You are on page 1of 437

EBD_7332

Corporate Office
DISHA PUBLICATION
45, 2nd Floor, Maharishi Dayanand Marg, Corner Market,
Malviya Nagar, New Delhi - 110017
Tel : 49842349 / 49842350

© Copyright Disha
No part of this publication may be reproduced in any form without prior permission of the
publisher. The author and the publisher do not take any legal responsibility for any errors
or misrepresentations that might have crept in. We have tried and made our best efforts
to provide accurate up-to-date information in this book.
All Right Reserved

Typeset by Disha DTP Team

www.dishapublication.com www.mylearninggraph.com
Books & ebooks Etests for
for School &
Competitive
Competitive
Exams Exams

Write to us at feedback@dishapublication.com
CONTENTS
NTSE Stage 1 (2020-21-2012) Solved Questions

• NTSE Solved Questions MAT (with solutions) A-1-108

• NTSE Solved Questions SAT (with solutions) B-1-204

Exemplar Practice Questions for MAT & SAT

• NTSE Solved Questions MAT M-1-31

• Hints & Solutions M-32-52

• NTSE Solved Questions SAT

Physics S-1-7

Chemistry S-7-12

Biology S-12-16

Maths S-16-21

History S-22-26

Civics S-26-30

Economics S-31-35

Geography S-36-40

• Hints & Solutions S-41-68


EBD_7332
Stage-1 (2020-21 – 2012) Solved Questions

Mental Ability Test – MAT

CHAPTER 1. KNOWLEDGE BASED REASONING

Directions: Qs. (30 - 33): In each of the following questions find 10. AJS, GPY, (?), SBK, YHQ
out the alternative which will replace the question mark. (1) DMV (2) MVE (3) OUA (4) QZI
(Andhra Pradesh-2020-21) Directions: Qs. (11 - 12) : Which of the following question is
1. Darvin : Evolution : : Archimedes : ? based on the following alphabet series:
(1) Friction (2) Lubrication (Bihar-2020-21)
(3) Buoyancy (4) Liquids ABCDEFGHIJKLMNOPQRSTUVWXYZ
2. Snake : Fang : : Bee : ? 11. Which letter is exactly midway between G and Q in the given
(1) Honey (2) Humming (3) Wax (4) Sting alphabet?
3. Tea : Cup : : Tobacco : ? (1) K (2) L (3) M (4) N \
(1) Leaves (2) Hookah 12. Which letter is sixteenth to the right of the letter which is
(3) Toxin (4) Cheroot fourth to the left of I?
(1) S (2) T (3) U (4) V
4. Aeroplane : Cockpit : : Train : ?
(1) Wagon (2) Coach Directions: Qs. (13 - 14) : Two matrices containing letters are
given below. The rows and columns are numbered 0 to 4 in matrix
(3) Compartment (4) Engine
1 and 5 to 9 in matrix II. Each letter from these matrices are
Directions: Qs. (5 - 6) : The letters in a word are replaced by represented by its row number and the next by its column number.
certain other letters according to a specific rule to form its code. (Karnatka-2020-21)
Answer the following questions accordingly.
(Andhra Pradesh-2020-21)
5. In a certain code "NEW YORK" is written as 111, how "NEW 0 1 2 3 4 5 6 7 8 9
JERSEY" will be written in that code ? 0 P W R MA 5 S T L K D
1 M A P WR 6 K D S T L
(1) 104 (2) 124 (3) 134 (4) 114
2 A P WR M 7 D S T L K
6. In a certain code "MADRAS" is coded as NBESBT, how
3 W R MA P 8 T L K D S
"BOMBAY" will be coded in that code ?
4 R M A P W 9 L K D S T
(1) CPNCBX (2) CPNCBZ Matrix - I Matrix - II
(3) CPOCBZ (4) CQOCBZ
Directions: Qs. (7 - 10) : In each of the following questions , 13. The set of numbers which represents the word PALM is
various terms an alphabet series/number series/ alpha-numeral (1) 12, 73, 21, 43 (2) 21, 33, 58, 03
series are given with one term missing as shown by (?). Choose (3) 34, 42, 86, 24 (4) 12, 20, 87, 32
the missing term out of the given.
14. The set of numbers which represents the word WARD is
(Bihar-2020-21)
(1) 01, 42, 43, 59 (2) 30, 33, 41, 88
3 4 9 13 (3) 44, 20, 31, 98 (4) 13, 04, 40, 75
7. , , (?), ,
7 11 31 17 Directions: Qs. (15 - 16) : Read the following information and
answer the questions :
6 6 6 5
(1) (2) (3) (4) (Karnatka-2020-21)
15 19 17 15
The order of the letters that needs to come in the boxes to complete
8. 625, 5, 125, 25, 25, (?), 5
a certain pattern is :
(1) 5 (2) 25 (3) 125 (4) 625
15. BY DW GT
9. 4, 8, 28, 80, 244, (?)
(1) 278 (2) 428 (3) 628 (4) 728 (1) JP (2) JQ (3) KQ (4) KP
EBD_7332
A-2 NTSE Stage 1 Question Bank

16. L MN K N KM 28. Arrange the following in a meaningful sequence :


(1) MLML (2) KLML (Rajsthan-2020-21)
(3) NMLL (4) KMML 1. Crop 2. Root 3. Stem 4. Seed
17. In a code 2357 stands for the word WORK and the code 5. Flower
14649 stands for the word STATE, then the code 14359 stands (1) 4, 1, 2, 3, 5 (2) 2, 4, 3, 5, 1
for: (Karnatka-2020-21) (3) 2, 3, 4, 5, 1 (4) 4, 2, 3, 5, 1
(1) S C O R E (2) S T O R E Directions: (Qs.29 - 31) : Each question has four terms. Three
(3) S T E O R (4) S T O E R terms are alike in some way. One term is different from three
18. If a word BALL is coded as 288, then the word JACK is others. Find out the correct term which is different from three
coded as: (Karnatka-2020-21) others and write its alternative number on your answer sheet
(1) 330 (2) 240 (3) 220 (4) 140 against the proper question number.
(Uttar Pradesh 2020-21)
Directions(Qs. 19 - 20): In each of the following questions write
which correct term in sequence replaces the question mark? 29. (1) Guru Ramdas (2) Guru Govind Singh
(Maharashtra-2020-21) (3) Guru Granth Sahib (4) Guru Nanak Dev
19. CD, HI, MN,? 30. (1) 289 (2) 216 (3) 512 (4) 729
(1) QS (2) OP (3) RS (4) PQ 31. (1) Sky–stars (2) Stadium–players
20. BM26 , EN70 , HO120 , KP176 ,? (3) Hospital–patient (4) Moon–bird
(1) NQ250 (2) NP224 Directions: (32 - 34): Question are based on the alphabet series
(3) MQ221 (4) NQ238 which is given below. Read the alphabet series carefully and find
out the correct answer for each question and write its alternative
Directions (Qs. 21 - 22): In the following specific group of letters
number on your answer sheet against the proper question number.
are given. From the given alternatives, find out the right letters
(Uttar Pradesh 2020-21)
which matches the given group.
noi j on p t ak t l n op uj e t bn ap ub
(Maharashtra-2020-21)
32. Which letter is been repeated the most in this series?
21. GECA ZXVT SQOM
(1) o (2) n (3) p (4) t
(1) YWUT (2) VTRQ (3) MKIH (4) LJHF 33. In this series, how many times the vowel occurs just before
22. BEIN EHLQ ILPU and just after consonant?
(1) NQUZ (2) HKOS (1) one times (2) two times
(3) LOSY (4) JMQT (3) three times (4) four times
Directions: Qs. ( 23 - 24) : In the following questions specific 34. In this series many time the vowel occurs just before the
group of numbers are given. From the given alternatives, find consonant but hot just after consonant?
out the right number which matches the given group. (1) One times (2) two times
(Maharashtra-2020-21) (3) three times (4) four times
35. Question is based on definite series. In given question some
23. 416 749 525
symbols are missing shown by (-). The missing symbols are
(1) 982 (2) 864 (3) 637 (4) 319
given in proper sequence as one of the four alternatives
24. 294 648 448
given under each question. Find out the correct alternative
(1) 84 (2) 94 (3) 100 (4) 194
and write number on the answer sheet against the question
Directions: (Qs. 25 - 27) : In each of the questions, a letter series is number. (Uttar Pradesh 2020-21)
given with one term missing shown by question mark (?). This term is c _ ba _ cb _ cc _ ac _ ba
one of the four alternatives given under it. Find the correct alternative. (1) cabac (2) ccabc (3) acabc (4) bcaac
(Rajsthan-2020-21) Directions: (Qs. 36 - 37) : In each of the questions, there are four items,
25. R, V, A, G, N, ? three of which are alike by some means or other while one is out of the
(1) U (2) V (3) W (4) X class. Find out the odd items and indicate your answer by filling the
26. COVIDNINETEEN, VIDNINETEENCO, DNINETEENCOVI, ? circle of the corresponding letter on the OMR Answer sheet.
(1) IDNINETEENCOV (2) NINETEENCOVID (West Bengal-2020-21)
(3) INETEENCOVIDI (4) INETEENCOVIDN
36. (1) Bar (2) Pie
27. 4, 5, 12, 39, 160, ?
(3) Rectangle (4) Pictogram
(1) 225 (2) 695 (3) 805 (4) 790
Solved Questions 3
A-

37. (1) SARS (2) COVID-19 50. Arrange the given words in the sequence in which they
(3) EBOLA (4) Typhoid appear in the dictionary and then choose the correct
sequence. (Bihar 2019)
Directions : (Qs.38 - 39): First pair is connected by some
(1) POWER (2) POWDER
relationship. The same relationship is applicable for the next
(3) POSITION (4) POSTER
pair. Identify the missing term in the second pair.
(5) POSITIVE
(Tamil Nadu-2020-21)
(1) 4, 5, 3, 2, 1 (2) 5, 3, 4, 2, 1
38. throw : collect : : push : __?___
(3) 3, 5, 4, 2, 1 (4) 2, 5, 1, 4, 3
(1) pull (2) door (3) window (4) knock
51. If the word GARDEN is coded as 49, then the word
39. apparel : shirt : : __?__ : necklace
FLOWER can be coded as: (Karnataka 2019)
(1) gold (2) jewellery (3) silver (4) ring
(1) 40 (2) 59 (3) 79 (4) 99
40. Arrange the given words in dictionary alphabetical order
52. In some code language if the word FRAGRANCE can be
and choose the word which comes in the middle.
coded as SBHSBODFG, then the word IMPOSING can be
(Tamil Nadu-2020-21)
coded as: (Karnataka 2019)
(a) credential (b) creed
(1) NQPTJHOJ (2) NQPTJOHI
(c) colour (d) credible
(e) create (3) NQPTJOHJ (4) NQTPJOHJ
(1) (a) (2) (e) (3) (b) (4) (d) Directions (Qs. 53 – 54): Given below are 2 matrices containing
Directions (Qs.41 – 44): Find the correct alternative that have letters. The rows and columns are numbered 0 to 4 in Matrix I and
same relation with the term right to symbol :: as the second term 5 to 9 in Matrix II. Each letter from these matrices are represented
have with first term. first by its row number and the next by its column number.

41. PMG : SIX :: TIN : _____ (Andhra Pradesh 2019) Example: Letter “T” is represented as any of the following: 59,
65, 76, 87, 98.
(1) WEB (2) SEE (3) WEE (4) BEE
Matrix I
42. AFC : DIF :: BED : _____ (Andhra Pradesh 2019)
0 1 2 3 4
(1) EHG (2) EGH (3) CGF (4) EIG
43. Doctor : Patient :: Lawyer : _____ (Andhra Pradesh 2019) 0 A R S N C
(1) Magistrate (2) Client 1 N C A R S
(3) Customer (4) Accused
44. P.V. Sindhu : Badminton :: Deepak Puniya :_______ 2 S N C A R
(Andhra Pradesh 2019) 3 R S N C A
(1) Cricket (2) Boxing
4 C A R S N
(3) Wrestling (4) Hockey
45. Which word cannot be formed from the letters of the word Matrix II
FRAGILE ? (Bihar 2019)
5 6 7 8 9
(1) LIFE (2) RAIL (3) EAGLE (4) RACE
46. In a certain code ‘CLOUD’ is written as ‘GTRKF’ then 5 O E L P T
how ‘SIGHT’ will be written in this code? (Bihar 2019) 6 T O E L P
(1) WGJHV (2) UGHHT
7 P T O E L
(3) UHJFW (4) WFJGV
8 L P T O E
Directions (Qs.47 – 49): In the following questions, select the
related letters/numbers from the given alternatives. 9 E L P T O
47. CE : 70 : DE : ? (Bihar 2019) 53. Which word will represent the set of numbers:
(1) 90 (2) 60 (3) 120 (4) 210 86, 12, 31, 76 (Karnataka 2019)
48. 7384 : 4837 : : 5291 : ? (Bihar 2019) (1) PAST (2) PATE (3) POTE (4) PSAT
(1) 1924 (2) 1925 (3) 1935 (4) 1915 54. Which word will represent the following set of numbers:
49. Find the missing term of given series? FOX, IQV, LST, 21, 67, 14, 59
OUR, ? (Bihar 2019) (1) PAST (2) RATE
(1) RWP (2) RPW (3) QVS (4) SXU (3) POET (4) NEST
EBD_7332
4
A- NTSE Stage 1 Question Bank

55. Complete the given analogy by choosing the correct Choose the correct code from the following alternatives
answer from the four options given below: for the word ‘DONAR’.
(1) 48391 (2) 54872 (3) 45392 (4) 53971
12 : 864 : : 18 : (Karnataka 2019)
64. In a mathematical code language (Maharashtra 2019)
(1) 1624 (2) 1824 (3) 1260 (4) 2916 88 – 7 = 39,
Directions (Qs. 56 – 57): In the following questions a specific 77 – 6 = 41,
group of numbers is given. From the given alternatives. Find out 99 – 5 = 74, then 55 – 4 = ?
the correct alternative that matches the given group. (1) 31 (2) 39 (3) 49 (4) 34
56. 150 576 252 (Maharashtra 2019) 65. In a coded language, BRAIN is written as *%÷#× and
TIER is written as $#+%; then in the same coded language,
(1) 393 (2) 466 (3) 80 (4) 182
RENT will be written as (Rajasthan 2019)
57. 193 454 265 (Maharashtra 2019)
(1) %×#$ (2) %#×$ (3) %+×$ (4) +×%$
(1) 572 (2) 823 (3) 734 (4) 367 66. In a coded language, ZEBRA is written as 2652181; then
58. In the following question there is a specific relation in the same coded language, COBRA will be written as
between first and second term. The same relationship exists (Rajasthan 2019)
between third and the fourth term. Considering the same (1) 3152181 (2) 1182153
relationship choose the correct alternative that will replace (3) 31822151 (4) 302181
the question mark. (Maharashtra 2019) 67. Arrange the following in a meaningful sequence:
11529 : 72135 :: 152943 : ? (Rajasthan 2019)
(1) 213549 (2) 223649 1. Probation
(3) 224194 (4) 215049 2. Interview
3. Selection
Directionss (Qs.59–60): Choose the correct alternative that will
4. Appointment
replace the question mark.
5. Advertisement
59. JDP, NGR, RJT, VMV, ? (Maharashtra 2019) 6. Application
(1) ZPW (2) ZQY (3) ZPX (4) ZRY (1) 5, 6, 2, 3, 4, 1 (2) 5, 6, 3, 2, 4, 1
60. ZAB, WDE, SHI, NMN, ? (Maharashtra 2019) (3) 5, 6, 4, 2, 3, 1 (4) 6, 5, 4, 2, 3, 1
(1) VEF (2) UFG (3) FUG (4) HSG
Directions (Qs.68 – 69): In each of the following questions, there
61. In the following table the digits are assigned with certain is a certain relationship between two given numbers on left side of
symbols. Observe them carefully and choose the correct (: :) and one number is given on the right side of (: :) while another
alternative to answer the questions. (Maharashtra 2019) number is to be found from the given alternatives, having the
Digit 9 0 8 1 7 2 6 3 5 4 same relationship with the number as the numbers of the given
Symbol pair bear. Choose the correct alternative.
68. 21 : 3 : : 574 : ? (Rajasthan 2019)
(1) 23 (2) 82 (3) 97 (4) 113
69. 42 : 56 : : 72 : ? (Rajasthan 2019)
(1)
(1) 81 (2) 90 (3) 92 (4) 100
(2) 70. Find out the correct term which is different from three
others (Uttar Pradesh 2019)
(3) (1) R81 (2) L19 (3) W25 (4) M16
71. Find out the correct term which is different from three
(4)
others (Uttar Pradesh 2019)
62. Choose the correct alternative that have same relation with (1) Lion (2) Deer (3) Wolf (4) Fox
third term as first term has with second term. 72. Find out the correct term which is different from three
ADE : FGJ : : KNO : ? (Maharashtra 2019) others (Uttar Pradesh 2019)
(1) PQR (2) PQT (3) RQP (4) TPR (1) 32.5 (2) 43.5 (3) 58.5 (4) 73.5
63. Observe the following code and answer the questions that
73. In a certain code language if BOUND is written as 112,
follow: (Maharashtra 2019)
what will be code of WHITE in same language?
Letters A T M G O D N R S
(Uttar Pradesh 2019)
Digits 9 8 7 6 5 4 3 21
(1) 132 (2) 103 (3) 130 (4) 123
Solved Questions A-5

74. In a certain code language if XDRL is written as 12296, what Directions (Qs.84 – 85): First two terms are connected by some
will be code of NHTV in same language? (Uttar Pradesh 2019) relationship. The same relationship is applicable for the next
(1) 72511 (2) 1481022 (3) 741011 (4) 7856 terms in which one is blank space. Identify the blank space.
75. In a certain code language if RUBBER is written as
84. Tree : Plant : : Woman :? (Tamil Nadu 2019)
BERRUB, what will be code of BUTTER in same language?
(1) boy (2) child
(Uttar Pradesh 2019)
(3) girl (4) mother
(1) TTBUR (2) TERBUT
85. Roof : floor : : ? : Dark (Tamil Nadu 2019)
(3) TUTREB (4) UBTTRE (1) Tube light (2) Bulb
Direction: In question (76) there are four terms in each question. (3) Bright (4) Black
The relation that exist between the terms left to the symbol :: is 86. The arrangement of alphabetical order of the words:
the same between the terms right to the symbol ::. Out of the four (Tamil Nadu 2019)
terms one term is missing in each question. The missing term is (a) Music (b) Monk
one of the four alternatives given below each question. Find out (c) Minimum (d) Maximum
the correct alternative and write its number on your answer (1) (d), (c), (b), (a) (2) (a), (c), (d), (b)
sheet against the proper question - (3) (d), (c), (a), (b) (4) (d), (a), (c), (b)
76. RSTU : YXWV :: GHIJ : ? (Uttar Pradesh 2019) 87. A group of letters is given which are numbered 1, 2, 3, 4, 5,
(1) NMLK (2) MLKJ (3) NLMK (4) MKJL 6. Choose the combination of numbers, so that letters
arranged form a meaningful word. (Tamil Nadu 2020)
Directions (Qs.77 – 78): In each question there are two words
R O P N E S
separated by ‘ : ’ and other two separated from the first two by
| | | | | |
the symbol ‘: :’. Find the relation between two sets of words and
1 2 3 4 5 6
select one word from the right side of ‘:’ which have the same
(1) 3, 5, 1, 6, 2, 4 (2) 1, 3, 5, 6, 2, 4
relation as left set of word of ‘: :’.
(3) 3, 5, 6, 1, 2, 4 (4) 3, 5, 2, 1, 6, 4
77. Lamp : Oil :: Bulb : ? (West Bengal 2019) 88. In a coded language, ‘SHOULDER’ is written as ‘TJSNMAGZ’
(1) Electricity (2) Bright and ‘BOXING’ is written as ‘RSYCPH’, then in the same lan-
(3) Holder (4) Switch guage, ‘HORN’ will be written as (Rajasthan 2018)
78. 841 : 29 :: 289 : ? (West Bengal 2019) (1) JSZP (2) JSNS (3) JNZS (4) JZSP
(1) 23 (2) 33 (3) 17 (4) 13 89. Arrange the following in a meaningful sequence:
A = Study, B = Service, C = Examination, D = Earning,
Directions (Qs.79 – 80): In each of the questions there are four E = Result (Rajasthan 2018)
items, three of which are alike by some means or while the other (1) EACDB (2) ABECD
one is out of the class. Find out the odd item. (3) ACEBD (4) AECBD
79. (1) Iron (2) Steel (3) Gold (4) Tin 90. Choose the number group which is different from others.
(West Bengal 2019) (Chhattisgrah 2018)
80. (1) 70, 80 (2) 54, 62 (3) 28, 32 (4) 42, 24 (1) 1, 5, 2 (2) 7, 113, 8 (3) 2, 20, 4 (4) 3,27,4
(West Bengal 2019) 91. Which term comes next in the series? (Chandigarh 2018)
81. Arrange the given words in the sequence in which they YEB, WFD, UHG, SKI ....
occur in dictionary and then choose the correct sequence. (1) QGL (2) TOL (3) QOL (4) QNL
(West Bengal 2019) 92. If CHARACTER is written 241612376 and CHILDREN is
(1) Leaf (2) Learned (3) Leave (4) Leak written 24859670 in sign language, then what will be written
(5) Leader for HIRALAL? (Gujarat 2018)
(1) 5, 1, 4, 2, 3 (2) 5, 1, 4, 3, 2 (1) 4861551 (2) 4861515
(3) 3, 5, 1, 4, 2 (4) 1, 4, 2, 3, 5 (3) 48651551 (4) 4681515
82. ‘Sea’ is related with ‘Waves’. Then ‘Candle’ is related to: 93. In a certain coding language, if GO = 32 & SHE = 49, then
(Tamil Nadu 2019) SOME will be equal to? (Odisha 2018)
(1) Rays (2) Wax (3) Stand (4) Melt (1) 56 (2) 58 (3) 62 (4) 64
83. In a certain code HONEY is written as HPPHC then FORGE 94. If ‘SCOILAND’ is written as ‘12345678’, LOAN is written as
is written as: (Tamil Nadu 2019) ‘8124’ and DAN is written as ‘537’, then what will be the
(1) FPTJJ (2) FPTJI (3) FQTJI (4) FPSJI code of ‘C’? (Odisha 2018)
(1) 6 (2) 9 (3) 5 (4) 4
EBD_7332
A-6 NTSE Stage 1 Question Bank

95. ‘O’ is smaller than ‘P’, but bigger than ‘Q’. ‘R’ is smaller Column-I Column-II
than ‘S’ but bigger than ‘T’. ‘Q’ is bigger than ‘S’. Who is ABLMS 24538
the biggest them? (Jharkhand 2018) QRLBA 93526
(1) Q (2) T (3) P (4) R PTQAB 52601
96. Choose the analogous pair............. LRNPQ 93716
Fan: Sweat :: ? : ? (Chhattisgarh 2018) ATRNP 29071
(1) Rain : Drought (2) Fire : Smoke MSPTQ 84106
(3) Wind : Evaporation (4) Walk : Dance QPNAR 16729
RABLS 29583
T X
97. Choose the best alternate in place of ‘?’ : 2 :: : ? TSLBA 80325
J H
PLQST 31860
(Chhattisgarh 2018) 103. The code for M is....
23 (1) 0 (2) 8 (3) 1 (4) 4
(1) 2 (2) 4 (3) 3 (4) 104. The code for N is ....
7
(1) 9 (2) 6 (3) 1 (4) 7
98. Complete the given series: (Delhi 2017)
25, 255, 2545, 25455, … Directions (Qs. 105 - 107): There are four terms in each
(1) 254545 (2) 25555 question. The term right to symbol : : have some relationship
(3) 254555 (4) 255454 as the term of the left to symbol : : and out of the four, one term
99. If ‘SKY WAS BLUE’ is 123 (Delhi 2017) is missing, which is among one of the given four alternatives.
‘SEA IS BLUE’ is 245 Find the correct alternatives.
‘PEOPLE SWIMMING IN SEA’ is 4678 (Andhra Pradesh 2017)
‘PEOPLE LIKE SKY’ is 801and 105. KMF: LLH : : RMS: ... ? ...
‘BIRDS IN SKY’ IS 169. Then ‘PEOPLE LIKE BIRDS’ will (1) TVT (2) SUS
have the number.
(3) SLR (4) SLU
(1) 809 (2) 104 (3) 036 (4) 806
106. Smoke: Pollution:: War: ......
Direction (Q. 100): In the number series given below, one number (1) Victory (2) Death
is missing. Each series is followed by four alternatives (1), (2), (3) Army (4) Enemy
(3), and (4). One of them is the right answer. Identify and indicate 107. Arrange the following words as per order in the dictionary.
it as per the “instructions”. 1. Dissident, 2. Dissolve, 3. Dissent, 4. Dissolute,
100. 13, 74, 290, 650, ..... (Andhra Pradesh 2017) 5. Dissolution (Haryana 2017)
(1) 1248 (2) 1370 (3) 1346 (4) 1452 (1) 3,1,4,2,5 (2) 3,1,4,5,2
(3) 3,2,4,5,1 (4) 3,2,1,4,5
Directions (Qs. 101 to 102): In each of the following questions,
a letter series is given, in which some letters are missing. The Direction (Q. 108): Select the one word-pair/number-pair/
missing letters are given in the proper sequence as one of the letter cluster which is different from the other three alternatives.
alternative. Find the correct alternative (Haryana 2017)
101. A_BBC_AAB_CCA_BBCC (Andhra Pradesh 2017) 108. (1) Light : Heavy
(1) BACB (2) ABBA (2) Kind : Cruel
(3) CABA (4) ACBA (3) Soft : Hard
102. BA_B_AAB_A_B (4) Vacant : Empty
(1) AABB (2) BABB 109. From the given alternative words, select the word which
(3) BAAB (4) ABBA cannot be formed using the letter of the given word:
COMMISSIONER (Haryana 2017)
Directions (Qs 103 & 104): Some letters are given in column I (1) COMMON (2) MISSION
and some digits are given in column II. Each digit of column II (3) MISSILE (4) SIREN
represents only letter of column II represent any letter of column I. 110. Which one number is wrong in the number series?
study the columns and write the alternative letter after choosing 13, 17, 19, 23, 27, 29 (Haryana 2017)
the correct alternative against the corresponding question. (1) 29 (2) 27 (3) 23 (4) 19
(Andhra Pradesh 2017)
Solved Questions A- 7

Directions (Qs. 111 & 112): In each of the questions 1 to 2 a 120. Institute : Academy :: Decree : ?
letter series is given with one term missing shown by question (1) Blame (2) Court
mark (?). This term is one four alternatives given under it. Find (3) Judge (4) Mandate
the right alternative. 121. 63 : 80 : : 120 : ?
(1) 125 (2) 137 (3) 170 (4) 180
111. G, K, O, S, ? (Haryana 2017)
(1) U (2) W (3) V (4) X Direction (Q. 122): In each of the following questions write
112. KLE, IND, GPC, ?, CTA. (Haryana 2017) which term in sequence replaces the questions mark?
(1) DRB (2) BSE (3) ERB (4) ECR (Maharashtra 2017)
Directions (Qs. 113 & 114): In each of the questions 3 to 4 a 122. BJ, DL, HP, PX, ?
number series is given with one term missing shown by question (1) FN (2) FX (3) TB (4) VD
mark (?). This term is one of the four alternatives given under it. Direction (Q. 123): Find the odd term
find the right alternative.
(Maharashtra 2017)
113. 1, 3, 7, 13, 21, ?, 43, 57. (Haryana 2017) 123. (1) 141 (2) 101 (3) 107 (4) 131
(1) 31 (2) 29 (3) 30 (4) 32 124. In the following question a specific group of numbers is
114. 5, 3, 10, 8, 17, 15, ?, 24. (Haryana 2017) given. From the given alternatives, find out the right number
(1) 25 (2) 23 (3) 26 (4) 27 which matches the given group.
341, 572, 781 (Maharashtra 2017)
Direction (Q. 115): In this questions alternatives are alike in a
(1) 634 (2) 891 (3) 909 (4) 990
certain way but the rest one is different. Find out the odd one and
125. In a certain code language ZEAL = 11, Written then in that
write correct answer,
language BEAT = ? (Maharashtra 2017)
(Haryana 2017) (1) 7 (2) 13 (3) 14 (4) 19
115. (1) Afghanistan (2) Kabul Direction (Q. 126): In chart one the rows and columns are
(3) Spain (4) Iraq.
labelled with 0 to 4 number. In chart two rows and columns are
116. ‘A+B’ means Ais the son of B. ‘A – B’ means A is the wife
labelled with the numbers 5 to 9. The letter in the chart is
of B. Then what does P + R – Q mean? (Haryana 2017)
identified firstly by its row number and then by its column
(1) Q is the father of P (2) Q is the son of P number. For example 5 is denoted by 22,41 number.
(3) P is the father of Q (4) R is the son of Q
(Maharashtra 2017)
Directions (Qs. 117 - 118): In each of the following questions, a CHART I CHART II
series of number/alphabets is given which follow certain rules.
One of the number/alphabet is missing. Choose the missing 0 1 2 3 4 5 6 7 8 9
number/alphabets from the alternatives given below and mark 0 F O M S R 5 A T D I P
it on your answer-sheet as directed. 1 S R F O M 6 I P A T D
(Jharkhand 2017) 2 O M S R F 7 T D I P A
117. 18, 24, 21, 27, ?, 30, 27 3 R F O M S 8 P A T D I
(1) 33 (2) 30 (3) 24 (4) 21 4 M S R F O 9 D I P A T
118. ELFA, GLHA, ILJA, – , MLNA
126. Which group of words represent the word MOST?
(1) OLPA (2) KLMA
(Maharashtra 2017)
(3) LLMA (4) KLLA
(1) 40, 44, 22, 89 (2) 33, 20, 11, 79
119. If in a certain code, SENIOR is written as NZIDJM. Then
(3) 21, 00, 03, 88 (4) 02, 13, 34, 56
which word is written in the same code as XDODUZI.
(Jharkhand 2017) Direction (Q. 127) : In the question three statements followed
(1) CISTERN (2) INQUIRE by two conclusions numbered I and II are given. Decide which
(3) CITIZEN (4) SUSTAIN of the given conclusions logically follows the given statement
disregarding commonly known facts. Give answer.
Directions (Qs. 120 - 121): In these questions pairs of words/
letters numbers to the left/right of :: have certain relationship (Haryana 2016)
with each other. You are required to select the correct alternative (1) If only conclusion I follows
so that similar relationship is established to the right/left of :: (2) If only conclusion II follows
(Jharkhand 2017) (3) If both I and II follow
(4) If neither I nor II follows
EBD_7332
8
A- NTSE Stage 1 Question Bank

127. Statements: (Haryana 2016) Directions (Qs. 134 - 135) : From each question has four terms.
All crows are parrots. All parrots are doves. Some doves Three terms are alike in some way one term is different from
are cats. three others. Find out the correct term which is different from
Conclusions: other three.
I Some cats are crows. II Some doves are crows. (Uttar Pradesh 2016)
134. (1) G I L P (2) D F I M (3) B D J K (4) EGJN
Direction (Q. 128) : A series of small letters is given which
follow a certain pattern. However some letters are missing from 135.
the series. You have to find out the right set of letters from
alternatives that can be inserted into the blanks of the series.
(1) (2) (3) (4)
128. h_eg_fegh_eghfe_ (Haryana 2016)
(1) gffh (2) hhgg (3) ffgh (4) fhfg Directions (Qs. 136 - 139) : Question in column I are coded in
Directions (Qs. 129 - 130): In each of the questions below two the form of numbers. Which are written in column II, but the
statements and two conclusions numbered I and II are given. order of numbers is different. Read carefully code of letters.
You have to take the given two statements to be true even if they Find out correct answer in given alternative and write its
seem to be at variance from commonly known facts. Read the alternative number against the corresponding question number
conclusions and then decide which of the given conclusions on your answer sheet.
logically follows from the two given statements. (Uttar Pradesh 2016)
129. Statements: (Rajasthan 2016) Column-I Column-II
I. All dancers are singers II. All singers are teachers CJ L 359
Conclusions: EJP 092
I. All dancers are teachers II. Some singers are dancers PCK 304
(1) Only conclusion I is true KN D 478
(2) Only conlusion II is true NE V 721
(3) Both conclusion I and II are true 136. What will be code of K N P?
(4) Neither conclusion I nor conclusion II is true. (1) 870 (2) 327 (3) 951 (4) 470
130. Statements: (Rajasthan 2016) 137. What will be code of C J E?
I. Some fruits are mangoes II. Some fruits are not guavas (1) 123 (2) 392 (3) 724 (4) 803
Conclusions: 138. What will be code of L J K?
I. All fruits are mangoes II. All mangoes are fruits (1) 270 (2) 903 (3) 594 (4) 741
(1) Only conclusion I is true 139. What will be code of P V D?
(2) Only conclusion II is true (1) 018 (2) 372 (3) 209 (4) 743
(3) Both conclusion I and II are true
(4) Neither conclusion I nor conclusion II is true. Direction (Q. 140): Find out the alternative which will replace
131. If in a coded language the word SCHOOL is written as UAJMQJ the question (?) mark. (Odisha 2016)
then PRINCIPLE will be written as (Rajasthan 2016) 140. REASON : SFBTPO : : THINK : ?
(1) RTKPEKRCN (2) NPGLAGNJB (1) SGHMJ (2) UIJOL
(3) RPKLEGRJG (4) RPKLEKRJG (3) UHNKI (4) UJKPM
Directions (Qs. 132 - 133): Following alphabets are written in Direction (Q. 141) Here are some words translated from an
a special coded language like: artificial language. Answer the related question. (Odisha 2016)
B LA C K WHITE 141. 'peslligen' means 'basketball court'
01 2 34 56 7 89 'ligenstrisi' means 'courtroom'
132. Then code 62830 will be written as (Rajasthan 2016) 'oltaganti' means 'placementtest'
(1) HATCB (2) HATEC Which word could mean "guest room"?
(3) HATBC (4) HATCE (1) peslstrisi
133. 'BHICK' will be coded as (Rajasthan 2016) (2) vosefstrisi
(1) 06734 (2) 67340 (3) gantipesl
(3) 67430 (4) 06743 (4) oltastrisi
Solved Questions 9
A-

142. Alphabet series: (Maharashtra 2016) Direction (Q. 151 - 152):- Observe the following pyramid of
x c w m vc x w m w m x c x w m x m alphabets and answer the following questions.
In the given alphabet series how many times m is succeeded a
by w and preceded by w? b c
(1) 1 (2) 0 (3) 2 (4) 3 f e d
g h i J
Directions (Qs. 143 & 144) : In each of the questions 1 and 2,
o n m l k
there are four items, three of which are alike by some means or
p q r s t u
other while one is out of the class. Find out the odd item and
b a z y x w v
indicate your answer by filling the circle of the corresponding
c d e f g h i j
letter on the O.M.R. Answer-Sheet. 151. onza, nmyz, mlxy, ? (Maharashtra 2014)
143. (1) Flourine (2) Bromine (1) nmst (2) hilm (3) abcd (4) lkwx
(3) Chromium (4) Chlorine 152. aqze : wtxh : : ghno : ? (Maharashtra 2014)
(West Bengal 2015) (1) abce (2) qrfe (3) jilk (4) ijlh
144. (1) Titan (2) Eris Directions (Qs. 153 - 154): In the given questions there are four
(3) Uranus (4) Haumea groups of numbers / pairs of numbers / group of letters of which
(West Bengal 2015) three are alike and one is different. Find the one which is different.

Directions (Qs. 145 – 147) : In each question below there are two (Karnataka 2014)
words separated by ‘:’ in the upper row. Below that there are some 153. (1) GAINFUL (2) SE CTARY
(3) FOUNDE R (4) Q UAD RI C
words on each side of the symbol ‘:’. Find the relation between
154. (1) 686,21 (2) 1024,24
two upper words and select one word from the right side of ‘:’
(3) 2000,30 (4) 2662,30
below which have the same relation as above. Fill the circle of the
letter denoting your selected answer on the O.M.R. Answer-Sheet. Directions (Qs. 155 - 157): Complete the given analogy by
145. Octopus : Mollusca selecting the correct answers from the alternatives.
Spider : ? (West Bengal 2015) 155. 147 : 49 : : ? : ? (Karnataka 2014)
(1) Arthropoda (2) Clean (1) 186 : 66 (2) 189 : 33 (3) 198 : 66 (4) 201:33
156. 195 : 15 : : ? : 25 (Karnataka 2014)
(3) Porifera (4) Genus
(1) 575 (2) 600 (3) 625 (4) 650
146. Pituitary : Brain
157. TPLNR : BXTVZ : : NJFHL : ? (Karnataka 2014)
Thymus : ? (West Bengal 2015) (1) VNRTP (2) VRNPT
(1) Throat (2) Chest (3) URMPT (4) RVPTN
(3) Spinal Cord (4) Larynx 158. Oriya is related to Orissa in the same way Dogri is related to;
147. Eye : Myopia (Madhya Pradesh 2013)
Teeth : ? (West Bengal 2015) (1) Himachal Pradesh (2) Sikkim
(1) Cataract (2) Trachoma (3) Jammu (4) Assam
(3) Pyorrhoea (4) Eczema Directions (Q. 159): In each of the following questions, a group
of three interrelated words is given. Choose a word from the
Directions (Qs. 148 - 149) : In each of the following questions
given alternative, that belongs to same group.
there is a specific relationship between the first and. second
term. The same relationship exists between the third and fourth 159. Ohm : Watt: Volt:? (Madhya Pradesh 2013)
term which will replace the question mark (?). Select the correct (1) Light (2) Electricity
term from the alternatives given : (3) Hour (4) Ampere
160. Four words are given below out of which three are alike in
148. 531 : 99 :: ? : ? (Maharashtra 2014)
some manner and fourth one is different. Choose out the
(1) 451 : 55 (2) 321 : 44 (3) 642 : 66 (4) 212:11
odd one : (Madhya Pradesh 2013)
149. ACG : ZXT : : HJN : ? (Maharashtra 2014) (1) Barauni (2) Trombay
(1) SQM (2) TRN (3) SQN (4) SOM (3) Neyveii (4) Mettur
161. Choose odd pair of the words : (Madhya Pradesh 2013)
Directions (Q. 150) : Find the odd term out. (Maharashtra 2014)
(1) Taiwan : Taipei (2) China : Mongolia
150. (1) 336 (2) 508 (3) 504 (4) 720 (3) Iran : Tehran (4) Japan : Tokyo
EBD_7332
A-10 NTSE Stage 1 Question Bank

Directions (Qs. 162): If MISTAKE is coded as 9765412 and 172. If second alphabet in each of the words is changed to next
NAKED is coded as 84123, how the words will be coded in the alphabet in the English alphabetical order, how many words
following questions; having no vowels will be formed?
(1) One (2) Two
162. DISTANT: (Madhya Pradesh 2013)
(3) Three (4) More than three
(1) 3765485 (2) 4798165
173. Arrange the given words in the sequence in which they
(3) 3697185 (4) 4768296
occur in the English dictionary and then choose the correct
Directions (Qs. 163 – 165) : Find out the missing one from the sequence. (Punjab 2012)
given alternatives. (A) Select (B) Seldom
163. 08 : 28 : : (?) : 65 (Rajasthan 2013) (C) Send (D) Selfish
(1) 9 (2) 12 (3) 15 (4) 18 (E) Seller
164. 35 : 91 : : 189 : (?) (Rajasthan 2013) (1) A, B, D, E, C (2) B, A, E, D, C
(1) 343 (2) 341 (3) 280 (4) 210 (3) B, A, D, E, C (4) B, E, D, A, E
7 13 19 Directions (Qs. 174 – 175) : In each of the following questions,
165. : : : : (?) (Rajasthan 2013)
11 17 23 find which word cannot be made from the letters of the given
word ? (Madhya Pradesh 2012)
25 29 23 29
(1) (2) (3) (4) 174. DEPARTMENT
27 31 29 33 (1) ENTER (2) PERMIT
166. Which of the following has the same relationship as that of (3) TEMPER (4) RENTED
‘PS’ : ‘TW’? (Delhi 2012) 175. CONSTITUTIONAL
(1) JM : RQ (2) AD : DI (3) AD : EH (4) FC:ZE (1) LOCATION (2) TUTION
167. 3 out of 4 words are alike in some way and form a group. (3) TALENT (4) CONSULT
Which one does not belong to the group? (Delhi 2012) 176. Which name will be in second place if given names arranged
(1) Tyre (2) Engine (3) Speed (4) Fuel as they appear in telephone directory?
Directions (Qs. 168 – 169) : Which of the following has the same (Madhya Pradesh 2012)
relationship as given in the question. (Delhi 2012) (1) Avdash (2) Avadhesh
168. Race : Fatigue: : ............. : ............ (3) Awadesh (4) Awdjiesh
(1) Fast : Hungry (2) Fast : Energy Directions (Qs. 177 – 178) : Read the following questions carefully
(3) Fast : Food (4) Fast : Fatigue and answer. (Madhya Pradesh 2012)
169. Penology : Punishment : Seismology : ............ . 177. Mathematics is related to Numbers in the same way History
(1) Law (2) Earthquake is related to :
(3) Liver (4) Medicine (1) People (2) Events
Directions (Qs. 170) : Choose the odd one out: (3) Dates (4) Wars
170. Arrange the words in the correct order: (Delhi 2012) 178. Accident is related to Carefulness in the same way Disease
(A) Tissues (B) Cells is related to :
(C) Organ System (D) Body (1) Sanitation (2) Treatment
(E) Organ (3) Medicine (4) Doctor
(1) B, A, C, E, D 179. Four words are given below out of which three are alike in
(2) C, A, B, E, D some manner and fourth one is different. Choose out the
(3) A, E, C, D, B odd one. (Madhya Pradesh 2012)
(1) Circle (2) Ellipse
(4) B, A, E, C, D
(3) Sphere (4) Cube
Directions (Qs. 171 – 173) : Read the data carefully and answer 180. Choose odd pair of the words : (Madhya Pradesh 2012)
the questions. (1) Volume : Litre (2) Time : Seconds
LAP TUB CAR SON HID (3) Length: Metre (4) Pressure : Barometer
(New words to be formed may or may not necessarily be meaningful 181. Four numbers are given below out of which three are alike in
English words.) (Delhi 2012) some manner and fourth is different. Choose the different
171. Which alphabet is on same place in two words given in the number: (Madhya Pradesh 2012)
question? (1) 11 (2) 13 (3) 15 (4) 17
(1) u (2) i (3) a (4) n
Solved Questions A-11

Directions (Qs. 182 – 183) : If MISTAKE is coded as 9765412 197. (1) EBA (2) XUT (3) TQP (4) GFE
and NAKED is coded as 84123, how the words will be coded in 198. (1) NPM (2) IJL (3) QSZ (4) XGT
the following questions: (Madhya Pradesh 2012)
Directions (Qs. 199 – 203) : In these questions, there is a certain
182. DISTANT: relationship between two given words on one side of :: and one
(1) 3765485 (2) 4798165 word is given on another side of ::, the other word is to be found
(3) 3697185 (4) 4768296 out from the given five alternatives so that the relationship is the
183. STAIN: same as for the other pair. Identify correct answer and indicate it
(1) 98175 (2) 89483 as per the “Instructions.” (Andhra Pradesh 2012)
(3) 68194 (4) 65478
199. Bengaluru : Karnataka :: Kohima :?
184. If REASON is coded as 5, BELIEVED as 7, what is the code
(1) Manipur (2) Meghalaya
number of GOVERNMENT? (Madhya Pradesh 2012)
(1) 6 (2) 8 (3) 9 (4) 10 (3) Mizoram (4) Nagaland
200. London : UK :: Bangkok: ?
Directions (Qs. 185 – 188) : Each question consists of four groups. (1) South Korea (2) North Korea
One set is different from other three in someway. Find out the (3) Thailand (4) Malaysia
different set :– (Rajasthan 2012) 201. Hockey : Stick :: Tennis : ?
185. (1) PROQN (2) DFCEG (l) Bat (2) Racket (3) Ball (4) Court
(3) GIFHE (4) KMJLI 202. Oxygen : Heart :: Fuel :?
186. (1) ACEGF (2) FHJLK (1) Car (2) Engine (3) Truck (4) Bike
(3) KMOQP (4) UWYZA 203. Orange : Fruit :: Spinach: ?
187. (1) CADGE (2) JHKNL (l) Vegetable (2) Root
(3) ECFGI (4) XVYBZ (3) Bush (4) Flower
188. (1) 6253 (2) 1881 (3) 7043 (4) 8210
Directions (Qs. 204 – 208) : If in a language (say English), A is
Directions (Qs. 189 – 192) : In each of the questions there is coded as 1, B is coded as 2 and so on, how are the following
some relationship between the two terms on the left side of the letters groups are coded in that language?
sign (::). The same relationship exists between the two terms on (Andhra Pradesh 2012)
the right side of the sign (::) of which one is missing. Find the 204. FHCI
missing one from the given four alternatives. (Odisha 2012) (1) 6389 (2) 8369 (3) 6893 (4) 6839
189. MUMBAI : LTLAZH : : DELHI : ____________ 206. GADBH
(1) CDKGJ (2) IHLED (1) 71248 (2) 81248 (3) 71428 (4) 81428
(3) CDKGH (4) BCKGH 206. HBDFE
190. corden : zrogbq :: ________ : pxivro (1) 82654 (2) 84265 (3) 82465 (4) 86426
(1) srspql (2) munmun 207. GCEAI
(3) mulmul (4) sulsul (1) 72519 (2) 73519 (3) 73591 (4) 75391
208. IHGFED
191. RATIONAL:RATNIOLA:: _________ :TRILBA
(1) 987651 (2) 978562
(1) TIRLAB (2) TRIBAL
(3) 897654 (4) 987654
(3) TRIALB (4) TIRBAL
192. HI : 36 :: DE : _________ Directions (Qs. 209 – 211) : In each of the following questions
(1) 10 (2) 15 (3) 20 (4) 25 there is a specific relationship between the first and second term.
The same relationship exists between the third and the fourth
Directions (Qs. 193 – 194) : In each of the questions choose that term which will replace the question mark. Select the correct
set of numbers from the four alternative sets, which is similar to term from the alternatives given. (Maharashtra 2012)
the given set. (Odisha 2012) 209. AYXD : ZBCW : : ? : QKLN
193. Given set (9, 15, 21) (1) MOPJ (2) JKLM (3) JPOM (4) MJKL
(1) (7, 21, 8) (2) (5, 10, 25) 210. KHOT : 45 : : VILE : ?
(3) (10, 14, 16) (4) (4, 8, 12) (1) 48 (2) 68 (3) 76 (4) 84
194. Given set (223, 324, 425) 211. AEH : IMP : : BFI : ?
(1) (554, 655, 756) (2) (225, 326, 437) (1) JMP (2) JNQ (3) JNP (4) JMQ
(3) (451, 552, 636) (4) (623, 723, 823) Directions (Qs. 212) : The following questions are based on the
given matrix. The value of each letter is the product of its row
Directions (Qs. 195 – 198) : In each of the questions choose the
and column numbers. (Karnataka 2011)
group of letters which is different from others. (Odisha 2012)
Example: The value of letter “N” is 4 × 3 = 12
195. (1) AI (2) IU (3) EO (4) OU
212. As per directions given, sum of letters “ABGENO” is:
196. (1) PUT (2) OL (3) END (4) ARM
(1) 61 (2) 57 (3) 51 (4) 41
Solved Questions A-47

Directions (Qs. 65 & 66) : Madhav and Govind play Hockey is a lawyer. (3) Farmer is the richest of all. (4) Occupation of the
and Volleyball, Hemant and Madhav play Hockey and Baseball. poorest is photography. (5) Amar is a doctor and Gopal is a
Ramesh and Govind play Cricket and Volleyball. Hemant, photographer. (6) Akhil is more richer than Amar but less richer
Ramesh and Anant play Football and Baseball. Then, answer than Bhanudas. Then: (Maharashtra 2016)
the following questions. (Maharashtra 2017) 74. What is the occupation of Chaitanya?
(1) Lawyer (2) Doctor
65. Who plays Hockey, Cricket and Volleyball? (3) Farmer (4) Teacher
(1) Madhav (2) Govind (3) Hemant (4) Anant 75. Who among the following is farmer?
66. Who does not play Baseball? Choose the correct alternative? (1) Akhil (2) Bhanudas
(Maharashtra 2017) (3) Gopal (4) Chaitanya
(1) Govind (2) Hemant 76. A rhythmic arrangement of alphabets is given. The missing
(3) Madhav (4) Ramesh letter alphabets appear in the same order as in one of
Direction (Q. 67): Read the following information carefully and —bcdbc—dcabd—bcdbc—dc—b.
answer the question given below: (1) aaaaa (2) bbbbb (3) ccccc (4) ddddd
M, P, J, B, R, T and F are sitting around a circle the centre. B is the third Directions (Qs. 77 - 78) : The information about the students
to the left of J who is second to the left of M. P is third to the left of B wearing T-shirts having numbers 7, 8, 9 and 10 is given below.
and second to the right of R. T is not an immediate neighbour of M. Read the given information and answer the following questions.
67. Who is fourth to the right of M? (Delhi 2016) (i) The players having number 7 and 9 play Football as well as
(1) B (2) T (3) R (4) M
Cricket.
Directions (Qs. 68 - 70) : Questions are based on the information (ii) The fat player having number 8 participates in all the games
given below. Read the information carefully and find out the except Kho-Kho and Football.
correct answer from the four alternative and write its alternative (iii) The player carrying number 9 is very short. He is expert in
number on you answer sheet against the proper question number. Kabaddi. The player having number 10 is very slim. He par-
For being graduate Dinesh opted Sanskrit, Science and Hindi, ticipates in all the games except Football.
Ganesh opted English, Mathematics and Hindi, Umesh opted (iv) The player having number 7 is very tall.
English, Science and Hindi. Nita opted Sanskrit, Science and 77. Which game can be played by one and only one player ?
Hindi. While Gita opted English, Sanskrit and Hindi. Then answer (Maharashtra 2014)
the following question. (Uttar Pradesh 2016) (1) Kabaddi (2) Kho-Kho
68. Which subject opted by the most students. (3) Cricket (4) Football
(1) Sanskrit (2) Science 78. Name the game, which is played by the slim player but can-
(3) Hindi (4) English not be played by the fat player ? (Maharashtra 2014)
69. Which subject opted by the least student. (1) Cricket (2) Football
(1) Science (2) Mathematics (3) Kho-Kho (4) Kabaddi
(3) English (4) Sanskrit Directions (Qs. 79 – 83) : Study the information given below
70. How many student opted Sanskrit subject. and answer the questions that follow:
(1) 2 (2) 4 (3) 5 (4) 3
(i) P, Q, R, S, T and U are six students procuring their Master’s
Directions (Qs. 71 - 73): Read the given information carefully degree in six different subjects-English, History, Chemistry,
to answer the question. Physics, Hindi and Mathematics.
A, B, C, D, E, F and G are sitting in a row facing North. F is to the (ii) Two of them stay in a hostel, two stay as Paying Guest (PG)
immediate right of E. E is 4th to the right of G. C is the neighbour of and the remaining two stay at home.
B and D. Person who is third to the left of D is at one of the ends. (iii) R does not stay as PG and studies Chemistry.
71. Who are to the left of C? (Odisha 2016) (iv) The students studying Hindi and History do not stay as
(1) Only B (2) G, B and D Paying Guest (PG)
(3) G and B (4) D, E,F and A (v) T studies Mathematics and S studies Physics.
72. Who are the neighbours of B? (Odisha 2016) (vi) U and S stay in a hostel. T stays as Paying Guest (PG) and
(1) C and D (2) C and G Q stays at home. (Punjab 2012)
(3) G and F (4) C and E 79. Who studies English?
73. Which of the following statements is not true? (1) R (2) P (3) S (4) T
(1) E is to the immediate left of D (Odisha 2016) 80. Which of the following combinations of subject and place
(2) A is at one of the ends of stay is not correct?
(3) G is to the immediate left of B (1) English-Hostel
(4) F is second to the right of D (2) Chemistry-Home
(3) Mathematics-Paying Guest
Directions (Qs. 74 - 75) : Answer the following questions based (4) Physics-Hostel
on the information given below: 81. Which of the following pairs of students stay one each at
(1) Bhanudas, Gopal, Amar, Akhil and Chaitanya each practice hostel and at home?
one of the professions. Farmer, Lawyer, Doctor, Teacher, (1) QR (2) SR (3) PQ (4) PS
Photographer. (2) Akhil who is richer than Amar and Chaitanya,
EBD_7332
A- 48 NTSE Stage 1 Question Bank

82. Which subject does Q study? (1) S is ranked third (2) T is ranked sixth
(1) History (2) Hindi (3) P is ranked sixth (4) V is ranked fourth
(3) History or Hindi (4) None of these 87. If S is ranked second, which of the following can be true ?
83. Which of the following pairs of students stay at home? (1) U gets more than V (2) V gets more than S
(1) PQ (2) QR (3) RS (4) ST (3) P gets more than R (4) P gets more than V
88. If V is ranked fifth, which of the following must be true ?
Directions (Q. 84) : Study the information given below and (1) S scores the highest (2) R is ranked second
answer the questions that follow: (3) T is ranked third (4) Q is ranked fourth
Six lectures A, B, C, D, E and F are to be organized in a span of
seven days-Sunday to Saturday, only one lecture on each day Directions (Qs. 89 – 90) : Read the following passage carefully
accordance with the following: and answer the questions.
(i) A should not be organized on Thursday. P, Q, R, S and T are five teachers. Each teacher teaches only one
(ii) C should be organized immediately after F. subject. T is not Hindi teacher and R, S do not teach English. P, Q
(iii) There should be a gap of two days and R are not related with History and Hindi. Neither-S nor T
(iv) One day there will be no lecture (Friday is not that day), just teaches Mathematics. Geography is not taught by P, R and T.
before that day D will be organized. (West Bengal 2012)
(v) B should be organized on Tuesday and should not be 89. The teachers of Geography and History are
followed by D. (Punjab 2012) (1) Q and S (2) R and T
84. Which of the following is the last lecture in the series? (3) Q and T (4) S and T
(1) A (2) B (3) C (4) D 90. The teacher of English and Mathematics are
(1) P and R (2) Q and T
Directions (Qs. 85 - 88) : Read the information carefully and (3) P and S (4) R and S
answer the questions given below :
(i) Seven students P, Q, R, S, T, U and V take a series of tests, Directions (Qs. 91 – 93) : Suman and Komal are experts in acting
(ii) No two students get similar marks, and singing. Geeta is well versed in singing and debating. Suman
(iii) V always scores more than P. and Preeti are experts in drawing. Komal and Shweta are well
(iv) P always scores more than Q. versed in drawing and debating. Komal and Preeti have a good
(v) Each time either R scores the highest and T gets least, or knowledge of sculpture. Then, answer the following questions.
alternatively S scores highest and U or Q scores least. (Maharashtra 2012)
(Madhya Pradesh 2012) 91. Who knows only drawing and debating ?
85. If S is ranked sixth and Q is ranked fifth, which of the (1) Geeta (2) Preeti
following can be true ? (3) Suman (4) Shweta
(1) V is ranked first or fourth 92. Who knows only singing and debating ?
(2) R is ranked second or third (1) Suman (2) Geeta (3) Shweta (4) Preeti
(3) P is ranked second or fifth 93. Who knows all art forms ?
(4) U is ranked third or fourth (1) Geeta (2) Komal
86. If R is ranked second and Q is ranked fifth, which of the (3) Preeti (4) Suman
following must be true ?

CHAPTER 5. NON-VERBAL REASONING

Directions: (Qs.1 - 3) : Out of the four figures (1), (2), (3), (4) 2. Choose the figure which is different from the rest.
given in each question, three are similar in certain way. Choose
the figure which is different from the other figures.
(Andhra Pradesh-2020-21)
1. Choose the figure which is different from the rest.
(1) (2) (3) (4)
3. Choose the figure which is different from the rest.

(1) (2) (3) (4)

(1) (2) (3) (4)


Solved Questions 49
A-

Directions: (Qs.4 - 6): Each of the following questions consists Answer Figures :
of five figures marked A, B, C and D constitute the problem set
while figures 1, 2, 3 and 4 constitute the answer set. There is a
(1) (2)
definite relationship between figures A and B. Establish a similar
relationship between figures C and D by selecting a suitable
figure from the Answer set that would replace the question mark
(?) in figure (D).
(3) (4)
(Andhra Pradesh-2020-21)
4. Problem Figures :
Directions: (Q. 7) : Find the missing part of the given figure from
the alternatives.
? (Karnatka-2020-21)

(A) (B) (C) (D)


Answer Figures :

7.

(1) (2)

(1) (2)

(3) (4)
(3) (4)

8. Complete the given figure analogy by choosing the correct


5. Problem Figures :
answers from the given alternative. (Karnatka-2020-21)

?
: :: : ?

(A) (B) (C) (D)


Answer Figures :

(1) (2)
(1) (2)

(3) (4)

6. Problem Figures :
(3) (4)

?
(A) (B) (C) (D)
EBD_7332
A- 50 NTSE Stage 1 Question Bank

9. Find number of triangles in the given figure. 12. Go through the diagram and choose the answer.
(Maharashtra-2020-21) The number of triangles in the given diagram is :
(Tamil Nadu-2020-21)

(1) 16 (2) 20 (3) 24 (4) 32 (1) 17 (2) 11 (3) 8 (4) 10


Directions (Q. 10): In the given question a complex figure is 13. Choose the figure which is different from the rest.
given. Find out which of the simple figures given in the (Andhra Pradesh 2019)
alternatives is hidden in the complex figure.
(Maharashtra-2020-21)
10. Question figure :
(1) (2) (3) (4)
(1) 1 (2) 3 (3) 2 (4) 4
14. Choose the figure which is different from the rest.
(Andhra Pradesh 2019)
Answer figure

(1) (2) (3) (4)


(1) (2) (1) 3 (2) 4 (3) 1 (4) 2
15. Choose the figure which is different from the rest.
(Andhra Pradesh 2019)

(3) (4)
(1) (2) (3) (4)
(1) 4 (2) 1 (3) 3 (4) 2
Directions: (Q. 11) : Find the correct mirror image of the given Directions (Qs.16 – 18): In the problem figures, a definite
figure, when mirror is placed on right side of the figure. relationship exists between fig. A and B. You are to establish a
(Rajsthan-2020-21) similar relationship between fig. C and D by choosing a suitable
11. Question Image : fig. D from the answer set.
16. Problem figures (Andhra Pradesh 2019)

Answer Image : (A) (B) (C) (D)

Answer figures

(1) (2)
(1) (2)

(3) (4)
(3) (4)
Solved Questions 51
A-

17. Problem figures (Andhra Pradesh 2019) 20. How many semicircles are there in the given figure?
(Karnataka 2019)

(A) (B) (C) (D)

Answer figures

(1) (2) (1) 8 (2) 10 (3) 12 (4) 14


21. Find the missing part of the given figure from the
alternatives.

(3) (4)

18. Problem figures (Andhra Pradesh 2019)

(A) (B) (C) (D)

Answer figures
(1) (2)

(1) (2)

(3) (4) (3) (4)

19. Complete the following figure analogy. (Karnataka 2019)


Directions (Qs. 22 – 23): In each of the following questions,
there is a specific relationship between the first and the second
figure. The same relationship exists between the third and the
fourth figure. Find the relation and choose answer to replace
the question mark.

22. Question Figure


(1) (2)
: :: : ?
(Maharashtra 2019)
(3) (4) Answer Figure

(1) (2) (3) (4)


EBD_7332
52
A- NTSE Stage 1 Question Bank

23. Question Figure


(1) (2)
: :: : ?
Answer Figure (3) (4)

28. The two positions of a single die are given below. Which
digit will be at the face opposite to the face having digit 4?
(Rajasthan 2019)
(1) (2) (3) (4)

24. Find the number of squares from the adjacent figure:


(Maharashtra 2019)

(1) 1 (2) 2 (3) 3 (4) 6


Directions: In question (29) there are four terms in each question.
The relation that exist between the terms left to the symbol :: is
the same between the terms right to the symbol ::. Out of the four
terms one term is missing in each question. The missing term is
(1) 6 (2) 11 (3) 13 (4) 10 one of the four alternatives given below each question. Find out
25. In the given question a complex figure is given. Find out the correct alternative and write its number on your answer
which of the figure given in the alternatives is hidden in sheet against the proper question -
the complex figure. (Maharashtra 2019)
29.
Question Figure

Answer Figure
(Uttar Pradesh 2019)
30. Find the number of quadrilaterals in the figure :
(1) (2) (Tamil Nadu 2019)
A

(3) (4)

(1) 0 (2) 2 (3) 3 (4) 4


26. How many triangles are there in the figure given below?
(Rajasthan 2019) Directions (Qs. 31 - 32): Select a figure from the four alternatives,
which when placed in the black space of given (X) would
complete the pattern. (Delhi 2018)

31.
(1) 10 (2) 8 (3) 11 (4) 12
27. Find the correct mirror image of the given fig. when mirror
is placed on the right side of the fig. (Rajasthan 2019)

(X)
Solved Questions 53
A-

37. In the adjoining figure, if the centres of all the circles are
(1) (2) joined by horizontal and vertical lines, then find the number
of squares that can be formed. (Odisha 2018)

(3) (4)

(1) 6 (2) 7 (3) 8 (4) 1


38. Based on the given positions of the dice find the number of
dots on the face opposite to the face with one dot.
(Odisha 2018)
32.

(X)
(i) (ii) (iii)
(1) 2 (2) 3 (3) 4 (4) 6
39. Find out from the four alternatives as to how the pattern
(1) (2) would appear when the transparent sheet is folded at the
dotted line. (Odisha 2018)

(3) (4)

33. Determine the number of rectangles in the following figure:


(Rajasthan 2018)
(1) (2)

(1) 7 (2) 8 (3) 9 (4) 10 (3) (4)


34. How many line segments are there in the given figure?
(Gujarat 2018)
Directions (Qs. 40 - 43): Match the following base on common
characteristics:
(Tamil Nadu 2018)

(1) 12 (2) 13 (3) 14 (4) 15


35. How many squares are there in the given figure? 40. (1)
(Gujarat 2018)

41. (2)
(1) 11 (2) 21 (3) 24 (4) 26
36. Find the set of figures which have similar characteristics,
choosing from the given alternatives. (Karnataka 2018)
42. (3)

A B C D E F
(1) A and C (2) B and E 43. (4)
(3) D and A (4) F and D
EBD_7332
A-54 NTSE Stage 1 Question Bank

Directions (Qs. 44 - 45) : In each of the following questions find


out which of the answer figures complete the figure. (1) (2)
44. Question Figure (Delhi 2017)

(3) (4)

47. ?

(1) (2)

(3) (4)

Answer Figure Directions (Qs. 48 to 50): Out of the four figure (1), (2), (3), (4),
given in each problem, three are similar in a certain way. Choose
the figure which is different from the other figures.
(1) (2) (Andhra Pradesh 2017)

(3) (4) 48.

45. Question Figure (1) (2) (3) (4)

49.

(1) (2) (3) (4)

Answer Figure 50.

(1) (2)
51. Select a suitable figure from the four alternatives that would
complete the given matrix. (Haryana 2017)
(3) (4)

Directions (Qs. 46 - 47): Select the correct alternatives which will


fit in the place of the sign of interrogation for a correct pattern.
(Delhi 2017)

46. ?
Solved Questions 55
A-

(1) (2)

(3) (4)

52. How many triangles are there in the following figure?


(Haryana 2017)
(1) (2)

(3) (4)

Directions (Q. 56) In each of the following question is there is


(1) 20 (2) 25 (3) 32 (4) 34 a specific relationship between the first and second figure. The
53. select the correct water image of the given figure. same relationship exists between the third and the fourth figure
(Rajasthan 2017) which will replace the question mark. Select the correct term
Question-figure from the given alternative.
A

56. ?
Answer-figures
(Maharashtra 2017)
(1) A (2)

(3) A (4) (1) (2)

Direction (Q. 54): A Square transparent sheet with a pattern is (3) (4)
folded along the dotted line. Which of the following answer
figures is formed after folding the transparent sheet? 57. If the figure given along side is folded to construct a cube,
find out the correct cubical figure from the given alternative
(Rajasthan 2017)
figures. (Maharashtra 2017)
54. Transparent sheet

Answer-figures

(1) (2) (1) (2) (3) (4)

58. The following figure is rotated in anticlockwise direction


and its mirror image is obtained. Select the correct mirror
image from four alternatives given. (Maharashtra 2017)
(3) (4)

55. Find out the correct image in place of question marks (?) (1) (2) (3) (4)
from the given alternatives. (Jharkhand 2017)
EBD_7332
56
A- NTSE Stage 1 Question Bank

Directions (Qs. 59 & 60) : Find the odd figure Answer Figures:
(Maharashtra 2017)

× ×=
59. (1) =* (2) * (1) (2) (3) (4)
° ° 65. Problem Figures: (Haryana 2016)

×= ×*
(3) * (4) =
° °
Answer Figures:
60. (1) (2) (3) (4)

61. Observe the adjoining figure and answer the following


question. Choosing the correct alternative.
(1) (2) (3) (4)
How many isosceles trapezium are in the figure?
(Maharashtra 2017) 66. The two positions on a dice are shown below. If 1 is at the
bottom then what will be on the top? (Rajasthan 2016)

(1) 16 (2) 10 (3) 8 (4) 14


62. The following question figure given at left side is incomplete.
Select the correct alternative which can complete the figure.
(Maharashtra 2017) (1) 2 (2 ) 3 (3) 4 (4) 5
Direction (Q. 67) : In question find the correct water image of
the given figure.
67. Questions Figure (Rajasthan 2016)

(1) (2) (3) (4)


720
63. Which one of the answer figure would occupy the next Answer Figures
position in the problem figure, if they continue in the same
order? (Haryana 2016)
Problem Figures
(1) (2) (3) (4)
X
X X ? Directions (Q. 68) : In the following question, figures showing
a sequence of folding and cutting a paper are given. Which
could resemble the figure in the Answer figure?
Answer Figures
68. Problem Figures (Rajasthan 2016)

X X X X
(1) (2) (3) (4)
Answer Figures
Direction (Qs. 64 - 65) : In the following questions, which one of
the answer figure would occupy the next position in the problem
figure. If they continue in the same order.
64. Problem Figures: (Haryana 2016)
(1) (2) (3) (4)
Solved Questions 57
A-

69. Look at the sequence of symbols to find the pattern. Select Direction (Q. 74) : In the following question there is a specific
correct pattern at the (?) mark. relationship between the first and second figure. The same
relationship exists between the third and fourth figure which
will replace the question mark. Select the correct term from the
?
alternatives given. (Maharashtra 2016)

(1) (2) (3) (4) 74. ?


(Odisha 2016)
(1) 1 (2) 2 (3) 3 (4) 4
70. Four positions of a dice are shown. Which symbol or number (1) (2)
will be on the face opposite to the face with symbol (star)?

@ @ $
(3) (4)
+ + +
– $ 8
75. A square piece of paper is folded and cut at specific spot as
(Odisha 2016) shown in the figure. The paper when unfolded will look like
(1) @ (2) $ (3) 8 (4) + as shown in one of the alternatives. Select the correct
71. Which letter represents the question mark? (Odisha 2016) alternative. (Maharashtra 2016)

D G
W T
NM J Q
C L
X ?
I B
R Y
(1) M (2) O (3) Q (4) R
72. Which number replaces the question mark? (Odisha 2016)

(1) (2)
4 13 3 21
12 7 9 5 9 6 8 5 2 5 12 ?
3 12 7 16

(1) 4 (2) 6 (3) 8 (4) 12


(3) (4)
Direction (Q. 73): Select a figure form amongst the Answer
figures which will continue the same series as established by
the five problem figures. 76. The following figure is folded to form a cube. Observe the
73. Problem Figures (Odisha 2016) cube. Of the following cube figure find the most appropriate
figure. (Maharashtra 2016)

(A) (B) (C) (D) (E)


EBD_7332
58
A- NTSE Stage 1 Question Bank

Directions (Q. 81) : In each of the following questions, find the


figure from the answer-set (i.e., 1, 2, 3 and 4) which will continue
(1) (2)
the series given in the problem set (i.e. A, B, C, D and E).

81. C C C
(3) (4)
C C
Direction (Q. 77) : In the following question figures change in a A B C D E
particular order. Find out the correct figure from the alternative
which will replace the question mark?
C C C C
(1) (2) (3) (4)
77. ?
(Jharkhand 2015)
(Maharashtra 2016) 82. Directions- Choose the water image of the ‘Question Figure’
from the given alternatives. (Maharashtra 2014)
Question Figure:
(1) (2)

(3) (4)

Directions (Qs. 78 - 79) : In every question a dice has been Answer Figures:
shown in different faces on which numbers/symbol/colour have
been written randomly. Carefully study the faces and answer the
question based on it.
(1) (2)
78. Which symbol is just opposite to symbol ' '?

0 + × 0 + + 0

(Jharkhand 2015)
(3) (4)
(1) (2) +
(3) 0 (4)
79. Which number will come at the bottom of last cube ? 83. Select a suitable figure from the answer figure that would
replace the question mark (?) (Maharashtra 2014)
3 Question Figure:
4 5 4 2 5
+
× + ?
(Jharkhand 2015) ×
(1) 3 (2) 4
Answer Figures:
(3) 6 (4) 1
Direction (Q. 80) : Choose the correct water image of the given ×
word numbers from amongst the alternatives.
80. PR9YA (Jharkhand 2015)
×+
(1) RPY9A (2) +
(3) (4) (1) (2) (3) (4)
Solved Questions 59
A-

Directions (Qs. 84 - 86) :- Observe the following figure and 87. (Karnataka 2014)
answer the following questions by choosing the correct
alternative given below.

?
(1) (2)

84. Find the number of equilateral triangles in the given figure. (3) (4)
(Maharashtra 2014)
(1) 32 (2) 18
Direction (Q. 88) : Identify the number of specified geometric
(3) 5 (4) 0
shapes in the given diagram and mark the correct answer.
85. Find the number of parallelograms which are not rectangles
from the given figure. (Maharashtra 2014) 88. How many triangles are in the given figure? (Karnataka 2014)
(1) 4 (2) 6
(3) 8 (4) 10
86. A square piece of paper is folded and cut at specific spots
as shown in the figure. The paper when unfolded will look
as shown in one of the alternatives. Select the correct
alternative. (Maharashtra 2014)
Question figure:
(1) 21 (2) 22 (3) 23 (4) 24
Direction (Qs. 88) : Find the correct mirror images for the
following problem figures choosing from the alternatives.
89. Problem figures (Karnataka 2014)

Answer Figures:

(1) (2)

(1) (2) (3) (4)


Direction (Q. 87) : Complete the following number/letter/figural
series by selecting from the given choices.
(3) (4)
EBD_7332
A-60 NTSE Stage 1 Question Bank

Direction (Q. 90) : Choose the figure from the alternatives which F F E D
is suitable to put the dot ( ) as found in the problem figure. (i) B E (ii) E D (iii) (iv) E A
B C
90. Problem figure (Karnataka 2014)
(1) A only (2) B only
(3) A and C only (4) A, B, C and D
Directions (Qs. 94 - 95) : Two sets of the figures are given. One
set of Question-figures and another set is of Answer figure.
Question-figures are arranged in a sequence. One figure from
(1) (2) the Answer figures is to be selected such that it can be placed
after the series of Question-figures. Find the correct serial number
of the selected Answer-figure.
94. Question-Figures : (Rajasthan 2013)
(3) (4)

Direction (Q. 91): Choose which one of the figures shown in the
alternatives will be formed by joining the given parts of the figure.
91. (Karnataka 2014) Answer-Figures :

(1) (2)

(3) (4)
(1) (2)
95. Questions-Figures : (Rajasthan 2013)

(3) (4)

Answer-Figures :
92. The following figure is converted into a cube. Its correct
shape will be: (Rajasthan 2013)
2 (1) (2)
5 1
4
6 3

3 1 2 3 (3) (4)
(1) 6 1 (2) 3 5 (3) (4) 1 4
6 4
93. The following figure is converted into a cube. Its four 96. What will be the number of Parallelograms in the given fig-
positions (a), (b), (c) and (d) are shown. On the basis of ure ? (Rajasthan 2013)
these select correct alternative. (Rajasthan 2013)
F
A E
B
C D (1) 15 (2) 17 (3) 13 (4) 16
Solved Questions A-61

Directions (Qs. 97 – 100) : Select the alternative which correctly Which number is opposite 3 ? (Delhi 2012)
(1) 1 (2) 2 (3) 4 (4) 6
depicts how a paper will appear, when it is folded along the
102. If the total number of dots on opposite faces of a cubical block
dotted line? is always 7, find the figure which is correct? (Delhi 2012)
(Delhi 2012)

(1) (2)
97.

X (A) (B) (C) (D) (3) (4)


(1) A (2) B (3) C (4) D

Directions (Qs. 103 - 105) : The five figures marked as A, B, C &


98. D in the following questions follow a series. Select a figure from
the given alternatives marked as 1, 2, 3 and 4 which will continue
X (A) (B) (C) (D) the same series.
(1) A (2) B (3) C (4) D (Punjab 2012)

••
99. •• 103.

(Delhi 2012)
A B C D

(1) (2)

(1) (2) (3) (4)


(3) (4)
C O C O

100. O C
104.
•• C O
•• A B C D

(Delhi 2012)
O C

(1) (2) O C O C C O
(1) (2) (3) (4)
105.

(3) (4)

101. Observe the dice given below and answer: A B C D


2 3
(1) 1 4 (2) 6 5

4 5
(3) 5 1 (4) 4 6 (1) (2) (3) (4)
EBD_7332
A-62 NTSE Stage 1 Question Bank

Directions (Qs. 106 - 109) : In each of the following questions,


figure X is given with a part missing. 112.
Choose the alternative which will complete the missing part. (1) (2) (3) (4)
(Punjab 2012)
106. 113.
(1) (2) (3) (4)

? 114.
X (1) (2) (3) (4) (1) (2) (3) (4)
107.

115.
(1) (2) (3) (4)
?
Directions (Qs. 116 – 117) : In each of the questions, there is a
X (1) (2) (3) (4) big figure at the top having a vacant chamber. Identify one
108. figure from the given choices which will fit into the vacant chamber.
(Odisha 2012)
116.

X (1) (2) (3) (4)


109.

? (1) (2) (3) (4)


X (1) (2) (3) (4) 117. Choose the mirror image of the figure given at the top from
amongst the four alternatives. (Odisha 2012)
Directions (Q. 110) : A wooden cube is painted Blue on all four
adjoining sides and Green on two opposite sides i.e. top and
bottom. It is then cut at equal distances at right angles four times
vertically (top to bottom) and two times horizontally as shown
in the figure where dotted lines represent the cuts made. Study
the diagram and answer the questions that follow.

(1) (2) (3) (4)


Directions (Qs. 118 – 119) : In each question there is a figure
and answer the question from the figure. (West Bengal 2012)

118.
(Punjab 2012)
110. How many cubes will have no side painted at all?
(1) 1 (2) 2 (3) 3 (4) 4 How many quadrilaterals are there in this figure?
(1) 8 (2) 9 (3) 10 (4) 11
Directions (Qs. 111 – 115) : In each of the questions, a part of
the figure is given. Select one from the given four figures in 119.
which that part is embedded. (Odisha 2012)

111. How many triangles are there in this figure?


(1) 10 (2) 12 (3) 8 (4) 9
(1) (2) (3) (4)
Solved Questions 63
A-

Directions (Qs. 120 – 121) : In these questions, there are four 123. Question figures :
problem figures (a), (b), (c) and (d). One of them is, however,
missing. You have to select one figure from the five answer figures,
(1), (2), (3), (4) and (5) such that the series is completed. Indicate
your answer as per the “Instructions”. (Andhra Pradesh 2012)
120. Problem figures:

? ?
(a) (b) (c) (d)
Answer figures:

(1) (2)

(1) (2) (3) (4) (5)


121. Problem figures:
(3) (4)

?
(a) (b) (c) (d) Directions (Q. 124 ) : Find the odd figure out.
Answer figures: (Maharashtra 2012)

O O
124. (1) (2)
X S X S
(1) (2) (3) (4) (5)
Directions (Qs. 122 – 123) : In each of the following, question
figures change in a particular order. Find out the correct figure
from the alternatives which will replace the question mark ? O O
(3) (4)
(Maharashtra 2012) X S S X
122. Question figures :
Directions (Qs. 125 – 126) : Question figure given to the left hand
side is held vertically on water surface. How will the reflection of
the given figure in water look ? Select the correct alternative.
(Maharashtra 2012)

?
125.

Question figure :

(1) (2)
(1) (2)

(3) (4)
(3) (4)
EBD_7332
A-64 NTSE Stage 1 Question Bank

12 12
11 1 11 1
126. 10 2 10 2
9 3 9 3

Question figure : 8 4 8 4
7 5 7 5
6 6

(1) (2)
12 12
11 1 11 1
10 2 10 2
9 3 9 3
(1) (2)
(3) (4) 8 4 8 4
7 5 7 5
6 6

Directions (Q. 127) : In the following figure, the solid cube is


12 12
painted on all sides by a single colour. Observe the given solid and 11 1 11 1
choose the correct alternative for the following questions. 10 2 10 2
9 3 9 3
(3) (4)
8 4 8 4
7 5 7 5
6 6

Directions (Q. 129) : Find the correct mirror is for the following
figures from the alternatives. (Karnataka 2011)
(Maharashtra 2012)
127. Find total number of blocks whose not even one surface is 129.
coloured.
(1) 12 (2) 16 (3) 8 (4) 24
Direction (Q. 128): Complete the figure series, choosing the (1)
correct figure from the given alternatives. (Karnataka 2011)

(2)
128. 12 12
11 1 11 1
10 2 10 2 (3)
9 3 9 3

8 8 (4)
4 4
7 5 7 5
6 6
Solved Questions 65
A-

HINTS & SOLUTIONS


CHAPTER 1. KNOWLEDGE BASED REASONING 12. (3) Fourth to left of I is (E) and 16th to the right of E is (U).
1. (3) Evolution is proposed by Darwin, Buoyancy is 13. (3) P A L M
proposed by Archimedes. 34 42 86 24
2. (4) Snake bite with Fang, Bee bite with sting.
3. (2) Tea is taken in cup, Tobacco enhanced through 14. (4) W A R D
Hookah. 13 04 40 75
4. (4) Aeroplane is controlled by cockpit, train is controlled
by engine. 15. (4) KP
5. (2) Assign numbers to alphabets and add them. B + 2 = D + 3 = G + 4 = K.
(NEW JERSEY = 14 + 5 + 23 10 + 5 + 18 + 19 + 5 + 25 Y – 2 = W – 3 = T – 4 = P.
= 124) 16. (2) KLML
6. (2) Replace each alphabet in BOMBAY with the next LKMN, LKMN
alphabet. 17. (2) WORK STATE
M A D R A S 2357 146 49
+1 +1 +1 +1 +1 +1 18. (1) BALL JACK
2 × 1 × 12 × 12 10 × 1 × 3 × 11
N B E S B T
Option (1) - 330
B O M B A Y
–5
+1 +1 +1 +1 +1 +1 –5

C P N C B Z 19. (3) C D H I M N R S
+5 +5
+1 +2 +3 +4
+5 +5

7. (2) 3 4 6 9 13 20. (4) B (2) × M (13) = 26


7 11 19 31 47
E (5) × N (14) = 70
+4 +8 +12 +16
H (8) × O (15) = 120
×5 ×5 K (11) × P (16) = 176
N (14) × Q (17) = 238
8. (3) 625, 5, 125, 25, 25, 125, 5 21. (4) GECA, ZXUT, SQOM
÷5 ÷5 ÷5 –2, –2, –2, –2 in each term
9. (4) 4, 8, 28, 80, 244, ____ So, option 4 satisfies the condition.
31 + 1 = 4 22. (1) BEIN, EHLQ, ILPU
32 – 1 = 8 +3, +4, +5, +6 in each term.
33 + 1 = 28 So, option (1) satisfy given condition.
34 – 1 = 80 T=4
35 + 1 = 244 STORE = 14359
36 – 1 = 728
+1 +1

10. (2) A J S, G P Y, M V E, S B K, Y H Q 23. (2) 416, 525 749 864


2 2 2 2
(4) (5) (7) (8)
1 7 13 19 25
+6 +6 +6 +6 24. (1)
10 16 22 2 8
+6 +6 25. (2) R V A G N V
=V
+4 +5 +6 +7 +8
19 25 31
+6 +6
= E = 31 – 26 = 5 26. (4) The first two letters of a term are shifted to the last to
form the next term of the series.
11. (2) H I J K L M N O P
So after, DNINETEENCOVI, the next term will be
INETEENCOVIDN.
EBD_7332
66
A- NTSE Stage 1 Question Bank

27. (3) The pattern is, As, Similarly,


46. (1)
4× 1+1=5
5 × 2 + 2 = 12 CONSECUTIVE CONSECUTIVE

12 × 3 + 3 = 39 C L O+3 U D S I G+3 H T
–1 –1 –1 –1
39 × 4 + 4 = 160 +3 +3 +3 +3
160 × 5 + 5 = 805
28. (4) The sequence is based on the growth of a crop from a
seed. G T R K F W G J H V
The sequence will be : Seed, Root, Stem, Flower, Crop 47. (1) CE : 70 : : DE :
i.e. 4, 2, 3, 5, 1.
29. (3) All except Guru Granth Sahib are coming among the C E 7 0
We have,
first ten Sikh Gurus. D E x y
30. (1) All are perfect cubes except 289. y=0
31. (4) All second word is the part of the first word except As, C = 7=3×2+1
Moon-Bird. (Here 3 is place value of C)
32. (2) ‘n’ repeated the most. Similarly, D = 4 4×2×+1=9
33. (3) Three times. (Here 4 is place value of D)
ijo opu apu DE = 90
34. (3) onp akt etb 48. (2)
Three times.
As,
35. (2) ccba | ccba | ccba | ccba
7 3 8 4
36. (2) Pie is circular representation of data, while rest are
rectangular / tabular format. 4 8 3 7
37. (4) All other are virus infections, except Typhoid.
Similarly,
38. (1) The words in each pair are antonyms.
5 2 9 1
39. (2) ‘Shirt’ belong to ‘apparel’, ‘necklase’ belongs to
‘jewellery’. 1 9 2 5
40. (1) (c) Colour (e) Create 49. (1) We can see the pattern follows by
(a) Credential (d) Credible +2 +2 +2 +2

(b) Creed
F O X, I Q V, L S T, OUR R WP
41. (3) As, P M G : S I X

+3
GH JK MN PQ
–4
P R T V
+17
50. (3) We can see in the option (3)
Similarly, T I N : W E E O comes before N 3, 5, which is the right sequence
+3 So, 3, 5, 4, 2, 1 is correct.
–4 51. (3) As, sum of the place value of the GARDEN = 7 + 1 +
+17
18 + 4 + 5 + 14 = 49
42. (1) As, A F C : D I F Similarly, sum of the place values of the FLOWER is
6 + 12 + 15 + 23 + 5 + 18 = 79
+3
+3 52. (3) As, F + 1 (on last written)
+3
F R A G R A N C E
Similarly, B E D : E H G +1 +1 +1 +1 +1 +1 +1 +1 +1

+3
S B H S B O D F G
+3 Similarly, I + 1 (on last written)
+3 I M P O S I N G
43. (2) First works for the second. +1 +1 +1 +1 +1 +1 +1 +1

Doctor is related to patient. Lawyer is related to client. N Q P T J O H J


44. (3) As, P.V. Sindhu is a badminton player. 53. (1) According to question,
Similarly, Deepak Puniya is a wrestler. 86 = P 12 = A 31 = S 76 = T
45. (4) R A C E So, option (1) is correct
Letter C is not available in FRAGILE
Solved Questions A-67
54. (4) According to the matrix code, 69. (2) Pattern follows as,
21 = N, 67 = E, 14 = S, 59 = T
So, option (4) is correct
12
55. (4) As, (12) 2 864
2
18 70. (2) Number along with alphabet is a squerge number.
Similarly, (18)2 2916
2 Hence, L19 is correct.
56. (1) The sum of the digits are in multiple of 3. Hence 393
is also multiple of 3. 71. (2) In the given option only Deer is Herbivorous rest are
carnivores
57. (2) Sum of digits are equal to 13. Hence 8 + 2 + 3 = 13
72. (1) In the following we get the pattern as like twice of
58. (1) Sum of digit is equal to 24.
32.5 is not divisible by 3
Hence, 2 + 1 + 3 + 5 + 4 + 9 = 24
73. (3) Here we apply the sum of place value of the letters.
59. (3) We get ZPX in the next.
BOUND 2 + 15 + 21 + 14 + 4 = 112
WHITE = 23 + 8 + 9 + 20 + 5 = 130
Position of X Position of D
74. (3) ,
2 2
60. (4) And the gap of +4 between the Alphabet Position of R Position of L
,
Z A B W D E S H I N M N H S G 2 2
–3 –4 –5 –6 So, code of XDRL is ‘12296’.
+3 +4 +5 +6 Simililarly,
+3 +4 +5 +6
61. (3) The given figure represents the symbol like as Position of N Position of H
,
872 + 634 = 1506. 2 2
Hence option (3) is correct.
Position of T Position of V
62. (2) As A D E:F G I ,
2 2
+3 +1 +1 +3 So, code of NHTV is ‘741011’.
Similarly, K N O: P Q T 75. (2) As RUBBER = BERRUB, Similarly BUTTER
+3 +1 +1 +3 = TERBUT
63. (3) Code for DONAR = 45392 76. (1) As, RSTU VWXY –
64. (2) Pattern follows as when applying the mathematical Reversing the letter – YXWV
operations. GHIJ KLMN
88 – 7 = 39, 77 – 6 = 41,
Similarly reversing the letters – NMLK
99 – 5 = 74, 55 – 4 = ?
77. (1) As oil is used for lamp, electricity is used for lighting
(88 – 7) = {88 – 7 – 7(7 – 1)} = (81 – 42) = 39 the bulb.
(77 – 6) = {77 – 6 – 6(6 – 1)} = (71 – 30) = 41 78. (3) Pattern for the given series follows similarly as;
(99 – 5) = {99 – 5 – 5(5 – 1)} = (94 – 20) = 74 841 = (29)2, 289 = (17)2
(55 – 4) = {55 – 4 – 4 (4 – 1 )} = (51 – 12) = 39
79. (2) In the above options others are the elements.
65. (3) Letter B R A I N T E But Steel is not element.
Code * % ¸ # × $ + 80. (4) In the above options first term is greater than second
Henc RENT is written as term
%+×$ 81. (1) Words arrangement according to dictionary are:
66. (1) Code for COBRA is Position of letters in alphabet. Leader (5), Leaf (1), Leak (4), Learned (2), Leave (3).
Hence code of COBRA is as 3152181. 82. (1) Sea is to waves. So, in the same way. Candle is related
67. (1) First we get the Advertisement, then we apply, after to the rays.
that we go for an interview, so the right sequence is 83. (2) We get the code as,
5, 6, 2, 3, 4, 1
68. (2) Here pattern is the Ist no.Which is divided by 7. As H O N E Y Similarly F O R C E
21 +0 +1 +2 +3 +4 +0 +1 +2 +3 +4
3 H P P H C F P T J I
7
84. (3) Plant is the smallest form of tree, in the same way girl
574 is the younger one women.
82
7
EBD_7332
68
A- NTSE Stage 1 Question Bank

85. (3) Floor is the opposite of Roof. In the same way Dark 98. (1) 25 255 2545 25455 254545
is the opposite of Bright.
86. (1) We can see in the option (1) ×10+5 255×10–5 2545×10+5 25455×10–5
99. (1) From given statements :–
A comes before I (d), (c), which is a right sequence. Blue 2
So, (d), (c), (b), (a) is correct. Sky 1
87. (1) Letters arranged from a meaningful word is Was 3
P E R S O N People 8
Like 0
| | | | | | In 0
3 5 1 6 2 4 Bird 9
88. (1) SHOULDER TJSNMAGZ ‘People like birds’ 809
BOXING RSYCPH 100. (2) The pattern is:
13, 74, 290, 650
Comparing the values of above examples- we get 22 + 32 = 13
HORN JSZP 52 + 72 = 74
89. (3) Study - Examination - Result - Service - Earning 112 + 132 = 290
172 + 192 = 650
ACEBD
232 + 292 = 1370
90. (4) By (1), 12 22 5 (1, 5, 2) So, missing number is 1370.
101. (4) A – BBC – AAB – CCA – BBCC
By (2), 72 + 82 = 113 (7, 113, 8) A A BB/CCAA/BBCC/AABB/CC
By (3), 22 + 42 = 20 (2, 20, 4) 102. (4) BAAB / BAAB / BAAB
ABBA is answer
By (4), 32 + 42 = 25 (3, 25, 4)
Sol. (103-104)
So option (4) should be (3, 25, 4) but not (3, 27, 4) M 4 (as 48 is common number in code for MSPTQ & ABLMS)
91. (3) Given series consists of 3 patterns. N 7 (as 1 is common number in code for PTQAB & ATRNP)
2 2 2 2
103. (4)
Y W U S Q 104. (4)
1 2 3 4 105. (4) AS, K M F Similarly, R M S
E F H K O
+1 –1 +2 +1 –1 +2
2 3 2 3
B D G I L L L H S L U
92. (2) CHARACTER 241612376 106. (2) As, Smoke Pollution
CHILDREN 24859670 Similarly, War Death
Comparing the values from above examples 107. (2) According to dictionary;
HIRALAL 4861515 3. Dissent
93. (1) Adding reverse positions of alphabets 1. Dissident
4. Dissolute
SOME = 8 + 12 + 14 + 22 = 56 5. Dissolution
94. (1) S C O I LAN D 1 2 3 4 5 6 7 8 2. Dissolve
LOA N 8 1 2 4 So, option (2) is correct.
DAN 537 108. (4) Except option (4) all are antonyms. But Option (4) is
Synonyms.
By eliminating the values, we get c 6
109. (3) There is L in MISSILE which is not present in COM-
95. (3) T< R<S< Q<O<P MISSIONER. So, MISSILE cannot be formed using the
P is the biggest of all letter of the given word.
96. (1) 110. (2) The pattern is :
25
T X
97. (3) : 2 :: : ?
J H 13 17 19 23 27 29

Position of T in alphabets = 20 4 2 4 2 4
So, 27 is wrong number.
Position of J = 10
111. (2)
G, K, O, S, w
T 20 7 11 15 19 23
2
J 10

X 24 +4 +4 +4 +4
Similarly, 3 So, missing term is W
H 8
Solved Questions 69
A-

112. (3) Similarly,


3 9 20 9 26 5 14
KLE, IND, GPC, ERB, CTA C I T I Z E N
–1 –1 –1 –1
+5 +5 +5 +5 +5 +5 +5
11 12 5 9 14 4 7 16 3 5 18 2 3 20 1
X D O D U Z I
24 4 15 4 21 26 9
+2 +2 +2 +2 120. (4)
121. (2) According to question,
–2 –2 –2 –2
63 : 80 : : 120 : 137
So, missing term is ERB.
+17 +17
113. (1) The pattern is:
122. (1) The Pattern is :
1, 3, 7, 13, 21, 31, 43, 57
+2 +4 +8 +16
+ 2 + 4 + 6 + 8 + 10 + 12 + 14 BJ DL HP PX FN
So, missing number is 31.
+2 +4 +8 +16
114. (3) The pattern is The next term = FN.
5 , 3 , 10 , 8 123. (1) Only 141 is not a prime number but other numbers
, 17 ,15 ,26 24
, are prime number.
–2 +7 –2 +9 –2 + 11 –2 124. (4) The pattern is :
+2 +2 +2
+2 +2 3 4 1, 5 7 2, 7 8 1, 9 9 0
So, missing number is 26.
3+1=4 7+1=8
115. (2) All are countries and kabul is city
116. (1) According to question, P + R – Q 5+2=7 9+0=9
– + So, option (4) is correct answer.
R Q 44
125. (1) As, ZEAL = 26 + 5 + 1 + 12 11
4
+
P 28
Similary, B E A T = 2 + 5 + 1 + 20 7
So, Q is father of P. 4
117. (3) The pattern is: Logic Sum of all place then divide by 4.
+3 +3
126. (4) 02, 13, 34, 56
Check row and column.
18 24 21 27 24 30 27 127. (2) The venn diagram is as following:

+3 +3 +3
So, missing number is 24
118. (4) The pattern is : Crows Doves
ts
rro
+2 +2 +2 Pa Cats

ELFA, GLHA, ILJA, KLLA, MLNA


Conclusions:
+2 +2 +2 I. II.
So, missing term is KLLA. So, only II follows.
119. (3) As, 128. (4) The pattern is as follows:
h feg/hfeg/hfeg/h feg
19 5 14 9 15 18
129. (3) The venn diagram is as shown below :
S E N I O R

+5 +5 +5 +5 +5 +5
Dancers Singers Teachers
N Z I D J M
14 26 9 4 10 13
Conclusions : I. II.
So, both conclusions I and II are true.
EBD_7332
70
A- NTSE Stage 1 Question Bank

130. (4) The venn diagram is as shown below : P C K – 3 0 4 ...(iii)


K N D – 4 7 8 ...(iv)
Fruits Mangoes From (i) (ii) and (iii) L 5
From (i) & (ii) J 9
From (iii) & (iv) K 4
So, the code for LJK will be 594.

Guavas 139. (1) E J P – 0 9 2 ...(i)


P C K – 3 0 4 ...(ii)
K N D – 4 7 8 ...(iii)
Conclusions : I. II.
So, Neither conclusion I nor conclusion II is true. N E V – 7 2 1 ...(iv)
S C H O O L From (i) & (ii) P 0
131. (3) As, +2 –2 +2 –2 +2 –2 From i iii and (iv) V 1
U A JM Q J From (ii), (iv) and (iii) D 8
So, the code for PVD will be 018.
P R I N C I P L E
R E A S O N
Similarly, +2 –2 +2 –2 +2 –2 +2 –2 +2
140. (2) As, +1 +1 +1 +1 +1 +1
R P K L E G R J G S F B T P O
132. (1) The code for 62830 will be written as HATCB T H I N K
133. (1) 'BHICK' will be coded as 06734. Similarly, +1 +1 +1 +1 +1
G I L P U I J O L
134. (3) As, ,
+2 +3 +4 141. (2) By observation, the 'guest room' Means 'vosefstrisi'
D F I M 142. (1) The given alphabet series is
+2 +3 +4 x c w m v c x w m w m x c wm x m
E G J N So, only 1 time m is succeeded by w and preceded
and by w.
+2 +3 +4
B D J K 143. (3) 144. (3) 145. (1) 146. (2) 147. (3)
But, 148. (2) 149. (1) 150. (2) 151. (4) 152. (3)
+2 +6 +1 153. (2) 154. (2) 155. (3) 156. (1) 157. (2)
So, BDJK is different.
158. (3) 159. (4) 160. (4) 161. (2) 162. (1)
135. (2) Except option (2), in all other 'T' and ' 'are opposite to
163. (3) 32 – 1 : 33 + 1 : : 42 – 1 : 43 + 1
each other. i.e. missing term is 15.
Sol (136-139) 164. (2) 23 + 33 : 33 + 43 : : 43 + 53 : 53 + 63
136. (4) E J P 0 9 2 ...(i) So missing term is 341.
K N D – 4 7 8 ...(ii) 7 13 19 29
165. (2) All are prime number : :: :
P C K – 3 0 4 ...(iii) 11 17 23 31
N E V – 7 2 1 ...(iv) 166. (3) S is the 3rd letter after P and W is the 3rd letter after T.
From (ii) & (iii) K 4 Similarly, D & H are the 3rd letters after A and E
From (ii) & (iv) N 7 respectively.
From (i) & (iii) P 0 167. (3) Except speed all are form a group. Tyre, Engine & Fuel
So, the code for KNP will be 470. are part of a vehicle.
137. (2) C J L – 3 5 9 ...(i) 168. (1) As race leads to Fatigue in the same way fast leads to
Hungry. Rest are irrelevant.
E J P – 0 9 2 ...(ii)
169. (2) Penology is the study of punishment in the same say
P C K – 3 0 4 ...(iii)
seismology is the study of earthquake.
N E V – 7 2 1 ...(iv) 170. (4) Option (4) is the correct order. Cell is the smallest unit
From (i) & (iii) C 3 of life. An organ has many tissues.
From (i) & (ii) J 9
From (ii) & (iv) E 2 171. (3) LAP and CAR both have ‘A’ at the same position.
So, the code for CJE will be 392. 172. (4) All 5 will have no vowels – LBP, TVB, CBR, SPN, HJD.
138. (3) C J L – 3 5 9 ...(i) 173. (3) The correct alphabetical order is:-
E J P – 0 9 2 ...(ii) Seldom > Select > Selfish > Seller > Send.
Solved Questions A-71
174. (2) Permit cannot be made from the letters of the given
word as I letter is not mentioned in the given word. s u l s u l : p x i v r o
175. (3) Talent cannot be made from the letters of the given
word as E letter is not mentioned in the given word. –3
176. (2) The correct order is– +3
–3
Avdash > Avadhesh > Awadesh > Awdijesh
+3
Avadhesh would be in second place. –3
177. (2) Mathematics is related to the numbers in the same way +3
History is related to Events.
178. (1) Sanitation related to public health. 191. (2) RATIONAL : RATNIOLA TRIBAL : TRILBA
179. (4) Except cube, all other are circular figures. First 3 Remain Same Place TRI
rd
Then coming III one LBA TRIBAL
180. (4) Except (4) all other are measurement unit like time is to
be measure is seconds. Comes in IVth place. Then
left two took place and last
181. (3) Except (3) all are prime numbers.
if digit comes inter change them.
181-182. M 9 I 7
192. (1) HI : 36
S 6 T 5
H=8
A 4 K 1 Place value
I=9
E 2 N 8
D 8 72
8×9= 36
182. (1) D 3, I 7, S 6, T 5, A 4, N 8, T 5 2
183. (4) S 6, T 5, A 4, I 7, N 8 DE : 10
184. (3) No. of alphabets in REASON = 6 – 1 = 5 D=4
No. of alphabets in BELIEVED = 8 – 1 = 7 E= 5
GOVERNMENT = 10 – 1 = 9.
185. (2) Pattern is– 20
4×5= 10
2 3 2 3
2
P R O Q N
6 6 4 4
So that, DF CEG is out.
193. (4)
186. (4) Pattern is– (9, 15, 21) (4, 8, 12)
2 2 2 1 Comm. Diff. = 15 – 9 = 6 Com. Diff. 8 – 4 = 4
A C E G F
21 – 15 = 6 12 – 8 = 4
So that, UWYZA is out.
2 3 3 2
194. (1) (223, 324, 425) (554, 655, 756)
187. (3) C A D G E
So that, ECFGI is out. +101 +101 +101 +101
188. (4) Except (4) all others are odd numbers. 195. (4) VOWELS Only “OU” Comes in
M U M B A I
A I O U E a sequence of cronical order.
–1 –1 –1 –1 –1 –1
189. (3)
196. (1) “PUT” Not start with vowel among these options.
L T L A Z H
Similarly, 197. (4) “GFE” Its only in sequence
E F G
D E L H I 198. (2) “IJL”. Its only has one vowel among these.
–1 –1 –1 –1 –1 199. (4) Bengaluru is the capital of Karnataka. Similarly, Kohima
is the capital of Nagaland.
C D K G H 200. (3) London is the capital of UK, similarly, Bangkok is the
190. (4) c o r d e n : z r o g b q capital of Thailand.
201. (2) In hockey, ball is hit with a stick, similarly in Tennis,
–3 ball is hit with a Racket.
+3 202. (2) Oxygen is processed by the heart, similarly fuel is
–3 processed by engine.
+3
203. (1) Orange is fruit while spinach is vegetable.
–3
+3
204. (4) F 6, H 8, C 3, I 9
EBD_7332
A- 72 NTSE Stage 1 Question Bank

205. (3) G 7, A 1, D 4, B 2, H 8 3. (1) 40 – 10 + 5 ÷ 4 × 5 = 21


206. (3) H 8, B 2, D 4, F 6, E 5 After changing signs
207. (2) G 7, C 3, E 5, A 1, I 9 40 ÷ 10 × 5 – 4 + 5 = 21
208. (4) I 9, H 8, G 7, F 6, E 5, D 4 20 4 5 21
209. (3) 1 25 24 4 Forward place value 21 21.
A Y X D of alphabets
4. (4) 10 25 108 154 225

Z B C W
1 25 24 4 Reverse place 10 25 108 154 15
value of alphabets
Similarly,
10 25 108 169
10 16 15 13
J P O M
10 25 108 13

Q K L N
10 25 121
10 16 15 13
210. (4) K H O T 10 25 11

10 36 10 6 16 4.
11 + 8 + 15 + 20 = 5 4 = 45
Similarly, 24 7
5. (4) tan , cos
V I L E 7 25
14 tan 75cos 7 sec

22 + 9 + 12 + 5 = 4 8 = 84 24 7 25
14 75 7
7 25 7
211. (2) +1 = 48 – 21 – 25 = 2.
A E H : I M P :: B F I : J N Q
6. (2) Horizontal and vertical distance are same.
+1
+1 perpendicular
+1 tan 1
+1 base
+1
45
212. (3) ABGENO = 6 + 4 + 8 + 9 + 12 + 12 = 51
7. (3) 18, 50, 98, .......
CHAPTER 2. ARITHMETICAL REASONING
3 2, 5 2, 7 2
1. (3) an 3n 4
Put n = 1, 1, 3 [series will be 7, 10, 13, ....] 9 2 162
a 7, d 3, n 12
8. (4) ap q m, a p q n
n
s12 [2a (n 1)d ] a ( p q 1)d m, a ( p q 1)d n
2
12 Add both equations
[2(7) (12 1)3]
2 2a 2( p 1)d m n
= 6 (14 + 33) = 282.
m n
2 1 pth term a ( p 1)d
2. (1) x 2 98 2 2
x2 9. (2) Students who passed the examination
1 = (16 + 29) – 1 = 44.
x 10
x So, total number of students in the class
3 = 44 + 6 + 5 = 55.
1 1 1
x3 x 3 x 10. (2) (3 15 19) 8 6 2
3 x x
x
103 3 10 1000 30 970. 11. (4) (252 9) 5 32 92 200.
Solved Questions 73
A-

12. (1) Student who don’t drink milk = 52 – 24 = 28


92 104 40000
Student who don’t drink Tea = 52 – 28 = 24 x 20000.
Student who don’t drink anything = 8 23 2 2

20
Don't drink anything No. of English books 20000 4000.
100
21. (1) Let the total no. of books be x.
Then
20 8 16
2% of (23% of x ) 1% of (20% of x)
555555555555 555555555555
Kannada English
Not drink Milk Not drink Tea
6% of (14% of x ) 4% of (16% of x )
No. of students who drink both Milk and Tea 555555555555 555555555555
Hindi Telugu
= 52 – (20 + 8 + 16)
= 52 – 44 = 8. 8% of (27% of x)
555555555555 = 860
13. (2) 10R192P48S48P96Q1 Tamil
10 192 48 48 96 1
2 23 1 20 6 14
10 4 0.5 1 10 2.0 1 11 2 9. x x x
100 100 100 100 100 100
14. (2) 45 ÷ 9 × 4 + 5 = 5 × 4 + 5 = 20 + 5 = 25.
15. (4) Sister 4 16 8 27
x x 860
100 100 100 100
I
46 x 20 x 84 x 64 x 216 x
4 4 4 4
860
10 10 10 10 104
F
430 x
860
104
MK M
860
x 10 4 2 10 4 x 20000.
N 430
16. (2)
10 1 1 1
22. (3)
2 a b 9
6
S 1 1
a 36
20th 30th
17. (2) Left k m
Right
1 1 1
m k b 9 36
19 35th
34 1 3 1
So total man = 19 + 1 + 34 = 54. 36 36 12
18. (4) Clearly from option if it is gain 6 hrs then it covers 30
b can complete in 12 days.
min. So at 6:00 o’clock incorrect watch shows 6:30 pm.
23. (1) Let the number be x.
19. (2) Hindi books = 14% of 10000 = 1400
1400 x x 2x
6 84. 5
6% are Bad = 6% of 1400 3 2 3
100
20. (4) Total no. of books = x 5x 2 x 15
2 3
23 x 6
23% of x (no. of Kannada books)
100 5x 4 x 30 x 30.
Bad condition 2% of 23x = 92
2 2
So, of x 306 12.
23x 5 5
2 92
104
EBD_7332
74
A- NTSE Stage 1 Question Bank

24. (4) CP of 72 oranges = `. 600 33. (2) The 7 multiples that leave a remainder of 1 when divided
SP of 72 oranges = 11 × 50 = `. 550 by 3 and 5 are to be counted.
SP of remaining (61) oranges = `. 236 In other words, the 7 multiples that leave a remainder
Total SP of oranges = 236 + 550 = `. 786 of 1 when divided by 15 are to be counted.
Profit = SP – CP = 786 – 600 = `.186. Between 1 and 100 :
25. (4) As 50% is 450 gm so 40% will be 360 gm + 150 = 510. 7 multiples are 7, 14, 21, 28, 35, 42, 49, 56, 63, 70, 77, 84,
26. (2) Increase by 4% as 91 and 98.
Let L = 10, B = 10 Area = 100
15 multiples between 1 and 100 are 15, 30, 45, 60, 75, 90.
4% (Multiples of 15) + 1 are 16, 31, 46, 61, 76, 91
New L = 13, B = 8 Area = 104 91 is the only multiple of 7 which is 1 more than a
27. (1) Ravi’s age = x multiple of 3 and 5.
Raju’s age = y 34. (2) The greatest 3-digit number having 8 in tens place and
x 4 divisible by 4 is 988.
y 3 The smallest 3-digit number having 8 in tens place and
divisible by 4 is 180.
x 6 26 x 20
So, the difference is 808.
20 4 20 3 35. (2) Let us say that the total number of books are N.
y 15 years
y 3 4 Number of English books = 0.5N
Number of Hindi books

28. (4) 20 m 7
( N 0.5 N ) 0.7 0.5 N 0.35 N
10
25 m The shaded portion, that represents the number of
30 m
books in Hindi, should be more than quarter and less
25 m than half.

29. (3) 8 + (8 – 2) = 70
2

92 + (9 – 3) = 87 36. (3) D 6 E 6 C
102 + (10 – 4) = 106 x
10
72 + (7 – 5) = 51
30. (4) When a + b = 2020 O
2020 is an even number. So, if a is even, b is also even 10 x
(to make sum even). Similarly, if a is odd, b is also odd
A 6 F 6 B
to make the sum even.
When a and b are even, (–1)a + (–1)b = 2
When a and b are odd, (–1)a + (–1)b = 2
x = OE 10 2 62 8 cm = OF
31. (3) Total successful surgeries = 5 + 4 + 5 + 6 + 4 + 3 + 2 = 29.
EF = EO + OF = 8 + 8 = 16 cm.
Total unsuccessful surgeries = 3 + 4 + 2 + 3 + 2 + 3 + 4
= 21. V1 a3 1
Total number of surgeries = 21 + 29 = 50. 37. (3) 3
V2 A 27
29
Percentage of successful surgeries 100 58%. a 1
50
A 3
32. (4) Total
Total Total Percentage of
Surgeries 2
Month Successful Unsuccessful
every
Successful S1 6a 2 1 1
Surgeries Surgiers
month
Surgeries
2
.
S2 6A 3 9
Jan 5 3 8 5 / 8 × 100 = 62.5%
Feb 4 4 8 4 / 8 × 100 = 50% 38. (3) sin cos tan 1
March 5 2 7 5 / 7 × 100 = 71.4%
45
April 6 3 9 6 / 9 × 100 = 66.6%
May 4 2 6 4 / 6 × 100 = 66.6% then, sin 45 cosec45 2 2 2 2
Jun e 3 3 6 3 / 6 × 100 = 50%
July 2 4 6 2 / 6 × 100 = 33.3% 39. (1) 7 P 24 M 8 Q 6 M 2 L 3
After putting signs we get
The percentage of successful surgery is highest in the
7 + 24 ÷ 8 – 6 ÷ 2 × 3
month of March.
= 7 + 3 – 3 × 3 = 7 + 3 – 9 = 10 – 9 = 1.
Solved Questions A-75
40. (2) Persons known the Bengali and English language but 49. (1) a + b = 51
not known the Hindi = 62.
41. (3) 53 persons knows all the three languages.
42. (1) Persons who do not know Hindi language 26 + 25 = 51
= 120 + 83 + 62 = 265.
(Alphabets from Right to left)

B x + y = 5
43. (2)
9 km
15 km 8 km
O C 3 + 2 = 5
6 km
p q 1 11 10 = 1.
A
Starting Point
S
50. (4)
AC = AO 2 + OC 2 = 62 + 82
20
= 36 + 64 = 100 = 10. 5
10 km North-East.
44. (2) If 1st Jan 2008 is Tuesday so 1st Jan 2009 is Thursday. 10
45. (4) Total students = 35
Ibrahim Rank = 8th S
So students after him in class = 27
150000
So his rank from last = 28th. 51. (2) = 9375
16
46. (b) Let ‘x’ be the common root.
52. (3) 10, 20, 30 4050 60 7080 90 100
x 2 + ax + b = 0 ...(1)

x 2 + bx + a = 0 ...(2) 110, 120 , 130, 140, 150 , 160, 170, 180 , 190, 200

(a b) x (b a) = 0 14

a b 53. (2) Sum of all + ve factor of 256


x= =1 x =1 256 1, 2, 4, 8, 16, 32, 64, 128, 256
a b
Number of factor = 9
Then, {1 + a + b = 0} Sum of all +ve factors = 29 – 1 = 512 – 1 = 511
From equation (1), a + b = 1 x x+1 1
54. (4) + – 2
47. (1) Let, P = x, A = 2x x+1 x x +x

x R 10 x 2 + (x + 1)2 1

2x R = 20% x +x2 2
x +x
100
2 2
Then, ATQ x + x + 2x + 1 – 1
x2 + x
x 20 T '
3x =
100 2x 2 + 2 x (2 x 2 + x)
= =2
x2 + x x2 + x
T'
3 T ' = 15 years 55. (1) 5 + 6 + 7 + 8 + ... + 19 = 1 + 2 + 3 + ... 19 – 1 + 2 + 3 + 4
5
And from formula, Sum of n consticutive positive
1 2 n(n + 1)
48. (3) V1 r h ...(1) integer =
3 2
Here n = 19.
1 2 19 20
Also, V2 r h(3h) ...(2) – (1 + 2 + 3 + 4)
3 2
Volume is increased by 300%. 190 – 10 = 180
EBD_7332
76
A- NTSE Stage 1 Question Bank

1 2 3 Sol. (65–68):Let the present age of Ram, and Ram’s son is p and
56. (1) Given ratio: : : q, then the present age of Ram’s Father is 100 – p
2 3 4
RF R RS
1 2 3 100 – p p q
12 : 12 : 12 6 :8:9
2 3 4 Now, Let after years Ram’s age will be equal to present age
Let common factor is x then of Ram’s Father and son
9x – 6x = 3x = 27 RF R RS
x=9 (100 – p + a) (p + a) (q + a)
6x 54 Given
8x 72 p + a = 100 – p ...(1)
9x 81 p + a = 5q ...(2)
57. (2) 325 + 326 + 327 + 328 q+a–p=8 ...(3)
325 (1 + 31 + 32 + 33) Now solve eq. (1), (2) and (3) for a, p and q
325 (1 + 3 + 9 + 27) a = 30 years
325 (40) p = 35 years
Hence, the value is divided by 40. q = 13 years
58. (4) Score in mid-term = 75 65. (2) Age of Ram’s Son is ‘q’ = 13 years
Score in find exam = 90 66. (1) Age of Ram 5 years ago = 35 – 5 = 30 years
Let weight of mid-term and find exam are W and 2W 67. (3) Present age of Ram’s father = 100 – 35 = 65 years
respectively After 10 years Ram’s father will be = 65 + 10 = 75 years
75 W 90 2W 255 68. (2) According to the question,
Rohan’s final score = 85
W 2W 3 “_” means division
59. (1) Total number of ways to arrange n obejects in row = n! “÷” means multiply
Here n = 3. “+” means subtract
Number of arrangement = 3! = 3 × 2 × 1 = 6 “×” means sum
60. (2) Given expression : 4 + 6 × 9 6 – 2 × 5 (i) 20 × 8 ÷ 7 × 6 + 4
8 25
After substitution 4 6+9–6×2+5= (ii) 20 ÷ 5 × 4 – 6 + 5
3
61. (2) From option (2): =4×4–6+5
59 + 21 = 8 × 10 = 80 = 16 – 6 + 5
62. (4) From option (4): = 21 – 6 = 15
6 × 12 – 48 = 24 (iii) 20 × 5 – 6 ÷ 7 + 4 = 28
72 – 48 = 24 6
24 = 24 100 – + 4 = 28
7
63. (2) (i) A = 3B 700 – 6 + 28 = 196
D 722 196
(ii) C=
2 (iv) 20 ÷ 4 – 8 × 10 + 6 = 36
(iii) B>C 5 – 80 + 6 = 36
Let B = C + x –69 36
(x > 0) 69. (3) If selling price is same and profit % of Ist article is
A = 3 (c + x) equal to the loss % of 2nd article say (L) then always
will be overall loss and loss % equal to
D
A=3 x L2 102
2 % %
100 100
3D 100
A= 3x Hence, = 1%
2 100
A>D 70. (1) Clearly, A’s new position, is 15th from the left end. But
64. (4) The average age of six member = 22 × 6 = 132 this is the same as B’s earlier position which is 9th
132 7 7 5 from the right end.
Now, Then the total no. of boys in a row = 15 + 9 – 1 = 23
5
71. (3) Here we have to convert the power of digit as,
90 63 × 5 × (2 × 5)98
= = 18 Year
5 We get,1080 × 1098 have 102 digits
Solved Questions 77
A-

72. (2) If, A = 7 and B = 5 By trial and error method 77. (4) Let the total number of students in the class = x =
78 35 = 2730 100,
73. (2) Let the total population of city E = x According to the question,
8 Girls – 40, Boys – 60
Now, x = 3360
100 3
Now, 75% of 40 = 40 = 30
100 4
x = 3360 But overall passed percent is 80% = 80
8
x = 42000 Passed girls are = 30
14% of E = 42,000 Passed boys are = 50
100 50 5
E = 4200 So, required ratio
14 60 6
E = 300000 78. (2) According to the question,
So, given D = 16% C.P. = 5 × 8 + x × 6
16 Now, C.P. = 40 + 6x
Hence, population of city D = 300000 = 48,000 S.P. = 7(5 + x)
100
74. (3) According to given information, Profit = S.P. – C.P. = (35 + 7x) – (40 + 6x) = 10
% of illiterate of city B = 5%, x = 15
% of literate of city = 95% 79. (4) At present 10 years ago
95 Father’s age: 4x 4x – 10
Required ratio = = 19 : 1 Daughter’s age: x x – 10
5
75. (2) Let total population of all the city = x According to question,
then, total population of illiterates. (4x – 10) + (x – 10) = 40
5x – 20 = 40
24 1 26 5 20 4 16 10 14 8
= x 5x = 60
100 100 100 100 100 100 100 100 100 100
x = 12
x present age of the daughter = 12.
= 24 130 80 160 112
10000
506 x 80. (3) 4 km
=
10000
5 km
Percentage of illiterate population. 11 km

506 x
= 100 5.06%
10000 x 12 km
76. (1) According to the question,
P 10

He is facing in south direction.


10
6 6 81. (4)

4 km
Q 8 10

N 5 km
5 km

B
W E 4 km 8 km C

6 km
S
A
2 2
PQ = (8) (6)
AC2 = AB2 + BC2
= 64 36 100 10m
AC2 = 62 + 82 = 100 10 km
EBD_7332
78
A- NTSE Stage 1 Question Bank

82. (4) 2x + y = 35 ...(1) 89. (2) Let mother’s present age be x years
3x + 4y = 65 ...(2) Son’s age = (x – 30) years
on solving eqn. (1) and (2) we get According to question
x 15 x + x – 30 = 40 2x = 40 + 30
y = 5 and x = 15 then = =3
y 5 2x = 70
83. (3) According to the question. x = 35 years
LCM = 12 HCF Son’s age after 10 years = 35 – 30 + 10
LCM + HCF = 403 = 15 years
Now, 90. (2) Volume of Spherical shell = Volume of cone
12 HCF + HCF = 403
We have R = 4 cm
403
HCF = 31 r = 2 cm
13
4 1
LCM = 12 × 31 ( R )3 ( r ) 3 = r2 h
3 3
LCM × HCF = 93 × b
Hence, b = 124 4 1
84. (1) According to the question. (4)3 (2)3 = 42 h
3 3
Let the two integers be a and b.
a–b=3 56
h= = 14 cm
a3 – b3 = 117 4
(a – b)3 = a3 – b3 – 3ab (a – b) 91. (3) According to question,
ab = 10 AOB = 130° (Given)
a+b= (a b) 2 4ab PAO = PBO = 90° (since PA and PB are tangents to
a+b=7 the circle). We know sum of the angle of the
85. (2) According to the question, quadrilateral = 360°
Total amount spent = 46000 APB = 360° – AOB – PAO – PBO
Food Share = 23% = 360° – 130° – 90° – 90° = 50°
Amount spent on food is = 46000 × 23 / 100 = 10580 Hence APB = 50°
86. (1) According to the question, 92. (1) We have,
Total amount spent = 46000 1
Clothing = 10% and Housing = 15% cos4 – sin4 =
3
Amount spent on Clothing + Housing both = 25% 1
Amount on both = 46000 × 25/100 = 11500 (cos2 – sin2 )(cos2 + sin2 =
3
87. (1) According to the question,
1 4
(2 cos2 – 1) = 2 cos2 =
x 2 y 5 3 3
x + y + z = 98
y 3 z 8 2
cos2 =
2 8 2 5 3
Now, y y y 98 x y, z y
3 5 3 8 1
sin2 = 1 – cos2 = ( cos2 + sin2 = 1)
10 y 15 y 24 y 3
98
15 sin 2 1
tan2 = 2
98 15 cos 2
y= Hence, y = 30
49 93. (3) 3 km
88. (4) According to question, 4 km 4 km
1 3 km
2x + 1 = 0 x= 8 km
2
5
1 1 33
x5 – 1 = 1 1 He is 12 km away from the starting point.
2 32 32
94. (1) We have P –, Q = +, R = , S = ×
33 then, 18 – 6 + 4 × 6 2 = 24
Absolute value =
32
Solved Questions 79
A-

95. (1) According to the question, 1


5 a
Suppose student got x sum right and 3x sum wrong 2
1
x + 3x = 120 13
4x = 120 1
5 a
x = 30 15
Wrong sums = 90 13
13
Hence the no. of problem incorrectly done is 90. 5 a
15
Left end 12
96. (1) 13 62
18 Right end a 5
15 15
Total strength = 12 + 18 – 1 = 29 1 1 1
The total number of students is a prime no. 102. (4) P Q 10
..... (i)

6,12,18, 24, 30 1 1 1
Q R 15
..... (ii)
36, 42, 48,54, 60
97. (3) Divisible by ‘6’
66, 72, 78,84,90 1 1 1
..... (iii)
96 R P 20
Adding eq. (i), (ii) and (iii),
Contains ‘6’ as digit
2 2 2 1 1 1
6, 36, 60, 66, 96
P Q R 10 15 20
Hence 5 numbers exactly divisible by 6 and also have
6 as digit. 1 1 1 6 4 3
2
a b c P Q R 60
98. (2) k say
3 5 7 1 1 1 13 13
a = 3k, b = 5k, c = 7k P Q R 60 2 120
a b c 3k 5k 7 k 15k 1 13 1
3
b 5k 5k (from (i))
R 120 10
99. (4) 17 x 11 11 6 1 13 12 1
Squaring both sides, R 120 120
Hence, R alone will complete the same work in 120 days.
17 x 11 11 6 2 66
103. (4) Checking option
17 x 11 17 2 66 (1) 36 × 4 ÷ 8 – 7 + 4 = 10
x 11 2 66 15 10
2 66 (2) 16 × 12 + 49 ÷ 7 – 7 = 200
x 2 6 192 200
11
2
(3) 32 ÷ 8 × 9 = 160 – 12 × 12
x2 2 6 24 36 16
(4) 8 × 8 + 8 ÷ 8 – 8 = 57
100. (1) Sum of roots 7 3 7 3 14
57 = 57
Product of roots 7 3 7 3 46 7n 3
14 7 n 4
104. (3)
Equation is 7n 3
(x2 – Sx + P) = 0 7n 3
14 7 7 n 3
x2 – 14x + 46 = 0
7n 3
1 n 3
101. (2) 5 a 7 1 14 7
1
1
1 7n 3
6 = 1 + 98 = 99
2
1 1 2 3 2 3
5 a 105. (4) a 2 3
1 2 3 2 3 4 3
1
13
1 2 3 2 3
2 b 2 3
2 3 2 3 4 3
EBD_7332
80
A- NTSE Stage 1 Question Bank

Substituting the values of a and b in the equation, 113. (1) No. of visitors from Maharashtra below 20 years
we get
5 60
100000 = 3000
7 a 2 11ba 7b 2 100 100
2 2 114. (3) No. of visitors below 20 years of age who were neither
7 2 3 11 2 3 2 3 7 2 3
from Karnataka nor Maharashtra or West Bengal
7 7 4 3 11 4 3 7 7 4 3 15 60
100000 = 9000
49 28 3 11 49 28 3 100 100
11 56 3 115. (2) Required ratio is :
106. (1) a + b = sum of roots = 2 and 60% of 20% of 100000 : 20% of 5% of 100000
ab = product of roots = (–1) 60 20 20 5
= 100000 : 100000
a2b + ab2 = ab (a + b) 100 100 100 100
= (–1) (2) = (–2) = 12000 : 1000 = 12 : 1
107. (4) 12 litres is taken out & replaced by water 116. (3) Visitors from West Bengal in the age group of 20 – 40
new ratio is 4 : 1 years = 20% of 20% of 100000
4 1 20 20
Now milk is part and water is part 100000
5 5 100 100
= 4000
4 48
Thus in 12 litres quantity of milk = 12 Ltrs. Visitors from Maharashtra above 40 years
5 5 = 20% of 5% of 100000
48 20 5
So remaining milk 48 38.4 Ltrs. 100000 1000
5 100 100
20 16 30 16 Difference = 4000 – 1000
108. (3) = 3000
Left AB Right Left BA Right 117. (2) 7 = 4 < 8 = 3 > 39 @ 3
From the diagram total no. of boys = 30 + 16 – 1 = 45 = 7 × 4 – 8 × 3 + 39 ÷ 3
109. (3) Number of apples in the basket initially = 12 × 12 = 144 = 28 – 24 + 13
After adding two dozen = 41 – 24 = 17
Number of apples = 144 + 24 = 168 118. (2) 24 – 36 ÷ 12 + 8 × 4 = 24 – 3 + 32
10 apples got rotten, = 56 – 3
Remaining apples = 168 – 10 = 158 = 53
No. of basket in which apples are to be distributed = 2 119. (4) After interchanging the signs × and + we have
(20 × 3) + 6 × 8 – 32 = 76
158
No. of apples in each basket 79 76 = 76.
2
120. (2) 90m
110. (2) The relative speed of minute hand and hour hand is 5.5°
Angle between hour & minute hand at 8 O’ clock North
= 240° F
For the hands to be together, they must coincide, the
angle of 240° must be covered by them by the relative
B 100m
240 7 A C East
speed in time per minute 43 minutes past 8.
5.5 11 20 m
111. (1) Let ‘x’ minutes be the time taken by pipe C
E 30 m D
For pipes A, B and C to fill the tank together
South
We have :
From figure,
10 10 10
1 2 2 2
12 15 x AF FB AB
x = 20 minutes 2 2
Solutions (112-116) : FE BE AC BC
112. (3) No. of visitors from Karnataka in the age group of 2 2
20 – 40 years 80 60
= 60% of 20% of total population = 6400 + 3600
60 20 = 10000
100000 = 12000
100 100 AF = 100 m
Solved Questions 81
A-

121. (1) 175 – 25 ÷ 5 + 20 × 3 + 10 127. (4) Let the number of wrong sums and right sums be 2x
and x respectively.
= 175 ÷ 25 + 5 × 20 – 3 × 10
Total no. of sums = 60
= 7 + 5 × 20 – 3 × 10
x + 2x = 60
= 7 + 100 – 30 3x = 60 x = 20
= 77 He solves 20 sums correctly.
122. (3) Let the no. of members excluding father be x 128. (1) Let the no. of children = x
Total members = (x + 1) 1 x
then, x x 16
1 8 2
Father’s share
4 x
1 1 3 x2 16 8
1 2
Share of each member
4 x 4x
x2
64
3 1 x
3
4x 4 x = 64
x =9 1
Number of notebooks x x
Hence, total members = x + 1 = 10 8
123. (2) Let the present ages of man and his son be x yrs and
y yrs respectively. x2 64 64
512
5 yrs ago, 8 8
(x – 5) = 7 (y – 5) 1 1 1 1
129. (2) ......
x – 5 = 7y – 35 5 6 6 7 7 8 24 25
x – 7y = – 30 ......(i) 1 1 1 1 1 1 1 1
.....
5 yrs hence 5 6 6 7 7 8 24 25
(x + 5) = 3 (y + 5) 1 1 5 1 4
0.16
x – 3y = 15 – 5 5 25 25 25
x – 3y = 10 ......(ii) 2
5 4 5 4 5 4
On solving eq. (i) and (ii), we get y = 10 and x = 40 130. (1) x
124. (1) Let four prime numbers in ascending order are x, y, z 5 4 5 4 5 4
and t respectively, 2
x 5 4 5 4 2 20 9 2 20
xyz 1001
yzt 2431 2
5 4 5 4 5 4
7 also, y
x 1001 7 5 4 5 4 5 4
t 2431 17
17 2
y 5 4 5 4 2 20 9 2 20
x 7
t 17 2 2
Now, x 2 y2 9 2 20 9 2 20
last number = 17
125. (2) Required largest number = HCF of (237 – 132) 81 80 36 20 81 80 36 20
(132 – 62) and (237 – 62)
= 322
= HCF of (105, 70, 175) = 35
126. (3) 48 sec, 72 sec, 108 sec 131. (4) 13 3.605
48 = 2 × 2 × 2 × 2 × 3
1.3 1300 0.013
72 = 2 × 2 × 2 × 3 × 3
108 = 2 × 2 × 3 × 3 × 3 130 130
LCM of 48, 72, 108 = 2 × 2 × 2 × 2 × 3 × 3 × 3 13 100
100 10000
= 432
432 11.40 11.40
mins 7 mins12 sec 3.605 10
432 secs 10 100
60
Hence, they all will again change simultaneously at = 1.14 + 36.05 + 0.114
7 : 07 : 12 hrs. = 37.304
EBD_7332
82
A- NTSE Stage 1 Question Bank

137. (3) 7 + 6 + 5 + 4 + 3 + 2 + 1 = 28
1 1 1 1 1 138. (4) Total hours = 89 hours
132. (2)
9 8 8 7 7 6 6 5 5 4 44 356
Faulty time 23 hours & 44 min = 23 i.e, hr
60 15
1 9 8 1 8 7 356
hr of incorrect watch = 24 hr of correct watch
9 8 9 8 8 7 8 7 15
24 15
89 hours of incorrect watch = 89 90 hrs
1 7 6 1 6 5 356
7 6 7 6 6 5 6 5 Actually watch will be 1 hr faster than faulty watch.
i.e., 11 pm
139. (4)
1 5 4 H F
5 4 5 4

9 8 8 7 7 6
J I G

6 5 5 4 means female member


means male member
9 4 3 2 5 means couple
133. (2) Volume of wood = volume of pencil-volume of lead — means brother/sister
means son/daughter
3
2
1 140. (2)
28 28 Standing Positions:
10 10
Facing North
9 1
28 Teacher Student Doctor Clerk Businessman
100 100
141. (2) 52 < 4 5 > 8 2
8 22 8 52 – 4 × 5 + 8 ÷ 2
28 28 52 – 4 × 5 + 4
100 7 100
56 – 20 = 36
= 7.04 cm3 142. (1) If the order is
×+ =
2 2 2
0.03 0.21 0.065 Then 6 × 3 + 4 = 22 is true
134. (3) ? So, option (1) is correct answer.
2 2 2
0.003 0.021 0.0065 143. (2) If the order is × – =
Then 7 × 3 – 8 = 13
0.0009 0.0441 0.004225 So, option (2) is correct answer.
144. (3) According to question,
0.000009 0.000441 0.00004225
15th September 2000 Friday
15th September 2001 Saturday [because 2001 is a
0.049225 non-leap year i.e. +1 odd day]
0.00049225 145. (2) According to question,
7 11
100 10
135. (3) Volume of cone : Volume of hemisphere : Volume of V k
cylinder L R

1 2 2 3 13
r h: r : r2 h Total = 13 + 11 – 1 = 23
3 3 Kamal new position = (23 – 7) + 1 = 17
1 2 2 146. (4) Aeroplanes fly in the sky, sky is called sea. so, sea
r r : r3 : r 2 r is answer.
3 3
147. (2) Here, S = +, D = –, Q = ÷ and p = x
1 2 35 ÷ 7 × 5 + 5 – 6 = 24
: :1 148. (2) According to question,
3 3 (8 × 5) + (1 × 4) = 44
=1:2:3 (6 × 8) + (2 × 8) = 64
136. (4) 15 – 2 ÷ 900 + 90 × 100 = ? Similarly,
15 × 2 + 900 ÷ 90 – 100 = 15 × 2 +10 – 100 (7 × 9) + (4 × 8) = 83
30 + 10 – 100 = 40 – 100 = – 60
Solved Questions 83
A-

(E) Now,
149. (2) N (S-E)
(N-E) x y y z z x
N-w N-E
+ 11
xyz
(N)W E(S) Put (ii) in the above equation, we get
( z )( x)( y)
S-W = 11
S-E xyz
(N-W)
S (S-W)
(W) xyz
= 11 –1 + 11 10
So, answer is East-South. xyz
150. (4) According to question, 158. (1) Given : + = 6, = p and 2 2
10
Let A scored = x, then A + E + D + B + C = (36 × 5) = 180 Now, ( + ) – 2
2
= 2+ 2
x + (x – 8) + (x – 3) + (2x – 11) + (118 – 2x) = 180 (6)2 – 2 × p = 10
3x = 84 36 – 2p = 10
x = 28
p = 13
E = x – 8 = 28 – 8 = 20
151. (2) According to questions, 90 10 90 10 80
LCM of 6, 5, 7, 10 and 12 is 420 so, that bells will toll 159. (2) Here, 10
8 8 8 8
together after every 420 seconds ie. 7 minutes
Now 7 × 8 = 56 & 7 × 9 = 63 ( x 1) ( x 1) (2 x 2 2)
160. (2) Given :
So, in 1 hour (60 min) the bells will toll together 8 times, ( x 1) ( x 1) ( x 2 1)
excluding the one at start Now,
152. (1) According to questions,
( x 1)2 ( x 2 1) ( x 1) 2 (x 2 1) (x 2 1)(2 x 2 2)
C 15 yrs
A 12 yrs (C – 3) ( x 2 1)( x 2 1)
D 11 yrs (A – 1) ( x 2 2 x 1)( x 2 1) (x 2 2 x 1)
B 10 years (D – 1) Age of B = 10 years.
(x 2 1) (x 2 1)(2 x 2 2)
153. (2) According to question,
( x 4 1)
B x4 x 2 2 x3 2 x x 2 1 x 4 x2
2 x3 2 x x 2 1 2 x 4 2 x2 2 x2 2
6 km (x 4 1)
2(2 x 2 )(2) 8 x 2
10 km C 3 km ( x 4 1) x4 1
D
161. (1) Let the speed of Parth in still water be x km/hr and
speed of stream be y km/hr.
4 km Then,
According to the Question,
16 8
A =6 ...(i)
x y x y
24 6
AD = 42 + 32 = 25 = 5km(North east) and =6 ...(ii)
x y x y
154. (1) According to question,
From (i) and (ii), we get
x + y = 8 and x – y = 2
On solving, we get
MT KA S N x = 5 km/hr and y = 3 km/hr.
T is sitting between M and K. Hence, the speed of Parth in still water is 5 km/hr.
155. (3) Logic Pattern N R M L J O Q P K 75 5 32
J is sitting at the centre. 162. (4) Given : log 2 log log
16 9 243
156. (2) 2
157. (4) Given : x + y + z = 0 ...(i) 75 5 32
log log log
From (i) 16 9 243
(x + y) = – z 75 81 32
(y + z) = – x and log
16 25 243
(x + z) = – y ...(ii) log 2
EBD_7332
84
A- NTSE Stage 1 Question Bank

163. (3) Given : 315 + 316 + 317 173. (4) Let the ten's digit be x and unit's digit be y.
315 (1 + 3 + 32) Then, number = 10x + y
315 (1 + 3 + 9) Number obtained by interchanging the digits = 10y + x
315 (13) (10x + y) + (10y + x) = 11(x + y)
Hence, it will be completely divisible by 13. which is divisible by 11.
164. (4) From the options : 174. (3) Birth date of Kiran = 12th September.
(1) –7 7 × 7 + 7 = 0
Soham is 12 days younger thus, his birth date is = 1st
(2) (7 + 7 × 7) 7 – 7 = 1
September.
(3) 7 – 7 × 7 7 + 7 = 7
(4) 7 – (7 7 × 7 + 7 ) = – 7 Hindi day = 14 September : (considered as Friday).
165. (1) Given : 6 R 8 S 1 R 3 Q 5 P 7 Q 4 P 2 Therefore, 1st September of same month is Saturday.
Now putting code : 175. (3) The direction diagram of a farmer is as follows:
= 6×8 1×3– 5+7–4+2 A 6m B N
= 6 × 8 × 3 – 5 + 7 – 4 + 2 = 144 – 5 + 7 – 4 + 2 = 144
12m
166. (1) By observation, 20 m 6m W E
D
= 12 : 00 – 9 : 30 = 2 : 30 8 C
167. (1) By option (1), O 10
S
6 20 × 12 + 7 – 1 = 70 Farmer's
House
By putting signs,
6 – 20 + 12 × 7 1 = 70 Here, In ODC,
6 – 20 + 12 × 7 = 70 D = 90°
6 – 20 + 84 = 70
70 = 70 OC 82 62 100 10 .
168. (2) So, he is 10 m from his Original position.
176. (4) 177. (3) 178. (2) 179. (4) 180. (4)
Fri Sat Sun Mon Tue Wed Thu Fri 181. (1) 182. (3) 183. (2) 184. (1) 185. (2)
Third 186. (2) 187. (3) 188. (4) 189. (4) 190. (4)
Today Day 191. (4) 192. (1) 193. (3) 194. (4) 195. (2)
So, if day before yesterday was friday, then friday will 196. (1) p × q × r means q is greater than or equal to r .
be the third day after the day after tomorrow. 197. (4) p + q × r means p is not equal to q which is not possible.
169. (4) As, (10)3 = 1000 198. (3) p q O r means q is greater than r.
(100)2 ÷ 10 = 10000 ÷ 10 = 1000 199. (4) Thursday because total number odd days will be three.
and 23 × 53 = (10)3 = 1000 200. (2)
But, 2000 – 2 = 1998 11. a.m.
st nd rd th th th
So, except (4), all result in 1000. 1 day 2 day 3 day 4 day 5 day 6 day
170. (1) The relation is as follows : 24 hrs 24 hrs 24 hrs 24 hrs 24 hrs 1. p.m.
Father wife Mother 122 hrs. of given clock is equal to 120 hrs of actual time.
Mother's only Uncle Total hrs (according to given clock = 24 × 5 + 2 = 122 hrs i.e.
brother actual time will be 2hrs back = 11 a.m.
Uncle's son Cousin 201. (1) OGEVNREMTN
So, Mahipal is cousin to sailesh 202. (1) South
171. (4) Given : 15 = 5 + 2 × 1 ÷ 7
By putting signs in option (4)
15 ÷ 5 × 2 + 1 = 7 203. (3)
Fr L
3×2+1=7
6 + 1= 7
7= 7 A B A B A B
172. (3) According to the question, if the first person face is 100 = 75.8 + 49.4 – A B
towards north then the right hand of second person
A B = 125.2 – 100 = 25.2
will be in west direction.
Students who took both 25.2% of 250 = 63.0 (Approx)
N Varun Raman
Ist Person facing North
204. (2) Left Right
W E 9th 14th
2nd Person facing South
Right
As Raman is 14th from right. So Varun will be 17th from
S right. So there are 8 people (25 – 17) to the left of Varun.
He is the nineth.
Solved Questions A- 85
205-206. Clearly Arrangement of boys is as shown:-
B A (140 m)
P
213. (3)
A 60 m B
20 m 20 m
30 m
40 m
Facing
N A K towards Distance travelled by A on road = 60 + 20 + 40 + 20 =
25 m 40 m north 140 m
Disance travelled by B on road = 60 + 40 = 100 m
60 m Required difference = 140 – 100 = 40 m
214. (4) B C N
NW NE
D
205. (4) Clearly, Atul is to the left of Kunal and Prashant is to W E
the north - east of Atul.
206. (3) Required distance = N A + A K + K D + D P SW SE
= (25 + 40 + 60 + 90)m = 215 m S
A
207. (2) Anuj reached the place at 08 : 15 a.m.
Clearly, the man who was 40 minutes late would reach A is to south west of C.
th th
the place at 8 : 45 a.m. So, the scheduled 8 : 05 time of 215. (3) Amrita is 10 from front and 41 from the behind.
meeting. Therefore, 15 persons are in between Mukul and Amrita
and Mamta will occupy 33rd position from behind.
36 4 8 4 Hence, Her front position is 18th.
208. (1)
4 8 2 16 1 216. (3) Let the no. of boys = x
Replacing sign according to question. We get, Let the no. of girls = 2x
36 4 8 432 8 4 4 4 According to question
0 x + 2x = 60
4 8 2 16 1 32 32 1 1
x = 20
209 – 211. We have, A + B > C + D ...(1)
Boys = 20
A+ C=B+D ...(2)
Girls = 40
1 If Kamal is at Seventeenth position and 9 girls are a
A = (B+D) ...(3)
2 head of him then out of 16 people standing ahead of
1 him, seven are boys. Therefore, there are 12 boys are
Putting the value of A in Eq. (2), we get C = (B+D)
2 after him in a rank.
i.e., A = C
Since, A + B > C + D and A = C, we get 217-221. P
B> D K R
Thus from eq. (iii), we get B > A and B > C.
209. (2) Clearly B has the highest income. M T
210. (1) Clearly B earns more than A and Hence option (1) is
false.
211. (4) 2A = B + D ...(1) D W
A=B–D ...(2) V
Adding eq. (1) and (2) 217. (2) R is second to the left of K.
3A = 2B 218. (1) D is to immediate left of V.
A = 80,000 (given) 219. (4) R is third to the right of V.
3 220. (1) R’s is third to the right of V.
B= 80000
2 221. (4) All groups have one person sitting between them while
B = ` 1,20,000 RD have 3 persons sitting between them.
212. (2) Short cut method- Person N 11
Take clockwise as ‘+’ 222. (3) Angle between hands of clock = 30H M
W E 2
and anticlockwise as ‘–’ where H Hour hand, M Minute hand
+ 90 ° – 135° = – 45° S 11
As person is facing North west. Now, rotate that person 30 6 30 15
45° anticlockwise. Now, he is facing west direction. 2
EBD_7332
86
A- NTSE Stage 1 Question Bank

Y (Cook) OP2 + OQ2 = QP2


223. (3)
QP = (300)2 (400) 2
P Z (Barber)
(Washerman) = 90000 160000 = 250000
QP = 500
X (Tailor) QP 500
QR = 250
224. (3) A 2 2
E 231. (4) Candidates failed in two subjects only = candidates
passed in one subject only = 62 + 48 + 52 = 162
C D 232. (2) Candidates passed in atleast one subject = (candidates
passed in only 1 subject) + (candidates passed in only
2 subjects) + (candidates passed in all the subjects)
F
B = (candidates failed in only 2 subjects) + (candidates
failed in only 1 subject) + (candidates passed in all the
Hence, D is between A and F.
subjects)
225. (4) = 162 + 61 + 167 = 390.
B M V K R Q
233. (2) Let the speed of the second train be x km/hr.
226. (2) 81 × 3 = 243 Relative speed = (x + 50) km/hr
20 × 4 = 80
Hence, 21 × 7 = 147 5 250 + 5x
= ( x 50) m/sec = m/sec
18 18
227. (2) Time 4.30 W
Distance covered = 100 + 120 = 220 m
220
=6
S N 250 5 x
18
(North-East) 220 18
E 250 + 5x =
228. (3) Total students = 39 6
250 + 5x = 660
6 17
x = 82 km/hr
Paresh Sumit 234. (1) Let the number to be multiplied = x
Sumit is 17th from last
Paresh 7 ranks a head of Sumit According to question
It means there are 6 students between them. 52x = 25x + 324
Total from last to Paresh 17 + 6 = 23 52x – 25x = 324 x = 12
Now 235. (2) No. of students in class
39 = student rank from starting + student rank from last –
23 1
Total last 16 16th Rank 45 = 18 + last rank – 1
th
Last rank = 45 – 18 + 1 = 28
229. (1) 236. (4) No. of boys = 18 + 18 – 1 = 35
B D E F C A 237. (1) No. of passed students in class = Rakhee
The right of B is D. Rank from first + Rakhee rank from last – 1
300 km P 21 + 34 – 1 = 54
230. (2)
O No. of students in class = No. of passed students +
failed students + absent students
54 + 4 + 9 67
400 km No. of girls in class = 67 – 37 = 30
R

Q
Solved Questions A-87

2 km
4. (4) CA / CCAA / CCCAAA / CCCCAAAA
1 km
5. (2) 25 + 17 = 42 = 6 × 7
238. (2) Shrikant starting 4 km 2 km 38 + 18 = 56 = 8 × 7
position
89 + 16 = 105 = 15 × 7
6. (4) 4 × 2 –1 = 7, 7 × 2 + 1 = 15, 15 × 2 – 1 = 29, 29 × 2 + 1 = 59
9 km 59 × 2 – 1 = 117, 117 × 2 + 1 = 235, 235 × 2 – 1 = 469.
8 km 7. (4) (93 – 27) = 66 – 63 = 3
(79 – 38) = 41 – 37 = 4
(67 – 16) = 51 – 42 = 9
8. (4) UNCLE QQYOA
Shrikant final
position U N C L E
–4 +3 –4 +3 –4

Distance from original position = 2 2 Q Q Y O A


(6) (8)
C O L U M N
= 36 64 100 10 km . –4 +3 –4 +3 –4 +3
239. (2) Sohan was born on 25th Feb. 2012. That means Hasan Y R H X I Q
was born on 29th Feb 2012 ( 2012 is a leap year).
If 26th January in that year falls on Thursday. Then 9. (4) By observation (A) is false but (R) is true.
from 27th Jan. to 29 Feb these is total 34 days. 10. (2) By observation Both (A) and (R) are true but (R) is not
34 correct explanation of (A).
= 6 odd days 11. (4) By observation.
7
Thursday + 6 = Wednesday 12. (1) Children who do not like black = 100 + 30 + 20 = 50.
Hence, Hasan’s birthday follow Wednesday. Children who only like black and white = 50
240. (3) 8 2 + 6 = 5 × 2
4 + 6 = 10 10 = 10 150 3
Ratio
241 - 242. 50 1
Required ratio = 3 : 1.
Hostel
13. (3) Top numbers square multiply given in bottom.
2 km (102 112 ) 100 121 12100.
Football (92 × 102) = 81 × 100 = 8100
stadium
(32 × 72) = 9 × 49 = 441
4 km 14. (2) In column : 3 (9 × 5) = 345
Similarly: 5 (7 ×11) = 577.
15. (3) Combinations of two digits number having 8.
85, 58, 82, 28, 81, 18, 87, 78, 85, 68, 88
2 km So, 11 combinations can be made.
Cricket stadium School
16. (4) W T
241. (2) The bus travel 8 kms from school to football stadium.
242. (3) The school is south-west direction with reference to
the football stadium.
243. (3) Effective gain in one minute = 15 – 10 = 5 feet R
It will move 85 feet in 85 17 minutes 17. (4)
5 18. (1) (4 + 5 + 9) = 18.
Next, 18th minute he climbed the pole. 19. (3)
20. (2) The pattern is as :
CHAPTER 3. LOGICAL REASONING
(4 + 12 + 6 + 4)2 –2 = 674
1. (1) (42 6) (15 3) 2 (4 + 14 + 6 + 18)2 –2 = 1762
(12 + 10 + 14 + 16)2 –2 = 2702
(36 9) (9 3) 1
21. (2) 1 5 6
(38 19) (20 10) 0
2 7
2. (2) Sequence AABBCC is repeated.
4 2 1 2
3. (4) BAAC / ACCB / CBBA / BAAC Maharashtra
EBD_7332
88
A- NTSE Stage 1 Question Bank

35. (2) Decoding AVOID = 73564 letter by letter we get,


22. (1) As, 43 42 80
A = 7, V = 3, O = 5, I = 6, D = 4
Similarly, 213 212 9702. Similarly, in CHINA = 28617;
23. (4) moving 1 place in right side. C = 2, H = 8, I = 6, N = 1, A = 7.
24. (1) Let most element is eliminating and new elements add So, the code for COVID :
at right most place remaining elements shift place C = 2, O = 5, V = 3, I = 6, D = 4
towards left side. i.e., COV ID = 25364.
E J O T : V Q L G :: B G L Q : S N I D 36. (1) The colours adjacent to Green and Yellow, Red, Pink
25. (3) 5 10 15 20 22 17 12 7 2 7 12 17 19 14 9 4 and Black. The only colour remaining is White which
+5+5+5 –5–5–5 +5+5+5 –5–5–5 will be on the opposite face to Green.

+2 +2 +2
Red
26. (1) By observations.
Yellow Black
27. (4) - 951874 Black Green Yellow

Pink
Red
28. (4) Multiply the corner elements with each other i.e. 15 × 6 Pink
× 4 = 360 then divided it by 10. White White
360 ÷ 10 = 36 is the middle number. White
Q 50 × 10 × 10 = 5000 ÷ 10 = 500
29. (3) 36 64 25 49 26 37. (4) The number in the intersection of triangle (Rural) and
circle (Unemployed), but not covered by square
9 25 16 81 21 (Hardworking), and rectangle (Intelligents), but not in
the circle (Unemployed), i.e. 6.
25 144 36 64 31
30. (3) “21” represent the common part of all three destination 20 2
38. (2) 11
that’s why 21 is answer. 2
31. (4) 52 + 41 + 46 = 139 102 2
52
32. (1) From the given statement on unemployment, it can be 2
concluded that, to end poverty more employment 39. (1) 62 – 1 = 35
opportunities need to be created. But the statement 72 – 1 = 48
doesn’t convey that all people in the country are 40. (3) 326 : 3649
unemployed. 32 = 9, 22 = 4, 62 = 36
So, the conclusion (I) follows but conclusion (II) Which is written in reverse order as 3649.
doesn’t follow. Similarly,
33. (3) Rupee and Dollar are currencies for different countries. 534 : 16925
They do not have anything in common. 52 = 25, 32 = 9, 42 = 16
Which is written in reverse order as 16925.
Currency 41. (3) The closed figure is separated into two equal parts,
then it is rotated 180 degree and inside figure is rotated
90 degree.
Rupee Dollar
42. (1) As, M A R C H
–2 +2 –2 +2 –2

K C P E F
Similarly,
34. (3) As, O N L I N E
O R D E R
L N O E N I –2 +2 –2 +2 –2
And, S I L E N T M T B G P
L I S T N E 43. (3) P = 16, E = 5, N = 14
So, (16 × 5 – 14) = 66.
Similarly, L I S T E N
Similarly, L = 12, E = 5, G = 7
S I L N E T So, (12 × 5 – 7 ) = 53.
Solved Questions 89
A-

53. (2) 720, 360, 120, 30, 6, 1


44. (3) P D 8
16 4 ÷2 ÷3 ÷4 ÷5 ÷6
54. (2) The pattern of the series is as :
16 4
5 + 7 = 12
4 × 2 = 8
12 + 14 = 26
H B
26 + 21 = 47
8 2
47 + 28 = 75
8 2 75 + 35 = 110
2 2 2 55. (3) The pattern of the series is as :
I. 2, 5, 10, 17, ?
2 2 2 4
+2 +5 +7 +9
23 26 29
45. (3) 17 9 26
II. 2, 4, 8, 14, 22
169 169 ?
+2 +4 +6 +8
16 12 15 13 14 17 56. (1) S.I. unit of ‘angle’ is ‘radian’.
Similarly S.I. unit of ‘Force’ is ‘Newton’.
16 × 12 – 23 = 169; 15 × 13 – 26 = 169; 14 × 17 – 29 = 209
57. (3) As,
46. (3) Two alternative series.
F R U I T
22 19 16 13 10 7
–6 –6 –6 –6 –6
–3 –3 –3 –3 –3
Z L O C N
3 9 27 81 243 Similarly,
×3 ×3 ×3 ×3 F L O W E R
243, 7 is the answer. –6 –6 –6 –6 –6 –6

47. (1) 392, 365, 342, 323, 308, 297, 290, 287 Z F I Q Y L
–27 –23 –19 –15 –11 –7 –3 58. (1) P O L I C E
48. (2) As lion is related with Roar, similarly, Ass is related 265143
with Bray.
59. (1) T
49. (3) 72 7 : 82 8 : : 112 11:122 12
42 56 110 132
F S
50. (2) Multiply all outer number divided by 10.
120 60. (1) The pattern is as :
5 × 6 × 4 = 120, 12
10 52 + 3 = 28
210 52 + 4 = 29
6 × 7 × 5 = 210, 21 52 – 2 = 23
10
320
4 × 8 × 10 = 320, 32 61. (1) 3C 27D 9E C(+1) D (+1) E
10
51. (1) Top numbers square’s difference. 27D
112 52 121 25 96 7I 84K 12M I(+2) K (+2) M
2 2 x
15 9 225 81 144
?
212 16 2 441 256 185 5A 13G A(+3) D (+3) G
65D
52. (3) 95, 94, 92, 89, 85, 80, 74
–1 –2 –3 –4 –5 –6
EBD_7332
90
A- NTSE Stage 1 Question Bank

74. (4) There are three male member in the family.


75. (1) F is mother of C
62. (2)
76. (2) C is brother-in-law of B.
Agricultural 77. (1) F is mother of A
Engineers
Officer 78. (4) E is daughter of C.
79. (2) 3 5
Professionals 5 8 13
4 6
2 6 12
63. (4) (i) ‘im be pee’ - ptals are blue 5 5
9 x 21
(ii) ‘sik hee’ means ‘red flowers’
(iii) ‘pee mit hee’ means ‘flowers are fragrant’ Place value of alphabet in alphabetic series as
From (i) and (iii), Pee - are E = 5, H = 8
From (ii) and (iii), hee - flower, Slk - red and mit - fragrant M = 13, B = 2
Then, ‘fragrant are red flowers’ - mit Pee Sik hee F = 6, L = 12
64. (4) As per the law and moral values, you have to deposit I = 9, U = 21
in the nearest police station whether its small or big then the pattern follows as
amount. 9 + 5 = x = 14 = N
65. (2) | 9 – 3|, | 1 – 6|, | 5 – 4|, = 651 and | 5 – 7|, |4 – 8| and 80. (2) The missing fraction is
|3 – 9| = 246. similarly |8 – 2|, |6 – 4|, |3 – 1| = 622 We get the pattern in Numerator ‘=’ ‘×’ 2 + 1
66. (3) The pattern is: We get the pattern in Denominator ‘=’ ‘×’ 2 + 1
+6 +6 +6 +6 19
=
–4
42
26 4 20 10 14 16 8 22 2 28
81. (3) bcab/bcab/bcab/bcab/bcab
Clearly option (3) is the correct answer.
67. (4) The pattern is:
82. (2) cabbac/cabbac/cabbac
39 56 73 90 107 124 141 Clearly option (2) is the correct answer.
83. (2) Pattern follows as:
+17 +17 +17 +17 +17 +17
2, 10 , 30 , 6 8 , m
68. (2)ABAB ABAB ABAB
69. (1)mopn mopn mopn mopn 8, 20 38 n
70. (4)UVWVU UVWVU UVWVU UVWVU
12 18 24
71. (2)As, the pattern is: n = 38 + 24 = 62
7 (6 – 1) = 7 35 m = 68 + 62 = 130
5 (7 – 4) 5 3 = 15 84. (3) Pattern follow as:
Similarly, 8, 15, 28, 53, m
8 (9 – 2) = 8 7 = 56 7 13 25
72. (4) As, (5 × 1) (4 × 2) (2 × 2) = 584
(6 × 1) (7 × 1) (2 × 4) = 678 1 less than double
Similarly, (3 × 2) (3 × 3) (2 × 2) = 694 m – 53 = 25 × 2 – 1 = 102
85. (2) We get the pattern of the following question
73. (4) As, (11+5) (11 – 5) = 16
(9 + 6) (9 – 6) = 15 3 = 45 15 + 12 = 27 44 + 28 = 72 64 + 53 = 117
Similarly, (15 + 9) (15 – 9) = 24 6 = 144
Solutions for (74 to 78):

F Male 3 8 x

Female
27 72 117
C A B Couple Now, 9, 9, 9
3 8 13
E D Hence here x = 13
Solved Questions A-91
97. (2) The common in all four are 10. Clearly we can conclude
86. (1) Human 10 artist are expert in all the arts.
98. (3) From the given figure 21 + 15 + 35 + 12 = 83 total 83
are good in only two arts.
Nurse Doctor
99. (4)
Doctor can’t be Nurse or Nurse can’t be a Doctor. So,
both different. But both comes in the category of human.
As, = 2 (8 × 9 + 7 × 5) = 2 × 107 = 214
87. (3) Singer

Dancer Teacher
Similarly, = 2 (3 × 6 + 4 × 9) = 108

Both conclusion I and II are true. Hence the missing number is = 108
M 100. (4) Let the mising no. be x
88. (4) F x = (7 + 4)2 = (11)2
Guavas Fruit Mango x = 121
101. (3) Pattern follows as a sequence with a gap of +2, +4,
G +6 and so on.
We have given that some fruits are mango. So, both A + 2 C + 4 G + 6 M + 8 U + 10 E + 12 Q + 14 E + 16 U
the conclusion is false. Hence neither I nor 102. (3) We get the pattern like +2, +2, +3, +3, +4, +4
conclusion II is true.
B1 D1 F1 I1 L1 P1 T1
18
89. (2) As, (15 4) 6 11 17, and +2 +2 +3 +3 +4 +4 +5
3
30 103. (4) We get the pattern as skip the first and last letter.
(18 8) 5 10 15
6 (1) P ERPENDICULA R
36
Similarly, (27 12) 4 15 19
9 (2) E RPLNDICUL A
Mother
90. (4) Teachers (3) R PENDICU L
Females (4) PENDICU
In each step one letter is removed from Starting and
End.
91. (1) Table Chair
104. (2) We get the pattern followed as
Furnitures 12 = 1, 32 = 9
92. (4) ‘6’ number represents those who drink coffee, tea and 13 = 1, 33 = 27
milk. 22 = 4, 42 = 16
93. (2) ‘8’ number represents those who drink only tea and not
coffee. 23 = 8, 43 = 64
94. (1) m n l l / m n l l / m n l l / m n l l 105. (2) As some Indians are educated and educated persons
95. (2) a a b b c c / a a b b c c / a a b b c c like small families. So we can conclude some Indians
96. (3) The pattern is: like small families.
Series I. 28, 14, 6, 3 Only conclusion II follows.
Series II. 77, 24, 7 106. (2) India and Pakistan different from each other but both
Now, are the parts of Asia.
Series I. 22 – 1 = 3
23 – 2 = 6
24 – 3 = 13
25 – 4 = 28
Series II. 7 × 3 + 3 = 24 107. (1) People who are educated and employed both are
24 × 3 + 5 = 77
12 + 6 = 18
Option (3) is correct.
EBD_7332
92
A- NTSE Stage 1 Question Bank

108. (2) Given, = =, = >, = < and = 118. (2) According to the question d = 3; last term = 148
When we apply these symbol we get,
148 4 144
6x = 5y So, 1 1 49
3 3
6 119. (3) The hour hand point is in east direction.
y= x
5
120. (2) Regions 8, 9 indicate villagers are neither employed
2y > 3z nor educated.
6 121. (4) Regions 7, 4, 6 represent educated persons are
2*x > 3z
5 villagers.
2.4x > 3z 122. (3) Region 4 represents educated persons are both
As 2.4x > 3z villagers and employed.
4x > 3z 123. (1) The person looking at his son’s photo.
4x 3z 124. (2) Common solution.
109. (3) Blood relation tree is Hence, E is nephew of A.
+
(+) P Q (–) D

M(+) C

T
A + B
Hence P is Paternal grandfather of T.
110. (2) 114466 114466 114466
E +
Clearly option (2) is correct answer.
111. (3) 3 × 3 –1 = 8, 8 × 3 – 1 = 23, 68 × 3 – 1 = 203 125. (4) Common solution.
112. (1) FOUR FOUR FOUR FOUR Hence, D is grandfather of B.
+
Clearly option (1) is correct answer. D
7 6 3 5 13 2
113. (3) As = 5, Similarly =8 –
2 2 C
114. (3)
As 36 = 18 2 26 = 13 2 Similary ? = 15 2 A + B
90 = 18 5 65 = 13 5 75 = 15 5
So, the missing no. is = 30 E +
115. (1)
126. (4) Common solution.
5 9 8
Hence, E is grandson of C.
+
7 13 4 8 5 4 9 ? 4 D

3 3 3 –
C
As, 7 × 4 – 5 × 3 = 13 8 × 4 – 9 × 3 = 5
Similarly, 9 × 4 – 8 × 3 = 12
A + B
So the missing no. is = 12
116. (3) The pattern for the given series is as follows,
E +
132 253 374 495 ?
127. (4) Pattern is followed as,
+121 +121 +121 +121

Hence, the missing no. is = 495 + 121 = 616


117. (2) The pattern for the given series is as follows,
8 18 32 50 72 ?
128. (3) Pattern is followed as,
+10 +14 +18 +22 +26
5040, 840, 168, 42, 14, 7
Hence, the missing no. is = 72 + 26 = 98 ÷6 ÷5 ÷4 ÷3 ÷2
Solved Questions 93
A-

129. (4) Let the missing no. be ‘x’ 135. (1)

4 5 6 7 a Mother
Women
6 18 28 ? b
Sister
10 7 8 7 c
Here, pattern is a2 = b + c. Based on that
42 = 6 + 10 Only conclusion (1) follows.
52 = 18 + 7
62 = 28 + 8

Tables
72 = ? + 7 136. (4) Books Pencils
49 – 7 = x
Hence, the missing no. is ‘x’ = 42
130. (4) Single alphabet is the sum of the alphabet value given Neither conclusion (1) nor (2) follows.
in the bunch. 137. (3) Count 13th element from the left to get required answer
As, as W.
138. (4) There is no such consonent in the series.
139. (2) After dropping all the symbols the arrangement be-
comes
7 M 4 PJ V 1 K 3 E W2 Q C 6 TA8 Z 1 5 F U 9 H N
D = B+A+A H= D+ B+ B ?=E + E + J
4=2+1+1 8=4+2+2 = 5 + 5 + 10 = 20 T 12th from right
131. (4) South becomes North East 140. (3) From the Venn diagram
North becomes South West The number of non-working females are the ones in the
North West becomes South triangle not in the circle = 17.
(S.W)N 141. (1) The number of non-educated urban women who are
unemployed = 0
(S)N.W N.E 142. (2) Of the total urban educated people, if we remove the
woman (triangle), we get the number of educated urban
males = 40, who are not working.
W(S.E) E(N.W)
143. (1) Ram’s grand father’s son is Ram’s father. Ram’s father’s
daughter is Ram’s sister.
S.W S.E
331 16 2
S(N.E) 144. (4) 100 50 3 25 20 3
132. (1) Here, all peacocks are birds but tigers are animal. 100 100 100 100 100 100
1 2 3 4 5 6
Peacocks
145. (3) Kitchen and bathroom are both parts of house.
(Qs. 146 - 148) :
Birds (Sister)
(Brother/Sister) B C F (Brother)

(Son) A (Son) D E (Daughter)


Tigers
146. (1) A is the cousin of D.
147. (3) E is the niece of F.
133. (1) A + B + C
Hence, A is the mother of C. 148. (3) A and D are both nephews of F.
134. (3) A × B ÷ C South
B
C

149. (1) East


C A

A + B –

D
So, A is son of B. North
EBD_7332
94
A- NTSE Stage 1 Question Bank

150. (2) All males are fathers but only some males can be 159. (3) Fourth Friday will be on 23rd of the month.
Doctors. Also some fathers can be doctors. Fourth Tuesday will be on 27th of the month.
Males 160. (2)
E
Fathers 12
11 1
10 2
N 9 3 S
8 4
Doctors 7 6 5
151. (2) No. of odd days from :
W
17th Dec 2002 – 17th Dec 2003 = 1 Minute hand point towards west and Hour hand
17th Dec 2003 – 17th Dec 2004 = 2 towards south-east
17th Dec 2004 – 22nd Dec 2004 = 5 Time = 1 : 30 hr
Time after three hours = 1 : 30 + 3 = 4 : 30 hours.
5 2 1 8
Total no. of odd days 1 odd day E
7 7 161. (2) P
22nd Dec 2004 will be Saturday + 1 = Sunday S
152. (3) Number of odd days in April = 2 N S
Q R
May = 3
W
June = 2
July = 3 S is to the south east of Q.
So, according to number of odd days April will have 162 (3) Hour hand rotates by 30° in 60 minutes (1 hour) in 10
same calender as July 30
minutes hour hand rotates by 10 5
153. (4) A + B > C + D ...... (1) 60
B+ C> D +A ...... (2) in 5 hours and 10 minutes hour hand rotates by
Adding (1) and (2) 5 × 30° + 5° = 155°.
A + 2B + C > A + 2D + C 3 3 3
2B > 2D 2 5 8 11
163. (3) 3 3 3
888 440 216 104 48 20 16 19 22 25
154. (4) 888 – 8 440 – 8 216 – 8 104 – 8 48 – 8 164. (4) 93 – 27 – 63 = 3
2 2 2 2 2 79 – 38 – 37 = 4
155. (2) Square of opposite number
67 –16 – 42 = 9
3 9
165. (1) The logic is
4 16
7 49 2 × 3 = 56
8 64
156. (1) BAAB AABAABAABAAB
(2 + 3) (2 × 3)
Letters BAA are repeated
BABAB 4 × 1 = 54
157. (1) (7 – 4)2 + (16 – 7)2 = 90 166. (4) 8 4 1 = 3
(28 – 19)2 + (42 – 28)2 = 277 a b c
(60 – 45)2 + (57 – 50)2 = 274 Here, logic is (a ÷ b) + c
North 2
217 7 9
1
North-East
60 m 167. (3) The given pattern is :

20 m 12 2 52 13,
158. (2)
10 m 152 82 17
10 m
East 72 242 25
20 m
So, I am in North - East direction from starting point. 92 402 41
Solved Questions A- 95
168. (2) 6, 24, 60, 120, ....... 178. (1)
1× 2×3=6 E
2 × 3 × 4 = 24 W
3 × 4 × 5 = 60
4 × 5 × 6 = 120 I
5 × 6 × 7 = 210 I
169. (2) By observing row wise, we get II
A = 1, E = 1 + 4 = 5, k = 5 + 6 = 11 So, only conclusion (I) is true
179. (3)
G = 7, k = 7 + 4 = 11, Q = 11 + 6 = 17
180. (1)
M = 13, Q = 13 + 4 = 17, 17 6 23 w 181. (2) 15 Students got distinction in both physics and maths
subject only.
170. (4) 3 × 4 = 12 (Alphabet Position of L) 182. (1) As the numbers 2, 3, 4 and 5 are adjacent to 6.
1 × 17 = 17 (Alphabet position of Q)
5 × 4 = 20 (Alphabet position of T) Hence, the number on the face opposite to 6 is 1.
171. (4) After removing even numbers :
2 2
183. (2) 5 (5 – 2) = 9 21 (21 – 19) = 4
15 97153597 5 9 13 513
Middel Number 16 109 2 22 53 19
172. (4) Total hours = 89 hours 2
(16 – 6) = 100
6 2
(22 – 15) = 49
15
44 356 100 + 9 = 109 49 + 4 = 53
Faulty time 23 hours & 44 min = 23 i.e, hr
60 15 2
51 (51– 48) = 9
356
hr of incorrect watch = 24 hr of correct watch
15 17 ? 48
24 15 13
89 hours of incorrect watch = 89 90 hrs (17 – 13)2 = 16
356 16 + 9 = 25
Actually watch will be 1 hr faster than faulty watch. 184. (2) The pattern is :
i.e., 11 pm 7 × 2 + 2 = 16
173. (2) Unemployment 16 × 2 + 2 = 34
Illiterates 34 × 2 + 2 = 70
70 × 2 + 2 = 142
142 × 2 + 2 = 286.
So, missing number is 70.
185. (2)
186. (2)
187. (2) The sequence is :
Poor
174. (4) The pattern is: ML K J I H G F E D C BA ZY XWV UT S R Q PO N
32 + 31 + 30 + 28 = 121 = (11)2
70 + 72 + 73 + 74 = 289 = (17)2 12th letter from left 7th letter right of B.
So, U is 7th to the right of the 12th letter from the left
112 + 108 + 106 + 115 = 441 = (21)2
missing number = 17 side.
1756.(4) The pattern is: 188. (1) The arrangement of five persons is as following:
5 + 12 + 13 = 30 = |3 – 0| = 3 Kamal > Rashi > Vinita > Priti > Leela
13 + 9 + 4 = 26 = |2 – 6| = 4 3rd in order of height is Vinita.
7+ 5 + 16 = 28 = |2 – 8| = 6 15 6 4
189. (3) As, In 1st figure, 36
missing number is 6. 10
176. (1) According to question, 6 7 5
and In 2nd figure, 21
3 30 – 20 2 10
50 10 10
3 67 – 40 3 Similarly, In 3rd figure , 500
10
3 416 – 200 6 190. (3) how can you go Je de ke pe ...(i)
So, missing number is 6. you come here ne ke se ...(ii)
177. (4) By observation. Option (4) is correct answer. come and go re pe se ...(iii)
From (ii) here ne
EBD_7332
96
A- NTSE Stage 1 Question Bank

191. (3) Alternate series: 199. (1) The pattern of the series is as follows :
+5 +5 +5 121 144 169 196 225 256
2 65 7 59 12 53 17 47
(11)2 +1 (12) 2 +1 (13) 2 +1 (14) 2 +1 (15)2 +1
(16) 2
–6 –6 –6 200. (2) The pattern of the series is as follows :
192. (4) As, (5 – 4) + (4 – 3) 1+1 2
and (6 – 0) + (5 – 1) 6 + 4 10 5 10 20 40 80
Similarly, (6 – 2) + (7 – 2) 4+5 9 5 10 20 40
193. (2) N N 5 10 20
12 12
5 10
W 9 3 E W 9 3 E
5
6 6 201. (3) The correct venn diagram represents female, mother
S S and doctor is as shown below.
Given : 12 : 30 2 : 45
It is clearly shown that the minutes hand will be in west Female
direction.
194. (3) The pattern is as follows: Mother
4×1=4 Doctor
4×2=8
8 × 3 = 24
24 × 4 = 96 202. (2) The sequence is as follows :
96 × 5 = 480
4 36 5 7 5 3 6 4 5 7 3 57 3 5 3
480 6 2880
So, it is clear that only one 5s which is immediately
2880 × 7 = 20160
followed by 3 but not immediately preceded by 7.
E
195. (1 ) A C 203. (3) Option (3) is correct answer.
204. (1) The pattern of the series is as follows:
16 19 28 43 64 91
G O H
+3 +9 +15 +21 +27
+6 +6 +6 +6
B 205. (3) The pattern of the series is as follows :
F D
Smallest triangles = AOE, EOC, COH, HOD, DOF, FOB, 63 58 51 40 27 10
BOG and GOA i.e. 8.
Contains 2 triangles = AOC, BOD, COD and AOB i.e. 4 –5 –7 –11 –13 –17
Contains 4 triangles =ABD, CBD, ABC and ACD i.e. 4 206. (2) The pattern of the series is as follows :
Total triangles = 8 + 4 + 4 = 16. 3 4 8 17 33 58 94
196. (3) The pattern is as follows:
9 + 3 = 12 +1 +4 +9 +16 +25 +36
12 + 6 = 18
18 + 12 = 30 12 22 32 42 52 62
+1 +1 +1 +1 +1
30 + 24 = 54
207. (2) As, I Figure : 48 ÷ 6 = (8)2 = 64
54 + 48 = 102
197. (4) The blood relation diagram is as shown below: II figure : 30 ÷ 5 = (6)2 = 36
Mother Similarly,
III figure : 35 ÷ 7 = (5)2 = 25
Father-in-law Amit 208. (1) Alternate series :
Grandson
582 588 634 600
Girl Wife Son
So, Amit is father-in-law to the girl. +6 +6 +6
198. (1) English and Hindi both are the languages. 605 611 617

+6 +6
English Hindi There should be 594 in place of 634.

Language
Solved Questions 97
A-

209. (3) Alternate series : (13 7) (28 25) 20 3 60 30


C F I L O R 2 2 2
Similarly,
+3 +3 +3 +3 +3
In 3rd figure :
Z X V T R
(2 8) (16 10) 10 6 60 30
–2 –2 –2 –2 2 2 2
210. (3) The figure may be labelled as shown. 215. (4) The code of EUKAR will be
A B 216. (2) As, 42 × (4 – 1) = 16 × 3 = 48
I J C Similarly, 52 × (5 – 1) = 25 × 4 = 100
O D 217. (3) 218. (2) 219. (1) 220. (2) 221. (2)
N
P 222. (1) 223. (4) 224. (4) 225. (1) 226. (2)
H QK 227. (1) 228. (2) 229. (4) 230. (3) 231. (4)
G M L 232. (4) 233. (1) 234. (3) 235. (4) 236. (4)
F E 237. (2) In the given numbers grid by observation.
The simplest triangles are IJO, BCJ, CDK, KQL, MLQ, 238. (4) In the given numbers grid by observation.
GFM, GHN and NIO i.e. 8 in number. 239. (1) In the given numbers grid by observation.
The triangles composed of two components each are 240. (1) In the given numbers grid by observation.
ABO, AHO, NIJ, IGP, ICP, DEQ, FEQ, KLM, LCP and 241. (3) In the given numbers grid by observation.
LGP i.e. 10 in number 242. (4) The different letters follows the given sequences:
The triangles composed of four components each are
HAB, DEF, LGI, GIC, ICL and GLC i.e. 6 in number. 1st letter B C D E (next letter from B to E)
Total number of triangles in the figure = 8 + 10 + 6 = 24. 2nd letter Y X W V (reverse from V to Y)
211. (1) The figure may be labelled as shown. 3rd letter A B C D (next letter from A to D)
C 243. (1) Father
B L
A N D
M
Q Son Wife Amir
K
I
P O
H E
J
Daughter
G F Amir’s daughter mother i.e. Amir’s wife
The simplest squares/rectangles are - ABCI, LCDK, Amir’s wife’s father’s son (Manjeet) i.e. brother of
HIJG, JKEF, MLQI, LNK, QKOJ and QJPIQ i.e 8. Amir’s wife hence Manjeet is Amir’s brother in law.
The squares/rectangles composed of two components 244. (3) 144 = 122; 121 = 112, 100 = 102, 81 = 92, 82 = 64
each are ABKJ, ILEF, HLKG, IJCD, MLQJP, LNOJ, MNKI 245. (4) In such questions we must try & see if the first/second
and IKOP i.e 8 alphabets are forming some sequences separately.
The squares/rectanglescomposedof three/four components
Z, Y, X is followed by W (reverse)
each are - ABEF, HGDC, ILKJ and MNOPL i.e 4.
Total no. of squares/rectangles in the figure = 8 + 8 + 4 = 20. A, B, C is followed by D
212. (2) As, 37 = 3 + 7 = 10 13, 15, 17 is followed by 19 (increase of 2)
46 = 4 + 6 = 10 246. (4) B, C, D is followed by E
and 28 = 2 + 8 = 10 N, M, L is followed by K (reverse order)
Similarly, In option (2), 55 = 5 + 5 = 10 Q, R, S is followed by T.
213. (3) The pattern of the series is as follows: 247. (2) M H S g
248. (4) Only S H d
100 , 25 , 100 , 50 , 100 i.e. 20
9 2 7 3 5 249. (3) M 4; E 3; N 8; T 1
–2 –2 250. (4) Each alphabet is replaced by its next alphabet in the
code.
214. (2) As, In 1st figure : So CORE will become DPSF.
(5 7) (21 17) 12 4 48 251. (2) 5, 6, 10, 19, 35, .........
24 The gap between two numbers is perfect square 1, 4, 9,
2 2 2
16 so next gap should be 25, so the next number in the
and In 2nd figure : series
= 35 + 25 = 60.
EBD_7332
A-98 NTSE Stage 1 Question Bank

1 1 1 1 Musician
252. (1) Ist Letter: L M N O P
259. (4)
Instrumentalists
2nd Letter :
4 4 4 4
X T P L H
Violinists
3rd Letter: All violinists are instrumentalists.
4 4 4 4
F J N R V All instrumentalist are musicians.
253. (2) 23 × 2 + 2 = 48 260-263.
48 × 2 + 3 = 99 E C A
99 × 2 + 5 = 203
203 × 2 + 7 = 413
F B D
413 × 2 + 11 = 837
254. (4) r s s r / r s s r / r s s r i.e. s r s r where, Male; Female; Sibling
T R I P P L E
255. (3) –1 +1 –1 +1 –1 +1 –1 relationship; spouse relationship.
260. (4) There are 4 male members are there in the family.
261. (4) A is mother of B
D M O Q H S S
262. (3) A have three children
Similarly,
263. (4) AD is a pair of females.
V I C T O R Y
–1 +1 –1 +1 –1 +1 –1
264. (2) H E A T E R
+3 –3 +3 –3 +3 –3
X S N U B J U
K B D Q H O
256. (2) Human beings
Similarly,
C O O L E R
+3 –3 +3 –3 +3 –3

Graduates F L R I H O
Teachers
265. (1) Cousion. Father
All teachers and graduates are human beings. But,
some teachers may be graduates and some graduates
Uncle Daughter
may be teacher.
Anti-social elements Girl Son
257. (2)
266. (1)
245
Black mailers
8 15 31 61 123 247 491

×2–1 ×2+1 ×2–1 ×2+1 ×2–1 ×2+1


Pickpockets 267. (3)
Both pickpockets and blackmailers are anti-social 120
elements. But, some pickpockets may be blackmailers
and vice-versa. 3 6 24 30 63 72 122 132

258. (1)
Students 2 2 2 2 2 2 2 2
2 –1 2 +2 5 –1 5 +5 8 –1 8+8 11 – 1 11 + 11
268. (1) By observation.
Boys
269. (3) By observation.
270-271. Letter Code Letter Code
Players
A s B l
Some boys are students. C q E t
Some students are players. G u H f
Some players are boys. I p J a
Solved Questions 99
A-

L g M d 278. (3) Only teachers, neither experienced nor trained work in


N k O b Rural Area–
P r R w A No which comes in But not in and .
S o T x It is = 3
279. (4) Only trained, not experience and not work in
U e V z
Rural Area–
W y X v
Z n A No which comes in But not in and .
It is = 1
270. (3) S o, I p, N k, E t 280. (2) NOIDA INDIA
271. (2) A s, R w, C q, H f 39658 63568
281. (2) acac/abab/acac/abab/acac
272 (3) A B Hence, a a c b is answer.
282. (1) aabb/aabb/aabb/aabb
Hence, b a a b b is answer.
? C D 283. (4) a b / b a / a b / b a / ab/ b a / a b
Hence, a b a a b a is answer.
Where, Male; Female 284. (3) aba/cac/aba/ cac/aba
Hence, a a b a b is answer.
Therefore, B is uncle to C.
285. (1) sdo/pdz/sdo/pdx/sdo
273. (1) P Q P is daughter of Q. 286. (4) wp qsu/wpqs v/wpq
Q–R Q is husband of R. 287. (2) In first figure;
18 × 2 = 36; 18 × 5 = 90
Represents this in diagrammatic form. We get, In second figure,
13 × 2 = 26; 13 × 5 = 65
Q R
Similarly,
15 × 2 = 30; 15 × 5 = 75
288. (1) In first figure : 5 × 4 + 6 = 26
P In second figure : 4 × 7 + 3 = 31
Similarly, 3 × 4 + 7 = 19
Then, R is mother of P.
289. (4)
6 10 14
274. (4) Males +6 +7 +8
12 17 22
Nephews +7 +8 +9
Nieces 19 25 31

290. (3) 7 × 3 – 1 = 20
8 × 3 – 1 = 23
275. (1) Each note contains its square or square root at opposite
9 × 3 – 1 = 26
side.
291. (4) In first figure :
81 (square)
In case of 9 3 × 3 + 5 × 6 = 9 + 30 = 39
3 (square rooot)
In second figure :
81 is present in option.
4 × 4 + 5 × 7 = 16 + 35 = 51
276. (2) 1 2 2 5 2 3 Similarly,
2 2 2 2 2 2 3 × 4 + 5 × 5 = 12 + 25 = 37
1 2 2 5 2 3
292. (4) Father
14 425 49

277. (4) Experienced but not trained teachers work in Lady/Mother


Rural Area–
A No which comes in and both. Ramu
But Not in Hence, lady is mother to Ramu.
It is = 6
EBD_7332
100
A- NTSE Stage 1 Question Bank

293. (1)
Millets
Photos

Pictures Cheap

299. (2) Oranges


Fruits
Conclusions: I : True
II : False
300. (3) Letter ‘e’ is common in all.
294. (2) Start
301. (3) Qualified
Moons Doctors

Planets Experienced
Doctors

Conclusions: I : False
II : True So, Letter ‘F’ is answer.
Guitarists Experienced
302. (1) Doctors

295. (1) Musicians Writers


g

Drs. working in villages

Insects 303. (1) Only letter ‘c’ do shows Doctors who are neither
qualified nor experienced but working in villages.

304. (4) Qualified


296. (3) Locusts Flies Doctors

a
d
Oranges Drs. Working
in villages

305. (1) Sum of terms is equal to 18. i.e.


297. (2) Professors 6 + 5 + 7 = 18; 3 + 6 + 9 = 18
People 9 + 5 + 4 = 18; 4 + 6 + 8 = 18
306. (4) Here,
7 × 6 = 42, 5 × 4 = 20, 8 × 7 = 56
So, 4 × 3 = 12
District 307. (4) Kamal Jaipur, Srinagar,Solapur
Lalit Nagpur, Jaipur, Srinagar
Vivek Solapur
298. (4) Mandal Panchayat Karan Nagpur
From table, we get to know that Kamal-Vivek has not
been to Nagpur.
Solved Questions 101
A-

308. (3) 1. (4) E is the mother.


309. (3) 2. (3) A is E’s husband.
310. (3) 21 12; 45 54; 17 71 12 + 54 + 71 = 137 3. (4) There are 4 male members in the group.
4. (1) F is son of E.
31 13; 73 37; 13 31 13 + 37 + 31 = 81
5. (1) ABF are brothers.
23 32; 51 15; 72 27 32 + 15 + 27 = 74
–5 –4 –3 Sol. (6-8) P
A Q C L F H J E
311. (1)
A L
+2 +3 +4

+1 +2 +2 +2 S K
312. (1) B C E G I
–1 –2 –2 –2 ( - Male, - Female)
Y X V T R
+1 +2 +2 +2
C D F H J 6. (2)
–1 –2 –2 –2 7. (1)
X W U S Q
8. (3)
–1 –2 –2 –2
W V T R P 9. (4) As,

+1 +1 R O M E
313. (1) +1
FOX GPY HQZ
M O R E
+1 +1 Similarly,

F B D A R E
314. (4) =
O E
The place value of F = 6 R A D E
The place value of O = 15
t
6 23 B 10. (1)
p q
15 5 E

L 12 3 C r
Thus, s
P 16 4 D
315. (2) L 12 ; D 4 12 – 4 = 8 H 11. (3) Sushil Vipin Prashant Amar
Hence, L 12, A 1 12 – 1 = 11 K. Drawing Music Drawing Dance
316. (4) The reverse place value of G = 20 Move Dance Dance Craft
The reverse place value of O = 12 Craft Craft
GO = 20 + 12 = 32 12. (3)
SHE = 8 + 19 + 22 = 49 13. (3)
Thus, SOME = 8 + 12 + 14 + 22 = 56
317. (1) C
Sol. (14-16) E F
CHAPTER 4. PROBLEM SOLVING

D(+) D B
Sol. (1-5)
H G
E A B
(–) (+) (+) A

C(–) F (+) 14. (3)


15. (2)
16. (2)
EBD_7332
102
A- NTSE Stage 1 Question Bank

Sol. (17-20)
As uncle

Subject Class A E F C B D

A English IX As uncle
B History VIII
A E F B D C B D
C Hindi VI
27. (2) Three Female F, B, D
D Chemistry V
28. (4) Cousin
E Maths X
29. (1/2)Cousin or Sister.
F Physics VII Sol. (30-32)
G Geograph y XI P is neither wearing White nor Green Cap
i.e. P – White, Green ...(1)
17. (1) From the table, F is in VII class. Q is not wearing White Cap
18. (3) From the table, G is in XI class. i.e. Q – White ...(2)
19. (3) From the table, A likes English. S wears Red Cap
20. (1) From the table, B is in VIII class. i.e. S – Red ...(3)
Sol. (21-22) From (1) P is either Red or Blue
Smt. Mittal Shri Mittal Smt. Rita Goyal Shri Sumer Goyal S is already Red so P is Blue and
Q is Green from (2).
Kriti Amit Ananya Sonu Rakesh
P Q R S

Urvi Poorvi Blue Green White Red


21. (3) From the family tree, Sumer is the maternal grandfather 30. (1) R wears White Cap.
of Poorvi. 31. (4) So P wears Blue Cap.
22. (2) From the family tree, Urvi is the niece of Kriti.
32. (2) So Q wears Green Cap.
D 33. (2) Number of girls in the row
Sol. (23-25)
= 17 + 27 – 1 = 43.
E– A+ B+ th
17
+ –
F C
Reena Malini Pallavi
23. (1) Three, A, B and F. st th st
31 27 21
24. (3)
34. (4) D
25. (4)
H F
26. (4) Mother
B A
F

G E
C
Woman P
Same So, H is facing E.
35. (1) Atul Cricket & Football
Tushar Cricket, Football, Kabaddi
Person
Nishant Kho-Kho, Kabaddi
Amar Cricket, Football, Kho-Kho, Kabaddi
Sol. (27-29) Kabaddi is being played by Tushar, Nishant and Amar
Two figures are possible. together.
( Male, Female) 36. (4) Amar plays all the games.
Solved Questions A-103
37. (1) C is sitting opposite to F. 44. (3) Three boys like English.
A E 45. (3) Three boys like Science.
46. (1) Let Sam, Ali, Rahul and Deepak age be S, A, R, D
F C According to the question,
A
B D S= …(i)
38. (3) B is sitting between D and F. 4
S = 11 + R …(ii)
D B
R–5=D …(iii)
Sequence of ages: Ali > Sam > Rahul > Deepak
C F
Hence, Ali is the eldest.
E A 47. (3) Deepak is the youngest.
48. (4) When Deepak’s age is nine year, then Rahuls’ age
39. (1) A is sitting between B and C.
= 9 + 5 = 14 years
A E Sam’s age = 14 + 11 = 25 years.
Ali’s age = 25 × 4 = 100 years.
F C 49. (4) Five boys are: P, Q, R, S and T.
1 2 3 4 5
B D
S R T P Q
40. (2) Hence, S won the race.
50. (1) T S
P R
Q T
Order in which five boys finished the race:
1 2 3 4 5
S R T P Q
Hence, Q got the last place.
G is sitting second to the right of E. C
41. (4) 51. (3) E F

A B
D
Between B and C.
52. (2) Correct sequence as per weight is :
T > U> R> S= P > Q
K is the professor.
53. (4) J and k, both are males.
42. (3) 54. (3) H, G and E
55. (3) J is sitting to the immediate left of F.
56. (3) H, L and J are sitting between k and F.
Sol. (57-59)
Sitting Arrangement.
A/C G B H C/A
Except option (3) all are sitting opposite to each other.
43. (3) Hindi is liked by 4 student, which is the maximum,

D I E J/F F/J
57. (2) 58. (1) 59. (4)
EBD_7332
104
A- NTSE Stage 1 Question Bank

Sol. (60-61): Sol. 68 -70 The arrangement is as following :

Rajesh Priya Madavi Sanskrit Science Hindi English Mathematics


Dinesh
Ganesh
Umesh
Girija Vinod
Nita
Gita

68. (3) Hindi subject opted by the most students.


Uma Satish Gyan 69. (2) Mathematics opted by the least students
70. (4) Three students opted sanskrit subject.
60. (1)
61. (3) Sol. 71-73
Solution (62-64) : The sitting arrangement is as following :
G B C D E F A Facing
C +
E+ North
A– 71. (3) G and B are to the left of C.
72. (2) C and G are the neighbours of B.
73. (1) E is to the immediate left of D is not true.
Sol. (74– 76)
Arrangement from Richest to poorest
Bhanudas > Akhil > Amar > Chaitanya > Gopal
D– B+ F+ (Farmer) (Lawyer) (Doctor) (Teacher) (Photographer)
74. (4) Chaitanya's occupation is Teacher.
62. (4)
75. (2) Bhanudas is a farmer.
63. (4) ‘A’ is mother of ‘B’
64. (3) 76. (1) The pattern is as follows:
abcd/bcad/ ca bd / abcd/bcad/ cab
65. (2) PLAYERS Foot Ball BaseBall Cricket VolleyBall Hockey 77. (2) 78. (3)
Hemant (79 – 83)
Madhav P PG Eng
Govind Q Home Hindi / History
Ramesh R Home Chemistry
Anant
S Hostel Physics
T PG Maths
66. (1) PLAYERS Foot Ball BaseBall Cricket VolleyBall Hockey U Hostel Hindi / History
Hemant
Madhav
79. (2) P studies English
Govind 80. (1) English - Hostel is not correct.
Ramesh 81. (2) SR
Anant 82. (3) Q studies either History or Hindi
83. (2) QR
67. (3) The arrangement is as following : 84. (1) A is the last lecture.
The correct order is:
M
F Sun Mon Tues Wed Thus Fri Sat
P
D B E F C A

J (85 -88). Given that:


B
V scores more than P, i.e., VP. ...:(1)
P scores more than Q, i.e., PQ ...(2)
T R (1) & (2) VPQ
It is clearly shown that, R is fourth to the right of M. If R scores the highest, one get. R.....T.
So, correct option. is (3).
If S scores the highest, one gets either S.....Q or S....V.
Solved Questions A-105

85. (4) If S ranked sixth and Q ranked fifth we have the sequence 10. (2) By observation.
...... QS ...... . In this case R will be ranked highest and T
the lowest, and we have the order R ...... QST. Also the 11. (4)
order VPQ will stay. So V and P will have second, third
or fourth place. So, the options a, b and c are wrong.
Hence option (4) is correct answer.
86. (2) If R is second, S will rank first and Q and U lowest. But Q
ranks fifth. So, U ranks least. Thus, in view of order VPQ,
the arrangement will be SRVPQ – U. So, T will be ranked
sixth. 12. (1) The triangles are
1, 2, 3, 4, 5, 6, 7, 8, (1, 2), (1, 3), (2, 4), (3, 4), (6, 7), (5, 6, 7),
87. (1) If S is second, R ranks first and T ranks least. So, the
(3, 4, 8), (5, 3, 4), (6, 7, 8)
arrangement, in view of order VPQ, will be R, S, ...... T. So, total triangles are ‘17’.
Thus, (2), (3) and (4) are not true. Hence, option (1) is
correct.
88. (1) If V ranks fifth, P and Q will occupy sixth and seventh 1 2
place. So, Q ranks least and thus, S will score the highest.
3 4
(89-90). Teacher Subjects 5 8
P English
Q Geography 6 7
R Maths
13. (3) By observing arrow direction, second figure direct
S Hindi
towards the x-y axis.
T History
14. (3) In first fig. lower half is not water image of upper half.
15. (3) By observing symbols.
89. (3) The teacher of geography and history are Q and T. 16. (4) By observing the shift and rotation.
90. (1) The teacher of English and Maths are P and R. 17. (3) By observing the symbols.
(41-43).
18. (4) By observing shift.
19. (4)
Experts Art forms 20. (2) There are 10 semicircles:
Suman Acting, Singing, Drawing ACB, ADB, CAD, CBD, HEG, HFG, EGF, EHF, IJ, and JI.
Komal Acting, Singing, Drawing, Debating, Sculpture I
G
Geeta Singing, Debating
Preeti Drawing, Sculpture C
Shweta Drawing, Debating
B A
E F
91. (4) Shweta only knows drawing and debating.
92. (2) Geeta only knows singing and debating. D
93. (2) Komal knows all the art forms. H
CHAPTER 5. NON-VERBAL REASONING J
21. (1)
1. (4) (1), (2), (3) figures contains number of sides 3, 4, 5, 6
22. (3) Outer part is inside the inner part.
either in clockwise (or) anticlockwise.
But (4) do not obey the same rule. 23. (3) The small quadrilateral is inside the big quadrilateral.
2. (3) 24. (2) There are 11 squares in the figure.
3. (4) Figure (4) contains both dots in same side of diagonal. 25. (1)
4. (2) First take mirror image and then water image.
5. (2) A : B : : C : D
Interchange positions of two figures.
6. (2) Take mirror image of figure and reverse the tail.
7. (1)
8. (2) By observation.
9. (3)
EBD_7332
106
A- NTSE Stage 1 Question Bank

G F
E
35. (3)
26. (1) H I
D

A C
B
Number of triangles in the given fig. are 10 and that are :
ABI, AHI, GHI, GFI, EFI, DEI, EGI, AGI, ACE, No. of squares = 24
AEG.. 36. (3) By observing the pattern figure D and A have similar
27. (4) Mirror image is option (4). characteristics.
28. (3) The nos. 1, 2, 3 and 4 are on adjacent faces of no. 5. 37. (3) 8 squares can be formed,
So, no. 3 lies opposite to face of 4. 38. (4) Adjacent faces of 1 are 2, 3, 4, 5.
So, opposite face of 1 is 6
39. (3) 40. (4) 41. (3)
42. (1) 43. (2) 44. (1)
45. (4) From one row to another the contents of each cell
moves one place towards right.
46. (3) 47. (4) 48. (4)
49. (4) 50. (4)
29. (2) Here the lower part of the Ist is in the IInd part.
51. (3) By observation. Option (3) is correct answer.
Hence, option (2) is correct.
52. (3) C
30. (4) There are (3) quadrilaterals in the figure.

B M D
D E
O A E
P O N

B C G F
P
ADOE, DOPB, EOPC 53. (2) A
31. (2) By visualizing the pattern, option (2) is the correct an-
swer.
32. (4) By visualizing the pattern, option (4) is the correct an-
swer. 54. (3) 55. (4)
56. (1) 4 side divided into 4
10 side divide into 10
P Q
33. (3) 57. (2) 58. (2) 59. (3)
R S
60. (4) 64. (3)
62. (3)
Rectangles are P, Q, R, S, PQ, RS, PR, QS, PQRS
63. (1) 'X' shifts to downwards and '–' shifts to left side.
No. of rectangles = 9
64. (2) Option (2) will be the next figure.
65. (4) Option (4) will be the next figure.

66. (2)
4
34. (2)
6 3 5
2

No. of line segments = 13 1

So, if 1 is at the bottom then 3 will be on the top.


Solved Questions 107
A-

67. (2) The water image is as shown below: 102. (2) 1 is wrong as 2 dots can’t be opposite 5 dots; 3 is
720 wrong as 3 dots can’t be opposite 4 dots; 4 is wrong as
water
6 can’t be opposite 1 dot.
68. (4) By observation, option (4) is correct answer.
103. (4) By visualizing the figure
69. (3) In each pair, both figures interchange their position.
104. (2) By visualizing the figure
Inner figures becomes small and stands on the top of
105. (2) Similar figure repeats in every second step. Each time
the first figure.
particular figure reappears, the outer element moves
– 90° ACW along the circumference of the circle and the
70. (3) $ @ inner black triangle moves 90° CW along the
circumference of the circle.
+
106. (4) 107. (3) 108. (1) 109. (4)
8 110. (1) Only one central cube in the middle layer has no face
opposite opposite
painted at all.
71. (2) As, B alphabet
Y, D alphabet
W
opposite Green
Similarly, L alphabet
O
72. (2) As, In Ist figure : 12 + 4 + 9 + 3 = 28 ÷ 4 = 7
In 2nd figure : 5 + 13 + 6 + 12 = 36 ÷ 4 = 9
and In 3rd figure : 8 + 3 + 2 + 7 = 20 ÷ 4 = 5
Similarly, In 4th figure : 21 + 5 + 16 + 6 = 48 ÷ 4 = 12
Blue
73. (3) By observation, option (3) is correct.
74. (1)Option (1) is correct.
75. (1)Option (1) is correct answer.
111.(4) This figure comes in
76. (3)Option (3) is correct answer.
77. (4)Option (4) is correct. 112. (2) 113. (2) 114. (1)
78. (3) 79. (3) 80. (2) 81. (1) 82. (2) 115. (2)
83. (4) 84. (4) 85. (1) 86. (3) 87. (2)
88. (1) 89. (2) 90. (4) 91. (4)
92. (4)Opposite pair are (2, 4), (1, 6) and (3, 5)
i.e. from option only (4) is in correct shape. 116. (1)
93. (2) Opposite pair are (B, F), (C, E) and (A, D) 117. (3) For mirror image watch the original figure from back in
i.e. from option only (2) is in correct shape. the shadow of light. That changed figure will be the proper
94. (2) By visualizing the figure option 2 is correct. mirror image.
95. (1) By visualizing the figure option 1 is correct. 118. (3) The figure may be labelled as shown :
96. (2) Total number of Parallelograms are 17.
97. (4) A F B

98. (3) The triangular portions will overlap with the semi-circle M
E G
on the left side.
99. (3) Shaded position is moved 45 degrees in anticlockwise D C
H
direction.
100. (4) One side is increased in the figure and outer line taken The quadrilaterals composed of two components each are –
inside and kept down opening upwards in ‘V’ shape.
AFME, FMGB, EMHD, MGCH i.e. 4 in number.
1
101. (3) The quadrilaterals composed of four components each are –
3
AEGB, AFHD, FBCH, EDCG, EFGH i.e. 5 in number
5
2 6 The quadrilateral composed of eight components is –
ABCD i.e. 1 in number
4 So, quadrilaterals are = (4 + 5 + 1) = 10
Number opposite to 3 is 4
EBD_7332
A-108 NTSE Stage 1 Question Bank

119. (2) The figure may be labelled as shown :


F
The simplest triangles are – A B
M
AEM, EMP, MDH, MHC, MCG, MGB, E G
125. (1)
0
MBF, MFA i.e. 8 in number.
D C
The triangles composed of two components H

each are –
AMD, ABM, DMC, BMC i.e. 4 in number
So, triangles are = (8 + 4) = 12
126. (4)
120. (3)
121. (5) In each step, new image comes in clockwise direction.
Hence, option (5) is correct answer. 127. (3) The central blocks of each middle layers are those
122. (1) In every step, the shaded square shifts one space having no surface uncloured. Hence, there are 8 such blocks.
anticlockwise in the same rows.
123. (1) By visualizing the figure. 128. (2)
124. (3) In all the figures except (3) outer curve elements count is 4 129. (1)
but in figure (3), it’s having 3 elements. Hence, it’s odd one out.
Stage-1 (2020-21 – 2012) Solved Questions

Scholastic Aptitude Test – SAT

SECTION 1. PHYSICS
1. Assertion (A) : Cooking food is difficult on hills. 9. A body is travelling with speed 20 m/sec having acceleration
Reason (R) : The boiling point decreases with increase of 4 m/sec2 the speed of the body after 2 sec is
pressure. (AP_FIITJEE 2020-21) (Bihar 2020-21)
(1) Both A and R are true and R is the correct explanation (1) 8 m/sec (2) 12 m/sec
of A (3) 16 m/sec (4) 28 m/sec
(2) Both A and R are true but R is not the correct explanation 10. The weight of a body is 9.8 Newton, when g = 9.8 m/sec2 the
of A mass of the body is (Bihar 2020-21)
(3) A is true and R is false (1) Zero (2) 9.8 kg
(4) Both A and R are false (3) 10 kg (4) 1 kg
2. A plane mirror is approaching you at a speed of 5 cm/sec.
11. Flying birds has (Bihar 2020-21)
The image will approach you with a speed
(AP_FIITJEE 2020-21) (1) only kinetic energy
(1) 0 cm/sec (2) 5 cm/sec (2) only potential energy
(3) 10 cm/sec (4) 20 cm/sec (3) both kinetic energy and potential energy
3. A body mass 50 kg has momentum of 3000 kg-m/sec. Then (4) only pressure energy
its kinetic energy is (AP_FIITJEE 2020-21) 12. Two bodies of mass 2 gram and 4 gram having same kinetic
(1) 900 J (2) 90,000 J energy having their ratio of linear momentum as
(3) 90 J (4) 9,00,000 J (Bihar 2020-21)
4. The distance between the near point and the eye of a person (1) 2 : 1 (2)2 :1
suffering with hypermetropia is 50 cm. Then what is the
power of the lens to be used by the person? (3) 1 : 2 (4) 1 : 16
(AP_FIITJEE 2020-21) 13. The sound of same pitch and loudness are "distinguished
from one another by their (Delhi 2020-21)
(1) +1D (2) +2D
(1) Wavelengths (2) Velocity
(3) +3D (4) +4D
(3) Quality (4) Tones
5. The power of a lens in the spectacles of a person is –2D.
14. A water pumps lifts water from a level 10 m below the ground.
The person suffers from (AP_FIITJEE 2020-21)
The water is pumped at the rate of 30 kg/min with negligible
(1) Hypermetropia (2) Myopia velocity. Calculate the minimum power the pump should
(3) Colour blindness (4) Presbyopia have to do this work. (Delhi 2020-21)
6. Two block of masses 8 kg and 12 kg connected at two ends (1) 49 J/s (2) 490 J/s
of a inextensible string. The string passes over a frictionless
(3) 500 J/s (4) 48 J/s
pulley. Then the acceleration of the system is
15. Six identical resistors connected between points A, B and C
(AP_FIITJEE 2020-21)
as shown in diagram. The equivalent resistance would be
g g maximum between (Delhi 2020-21)
(1) (2)
4 5 C
g g
(3) (4) R R
8 6 R
7. In Circular motion, particle moves with (Bihar 2020-21) R R
(1) Constant Velocity (2) Zero Acceleration R
(3) Constant Speed (4) Speed changes A B
8. A car travels half distance with speed 20 m/sec and next half
(1) A and B
distance travel with 30 m/sec, the average speed of the car
is (Bihar 2020-21) (2) B and C
(1) Zero (2) 25 m/sec (3) A and C
(3) 24 m/sec (4) 5 m/sec (4) Option (1), (2) & (3) are correct
EBD_7332
2
B- NTSE Stage 1 Question Bank

16. A particle of mass 0.3 kg is subjected to a force F = Kx with t nt


K = 15 N/m, what will be its acceleration if it is released from (1) (2)
a point x = 20 cm. (Delhi 2020-21) nc c
(1) 1 m/s2 (2) 10 m/s2 n2t t
(3) 100 m/s2 (4) 0.1 m/s2 (3) (4)
c n2 c
17. An object is moving in a straight line. The velocity time 23. The resistance of rheostat shown in the figure is 0 - 30 ,
graph is as shown in figure. Then (Delhi 2020-21) neglecting the resistance of ammeter and connecting wire
B the minimum and maximum currents through the ammeter
will be (Delhi 2020-21)

velocity A 2.2v

O A
time 10
(1) In part OA acceleration is increasing constant
(2) In part AB acceleration is increasing 30
20
(3) In part OA acceleration is decreasing
(4) In part AB acceleration is decreasing (1) (0.08 A, 0.33 A) (2) (0.06 A, 0.08 A)
18. A force of 100 N acts on a body so that the body acquire a (3) (0.06 A, 0.33 A) (4) (0.33 A, 0.09 A)
velocity of 10 m/s after some time. Now the force of 100 N is 24. Three particles A, B and C are thrown from top of a building
replaced by another force F which decelerates the body and with same speed. A is thrown upwards, B is thrown
body come to the rest then. (Delhi 2020-21) downwards and C is thrown horizontally, they hit the ground
(1) F > 100 N (2) F < 100 N with speed VA, VB and VC respectively then
(3) F = 100 N (4) All options are possible (Delhi 2020-21)
19. 2 points A and B are at electric potentials 10 V and 100 V (1) VA VB VC (2) VB VC VA
respectively. A charge q is taken from A to B and 18 Joule of
work is done. The value of q is (Delhi 2020-21) (3) VA VB VC (4) VA VB VC
(1) 2 Coulomb (2) 0.2 Coulomb 25. An object of height 2.0 cm is placed on the principal axis of
(3) 20 Coulomb (4) 0.02 Coulomb a concave mirror at a distance of 12 cm from the pole. If the
image is inverted, real and 5 cm in height then location of
20. Which of the following is NOT correct for magnetic field
the image and focal length of the mirror respectively are
lines? (Delhi 2020-21)
(Delhi 2020-21)
(1) The direction of magnetic field lines outside the magnet
is from north pole to south pole. (1) (–30 cm, +8.6 cm) (2) (–30 cm, –8.6 cm)
(2) The direction of magnetic field lines inside the magnet (3) (+30 cm, +8.6 cm) (4) (+30 cm, –8.6 cm)
is from south pole to north pole. 26. Luggage porters place a round piece of cloth on their head
(3) The degree of closeness of magnetic field lines tells when they have to carry heavy loads. By doing this, the
the relatives strength of magnetic field. area of contact between the load and head :
(4) Magnetic field lines never from closed loop. (Karnataka 2020-21)
21. A car moving along straight line covers 1/5th of total distance (1) Decreases and reduces the pressure
with speed v1 and remaining part of distance with speed v2. (2) Increases and reduces the pressure
The average speed of car over entire distance is (3) Decreases and increases the pressure
(Delhi 2020-21) (4) Increases as well as increases the pressure
27. A boy covers a distance of 260 m in 20 s and another distance
5v1v2 4v1v2
(1) (2) of 300 m in 20 s. The average speed of the boy is :
v2 4v1 5v1 v2 (Karnataka 2020-21)
–1 –1
5v1v2 4v1v2 (1) 14 ms (2) 28 ms
–1 –1
(3) (4) (3) 42 ms (4) 56 ms
4v2 v1 4v1 v2
28. In the following, the phenomena which were successfully
22. Light travels through a glass slab of thickness t and having explained by the law of gravitation are :
refractive index n. If c is the velocity of light in vacuum then (Karnataka 2020-21)
the time taken by light to travel this thickness of glass is
a. Motion of moon around the earth
(Delhi 2020-21) b. Formation of tides
Solved Questions B-3
c. Motion of an aeroplane (1) Atomic number of 'c' is 8. So it is inert gas.
d. Force that binds us to the earth (2) 'a' is the schematic atomic structure of oxygen.
e. Rotation of blades in a ceiling fan (3) 'b' has 8 electrons and it represents octet configuration.
(1) b, c, d (2) d, e, a (4) 'd' is the schematic atomic structure of sodium.
(3) c, d, e (4) a, b, d 34. The bulb is not glowing in this experimental setup. Because
29. The image formation in a convex lens is shown in the following
(Karnataka 2020-21)
figure. (Karnataka 2020-21)
A
6V
C F2 B' Bulb
B F1 O 2F2 Switch

20 cm 20 cm A'
AB is the object and A'B' is the image. The focal length of
the lens is
(1) 40 cm (2) 10 cm
(3) 20 cm (4) 05 cm
30. The image formation in human eye is shown in the given Electrode
diagram. (Karnataka 2020-21) Alcohol solution
O - Object
I - Image
(1) The circuit is open
O I (2) The battery is not connected properly
(3) The electrodes are too short
(4) Alcohol does not conduct electricity
35. A ray of light follows the path as shown in figure as it travels
The lens used to correct this type of eye defect is
through different media. Choose the correct relation
(1) Plano concave lens (2) Concave lens
regarding refractive indices from the given alternatives.
(3) Cylindrical lens (4) Convex lens
(Maharashtra 2020-21)
31. The lenses of power +3.5D, +2.5D and +1D are placed in
contact with each other in an optical device. The effective 40°
power of combination of these lenses is : Medium (1) 1
(Karnataka 2020-21)
(1) 5 D (2) 6 D Medium (2) 50° 2
(3) 7 D (4) 8 D 30°
32. Read the following and select the correct option. Medium (3) 3

(Karnataka 2020-21) Medium (4) 4


Statement (a) : When we pour little petrol on our palm, it
45°
causes the palm to feel cool. Medium (5) 5

Reason (b) : Particles of petrol evaporate using the energy


(1) 1 3 4 5
from our palm which causes the palm to feel cool.
(1) 'a' is correct and 'b' is not the correct reason. (2) 1 3 4 5
(2) 'a' is correct and 'b' is the correct reason.
(3) Both 'a' and 'b' are correct but 'b' is not the reason for 'a' (3) 1 3 4 5
(4) 'a' is not correct so 'b' is not the correct reason (4) 1 3 4 5
33. Schematic atomic structure of four elements are given below.
Observe and choose the right statement. 36. The main objective of ______ satellite launched by COEP
(Karnataka 2020-21) (College of Engineering, Pune) orbiting at a height of
_________ is to provide point-to-point messaging services.
(Maharashtra 2020-21)
(1) Samarpan, 550 km (2) Sampoornam, 540 km
(3) Swayam, 515 km (4) Sayam, 500 km
a b c d
EBD_7332
4
B- NTSE Stage 1 Question Bank

37. Observe the columns I and II, match them and select the 42. If force, momentum and displacement are represented by A,
correct alternative form the given options. AC
(Maharashtra 2020-21) B and C respectively, then the term will represent:
B
Column I Column II (Rajasthan 2020-21)
A. Motion of earth around (i) Nuclear force (1) momentum (2) acceleration
sun (3) velocity (4) displacement
B. Motion of stone tied to (ii) Electromagnetic force 43. The mass of a person on earth surface is 60 kg then his mass
on moon will be (Rajasthan 2020-21)
a string and whirled in
(1) 60 kg (2) 360 kg
a circle (3) 20 kg (4) 10 kg
C. Motion of electron (iii) Gravitational force 44. On halved th e distance between two masses, the
around nucleus gravitational force between them will be (Rajasthan 2020-
21)
D. Motion of blades of fan (iv) Tension
(1) half (2) one-fourth
(1) (A)-(i), (B)-(ii), (C)-(iv), D-(iii)
(3) four times (4) double
(2) (A)-(ii), (B)-(iii), (C)-(i), D-(iv)
45. If the speed of wave is 250 m/s and its wavelength is 50 cm
(3) (A)-(iv), (B)-(i), (C)-(iii), D-(ii) then the frequency will be (Rajasthan 2020-21)
(4) (A)-(iii), (B)-(iv), (C)-(i), D-(ii) (1) 5 Hz (2) 500 Hz
38. If mass of a planet is 25 times mass of earth and radius of the (3) 50 Hz (4) 12500 Hz
planet is 125 times radius of earth then escape velocity of an 46. An object of mass 10 gm is moving with an acceleration of
object from the plant (VP) is _______ times the escape 10 m/s2. Force acting on the object will be
velocity from earth (VE). (Maharashtra 2020-21)
(Rajasthan 2020-21)
1 (1) 1 N (2) 0.1 N
(1) (2) 5
5 (3) 1000 N (4) 100 N
1 47. Lens formula is (Rajasthan 2020-21)
(3) (4) 5
5 1 1 1 1 1 1
39. If approximate refractive index of sapphire is 1.8, then (1) (2)
v u f v 2u f
approximate decrease in velocity of light, when light enters
sapphire is ________ (Maharashtra 2020-21) 1 1 1 1 1 1
(1) 40% (2) 45% (3) v u f
(4) v 2u f
(3) 50% (4) 55% 48. Focal length of a lens is 50 cm. In dioptre, power of lens will
40. Group of vector quantities are : (Rajasthan 2020-21) be (Rajasthan 2020-21)
(1) displacement, velocity, time (1) 0.02 D (2) 2 D
(2) area, density, mass (3) 0.2 D (4) 50 D
(3) speed, length, impulse 49. Correct relation between radius of curvature (R) and focal
(4) velocity, acceleration, force length (F) of spherical mirror is (Rajasthan 2020-21)
41. The velocity-time graph of an object moving with uniform F
velocity is (Rajasthan 2020-21) (1) R (2) R = F
2
y y (3) R = 2F (4) R
50. Refraction from denser to rarer medium for a light ray, the
value of angle of refraction at the condition of critical angle
Velocity

Velocity

is (Rajasthan 2020-21)
(1) (2) (1) 0° (2) 180°
x x (3) 45° (4) 90°
Time Time 51. The resistance of a bulb marked 220V, 10W is
y y (Rajasthan 2020-21)
(1) 242 (2) 4840
(3) 121 (4) Zero
52. A person of mass 100 kg reaches a height of 5 meters in 10
Velocity

Velocity

(3) (4) seconds .............. power used by the person (g = 10 m/s2)


x x (Rajasthan 2020-21)
Time Time (1) 500 watt (2) 250 watt
(3) 5000 watt (4) 50 watt
Solved Questions B-5

R R (1) +2D (2) –2D


53. , where R , R – Resistance, T, T 0 – (3) +5D (4) –5D
R (T T0 )
62. In an electric circuit, the voltmeter is used :
Temperature, then the unit of is : (Tamil Nadu 2020-21)
(Uttar Pradesh 2020-21)
(1) / C (2) C
(1) in series (2) in parallel
(3) / C (4) C /
(3) in both manner (4) None of these
54. A girl of mass 40 kg stands on her feet of surface area 80 cm2
(g = 10 m/s2). The pressure exerted by her feet is ? 63. One horse power (H.P.) is equal to:
(Tamil Nadu 2020-21) (Uttar Pradesh 2020-21)
3
(1) 5 × 10 Pa (2) 5 × 104 Pa (1) 467 watt (2) 500 watt
(3) 0.5 Pa (4) 2 Pa (3) 746 watt (4) 1000 watt
55. If the EMF of a cell is 1.5 V, then the energy provided by the 64. A magnet attracts : (Uttar Pradesh 2020-21)
cell to drive 0.5 C of charge around the circuit : (1) only iron (2) only cobalt
(Tamil Nadu 2020-21)
(3) only nickel (4) All the above
(1) 1 J (2) 0.75 J
65. The mass, linear momentum and kinetic energy of a body
(3) 0.5 J (4) 0.25 J
are m, p and E respectively, then (West Bengal 2020-21)
56. North
(1) p 2mE (2) E 2mp
5N
3N (3) p 2E (4) E 2p
East 66. A stone is allowed to fall freely downwards initially at rest
West 7N
4N from the top of a tower. The time taken by the stone to reach
the bottom of the tower is 4 seconds. What is the height of
4N the tower/ Take, acceleration due to gravity = 32 ft/s2
South (West Bengal 2020-21)
In which direction does the net force act? (Tamil Nadu 2020-21) (1) 64 ft. (2) 32 ft.
(1) North (2) South (3) 48 ft. (4) 256 ft.
(3) East (4) West 67. The coefficient of linear expansion of a solid is x and the
57. Pick out the correct statements regarding the properties of coefficient of volume expansion of the solid is y, then
Magnetic Lines of force. (Tamil Nadu 2020-21) (West Bengal 2020-21)
(a) Magnetic Lines of force never intersect.
y x
(b) They will be maximum at the equator than at the poles. (1) x (2) y
3 3
(c) Magnetic Lines of force start from South pole and end
at the North pole. y x
(3) x (4) y
(d) Magnetic Lines of force are closed continuous curves 2 2
extending to the body of the magnet. 68. In case of refraction of light from a medium to air the critical
(1) (a), (b) and (c) only (2) (a), (b) and (d) only angle is found to be 45°. What is the refractive index of the
(3) (a) and (d) only (4) (b) and (c) only medium with respect to air ? (West Bengal 2020-21)
58. Pick out the right answer to correct the given statement. (1) (2)
2 3
The distance travelled by sound in one second is called
time period of the sound. (Tamil Nadu 2020-21) (3) 2 (4) 3
(1) Loudness (2) Pitch 69. Which of the following pair have same unit?
(3) Velocity (4) Frequency (West Bengal 2020-21)
59. What is the far point for normal human eyes? (1) Heat and Specific heat
(Uttar Pradesh 2020-21) (2) Thermal capacity and water equivalent
(1) 25 cm (2) 50 cm (3) Specific heat and Thermal capacity
(3) 100 cm (4) Infinity (4) Heat and Work
60. Refractive index of water is : (Uttar Pradesh 2020-21) 70. Which of the following is an electromagnetic wave?
(1) 1.00 (2) 1.33 (West Bengal 2020-21)
(3) 1.52 (4) 2.42 (1) -ray (2) -ray
61. A man used a convex lens of focal length of 20 cm in his (3) -ray (4) cathode ray
spects, the power of this lens is: (Uttar Pradesh 2020-21)
EBD_7332
6
B- NTSE Stage 1 Question Bank

71. In case of a convex lens, what is the minimum distance 4


3
between an object and its real image ? 80. A ray of light passes from glass to water .
2 3
(West Bengal 2020-21)
The value of critical angle will be (Uttar Pradesh 2019)
(1) 2.5 times of focal length (2) 2 times of focal length
(3) 4 times of focal length (4) equal to focal length 1 1 1 8
(1) sin (2) sin
72. What will be the power consumed by a 50 wire if it is kept 2 9
across a potential difference of 200 V?
(West Bengal 2020-21) 1 8 1 5
(3) sin (4) sin
(1) 0.8 kW (2) 80 kW 9 7
(3) 400 W (4) 8 kW 81. The value of acceleration due to gravity (g) on the earth
will be maximum at- (Uttar Pradesh 2019)
73. A person cannot clearly see object at a distance more than
40 cm. He is advised to use lens of power (Delhi 2019) (1) Surface (2) Poles
(1) –2.5 D (2) 2.5 D (3) Equator (4) Center
(3) –1.5 D (4) 1.5 D 82. Which one of the following is an example of Biomass
Energy source? (Uttar Pradesh 2019)
74. Gravitational force is essentially required for (Delhi 2019)
(1) Nuclear Energy (2) Sun Energy
(1) Stirring in liquid (2) Convection
(3) Gobar Gas (4) Wind Energy
(3) Conduction (4) Radiation
83. The circuit shown has 5 resistors of equal resistance R.
75. If the ammeter in the given circuit reads 2 A, What is the Calculate equivalent resistance across points A and B
value of resistence R (the resistance of ammeter is
negligible). (Delhi 2019) (Bihar 2019)
3 A
R

6
B
11 13R
(1) (2)
6V R 12
A R 15R
(3) (4)
5 6
(1) 1 (2) 2
84. For an object thrown at 45° to horizontal, the maximum
(3) 3 (4) 4 height (H) and horizontal range (R) related as :
76. Two planets of radii r1 and r2 are made from the same (Bihar 2019)
material having same density. The ratio of acceleration due
(1) R = 16 H (2) R = 8 H
to gravity g1|g2 at the surfaces of the planets is
(3) R = 4 H (4) R = 2 H
(Delhi 2019)
85. A simple pendulum has a time period T1 when on the earth’s
(1) r1|r2 (2) r2|r1
surface and T2 when taken to a height R above earth’s
(3) (r1|r2)2 (4) (r2|r1)2 surface, where R is the radius of the earth.
77. An electron move with velocity v in a uniform magnetic
field B. The magnetic force experienced by the electron is T2
The value of ratio (Bihar 2019)
(Delhi 2019) T1
(1) Always zero (1) 1 : 1 (2) 2 :1
(2) Never zero (3) 4 : 1 (4) 2 : 1
(3) Zero if v is perpendicular to B 86. Under the influence of a uniform magnetic field a charged
(4) Zero if v is parallel to B particle is moving in a circle of radius R with constant
78. Unit of momentum is (Uttar Pradesh 2019) speed v. The time period of the motion : (Bihar 2019)
(1) meter/second (2) Newton × meter (1) Depends on both R and v
(3) kg-meter/second (4) kg-1 meter second (2) Is independent of both R and v
79. Which one of the following physical quantity is constant
in simple harmonic motion? (Uttar Pradesh 2019) (3) Depends on R but not on v
(1) Restoring force (2) Kinetic energy (4) Depends on v but not on R
(3) Potential energy (4) Total energy
Solved Questions 7
B-

87. A machine gun fires a bullet of mass 40 gram at a speed of 94. A house hold consumes 1 kWh of energy per day. This
1200 ms–1. The man holding it can exert a maximum forces energy in joules is (Karnataka 2019)
of 144 N on the gun. How many bullets can he fire per (1) 3.6 × 105 J (2) 6.3 × 105 J
second at the most? (Bihar 2019)
(3) 6.3 × 106 J (4) 3.6 × 106 J
(1) One (2) Four
(3) Two (4) Three 95. Resistors are connected as shown in the circuit diagram.
The effective resistance between A and B is
88. Bulb ‘P’ marked as 100 W, 220 V and bulb Q marked as 60 W,
110 V. The resistance ratio of P and Q is (Andhra Pradesh) (Karnataka 2019)
(1) 12 : 7 (2) 5 : 7
4 3
(3) 5 : 12 (4) 12 : 5
89. Assertion (A): The velocity of a particle may vary even
when it’s speed is constant. (Andhra Pradesh)
Reason (R): The particle is moving in circular path. A 3 B
(1) (1) is true, but (R) is false.
(2) Both (A) and (R) are true, but (R) is not correct
explanation to (A).
(3) (1) is false, but (R) is true. 4
3
(4) Both (A) and (R) are true and (R) is correct explanation
to (A). (1) 3 (2) 3.5
90. A convex lens of focal length 20 cm is cut into two halves. (3) 11 (4) 5.5
Each of which is placed 0.5 mm and a point object placed
at a distance of 30 cm from the lens as shown. 96. Two steel balls of mass 1 kg and 2 kgs and a lead ball of 10
kgs are released together from the top of tower 30 m high.
(Andhra Pradesh) Assuming the path to be in vacuum: (Karnataka 2019)
(1) The lead ball reaches the ground earlier
(2) The 1 kg iron ball reaches the ground earlier
(3) All the balls reach the ground simultaneously
(4) The 2 kgs steel ball reaches the ground earlier
97. The maximum number of 60 W bulbs that can be run from
Then the image is at the main supply of 220 V if you do not want to overload a
(1) 60 cm (2) 30 cm 5 A fuse: (Karnataka 2019)
(3) 70 cm (4) 50 cm (1) 18 (2) 11
91. Identify the following colours in the ascending orders of (3) 20 (4) 10
their frequencies. (Andhra Pradesh) 98. The diagram shows a current carrying wire passing through
(1) Red, blue, yellow, green the centre of a square cardboard. The magnetic field at
(2) Blue, green, yellow, red points A , B and C is: (Karnataka 2019)
(3) Red, green, yellow, blue
(4) Red, yellow, green, blue C
92. Which of the graph represents non-uniform acceleration?
(Andhra Pradesh)

B
Speed

Speed

(1) (2)
A

Time
Time
(1) Equal at A, B and C
(2) Stronger at B than A, equal at B and C
Speed
Speed

(3) (4) (3) Stronger at B than C, weaker at B than A


(4) Stronger at B than A, weaker at C than A
99. A Tennis ball is thrown up and reaches a certain height
Time Time
and comes down in 8 s. If value of acceleration due to
93. A person fired a gun standing at a distance of 55 m from a gravity (g) = 10 m/s², then height reached by tennis ball
wall. If the speed of sound is 330 m/s, the time for an echo and velocity with which it strikes the ground respectively
heard is (Andhra Pradesh) is ______ and ______. (Maharashtra 2019)
(1) 0.4 s (2) 0.6 s (1) 640 m, 160 m/s (2) 320 m, 120 m/s
(3) 0.5 s (4) 0.3 s (3) 160 m, 80 m/s (4) 80 m, 40 m/s
EBD_7332
8
B- NTSE Stage 1 Question Bank

100. For a colour blind person choose the incorrect statement (1) 4 m/sec2 (2) 6 m/ sec2
from the following: (Maharashtra 2019) 2
(3) 8 m/sec (4) 9 m/sec2
(1) rod cells are present on retina
112. The spectacular glow of diamond is due to:
(2) cone cells are present on retina
(Tamil Nadu 2019)
(3) Eye sight of person is normal
(1) Refraction (2) Reflection
(4) Proper information about intensity of light of object
is given to brain. (3) Total Internal Reflection (4) Scattering of Light
101. Unit of gravitational potential energy _____ 113. If a current of 5 A flows through the heater and the amount
(Maharashtra 2019) of heat produced is 54000 J in 6 minutes, then find the
resistance of the electric heater. (Tamil Nadu 2019)
(1) J/s (2) Js
(1) 6 (2) 5
(3) Nm (4) N/m
(3) 7 (4) 4
102. A ray of light is incident on the surface of transparent
medium at an angle of 45º and is refracted in the medium 114. The only moon in the solar system that moves in the opposite
at an angle of 30º. What will be the velocity of light in the direction to the direction in which its planet spins?
transparent medium ? (Maharashtra 2019) (Tamil Nadu 2019)
(1) 1.96 × 108 m/s (2) 2.12 × 108 m/s (1) Sputnik (2) Titan
(3) 2.65 × 108 m/s (4) 1.25 × 108 m/s (3) Ganymede (4) Triton
103. 250 kg of water per minute is to be drawn from a well 115. The reason for using red light in traffic signals to stop
150 m deep. An electric pump of __________ can be used. vehicles. (Tamil Nadu 2019)
(g = 10 m/s2) (Maharashtra 2019) (1) Red light has shorter wavelength
(1) 6 horse power (2) 7 horse power
(2) Red light has longer wavelength
(3) 8 horse power (4) 9 horse power
(3) Red light is very bright and attractive
104. ________ metal generally occurs in free state.
(4) Red light has highest angle of refraction
(Maharashtra 2019)
116. The distance-time graph of a particle at time t makes an
(1) Sodium (2) Platinum
angle 45° with the time axis. After 1 s it makes an angle 60°
(3) Magnesium (4) Potassium with the time axis. What is the average acceleration of the
105. The initial velocity of a particle is 10 m/s. It is moving with particle during this time interval ? (West Bengal 2019)
an acceleration of 4m/s2. The distance covered by the
particle after 2s is (Rajasthan 2019) (1) 3 1 unit (2) 3 1 unit
(1) 6 m (2) 18 m
(3) 22 m (4) 28 m (3) 3unit (4) 1 unit
106. If the speed of wave is 350 m/s and its wavelength is 100 117. Two blocks of mass 4 kg and 2 kg are placed side by side
cm, then the frequency of the wave will be on a smooth horizontal table and a horizontal force of 20
(Rajasthan 2019) N is applied on the 4 kg block as shown in the figure. The
(1) 35 Hz (2) 350 Hz normal reaction between the two blocks will be
(3) 700 Hz (4) 3500 Hz (West Bengal 2019)
107. The wave having compression and refraction is known as m1
(Rajasthan 2019) m2
(1) Transverse wave (2) Longitudinal wave 20 N 4 kg
2 kg
(3) Light wave (4) Ultraviolet wave
108. Focal length of a lens is 25 cm. In dioptre, power of lens
will be (Rajasthan 2019) (1) 10/3 N (2) 20/3 N
(1) 0.04 (2) 0.4 (3) 25/3 N (4) 40/3 N
(3) 4 (4) 2.5 118. All other conditions remaining same, if the velocity of
109. Unit of magnetic flux is (Rajasthan 2019) sound in oxygen and hydrogen gases are given by VO
(1) volt (2) weber and VH respectively, then which one of the following is
(3) hertz (4) ohm-metre correct ? (West Bengal 2019)
110. Lactometer is an instrument which works on the principle (1) VH = 2VO (2) VH = 4VO
of ? (Tamil Nadu 2019) (3) VH = VO (4) VO = 4VH
(1) Law of Floatation (2) Newton’s Law 119. All other conditions remaining same, if the temperature of
(3) Ohm’s Law (4) Avogadro’s Law a gas medium drops by 1%, the velocity of sound in that
111. A 250 kg bike is ridden by a circus man at a speed of 20 m/ medium will (West Bengal 2019)
sec, in a circular path of diameter 100 m. (1) Increase by 0.5% (2) Remain unchanged
Calculate its acceleration. (Tamil Nadu 2019) (3) Decrease by 0.5% (4) Decrease by 2%
Solved Questions 9
B-

120. A beam of light is incident at 60° to a plane separating two 127. The mass of a planet is twice and its radius is three times
medium. The reflected and refracted rays are found to be that of the earth. The weight of a body, which has a mass of
perpendicular to each other. What is the refractive index 5 kg, on that planet will be (Delhi 2018)
of the second medium with respect to the first medium ? (1) 11.95 N (2) 10.88 N
(West Bengal 2019) (3) 9.88 N (4) 20.99 N
1 128. The inertia of a body depends upon (Rajasthan 2018)
(1) (2) 1/3
3 (1) Gravitational acceleration
(3) 3 (4) 3 (2) Centre of gravity of body
(3) Shape of body
121. A boy starts from rest is accelerated uniformly for 30s. If x1, (4) Mass of body
x2, x3 are the distance travelled in first 10s, next 10s and last 129. Velocity-time graph of a body moving with uniform
10s respecrively, then : x1 : x2 : x3 is (Delhi 2018) acceleration is shown in the diagram. The distance travelled
(1) 1 : 2 : 3 (2) 1 : 1 : 1 by the body in 3 seconds is (Rajasthan 2018)
(3) 1 : 3 : 5 (4) 1 : 3 : 9
122. A bomb of mass 3m kg explodes into two pieces of mass

velocity(m/s)
m kg and 2 m kg. If the velocity of m kg mass is 16 ms–1, the 30
total kinetic energy in the explosion is (Delhi 2018)
20
(1) 192 mJ (2) 96 mJ
(3) 384 mJ (4) 768 mJ 10
123. Figure shows a ray of light as travels from medium 1 to Time(s)
medium 2. If reflective index of medium 1 with respect to 0 1 2 3
2 (1) 90 m (2) 45 m
medium 2 is then the value of angle x is (Delhi 2018) (3) zero (4) 10 m
3
130. The distance between two masses is to be halved. The
Medium 2 gravitational force between them will be-(Rajasthan 2018)
45° (1) Double (2) One-fourth
(3) Quadruple (4) Half
45°
131. The speed of a waves is 350 m/s and wavelength is 70 cm.
The frequency of wave is (Rajasthan 2018)
(1) 500 Hz (2) 700 Hz
x=? (3) 50 Hz (4) 200 Hz
Medium 1 132. Which one of the following is not a part of direct current
generator? (Rajasthan 2018)
(1) 30° (2) 60° (1) Commutator (2) Sliprings
(3) 15° (4) 45° (3) Armature (4) Carbon brushes
133. The equivalent resistance of the given circuit between points
124. Which of the following statements is true? (Delhi 2018)
A and B is (Rajasthan 2018)
(1) A convex lens with power + 4D has a focal length
– 0.25 m 10
(2) A convex lens with power –4D has a focal length A
+ 0.25 m
10
(3) A concave lens with power + 4D has a focal 10 10
length –0.25 m
(4) A concave lens with power – 4D has a focal length
– 0.25 m 10 B
125. If the current through a resistor is increased by 50%, the
increase in power dissipated will be (assume the temperature (1) 40 (2) 4
remains constant) (Delhi 2018) (3) 5 (4) 0.2
(1) 225% (2) 200% 134. A solid cube of silver has a mass of 84 g. What is the
(3) 250% (4) 125% resistance between the opposite faces, given that density
126. The linear distance between a consecutive compression and of silver is 10.5 g/cm3 and resistivity is 1.6 × 10–4 cm?
a rarefaction in longitudinal wave is (Delhi 2018) (Chandigarh 2018)
(1) 0.4 × 10–4 (2) 0.8 × 10–4
(1) (2) (3) 0.4 × 10–8 (4) 0.8 × 10–8
2
3
(3) (4)
4 4
EBD_7332
10
B- NTSE Stage 1 Question Bank

135. Farsighted people, who have lost their spectacles, can still 142. Conductivity of superconductors is :
read a book by looking through a small (3 - 4 mm) hole in a (Madhya Pradesh 2018)
sheet of a panel because (Chandigarh 2018) (1) infinite (2) very large
(1) the fine hole produces an image of the letters at a longer (3) very Small (4) zero
distance
143. How many time does a ray bend on passing through a prism?
(2) in doing so, the distance of the object is increased.
(Madhya Pradesh 2018)
(3) in doing so, the focal length of the eye lens is
(1) Once (2) Twice
effectively decreased.
(3) Thrice (4) None
(4) in doing so, the focal length of trie eye-lens is
effectively increased. 144. Which is the colour at lower end of visible spectrum?
136. A concave mirror is placed on a table with its pole touching (Madhya Pradesh 2018)
the table. The mirror is rotated about its principal axis in (1) Red (2) Green
clockwise direction. The image of a person looking straight (3) Yellow (4) Violet
into it (Chandigarh 2018)
145. A wire of resistance 12 ohm is bent in the form of a circular
(1) rotates in clockwise direction ring. The effective resistance between the two points on
(2) rotates in anti-clockwise direction any diameter of the circle is (Karnataka 2018)
(3) is inverted (1) 24 (2) 12
(4) does not rotate
(3) 6 (4) 3
137. “Metal dishes” (Dish Antennas) are used for receiving TV
signals from distant communication satellites. These ‘Metal 146. Sheela cannot read newspaper when she holds it closer than
Dishes’ are (Chandigarh 2018) 100 cm. The defect in her eye and the power of lens pre-
(1) Convex Reflectors scribed to her (Normal eye near point = 25 cm)
(2) both convex and concave reflectors (Karnataka 2018)
(3) Concave reflector (1) Myopia with + 3D lens
(4) Convex refractors
(2) Myopia with – 3D lens
138. If a symmetrical convex lens of focal length ‘f’ is cut into
two parts along the principal axis as shown in the figure, the (3) Hypermetropia with – 3D lens
focal length of each part will be (Chandigarh 2018) (4) Hypermetropia with + 3D lens
147. An athlete completes one round of a circular track of radius
R in 40 seconds. The displacement at the end of 2 minutes
20 seconds will be (Karnataka 2018)
f’
(1) Zero (2) 2R
(3) R (4) 7 R
148. For a nuclear reactor 48 KJ of energy is produced per minute.
If the energy released per fission is 3.2 × 10–11 J then the
(1) f/2 (2) f/4 number of fissions which would be taking place in a reactor
per second is (Karnataka 2018)
(3) f (4)
139. What is speed of light in glass? (Gujarat 2018) (1) 5 × 1014 (2) 2 × 1014
8 –1 (3) 5.2 ×1013 (4) 2.5 ×1013
(1) 2 × 10 ms (2) 2.25 × 10 ms–1
8
8
(3) 3 × 10 ms –1 (4) 1.75 × 108 ms–1 149. The speed of sound in air at NTP is 332 m/s. If air pressure
becomes four times the normal then the speed of sound
140. When a milky and cloudy layer is formed on the eye lens of
old age person, they lose their vision partially or completely. waves will (Karnataka 2018)
This type of situation is called _______. (Gujarat 2018) (1) Double
(1) myopia (2) cataract (2) Quadruple
(3) hypermetropia (4) presbyopia (3) Remain the same
141. Determine the equivalent resistance between points x and y 1
in the following circuit. (Gujarat 2018) (4) become of the original value
4
3
150. How much time the satellite will take to complete one revo-
3 lution around the earth, if velocity of satellite is 3.14 km/s
and its height above earth’s surface is 3600 km (Radius of
3 3 earth is 6400 km)? (Maharashtra 2018)
X (1) 2000 S (2) 20000 S
(3) 1000 S (4) 10000 S
Y
(1) 5 (2) 12
(3) 9 (4) 6
Solved Questions 11
B-

151. Device/device changing electrical energy into mechanical 160. The mass of earth is 80 times that of moon and its diameter
energy is/are ........ (Maharashtra 2018) is double that of moon. If the value of acceleration due to
gravity on earth is 9.8 ms–2 then the value of acceleration
I. Electrical generator II. Electric motor
due to gravity on moon will be (Delhi 2017)
III. Voltmeter IV. Ammeter (1) 0.98 ms–2 (2) 0.49 ms–2
(1) I and II (1) II and III (3) 9.8 ms –2 (4) 4.9 ms –2
161. Two lenses of focal length f1 and f2 are kept in contact coaxially.
(3) II, III and IV (4) Only II The power of the combination will be (Delhi 2017)
152. If V1 and V2 are the volume of one gm water at 0°C and 4°C f1f 2 f1 f 2
respectively, then- (Uttar Pradesh 2018) (1) (2)
(1) V1 > V2 (2) V1 = V2
f1 f 2 f1f 2

(3) V1 < V2 (4) V1 V2 f1f 2


(3) (4) f1 + f2
153. The displacement time graph of a body in motion is given as f1 f 2
below- (Haryana 2018) 162. In figure, a ray of light undergoes refraction from medium A
to medium B. If the speed of light in medium A is then the
speed of light in medium B will be (Delhi 2017)

N
Time (s)

X 60o Y
Displacement (m) 30o
Velocity of body is (in m/s)-
N1
1
(1) 3 (2)
3
(1) 3 (2)
1 3
(3) 3 (4)
3
154. If a body is in equilibrium under the effect of some collinear (3) 2 (4)
forces, then the minimum number of such forces acting upon 2
the body are (Delhi 2017) 163. A body falls freely from a tower and travels a distance of 40
(1) 3 (2) 2 m in its last two seconds. The height of the tower is
(3) 5 (4) 4 (Delhi 2017)
155. A heater coil is cut into two equal parts and only one part is (1) 54 m (2) 45 m
used in the heater, the heat generated now will be (3) 80 m (4) 65 m
(Delhi 2017) 164. Two bodies of masses ma and mb are dropped from different
(1) doubled (2) four times heights ‘a’ and ‘b’. The ratio of time taken by them to reach
(3) one fourth (4) halved the ground is: (Haryana 2017)
156. A bar magnet placed in non-uniform magnetic field (1) a: b (2) a : b
experiences (Delhi 2017)
1 1
(1) only torque (2) only force (3) : (4) ma : mb
(3) both torque and force (4) neither force nor torque a b
157. How much water a pump of 2 kW power can raise in one 165. A person throws ball with a velocity ‘v’ from top of a building
minute to a height of 10 m? (g = 10 m/s2) (Delhi 2017) in vertically upward direction. The ball reaches the ground
(1) 1000 litre (2) 1200 litre with a speed of ‘3v’. The height of the building is:
(3) 10 litre (4) 2000 litre (Haryana 2017)
158. The Kinetic energy of a body becomes 4 times of its initial 4v 2 3v 2
value. The new linear momentum will be (Delhi 2017) (1) (2)
(1) same as initial momentum g g
(2) four times the initial momentum
6v 2 9v2
(3) two times the initial momentum (3) (4)
(4) eight times the initial momutum g g
159. In a simple pendulum mass of bob is m and effecting length 166. A bottle full of water containing an air bubble is rotated in
is L. Work done on the pendulum in one complete oscillation horizontal circle by a string tied to the neck of the bottle.
in gravitational field of earth is (Delhi 2017) Then air bubble will: (Haryana 2017)
(1) be collected at bottom
1 1
(1) mgL (2) mgL (2) remain unaffected
4 2 (3) be collected at the wall of bottle
(3) zero (4) mgL (4) be collected at the neck
EBD_7332
12
B- NTSE Stage 1 Question Bank

167. Three voltmeters all having different resistances are joined


as shown. When some potential difference is applied across

Distance
A and B, then readings in voltmeter V1, V2 and V3 are :

Speed
(Haryana 2017) (1) (2)
V1 V2 Time Time

A B

Acceleration

Velocity
V3 (3) (4)

(1) V1 = V2 (2) V2 < V2 Time Time


(3) V1 + V2 = V3 (3) V1 + V2 > V3
168. What is potential difference across AB?

Acceleration
A B
3A 1 6 (E)
3V
(Haryana 2017) Time
(1) 24 V (2) 0 V (1) D only (2) D and E
(3) 6 V (4) 18 V (3) A, B and C (4) A and D
169. Three equal resistors connected in series across a source of 175. A bullet of mass 50 gm is horizontally fired with a velocity
emf, dissipate 10 watt. If the same resistors are connected in 100 ms –1 from a gun of mass 10 kg. What will be the recoil
parallel across the same emf, the power dissipated will be: velocity of the gun? (Rajasthan 2017)
(Haryana 2017) (1) 100 ms –1 (2) 500 ms –1
(1) 10 watt (2) 30 watt (3) 0.5 ms –1 (4) Zero
10 176. A ball is shot vertically upward with a given initial velocity.
(4) watt (4) 90 watt It reaches a maximum height of 100 m. If on a second shot,
3
170. A long wire carries a steady current, is bent into a circle of the initial velocity is doubled then the ball will reach a
one turn and magnetic field at the centre of coil is B. When maximum height of (Rajasthan 2017)
it is bent into n-turns, magnetic field at centre of coil will be: (1) 70.7 m (2) 141.4 m
(Haryana 2017) (3) 200 m (4) 400 m
(1) 2n2B (2) 2nB 177. Let M denotes the mass of earth and let R denotes its radius.
(3) n2B (4) nB The ratio g/G at earth’s surface is (Rajasthan 2017)
171. If ‘p’ and ‘q’ are distance of object and image from principal (1) R2/m (2) M/R2
focus ‘f’ of a concave mirror then what is the relation (3) M/R (4) R/M
between ‘p’, ‘q’ and ‘f’? (Haryana 2017) 178. The unit ‘hertz’ is same as (Rajasthan 2017)
(1) second (2) second–1
(1) pq f (2) pq = f
(3) metre (4) metre–1
1 179. A sound wave has a frequency of 10 kHz and wavelength 3
(3) pq = f 2 (4) pq mm. How much time will it take to travel 3 metre?
f
(Rajasthan 2017)
172. When the object is at distances u1 and u2 from a lens, a real (1) 0.1 sec (2) 1 sec
and virtual images are formed respectively having the same (3) 10 sec (4) 0.01 sec
magnification. The focal length of lens is: (Haryana 2017) 180. The size of image formed by a concave mirror is same as the
u2 u1 – u 2 size of object. The position of the object will be
(1) u1 (2) (Rajasthan 2017)
2 2
(1) at F (2) between F and C
u1 u 2 (3) at C (4) between C and infinity
(3) (4) u1 + u2
2 181. A convex lens has focal length 30 cm. If an object is placed
173. A pump motor is used to deliver water at a certain rate from at a distance of 15 cm from it then the magnification produced
a given pipe. To obtain twice as much water from the same by the lens is (Rajasthan 2017)
pipe in same time, power of motor has to be increased: (1) 6.66 (2) 0.5
(Haryana 2017) (3) 1 (4) 2
(1) 16 times (3) 4 times 182. The electrical resistivity of a conducting wire is K. If its
(3) 8 times (4) 2 times length and area of cross-section are doubled then the new
174. Consider the following five graphs (note the axes carefully). resistivity of the wire will be (Rajasthan 2017)
Which of the following represents motion at constant speed? (1) K (2) 2K
(Rajasthan 2017) (3) K/2 (4) K/4
Solved Questions 13
B-

183. What is the equivalent resistance of the given circuit 190. The total electrical resistance between the points A and B of
between points A and B? (Rajasthan 2017) the circuit shown in: (Madhya Pradesh 2017)
10 10 10
1 2
1 3 2 A 20 B
A B
1 2
20 20
(1) 10 (2) 4
(1) 9.23 (2) 15
14 17 (3) 30 (4) 100
(3) (4) 191. Two wires of same material have lengths L and 2L and cross-
3 6
sectional areas 4A and A respectively. The ratio of their
184. A car covers 30 km at a uniform speed of 60 km/hr. and the resistances would be: (Madhya Pradesh 2017)
next 30 km at a uniform speed of 40 km/hr. The total time (1) 1 : 1 (2) 1 : 8
taken is: (Madhya Pradesh 2017) (3) 8 : 1 (4) 1 : 2
(1) 30 min (2) 45 min 192. Wire of resistance R is streched to thrice of its original length,
(3) 75 min (4) 120 min what is its new resistance? (Madhya Pradesh 2017)
185. A stone is thrown upwards with a speed “u” from the top of R
a tower. It reaches the ground with a velocity “3u”. The (1) 9 R (2)
9
height of the tower is: (Madhya Pradesh 2017)
(3) 3R (4) R/3
u2 2u 2 193. Galaxy in which we live is: (Madhya Pradesh 2017)
(1) (2) (1) Milky way (2) radio galaxy
g g
(3) circular galaxy (4) irregular galaxy
194. Opposition of flow of electric current is called–
3u 2 4u 2 (Uttar Pradesh 2017)
(3) (4)
g g (1) Potential difference
186. When the speed of a particle is doubled, the ratio of its (2) Electric charge
(3) Resistance
kinetic energy to its momentum: (Madhya Pradesh 2017)
(4) Electromagnetic induction
(1) remains the same (2) gets doubled 195. Which of these does not require a medium–
(3) becomes half (4) becomes four times (Uttar Pradesh 2017)
187. Calculate the wavelength of radio waves of frequency 109 (1) Conduction (2) Convection
Hz. The speed of radio waves is 3 × 108 m/s. (3) Radiation (4) None of these
(Madhya Pradesh 2017) 196. Capacity of a measuring flask is 1 litre. What it will be in
(1) 60 cm (2) 40 cm cubic centimetre– (Uttar Pradesh 2017)
(3) 30 cm (4) 10 cm (1) 1 Cubic Centimetre (2) 10 Cubic Centimetre
188. A force of 16 N is distributed uniformly on one surface of a (3) 100 Cubic Centimetre (4) 1000 Cubic Centimetre
cube of edge 8 cm. The pressure on this surface is: 197. Noise is produced by– (Uttar Pradesh 2017)
(Madhya Pradesh 2017) (1) Vibration with high frequency
(1) 3500 Pa (2) 2500 Pa (2) Regular vibration
(3) Regular and periodic vibration
(3) 4500 Pa (4) 5500 Pa
(4) Irregular and non periodic vibration
189. In which of the following caes, is the work done maximum? 198. A ray passing through which part of a lens emerges
(Madhya Pradesh 2017) undeviated– (Uttar Pradesh 2017)
(1) Focus
F (2) Centre of curvature
F
(3) Optical centre
(1) (2) (4) between Focus and centre of curvature
30° 45°
S S 199. To convert temperature in ‘F into °C we use the formula–
(Uttar Pradesh 2017)
F C – 32 5
(1) (2) C [ F – 32]
F 100 180 9
F
(3) (4) 5 9
S F [C – 32] C [ F – 32]
(3) (4)
S 9 5
EBD_7332
14
B- NTSE Stage 1 Question Bank

200. A swimming pool appears less deeper than its real depth– 208. A bomb of mass 9 kg initially at rest explodes into two
(Uttar Pradesh 2017) pieces of masses 3 kg and 6 kg. If the kinetic energy of 3 kg
(1) due to reflection mass is 216 J then the velocity of 6 kg mass will be
(2) due to refraction (Delhi 2016)
(3) due to dispersion (1) 4 m/s (2) 3 m/s
(4) due to lateral displacement (3) 2 m/s (4) 6 m/s
201. A person is standing 4m away from plane mirror. Distance 209. Which of the following situation is not possible:
between mirror and image is– (Uttar Pradesh 2017) (Delhi 2015)
(1) 4 metre (2) 8 metre (1) An object moving in certain direction with an
(3) 2 metre (4) 6 metre acceleration in perpendicular direction.
202. According to law of floating weight of a floating body is– (2) An object moving with constant acceleration but zero
(Uttar Pradesh 2017) velocity.
(1) Equal to the weight of liquid displaced (3) An object moving with variable speed and uniform
(2) Equal to the volume of liquid displaced velocity.
(3) Is greater than the weight of liquid displaced (4) None of the above
(4) Is less than the weight of liquid displaced 210. A particle starts its motion from rest under the action of a
203. Two resistance each of 2 ohm are connected in series and in constant force. If the distance covered in first 10s is S1 and
parallel separately. Equivalent resistance is– that covered in first 20s is S2 then (Delhi 2015)
(Uttar Pradesh 2017) (1) S2=S1 (2) S2=2S1
(1) 4 , 1 (2) 1 , 4 (3) S2=3S1 (4) S2=4S1
211. The volume of a substance is 20 cm3. The mass of the
(3) 2 , 2 (4) 4 , 4
substances if its relative density is 2.5 will be (Delhi 2015)
204. The distance travelled by a body falling freely from rest in
(1) 5g (2) 50g
2nd, 3rd and 5th second of its motion are in the ratio
(3) 500g (4) 5 kg
(Delhi 2016) 212. Sound travels fastest in (Delhi 2015)
(1) 7 : 5 : 3 (2) 3 : 5 : 7
(1) Aluminium (2) Water
(3) 5 : 3 : 7 (4) 5 : 7 : 3
(3) Hydrogen (4) Oxygen
205. Two extremes ends of a moving train (engine and guard
213. A real and enlarged image can be formed by using a
coach) pass a pole speeds U and V respectively with a
(1) convex mirror (Jharkhand 2015)
constant acceleration. The speed with which the middle
(2) plane mirror
point of the train will pass the same pole is (Delhi 2016) (3) concave mirror
U V V2 U2 (4) either convex or a plane mirror
(1) (2) 214. How many planets are there in our solar system
2 2
(1) 5 (2) 7 (Jharkhand 2015)
UV U2 V2 (3) 9 (4) 8
(3) (4) 215. In house electrical circuits the fuse wire for safety should be
2 2 of (Jharkhand 2015)
206. An athlete completes one round of circular track of radius r (1) High resistance – high melting point
in 30s with uniform speed. The ratio of distance to the (2) Low resistance – high melting point
displacement traveled by the athlete at the end of 4s is (3) Low resistance – low melting point
(Delhi 2016) (4) High resistance – low melting point
2 216. What is the equivalent resistance of the following
(1) 2r (2) r arrangement between M and N (Jharkhand 2015)
3
3
(3) (4) 2
2 M N
207. Five resistances of same value 'x' are joined in an electric R R R
circuit as shown in figure. The equivalent resistance (1) R 2 (2) R 3
between ends P and Q is 3 . The value of x (Delhi 2016) (3) R 4 (4) R 6
X 217. For an incident ray directed towards centre of curvature of a
P spherical mirror the reflected ray (Jharkhand 2015)
(1) retraces its path
X X (2) passes through focus
(3) passes through the pole
X
X (4) becomes parallel to the principal axis
Q 218. If a wire of resistance 1 is stretched to double its length,
then resistance will be (West Bengal 2015)
1 5
(1) (2) 1
5 4 (1) (2) 2
2
21 7
(3) (4) 1
4 4 (3) (4) 4
4
Solved Questions 15
B-

219. How many times does the polarity of an A.C. suppy of


frequency 50 Hz change in one second? +
(West Bengal 2015) 10V 2k
(1) 50 (2) 100 –
(3) 25 (4) 200 R
220. In electromagnetic wave, the direction of electric and V
magnetic field are (West Bengal 2015)
(1) parallel to each other (1) 15 mW (2) 14 mW (3)
(2) Perpendicular to each other 12 mW (4) 10 mW
(3) at acute angle to each other 233. A Parachutist of weight W strikes the ground with his legs
(4) at obtuse angle to each other fixed and come to rest with an upward acceleration of
221. Vm–1 is the unit of (West Bengal 2015) magnitude 3g. Force exerted on him by the ground during
(1) Potential (2) Electric field intensity landing is (Delhi 2012)
(3) Electric current (4) Electric potential energy (1) W (2) 2W
222. Two bodies with kinetic energies in the ratio 2 : 3 are moving (3) 3W (4) None of the above
with equal momentum. The ratio of their masses 234. The speeds of sound in air and sea-water are given to be 340
(West Bengal 2015) m/s and 1440 m/s. respectively. A ship sends a strong Signal
(1) 1 : 3 (2) 1 : 2 straight down and detects its echo after 1.5 secs. The depth
(3) 3 : 2 (4) 2 : 3 of the sea at that point is (Delhi 2012)
223. A comb run through one's dry hair attracts small bits of (1) 2.16 km (2) 1.08 km
paper. This is due to : (Delhi 2014) (3) 0.51 km (4) 0.255 km
(1) comb is a good conductor 235. Eight identical spherical mercury drops charged to a potential
(2) paper is a good conductor of 20 v each are coalesced into a single spherical drop
(3) The atoms in the paper gets polarised by the charged comb (Delhi 2012)
(4) the comb possesses magnetic properties (1) The internal Energy of the system remains the same
224. On a cold morning, a metal surface will feel colder to touch (2) The new potential of the drop is 80 v
than a wooden surface, because : (Delhi 2014) (3) Internal Energy of the system decreases
(1) metal has high specific heat (4) The potential remains the same i.e. 20 v
(2) metal has high thermal conductivity 236. Two masses of 1 gm and 4 gm are moving with equal kinetic
(3) metal has low specific heat energies. The ratio of the magnitudes of their linear moments
(4) metal has low thermal conductivity is (Delhi 2012)
225. The resultant focal length of the lenses as shown in the
figure is : (Delhi 2014) (1) 4 : 1 (2) 2 :1
(1) 2f (2) f/ 2 (3) 1 : 2 (4) 1 : 6
(3) f/ 4 (4) f 237. The far point of a myopic person is 40 cm. To see the distant
226. White colour of the cloud is due to : (Delhi 2014) objects clearly, the focal length and the power of the lens
(1) reflection of seven colors of light used should be : (Punjab 2012)
(2) refraction of seven colors of light (1) – 40 cm, – 2.5 D (2) – 25 cm, – 4.0 D
(3) scattering of seven colors of light (3) + 40 cm, + 2.5 D (4) – 40 cm, + 2.5 D
(4) absorption of seven colors of light 238. A copper ring is suspended by a thread in a vertical plane. If
one end of a magnet is brought horizontally towards the
227. Which is not the unit of force ? (Rajasthan 2013) ring as shown, the ring will: (Punjab 2012)
(1) Poundal (2) Dyne
(3) Joule (4) Newton
228. Which country is called the country of Wind mills?
(Rajasthan 2013)
(1) Holland (2) Britain
(3) India (4) America S N
229. Who was the first President of Atomic Energy Commission
in India ? (Rajasthan 2013) (1)
move towards the magnet.
(1) Homi Jahangir Bhabha (2) Vikram Sarabhai (2)
not change its position.
(3) Prof. M.S. Swaminathan(4) Prof. Meghnad Saha (3)
move away from the magnet.
230. The lens used in the camera is (Rajasthan 2013) (4)
first move towards and then move away from the
(1) Convex Lens (2) Concave Lens magnet.
(3) Biconvex Lens (4) Biconcave Lens 239. What is meant by one cycle of a.c. ? (Punjab 2012)
231. The first astronaut in space was (Rajasthan 2013) (1) going from zero to + maximum.
(1) H.G. Wales (2) Neel Armstrong (2) going from + maximum to zero.
(3) Uri Gagarin (4) Robert Godai (3) going from zero to – maximum and – maximum to zero.
232. In the given circuit voltmeter shows a reading of 4V, then (4) all the three mentioned above combined together in
the power developed across R resistance will be same order.
(Delhi 2012)
EBD_7332
16
B- NTSE Stage 1 Question Bank

240. A ball of mass 50 g is thrown upwards. It rises to a maximum 252. A single electron has change | e | = 1.6 × 10–19 C. If the
height of 100 m. At what height its kinetic energy will be current passing through a conducting wire is 0.32 A, how
reduced to 70 %? (Punjab 2012) many electrons would pass through the wire in one hour?
(1) 30m (2) 40m (Odisha 2012)
(3) 60m (4) 70m (1) 72 × 1020 (2) 20 × 1015
241. Which of the following substances has lowest electrical (3) 7.2 × 1019 (4) 2 × 1018
resistivity at room temperature ? (Madhya Pradesh 2012) 253. A piece of wire of resistance R is cut into five equal parts out
(1) Aluminium (2) Iron of which four of them are connected to form a square ABCD.
(3) Nichrome (4) Diamond The fifth one is connected diagonally across BD. If a battery
242. An electric current through a horizontal metal wire flows in is connected across AC, what would be the equivalent
East to West direction, direction of magnetic field at point resistance in the circuit? (Odisha 2012)
directly above it is from : (Madhya Pradesh 2012) (1) R/25 (2) 4R/5
(1) East to West (2) West to East (3) R/5 (4) 2R/5
(3) North to South (4) South to North 254. A circular coil carrying current produces a magnetic field B0
243. If the displacement-time graph for the motion of a car is at its centre. This coil is made itself to 10 turns and the same
parallel to the time axis, the velocity of that car is : current is set up in it. The magnetic field B at its centre
(1) constant but not zero (Madhya Pradesh 2012) would be (Odisha 2012)
(2) zero (1) B = B0 (2) B = 10B0
(3) infinite (3) B = 50B0 (4) B = 100B0
(4) linearly increasing 255. A virtual image larger than the object can be produced by
244. The density of a substance is 7100 kg m–3. Its relative density (Odisha 2012)
is : (Madhya Pradesh 2012) (1) Convex mirror (2) Concave lens
(1) 7100 (2) 71 (3) Concave mirror (4) Plane mirror
(3) 7.1 (4) 71 × 105 256. A concaved lens has focal length of 15 cm. At what distance
245. If the difference of temperature of two bodies is 5°C, then should the object from the lens be placed so that it forms an
the difference of temperature on Kelvin scale is : erect and virtual image at 10 cm from the lens?(Odisha 2012)
(Madhya Pradesh 2012) (1) 30 cm (2) 15 cm
(1) 268 K (2) 278 K (3) 60 cm (4) 10 cm
(3) 5K (4) 54.6 K 257. Dimensions of power are [West Bengal 2012]
246. The expansion of a solid body depends on- (1) [M2 L2 T–2] (2) [M2 L2 T–3]
(Rajasthan 2012) (3) [M2 L T–2] (4) [M L T–2]
(1) Mass of the solid only 258. In which case of a moving body force is not needed?
(2) Nature of the solid only [West Bengal 2012]
(3) Temperature of the solid only (1) To increase the speed of the body
(4) Nature and temperature of the solid (2) To decrease the momentum of the body
247. The particles of the medium in the transverse wave move- (3) To change the direction of motion
(Rajasthan 2012) (4) To keep the body in uniform velocity
(1) Circular Motion (form) 259. A single horizontal force F is applied to a block of mass M 1
(2) Oval form which is in contact with another block of mass M 2 as shown
(3) In direction of the wave motion in the figure. If the surfaces are frictionless, the force between
(4) Perpendicular to the direction of the wave motion the block is [West Bengal 2012]
248. Velocity of sound is maximum in- (Rajasthan 2012)
(1) Water (2) Oil F M1
(3) Air (4) Iron M2
249. On a bulb is written 220 Volt and 60 watt. Find out the
resistance of the bulb and the value of the current flowing M1F M1M 2 F
through it. (Rajasthan 2012) (1) (2)
M2 M1 + M 2
(1) 806.66 ohm or 0.27 ampere
(2) 500 ohm or 2 ampere M2F M2F
(3) 200 ohm or 4 ampere (4) 100 ohm or 1 ampere (3) (4)
250. The maximum attraction in a magnet is- (Rajasthan 2012) M1 + M 2 M1
(1) In the centre (2) On the sides 260. At what temperature will the Fahrenheit scale have the double
(3) On the poles (4) On the surface reading as that of Celcius? [West Bengal 2012]
251. Two bodies of masses m and 3m, moving with velocities 3v (1) 260 °C (2) 240°C
and v respectively along same direction, collide with each (3) 160°C (4) 370°C
other. After collision they stick together and move with a 261. The minimum distance between an object and its real image
velocity V in the same direction. Then: (Odisha 2012) in a convex lens is ( f = focal length of the lens)
[West Bengal 2012]
3 (1) 2.5 f (2) 2 f
(1) V = v (2) V= v
2 (3) 4 f (4) f
4
(3) V = 2v (4) V= v
3
Solved Questions B-17
262. The ratio of electric field intensity at distance 5 cm to that at (1) Refraction, reflection
10 cm from a point charge 5Q in air is [West Bengal 2012] (2) Dispersion, refraction
(1) 2 : 1 (2) 1 : 2 (3) Refraction, dispersion, reflection
(3) 1: 4 (4) 4 : 1 (4) Reflection, dispersion
263. The solar energy is due to the reactions 267. Stars twinkle but planets do not twinkle because :
[West Bengal 2012] [Maharashtra 2012]
(1) Nuclear fusion (1) Stars emit their own light but planets receive light from
(2) Nuclear fission the stars.
(3) Combination of nuclear fusion and fission (2) Stars do not form a part of solar system.
(4) Chemical (3) Stars form a point source of light while planets are
264. Three resistances A, B, C are connected as shown in the considered as a collection of a large number of point
figure. Their resultant resistance is[Andhra Pradesh 2012] sources of light.
B (4) During refraction of star light from the atmosphere,
A star light bends more towards the normal as compared
3 to the planets.
6 C 268. A horse runs on a circular track of length 720 metres in 20
seconds and returns to the starting point. Calculate the
2
average speed. [Maharashtra 2012]
(1) 11 (2) 7.2 (1) 36 metre/second (2) 0 metre/second
(3) 6 (4) 5 (3) 18 metre/second (4) 72 metre/second
265. Which of the following statements given below is not true 269. Which one among the following is doping material in
in the context of magnetic lines of force? P-type semiconductor ? [Karnataka 2012]
[Maharashtra 2012] (1) Carbon (2) Indium
(1) Magnetic lines of forces are closed continuous curves. (3) Arsenic (4) Phosphors
(2) No two magnetic lines of force can intersect each other 270. Escape velocity of the rocket depends on:
(3) Magnetic lines of force are far away from each other at [Karnataka 2012]
the poles (1) Mass of the rocket (2) Mass of the satellite
(4) The start from the north pole and end on the south (3) Radius of the earth (4) Speed of the rocket
pole. 271. The amount of energy consumed by the 10HP water upmp
266. The beautiful natural phenomenon of the rainbow is a in 30 minutes to lift the water to overhead tank?
combination of which of the following different phenomena? [Karnataka 2012]
[Maharashtra 2012] (1) 26.856 × 106 J (2) 0.8946 × 106 J
(3) 53.712 × 106 J (4) 13.428 × 106 J

SECTION 2. CHEMISTRY
1. When 4 g of carbon is completely burnt in air, the volume 1
occupied by the liberated gas at STP is ______ (4) n = 4, l = 1, m1 = 0, ms =
2
(AP_FIITJEE 2020-21)
5. A solution of sodium sulphate in water is electrolysed using
(1) 11.2 L (2) 6.46 L inert electrodes. The products at the anode and cathode are
(3) 7.46 L (4) 22.4 L respectively - (Bihar 2020-21)
2. Copper present in 50 g of CuSO4 is (AP_FIITJEE 2020-21) (1) H2, O2 (2) O2, H2
(1) 19.90 g (2) 39.81 g (3) O2, Na (4) O2, SO2
(3) 29.5 g (4) 31.71 g 6. A substance on treatment with dilute H2SO4 liberates a
3. The mass of 3.011 × 1023 number of nitrogen atoms is colourless gas which produces (Bihar 2020-21)
(AP_FIITJEE 2020-21) (i) turbidity with baryta solution and
(1) 14 g (2) 3.5 g (ii) turns acidified dichromate solution green.
(3) 7 g (4) 28 g The reaction indicates the presence of -
4. The four quantum numbers of 19th electron of potassium is
(1) CO32– (2) S2–
(AP_FIITJEE 2020-21) 2–
(3) SO3 (4) NO3–
1 7. A gas formed by the action of alcoholic KOH on ethyl iodide
(1) n = 4, l = 3, m1 = 0, ms = decolourless alkaline KMnO4 solution, the gas is -
2
(Bihar 2020-21)
1
(2) n = 4, l = 0, m1 = 0, ms = (1) CH4 (2) C2H6
2
(3) C2H4 (4) C2H2
(3) n = 4, l = 0, m1 = 1, ms = 0
EBD_7332
18
B- NTSE Stage 1 Question Bank

8. A light greenish salt is soluble in water. On passing H2S gas (1) Cl2, Na2CO3 and CO2 (2) Cl2, NaCl and H2
into the solution, a black precipitate is obtained which (3) Cl2, NaOH and H2 (4) Cl2, HCl and NaOH
dissolves readily in HCl. The metal ion present is - 18. The equation of the reaction when aluminium is heated in air
(Bihar 2020-21) is given below. (Karnataka 2020-21)
(1) Fe 2+ (2) Co 2+
xAl yO 2 zAl 2O3
(3) Ni2+ (4) Mn 2+ The value of 'z' when the reaction is balanced is :
9. An organic compound (X) on treatment with acidified (1) 2 (2) 1
K2Cr2O7 gives a compound (Y) which reacts with I2 and
sodium carbonate to form tri-iodomethane. The compound (3) 4 (4) 3
(X) is - (Bihar 2020-21) 19. Observe the following chemical equation of carbon compound.
In presence
O CH3Cl Cl 2 x HCl
|| of sunlight
(1) CH3OH (2) CH3 C CH3 Name of the chemical reaction and the product 'x' respectively
(3) CH3CHO (4) CH3 CH CH3 (Karnataka 2020-21)
| (1) Oxidation reaction and CH2Cl
OH
(2) Combustion reaction and CH2Cl2
10. How many litres of CO2 at STP will be formed when 100 mL
(3) Substitution reaction and CH2Cl2
of 0.1 M H2SO4 reacts with excess of Na2CO3?
(4) Addition reaction and CH2Cl
(Bihar 2020-21)
20. Observe the given diagram and read the following statements
(1) 22.4 (2) 2.24 (Karnataka 2020-21)
(3) 0.224 (4) 5.6
11. A bottle of ammonia and a bottle of dry hydrogen chloride Cell wall
connected through a long tube are opened simultaneously
at both ends. The white ammonium-chloride ring first formed Nucleus
will be - (Bihar 2020-21)
(1) At the centre of the tube
(2) Near the hydrogen chloride bottle
(3) Near the ammonia bottle (i) These are cells of collenchyma tissue
(4) Throughout the length of the tube (ii) At maturity these cells lose their nucleus and cytoplasm
12. When lead nitrate is heated a brown gas is evolved, the (iii) The cells are living and thick at their corners
evolved gas is ________ (Delhi 2020-21) (iv) This tissue gives flexibility to plant parts
(1) Dioxygen (2) Nitrogen dioxide Correct statements are :
(3) Nitrous oxide (4) Dinitrogen
(1) (i), (iii) and (iv) (2) (i), (ii) and (iv)
13. When a solution of lead (II) nitrate and potassium iodide are
mixed, the yellow ppt is formed, the ppt is of ________ (3) (i) and (iii) only (4) (i) and (iv) only
21. How many covalent bonds are present in propyne molecule?
(Delhi 2020-21)
(Maharashtra 2020-21)
(1) KNO3 (2) KCl
(1) 5 (2) 7
(3) PbI2 (4) PbI4
(3) 6 (4) 5
Conc. H 2SO4
14. CH3 CH 2 OH Products (Delhi 2020-21) 22. Oxidation product of methyl alcohol is ________
443 K
(Maharashtra 2020-21)
The products formed in the above reaction is/are (1) acetic acid (2) methyl amine
(1) Ethene and H2O (2) Ethyne and H2O (3) formic acid (4) ethyl acetate
(3) Ethane and H2O (4) Methane and H2O 23. From the given diagram, identify the element 'X' and 'Y'?
15. Which one of the following oxide is insoluble in water? (Maharashtra 2020-21)
(Delhi 2020-21) Group 13 14 15 16 17 18
(1) Na2O (2) CuO Period
(3) K2O (4) CaO II B
III Si
16. Which one of the following will have the largest number of
atoms ? (Delhi 2020-21) IV 'x' As
(1) 100 g of He (2) 100 g of Na V 'y' Te
(3) 100 g of Li (4) 100 g of Al
VI At
17. The set of products formed when electricity is passed
through brine is : (Karnataka 2020-21) (1) Ga and Sn (2) Ge and Po
(3) Ge and Sb (4) Sb and Po
Solved Questions B-19
24. How many atoms are present in a mole of Ca(HCO3)2? 35. If the difference in electronegativity between two elements
(Maharashtra 2020-21) is 1.7, then the bond is _______ (Tamil Nadu 2020-21)
(1) 5 × 6.02 × 1023 (2) 7 × 6.02 × 1023 (1) 60% covalent 40% ionic
(3) 9 × 6.02 × 1023 (4) 11 × 6.02 × 1023 (2) 50% covalent 50% ionic
25. Carbon dating method developed by Willard Libby is based (3) 40% covalent 60% ionic
upon the radioactive decay of naturally occuring carbon (4) 70% covalent 30% ionic
___________ (Maharashtra 2020-21) 36. The acid which makes iron passive is _______
(1) C14 (2) C11 (Tamil Nadu 2020-21)
(3) C6 (4) C22 (1) Conc. HCl (2) Conc. H2SO4
(3) Conc. HNO3 (4) Conc. HF
26. Number of molecules present in 0.25 moles of water is
37. The value of pOH of rain water is : (Tamil Nadu 2020-21)
(Rajasthan 2020-21) (1) 7 (2) 6
(1) 3.011 × 1023 (2) 30.11 × 1023 (3) 8 (4) 9.5
(3) 1.5055 × 1023 (4) 15.055 × 1023 38. Main component of LPG is (Uttar Pradesh 2020-21)
27. Substance having equivalent number of molecules as in 9 g
of water is (Rajasthan 2020-21) (1) Methane + Ethane (2) Ethane + Propane
(1) 12 g of Magnesium (2) 12 g of Carbon (3) Propane + Butane (4) None of these
(3) 17 g of Ammonia (4) 11 g of Carbondioxide 39. What is the product when chlorine gas passes through dry
slaked lime- (Uttar Pradesh 2020-21)
28. Molecular formula of chloride of a metal 'M' is MCl2.
Molecular formula of oxide of 'M' will be (Rajasthan 2020- (1) CaCl2 (2) CaO
21) (3) CaOCl2 (4) None of these
(1) MO (2) M2O 40. An alloy which does not contain copper is
(3) MO2 (4) M2O2 (Uttar Pradesh 2020-21)
29. Conjugate acid-base pair is (Rajasthan 2020-21) (1) Magnalium (2) Bronze
(1) HCO3 , CO32 (2) NH 4 , NH 2– (3) Brass (4) German Silver
41. Which of the following does not belong to a group-
(3) OH , H 2 O 2 (4) NO 2 , NO3 (Uttar Pradesh 2020-21)
30. Gas used for precipitation of pure common salt (NaCl) from (1) Li, Na, K (2) Be, Mg, Ca
saturated solution of common salt is -(Rajasthan 2020-21)
(3) N, O, F (4) He, Ne, Ar
(1) H2 (2) Cl2
42. A byproduct of soap industry is- (Uttar Pradesh 2020-
(3) HCl (4) CO2 21)
31. Correct increasing order of reactivity of elements is -
(1) Sodium hydroxide (2) Sodium palmitate
(Rajasthan 2020-21)
(3) Glycerol (4) Gat or Oil
(1) Au, Cu, K, H (2) Au, Cu, H, K
43. The ratio of and bonds in propyne is
(3) Cu, Au, K, H (4) Cu, Au, H, K
32. An element 'X' has 8 electrons in the M shell when it gains (West Bengal 2020-21)
one electron. Which statements given below are not correct (1) 1 : 3 (2) 3 : 1
about the element 'X'? (Tamil Nadu 2020-21) (3) 2 : 3 (4) 3 : 2
(i) It belongs to 3rd period in the modern periodic table. 44. At a given temperature what will be the percentage increase
(ii) It has a valency of 7. in pressure for a 5% decrease in the volume of the gas
(iii) It belongs to 18th group in the modern periodic table. (West Bengal 2020-21)
(iv) Its valency is 1. (1) 5% (2) 5.26%
(1) (i) and (ii) (2) (ii) and (iii) (3) 6.26% (4) 10.26%
(3) (ii) and (iv) (4) (i) and (iv) 45. The gas that gives a black precipitate with aqueous Pb (NO3)
33. The Rule of Eight was proposed by :(Tamil Nadu 2020-21) solution and a white precipitate with aqueous ZnCl2 solution
(1) Kossel and Lewis (2) Henry Moseley is (West Bengal 2020-21)
(3) Dimitri Mendeleev (4) John Newland (1) CO2 (2) NO2
34. (a) CH3 CH 2 OH (b) CH3 O CH3 (3) NH3 (4) H2S
If compounds (a) and (b) have same molecular formula but 46. Find out the position isomers from the following pairs of
different kind of arrangements, then compound (b) is an : compounds (West Bengal 2020-21)
(1) alcohol (2) ether (1) H 2C C CH 2 OH, H 2 C C OCH 3
(3) aldehyde (4) acid | |
CH3 CH3
EBD_7332
20
B- NTSE Stage 1 Question Bank

(2) H2C = CH – CH2COOH, H2C = CH – COOCH3 55. Which compound has both covalent as well as co-ordinate
bond? (Uttar Pradesh 2019)
(3) H 2C CH O CH 3 , H 3C CH 2 CH 2 O CH 3
| (1) H2S (2) CO2
CH3 (3) H2O (4) SO2
O O 56. Which type of ores are concentrated by Froth floatation
|| || process: (Uttar Pradesh 2019)
(4) H 2 C CH 2 C CH3 , H3C CH C O H (1) Oxide ores (2) Sulphide ores
|
CH3 (3) Carbonate ores (4) Nitrate ores
57. Which of the following is amphoteric oxide:
47. Egg albumin in water is a (West Bengal 2020-21)
(Uttar Pradesh 2019)
(1) True solution
(1) Na2O (2) SO2
(2) Colloid
(3) Al2O3 (4) CaO
(3) Suspension
58. The following is the correct decreasing order of the ionic
(4) Supersaturated Solution radii- (Bihar 2019)
48. NH3 O 2
Pt Rh
A H2O (West Bengal 2020-21) (1) K > Ca > S > Cl (2) K > Ca > Cl– >S2–
+ 2+ 2– – + 2+

(3) Ca2+ > K+ >> Cl– >S2– (4) S2– > Cl– > K+ > Ca2+
A O2 B
59. Propyne and Propene can be distinguished by -
B O2 H2O C (Bihar 2019)
A, B and C respectively are (1) Conc. H2SO4 (2) Br2 in CCl4
(1) N2O, NO2 and HNO3 (3) Dil KMnO4 (4) AgNO3 in ammonia
(2) NO, NO2 and HNO3 60. Identify the correct order of boiling points of the following
(3) NO2, NO and HNO3 compounds- (Bihar 2019)
(4) N2O, NO and HNO3 (A) CH3CH2CH2CH2OH
49. Products obtained on electrolysis of brine are : (B) CH3CH2CH2CHO
(Delhi 2019) (C) CH3CH2CH2COOH
(1) NaHCO3, H2, CO3, H2, Cl2 (1) (A) > (B) > (C) (2) (C) > (A) > (B)
(2) H2, NaOH, NaHCO3
(3) (A) > (C) > (B) (4) (C) > (B) > (A)
(3) Cl2, NaOH, Na2O2
(4) NaOH, H2, Cl2 61. Elimination of hydrogen bromide from 2-bromobutane
50. Benzene (C6H6) have : (Delhi 2019) results in the formation of - (Bihar 2019)
(1) 12 Covalent bonds (2) 15 Covalent bonds (1) Predominantly 1-butene
(3) 18 Covalent bonds (4) 9 Covalent bonds (2) Predominantly 2-butyne
51. Aqueous solution of CsO2 is : (Delhi 2019) (3) Equimolar mixture of 1-butene and 2-butene
(1) Basic (2) Neutral (4) Predominantly 2-butene
(3) Acidic (4) Amphoteric 62. Assertion (A): Isotopes are electrically neutral.
52. A student added a drop of universal indicator to 1.00 mL
Reason (R): Isotopes are species with same mass number
of given solution and found that a green colour is
produced. The pH value of the solution will be : but different atomic number. (Andhra Pradesh)
(Delhi 2019) (1) Both (A) and (R) are true, but (R) is not the correct
(1) 7 – 9 (2) 0 – 3 explanation to (A).
(3) 10 – 12 (4) 4 – 6 (2) Both (A) and (R) are true and (R) is the correct
53. On heating camphor in a porcelain dish it got mixed in air explanation to (A).
without melting. This phenomenon is known as: (3) (A) is true, but (R) is false.
(Uttar Pradesh 2019) (4) (A) is false but (R) is true.
(1) Condensation (2) Sublimation 63. An atom ‘A’ belongs to III B group and another atom “B”
(3) Suspension (4) Evaporation belongs to VI B group. The formula of the compound
54. The H+ ion concentration of a solution is 2 10–8 mol L–1. formed is (Andhra Pradesh)
The PH value of the solution is- (log10 2 = 0.3010) (1) A2B3 (2) A3B6
(Uttar Pradesh 2019) (3) A2B (4) A3B2
(1) 7.699 (2) 7.599
(3) 7.799 (4) 7.899
Solved Questions 21
B-

(1) a, b, c and d (2) a, b and d


64. A B C D
(3) a, c and d (4) a, b and c
71. The molecular formula of Ethyne is_________
(Maharashtra 2019)
(1) C2H5 (2) C2H4
(3) C2H2 (4) C2H6
72. Weak base is ___________. (Maharashtra 2019)
(1) NaOH (2) KOH
(3) NH4OH (4) Na2O
Cu Al Fe Zn 73. Adding zinc to blue coloured copper sulphate solution, a
If we added FeSO4 to above four test tubes, in which test __________ solution of zinc sulphate is formed.
tube we observe black residue? (Andhra Pradesh) (Maharashtra 2019)
(1) “A” and “B” (2) “B” and “C” (1) Reddish (2) Colourless
(3) “A” and “C” (4) “B” and “D” (3) Greenish (4) Purple
65. Electro-negativity of the following elements increase in the 74. Number of moles present in 0.36 g of water is
order: (Andhra Pradesh) (Rajasthan 2019)
(1) C, N, Si, P (2) Si, P, C, N (1) 0.1 (2) 0.2
(3) P, Si, N, C (4) N, Si, C, P (3) 0.01 (4) 0.02
66. A reaction is described below, 75. Radioactive isotope used in the treatment of cancer disease
2Na + X Sodium ethoxide + Hydrogen is (Rajasthan 2019)
Hot
(1) Iodine-131 (2) Cobalt-60
X Conc. H 2SO 4
CH2 = CH2 + H2O (3) Sodium-24 (4) Chlorine-37
Identify ‘X’: (Karnataka 2019) 76. The conjugate bases of Bronsted acids H2O and HCl are
(1) CH3 – COOH (2) CH3 – CHO respectively (Rajasthan 2019)
(3) CH3 – CH2 – OH (4) CH3 – CH3 (1) OH–, Cl– (2) H3O+, Cl–
67. Match the salts with their common name. (Karnataka 2019) (3) H3O+, Cl+ (4) OH–, Cl+
77. The chemical formula of ‘Plaster of Paris’ is
Hydrated Salt Common Name
(Rajasthan 2019)
A MgSO4.7H2O i Gypsum
1
B CuSO4.5H2O ii Green Vitriol (1) CaSO 4 . H 2 O (2) CaSO 4 .2H 2 O
2
C FeSO4.7H2O iii Blue Vitriol
D CaSO4.2H2O iv Epsum 3
(3) CaSO 4 .H 2 O (4) CaSO4 . H 2 O
2
(1) A-iv, B-iii, C-ii, D-i (2) A-iv, B-ii, C-i, D-iii
78. The oxidation reaction in the following chemical changes
(3) A-iv, B-ii, C-iii, D-i (4) A-iv, B-i, C-iii, D-ii is (Rajasthan 2019)
68. The volume occupied by half a mole of a gas at standard (1) Cl + e– Cl – (2) Mg + 2e–
+2 Mg
temperature and pressure is: (Karnataka 2019)
(1) 5.6 L (2) 11.2 L (3) MnO4 e MnO42 (4) Fe+2 Fe+3 + e–
(3) 22.4 L (4) 2.8 L 79. 2PbO + C 2Pb + CO2 is an example of ________ reaction.
69. Choose the correct statements about the given chemical (Tamil Nadu 2019)
reaction: (Karnataka 2019) (1) Reduction (2) Redox
3MnO2 (S) + 4Al (S) 3Mn(l) + 2Al2O3 + Heat (3) Oxidation (4) Decomposition
(a) Reaction is exothermic. 80. Which among the following is highly toxic and inflammable
(b) Al is acting as oxidizing agent. gas ? (Tamil Nadu 2019)
(c) MnO2 is getting reduced. (1) CO (2) CO2
(d) Al is more reactive than Mn. (3) CS2 (4) CaC2
(1) a, c and d (2) a, b and c 81. Pick out the correct formula for blue vitriol :
(3) a, b and d (4) a, b c and d (Tamil Nadu 2019)
70. Consider the elements of group-14. Choose the correct (1) CuSO4 5H2O (2) CuSO4 7H2O
alternative: (Karnataka 2019) (3) CuSO4 6H2O (4) CuSO4 9H2O
(a) Si and Ge are semi conductors 82. The anhydride of Pyrosulphuric acid is
(b) Carbon and silicon are non-metals (West Bengal 2019)
(c) Sn and Pb are metals (1) SO2 (2) SO3
(d) Si and Ge are metalloids (3) S2O3 (4) S2O7
EBD_7332
22
B- NTSE Stage 1 Question Bank

83. Which ammonium compound does not produce ammonia 95. The compound used for removal of acidity in stomach is
gas on heating? (West Bengal 2019) (Rajasthan 2018)
(1) (NH4)2SO4 (2) (NH4)2CO3 (1) NaCl (2) MgCl2
(3) NH4NO2 (4) NH4Cl
(3) Mg(OH)2 (4) CaCl2
84. In the following compounds which two are not isomer to
each other ? (West Bengal 2019) 96. The chemical formula of dead burn plaster is :
(1) (CH3)2CHCH3, (CH3)2CHCH2CH3 (Rajasthan 2018)
(2) CH3CH2OH, CH3–O–CH3 1
(1) CaSO 4 H 2 O (2) CaSO4 2H 2 O
(3) C2H5–O–C2H5, CH3–O–C3H7 2
(4) CH3CH2CHO, CH3COCH3 (3) CaSO4 H 2 O (4) CaSO 4
85. Which reagent will be helpful in differentiating ethanoic 97. Element having highest atomic radius among the following
acid from ethanol ? (West Bengal 2019) is (Rajasthan 2018)
(1) Br2/CCl4 (2) Dilute NaOH solution (1) Li (2) Be
(3) Dilute HCl solution (4) NaHCO3 solution (3) B (4) C
86. Among the four element Li, Na, K, Be, which one has the 98. What is the formula for Ferric Oxide?
highest first Ionization energy? (West Bengal 2019) (Chandigarh 2018)
(1) Li (2) Be (1) FeO (2) Fe2O3
(3) K (4) Na (3) Fe3O4 (4) Fe2O
87. What happen when methyl orange solution is mixed with 99. In the presence of concentrated H2SO4 acetic acid reacts
HCl? (Delhi 2018) with ethyl alcohol to produce (Chandigarh 2018)
(1) Solution becomes yellow (1) In Aldehyde (2) Carboxylic Acid
(2) Solution becomes Red (3) Sulphur Dioxide (4) Ester
(3) Solution becomes Blue
100. Which of the following is also known as laughing gas?
(4) Solution becomes pink
(Chandigarh 2018)
88. Which of these salts will give acidic solution?
(Delhi 2018) (1) Methyl isocyanate (2) Sulphur Dioxide
(1) Na2CO3 (2) NaCl (3) Nitrous Oxide (4) Methyl phosphate
(3) NH4CI (4) COONa 101. Which of the following is responsible for the blackening of
silver jewellery on prolonged exposure to air?
89. pH is defined as : (Delhi 2018)
(Chandigarh 2018)
(1) log H 3O (2) log H 2O (1) Ag3N (2) Ag2O
(3) Ag2S and Ag3N (4) Ag2 S
(3) log OH (4) log H OH 102. Which substance is chemically resistant and can hold aqua
regia? (Chandigarh 2018)
90. A solution turns methyl orange into yellow. The approximate (1) Ceramics (2) Glass
pH of solution is (Delhi 2018)
(3) Plastic (4) Fibre
(1) 1.2 – 2.8 (2) 3.1 – 4.4
103. On which of the following substances will you pour
(3) 6.0 – 7.6 (4) 8.3 – 10.00 Hydrochloric acid if you wish to prepare carbon dioxide gas
91. Aqueous solution of SO2 is (Delhi 2018) in _______ laboratory? (Chandigarh 2018)
(1) Acidic (2) Basic (1) Zinc particles (2) Copper sulphate particle
(3) Neutral (4) Amphoteric (3) Pieces of marbles (4) Ammonium chloride
92. The substance showing sublimation property among the 104. Shweta went for a journey by train in June 2018. Her train
following is (Rajasthan 2018) was to depart at 7 am. Her mother packed ‘Bread Pakodas’
for her lunch. However Shweta did not eat them till evening.
(1) common salt (2) copper sulphate In the evening when she opened her lunch box, she felt an
(3) potassium nitrate (4) camphor unpleasant smell. The taste of pakodas was also sour. This
93. Number of neutrons in isotope of hydrogen, tritium is happened due to _____ (Chandigarh 2018)
(Rajasthan 2018) (1) thermal Decomposition (2) calcinations
(1) 0 (2) 1 (3) isomerism (4) rancidity
105. What is the chemical formula of milk of magnesia?
(3) 2 (4) 3
(Gujarat 2018)
94. Molecule containing coordinate covalent bond among the
(1) MgNO3 (2) MgSO4
following is (Rajasthan 2018)
(3) Mg(OH)3 (4) Mg(OH)2
(1) H2O (2) HNO3
(3) BaCl2 (4) CaO
Solved Questions 23
B-

106. Which alloy is used for preparation of statues? 118. In which of the following ink silver nitrate is used?
(Gujarat 2018) (Maharashtra 2018)
(1) Bronze (2) Brass (1) Voting ink (2) Writing ink
(3) Steal (4) Duralumin (3) Printing ink (4) Marker pen ink
107. Which is not a step of metallurgy? (Gujarat 2018) 119. 15 ml of NaOH solution gets complete neutrilised with 10 ml
(1) Reduction (2) Roasting of HCl solution. What volume of the same HCI solution will
(3) Corrosion (4) Concentration of ore be required to neutralise 30 ml of same NaOH solution?
108. What is the formula of carbon tetrachloride? (Gujarat 2018) (Uttar Pradesh 2018)
(1) CCl4 (2) CCl3 (1) 5 ml (2) 10 ml
(3) CCl2 (4) CCl (3) 15 ml (4) 20 ml
109. Valency electron in Cl– ions is (Madhya Pradesh 2018) 120. When water gas mixed with half its volume of hydrogen and
the mixture is compressed to 300 atm. pressure and passed
(1) 16 (2) 8 over ZnO-Cr2O3 catalyst a colourless liquid is obtained
(3) 17 (4) 18 which is used as solvent for paints & varnishes. The liquid
110. Physical state of water at 0°C is (Madhya Pradesh 2018) will be- (Haryana 2018)
(1) Solid (2) Liquid (1) methanol (2) ethanol
(3) Gas (4) None of the above (3) ether (4) acetone
111. Isoelectronic species are (Karnataka 2018) 121. Reaction with sodium hydrogen carbonate can be used to
a. Na + b. Al 3+ distinguish between- (Haryana 2018)
c. Mg2+ d. Ca2+ (1) Ethanoic acid & Methanoic acid
(1) a, b and c (2) a, c and d (2) Ethanol and Methanol
(3) a, b and d (4) a, b, c and d (3) Ethanol and Ethanoic acid
112. Identify the correct order of elements according to their (4) Ethylacetate and Ethanol
metallic character. (Karnataka 2018) 122. Which of the following sub shells present in atom?
(1) B > Al > Mg > K (2) Al > Mg > B > K (Delhi 2017)
(3) Mg > Al > K > B (4) K > Mg > Al > B (1) s, p, d, f (2) a, b, c, d
(3) s, d, n, g (4) None
113. Identify the solid acid at room temperature. 123. Which elements are used in atomic reactors to control the
(Karnataka 2018) speed of neutrons? (Delhi 2017)
(1) Boron and cadmium
(1) COOH (2) CH3COOH (2) Cadmium and aluminium
|
COOH (3) Boron and iron
(4) Sodium and potassium
(3) H2CO3 (4) HCOOH 124. How many atoms are present in 1 kg of silver (atomic mass
114. Choose the correct statements about the given chemical of silver = 108)? (Delhi 2017)
reaction. (Karnataka 2018) (1) 2.03 1023 atoms (2) 5.57 1024 atoms
3Fe s 4H 2 O g Fe3O4 s 4H 2 g (3) 4.27 10–23 atoms (4) 6.23 1023 atoms
125. Buckminister fullerenes is (Delhi 2017)
a. Iron is getting exidised. (1) isotope of carbon (2) isobar of carbon
b. Water is getting reduced. (3) allotrope of carbon (4) none of these
c. Water is acting as reducing agent. 126. What is the correct electronic configuration of Cr? (At. No. 24)
(Delhi 2017)
d. Water is acting as oxidising agent. (1) (Ar) 4s1 3d 5 (2) (Ar) 4s2 3d 4
(1) a, b and c (2) c and d (3) (Ar) 4s0 3d 6 (4) None of these
(3) a, b and d (4) b and c 127. Nature of Al2O3 (Aluminium oxide) is (Delhi 2017)
(1) Acidic (2) Basic
115. Which is the chemical formula of red oxide?
(3) Amphoteric (4) Neutral
(Maharashtra 2018) 128. What is the pH of dil. HCl solution which have conc. 10–8
(1) Fe2O3 (2) FeO3 mol/L? (Delhi 2017)
(3) FeO (4) FeO2 (1) 7 (2) 8
116. Which block elements are called transition elements? (3) 6.98 (4) 10
129. Which colour appears when few drops of phenolphthalein
(Maharashtra 2018) put into test tube contains lime water? (Delhi 2017)
(1) S-block (2) P-block (1) Yellow (2) Orange
(3) D-block (4) F-block (3) Pink (4) Colourless
117. In which of the following elements does not consist 130. Which is the correct answer, if n = 4 (where n is number of
isotopes? (Maharashtra 2018) shell) then number of sub shells and electron present in
atom. (Delhi 2017)
(1) Carbon (2) Neon
(1) 16, 32 (2) 32, 16
(3) Chlorine (4) Iodine (3) 32, 32 (4) 16, 16
EBD_7332
24
B- NTSE Stage 1 Question Bank

131. Which of the following are the Green house gases? (1) Mg2X (2) MgX2
(Delhi 2017) (3) MgX (1) Mg2X3
(1) CO2, CH4, N2O and O3 142. KMnO4 is a strong oxidizing agent in acidic medium. To
(2) CO2, Octane, Chlorine, Nitrogen provide acidic medium H2SO4 is used instead of HCl
(3) Methane, Oxygen, Helium, Neon beacuse: (Haryana 2017)
(4) None of these (1) H2SO4 is stronger acid than HCl
132. Which salts are responsible for yellow colour of Taj Mahal (2) H2SO4 is a dibasic acid
in Agra due to acid rain (Delhi 2017) (3) HCl is oxidized by KMnO4 to Cl2
(1) CaCl2 & CaSO4 (2) Ca (NO3)2 & CaSO4 (4) Only H2SO4 is completely ionized
143. Read the statements about carbon and choose correct
(3) Ca (NO3)2 & BaSO4 (4) CaSO4 & BaCl2 option: (Haryana 2017)
133. Choose the correct option about cheese: (Haryana 2017) A. It has small atomic size
(1) Cheese is an example of emulsion in which dispersed B. Its melting & boiling point is low as compared to other
phase is a liquid and dispersing medium is solid. members of group
(2) Example of gel in which dispersed phase is solie=d C. It shows electropositive character
and dispersing medium is liquid. D. It shows maximum tendency of catenation
(3) Example of emulsion in which dispersed phase is solid (1) A and B are correct (2) B and D are correct
and dispersing medium is liquid. (3) A, C and D are correct (4) A and D are correct
(4) Example of gel in which dispersed phase is liquid and 144. A metal 'X' is placed below Al and above Pb. The extraction
dispersing medium is solid. of metal is done by reacting carbon with its oxide. Metal
134. On reacting a compound of calcium (X) with water, oxide is used to join cracks of machine parts and rail lines by
compound (Y) is obtained. (Y) on boiling with NH4Cl a gas reacting it with Al. The metal is: (Haryana 2017)
(Z) is obtained. X, Y and Z respectively, are: (1) Zn (2) Cu
(Haryana 2017) (3) Fe (4) Mg
(1) CaCO3, CaO, NH3 (2) CaCO3, CaO, Cl2 145. The method for separation of mixture of common salt and
(3) CaO, CaCl2, Cl2 (4) CaO, Ca(OH)2, NH3 ammonium chloride is (Rajasthan 2017)
135. A colourless gas with choking smell is evolved when Cu (1) fractional distillation (2) sublimation
turning are heated with conc. H2SO4. The gas is: (3) chromatography (4) crystallization
(Haryana 2017) 146. Conjugate base of HCl in the following reaction is
(1) SO2 (2) SO3 HCl (aq) + H2O Cl– (aq) + H3O+ (Rajasthan 2017)
(3) H2S (4) S (1) H3O+ (2) H2O
136. Acetic acid is reduced with LiAlH4 to give: (3) Cl– (4) HCl
(Haryana 2017) 147. The chemical formula of Plaster of Paris is
(1) CH3CH2OH (2) CH3CHO (Rajasthan 2017)
(3) CH3OH (4) CH3CH3 (1) CaSO4 (2) CaSO4. 2H2O
137. Which of the following metal is not placed in eighth group (3) CaSO4. 1/2H2O (4) CaSO4. H2O
of Mendeleev’s periodic table? (Haryana 2017) 148. Which type of catalyst is ethanol in the following reaction?
(1) Fe (2) Na C 2 H 5OH
CHCl3 + O2 2COCl2 + 2HCl
(3) Pt (4) Ni
138. Baking powder is a mixture of: (Haryana 2017) (Rajasthan 2017)
(1) Sodium carbonate and sodium hydrogen carbonate (1) Positive catalyst (2) Negative catalyst
(2) Sodium carbonate and acetic acid (3) Bio-catalyst (4) Autocatalyst
(3) Sodium hydrogen carbonate and methanoic acid 149. Metalloid among the following is: (Rajasthan 2017)
(4) Sodium hydrogen carbonate and tartaric acid (1) lithium (2) sulphur
139. Assertion: CO2 is a gas but SiO2 is a solid at room (3) sodium (4) silicon
temperature CH 3
150. The IUPAC name of C = CH 2 is
Reason: CO2 contain C = O bonds but SiO2 does not contain CH 3
Si = O bonds (Haryana 2017)
(1) Both Assertion and Reason are true and Reason is a (Rajasthan 2017)
correct explanation of Assertion. (1) 1,1–dimethyl–2–ethene (2) 2–methyl–1–propene
(2) Both Assertion and Reason are true but Reason is not (3) 2,2–dimethyl ethene (4) 2–methyl prop–2–ene
correct explanation of Assertion. 151. Which of the following elements has an electronic
(3) Assertion is true Reason is false configuration 2, 8, 6? (Rajasthan 2017)
(4) Assertion is false Reason is true. (1) Sulphur (2) Oxygen
140. Which has more number of particles? (Haryana 2017) (3) Phosphorus (4) Chlorin
(1) 46 g of Na atom 152. Which of the following elements shows variable valency?
(2) 8 g of O2 molecules (Rajasthan 2017)
(3) 0.1 mole of carbon atom (1) Na (2) Mg
(4) 28 g of N2 molecules (3) Fe (4) Zn
141. If the aluminium salt of anion 'X' is Al2X3 the formula of 153. Formula of aluminium carbonate is: (Rajasthan 2017)
magnesium salt of 'X' will be: (Haryana 2017) (1) Al2(CO3)3 (2) Al2CO3
(3) Al2HCO3 (4) AlCO3
Solved Questions B-25
154. Formula of Freon-112 is: (Rajasthan 2017) 170. Stainless steel is an alloy in which following is added along
(1) C2F2Cl4 (2) CF2Cl2 with iron- (Uttar Pradesh 2017)
(2) CFCl3 (4) CCl3F (1) Zinc (2) Chromium
155. Number of molecules present in 14 g of N2 molecules is (3) Tin (4) Copper
(Rajasthan 2017) 171. Renewable source of energy is: (Uttar Pradesh-2017)
(1) 6.022×1023 (2) 3.011×1023 (1) Coal (2) Petroleum
(3) 1.51×1023 (4) 3.011×1022 (3) Natural gas (4) Energy of flowing water
156. The element ‘X’ reacting with chlorine forms a water soluble 172. Which of following compound is alkaline in aqueous
compound having high melting point. Element ‘X’ is medium. (Maharashtra 2017)
(Rajasthan 2017) (1) Na2CO3 (2) NaCl
(1) magnesium (2) argon (3) H2CO3 (4) CuSO4
(3) carbon (3) neon 173. Which of the following elements loses electron most readily?
157. The polymer formed by condensation of adipic acid and (Maharashtra 2017)
hexamethylene diamine is (Rajasthan 2017)
(1) Na (2) Mg
(1) isoprene (2) rayon
(3) terylene (4) nylon-6, 6 (3) K (4) Ca
158. What type of colloidal system is fog? 174. Which of the following species does not have electrons
(Uttar Pradesh 2017) equal to 18? (Maharashtra 2017)
(1) Gas in liquid (2) Liquid in gas (1) K+ (2) Cl–
(3) Liquid in liquid (4) Solid in gas (3) Ca2+ (4) K
159. Unsaturated hydrocarbon among the following is: 175. Which of the following is a double displacement reaction?
(Uttar Pradesh 2017) (Maharashtra 2017)
(1) Ethane (2) Methane (1) NH3 + HCl NH4Cl
(3) Ethylene (4) Propane
160. Nature of oxides of non-metal is: (Uttar Pradesh 2017) (2) CuSO4(aq) + Fe(s) FeSO4 + Cu
(1) acidic (2) basic (3) Na2SO4 + BaCl2 BaSO4 + 2NaCl
(3) amphoteric (4) neutral (4) CaCO3 (s) CaO (s) + CO2 (g)
161. Which among the following coal contains highest % of 176. The colour of anhydrous copper sulphate is:
carbon? (Uttar Pradesh 2017) (Maharashtra 2017)
(1) Peat (2) bituminous
(1) Blue (2) White
(3) anthracite (4) lignite
162. Most reactive metal is: (Uttar Pradesh 2017) (3) Pink (4) Green
(1) Na (2) M g 177. Ajay has a stung by red ant, it causes itching and irritation.
(3) Cu (4) Au The sting consists of which of the following acid?
163. Zinc and HCl react to form: (Uttar Pradesh 2017) (Maharashtra 2017)
(1) H2 (2) N2 (1) Acetic acid (2) Butyric acid
(3) CO2 (4) He (3) Carbonic acid (4) Formic acid
164. Which among the following non metal is liquid at room temp? 178. Which of the following compound conduct electricity in
(Uttar Pradesh 2017) aqueous solution which is a covalent compound ?
(1) Chlorine (2) Bromine (Maharashtra 2017)
(3) Mercury (4) Phosphorus (1) Calcium Chloride (2) Hydrogen Chloride
165. Which metal forms hydrogen gas when reacts with cold (3) Magnesium Chloride (4) Lithium Fluoride
water? (Uttar Pradesh 2017) 179. ______ of the following metal does not react with dil. HCl
(1) Na (2) Mg
(Maharashtra 2017)
(3) Fe (4) Zn
166. Thermosetting plastic is: (Uttar Pradesh 2017) (1) Copper (2) Aluminium
(1) Polythene (2) PVC (3) Iron (4) Zinc
(3) Bakalite (4) Polypropene 180. Select a pair of homologous from the following
167. Lens of Spectacles are made from: (Uttar Pradesh 2017) (Maharashtra 2017)
(1) Soft glass (2) Hard glass (1) C3H6 and C4H10
(3) Pyrex glass (4) Flint glass (2) CH3COOH and C2H5COOH
168. The reaction that take place in nuclear reactor is:
(Uttar Pradesh 2017) (3) C4H8 and C3H4
(1) Nuclear fusion (4) (CH3)2CO and C3H7CHO
(2) Nuclear fission 181. According to IUPAC rule, which of the following compound
(3) Controlled nuclear fission is prop–1–ene? (Maharashtra 2017)
(4) Double decomposition (1) CH3–CH2–CH3 (2) CH3–CH = CH2
169. Calcium carbide when reacts with water gives (3) CH3–CH = CH – CH3 (4) CH3–C = CH
(Uttar Pradesh 2017) 182. Stainless steel alloy is a mixture of (Maharashtra 2017)
(1) Methane gas (2) Ethane gas
(1) Fe + C + Cr + Ni (2) Ni + C + Cr + Al
(3) Ethylene gas (4) Ethyne gas
(3) Fe + Cu + Al + C (4) Fe + Zn + C + Ni
EBD_7332
26
B- NTSE Stage 1 Question Bank

183. Which of the following elements will form an acidic oxide? (3) Fossils are obtained along with mining of coal.
(Haryana, Maharashtra 2017) (4) It was formed from the dead remains of living organisms.
(1) An element with atomic no. 7 195. An organic compound butanal has functional group:
(2) An element with atomic no. 3 (Haryana 2016)
(3) An element with atomic no. 12
(1) Aldehyde (2) Ketone
(4) An element with atomic no. 19
184. Which of the general formulae represents the alkyl group? (3) Alcohol (4) Amine
(Maharashtra 2017) 196. The compound containing both covalent and ionic bond
(1) CnH2n (2) CnH2n+1 is: (Haryana 2016)
(3) CnH2n+2 (4) CnH2n–1 (1) MgCl2 (2) NH4Cl
185. Raw material required for photosynthesis is _____ and (3) CaO (4) AlBr3
water. (Maharashtra 2017) 197. Identify X in the following reaction– (Rajasthan 2016)
(1) chloroplast (2) sunlight hot, conc.
(3) nitrogen (4) carbon-dioxide CH3 CH 2 OH X H2O
H 2SO4
186. A green house gas N2O remains for how many year in the
(1) Ethane (2) Methane
atmosphere? (Maharashtra 2017)
(3) Ethene (4) Ethanol
(1) 100 (2) 114
198. Electronic configuration of an atom is 2, 8, 1. Which of the
(3) 104 (4) 109 following elements is similar with it in chemical reactivity?
187. The German silver, an alloy, has the composition (Rajasthan 2016)
(Delhi 2016) (1) K (2) Cl
(1) Cu + Sn + Zn (2) Cu + Zn + Ni (3) N (4) Ar
(3) Cu + Ag + Zn (4) Ag + Hg + Sn 199. Ethanol is made unfit for drinking by adding
188. Out of the following, which is the incorrect statement? (Rajasthan 2016)
(Delhi 2016) (1) Propanol (2) Methanal
(1) Adsorption is always an exothermic process (3) Methanol (4) Ethanal
(2) The soap solution is not a colloidal solution below its 200. Which types of radiation absorbed by CO2 molecules in
CMC. atmosphere? (Uttar Pradesh 2016)
(3) 'Argyrol' used in eye – lotion is a colloidal solution (1) X-rays (2) gamma rays
(4) Gold number is the number of moles of gold formed in (3) infra-red rays (4) UV rays
anode mud during copper refining. 201. The number of carbon atom in kerosene oil is
189. A mixture of non- reacting gasses contains hydrogen and (Uttar Pradesh 2016)
oxygen gases in the mass ratio of 1 : 4 respectively. What (1) C6 –C11 (2) C20–C30
will be the molar ratio of the above two gases in the mixture? (3) C11–C16 (4) C18–C22
(Delhi 2016) 202. Which of the following salt does not contain the water of
(1) 16 : 1 (2) 1 : 4 crystallization? (Uttar Pradesh 2016)
(3) 4 : 1 (4) 1 : 6 (1) Blue vitriol (2) Baking soda
190. An element 'X' has the same number of electrons in the first (3) Washing soda (4) Gypsum
and the fourth shell as well as in the second and the third 203. In the following structural formulae one IUPAC name is
shell. What is the formula and nature of its oxide? incorrect. Identify it: (Maharashtra 2016)
(Delhi 2016) (1) CH3–CH2–C–CH3 Butanone
(1) XO, Neutral (2) XO2, Acidic
(3) XO2, Amphoteric (4) XO, Basic O
191. Which of the following is not used as food preservative? H
(Delhi 2016)
(2) CH3–CH2–C = O Propanal
(1) Alitame (2) BHA
(3) BHT (4) Na2SO3 O
192. Which natural indicator (acid-base) is used in kitchens of (3) CH3–CH2–C–OH Ethanoic acid
our homes? (Haryana 2016) (4) CH3–CH2–CH2–CH2–OH Butanol
(1) Sodium chloride (2) Tea leaves 204. Select a compound which gives effervescence with
(3) Sodium carbonate (4) Turmeric NaHCO3 solution: (Maharashtra 2016)
193. Which of the following statement is correct regarding metals (1) C2H6O (2) C2H4O2
and non-metals? (Haryana 2016) (3) C2H4O (4) C3H8O2
(1) All metals are brittle. 205. What is the IUPAC name of the following compounds?
(Maharashtra 2016)
(2) All non-metals are ductile.
CH3 CH2 CH C3H7
(3) Generally metals are ductile.
(4) Non-metals are good conductor of electricity.
194. Coal is called fossil fuels because: (Haryana 2016) Cl C Cl
(1) In future coal will become completely exhausted.
(2) It was formed 300 million years ago. C2H5
Solved Questions B-27
(1) 4-Ethyl-3, 3-dichloro heptane (1) B C E A D
(2) 4-Ethyl-3, 3-dichloro hexane (2) B E C D A
(3) 4-Ethyl-3-chlorohexane (3) B A C E D
(4) 3, 3-dichloro-4-butyl heptane (4) B D E C A
206. X and Y are the two atomic species (Maharashtra 2016) 212. The total number of electrons present in 16 g of methane
X Y gas is: (Delhi 2015)
(1) 96.532 1023 (2) 48.176 1023
Number of Proton 8 8 (3) 60.22 1023 (4) 30.110 1023
Number of Neutron 8 10 213. Identify the correct oxidant and reductant in the following
Select the correct statement about X and Y. reaction: (Delhi 2015)
(1) X and Y are isobars PbS + 4H2O2 PbSO4 + 4H2O
(2) X and Y have different chemical properties (1) PbS – Oxidant (2) PbS – Reductant
(3) X and Y have different physical properties H2O2 – Reductant PbSO4 – Oxidant
(4) X and Y are the atoms of different elements (3) PbS – Reductant (4) H2O2 – Oxidant
207. How many electrons are present in M-shell of an element H2O2 – Oxidant H2O – Reductant
with atomic number 20? (Maharashtra 2016) 214. The pH of pure water at 25°C is 7. If water is heated to boil,
(1) 8 (2) 6 then (Delhi 2015)
(3) 18 (4) 2 (1) pH will decrease and water will become acidic
208. Which gas emits on burning of rice straw? (2) pH will increase and water will become basic
(Maharashtra 2016) (3) pH will decrease but water will remain neutral
(1) SO2 (2) NH3 (4) pH will remain 7
(3) O3 (4) H2S 215. The correct order of acidic strength of the following oxides
209. The chemical reactions and its corresponding observable is: (Delhi 2015)
features are matched below. The correct option is (1) CaO < Na2O < CO2 < ZnO
(Karnataka 2016) (2) Na2O < CaO < ZnO < CO2
A. Change in temperature i. Magnesium reacting (3) ZnO < CO2 < CaO < Na2O
with dilute sulphuric (4) CO2 < ZnO < Na2O < CaO
acid 216. Consider the chemical formulae CH3COOH and HCOOCH3
and choose the incorrect (Delhi 2015)
(1) Both have the equal boiling point
B. Evolution of a gas ii. Potassium iodide (2) Both have the equal molecular weight
reacting with lead (3) Both have the equal number of covalent bond
nitrate (4) Both are not the same compound
C. Formation of a iii. Sulphur dioxide gas 217. The substance which is chemically resistant, and can hold
precipitate reacting with acidified aqua regia: (Karnataka 2015)
potassium dichromate (1) Ceramics (2) Glass
solution (3) Fibre (4) Thermosetting plastic
D. Change in colour iv. Zinc granules reacting 218. China dish is (Karnataka 2015)
with dilute sulphuric (1) Brittle and heat resistant
acid (2) Durable and heat, resistant
(1) A - iii, B - iv, C - i, D - ii (3) Brittle and corrosive
(2) A - iv, B - i, C - ii, D - iii (4) Durable and non-corrosive
(3) A - i, B - iv, C - iii, D - ii 219. In saponification process, the fatty acid present in the oils
is neutralised by adding (Karnataka 2015)
(4) A - iv, B - ii, C - iii, D - i
(1) Sodium hydroxide (2) Aluminium hydroxide
210. The solution in which an immersed glass exhibits apparent (3) Calcium hydroxide (4) Magnesium hydroxide
disappearance is a mixture of (Karnataka 2016) 220. This is not a characteristic of members of a homologous
(1) Hydrogen fluoride and acetic acid series: (Karnataka 2015)
(2) Hydrofluoric acid and sulphuric acid (1) They possess varying chemical properties
(3) Conc. HCl and HNO3 in the ratio of 3 : 1 (2) Their properties vary in regular and predictable
(4) Acetone and chloroform in suitable proportion manner.
211. The following substances are arranged in the increasing (3) Their formulae fit the general molecular formula
order of their pH values. The correct option is (4) Adjacent members differ by one carbon and two
(Karnataka 2016) hydrogen atoms.
A. Tomato juice 221. The electronic configuration of copper can be represented
B. Tooth paste in this/these way/ways: (Karnataka 2015)
C. Saliva (after meals) (1) 1s2 2s2 2p6 3s2 3p6 4s1 3d10
D. Coffee (2) (Ar) 3d10 4s2
E. Blood (3) 1s2 2s2 2p6 3s2 3p6 4s2 3d9
(1) Only 1 & 2 are correct (2) Only 2 is correct
(3) Only 2 & 3 are correct (4) Only 1 & 3 are correct
EBD_7332
28
B- NTSE Stage 1 Question Bank

222. Fe2O3 + 2Al Al2O3 + 2Fe, this reaction is an example of a: 234. 10 g H2 and 64 g O2 were taken in a sealed vessel and
(Uttar Pradesh 2015) exploded. The amount of water produced in the reaction
(1) combination reaction was— (West Bengal 2015)
(2) double displacement reaction (1) 3 mole. (2) 4 mole.
(3) decomposition reaction (3) 1 mole. (4) 2 mole.
(4) displacement reaction 235. Which of the following are isomers? (West Bengal 2015)
223. The chemical formula of baking soda is: (1) Butane and Isobutene
(Uttar Pradesh 2015) (2) Ethane and Ethene
(1) NaHCO3 (2) Na2CO3 (3) Propane and Propyne
(3) CaOCl2 (4) CaSO4 (4) Butane and Isobutane
224. Which one of the following types of medicines is used for 236. Which of the following is a double salt?
treating indigestion? (Uttar Pradesh 2015) (1) Blue vitriol (West Bengal 2015)
(1) Antibiotic (2) Analgesic (2) Glauber’s salt
(3) Antacid (4) Antiseptic (3) Potash alum
225. MPO4 is the formula of phosphate of an element. The (4) Potassium ferrocyanide
molecular formula of its nitrate will be: 237. Which of the following has shortest carbon-carbon bond
(Uttar Pradesh 2015) length? (Delhi 2014)
(1) MNO3 (2) M(NO3)3 (1) C2H2 (2) C2H4
(3) M2(NO3) (4) M(NO3)2 (3) C2H6 (4) C6H6
226. The main factor of depletion of ozone layer is: 238. Which of the following may be isomer of aldehyde having
(Uttar Pradesh 2015) general formula CnH2nO? (Delhi 2014)
(1) chlorofluorocarbons (2) oxygen (1) Alcohol (2) Ether
(3) sulphur (4) nitrogen (3) Ester (4) Ketone
227. Methane on combustion gives (Jharkhand 2015) 239. Which of the following solution has the lowest pH value?
(1) CO2 (2) H2O (Delhi 2014)
(3) both CO2 and H2O (4) Neither CO2 nor H2O (1) 0.1 Molar NaCl solution
228. Which of the following methods is suitable for preventing (2) 0.01 Molar NaHCO3 solution
an iron frying pan from rusting? (Jharkhand 2015) (3) 0.001 Molar Na2CO3 solution
(1) Applying grease (4) 0.01 Molar NaOH solution
(2) Applying paints 240. Which one of the following is not a base?
(3) Applying a coating of zinc (Maharashtra 2014)
(4) All of the above (1) B(OH)3 (2) KOH
229. Rusting of iron takes place in (Jharkhand 2015) (3) Ca(OH)2 (4) NH4OH
(1) ordinary water 241. Which of the following orders of atomic radii is correctly
(2) distilled water represented? (Maharashtra 2014)
(3) both ordinary and distilled water (1) B < Al < Ga (2) B < Ga < Al
(4) none of the above (3) Al < B < Ga (4) B > Ga > Al
230. Mg dissolves in hot water to form (Jharkhand 2015) 242. A sample of MgCO3 contains 3.01 × 1023 Mg2+ ions and
(1) MgO (2) Mg(OH)2 3.01 × 1023 CO32 ions. The mass of the sample is
(3) MgOH (4) MgOMg(OH)2
231. While cooking, if the bottom of the vessel is getting blackend (Maharashtra 2014)
on the outside, it means that (Jharkhand 2015) (1) 42 mg (2) 84 g
(1) The food is not cooked completely (3) 0.042 kg (4) 42 mol
(2) The fuel is not burning completely 243. What are the electronic configurations of Na + and Cl– ions?
(3) The fuel is wet (Maharashtra 2014)
(4) The fuel buring completely (1) Na+ = 2, 8, 1 and Cl– = 2, 8, 7
232. Electric current is passed through a concentrated aqueous (2) Na+ = 2, 8 and Cl– = 2, 8, 8
solution of sodium chloride by means of a battery. After (3) Na+ = 2, 8, 2 and Cl– = 2, 8, 6
disconnecting the circuit the solution thus obtained is (4) Na+ = 2, 8 and Cl– = 2, 8, 7
divided into two equal parts by volume. In one part 244. Which of the following solution has the lowest pH value?
phenolphthalein and in another part methyl orange indicator (Maharashtra 2014)
was added. The colour of the solutios will be respectively— (1) 0.1 Molar NaCl solution
(West Bengal 2015) (2) 0.01 Molar NaHCO3 solution
(1) Pink and Red. (3) 0.001 Molar Na2CO3 solution
(2) Colourless and Yellow. (4) 0.01 Molar NaOH solution
(3) Pink and Yellow. 245. On addition of which metal the blue coloured copper
(4) Colourless and Red. sulphate solution turns into colourless solution?
233. Among LiCl, RbCl, BeCl2 and MgCl2 which two are the (Rajasthan 2014)
highest and the lowest ionic compounds? (1) Ag (2) Hg
(West Bengal 2015) (3) Zn (4) Au
(1) LiCl & RbCl. (2) RbCl & BeCl2.
(3) RbCl & MgCl2. (4) MgCl2 & BeCl2.
Solved Questions B-29
246. IUPAC name of the first member of homologous series of (1) N3+ (2) O2–
Ketones is (Rajasthan 2014) (3) F – (4) Na+
(1) Ethanone (2) Propanol 260. Which chemical substance is added to LPG to help in
(3) Methanone (4) Propanone detection of its leakage? (Haryana 2013)
247. The nature of solution when sodium carbonate is dissolved (1) Isobutane (2) Ethanethiol
in water will be (Rajasthan 2014) (3) Propane (4) Hydrogen sulphide
(1) acidic (2) basic 261. The ion of an element has 3 positive charge, 27 mass number
(3) neutral (4) amphoteric and 14 neutrons. What is the number of electrons in this
248. The chemical reaction HNO3 + KOH KNO3 + H2O is an ion? (Haryana 2013)
example of (Rajasthan 2014) (1) 13 (2) 14
(1) neutralisation (3) 10 (4) 16
(2) double displacement 262. Which one of the following is the natural fruit ripening
(3) neutralisation and double displacement hormone? (Uttar Pradesh 2013)
(4) combination (1) Ethane (2) Ethene
249. pH of a solution is zero. The nature of this solution is (3) Ethyne (4) Carbide
(Rajasthan 2014) 263. Silver jewellery becomes black on prolonged exposure to
(1) acidic (2) basic air, it is due to the formation of (Uttar Pradesh 2013)
(3) neutral (4) amphoteric (1) Ag3N (2) Ag2O
250. Graphite is used as a lubricant in machines because it has a (3) Ag2S and Ag3N (4) Ag2S
very high melting point and also it. (Jharkhand 2014) 264. The volume of 1 mole of an ideal gas at 25°C and 1 bar
(1) is crystalline pressure is: (Uttar Pradesh 2013)
(2) has layer structure (1) 22.4 L (2) 22.7 L
(3) is a giant molecule (3) 24.8 L (4) 24.4 L
(4) is a liquid at room temperature 265. In the year 1984, the Bhopal gas tragedy was caused by the
251. Which of the following oxyacid of phosphorus are leakage of : (Madhya Pradesh 2013)
monobasic (monoprotic)? (Jharkhand 2014) (1) Carbon monoxide (2) Methyl isocyanate
(1) H3PO4 (2) H3PO3 (3) Nitrogen oxide (4) Sulphur oxide
(3) H3PO2 (4) H4P2O7 266. Oxidation is defined as : (Madhya Pradesh 2013)
252. The percentage of gold present in 20 carat gold is (1) loss of electron (2) gain of electron
(Jharkhand 2014) (3) loss of proton (4) gain of proton
(1) 100 (2) 73.86 267. The process by which a mixture of sodium chloride and
(3) 50 (4) 83.33 ammonium chloride can be separated, is called
253. Which of the following aldehyde undergo cannizaro (Rajasthan 2013)
reaction? (Jharkhand 2014) (1) Sublimation (2) Chromatography
(1) C3H7CHO (2) C6H5CHO (3) Evaporation (4) Distillation
(3) CH3CHO (4) CH3CH2CHO 268. Isotopes of an element have (Rajasthan 2013)
254. An element have atomic number 19 and mass number 39. (1) same physical properties
The number of neutron in its nucleus is- (2) different chemical properties
(Uttar Pradesh 2014) (3) different no. of neutrons
(1) 20 (2) 58 (4) different atomic number
(3) 19 (4) 39 269. A non metal, which is found in liquid state is
255. Mass percentage of nitrogen in the compound N2O3 is (Rajasthan 2013)
(Uttar Pradesh 2014) (1) Bromine (2) Iodine
(1) 36.84 (2) 46.70 (3) Oxygen (4) Carbon
(3) 82.40 (4) 63.60 270. Element “X” which is solid and having high melting point,
256. A metal M has its Chloride formula MCl3 and equivalent from a Chloride “XCl3”. This element “X” would be in which
weight of metal is 9. Atomic weight of element is group of periodic table : (Rajasthan 2013)
(Uttar Pradesh 2014) (1) Na (2) Mg
(1) 9 (2) 18 (3) Al (4) Si
(3) 27 (4) 3 271. Methane with the molecular formula “CH4” has
257. Atomic weight of an element is 30. Its equivalent weight is (Rajasthan 2013)
10. The velocity of element will be: (Uttar Pradesh 2014) (1) 4 Covalent bonds (2) 8 Covalent bonds
(1) 1 (2) 2 (3) 6 Covalent bonds (4) 2 Covalent bonds
(3) 3 (4) 4 272. If the quantity of metal in a metallic oxide is 60%, then its
258. Specific heat of any element is 0.1. The nearest atomic equivalent weight is: (Delhi 2012)
weight of that element will be: (Uttar Pradesh 2014) 1 1
(1) 32 (2) 64 (1) of molecular weight (2) of molecular weight
(3) 62 (4) 48 5 2
259. Which one of the following is the smallest in size? 3 3
(Haryana 2013) (3) of molecular weight (4) of molecular weight
2 5
EBD_7332
30
B- NTSE Stage 1 Question Bank

273. Arrange the following elements in order of their increasing (2) Neutron have no charge
ionization energies O, S, Se, Te, Po: (Delhi 2012) (3) Electron and proton have same mass
(1) Se, Te, S, Po, O (2) O, S, Se, Te, Po (4) Masses of proton and neutron are nearly the same
(3) Po, Te, Se, S, O (4) Te, O, S, Po, Se 283. The electronic structure of four elements A, B, C, D
274. Structures of nuclei of three atoms A, B and C are given respectively are : (Madhya Pradesh 2012)
below: (Delhi 2012) (A) 1s2 (B) 1s2 2s2 2p2
A has 90 protons and 146 neutrons (C) 1s2 2s2 2p5 (D) 1s2 2s2 2p6
B has 92 protons and 146 neutrons The tendency to form electrovalent bond will be largest in :
C has 90 protons and 148 neutrons (1) A (2) B
Based on the above data, which of these atoms are isotopes (3) C (4) D
and which are isobars? 284. If 0.5 g of any substance is completely transformed into energy,
(1) A and C are isotopes; B and C are isobars then how much energy in kilo-joule will be obtained ?
(2) A and B are isotopes; A and C are isobars (Madhya Pradesh 2012)
(3) B and C are isobars; A and B are isotopes (1) 1.5 × 1010 kilo-joule (2) 3.0 × 1010 kilo-joule
(4) A and C are isotopes; A and B are isobars (3) 4.5 × 1010 kilo-joule (4) 6.0 × 1010 kilo-joule
275. Which one of the following metal oxides shows both acidic 285. On passing CO2 in excess in aqueous solution of sodium
and basic characters? (Delhi 2012) carbonate the substance obtained is :
(1) Na2O (2) K2O (Madhya Pradesh 2012)
(3) CuO (4) Al2O3 (1) NaOH (2) NaHCO3
276. Which of the following will have equal number of electrons? (3) Na2CO3.10H2O (4) Na2CO3.H2O
(Delhi 2012) 286. A Brown and bright element “x” when heated in presence of
(1) Cl– and Br– (2) Na+ and Mg2+ air turns in to black substance “y”. If hydrogen gas is passed
(3) Ar and Ne (4) Mg2+ and Ca2+ over this heating material again “x” is obtained. “x” and
277. Mg has three natural isotopes whose isotopic masses and “y” are- (Rajasthan 2012)
relative abundances are respectively 23.98(78.60%), (1) Cu and CuO (2) S and SO2
24.98(10.11%) and 25.98(11.29%). The atomic mass of Mg (3) C and CO2 (4) Na and NaH
will be (Delhi 2012) 287. The pH of a solution which turns red litmus Blue will be-
(1) 23.42 (2) 24.31 (Rajasthan 2012)
(3) 24.95 (4) 23.95 (1) 1 (2) 9
278. Choose the correct from the following (Punjab 2012) (3) 4 (4) 5
(i) Salt of a strong acid and a strong base are neutral with 288. Which of the following is not a pure substance?
pH value of 7. (Odisha 2012)
(ii) Salt of a strong acid and a weak base are basic with pH (1) Gold (2) Glucose
value more than 7. (3) Pure Milk (4) Water
(iii) Salt of a weak acid and a strong base are acidic with pH 289. Which of the following element has highest electron-gain
value less than 7. enthalpy? (Odisha 2012)
(1) (i) and (ii) (2) (ii) and (iii) (1) Oxygen (2) Fluorine
(3) only (i) (4) (i) and (iii) (3) Chlorine (4) Neon
279. Which out of following hydrocarbons undergo addition 290. The geometry of Ni(CO)4 is (Odisha 2012)
reactions? (Punjab 2012) (1) tetrahedral (2) octahedral
C2H6, C3H8, C3H6, C2H2 and CH4 (3) square planar (4) pyramidal
(1) C2H6 and C3 H8 (2) C3H6 and C2H2 291. ___________ is used as electrodes in batteries and
(3) CH4 and C2H6 (4) C3H8 and C2H2 composites in aircrafts. (Odisha 2012)
280. Match the following. (Punjab 2012) (1) Diamond (2) Graphite
Natural Source Acid (3) Activated charcoal (4) Carbon black
(i) Vinegar P. tartaric acid 292. Identify the metal that occurs in the native state in nature.
(ii) Orange Q. oxalic acid (Odisha 2012)
(iii) Tamarind R. acetic acid (1) Potassium (2) Copper
(iv) Tomato S. citric acid (3) Aluminium (4) Zinc
(1) (i)–Q, (ii)–R, (iii)–P, (iv)–S 293. The chemical formula of white vitriol is _________.
(2) (i)–R, (ii)–Q, (iii)–P, (iv)–S (Odisha 2012)
(3) (i)–R, (ii)–S, (iii)–P, (iv)–Q (1) CuSO4.5H2O (2) ZnSO4.7H2O
(4) (i)–S, (ii)–Q, (iii)–R, (iv)–P (3) FeSO4.7H2O (4) MgSO4.7H2O
281. Arrange the following atoms in the order of increasing atomic 294. 2 litre of hydrogen and 1.2 litre chlorine are mixed and
radius: (Punjab 2012) exploded. The composition by volume of the resultant
F, Cl, C, O mixture will be (West Bengal 2012)
(1) F, Cl, O, C (2) C, O, F, Cl (1) 0.8 litre of hydrogen and 2.24 litre of chlorine
(3) O, C, F, Cl (4) F, O, C, Cl (2) 0.8 litre of hydrogen and 2.24 litre of hydrogen chloride
282. Which of the following statements is incorrect ? (3) 2.24 litre of hydrogen chloride
(Madhya Pradesh 2012) (4) 0.8 litre of chlorine and 22.4 litre of hydrogen chloride
(1) Charges on an electron and proton are equal and
opposite
Solved Questions B-31
295. The correct order of increasing acidic nature of SO2, SiO2, (1) builders (2) stearic acid
P2O3 and Al2O3 is (West Bengal 2012) (3) perfume (4) glycerol
(1) SO2 < P2O3 < SiO2 < Al2O3 307. Acidity in the Sugarcane juice is removed by adding:
(2) Al2O3 < SiO2 < SO2 < P2O3 (Andhra Pradesh 2012)
(3) Al2O3 < SiO2 < P2O3 < SO2 (1) Ca(OH)2 (2) CO2
(4) SiO2 < SO2 < Al2O3 < P2O3 (3) SO2 (4) H2O
296. 1.35 g ‘X’ element is completely converted to 1.88 g compound 308. If the pH of, a solution is 8, .its (H+) is ...
‘XO’. What is the atomic weight of ‘X’? (Andhra Pradesh 2012)
(West Bengal 2012) (1) log 10–8 (2) 108
(1) 20.32 (2) 40.75 (3) 10–8 (4) 8
(3) 16.25 (4) 56.10 309. In modern periodic table, arrange the third row elements Na,
297. One drop of methyl orange solution when added to the Mg, Al and Si in the increasing order of their atomic size :
solution obtained after electrolysis of a concentrated (Maharashtra 2012)
solution of NaCl with Pt electrodes, the colour of the solution (1) Na > Mg > Al > Si (2) Al > Si > Na > Mg
will turn (West Bengal 2012) (3) Si > Al > Mg > Na (4) Na > Al > Si > Mg
(1) Orange (2) Pink 310. From the given reactions which is a redox reaction?
(3) Yellow (4) Colourless (Maharashtra 2012)
298. How many isomers are possible for an alkane having (1) CuO(s) + H2(g) Cu(s) + H2O(1)
molecular formula C6H14? (West Bengal 2012) (2) 2Cu(s) + O2(g) 2CuO(s)
(1) 3 (2) 4 (3) C(s) + 2H2(g) CH4(g)
(3) 5 (4) 6 (4) 4Al(s) + 3O2(g) 2Al2O3(s)
299. Which of the following can not be used to extract a metal 311. Molecular formula of concrete is : (Maharashtra 2012)
from its ore? (West Bengal 2012) (1) 3CaO · Al2O3 (2) 2CaO · Al2O3 · 6H2O
(1) Electrolytic reduction (3) CaO · Al2O3 · H2O (4) CuSO4 · 3H2O
(2) Carbon reduction 312. .......................... is called impure sodium chloride.
(3) Reaction with oxygen (Maharashtra 2012)
(4) Reaction with a more electropositive metal (1) Gypsum salt (2) Rock salt
300. Which one among the following is not a periodic property (3) Carbonate salt (4) Rochel salt
(West Bengal 2012) 313. From the given alternatives, whose chemical and physical
(1) Electronegativity (2) Atomic volume properties are not same? (Maharashtra 2012)
(3) Ionization potential (4) Radioactivity (1) Graphite and diamond
301. Which test can be used to distinguish between acetylene (2) Phosphorous and sulphur
and ethylene gases? (West Bengal 2012) (3) Carbon and hydrogen
(1) Reaction with Br2 in CCl4 solvent (4) Methyl alcohol and acetic acid
(2) Conc. H2SO4 314. DHOKALA is a ............................ type of solution.
(3) Ammoniacal cuprous chloride solution (Maharashtra 2012)
(4) Hydrogen gas in presence of Pt. (1) Solid-in-solid (2) Solid-in-gas
302. The light producing gas in the lamp of the hawkers is (3) Solid-in-liquid (4) Gas-in-solid
(West Bengal 2012) 315. In iron metallurgy, lime stone is used : (Karnataka 2012)
(1) butane (2) ethylene (1) to obtain heat energy
(3) acetylene (4) methane (2) to reduce iron oxide into iron
303. Which of the following is aromatic hydrocarbon? (3) as an iron ore
(Andhra Pradesh 2012) (4) to remove sand (SiO2)
(1) C2H2 (2) C3H8 316. The compound which shows hydrogen bonding is
(3) C5H12 (4) C6H6 (Karnataka 2012)
304. Most Electronegative element is (Andhra Pradesh 2012) (1) Hydrogen sulphide (2) Table salt
(1) F (2) Cs (3) Methane (4) Ammonia
(3) He (4) I 317. Which of the following statements is true?
305. Molecule with double bond (Andhra Pradesh 2012) (Karnataka 2012)
(1) H2 (2) F2 (1) Colloidal solutions does not show Tyndal effect
(3) Cl2 (4) O2 (2) Colloidal solutions show Brownian movement
306. Shaving soap contains excess of .... to slow lather drying (3) Colloidal solutions are heterogeneous
(Andhra Pradesh 2012) (4) Size of the colloidal particles is less than 1 nm
EBD_7332
32
B- NTSE Stage 1 Question Bank

SECTION 3. BIOLOGY
1. Match the following : (AP_FIITJEE 2020-21) 10. Pure Bred Pea Plant A is crossed with pure bred pea plant B.
Group - A Group - B It is found that the plants which look like A do not appear in
F1 generation but re-emerge in F2 generation, which of the
(i) Ovary (A) development of male sex
plants A and B are tall and dwarf? (Bihar 2020-21)
organs
(1) A are tall and B are also tall
(ii) Pituitary gland (B) increases heart beat rate
(2) A are tall and B are dwarf
(iii) Testis (C) menstrual cycle
(3) A are dwarf and B are tall
(iv) Adrenal gland (D) growth of bones
(4) A are dwarf and B are also dwarf
(1) (i)-(C), (ii)-(D), (iii)-(A), (iv)-(B) 11. UV rays cause cancer but in stratosphere the same UV rays
(2) (i)-(D), (ii)-(C), (iii)-(A), (iv)-(B) are helping us, how? (Delhi 2020-21)
(3) (i)-(C), (ii)-(D), (iii)-(B), (iv)-(A) (1) They divert harmful UV rays back to sun.
(4) (i)-(A), (ii)-(D), (iii)-(B), (iv)-(C) (2) They convert oxygen in stratosphere into ozone.
2. Identify the right statement about Abscisic acid (3) UV rays are not present in stratosphere.
(AP_FIITJEE 2020-21) (4) UV rays reach the earth surface then bounce back
(1) It helps in ripening of fruits carrying ozone to stratosphere.
(2) It promotes cell division 12. Match the following : (Delhi 2020-21)
(3) It promotes seed dormancy Column-I Column-II
(4) It causes stem elongation in plants (i) Regeneration (a) Shoot
3. Identify the mismatched one (AP_FIITJEE 2020-21) (ii) Rhizophus (b) Pollen grain
(1) Tridax - Pyrethroids (2) Datura - Cocaine (iii) Plumule (c) Vegetative Propagation
(3) Azardiachta - Nimbin (4) Papaver - Morphine (iv) Rose (d) Planaria
4. Plasma membrane of a cell is made up of (v) Stigma (e) Spores
(AP_FIITJEE 2020-21) (1) (i)-(a); (ii)-(e); (iii)-(d); (iv)-(b); (v)-(c)
(1) Proteins and Carbohydrates (2) (i)-(b); (ii)-(d); (iii)-(a); (iv)-(c); (v)-(e)
(2) Proteins and Lipids (3) (i)-(b); (ii)-(a); (iii)-(d); (iv)-(c); (v)-(e)
(3) Carbohydrates and Lipids (4) (i)-(d); (ii)-(e); (iii)-(a); (iv)-(c); (v)-(b)
(4) Lipids and Nucleic acids 13. The opening and closing of the stomata depends upon -
5. When the blood flows out through a wound, the platelets (Delhi 2020-21)
release an enzyme called _____ to stop the blood flow.
(1) Oxygen (2) Temperature
(AP_FIITJEE 2020-21)
(3) Carbon dioxide (4) Water in guard cells
(1) Prothrombin (2) Fibrinogen 14. Match the Column-I and Column II and select correct option.
(3) Thrombokinase (4) Thrombin (Delhi 2020-21)
6. The disease that affects our lungs is - (Bihar 2020-21) Column-I Column-II
(1) AIDS (2) Rabies (A) Ribosome (I) ATP formation
(3) Polio (4) Tuberculosis (B) Mitochondria (II) Photosynthesis
7. Which of the following is not a plant hormone? (C) Centriole (III) Protein synthesis
(Bihar 2020-21) (D) Chloroplast (IV) Cell division
(1) Auxin (2) Florigen (1) (A)-(I); (B)-(II); (C)-(IV); (D)-(II)
(3) Cytokinin (4) Oxytocin (2) (A)-(III); (B)-(I); (C)-(IV); (D)-(II)
8. Which of the following composition represents energy rich (3) (A)-(IV); (B)-(III); (C)-(II); (D)-(I)
food? (Bihar 2020-21) (4) (A)-(II); (B)-(I); (C)-(III); (D)-(IV)
(1) Vitamins and minerals 15. Which of the following is a barrier method of contraception?
(2) Carbohydrates and fats (Delhi 2020-21)
(3) Water and roughage (1) Diaphragm (2) Contraceptive pills
(4) Proteins and mineral salts (3) Tubectomy (4) All of the above
9. Which of the following is not a natural resources? 16. Sperms are produced in the (Delhi 2020-21)
(Bihar 2020-21) (1) Seminiferous tubules (2) Interstitial cell
(1) Soil (2) Water (3) Vas deferens (4) Prostate gland
(3) Electricity (4) Air
Solved Questions 33
B-

17. The function of finger like projections called 'villi' in small (1) Algae (2) Fungi
intestine is to : (Karnataka 2020-21) (3) Thallophyta (4) Bryophyta
(1) Supply the digested food to each and every cell of the 27. Lack of which element occurs when Algal Bloom is formed
body in a waterbody. (Rajasthan 2020-21)
(2) Separate the water from undigested food (1) Oxygen (2) Nitrogen
(3) Convert the glucose into carbohydrates (3) Hydrogen (4) Calcium
(4) Increase the surface area for absorption of digested 28. The plant group called "pollution indicator" is
food
(Rajasthan 2020-21)
18. Because of lack of oxygen in muscle cells, 3-carbon molecule
(1) Bryophyta (2) Lichen
pyruvate is converted into : (Karnataka 2020-21)
(3) Gymnosperm (4) Pteridophyta
(1) Lactic acid (2) Ethanoic acid
(3) Hydrochloric acid (4) Acetic acid 29. Which of the following is not a member of phylum
Arthropoda (Rajasthan 2020-21)
19. Identify the relationship between the first two words and suggest
the suitable word for the fourth place related to the third word. (1) House-fly (2) Earthworm
Adrenal gland : Adrenalin : : Pancreas : ____________ (3) Shrimp (4) Cockroach
30. DNA synthesis occurs, in which stage of cell cycle.
(Karnataka 2020-21)
(1) Oestrogen (2) Thyroxin (Rajasthan 2020-21)
(3) Insulin (4) Testosterone (1) G-I phase (2) G-II phase
20. Pair of an organism and its asexual method of reproduction (3) M-phase (4) S-phase
is given below. Choose the correct pair : 31. Example of Fat digesting enzyme is (Rajasthan 2020-21)
(Karnataka 2020-21) (1) Amylase (2) Pepsin
(1) Amoeba - Regeneration (3) Lipase (4) Nucleases
(2) Bryophyllum - Vegetative reproduction 32. Hydroponics was demonstrated by a German Botanist
(3) Yeast - Fragmentation ______ in 1980. (Tamil Nadu 2020-21)
(4) Leishmania - Budding (1) Julius Von Sachs (2) Nehemiah Grew
21. ______ is most clever animal among all non-chordates and (3) Robin Hill (4) Robert Brown
which can change its colour. (Maharashtra 2020-21) 33. Which is not a component of stele? (Tamil Nadu 2020-21)
(1) Octopus (2) Lizard (Chameleon) (1) Pitch (2) Pericycle
(3) Snail (4) Balanoglossus (3) Cambium (4) Vascular tissue
22. Which is effective antibiotics against tuberculosis? 34. Choose the incorrect statement. (Tamil Nadu 2020-21)
(Maharashtra 2020-21) (1) Epithelium helps in absorption of nutrients.
(1) Cephalosporins (2) neomycin (2) Epithelium allows the normal movement.
(3) streptomycin (4) rifamycin (3) Epithelium helps in elimination of waste products.
23. Glucose and fructose syrup can be obtained from cornflour (4) Epithelium secrets chemical substances.
by action of enzymes obtained from ___________ 35. Find out the true and false statements.
(Maharashtra 2020-21) (Tamil Nadu 2020-21)
(1) Brevibacterium and Corynebacterium (a) A recessive factor is altered by the presence of a dominant
(2) Hansenula and lacto-bacillus brevis factor.
(3) Saccharomyces cerevisine and Candida (b) Dominant factor is expressed in both F1 and F2 generations.
(4) Bacillus and Streptomyces (c) Recessive factor is expressed only in F1 generation of
24. Consumption of tobacco products lead to which disease in a monohybrid cross.
human beings? (Maharashtra 2020-21) (d) Gregor Mendel conducted his experiment on bean plant.
(1) tuberculosis (2) AIDS (1) (a) false, (b) false, (c) true, (d) false
(3) cancer of the lungs (4) leprosy (2) (a) true, (b) false, (c) true, (d) true
25. Amongst the following which disease spread through (3) (a) true, (b) true, (c) false, (d) false
bacteria? (Maharashtra 2020-21) (4) (a) true, (b) false, (c) false, (d) false
(1) Hepatitis (2) Pneumonia 36. Assertion (A) : RBC plays an important role in the transport
(3) Influenza (4) Chicken pox of respiratory gases.
26. In which of the plant group chitineous cell wall is found Reason (R) : RBC do not have cell organelles and nucleus.
(Rajasthan 2020-21) (Tamil Nadu 2020-21)
EBD_7332
34
B- NTSE Stage 1 Question Bank

(1) Both (A) and (R) are true and (R) is correct explanation 47. The part of human alimentary canal where no enzyme
of (A) secretion takes place is (West Bengal 2020-21)
(2) Both (A) and (R) are true but (R) is not the correct (1) Mouth (2) Oesophagus
explanation of (A) (3) Stomach (4) Ileum
(3) (A) is true but (R) is false
48. The immunoglobulin which is transported to the foetus
(4) Both (A) and (R) are false through placenta from mother is (West Bengal 2020-21)
37. Which organele cell is called power house of cell?
(1) lgG (2) lgA
(Uttar Pradesh 2020-21)
(3) lgE (4) lgM
(1) Mitochondria (2) Chloroplast
(3) Ribosome (4) Lysosome 49. DDT is non-biodegradable chemical when it enters food
chain it gets accumulated in each trophic level. This
38. Nematoblast or stinging cells are found in which phylum of phenomenon is called as - (Delhi 2019)
animals (Uttar Pradesh 2020-21)
(1) Eutrophication (2) Chemical amplification
(1) Porifera (2) Annelida (3) Biomagnification (4) Chemical magnification
(3) Cnideria (4) Arthopoda 50. Correct sequence of reflex arc is (Delhi 2019)
39. In which organ, bile juice formation takes place? (1) Receptor Motor Neuron Sensory Neuron
(Uttar Pradesh 2020-21) Effector organ Relay Neuron
(1) Liver (2) Gall bladder (2) Receptor Sensory Neuron Motor Neuron
Effector organ Relay Neuron
(3) Pancreas (4) Stomach
(3) Receptor Sensory Neuron Motor Neuron
40. Where, glycolysis occurs in cell (Uttar Pradesh 2020-21) Relay Neuron Effector organ
(1) In Mitochondria (2) In Chloroplast (4) Receptor Sensory Neuron Relay Neuron
(3) In Cytopasm (4) In Nucleus Motor Neuron Effector organ
41. Which plant hormone causes apical dominance? 51. Tricuspid valve is present in (Delhi 2019)
(Uttar Pradesh 2020-21) (1) Right atria and right ventricle
(1) Auxine (2) Gibbereline (2) Left atria and left ventricle
(3) Wall of atrium
(3) Cytokinine (4) Ethylene
(4) Wall of vetricle
42. Cholera diseases caused by which pathogen-
52. Root pressure is effective way transporting water in xylem.
(Uttar Pradesh 2020-21) This pressure is generated (Delhi 2019)
(1) Virus (2) Bacteria (1) In bright sunlight
(3) Fungus (4) Protozoa (2) During night
43. In human eye, at the blind spot (West Bengal 2020-21) (3) At very low temperature
(1) Only rod cells are present (4) In high trees
(2) Only cone cells are present 53. Which of the following reactions take place during break
down of molecules in the respiration in our body?
(3) Both rod and cone cells are present
(Delhi 2019)
(4) Neither rod nor cone cells are present
(1) Oxidation (2) Reduction
44. The disease which usually spreads through cuts and wounds
(3) Oxidation – reduction (4) Photo-oxidation
is (West Bengal 2020-21)
54. Lactic acid is produced when pyruvate is broken down
(1) Chicken pox (2) Malaria
(1) In presence of oxygen in mitochondria
(3) Tuberculosis (4) Tetanus (2) In absence of oxygen in mitochondria
45. Lysosome stores (West Bengal 2020-21) (3) In presence of oxygen in muscle cells
(1) ATP (2) Hydrolytic enzymes (4) In absence of oxygen in muscle cells
(3) Carbohydrate (4) Protein 55. Separation of oxygenated and deoxygenated blood.
46. Cardiac muscle is (West Bengal 2020-21) (Delhi 2019)
(1) striated and voluntary I. Fulfils energy requirments of body
(2) smooth and voluntary II. Ensures the effect transfer of oxygen in the body
(1) Both statements are true
(3) striated and involuntary
(2) Statement I is true but statement II is false
(4) smooth and involuntary
(3) Statement I is false but statement II is true
(4) Both the statements are false.
Solved Questions B-35
56. Choose the correct option to complete ‘A’, ‘B’, ‘C’ and ‘D’ 63. Which of the following is not a part of the female
in the following table (Delhi 2019) reproductive system in human beings? (Uttar Pradesh 2019)
(1) Ovary (2) Uterus
Hormone Function
(3) Fallopian tube (4) Vas deferens
A Stimulates growth in all organs
64. Causative agent of ‘Kala azar’ (Black fever) is:
B Stimulates pituitary to release growth hormone (Uttar Pradesh 2019)
C Controls blood sugar lever (1) Bacteria (2) Virus
D Regulates carbohydrate metabolism (3) Protozoan (4) Fungi
65. Biotic components of ecosystem are- (Uttar Pradesh 2019)
(1) A – Insulin, B – Thyroxine, C – Growth Hormone, D –
Growth Hormone Release Factor (1) Producers (2) Consumers
(2) A – Growth Hormone, B – Insulin, C – Thyroxine, D – (3) Decomposers (4) All of above
Growth Hormone Release Factor 66. Which of the following is known as the ‘suicide bag’ of
(3) A – Thyroxine, B – Insulin, C – Growth Hormone, D – the cell? (Uttar Pradesh 2019)
Growth Hormone Release Factor (1) Plastid (2) Mitochondria
(4) A – Growth Hormone, B – Growth Hormone Release (3) Ribosome (4) Lysosome
Factor, C – Insulin, D – Thyroxine 67. A plant cell, an animal cell and a bacterial cell share the
57. If a pea plant with wrinkled seed and heterozygous tall following structure features - (Bihar 2019)
plant were self pollinated, what will be the phenotypes of (1) Cell membrane, Endoplasmic reticulum and Vacuole
plants of F2 generation. (Delhi 2019) (2) Cell wall, Plasma membrane, Mitochondria
(1) 75% plants will be tall and have wrinkled seeds and (3) Cell wall, Nucleus and Cytoplasm
other 25% will be dwarf with wrinkled seeds (4) Plasma membrane, Cytoplasm, Ribosome
(2) 50% plants will be tall and have wrinkled seeds and 68. Most fishes do not sink in water due to the presence of-
50% will be dwarf with wrinkled seeds
(Bihar 2019)
(3) 50% plants will be tall and have wrinkled seeds and
(I) Swim bladder (II) Air bladder
other 50% will be dwarf with round seeds
(III) Air sacs (IV) Air in spongy bones
(4) 25% plants will be tall and have wrinkled seeds and
other 75% will be dwarf with wrinkled seeds. (1) I & II are correct (2) II & III are correct
58. Two similar pea plants are growing in two different islands (3) III & IV are correct (4) I, II & III are correct
separated by a vast ocean. The phenonmen on of 69. The gene for the genetic disease “Haemophilia” is present
geographical isolation will (Delhi 2019) on the ‘X’ chromosome. If a haemophilic male marries a
(1) Not be seen as the plants get self pollinated normal female, what would be the probability of their son
being haemophilic. (Bihar 2019)
(2) Be seen as the plants are growing in isolated regions
(1) 50% (2) 100%
(3) Not be seen as the plants get pollinated by ocean
water currents (3) Nil (4) 3 : 1
(4) Be seen as the plants do not get pollinated and 70. The following diagram shows a simple version of energy
reproduces asexally flow through food web. (Bihar 2019)
59. Presence of _______ is an indicator of pollution level in Sunlight
water. (Delhi 2019)
Plants
(1) Colour (2) Coliform bacteria
(3) Rhizo bacteria (4) Spiral bacteria Animals
60. Leaves of tendu are the source of income of large number
Decomposers
of people in India. These leaves are used to make
(Delhi 2019) ?
(1) Thatched roofs (2) Bidis What happens to energy having the decomposers?
(3) Leaf plates (4) Teeth cleaning agent (1) It is used by the decomposers itself.
61. Maximum number of trophic levels supported in any (2) It is reflected from the surface of earth.
ecosystem is (Delhi 2019)
(3) It is lost as heat
(1) One (2) Two
(4) It is used in natural biocomposting
(3) Three (4) Four
71. If you think chest cavity is a room, in this the diaphragm
62. BCG vaccine provide protection against (Delhi 2019) may be (Andhra Pradesh)
(1) Measles (2) T.B. (1) Floor (2) Windows
(3) Cholera (4) Small pox (3) Roof (4) Walls
EBD_7332
36
B- NTSE Stage 1 Question Bank

72. One of the following is not related to “Pea” plant: 77. Match column-I with column-II and identify the correct
(Andhra Pradesh) answer: (Karnataka 2019)
(1) It has well defined characters. Column-I Column-II
(2) It prefers self fertilization. A Insulin i Promotes tissue
(3) Presence of bisexual flowers. metabolism
(4) It is a biennial plant. B Thyroxine ii Stimulates milk
73. The scientific name of Human is “Homo sapiens”. In this secretion
the word “sapiens” represents (Andhra Pradesh)
C Antidiuretic iii Testosterone
(1) Species (2) Family
Hormone (ADH) secretion
(3) Class (4) Genera
D Oxytocin iv Promotes glucose
74. Match the item in Column – I with Column – II
utilization by the
(Andhra Pradesh)
body cells
Column – I Column – II
v Increase
A. Plants Excrete Material 1. Tears
reabsorption of
B. Animals Excrete Material 2. Saliva
water from kidney
C. Plants Secretion 3. Falling of leaves
(1) A-iv, B-i, C-v, D-ii (2) A-ii, B-iii, C-iv, D-i
D. Animals Secretion 4. Gums
(3) A-iii, B-v, C-i, D-ii (4) A-v, B-i, C-ii, D-iii
(1) A-2, B-1, C-3, D-4 (2) A-1, B-3, C-2, D-4
78. The correct route that sperm follows when it releases from
(3) A-3, B-1, C-4, D-2 (4) A-4, B-2, C-1, D-3 the testis of a mammal: (Karnataka 2019)
75. The following statements about blood vessels is/are (1) Vas deferens Epididymis Urethra
correct. (Karnataka 2019) (2) Urethra Epididymis Vas deferens
(A) Arteries have thin and less muscular walls (3) Epididymis Urethra Vas deferens
(B) Walls of veins are non-elastic (4) Epididymis Vas deferens Urethra
(C) Arteries have no valves in their inner lining 79. Coralloid root is found in (Rajasthan 2019)
(D) Veins do not collapse when empty (1) Cycas (2) Pinus
(1) A and D (2) Only D (3) Marsilia (4) Azolla
(3) B and C (4) A and C 80. From which district of Rajasthan did Chipko movement
76. The changes that are likely to happen in the plant in the begin? (Rajasthan 2019)
following picture: (Karnataka 2019) (1) Jodhpur (2) Jaipur
(3) Ajmer (4) Jaisalmer
81. The parts of large intestine are (Rajasthan 2019)
(1) Duodenum, Ileum, Colon
(2) Caecum, Colon, Rectum
(3) Duodenum, Jejunum, Ileum
(4) Jejunum, Ileum, Caecum
82. The disease caused by deficiency of Vitamin K is
(Rajasthan 2019)
(1) Haemorrhage (2) Sterility
(A) A hormone abscisic acid is synthesized at the shoot (3) Rickets (4) Scurvy
tip. 83. The enzyme, secreted in your mouth helps to digest the
(B) When light is coming from one side of the plant, auxin rice that you are having in your lunch is
diffuses towards the shady side of the plant. (West Bengal 2019)
(C) Abscisic acid (ABA) helps the cells to grow longer. (1) Salivary amylase (2) Pepsin
(D) Cells grow longer on the side of the shoot which is (3) Trypsin (4) Intestinal lipase
away from light. 84. Analogous organs are those which have
Select the correct option. (West Bengal 2019)
(1) A and B (2) B and C (1) Common origin and common functions
(3) C and D (4) B and D (2) Common origin but different functions.
(3) Similar functions but different origins.
(4) Different functions and different origins.
Solved Questions B-37
85. Passive immunity is obtained through injecting 97. If a potted plant and a dish containing potassium hydroxide
(West Bengal 2019) are covered by a sealed container, made up of glass, are
kept in sunlight for a week, what will happen
(1) Antibiotics (2) Vaccines
(3) Antigens (4) Antibodies (Chandigarh 2018)
86. Identify the wrong one. (West Bengal 2019) (1) Plant will grow taller
(1) Mollusca Pseudopodia (2) Leaf will turn yellow due to no photosynthesis
(2) Cnidaria Nematocyst (3) Leaf will turn green due to excess photosynthesis
(3) Annelida True coelome (4) Leaf will turn yellow due to no oxygen in the jar
(4) Echinodermata Water vascular system 98. Sex determination in humans is due to the presence of :
87. A flagellum is present at one end of a protozoan. It is (Chandigarh 2018)
(Delhi 2018) (1) Presence of X-chromosome in female
(1) Planaria (2) Paramecium (2) Presence of only Y-chromosome in male
(3) Hydra (4) Leishmania (3) Formation of two types of eggs by female
88. The wings of house fly and the wings of a sparrow are an (4) Formation of two types of spems by male
example of (Delhi 2018) 99. _________ respire through lungs. (Gujarat 2018)
(1) Analogous organs (2) Vestigial organs (1) Crabs (2) Lizard
(3) Respiratory organs (4) Homologous organs (3) Sepia (4) Prawns
89. Cell division in plants is promoted by : (Delhi 2018) 100. Anaerobic respiration takes place only in ________.
(1) Abscisic acid (2) Gibberllin (Gujarat 2018)
(3) Ethylene (4) Cytokinin (1) mitochondria (2) gland
90. Flight and fight hormone is: (Delhi 2018) (3) lungs (4) cytoplasm
(1) Adrenaline (2) Thyroxin 101. In the villi of ileum the absorption of lipids takes place
through______ (Gujarat 2018)
(3) Oxytocin (4) Insulin
(1) Lymph ducts (2) Lymph capillaries
91. ATP is formed by photosynthesizing plant cell by :
(3) Blood capillaries (4) Lymph vessels
(Delhi 2018)
102. Whose excessive secretion causes the body to look like
(1) Photophosphorylation gorilla? (Gujarat 2018)
(2) Oxidative Phosphorylation (1) GTH (2) PRL
(3) Substrate level phosphorylation (3) GH (4) MSH
(4) All of the above 103. An exception to cell theory is (Madhya Pradesh 2018)
92. Breathing rate in human is controlled by : (Delhi 2018) (1) Bacteria (2) Virus
(1) Thalamus (2) Hypothalamus (3) Algae (4) All
(3) Cerebellum (4) Medulla oblongata 104. Association of algae and fungi forms
(Madhya Pradesh 2018)
93. Number of male gametes in the growing pollen tube is
(1) mycomhiza (2) lichen
(Rajasthan 2018)
(3) flower (4) bio fertilizer
(1) one (2) two
105. When ATP is converted into ADP, it releases
(3) three (4) seven
(Madhya Pradesh 2018)
94. Bacterial disease is (Rajasthan 2018) (1) enzymes (2) secretions
(1) Dengue (2) Poliomyelitis (3) energy (4) hormones
(3) Tuberculosis (4) Chicken pox 106. The recessive character in pea plant in the following
95. Which of the following is not a secondary reproductive (Karnataka 2018)
organ? (Rajasthan 2018)
(1) Violet flower (2) Axillary flower
(1) Fallopian tube (2) Uterus
(3) Round seed (4) Green seed
(3) Ovary (4) Vagina
107. Haemophilia is more common in males because it is a
96. In a practical laboratory, a student while observing the slide
of tissue with the help of a microscrope, found a bunch of (Karnataka 2018)
cylindrical shaped cells having interconnections belong to (1) Recessive character carried by Y chromosome
the category of : (Chandigarh 2018)
(2) Dominant character carried by Y chromosome
(1) Adipose tissue (2) Heart muscle
(3) Dominant trait carried by X chromosome
(3) Smooth muscle (4) Skeletal muscle
(4) Recessive trait carried by X chromosome
EBD_7332
38
B- NTSE Stage 1 Question Bank

108. Identify the correct statements about blood. 117. Which of the following plants store food in their root ?
(Karnataka 2018) (Chhattisgarh 2018)
(1) Radish (2) Potato
A. Platelets are produced in the bone marrow. (3) Tomato (4) Maize
B. When haemoglobin combines with oxygen it forms 118. Reena has a burning sensation in her stomach due to
carboxyhaemoglobin. acidity. She needs to eat ________ (Chhattisgarh 2018)
(1) tomato (2) apple
C. Calcium ions play an important role in clotting of blood.
(3) tamarind (4) baking soda
D. Fibrins are formed by the conversion of fibrinogen by 119. Which of the following carry hereditary characters to the
the enzyme thrombin. off spring in the organism? (Delhi 2017)
(1) A and B only (2) B, C and D only (1) Ribosome (2) Chromosome
(3) Plasma (4) Lysosome
(3) B and D only (4) A, C and D only
120. Which organelle of the cell is called the power house of the
109. By using only one of the two strands of DNA, mRNA is cell? (Delhi 2017)
produced the process called as _______ (1) Cell - Wall (2) Nucleus
(Maharashtra 2018) (3) Mitochondria (4) Complete cell
121. Plasma membrane is made up of (Delhi 2017)
(1) transcription (2) translation (1) Protein (2) Lipid
(3) translocation (4) replacement (3) Carbohydrate (4) Both (1) and (2)
110. Which is the sequence of four whorls of flower from 122. Which of the following is the site of fertilisation in
humans? (Delhi 2017)
outside to inside? (Maharashtra 2018)
(1) Uterus (2) Oviduct
(1) Calyx Corolla Androecium Gynoecium (3) Ovary (4) Vagina
(2) Gynoecium Androecium Corolla Calyx 123. What is the time of rest in the heart? (Delhi 2017)
(3) Calyx Androecium Corolla Gynoecium (1) Never
(2) While sleeping
(4) Gynoecium Corolla Androecium Calyx (3) Between two beats
111. Identify the correct sequence for process of energy (4) While doing yogasan
production from carbohydrates. (Maharashtra 2018) 124. Lacteal present in the villi of the small intestine :
(1) Carbohydrates Glycolysis Pyruvic acid Acetyl (Delhi 2017)
CoA Krebs cycle CO2 + H2O + energy (1) Help to absorb fatty acids and glycerol
(2) Secrete enzymes for digestion
(2) Carbohydrates Glycolysis Pyruvic acid Krebs (3) Secrete hormones
cycle Acetyl CoA CO2 + H2O + energy (4) Help to absorb proteins
(3) Carbohydrates Glycolysis Acetyl CoA Pyru- 125. How primitive life might have originated on earth was
vic acid Krebs cycle CO2 + H2O + energy experimentally shown by (Delhi 2017)
(4) Carbohydrates Glycolysis Acetyl CoA Krebs (1) Urey and Miller
cycle Pyruvic acid CO2 + H2O + energy (2) Watson and Crick
(3) Oparin and Haldane
112. The source of oxygen released during photosynthesis is-
(4) Hershey and Chase
(Maharashtra 2018) 126. Bicuspid valve is present in the human heart in between
(1) Carbon dioxide (2) Water which of the following (Delhi 2017)
(3) Glucose (4) Chlorophyll (1) Right atrium and right ventricle
(2) Left atrium and left ventricle
113. In herbaceous plants ‘guttation’ takes place by
(3) Right and left atria
(Maharashtra 2018) (4) Left atrium and systemic aorta
(1) Stomata (2) Hydathodes 127. Which of the following products of light dependent phase
(3) Root hair (4) Flowers are used during the light independent phase of photosyn-
114. Digestive fluids in stomach has approximate pH of- thesis? (Delhi 2017)
(Haryana 2018) (1) RUBP and ATP (2) H2O and O2
(1) 0 (2) 2 (3) NADPH and ATP (4) ATP and O2
(3) 4 (4) 6 128. Grafting in monocot plants is not possible because they
115. Tendons & ligaments are types of tissue. have (Delhi 2017)
(Haryana 2018) (1) Parallel venation
(1) Muscular tissue (2) Epithelial tissue (2) Have only one cotyledon
(3) Nervous tissue (4) Fibrous tissue (3) Have cambium
116. Which of the following diseases is only due to external (4) Have scattered vascular bundles
causes? (Haryana 2018) 129. Haemophilia disease is linked with (Delhi 2017)
(1) Diabetes (2) Arthritis (1) Sex chromosome (2) Autosome
(3) Jaundice (4) Cataract (3) Bacteria (4) Virus
Solved Questions 39
B-

130. They primary building blocks of DNA are (Delhi 2017) 145. Animals of which phylum are pseudocoelomate?
(1) Nitrogenous base, phosphorus and ribose (Rajasthan 2017)
(2) Nitrogenous base, Sulphur and deoxyribose (1) Porifera (2) Platyhelminthes
(3) Nitrogenous base, phosphorus deoxyribose (3) Aschelminthes (4) Mollusca
(4) Nitrogenous base, sulphur and ribose 146. Raw material required for photosynthesis is ___________
131. Which of the following helps in formation of insulin?
and water. (Maharashtra 2017)
(Delhi 2017)
(1) Chloroplast (2) Sunlight
(1) Islets of Langerhans (2) Pituitary gland
(3) Thyroid gland (4) Adrenal gland (3) Nitrogen (4) Carbon-dioxide
132. Which tissue is found in fibrous covering of coconut? 147. Find the odd one out (Maharashtra 2017)
(Rajasthan 2017) (1) Uterus (2) Ovary
(1) Parenchyma (2) Collenchyma (3) Vagina (4) Testis
(3) Sclerenchyma (4) Meristematic tissue 148. The prescribed limit of sound in decibels in silent zone
133. Nucleus of the cell was discovered by during daytime is. (Maharashtra 2017)
(Rajasthan 2017) (1) 50 (2) 60
(1) Robert Hooke (2) Leeuwenhoek (3) 70 (4) 40
(3) Robert Brown (4) Virchow 149. A green house gas N2O remains for how many years in the
134. Which of the following is a plant hormone? atmosphere? (Maharashtra 2017)
(Rajasthan 2017) (1) 100 (2) 114
(1) Insulin (2) Thyroxine
(3) 104 (4) 109
(3) Cytokinin (4) Oestrogen
150. In human being, blood goes through the heart ....................
135. Plant group more sensitive to the levels of sulphur dioxide
in air is (Rajasthan 2017) times during each cycle. (Maharashtra 2017)
(1) Thallophyta (2) Lichen (1) one (2) three
(3) Pteridophyta (4) Gymnosperm (3) two (4) four
136. Examples of perennial, evergreen and woody plants are 151. Response to stimulus of touch is called ___________
(Rajasthan 2017) (Maharashtra 2017)
(1) Funaria, Marchantia (2) Marsilea, Horse-tail (1) Tropic movement
(3) Cycas, Pinus (4) Ulothrix, Spirogyra (2) Photo-tropic movement
137. Turgidity of cell is maintained by (3) Hydro-tropic movement
(Rajasthan 2017) (4) Seismonastic movement
(1) Vacuole (2) Lysosome 152. Find the odd one out (Maharashtra 2017)
(3) Plastid (4) Golgi body (1) Fragmentation (2) Regeneration
138. The substance not essential for photosynthesis is (3) Budding in Yeast (4) Budding in Hydra
(Rajasthan 2017) 153. The total no. of pairs of chromosomes in human beings are
(1) sunlight (2) chlorophyll ........................... (Maharashtra 2017)
(3) nitrogen (4) carbon dioxide (1) 22 (2) 23
139. The nature of nerve impulse is (3) 46 (4) 44
(Rajasthan 2017)
(1) chemical (2) magnetic Column A Column B
(3) electrochemical (4) electromagnetic (i) Darwin (a) heritability of acquired characteristics
140. The example of uricotelic animals is 154. (ii) Lamarck (b) inheritance
(Rajasthan 2017)
(iii) Mendel (c) natural selection
(1) fishes (2) reptiles
(3) amphibians (4) mammals (Maharashtra 2017)
141. According to Mendel in monohybrid cross the genotypic (1) (i) – (c), (ii) – (a), (iii) –(b)
ratio of F2 generation is (Rajasthan 2017) (2) (i) – (b), (ii) – (c), (iii) –(a)
(1) 3 : 1 (2) 9 : 3 : 3 : 1 (3) (i) – (b), (ii) – (a), (iii) –(c)
(3) 1 : 1 (4) 1 : 2 : 1 (4) (i) – (a), (ii) – (c), (iii) –(b)
142. Example of connective tissue is (Rajasthan 2017) 155. Which plant does not belong to group Thallophyta
(1) cartilage (2) skeletal muscles (Maharashtra 2017)
(3) skin of animals (4) nerve cells (1) Ulothrix (2) Spirogyra
143. The example of egg laying mammal is (Rajasthan 2017)
(3) Chara (4) Funaria
(1) Bat (2) Kangaroo
156. The excretory product in crystalline form of the plants
(3) Pigeon (4) Echidna
................ causes itching (Maharashtra 2017)
144. Non-communicable disease is (Rajasthan 2017)
(1) Phyroid (2) Raphide
(1) Cancer (2) AIDS
(3) Amoebiasis (4) Jaundice (3) Graphite (4) Cyanide
EBD_7332
40
B- NTSE Stage 1 Question Bank

157. From different areas of the brain which is vision area? 168. DNA (De-oxyribonucleic acid) is not present in one of the
(Maharashtra 2017) following (Jharkhand 2017)
A (1) Chloroplast
B (2) Nucleus
(3) Mitochondria
C (4) TMV (Tobacco Mosaic Virus)
D 169. Due to the discovery of one of the following in 1980, the
(1) A (2) B evolution was termed as RNA world: (Jharkhand 2017)
(3) C (4) D (1) RNA present in some viruses as genetic material
158. In female reproductive system ovaries secrete ............... hor- (2) RNA has enzymatic property
(3) RNA is found in all living cells
mone. (Maharashtra 2017)
(4) RNA is found to be associated with protein synthesis
(1) testosterone (2) estrogen 170. In plants, the developing embryo is nourished by
(3) auxin (4) thyroxine endospermic tissues its cell consist of : (Jharkhand 2017)
159. Find the odd one out (Maharashtra 2017) (1) One genome (Haploid)
(1) Adiantum (2) Equisetum (2) Two genomes (Dipioid)
(3) Selaginella (4) Riccia (3) Three genomes (Triploid)
160. Plants normally growing on sand are known as- (4) Four genomes (Tetraploid)
(Jharkhand 2017) 171. One of the following is not assoicated with gametogenesis :
(1) Lithophytes (2) Xerophytes (Jharkhand 2017)
(3) Chasmophytes (4) Psammophytes (1) Formation of Ova
161. Our skin becomes dark in colour when exposed to excess of (2) Formation of sperm
sunlight. It is due to the presence of : (3) Change of spermatids to spermatozoa
(Jharkhand 2017) (4) Release of ova
(1) Carotene (2) Melanin 172. The part of biosphere dominated by human beings is known
(3) Flavoxanthin (4) Haemotoxylene as : (Jharkhand 2017)
162. Famous scientist Carolus Linnaeus is associated with one (1) Troposphere
the following : (Jharkhand 2017) (2) Hemisphere
(1) Plant Classification (3) Stratosphere
(2) Binomial Nomenclature (4) Noosphere
(3) Identification of plants 173. The excretory organs in Earthworm is known as -
(4) Identification of Animals (Jharkhand 2017)
163. Ozone hole or hole in the ozone layer in the atmosphere (1) Malphigian cells
(2) Renal cells
refers to : (Jharkhand 2017)
(3) Nephridia
(1) Development of a hole in the Ozone layer (4) Flame cells
(2) Decrease in the Ozone layer in troposphere 174. Which one of the following is made of only one type of
(3) Decrease in the Ozone layer in stratosphere macromolecule? (Haryana 2017)
(4) All of above (1) Virus (2) Plasmid
164. In living cells synthesis of ribonucleic acid (RNA) (3) Nucleosome (4) Ribosome
takes place in : (Jharkhand 2017) 175. Among carbohydrates, lipids, proteins and ATP, the relative
(1) Cytoplasm energy yield in kcal/gm is best represented by:
(2) Nucleus (Haryana 2017)
(3) Golgi body (1) Lipids > Carbohydrates > ATP
(4) Nephron (2) ATP > Lipids > Protein
165. Deficiency of one of the under mentioned vitamins causes (3) Lipids > ATP > Carbohydrates
cracking of lips in human beings : (Jharkhand 2017) (4) Lipids > Proteins > ATP
(1) Vitamin A (2) Vitamin B2 176. The sub units of ribosomes in cells of nephron of mouse are:
(3) Vitamin K (4) Vitamin C (Haryana 2017)
166. Insectivorous plants grow only on such soils which are de- (1) 50S & 30S (2) 40S & 23S
ficient in : (Jharkhand 2017) (3) 70S & 16S (4) 60S & 40S
(1) Calcium (2) Nitrogen 177. Involuntary muscles are not found in : (Haryana 2017)
(3) Magnesium (4) Phosphorus (1) Iris (2) Bronchi of lung
167. What will happen to the body of an adult human being if his (3) Tongue (4) Heart
spleen is removed : (Jharkhand 2017) 178. Different microorganisms taking part in nitrogen cycle are:
(1) RBC production will be reduced (Haryana 2017)
(i) Rhizobium in roots (ii) Ammonifying bacteria
(2) Antibodies production will be less
(iii) Nitrifying bacteria (iv) Denitrifying bacteria
(3) WBC production will be less
Which of them strictly work under anaerobic conditions ?
(4) Filtration of dead RBCs would not be possible (1) only iv (2) i & iv
(3) i, ii & iv (4) ii & iv
Solved Questions B-41
179. The following pictures were drawn by a student to show 189. Growth of pollen tube in the style towards the ovule in
different stages of binary fission: (Haryana 2017) plants is an example of (Delhi 2016)
(1) Geotropism (2) Hydrotropism
(3) Phototropism (4) Chemotropism
190. The common passage of urine and sperm in human male is
(Delhi 2016)
(1) Seminal vesicle (2) Ureter
(3) Vas deferens (4) Urethra
(i) (ii) (iii) (iv) 191. Out of the following, which enzyme is active in acidic medium
The correct sequence of these figures is: (Delhi 2016)
(1) iii, ii, iv, i (2) iii, iv, ii, i (1) Pepsin (2) Trypsin
(3) ii, iii, iv, i (4) iv, iii, ii, i (3) Lipase (4) Amylase
180. Which of the following is not strictly considered as a part of 192. Bowman capsule is found in (Delhi 2016)
neuron? (Haryana 2017) (1) Small intestine (2) Kidneys
(1) Dendrite (2) Myelin sheath (3) Heart (4) Brain
(3) Axon (4) Cell body 193. In a cell which cell organelle other than nucleus contains
181. Which of the following statement about autotrophs is in- DNA? (Rajasthan 2016)
correct? (Haryana 2017) (1) Lysosome (2) Golgi bodies
(1) They synthesize carbohydrates from carbon dioxide (3) Endoplasmic reticulum (4) Mitochondria
and water. 194. Which plant group is called amphibious plants?
(2) They store carbohydrate in the form of starch. (Rajasthan 2016)
(3) They convert water & CO2 into carbohydrate only in (1) Algae (2) Fungi
the absence of light. (3) Bryophyta (4) Pteridophyta
(4) They constitute first trophic level in the food chain. 195. Enzyme responsible for digestion of protein is
182. Correct pathway of blood in circulatory system is: (Uttar Pradesh 2016)
(Haryana 2017) (1) ptylin (2) pepsin
(1) atria ventricles artery veins (3) amylopsin (4) steapsin
(2) ventricles atria veins arteries 196. Ethylene hormone is found in the form of
(3) ventricles veins atria arteries (Uttar Pradesh 2016)
(4) atria arteries ventricles veins (1) gas (2) liquid
183. Which of the following is essential for formation of (3) solid (4) all of the above
thyroxine hormone in the thyroid gland? (Haryana 2017) 197. Calciferol is (Uttar Pradesh 2016)
(1) Sodium (2) Chloride (1) vitamin A (2) vitamin B
(3) Potassium (4) Iodine (3) vitamin C (4) vitamin D
184. In a given food chain if frog has 100 J of energy then the 198. Which one of the following does not contain any enzyme?
(Odisha 2016)
energy available with plants and snake respectively will be:
(1) Bile (2) Gastric Juice
(Haryana 2017)
(3) Saliva (4) Pancreatic Juice
(1) 1000 J and 10 J (2) 10000 J and 10 J
199. Mark the tissue in which the starch is stored in the body of
(3) 10 J and 1000 J (4) 1000 J and 100 J
plants (Odisha 2016)
185. Characters that are transmitted form parents to offspring
(1) Spongy parenchyma (2) Aerenchyma
during reproduction show: (Haryana 2017)
(3) Apical meristem (4) Stomata
(1) Only similarites with parents
200. Which one of the following tissues contains stone cells?
(2) Only variations with parents
(Odisha 2016)
(3) Both similarities and variation with parents
(1) Parenchyma (2) Collenchyma
(4) Neither similarities nor variation with parents
186. Rajiv was absent in class due to muscle pain which he claims (3) Sclerenchyma (4) Tracheids
was due to excess of physical (Haryana 2017) 201. Which of the following is the correct scientific name of
man? (Odisha 2016)
(1) Formation of Pyruvic Acid
(1) Homo Sapiens (2) Homo sapien
(2) Formation of Acetic Acid
(3) Homosapien (4) Homo sapiens
(3) Formation of Lactic Acid
202. Which of the following harmful products is not produced
(4) Formation of Hydrochloric Acid
in the biochemical reactions of the cell of living organisms?
187. Which of the following constitutes a good food chain?
(Maharashtra 2016)
(Haryana 2017)
(1) Urea (2) Uric acid
(1) Grass, Wheat, Mango (2) Grass, Goat, Lion
(3) Ammonia (4) Lymph
(3) Goat, Cow, Elephant (4) Grass, Fish, Goat
203. Identify a fish who breathes air through its lungs:
188. The oxygen rich blood rich from lungs comes to the heart in
(Maharashtra 2016)
(Delhi 2016)
(1) Lungfish (2) Rohu
(1) Left atrium (2) Right atrium
(3) Dogfish (4) Siting Ray
(3) Right ventricle (4) Left ventricle
EBD_7332
42
B- NTSE Stage 1 Question Bank

204. A pea plant with yellow and round seeds (YYRR) is crossed (4) Digestion of nucleic acids
with a pea plant having green and wrinkled (yyrr) seeds 212. The lipids and proteins essential for the formation of cell
then in F2 generation of this dihybrid cross 320 plants are membrane are synthesized in (Delhi 2015)
produced. Out of which plants have same phenotypic (1) Golgi bodies (2) Mitochondria
characters. Identify this phenotype. (Maharashtra 2016) (3) Endoplasmic reticulum (4) Lysosomes
(1) Yellow and wrinkled seeds 213. The part of brain which controls the balance and posture of
(2) Yellow and round seeds the body is (Delhi 2015)
(3) Green and round seeds 1. Cerebellum 2. Cerebrum 3. Pons 4. Medulla
(4) Green and wrinkled seeds 214. Uric acid is the main excretory product of: (Delhi 2015)
205. 'Earthworm' a farmer’s friend belongs to .......... phylum. (A) Insects (B) Birds
(1) Arthropoda (2) Echinodermata (C) Terrestrial reptiles (D) Human being
(3) Mollusca (4) Annelida (1) A, B and C are correct
206. Read the following statements: (Karnataka 2016) (2) A and B are only correct
A. It forms a thick, though barrier and protects the (3) Only B and D are correct
underlying tissues in the skin. (4) Only A and C are correct
B. When it is present in sense organs contain receptor 215. Which organism will break down the food material into
cells. simple substances outside the body and then absorb it?
C. It also helps in absorption of nutrients. (Maharashtra 2015)
D. When it is present in glands helps in secretion. (1) Mushroom (2) Cuscuta
Which one of the following represent the above mentioned (3) Ticks (4) worms Tape
characteristics? 216. The vegetative reproduction in Bryophyllum takes place
(1) Epithelial tissue (2) Connective tissue through which organ? (Maharashtra 2015)
(1) Root (2) Stem (3) Leaf (4) Seed
(3) Muscular tissue (4) Nerve tissue
217. Increase in the height of the plant is due to
207. Colour blindness is more common in men than in women
(Jharkhand 2015)
due to (Karnataka 2016)
(1) Auxins (2) Cytokinins
(1) Dominant genes of such trails are found on 'Y' (3) Gibberellins (4) Ethylene
chromosome 218. Ginger is (Jharkhand 2015)
(2) Dominant genes of such traits are found on 'X' (1) Root (2) Yeast
chromosome (3) Plasmodium (4) Leishmania
(3) Recessive genes of such traits which occur on the 'X' 219. The liver stores food in the form of (Jharkhand 2015)
chromosome (1) Glucose (2) Glycogen
(4) Recessive genes of such traits which occur on the 'Y' (3) Albumen (4) ATP
chromosome 220. Which of the following is a plant harmone?
208. The crop that would require minimum quantity of urea of (Jharkhand 2015)
NPK for its growth. (Karnataka 2016) (1) Insulin (2) Cytokinin
(1) Blackgram (2) Paddy (3) Thyroxine (4) Oestrogen
(3) Sugarcane (4) Groundnut 221. The centre of sense of smell in brain is (Jharkhand 2015)
209. Read the following statements and select the correct option. (1) Midbrain (2) Olfactrory lobes
(Karnataka 2016) (3) Cerebellum (4) Cerebrum
A. Wind pollinating flowers need to produce more 222. The cell organelle storing substances like starch, oil and
quantity of pollen grains proteins is (Rajasthan 2015)
B. Seeds from cross pollinated flowers produce weaker (1) Vacuole (2) Lysosome
and less healthy plants. (3) Plastid (4) Golgi body
(1) 'A' is false 'B' is true 223. The hormone present in higher concentration in fruits and
(2) 'A' is true 'B' is false seeds is (Rajasthan 2015)
(3) Both 'A' and 'B' are true (1) Auxin (2) Gibberellin
(4) Neither 'A' nor 'B' are true (3) Cytokinin (4) Ethylene.
210. Protein catalysts of chemical reactions in biological system. 224. Presence of which of the following compounds causes algal
(Karnataka 2016) bloom. (Odisha 2015)
(1) Hormones (1) Carbonate + Nitrate
(2) Vitamins
(2) Sulphate + Phosphate
(3) Enzymes
(3) Phosphate + Nitrate
(4) Both hormones and enzymes
(4) Sulphate + Nitrate
211. If pepsin is lacking in gastric juice, then the event in the
225. Which one of the following diseases is water borne?
stomach will be affected: (Delhi 2015)
(1) Digestion of starch into sugars (Odisha 2015)
(2) Proteins break down into peptides (1) Hepatitis B (2) Hepatitis C
(3) Breaking of fats into glycerol and fatty acids (3) Hepatitis D (4) Hepatitis E
Solved Questions B-43
226. A cell will plasmolyse, if it is placed in: (Delhi 2014) 239. Fossils are found in ............... (Madhya Pradesh 2014)
(1) Hypertonic solution (1) Igneous rocks (2) Sedimentary rocks
(2) Hypotonic solution (3) Metamorphic rocks (4) All of the above
(3) Isotonic solution 240. A person is excreting about10 litres of urine per day. Which
(4) Concentration of water molecules does not matter of the following endocrine gland is responsible for this?
227. Phototropic and geotropic response of a plant is under (Odisha 2014)
control of following hormone: (Delhi 2014) (1) Pituitary (2) Thyroid
(3) Parathyroid (4) Adrenal
(1) Auxin (2) Gibberlin
241. Which one of the following does not enter in to the Calvin
(3) Cytokinin (4) Ethylene cycle? (Odisha 2014)
228. The following blood vessel does not contain deoxygenated (1) Carbon dioxide (2) Enzyme
blood: (Delhi 2014) (3) ATP (4) NADP
(1) Pulmonary artery (2) Vena Cava 242. The cell organelle in which hydrolytic enzymes are stored is:
(3) Hepatic vein (4) Pulmonary Vein (Uttar Pradesh 2013)
229. During lack of oxygen in tissues of our body, the pyruvate is (1) Plastid (2) Mitochondria
converted into lactic acid in: (Delhi 2014) (3) Centrosome (4) Lysosome
(1) Mitochondria (2) Nucleus 243. Choose the incorrect statement about insulin:
(3) Cytoplasm (4) Ribosome (Uttar Pradesh 2013)
230. In Xerophytes, the rate of water loss get reduced due to (1) Deficiency of insulin leads to diabetes.
covering of epidermis by (Maharashtra 2014) (2) It regulates the growth and development of the body.
(1) Cutin (2) Suberin (3) It controls sugar level in the blood.
(3) Lignin (4) Gum (4) It is produced from the pancreas.
231. Given below are the pairs of disease and causal pathogen. 244. The animal which belongs to class pisces is:
Which one of these is not a matching pair? (Uttar Pradesh 2013)
(1) Silver fish (2) Jelly fish
(Maharashtra 2014)
(3) Star fish (4) Dog fish
(1) Kala azar – Leishmania
245. Most of the plants absorb nitrogen in the form of:
(2) Sleeping sickness – Trypanosoma
(Haryana 2013)
(3) Malaria – Salmonella (1) Uric acid (2) Amino acids
(4) Acne – Staphylococci (3) Atmospheric nitrogen (4) Nitrates and Nitrites
232. The accumulation of non-biodegradable substances in a 246. In a synapse, the chemical signal is transmitted from:
food chain in increasing amount at each higher trophic level (Haryana 2013)
is known as (Maharashtra 2014) (1) Axonal end of a neuron to dendritic end of another
(1) Accumulation (2) Eutrophication neuron.
(3) Pollution (4) Bio-magnification (2) Axonal end to the cell body of the same neuron.
233. Kidneys do not perform the function of: (3) Cell body to axonal end of the same neuron.
(Maharashtra 2014) (4) Dendritic end of one neuron to axonal end of another
(1) Regulation of blood pressure neuron.
(2) Filtration of blood 247. Ribosomes are the centre for : (Madhya Pradesh 2013)
(3) Regulation of ions concentration (1) Respiration (2) Protein synthesis
(4) Secretion of antibodies (3) Photosynthesis (4) Fat synthesis
234. Phototropic and geotropic response of a plant is under 248. Binomial nomenclature was introduced by :
control of following hormone: (Maharashtra 2014) (Madhya Pradesh 2013)
(1) Auxin (2) Gibberlin (1) John Ray (2) Aristotle
(3) Cytokinin (4) Ethylene (3) A.P. DeCandolle (4) Carolus Linnaeus
235. In simple organisms, exchange of gases and excretion occur 249. Oxygen present in the glucose molecule formed during
through (Jharkhand 2014) photosynthesis is obtained from: (Madhya Pradesh 2013)
(1) Osmosis (2) Diffusion (1) Water molecule
(3) Imbibition (4) All of the above (2) Carbondioxide molecule
236. The site of photosynthesis in plant is (Jharkhand 2014) (3) Chlorophyll
(1) Mitochondria (2) Chloroplast (4) Oxygen in air
(2) Leucoplast (4) Dictyosomes 250. The percentage of oxygen in air is :
237. Which scientist discovered "Tricarboxylic acid"? (Madhya Pradesh 2013)
(Madhya Pradesh 2014) (1) 78% (2) 0.03%
(1) Krab (2) Watson and Crick (3) 21% (4) 80%
(3) Sanger (4) Edison 251. Cell organelle which differentiates plant cell from animal cell
238. Which nitrogen base is absent in D.NA? is : (Rajasthan 2012, 2013)
(Madhya Pradesh 2014) (1) Cell Membrane (2) Plastids
(1) Adenine (2) Guanine (3) Nucleouls (4) Vacuoles
(3) Uracil (4) Cytosine
EBD_7332
44
B- NTSE Stage 1 Question Bank

252. Blood is a type of connective tissue, which has (1) endoplasmic reticulum (2) mitochondria
(Rajasthan 2012, 2013) (3) Golgi apparatus (4) Lysosome
(1) R.B.C (2) W.B.C 267. Normally in a healthy adult the daily initial filtrate in the
(3) Platlets (4) All of the above kidneys is : (Punjab 2012)
253. Bile Juice is secreted from (Rajasthan 2012, 2013) (1) 18 L (2) 1.8 L (3) 180 L (4) 9 L
(1) Salivary glands (2) Intestinal glands 268. Which part of the heart receives deoxygenated blood?
(3) Stomach (4) Liver (Punjab 2012)
254. When acidity in stomach increases, the medicine generally (1) Right atrium (2) Right ventricle
used is (Rajasthan 2012, 2013) (3) Left atrium (4) Left ventricle
(1) Sodium bicarbonate (2) Sodium Carbonate 269. Choose the right from the following. (Punjab 2012)
(3) Ammonium Carbonate (4) Ammonium bicarbonate (i) In light, hormone auxin, helps the cells to grow longer
255. Planaria is kept in which group (Rajasthan 2013) in plants.
(1) Coelenterata (2) Platyhelminthes (ii) Plant hormone gibberellins help in growth of a stem.
(3) Nematoda (4) Annelida
(iii) Cytokininis inhibits cell division.
256. Which of the following is an example of Bryophyte
(iv) Abscisic acid promotes growth in plants.
(Rajasthan 2013)
(1) (i) and (iii) are correct. (2) (ii) and (iv) are correct.
(1) Moss (2) Fern
(3) Pinus (4) Algae (3) (i) and (ii) are correct. (4) (i) and (iv) are correct.
257. Dissimilarity found in Aves and Mammalia is 270. The experiment conducted by Stanley L. Miller and Harold
(Rajasthan 2013) C. Urey in 1953 to show how organic molecules arise in
(1) Warm Blooded Animal (2) Lay eggs nature, they assembled an atmosphere consisted of :
(3) Breathe through Lungs (4) Four chambered Heart (1) ammonia, methane and oxygen. (Punjab 2012)
258. Substances necessary for autotrophic nutrition are (2) ammonia, hydrogen sulphide and oxygen.
(Rajasthan 2013) (3) ammonia, hydrogen sulphide and methane.
(1) CO2 and H2O (2) Chlorophyll (4) methane, hydrogen sulphide and oxygen.
(3) Sun light (4) All of the above 271. Sperm formation requires ____________ temperature as in
259. Blood Pressure is measured with an instrument called the normal body temperature. (Punjab 2012)
(Rajasthan 2012, 2013) (1) same (2) high
(1) Thermometer (2) Stethoscope (3) low (4) not sure
(3) Sphygmomanometer (4) Clinical Thermometer 272. Typhoid is caused by : (Madhya Pradesh 2012)
260. Wings of birds and insects are (Delhi 2012) (1) Streptococcus (2) Salmonella
(1) Vestigial organs (2) Homologous organs (3) Giardia (4) Mycobacterium
(3) Paralogous organs (4) Analogous organs 273. In the year 1984, the Bhopal gas tragedy was caused by the
261. Neurons have a unique property that makes them to leakage of : (Madhya Pradesh 2012)
communicate with other cells via: (Delhi 2012) (1) Carbon monoxide (2) Methyl isocyanate
(1) Nerve cords (2) Glial cells (3) Nitrogen oxide (4) Sulphur oxide
(3) Synapses (4) Schwann cells 274. Botanical name of amla is : (Madhya Pradesh 2012)
262. Cramps in the leg muscle after running a long distance are (1) Medicago sativa (2) Emblica officinalis
because of (Delhi 2012)
(3) Zingiber officinale (4) Ocimum sanctum
(1) Build up of acetic acid (2) Build up of oxalic acid
275. Hormone which stimulate initiation of flowering process is :
(3) Build up of lactic acid (4) Build up of Pyruvic acid
(Madhya Pradesh 2012)
263. The enzyme pepsin is secreted by (Delhi 2012)
(1) Gibberellin (2) Ethylene
(1) Inner lining of oesophagus
(3) Vernalin (4) Florigen
(2) Gastric lining of stomach
276. What happens when a cell placed in hypertonic solution ?
(3) Inner lining of duodenum
(Madhya Pradesh 2012)
(4) Gall bladder
(1) Endosmosis (2) Exosmosis
264. Pick the odd one out (Delhi 2012)
(3) Deplasmolysis (4) Imbibition
(1) Down syndrome (2) Haemophilia
277. Organisms lacking nuclear membrane and cell organelles is
(3) Malaria (4) Phenylketonuria called as : (Madhya Pradesh 2012)
265. Which of the following class of animals has coelomic cavity (1) Prokaryotes (2) Eukaryotes
filled with blood? (Punjab 2012)
(3) Protozoa (4) Virus
(1) Nematoda (2) Annelida
278. The plants in which vegetative propagation is found, are-
(3) Arthropoda (4) Mollusca
(Rajasthan 2012)
266. Lipids and proteins constituting the cell membrane are
(1) Bryophyllum (2) Sugarcane
synthesized at : (Punjab 2012)
(3) Rose (4) All of the above
Solved Questions B-45
279. Which one of the following is secreted by pituitary gland? (1) Palisade tissue (2) Phloem tissue
(Odisha 2012) (3) Spongy tissue (4) Xylem tissue
(1) Insulin (2) Estrogen 291. The cell organelle pertaining to energy release process is
(3) Adrenaline (4) Growth hormone (Andhra Pradesh 2012)
280. Which one of the following first receives the pollen during (1) Lysosome (2) Chloroplast
fertilization? (Odisha 2012) (3) Mitochondria (4) Endoplasmic reticulum
(1) Stigma (2) Style 292. The site of intelligence, thinking and judgment in human
(3) Pollen tube (4) Female gamete brain is —— (Andhra Pradesh 2012)
281. Which two of the following animals belong to the same (1) Cerebrum (2) Medulla oblongata
Phylum? (Odisha 2012) (3) Cerebellum (4) Mid brain
(1) Starfish and Nereis (2) Antedon and Starfish 293. An example for essential fatty acids is
(3) Antedon and Chiton (4) Nereis and Chiton (Andhra Pradesh 2012)
282. Which of the following two are only applicable for birds? (1) Glutamic acid (2) Aspartic acid
(Odisha 2012) (3) Linoleic acid (4) Tartaric acid
(1) Warm blooded and four chambered heart 294. The cell-division which is also known as reduction
(2) Feather and breathing by lungs cell-division is …. (Andhra Pradesh 2012)
(3) Warm blooded and egg laying (1) Fission (2) Meiosis
(4) Egg laying and with Feather (3) Mitosis (4) Amitosis
283. Which of the following is not a foreign breed cow? 295. In glycolysis, glucose molecule is converted into which
(1) Sahiwal (2) Holstein (Odisha 2012) substance? (Maharashtra 2012)
(3) Brown swiss (4) Jersy (1) Lactic acid (2) Ethanol
284. Read the following two statements and choose the correct (3) Pyruvate (4) Amino acid
answer. (Odisha 2012) 296. This part of alimentary canal absorbs maximum amount of
(i) Stomata regulates the body temperature in plants. water and minerals : (Maharashtra 2012)
(ii) Stomata helps in the absorption of minerals from the (1) Small intestine (2) Large intestine
soil. (3) Stomach (4) Oesophagus
(1) (i) is true but (ii) is false 297. Food becomes ................. due to bile juice.
(2) (i) is false but (ii) is true (1) Acidic (Maharashtra 2012)
(3) Both (i) and (ii) are true (2) Alkaline
(4) Both (i) and (ii) are false (3) Neutral
285. Chromosome consists of (West Bengal 2012) (4) First neutral and then acidic
(1) DNA only 298. Which is the hormone that brings about changes in girls
(2) DNA and histone protein during puberty? (Maharashtra 2012)
(3) DNA, histone protein and non histones proteins (1) Estrogen (2) Progesterone
(4) DNA and non–histones protein (3) Testosterone (4) Thyroxine
286. Which one is anti transpirant hormones? 299. How many pairs of autosomes are seen in human beings?
(West Bengal 2012) (1) 23 (2) 22 (Maharashtra 2012)
(1) Auxin (2) Cytokinin
(3) 46 (4) 44
(3) Abscisic Acid (4) Gibberellin
300. Variation combined with geographical isolation may result in:
287. Which one is the most poisonous excretory substance
(Karnataka 2012)
(West Bengal 2012)
(1) Uric Acid (2) Urea (1) Mutation (2) Speciation
(3) Ammonia (4) Hippuric Acid (3) Fossilisation (4) Genetic drift
288. Which Vector Spread the yellow fever disease? 301. In an experiment where two potted plants kept in a dark
(West Bengal 2012) room are used to demonstrate that carbon dioxide is essential
(1) Musca sp (2) Anopheles sp for photosynthesis, potassium hydroxide is used because it:
(3) Culex sp (4) Aedes aegypti (1) Releases oxygen (Karnataka 2012)
289. Which of the following is not directly formed from DNA (2) Absorbs carbon dioxide
(West Bengal 2012) (3) Releases carbon dioxide
(1) DNA (2) Protein (4) Absorbs oxygen
(3) m RNA (4) t RNA 302. Enzyme linked immuno sorbent assay test is used to detect
290. The product of Photosynthesis is transported from source (1) HBV (2) TMV (Karnataka 2012)
of production to the storage organs through (3) HIV (4) HCV
(Andhra Pradesh 2012)
EBD_7332
46
B- NTSE Stage 1 Question Bank

SECTION 4. MATHEMATICS
1. P, R and S are three persons with ages 26 years, 27 years and 1
28 years respectively. In what ratio must they invest money (iv) The probability of the number that is (s)
at 10% per annum compounded yearly so that each gets 16
same sum at the age of their retirement? on the card drawn is a composite number.
(AP_FIITJEE 2020-21) (1) (i)-(s), (ii)-(r), (iii)-(q), (iv)-(p)
(1) 100 : 121 : 132 (2) 100 : 110 : 121 (2) (i)-(s), (ii)-(q), (iii)-(p), (iv)-(r)
(3) 80 : 100 : 99 (4) 89 : 95 : 100 (3) (i)-(p), (ii)-(q), (iii)-(r), (iv)-(s)
2. If the mean of 28, 34, 41, 23, 45, 18, 21 is 'x', then the value of (4) (i)-(r), (ii)-(p), (iii)-(s), (iv)-(q)
2 – logx2 – logx3 – logx5 is ______ (AP_FIITJEE 2020-21)
7. Three numbers prime to each other are such that the product
(1) 1 (2) –1 of the first two is 437 and the product of the last two is 551.
1 The sum of the numbers is _______ (Bihar 2020-21)
(3) 0 (4)
2 (1) 59 (2) 63
(3) 69 (4) 71
3 x 2 (5 x ) x2 x 3
3. If x 5, then the value of ? 8. If the sum of the remainders obtained by dividing each of
2 2
x x x 3 x (5 x )
x3 8 x 2 3kx 7 and 2 x 3 kx 2 5 x 6 by x – 1 is 9 then
(AP_FIITJEE 2020-21) k = _______ (Bihar 2020-21)
(1) 2 (2) 6 (1) 0 (2) 2
5 9 (3) 3 (4) 5
(3) (4)
2 2
9. If the equations x 2 bx a 0 and x 2 ax b 0, (a b)
4. The average age of students of a class is 15.8 years. The have equal roots, then a + b = ______
average age of boys in the class is 16.4 years and that of the
girls is 15.4 years. The ratio of the number of boys to the (Bihar 2020-21)
number of girls in the class is _____ (AP_FIITJEE 2020-21) (1) –1 (2) 0
(1) 1 : 2 (2) 2 : 3 (3) 1 (4) None of these
(3) 3 : 4 (4) 3 : 5 10. In a right-angled triangle, the difference of the hypotenuse
x y x and the base is 2 cm. The hypotenuse exceeds twice the
5. If a, then the value height by 1 cm. The base of the triangle is ______ cm.
2x y z x 2 y z x y 2 z
of a = _______ (Given x + y + z 0) (AP_FIITJEE 2020-21) (Bihar 2020-21)
(1) 8 (2) 15
1 1
(1) (2) (3) 17 (4) 21
3 4
1 1
1 1 11. If in an A.P., the pth term and the qth term , then
(3) (4) q p
8 2
6. "Cards marked with numbers 13, 14, 15, ......, 60 are placed in the pqth term (Bihar 2020-21)
a box and mixed thoroughly. One card is drawn at random (1) –1 (2) 0
from the box." Read the information carefully and match the (3) 1 (4) None of these
following : (AP_FIITJEE 2020-
2 2
21) 12. If x 2sin , y 2cos for all real values of , then
1 (Bihar 2020-21)
(i) The probability of the number that is (p)
4
(1) x + y = 1 (2) x + y = 2
on the card drawn is divisible by 5.
36 (3) x y 2 2 (4) x y 2 2
(ii) The probability of the number that is (q) 13. If ABC is an equilateral triangle such that A(2, 2) and centroid
48
one the card drawn is a prime of the triangle is (–2, 2) then find the length of its side.
(Delhi 2020-21)
5
(iii) The probability of the number that is (r) (1) 4 units (2) 6 units
24
(3) 4 3 units (4) 9 units
on the card drawn is a multiple of 19.
Solved Questions B-47
14. In a ABC, AX bisects BC and AX is also the angle bisector (1) 10 (2) 30
of angle A. If AB = 12 cm, BX = 3 cm, then what is the area of (3) 40 (4) 60
ABC. (Delhi 2020-21) 21. Four circular plates of same size having diameter 'a' units
(1) 9 cm2 (2) 2 cm2 each are placed on the square mat such that each plate
touches externally the other two as shown in the figure. The
(3) 9 5 cm2 (4) 9 15 cm2 area of the shaded region is : (Karnataka 2020-21)
15. A 'p' m long wire is cut into two pieces one of which is bent
into a circle and the other into a square enclosing the circle.
What is the radius (in meter) of the circle? (Delhi 2020-21)
p 2p
(1) (2)
4 4
p 2p
(3) (4)
2 8 4 a2
(1) (4 ) square units
16. If (x + k) is a common factor of (x2 + px + q) and 4
(x2 + lx + m), then the value of k is : (Delhi 2020-21) a2
(2) (4 ) square units
(1) l p (2) m q 4

l p m q
(3) 4a 2 (4 ) square units
(3) (4)
m q l p (4) 4a 2 (4 ) square units
17. As a result of 40% hike in the price of rice per kg., a person is 22. The value of 'p' for which the pair of linear equations
able to purchase 10 kg less rice for Rs. 1400. What was the px 3 y ( p 3) 0 and 12 x py p 0 has infinitely
original price of rice per kg? (Delhi 2020-21) many solutions is : (Karnataka 2020-21)
(1) Rs. 50 (2) Rs. 60 (1) 0 (2) –6
(3) Rs. 40 (4) Rs. 30 1
18. One box contains four cards numbered 1, 3, 5 and 7 and (3) (4) 6
another box contains four cards numbered as 2, 4, 6 and 8. 6
One card is drawn from each box at random. The probability 23. 10th term of an AP is 16 and 16th term of it is 10. The common
that the product of the numbers so drawn is more than 14 is: difference and the 26th term of the AP are respectively.
(Karnataka 2020-21)
(Delhi 2020-21)
(1) 1 and 0 (2) –1 and 0
1 7 (3) 0 and 1 (4) 0 and –1
(1) (2)
2 10 24. The angles of depression of two ships as observed from the
top of a 80 m high light house from the sea-level are 30° and
3 5 45°. If two ships are on the same side of the light house and
(3) (4)
8 8 all the three are along a straight line, then distance between
19. If sin A sin 2 A 1 (A is acute angle) then the value of the two ships is : (Karnataka 2020-21)
cos 2 A cos 4 A is: (Karnataka 2020- (1) 80 m (2) 80( 3 1) m
21)
(1) 0 (2) 1 (3) 80( 3 1) m (4) 80 3 m
(3) 2 (4) 3 25. A storm broke a tree and the tree top rested 13 6 m from
20. The cumulative frequency curve for a distribution is the base of the making an angle 45° with the horizontal. Find
represented in the graph. The median for the data is : the height of the tree (Maharashtra 2020-21)
(Karnataka 2020-21)
(1) 26 6 m (2) 13 3(2 2) m
60
Cumulative Frequency

(3) 26 3 m (4) 26 2 13 3 m
50
3
40 Less than ogive 26. If sin + cos = then sin cos = ?
2
30 (Maharashtra 2020-21)
More than ogive
20 2
(1) 1 (2)
10 3

0 10 20 5
30 40 50 60 70 (3) (4) 0
8
Weight (in kg)
EBD_7332
48
B- NTSE Stage 1 Question Bank

27. The sum of the 3rd and 8th term of an A.P. is 7 and the sum of
1 2
the 7th term and 14th term is –3. Find the 10th term of the A.P. (1) (2)
(Maharashtra 2020-21) 2 3 3
(1) –1 (2) +1
1
(3) 0 (4) 2 (3) (4) 2 3
3
28. A bag contains 5 red and some blue balls. One ball is taken
out of the bag a random. The probability that the balls taken 35. Following are the points obtained by a Kabaddi team in
out is blue is double of the probability that the ball drawn is various matches. (Rajasthan 2020-21)
red. Find the total number of balls in the bag. 17, 2, 7, 27, 15, 5, 14, 19, 10, 24, 9, 8, 6, 18, 28
(Maharashtra 2020-21) the median of the points obtained by the team will be -
(1) 15 (2) 10
(Rajasthan 2020-21)
(3) 5 (4) 20
(1) 14 (2) 16
29. If a + b = 2 3, and ab = 3, then a 4 + b 4 = ? (3) 15 (4) 17
(Maharashtra 2020-21) 36. Discriminant of the quadratic equation 3 3 x 2 + 7 x + 3 = 0
(1) 14 (2) 16 will be (Rajasthan 2020-21)
(3) 18 (4) 20
(1) 49 (2) 13
30. PQRS is a square, PQ = 7 3 cm with centre R and radius (3) 36 (4) 62
RS, sector R QAS is drawn. Find the area of the shaded portion. 37. In a party of 100 people, 60 had Vanilla ice-cream and 40 had
(Maharashtra 2020-21) chocolate ice-cream. 10 people did not have any ice-cream.
How many had only chocolate ice-cream?
P S (Tamil Nadu 2020-21)
(1) 10 (2) 0
A (3) 20 (4) 30

38. A B
28°
E
Q R
(1) 29.5 cm2 (2) 17.5 cm2 y
(3) 23.7 cm2 (4) 31.5 cm2 40°
C D
31. Numerator of a fraction is 3 less than its denominator. If 4 is In the given figure, AB is parallel to CD. The angle y is :
added to the numerator and 2 is subtracted from the
denominator then the numerator becomes 2 times of the (Tamil Nadu 2020-21)
denominator. What will be the fraction? (1) 68° (2) 34°
(Rajasthan 2020-21) (3) 112° (4) 56°
39. If the total surface area of a cube is 96 cm2, then its volume
5 4 is : (Tamil Nadu 2020-21)
(1) (2)
2 5 (1) 64 cm3 (2) 512 cm3
2 5 (3) 8 cm3 (4) 27 cm3
(3) (4)
5 4 40. The probability of choosing a vowel in the word 'EDUCATION'
32. The measures of three angles of a triangle are in ratio is : (Tamil Nadu 2020-21)
3 : 4 : 5, then the measure of its greatest angle is - 3 4
(Rajasthan 2020-21) (1) (2)
9 9
(1) 55° (2) 65°
(3) 80° (4) 75° 5 7
(3) (4)
33. Each face of a cube is 144 cm2. If cube is cut by a plane, 9 9
parallel to it's base in two equal parts, then the total surface 41. A girl of height 100 cm is walking away from the base of a
area of both the parts will be - (Rajasthan 2020-21) lamp post at a speed of 1.9 m/sec. If the lamp is 5 m above the
(1) 1152 cm2 (2) 1052 cm2 ground, the length of the shadow after 4 sec. is :
(3) 288 cm2 (4) 576 cm2 (Tamil Nadu 2020-21)
cos A (1) 160 cm (2) 180 cm
34. If cosec A = 2, then the value of tan A + will be (3) 200 cm (4) 190 cm
1 + sin A
(Rajasthan 2020-21)
Solved Questions 49
B-

42. If the base circumference of a right circular cone and its 51. 125 identical cubes are cut from a big cube and all the smaller
height are 220 cm and 21 cm respectively, then the volume of cubes are arranged in a row to form a long cuboid. What is
the cone is : (Tamil Nadu 2020-21) the percentage of increase in total surface area of the cuboid
(1) 24,650 cm 3 (2) 21,750 cm3 over the total surface area of the cube?
(3) 26,950 cm 3 (4) 25,400 cm3 (West Bengal 2020-21)
x y z
43. If a = b, b = c & c = a, then the value of xyz is
2 1
(Uttar Pradesh 2020-21) (a) 234 % (b) 235 %
3 3
(1) 1 (2) 0
2
1 (c) 134 % (d) None of these
(3) (4) abc 3
abc
3 5 19
3 1 52. The value of .... is
44. If a b 3 , then the value of ‘a’ and ‘b’ is 2
1 .2 2
2 .32 2
9 .10 2
2
3 1
(Uttar Pradesh 2020-21) (West Bengal 2020-21)
(1) a = 2, b = –1 (2) a = 2, b = 1 99 1
(a) (b)
(3) a = –2, b = 1 (4) a = –2, b = –1 100 100
45. What is the probability that a leap year contains 53 Sundays
101
(c) (d) 1
(Uttar Pradesh 2020-21) 100
53. Given : 0° < < 90°, then, if sin + cos = x, which one of the
2 7 following is correct? (West Bengal 2020-21)
(1) (2)
7 13 (a) x < 1 (b) x > 1.5
2 (c) 1 x 2 (d) 1 x 2
(2) (4) None of these
13 54. The ratio of in which 9x – 3y – 14 = 0 divides the join of
46. The minimum value of sin cos is (Uttar Pradesh 2020-21) (2, –4) and (3, 7) is (West Bengal 2020-21)
(1) 0 (2) –1 (a) 2 : 1 (b) 1 : 2
1 1 (c) 2 : 3 (d) 3 : 2
(3) (4)
2 2 55. Find the area of the square ABCD. (Delhi 2019)
47. If radius of a right circular cylinder is increased by 10%, then D C
by what percent its height should be decreased so that its
volume remains unchanged
6

(Uttar Pradesh 2020-21)


m

(1) 17.26% (2) 17.36%


8

(3) 17.46% (4) None of these


10
m

48. If x 1 x 1 1 then value of x is


(Uttar Pradesh 2020-21)
5 2 A B
(1) (2) (1) 160 m2 (2) 140 m2 (3) 125 m2 (4) 120 m2
4 3
x 3 x 3 x x 3
56. If (2 – 4) + (4 – 2) = (4 + 2 – 6) , then the sum of all
4 3 real values of x is: (Delhi 2019)
(3) (4)
5 5 (1) 0.5 (2) 1.5 (3) 2.5 (4) 3.5
49. The solution set for the equation 57. If 2019x + 2019–x = 3, then the value of (Delhi 2019)
2 2
32 x 2.3x x 2 32( x 2) 0 is (West Bengal 2020-21) 20196 x – 2019–6 x
is
(a) {–1, 2} (b) {0, 2} 2019 x – 2019– x
(c) {0, –1} (d) {2, 4} (1) 3 (2) 6 (3) 9 (4) 12
50. For 3x+y = 81, 81x–y = 3, we get (West Bengal 2020-21) 58. Let ‘p’ be a root of the equation x2 – 5x + 7 = 0, then the
area of circle with centre at (P, P) and passing through
21 21 point (1,4) is
(a) no solution (b) x ,y
2 2 (Delhi 2019)
2 17 15 (1) 3 sq. units (2) 5 sq. units
(c) x 2, y (d) x ,y (3) 7 sq. units (4) None of these
3 8 8
EBD_7332
50
B- NTSE Stage 1 Question Bank

6 3
1 1 1 x x 3 5 7 17 19
59. If , then the value of is:- 68. The value of ... is
x y x y y y 4 36 144 5184 8100
(Delhi 2019) (Bihar 2019)
(1) 0.95 (2) 0.98
1 (3) 0.99 (4) 1
(1) 0 (2) (3) 1 (4) 2
2 69. The polynomial, f(x) = (x – 1) + (x – 2)2 + (x – 3)2 + (x – 4)2
2
60. The mean of 15 observations written in some order is 50. If has minimum value, when x = ................... (Bihar 2019)
the mean of first eight observations and last eight (1) 40 (2) 20
observations are 48 and 53 respectively, then the eighth (3) 10 (4) 2.5
observation is (Uttar Pradesh 2019) 70. From a bag containing 100 tickets numbered 1, 2, 3, ...........,
(1) 35 (2) 80 100 one ticket is drawn. If the number on this ticket is x,
(3) 72 (4) 58 1
61. If points (a, 0), (0, b) and (1, 1) are collinear, then the value then the probability that x 2 is ...... (Bihar 2019)
x
1 1 (1) 0 (2) 0.99
of is: (Uttar Pradesh 2019)
a b (3) 1 (4) None of these
(1) 1 (2) 2 71. Two circles of radii r1 cm and r2 cm (r1 > r2) touches each
other internally. The sum of their areas is A2 cm2 and the
(3) 0 (4) –1
distance between their centres is d cm, then,
62. If the ratio of volumes of two cubes is 27 : 64, then the (Bihar 2019)
ratio of their surface area is: (Uttar Pradesh 2019)
(1) A > d (2) A < d
(1) 3 : 4 (2) 4 : 3
(3) 9 : 16 (4) 16 : 9 (3) A 2 > d (4) A > 2d
63. If the base of a triangle is decreased by 30% and its height 72. A triangular field, having grass, has sides 20 m, 34 m, and
is increased by 25%, then the percentage decrease in area 42 m respectively. Three horses are tied to each of the
of triangle is: (Uttar Pradesh 2019) vertices with a rope of length 7m each. The horses start
grazing the field. The area of the portion of the field that is
(1) 20% (2) 15% ungrazed by the horses is ..............m2. (Bihar 2019)
(3) 22.5% (4) 12.5% (1) 250 (2) 255
x (3) 258 (4) 259
64. If cos 43 , then the value of tan 47° is 73. The ratio of the roots of the equation ax2 + bx + c = 0 is
2
x y2 same as the ratio of the roots of the equation px2 + qx + r = 0.
(Uttar Pradesh 2019)
If D1 and D2 are the discriminates of ax2 + bx + c = 0 and
y y px2 + qx + r = 0 respectively, then D1 : D2=
(1) (2) (Bihar 2019)
x x 2
y2
x x a2 b2
(3) (4) (1) (2)
y x2 y2 p2 q2

65. If the quadratic equations 2x2 + 4x + (a + 5) = 0 have equal c2


roots and (a + 4)x2 + ax – 3b = 0 have distinct real roots (3) (4) None of these
r2
then which of the following is true: (Uttar Pradesh 2019)
74. A conical shaped container, whose radius of base is r cm
3 3 and height is h cm, is full of water. A sphere of radius R is
(1) a = –3, b < (2) a = 3, b > completely immersed in the container in such a way that
4 4
the surface of sphere touches the base of the cone and its
3 3
(3) a = –3, b > – (4) a = 3, b < surfaces. The portion of water which comes out of the
4 4 cone is ......... (Bihar 2019)
66. The length of tangent drawn from a point Q to a circle is 24
cm and distance of Q from the centre of circle is R2 r2
(1) (2)
25 cm. The radius of circle is (Uttar Pradesh 2019) r2h R2h
(1) 7 cm (2) 12 cm
4R 2 4R 2
(3) 15 cm (4) 24.5 cm (3) (4)
r2h R2h
67. The unit digit in the expression 55725 + 735810 + 22853 is 75. In ABC, B = 90° and points D and E divide BC into
(Bihar 2019) three equal parts. If 3AC2 + 5AB2 = p AE2 then p = ..........
(1) 0 (2) 4 (Bihar 2019)
(3) 5 (4) 6 (1) 2 (2) 4
(3) 6 (4) 8
Solved Questions B-51
76. If ABC is an equilateral triangle such that AD BC, then 82. The 10th term from end of the A.P. 5, 12, 19, ....,173 is
AD2 = (Andhra Pradesh 2019) (Andhra Pradesh 2019)
3a 2 3a 2 (1) 96 (2) 103
A. B.
4 2 (3) 117 (4) 110
83. If the zeros of polynomial f (x) = x3 – 3x2 + m are in
3 3
C. BC2 D. a Arithmetic Progression, then the value of ‘m’ is
4 2
(Karnataka 2019)
(1) A and C (2) A
(3) D (4) B and C (1) 3 (2) 2
77. Which of following is not correct? (Andhra Pradesh 2019) (3) –3 (4) –2
(1) If tan + cot = 5, then tan2 + cot2 = 23 84. The Least Common Multiple (LCM) and the Highest
(2) If the pair of linear equations 4x + 5y = 9 and 8x + ky =18 Common Factor (HCF) of two numbers are 168 and 12
has infinitely many solutions, then k = 10. respectively. If the sum of those two numbers is 108, find
(3) The line cosec 60°x + cos45 y = 4 passing through the the difference between them: (Karnataka 2019)
point (tan60 , sec 45 ).
(1) 36 (2) 40
(4) If , are the zeroes of the quadratic polynomial x2 –2x + 1,
then 3 + 3 = 2. (3) 60 (4) 64
78. If –2 is a root of the quadratic equation x2 – px + 6 = 0 and
85. MNO is an isosceles triangle where MN MO. A circle
x2 + px – k = 0 has equal roots, then the value of k is
(Andhra Pradesh 2019) through the vertex ‘N’ touches side MO at its middle point
(1) 10 (2) 14
(3) 18 (4) 6 ‘P’ and intersects side MN at point ‘A’. If AM = 3 cm, then

a the length of MO is: (Karnataka 2019)


79. If cos = then cosec + cot in terms of a and b is
b M
(Andhra Pradesh 2019)
a b b a A
(1) (2)
a b b a P
a b b a
(3) (4)
a b b a
80. In the adjacent figure if AB = 10 cm, BC = 12 cm and N O
AC = 14 cm, then AD = (Andhra Pradesh 2019)
C (1) 6 cm (2) 9 cm
(3) 12 cm (4) 15 cm

F E 86. In PQR, PQR = 90° and QS PR. The true relation


among the following. (Karnataka 2019)
P
A B
D
(1) 8 cm (2) 5 cm
(3) 6 cm (4) 7 cm S
81. From the adjacent figure ABC, DE || BC and
1
AD = BD. If BC = 6cm, then DE is Q R
2
(Andhra Pradesh 2019)
2
A A. QS PS SR
2
B. PQ RS PR
D E
2
C. QR PS SR

B C D. PQ QR QS PR
(1) 3 cm (2) 2 cm (1) Only A (2) Only A and B
(3) 4 cm (4) 1.5 cm (3) Only A and C (4) Only A and D
EBD_7332
52
B- NTSE Stage 1 Question Bank

87. Following two statements speak about Arithmetic 93. 160 shares of face value Rs. 100 were purchased when the
Progression: (Karnataka 2019) market value was Rs. 120. Company had declared 20%
Statement (A): In an Arithmetic Progression series: dividend. Find the rate of return on the investment.
(Maharshtra 2019)
1 2 (1) 16.67% (2) 15.67%
20 19 18 ......... 25 terms is 300
3 3 (3) 14.67% (4) 13.67%
Statement (B): In an Arithmetic progression series: 94. A boat takes 3 hours to travel 30 km downstream and takes
5 hours to return to the same spot upstream. Find the speed
1 2
20 19 18 ......... 36 terms is 300 of the boat in still water. (km/hr)
3 3 (Maharshtra 2019)
Pick the correct option from below: (1) 10 km/hr (2) 8 km/hr
(1) Only statement A is true (3) 6 km/hr (4) 5 km/hr
(2) Only statement B is true 95. A shopkeeper sold a bicycle to a customer for Rs. 10304
(3) Both the statements are true including GST. The rate of GST was 12%. Find SGST
(4) Both the statements are false payable to him. (Maharshtra 2019)
88. ‘O’ is the centre of the circle. The area of the shaded region (1) Rs. 1104 (2) Rs. 552
(3) Rs. 1210 (4) Rs. 605
in the given figure is 126 cm2, PQ PR, then the diameter
QR is: (Karnataka 2019) 96. If the volume of a cuboid is 3000 cm3 and area of its base
P is 150 cm2, then the height of the cuboid is
(Rajasthan 2019)
(1) 10 cm (2) 15 cm
(3) 20 cm (4) 25 cm
97. How much time the minute hand of a clock will take to
Q R 2
O
describe an angle of radians? (Rajasthan 2019)
3
(1) 15 minutes (2) 20 minutes
(3) 10 minutes (4) 25 minutes
98. Find the area of shaded portion in the figure given below,
where ABCD is a square of side 28 cm: (Rajasthan 2019)
(1) 21 cm (2) 21 2 cm A B
(3) 7 cm (4) 7 2 cm
89. The radius of a solid right circular cone increases by 20%
and its height decreases by 20%. The percentage change
in its volume is (Karnataka 2019)
(1) 15.2% (2) 20% D C
(3) 25.2% (4) 30%
90. From the top of a building of height ‘h’ meter, the angle of (1) 784 cm2 (2) 616 cm2
(3) 668 cm 2 (4) 168 cm2
elevation of the top of the tower is ‘ ’ and angle of
depression of the foot of the tower is ‘ ’. The height of 99. Which of the following numbers will completely divide
the tower is (Karnataka 2019) 461 + 462 + 463 + 464 ? (Tamil Nadu 2019)
(1) 3 (2) 10
h (tan tan ) h(tan tan ) (3) 11 (4) 13
(1) (2)
tan tan
100. If sin + sin3 = cos2 , then the value of
h tan( ) h tan( ) cos6 – 4cos4 + 8cos2 is (West Bengal 2019)
(3) (4) (1) 1 (2) 4
tan tan
(3) 2 (4) 0
91. A metal cube is completely submerged in a cylindrical 101. If x + y = 12, then the maximum value of xy will be
vessel containing water. The diameter of the vessel is 30 (West Bengal 2019)
41 (1) 20 (2) 30
cm, the level of water is raised by1 cm. The length of (3) 36 (4) 40
99
the edge of the cube is (Karnataka 2019) 102. In a cuboid the length of the diagonal is p, the sum of
(1) 40 cm (2) 30 cm areas of all the surfaces is q and the sum of lengths of
coinitial edges is r. Then which one of the following
(3) 20 cm (4) 10 cm
relations is true? (West Bengal 2019)
92. In trapezium PQRS, PQ || RS and its diagonal intersect at
‘O’. If PQ = 6 cm and RS = 3 cm, then the ratio of the areas (1) r = 4 p 2 q2 (2) r = 4( p2 q)
of POQ and ROS is (Karnataka 2019)
(1) 4 : 1 (2) 1 : 2 (3) r = p2 q (4) r = 4 p 2 q
(3) 2 : 1 (4) 1 : 4
Solved Questions 53
B-

103. In the figure given below, ABCD is a quadrilateral and if 110. If x, y and z are positive real numbers and a, b and c are
rational numbers, then value of (Delhi 2018)
AB = 5 cm, AD = 12 cm, BC = CD = 13 cm, then the
area of the quadrilateral ABCD is (West Bengal 2019) 1 1 1
C b a c a a b c b b c is
13 cm
1 x x 1 x x 1 x xa c
(1) –1 (2) 1 (3) 0 (4) 2
D 111. The value of (Bihar 2018)
13 cm 11 13
12 cm sin sin sin sin sin sin sin is
14 14 14 14 14 14 14
90° 1 1
A B (1) (2)
5 cm 16 64
1 1
(1) 120 169 3 sq.cm (3) (4) None of these
4 128
n
1 112. If Sn nP n 1 Q, where Sn denotes the sum of the
(2) 120 169 3 sq.cm 2
4 first a terms of an Arithmetic Progression (A.P.), then the
1 common difference is (Bihar 2018)
(3) 60 169 3 sq.cm (1) P + Q (2) 2P + 3Q
4
(3) 2Q (4) Q
1 113. The mean of 25 observations is 36. If the mean of the first 13
(4) 60 169 3 sq.cm
4 observations is 32 and that of the last 13 observations is 39
then the 13th observation is (Bihar 2018)
104. If fi xi 216, fi 16 and weighted mean = 13.5 + P, (1) 32 (2) 30 (3) 28 (4) 23
then the value of P will be (West Bengal 2019) 114. The average weight of pupils of a class is 46 kg. The
(1) 1 (2) 0.1 average weights of boys and girls are respectively 50 kg
(3) 0.01 (4) 0 and 40 kg. The ratio of the number of boys to the number of
girls is ........... (Bihar 2018)
2ax 25by (1) 2 : 3 (2) 3 : 2 (3) 2 : 5 (4) 5 : 2
105. If a : b = 2 : 3 and x : y = 3 : 4, then is
3ay 4bx 115. The capacity of two pots are 240 litres and 112 litres
(Delhi 2018) respectively. Find the capacity of a container which can
exactly measure the contents of the two pots
24 5 24 12
(1) (2) (3) (4) (Jharkhand 2018)
5 24 5 13 (1) 9000 cm3 (2) 12000 cm3
106. A square is inscribed in a circle of radius ‘a’. Another circle (3) 16000 cm3 (4) 8000 cm3
is inscribed in that square and again a square is inscribed in 116. If 2 x 1 2 x 1 2560, find the value of x
this circle. The side of this square is : (Delhi 2018) (Jharkhand 2018)
a a (1) 10 (2) 12 (3) 9 (4) 8
(1) 2a (2) (3) (4) a 117. The product of two numbers is 12960 and their HCF is 36.
2 2
Number of pairs of such numbers that can be formed is
107. If a cos b sin c, then a sin b cos ? (Jharkhand 2018)
(Delhi 2018) (1) 2 (2) 3 (3) 4 (4) 5
118. In the given figure, AD is the bisector of BAC. If
(1) a 2 b2 c2 (2) a 2 b2 c2 AB = 10 cm., AC = 6 cm, and BC = 12 cm, find BD
(3) (d) (Jharkhand 2018)
a 2 b2 c2 a 2 b2 c 2 A
2 4 2
108. If x – 3x + 2 is a factor of x – px + q, then the value of p and
q are respectively are (Delhi 2018)
(1) –5, 4 (2) –5, –5
(3) 5, 4 (4) 5, –4
109. If x 1 , x 2 , x 3 , ..., x n are in A.P., then the value of
1 1 1 1
... is: (Delhi 2018)
x1 x2 x2 x3 x3 x4 xn 1 xn
B D C
(1) 4.5 cm (2) 9 cm
n 1 n 1
(1) (2) (3) 7.5 cm (4) 3 cm
x1 xn x2 xn 1 119. A sum becomes 6 times at 5% per annum. At what rate, the
sum becomes 12 times? (Jharkhand 2018)
n n 1
(3) (4) (1) 10% (2) 12%
x1 xn x1 xn (3) 9% (4) 11%
EBD_7332
54
B- NTSE Stage 1 Question Bank

120. In the given figure, three circles with centres A, B, C 130. If 102017 – 2017 is expressed as integer, what is the sum of its
respectively touch each other externally. If AB = 5 cm, digits? (Telangana 2018)
BC = 7 cm and CA = 6 cm, then the radius of the circle with (1) 18,144 (2) 17, 468
A is: (Jharkhand 2018) (3) 16,466 (4) 18,564
131. If P(x) = x4 + ax3 + bx2 + cx + d, where a, b, c, d Z and P(1)
C = P(2) = P(3) = 0, then the value of P(4) + P(0) will be....
(Telangana 2018)
O
(1) –12 (2) 24 (3) 16 (4) 18
R 132. If 39 + 312 + 315 + 3n is a perfect cube (of an integer) where
P n N, then find the value of n. (Telangana 2018)
A B
(1) 18 (2) 14 (3) 16 (4) 17
(1) 1.5 cm (2) 2 cm (3) 2.5 cm (4) 3 cm 1 2x
133. If x 5, then is equal to .......
121. If p, q, r are the roots of the equation x3 + 5x2 – 16x + 48 = 0 x 3x 2
5x 3
then value of the expression p(qr + q + r) + qr is (Telangana 2018)
(Himachal Pradesh 2018)
(1) 43 (2) –32 (3) –64 (4) 32 1 1
(1) 5 (2) (3) 3 (4)
122. If f(x) = 2x + 1 then number of real values of x for which three 5 3
unequal functions f(x), f(2x), f(4x) are in G.P.
134. In sin 3 cos3 0, then will be ....
(Himachal Pradesh 2018)
(1) 0 (2) 1 (3) 2 (4) 3 (Telangana 2018)
(1) 60° (2) 45° (3) 0 (4) –45°
2 2 135. If log4 7 = x, then the value of log7 16 will be ......
123. The equation x 1 has (Telangana 2018)
x 1 x 1
(Himachal Pradesh 2018) 2
(1) no roots (2) one root (1) x2 (2) 2x (3) x (4)
x
(2) two equal roots (4) infinite roots
136. The sum of the integers from 1 to 100 that are divisible by 2
124. If x is real then which equation is not possible?
or 5 is.... (Telangana 2018)
(Himachal Pradesh 2018)
(1) 2550 (2) 3050
1 1 (3) 3550 (4) 3600
(1) sin x (2) sin x2 137. If a, b, c are in A.P., then ax + by + c = 0 will always pass
x x3
through a fixed point whose co-ordinates are
1 1 (Andhra Pradesh 2018)
(3) sin x (4) tan x
x x (1) (–1, –2) (2) (–1, 2)
(3) (1, –2) (4) (1, 2)
1 1 138. In ABC, D, E and F are respectively mid points of the sides
125. The value of cos15 cos 7 sin 7 is
2 2 BC, CA and AB and P is a point on BC such that AP BC.
(Himachal Pradesh 2018) If DEF = 50º, then FPD = (Andhra Pradesh 2018)
1 1 1 1
(1) (2) (3) (4) A
2 8 4 16
126. In a hostel 60% of students read Hindi newspapers, 40%
read English and 20% read both. A student is selected at
random. Find the probability that she reads neither Hindi F E
nor English newspaper. (Himachal Pradesh 2018) 50º
1 1 1
(1) 0 (2) (3) (4)
5 3 2 B C
P D
127. The centroid of a triangle is (2, 4) and circumcentre is (1, 7)
then find the orthocentre. (Himachal Pradesh 2018) (1) 120° (2) 110° (3) 135º (4) 130º
139. If x < 1, y < –1, then (x–1, y – 3) lies in
(1) 3, 1 (2) 4, –2
(Andhra Pradesh 2018)
(3) –2, 4 (4) –8, 4
(1) Q3 (2) Q4 (3) Q2 (4) Q1
128. Point R(h, k) divides line segment AB between axes in the
140. 3(sin x – cos x)4 + 6(sin x + cos x)2 + 4(sin6x + cos6x) =
ratio 1 : 2 where A lies on X-axis. Find equation of line.
(Andhra Pradesh 2018)
(Himachal Pradesh 2018) (1) 14 (2) 13 (3) 7 (4) 9
(1) 2hx + ky = 3hk (2) 2kx + hy = 3hk
141. If ax–1 = bc, by–1 = ca, cz–1 = ab, then xy + yz + zx
(3) kx + hy = 2hk (4) 3kx + hy = 2hk (Andhra Pradesh 2018)
129. Let x be real number such that x3 + 4x = 8, then the value of (1) xyz (2) 0
x7 + 64x2 is ... (Telangana 2018)
(1) 136 (2) 146 (3) 128 (4) 156 1 1 1
(3) 1 (4)
x y z
Solved Questions 55
B-

142. If the equation (k + 3) x2– (5 – k) x + 1 = 0 has distinct roots,


Y 13)
the value of k will be (Andhra Pradesh 2018) M,
R(
(1) k = 1 or k = 13 (2) k < 13 or k > 1 5,5
)
S( N)
(3) k > 12 or k > 1 (4) k > 13 or k < 1 15,
Q(
143. If ax = by+z then (Andhra Pradesh 2018) 9,2
)
P(
log a x log b y z X
(1) (2) O
log b y z log a x
(1) 4 (2) –2 (3) –1 (4) 1
151. What will be the area of largest triangle that can be
log a y z
(3) log b x
(4) x log a = yz log b r
inscribed in a semicircle of radius 16 (Chandigarh 2018)
144. If the roots of the equation (a 2 + b 2 ) x 2 – 2b (a + c)
r2 r2 r2
x + (b2 + c2 ) = 0 are equal, then (Tamil Nadu 2018) (1) 16 r2 (2) (3) (4)
64 32 256
2ac 152. If tan5 tan4 = 1, then is _________. (Gujarat 2018)
(1) 2b = a + c (2) b=
a+c (1) 7 (2) 3 (3) 10 (4) 9
153. For right angled ABC, sin 2 A + sin 2 B + sin2 C = _______.
(3) b2 = ac (4) b = ac
(Gujarat 2018)
145. A circle is inscribed in a triangle ABC with right angle at A. (1) 2 (2) 1 (3) 0 (4) –1
The length of the two sides containing the right angle are 154. If the sum of ‘n’ terms of an arithmetic progression is
6 cm and 8 cm respectively. The radius of the circle is: Sn = 3n + 2n 2 then its common difference is
(Tamil Nadu 2018) (Karnataka 2018)
(1) 2 cm (2) 6 cm (3) 8 cm (4) 10 cm (1) 9 (2) 6 (3) 4 (4) 3
146. In the given figure. AB || ED and BC || EF, then the value of 155. If the graphs of x–y = 2 and kx + y = 3 (k is constant) intersect
ABC + DEF is (Rajasthan 2018) at a point in first quadrant then the value of k is
(Karnataka 2018)
E F (1) equal to 1 (2) greater than –1
A (3) less than 4
3 (4) between –1 and 4
3
156. If the roots of x2 – px + q = 0 are two consecutive integers
D the value of p2–4q is (Karnataka 2018)
B (1) 4 (2) 3 (3) 2 (4) 1
C
157. Three squares of a chess board are selected at random. The
(1) 90º (2) 180º (3) 120º (4) 360º probability of getting two squares of one colour and other
147. In the given figure, AB = AC, BAC = 40º, BE and CD are of a different colour is (Karnataka 2018)
angle bisectors of B and C respectively. If DOE = x, the
16 8 3 3
value of x is (Rajasthan 2018) (1) (2) (3) (4)
A 21 21 32 8

40º 5 7
D x E
3 2
O 158. 3 3 Choosse correct alternative for the value of
B C 4 2
(1) 140º (2) 70º (3) 110º (4) 40º determinant. (Maharashtra 2018)
148. The shadow of a tower, when the angle of elevation of the 1 1
sun is 30º is found to be 10 metre longer than when it was (A) (B)
8 8
60º. The height of the tower will be (Rajasthan 2018) 3
1 1
5 3 1 m (C) (D) 3
(1) 5 3m (2) 2 512
(1) A and C (2) B, C and D
(3) 5 3 1 m (4) 3 5m (3) A, B and C (4) A, C and D
159. Read the following statements carefully and choose the
149. If x : y =3 : 5 and x : 2 = 5 : 7, then what is (y – z) (y +z) equal to? correct alternative. (Maharashtra 2018)
(Chandigarh 2018) (A) The slope of the line parallel to X-axis can be derived
(1) 2/23 (2) 27/46 (3) 18/46 (4) 15/46
x2 x1
150. Find the value of m – n in the rectangles PQRS. by the formula
(Chandigarh 2018) y2 y1
EBD_7332
56
B- NTSE Stage 1 Question Bank

(B) The slope of the line parallel to Y- axis is 1


3 2 3
(C) The cotangent ratio of an angle made by the line with 165. If a 3 b, then the value of a b where a
the positive direction of X- axis is called the slope of 3 3
that line. and b are rational number is (Rajsthan 2017)
(D) The slope of the line which makes acute angle with (1) 5 (2) 8
(3) 2 (4) 16
X- axis is less than zero and the slope of the line making
166. For which positive values of k and p, equations
obtuse angle with X-axis is greater than zero. 2x2 + px + 8 = 0 and p (x2 + x) + k = 0 have equal roots?
Alternative: (Rajsthan 2017)
(1) Statement A and B correct (1) k = 1, p = 4 (2) k = 2, p = 8
(2) Statement C and D correct (3) k = 4, p = 8 (4) k = 2, p = 4
(3) Only statement C is wrong 167. If , are zeros of polynomial x2 – p(x + 1) – k such that
(4) All statements are wrong ( + 1) ( + 1) = 6, then value of k is (Rajsthan 2017)
160. In the figure, semi-circles are drawn whose centre are X, Y, Z (1) 5 (2) – 1
(3) – 3 (4) – 5
respectively. Points (X, Y, Z) are collinear points. AX = 2.5, 168. Which is unit digit of 618 – 510? (Rajsthan 2017)
BY = 6.5, CZ = 8.5 and AP + QC = 16; QC + CR = 27 and (1) 5 (2) 8
CR + AP = 19 then find the value of AP + PB + BQ + QC (3) 1 (4) 9
= CR + RD = ? (Maharashtra 2018) 169. In the following figure QT PR and QS = PS. If TQR = 40°
P and RPS = 20° then value of x is (Rajsthan 2017)
Q
R
P

T
A X B Y C Z D
(1) 37 (2) 41 (3) 53 (4) 47 x
2
161. The sum of areas of two squares is 468 cm . If the sum of their Q R
S
perimeters is 120 cm, then the difference of their sides is –
(Uttar Pradesh 2018) (1) 80° (2) 25°
(1) 1.5 cm (2) 2 cm (3) 4 cm (4) 6 cm (3) 15° (4) 35°
x+1 2–x
162. If 5 + 5 = 126 then x is equal to 1 1
170. Which term of A.P. 20, 19 , 18 , ... is first negative term?
(Uttar Pradesh 2018) 4 2
(1) –2, –1 (2) 1, –2 (Rajsthan 2017)
(3) –1, 3 (4) 2, –1 (1) 18th (2) 15th
163. In figure, PQ is a chord of a circle with centre O and PT is its (3) 28th (4) 27th
tangent at P. If QPT = 60º, then PRQ is– 171. The angles of elevation of the top of a 12 m high tower from
(Haryana 2018) two points in opposite directions with it are complementary.
If distance of one point from its base is 16 m, then distance
of second point from tower’s base is
Q
O (Rajsthan 2017)
(1) 24 m (2) 9 m
R (3) 12 m (4) 18 m
cos A cos A
P T 172. If m = and n = , then (m2 + n2) cos2 B is equal to
cos B sin B
(1) 105º (2) 115º (3) 120º (4) 130º
(Rajsthan 2017)
164. ABC is a right angled triangle, right angled at B. If side AB
is divided into three equal parts by points D and E such that (1) m2 (2) n2
2
(3) m + n 2 (4) m + n
AC2 EC 2 173. If ratio of heights of two similar triangles is 4 : 9, then ratio
D is nearest to A, than is equal to–
DC2 BC 2 between their areas is (Rajsthan 2017)
(Haryana 2018) (1) 2 : 3 (2) 3 : 2
1 1 (3) 81 : 16 (4) 16 : 81
(1) 3 (2) 2 (3) 2 (4) 2
2 4
Solved Questions B-57

174. In a circle of 10 cm radius, two chords AB = AC = 12 cm, then 182. If the distance between the points (4, q) and (1, 0) is 5 units
the length of the chord BC is (Rajsthan 2017) then the value of q is: (Punjab 2017)
(1) 12 cm (2) 9.6 cm (1) 4 (1) – 4
(3) 19.2 cm (4) 7.2 cm (3) 4 (4) 0
175. If p + q + r = 0 then the value of 183. The area of an equilateral triangle is 49 3 cm2.
2p 2 q r 2q 2 p r 2r 2 p q Taking each vertex as centre, circles are described with
will be radius equal to half the length of the side of the triangle.
pqr
Find the area of the part of the triangle which is not
(Punjab 2017)
included in these circles.
1
(1) 3pqr (2) 22
pqr ( 3 = 1.73, = ) (Punjab 2017)
7
(3) 6 (4) – 6 (1) 84 cm2 (2) 77.7 cm2
176. A circle is inscribed in a square of side 2.5 cm. Another circle (3) 7.77 cm2 (4) 70.7 cm2
is circumscribing this square. The ratio of areas of outer
tan A tan B tanAcotB sin 2 B
circle and inner circle is (Punjab 2017) 184. If A + B = 90° then is
sin A sec B cos 2 A
(1) 1 : 2 (2) 2 :1 equal to (Delhi 2017)
(3) 2 : 1 (4) 3 :1 (1) cot2 A (2) cot2 B
(3) – tan2A (4) – cot 2A
1 185. The simplified form of the expression given below is
177. If x = then find the value of 4x3 + 2x2 – 8x –3.
3 –1 (Delhi 2017)
y 4 x4 y3
(Punjab 2017) x( x y) x
(1) 0 (2) 2
y2 xy x 2
(3) – 2 (4)
2 (1) 1 (2) 0
178. How many revolutions will a circular wheel of radius r units (3) –1 (4) 0
will make to cover a distantce of 100 times its diameter? x 2 bx m 1
(Punjab 2017) 186. If , has roots which are numerically equal
ax c m 1
100 but of opposite signs, the value of m must be
(1) (2) 100 (Delhi 2017)
(1) (a – b)/(a + b) (2) (a + b)/(a – b)
50 1
(3) (4) (3) c (4)
100 c
179. If pth term of an AP is q and qth term is p then mth term of 187. If we divide a two digit number by the sum of its digits we
this AP will be: get 4 as quotient and 3 as remainder. Now if we divide that
(1) p + q + m (2) p + q – m two digit number by the product of its digits, we get 3 as
(3) p – q – m (4) p – q + m quotient and 5 as remainder. The two digit number is
180. Two dice are thrown. Find the probability that sum of (Delhi 2017)
(1) even (2) odd prime
numbers of both up sides of both dice is a perfect square.
(3) odd composite (4) odd
(Punjab 2017)
188. Four positive integers sum to 125. If the first of these
1 7 numbers is increased by 4, the second is decreased by 4,
(1) (2)
6 36 the third is multiplied by 4 and the fourth is divided by 4 we
5 find four equal numbers then four original integers are
(3) (4) 0 (Delhi 2017)
36
(1) 16, 24, 5, 80 (2) 8, 22, 38, 57
181. From the top of a tower of h m high, the angles of depres-
(3) 7, 19, 46, 53 (4) 12, 28, 40, 45
sion of two objects, which are in line with the foot of the
189. When 31513 and 34369 are divided by a certain three digit
tower are and ( > ). Find the distance between two
number, the remainders are equal, then the remainder is
objects. (Punjab 2017)
(Andhra Pradesh 2017)
(1) h (tan – tan ) (2) h (cot – tan )
(1) 86 (2) 97
(3) h (cot – cot ) (4) h (cot + cot )
(3) 374 (4) 113
EBD_7332
58
B- NTSE Stage 1 Question Bank

190. The greatest number of four digits which when divided by 199. On increasing the radius of the base and height of a cone
3, 5, 7, 9 leaves the remainder 1, 3, 5, 7 respectively, is each by 20%, then the percentage increase in the volume
(Andhra Pradesh 2017) will be (Karnataka 2017)
(1) 9763 (3) 9673 (1) 20% (2) 40.8%
(3) 9367 (4) 9969 (3) 60% (4) 72.8%
191. e f g h is a four digit number. One hundredth of e f g h is the 200. If – 4 is a root of the quadratic equation x 2 + px – 4 = 0 and
mean of e f and g h, then the four digit number is equation x2 + px + m = 0 has equal roots, then the value of
(Andhra Pradesh 2017) “m” is (Karnataka 2017)
(1) 3648 (2) 4950 25
(1) – 4 (2)
(3) 4590 (4) 3468 4
192. If x2 + xy + x = 12 and y2 + xy + y = 18, then the value of – 25
(3) + 4 (4)
x + y is ... (Andhra Pradesh 2017) 4
(1) 5 or – 6 (2) 3 or 4 201. Statement-I: If 5 24 x y then x + y = 5 and
(3) 5 or 3 (4) 6 or –3 xy = 24
193. If 217x + 131y = 913 and 131x + 217y = 827, then the value of
Statement-II: The square root of 5 – 24 is 3– 2
x + y is (Andhra Pradesh 2017)
(1) 8 (2) 5 (Karnataka 2017)
(3) 7 (4) 6 (1) Both statement-I and II are wrong
(2) Statement-I is wrong, but statement-II is right
1
194. If x = , (x 2), , then the value of x is (3) Both statement-I and II are right
1
2 (4) Statement-I is right, but statement-II is wrong
1
2 202. The value of cos2 50 + cos2 100 + cos2 150 + ... + cos2 850 +
2 x
cos2 900 is (Karnataka 2017)
(Andhra Pradesh 2017)
1
(1) 1 (2) 3 (1) 9 (2) 9
2
(3) 2 (4) 5 1
195. x1, x2, x3, ..., are in A.P. If x1 + x7 + x10 = – 6 and (3) 8 (4) 8
2
x3 + x8 + x12 = – 11, then x3 + x8 + x22 = ? 203. If (x + a) is the factor of the polynomials (x2 + px + q) and
(Andhra Pradesh 2017) (x2 + mx + n) then the vlaue of ‘a’ is (Karnataka 2017)
(1) – 21 (2) – 15
n–q m–p
(3) – 18 (4) – 31 (1) (2)
m–p n–q
2 5 8 ...n.terms 23 q–n m–p
196. If , then n value is (3) (4)
7 11 15 ...n.terms 35 m–p q–n
(Andhra Pradesh 2017) 204. The lowest common multiple of two numbers is 14 times
(1) 17 (2) 15 their greatest common divisor. The sum of LCM and GCD is
(3) 18 (4) 23 600. If one number is 80 then other number is
197. If the co-ordinates of the midpoints of the sides of a triangle (Karnataka 2017)
are (1, 1), (2, –3) and (3, 4), then the centroid of the triangle (1) 600 (2) 520
is ... (Andhra Pradesh 2017) (3) 280 (4) 40
1 2 205. The shaded protion in the given Venn diagram represents
(1) 3, (2) 1,
3 3 P Q U
2
(3) (3, 1) (4) 2,
3
198. The area of two concentric circles are 1386 cm 2 and
962.5 cm 2. The width of the ring is (Karnataka 2017)
(1) 4.2 cm (2) 3.8 cm (Karnataka 2017)
(3) 3.5 cm (4) 2.8 cm (1) P Q (2) P Q
(3) (P Q ) (4) (P Q )
Solved Questions B-59

206. In ABC, the altitudes AL, BM and CN are intersect at ‘O’.


a b
The value of AN × BL × CM is same as 217. If a + b = 2c, the value of is
(Karnataka 2017) a c b c
(1) BN × LC × AM (2) AL × CN × BM (West Bengal 2017)
(3) OL × OM × ON (4) OC × OB × OA (1) 0 (2) 1
207. If the letters of the word “FATE” are arranged as in a (3) 2 (4) – 1
dictionary without repetition, then the rank of the 218. The compound interest for two years of the amount
arrangement of “FATE” is (Karnataka 2017) Rs. 7500 at the rate of 8% per annum would be
(1) 12 (2) 13 (West Bengal 2017)
(3) 14 (4) 15 (1) Rs. 1,248 (2) Rs. 1,260
(3) Rs. 1,300 (4) Rs. 1,352
208. If ax2 + bx + c = a(x – p)2, the relation among a, b, and c is 219. A businessman fixed the selling price of an article after in-
(West Bengal 2017) creasing the cost price by 40%. Then he allowed his cus-
(1) abc = 1 (2) 2b = a + c tomer a discount of 20% and gained Rs. 48. The cost price
(3) b2 = ac (4) b2 = 4ac of the article is (West Bengal 2017)
(1) Rs. 200 (2) Rs. 248
209. The identity (x 4)2 x 4 is possible, when (3) Rs. 400 (4) Rs. 448
(West Bengal 2017) 220. The curved surface area of a right circular cylinder and that
(1) x – 4 (2) x – 4 of a sphere are equal. If their radii are equal, the ratio of their
(3) x – 16 (4) Not possible volume is (West Bengal 2017)
210. The number of real roots of the quadratic equation (1) 3 : 2 (2) 2 : 3
3x2 + 4 = 0 is (West Bengal 2017) (3) 3 : 4 (4) 4 : 3
(1) 0 (2) 2 221. The sum of the length, breadth and height of a rectangular
(3) 1 (4) 4 parallelopiped is 25 cm and its whole surface area is
211. The solution of the equation 9x + 6x = 2.4x 264 sq. cm. The area of the square whose sides are equal to
(West Bengal 2017) the length of the diagonal of that parallelopiped is
(1) 0 (2) 1 (West Bengal 2017)
(3) ± 2 (4) – 1 (1) 256 sq. cm. (2) 361 sq. cm.
212. If f(x) = 2x3 – 3x + 4, the value of f(x) + f(–x) is (3) 225 sq. cm. (4) 324 sq. cm
(West Bengal 2017) 222. The radii of two circles with center at A and B are 11 cm and
(1) 4 (2) 6 6 cm respectively. If PQ is the common tangent of the circles
(3) 0 (4) 8 and AB = 13 cm, length of PQ is
(West Bengal 2017)
x2 y2 z2 (1) 13 cm (2) 12 cm
213. If 2, the value of
by cz cz ax ax by (3) 17 cm (4) 8.5 cm
223. The chords PQ and RS of a circle are extended to meet at the
c b a point Q. If PQ = 6 cm, OQ = 8 cm, OS = 7 cm, then RS =
is (West Bengal 2017)
2c z 2b y 2a x (West Bengal 2017)
(1) 12 cm (2) 9 cm
1 (3) 10 cm (4) 16 cm
(1) 2 (2)
2 224. ABC is a right angled triangle and AD is perpendicular to
1 the hypotenuse BC. If AC = 2 AB, then BC =
(3) 4 (4)
4 B
214. If log4(log4{log4.(log4.x)}) = 0, ‘x’ is equal to
(West Bengal 2017)
(1) 256 (2) 418
x D
(3) 2512 (4) 2564
1 1
215. If x2 + y2 = z2 the value of is
log z x log x y x
y A 2x C
(West Bengal 2017)
(1) x (2) y (West Bengal 2017)
(1) 2 BD (2) BD
(3) x + y (4) 2 (3) 5 BD (4) 4 BD
216. If (x + 2) and (2x – 1) are factors of (2x3 + ax2 + bx + 10), the 225. (x + 2), x and (x – 1) are the frequencies of the numbers 12, 15
value of (a2 + b2) is (West Bengal 2017) and 20 respectively. If the mean of the distribution is 14.5,
(1) 338 (2) 218 the value of x is (West Bengal 2017)
(1) 2 (2) 3
(3) 74 (4) 198
(3) 4 (4) 5
EBD_7332
60
B- NTSE Stage 1 Question Bank

226. If two angles of a triangle are 87° 24 54 and 32° 31 6 , the


third angle is (West Bengal 2017) –101 123
(1) (2)
123 101
(1) (2) (3) –1 (4) 1
6 2 234. If x3 + y = 2249, where x and y are natural numbers and
HCF of x and y is not 1, then the value of (x + y) is
(3) (4) (Haryana 2016)
3 4
227. If xsin3 + ycos3 = sin cos and x sin – ycos = 0, the (1) 26 (2) 39
value of x2 + y2 is (West Bengal 2017) (3) 52 (4) 65
(1) 0 (2) 1
235. If a 6 11 and b 6 11 then the value of
1 1 (a + b) is : (Haryana 2016)
(3) (4)
2 3 (1) (2)
22 2 11
228. Four positive integers sum to 125. If the first of these numbers
is increased by 4, the second is decreased by 4, the third is (3) 6 (4) 12
multiplied by 4 and the fourth is divided by 4 we find four 236. If 2a2 –2ab + b2 + 6a + 9 = 0, then (a + b) + ab equals:
numbers, equal then four original integers are (Haryana 2016)
(Delhi 2016) (1) 1 (2) 2
(1) 16, 24, 5, 80 (2) 8, 22, 38, 57 (3) 3 (4) 4
(3) 7, 19, 46, 53 (4) 12, 28, 40, 45 237. The sum of all the terms common to the arithemetic
229. The total number of squares on a chessboard is (Delhi 2016) progressions 1, 3, 5,..........., 1991 and 1, 6, 11, ........., 1991, is :
(1) 206 (2) 205 (Haryana 2016)
(3) 204 (4) 202 (1) 199100 (2) 199200
230. In the figure, the area of square ABCD is 4 cm2 and E is mid (3) 199300 (4) 200196
point of AB, F, G, H and K are the mid points of DE, CF, DG 238. The cube root of
and CH respectively. The area of KDC is x + y + 3x1/3 y1/3 (x1/3 + y1/3) is (Rajasthan 2016)
(Delhi 2016) (1) x + y (2) x1/3 + y1/3
(3) (x + y)1/3 (4) (x + y)3
A E B
239. Expressing 0.23 0.23 as a single decimal, we get
(Rajasthan 2016)
F (1) 0.465 (2) 0.465
G (3) 0.465 (4) 0.4654
H
K
2
D C 240. If x 2 is a factor of kx 2x 1, then the value of k
is (Rajasthan 2016)
1 2 1 2
(1) cm (2) cm 3 2
4 8 (1) (2)
2 3
1 1
(3) cm 2 (4) cm 2 3 2
16 32 (3) (4)
2 3
231. If x% of y is equal to 1% of z, y % of z is equal to 1% of x and
z% of x is equal to 1% of y, then the value of xy + yz + zx is 241. In the equations 3x + 2y = 13xy and 4x – 5y = 2xy, the values
(Delhi 2016) of x and y that satisfy the equations are (Rajasthan 2016)
(1) 1 (2) 2 (1) (2, 3) (2) (3, 2)
(3) 3 (4) 4 1 1 1 1
232. The volume and whole surface area of a cylindrical solid of (3) , (4) ,
2 3 3 2
radius 'r' units are v and s respectively. If the height of
242. The angles of elevation of the top of a tower from two points
v
cylinder is 1 unit then is equal to (Delhi 2016) at a distance of 9 m and 16 m from the base of the tower and
s in the same straight line in the same direction with it are
1 1 1 1 complementary. Then height of the tower is
(1) 1 (2) 1 (Rajasthan 2016)
2 r 1 2 r 1
(1) 12 m (2) 15 m
1 1 1 1 (3) 20 m (4) 25 m
(3) 1 (4) 1
2 r 2 r 243. If log5 (log2 (log3x)) = 0 then the value of x is
233. The sum of the reciprocals of the roots of the equation (Uttar Pradesh 2016)
(1) 3 (2) 6
101 1 (3) 9 (4) 0
x 1 0 is (Haryana 2016)
123 x
Solved Questions 61
B-

244. Angle between the lines 6 + x = 0 and 3 – y = 0 will be 253. One of the root of a quadratic equation is 3 – 2 , then
(Uttar Pradesh 2016)
(1) 0° (2) 90° which of the following is that equation.
(3) 180° (4) 60° (Maharashtra 2016)
245. If number 6, 8, 2x – 5, 2x – 1, 15, 17, 20 and 22 are in ascending (1) (x2 – 6x – 7) = 0 (2) (x2 + 6x – 7) = 0
order and its median is 14 then the value of x will be (3) (x2 + 6x + 7) = 0 (4) (x2 – 6x + 7) = 0
(Uttar Pradesh 2016) 254. In ABC, m B 90 , AB = 4 5. BD AC, AD = 4, then
(1) 14 (2) 7
(3) 15 (4) 20 A( ABC) = ? (Maharashtra 2016)
246. If u = {1, 2, 3, 4,5,6,7,8} (1) 96 sq. units (2) 80 sq. units
A = {3, 4, 5, 6} and B = {1, 3, 5, 7} then the value of (A – B ) (3) 120 sq. units (4) 160 sq. units
is (Uttar Pradesh 2016) 255. Side of a cube is increased by 50%, then what percent
(1) {2, 8} (2) {3, 5} increase will be in the area of the vertical faces of the cube?
(3) {1, 7} (4) {1, 2, 4, 6} (Maharashtra 2016)
1 2 (1) 125% (2) 150%
247. Factors of c 2c 9 are (Uttar Pradesh 2016) (3) 100% (4) 50%
3 256. A 100 meter long train moving with constant speed of 90
1 1 km/h crosses a tunnel of 300 meter long. The time taken by
(1) c 3 c 3 (2) c 3 c 3 the train to cross the tunnel completely is
3 3
(Karnataka 2016)
1 1 (1) 16 seconds (2) 8 seconds
(3) c 3 c 3 (4) c 3c 1
3 3 (3) 4 seconds (4) 2 seconds
248. In the given figure, ABC has points D and F in AC and 257. The variance of 20 observation is 5. if each observation is
multiplied by 2, then new variance of the resulting
point E in BC such that DE || AB and EF || BD. If CF = 4 cm observations is (Karnataka 2016)
(1) 7 (2) 10
and AC = 9 cm, what is the length of DC ? (Odisha 2016) (3) 20 (4) 40
A 258. The polynomial (x2 –1) leave remainder as R1 and R2. The
D true statement among the following is
F (Karnataka 2016)
(1) R1 + R2 = 0 (2) R1 + R2 = 0
(3) 2R1 + R2 = 0 (4) R1 + 2R2 = 0
259. In ABC, XY || BC and xy divides the traingle into two parts
B E C
AX
(1) 7 cm (2) 6 cm of equal area. the value of is (Karnataka 2016)
(3) 5 cm (4) 4 cm BX
249. If a : b = 3 : 5 and a : c = 5 : 7, what (b – c) : (b + c) equal to? 2
(1) 4 : 46 (2) 46 : 4 (Odisha 2016) (1) 2 –1 (2)
2
(3) 5 : 48 (4) 48 : 5 2
(3) (4) 2 1
1 1 2
250. If the mth term of an A.P. is and then nth term of its is ,
n m
1 a4
then what is the mnth term equal to? (Odisha 2016) 260. If a 5 2 6 then (Karnataka 2016)
a
(1) 1 (2) 2
m n (1) 4 6 (2) 10
(3) (4)
n m (3) 5 (4) 2 6
251. 4 4 6 2 2 3 2 2 k
m n m n (mn) , then find the value of
m n B
2
k. (Maharashtra 2016)
(1) 6 (2) 3 D
(3) 2 (4) 1
252. In a group of students, 10% students scored marks less
than 20, 20% students scored marks between 20 to 40, 35% 261.
students scored marks between 40 to 60 and 20% students C 4 A
scored marks between 60 to 80. Remaining 30 students
scored marks between 80 to 100. Find the mode of marks. In the given figure, if AD BC, AC = 4, BD = 2, AB = a and
(Maharashtra 2016) CD = b then a2 + b2 = (Karnataka 2016)
(1) 30 (2) 50 (1) 6 (2) 8
(3) 60 (4) 70 (3) 12 (4) 20
EBD_7332
62
B- NTSE Stage 1 Question Bank

262. If be the zeros of the polynomial 2x2 + 5x + k such that (1) 90° (2) 180°
21 (3) 270° (4) 360°
2+ 2+ = , then K = ? (Jharkhand 2015)
4 273. Ihe mean of 100 observations is 24. If 4 is added to each of
(1) 3 (2) –3 the observation and then each of them is multipled by 2.5
(3) –2 (4) 2 then new mean is (Delhi 2015)
263. The sum of three consecutive terms of an AP is 21 and the (1) 70° (2) 80
sum of the squares of these terms is 165. The middle term of (3) 65 (4) 75
the three terms is (Jharkhand 2015) 274. If 2m – 2m –1 – 4 = 0. Then value of mm is
(1) 10 (2) 4 (Delhi 2015)
(3) 6 (4) 7 (1) 4 (2) 27
264. If the sum of the first "p" terms of an AP is the same as the (3) 6 (4) 29
sum of its first "q" terms (where p q) then sum of (p + q)th
275. If 23x 64 1
and 10y 0.01 , th en the value of
terms is (Jharkhand 2015)
(1) 1 (2) 0 (50 x) 1 (10 y) 1
is (Telangana 2015)
(3) p + q – 1 (4) p + q + 1
(1) –1 (2) 1
1 1 1
265. The value of 2 is (Jharkhand 2015) (3) (4) 2
1 cot 1 tan 2 2
1 276. Given that x, a1, a2, y are in A.P. and x, b1, b2, b3, y are also in
(1) 1 (2)
2 a 2 a1
A.P, then the value of if (x y) (Telangana 2015)
1 b2 b3
(3) (4) 2
4 1 3
(1) (2)
x 2 4
266. If cos 43° = , then the value of tan 47°.
x2 y2 4 4
(3) (4)
3 3
x y 277. Sum of the roots of the equation 4x – 3(2x + 3) + 128 = 0,
(1) (2) (Jharkhand 2015)
y x is (Maharashtra 2014)
(1) 0 (2) 7
x y
(3) 5 (4) 8
(3) 2 2 (4) 2
x y x y2 278. If sinx + cosecx = 2, then (sin10x + cosec10x) is equal to
267. If f(x) = x6 – 10x5 – 10x3 – 10x2 – 10x + 10, the value of f(11) is (Maharashtra 2014)
(1) 1 (2) 10 (West Bengal 2015) (1) 3 (2) 0
(3) 11 (4) 21 (3) 1 (4) 2
268. If a = 23, b = 27 and c = 50, the value of a3 + b3 – c3 + 3abc is 279. The hypotenuse of a right triangle is 10 cm and radius of the
(1) 100 (2) 73 (West Bengal 2015) inscribed circle is 1 cm. The perimeter of the triangle is
(3) 77 (4) 0 (Maharashtra 2014)
269. If (3p + 4q) : (3r + 4s) = (3p – 4q) : (3r – 4s), the value of ps : (1) 15 cm (2) 22 cm
qr is (West Bengal 2015) (3) 24 cm (4) 18 cm
(1) 3 : 4 (2) 1 : 1 280. A shopkeeper blends two varieties of tea costing ` 18 and `
(3) 4 : 3 (4) 9 : 16 13 per 100 gms in the ratio 7 : 3. He sells the blended variety
270. The solution of the inequation x2 – 4 0 is at the rate of ` 18.15 per 100 gm. His percentage gain in the
(West Bengal 2015) transaction is: (Maharashtra 2014)
(1) x 2 (2) 2 x 2 (1) 8% (2) 12%
(3) 2 x (4) x (3) 15% (4) 10%
2
tan cot
271. The number of solution of the equation x 2 x 2 is 281. If 2, (0° ), then the value of is
tan cot
(1) 0 (2) 1 (West Bengal 2015)
(3) 2 (4) 4 (Maharashtra 2014)
272. In given figure measure of 1 + 2 + 3 + 4 + 5 + 6 is (1) 60° (2) 30°
(Delhi 2015) (3) 90° (4) 45°
282. If the line segments joining the points (a, b) and (c, d)
1 subtends a right angle at the origin, then which of the
2 6 equation is correct? (Karnataka 2014)
(1) ac – bd = 0 (2) ac + bd = 0
3 5 (3) ab + cd = 0 (4) ab – cd = 0
4
Solved Questions 63
B-

283. If the pair of linear equations a1 x + b1 y + c1 = 0 and a2 x + b2 289. In the given figure, ODC ~ OBA, BOC = 115° and
+ c2 = 0 has infinite number of solutions then the correct CDO = 80°. Then OAB is equal to
condition is (Karnataka 2014) (Rajasthan 2014)
a1 b1 c1 a1 b1 c1 D C
(1) (2) 80°
a2 b2 c2 a2 b2 c2
65°
a1 b1 a1 c1 O 115°
(3) (4)
a2 b2 a2 c2

1 1 A B
2
284. If x then the value of x is (1) 80° (2) 35°
2 3 x2 (3) 45° (4) 65°
(Karnataka 2014) 290. The probability of getting a number greater than 2 by
(1) 12 3 (2) 8 3 throwing a fair dice is : (Madhya Pradesh 2013)
(1) 2/3 (2) 1/3
(3) 14 (4) 12 (3) 1 (4) 3/5
285. In the triangle PQR, MN || QR and MN divides the triangle 291. Which one of the following is a factor of the expression
QM (a + b)3 – (a– b)3 ? (Madhya Pradesh 2013)
into two parts of equal areas, then (1) a (2) 3a2 – b
PQ (3) 2b (4) (a + b) (a – b)
(Karnataka 2014) 292. If the sum of the roots of the equation ax2 + bx + c = 0 is
equal to product of their reciprocal, then:
P (Madhya Pradesh 2013)
(1) a2 + bc = 0 (2) b2 + ca = 0
(3) c2 + ab = 0 (4) b + c = 0
293. In the figure, triangle ABC is similar to triangle EDC :

M N A

D
Q R B C

1 2 2
(1) (2) E
2 1 2
If we have AB = 4 cm, ED = 3 cm, CE = 4.2 cm and CD = 4.8 cm,
2 1 1 then the values of CA and CB respectively are
(3) (4) (Madhya Pradesh 2013)
2 2 1
(1) 6 cm, 6.6 cm (2) 4.8 cm, 6.6 cm
286. The arithmetic mean of two given positive numbers is 2. If (3) 5.4 cm, 6.4 cm (4) 5.6 cm, 6.4 cm
the larger is increased by 1, the geometrical mean of the
1
numbers becomes equal to the arithmetic mean of the given 294. cot 2 is equal to : (Madhya Pradesh 2013)
numbers. Then, the harmonic mean of the given numbers is sin 2
(1) 1 (2) – 1
2 (3) 2 (4) – 2
(1) (2) 2 (Karnataka 2014)
3 295. If b2 – 4ac > 0 then the roots of quadratic equation ax2 + bx
+ c = 0 is: (Rajasthan 2013)
1 3
(3) (4)
2 2 b b2 4ac b b2 4ac
(1) (2)
x a x b x c 2a 2a 2a 2a
287. If 3 0,a 0, b 0,c 0 then the
b c c a a b b b2 4ac b b2 4ac
value of x is: (Delhi 2014) (3) (4)
2a 2a 2a 2a
2 2 2 296. The sum of the third and seventh terms of an A.P. is 6 and
(1) (a b c ) (2) (a + b + c)
their product is 8, then common difference is:
(3) –(a + b + c) (4) a b c (Rajasthan 2013)
288. If sin x + cosec x = 2, then (sin10 x + cosec10 x) is equal to: (1) ± 1 (2) ± 2
(Delhi 2014) 1 1
(1) 3 (2) 0 (3) (4)
2 4
(3) 1 (4) 2
EBD_7332
64
B- NTSE Stage 1 Question Bank

297. In a given figure in trapazium ABCD if AB || CD, then value


area DABC 16
of x is: (Rajasthan 2013) 305. ABC ~ PQR and = If PQ = 18 cm and
area DPQR 9
A B BC = 12 cm. then AB and QR are respectively (Delhi 2012)
(1) 9 cm, 24 cm (2) 24 cm, 9 cm
2 2
x– (3) 32 cm, 6.75 cm (4) 13.5 cm, 16 cm
O 306. E and F are respectively, the mid points of the sides AB and
5 AC of ABC and the area of the quadrilateral BEFC is k
2 x+ 5
times the area of ABC. The value of k is (Delhi 2012)
D C
1 3
(1) (2) 3 (3) (4) 4
2 4
29 8
(1) (2) 307. In the figure, PQ is a chord of a circle with centre O and PT
8 29
is the tangent at P such that QPT = 70°. Then the measure
1 of PRQ is equal to (Delhi 2012)
(3) 20 (4)
20 Q
298. If points (x, 0), (0, y) and (1, 1) are collinear then the relations O
is: (Rajasthan 2013)
R
(1) x + y = 1 (2) x + y = xy
(3) x + y + 1 = 0 (4) x + y + xy = 0 P T

299. If sin (A + B) cos (A – B) and O < A + B < 90o, if A > B then (1) 135° (2) 150° (3) 120° (4) 110°
the value of A and B are: (Rajasthan 2013) 308. AB and CD are two parallel chords of a circle such that AB
o
(1) A = 45 , B = 15 o (2) A = 60o, B = 30o = 10 cm and CD = 24 cm. If the chords are on the opposite
o
(3) A = 0 , B = 30 o (4) A = 30o, B = 0o sides of the centre and the distance between them is 17 cm,
300. If the length of diagonal of a square is (a + b), then the area the radius of the circle is (Delhi 2012)
of the square will be : (Goa 2013) (1) 14 cm (2) 10 cm (3) 13 cm (4) 15 cm
309. From a 25 cm × 35 cm rectangular cardboard, an open box is
1 to be made by cutting out identical squares of area 25 cm2
(1) (a + b)2 (2) (a b)2
2 from each comer and turning up the sides. The volume of
the box is (Delhi 2012)
1 2 2
(3) (a 2 b2 ) (a b )
(4) (1) 3000 cm3 (2) 1847 cm3
2 (3) 21875 cm3 (4) 1250 cm3
301. A shopkeeper prefers to sell his goods at the cost price but 2
310. Number of real solutions of (x 2 –7x 11) x
11x 30
uses a weight of 800 gm instead of 1 kg weight. He earns a 1 is :
profit of (Delhi 2012) (Punjab 2012)
(1) 2% (2) 8% (3) 20% (4) 25% (1) 4 (2) 5
302. The compound interest on a certain sum for two years is (3) 6 (4) No solution
` 618, whereas the simple interest on the same sum at the 311. If tan2 . tan2 + tan2 . tan2 + tan2 . tan2
same rate for two years is ` 600. The ratio of interest per + 2 tan2 . tan2 . tan2 = 1, then the value of sin2 + sin2
annum is (Delhi 2012) + sin2 is (Punjab 2012)
(1) 18% (2) 9% (3) 6% (4) 3% 1
303. If log12 27 = a, then log6 16 is (Delhi 2012) (1) 0 (2) – 1 (3) 1 (4)
2
4(3 – a) 4(3 a) 312. If 3 sin + 5 cos = 5, then the value of 5 sin – 3
(1) (2) cos = ? (Punjab 2012)
(3 a) (3 – a)
(1) ± 4 (2) ± 3 (3) ± 5 (4) ± 2
(3 a) (3 – a) 2010
(3) (4)
313. 2 7 3 3 4020 55 12 21 ? (Punjab 2012)
4(3 – a) 4(3 a)
(1) – 1 (2) 1 (3) 0 (4) 2
304. Four circular cardboard pieces. each of radious 7 cm are 314. If the quotient of x4 – 11x3 + 44x2 – 76x + 48. When divided
placed in such a way that each piece touches the two other by (x2 – 7x + 12) is Ax2 + Bx + C, then the descending order
pieces. The area of the space enclosed by the four pieces is of A, B, C is : (Punjab 2012)
(1) 21 cm2 (2) 42 cm2 (Delhi 2012) (1) A, B, C (2) B, C, A
(3) 84 cm 2 (4) 168 cm2 (3) A, C, B (4) C, A, B
Solved Questions B-65

315. In the given figure, AD = AE and BAD = EAC, then 2


(Punjab 2012) 3cos 43 cos 37 .cosec53
322. ?
A
sin 47 tan 5 .tan 25 .tan 45 .tan 65 .tan 85
(1) 7 (2) 0 (Punjab 2012)
(3) 1 (4) 8
323. In the figure, triangle ABC is similar to triangle EDC :
(Madhya Pradesh 2012)
2y+3 43
A

10 x–1 D
B D E C B C
(1) x = 11 (2) x = 13
(3) y = 21 (4) y = 11
316. In the given circle with centre ‘O’, the mid points of two E
equal chords AB & CD are K & L respectively. If we have AB = 4 cm, ED = 3 cm, CE = 4.2 cm and CD = 4.8
If OLK = 25°, Then LKB = ? (Punjab 2012) cm, then the values of CA and CB respectively are
(Madhya Pradesh 2012)
(1) 6 cm, 6.6 cm (2) 4.8 cm, 6.6 cm
B (3) 5.4 cm, 6.4 cm (4) 5.6 cm, 6.4 cm
K
A 324. If two circles are such that one is not contained in the other
and are non-intersecting, then number of common tangents
O are : (Madhya Pradesh 2012)
C (1) One (2) Two (3) Three (4) Four
L
D 1 1
325. If the mean of x and is M, then the mean of x2 and is :
(1) 125° (2) 115° (3) 105° (4) 90° x x2
317. The length 'L' of a tangent, drawn from a point 'A' to a circle (Madhya Pradesh 2012)
4 M2
is of the radius 'r'. The shortest distance from A to the (1) M 2 (2)
3 4
circle is : (Punjab 2012) (3) 2M2 – 1 (4) 2M2 + 1
1 1 2 326. Number of zero's in the product of
(1) r (2) r (3) L (4) L 5 × 10 × 25 × 40 × 50 × 55 × 65 × 125 × 80
2 2 3
(Madhya Pradesh 2012)
318. A set of numbers has the sum ‘S’. Each number of the set is
(1) 8 (2) 9 (3) 12 (4) 13
increased by 20, them multiplied by 5, and then decreased
by 20. The sum of the numbers in the new set thus obtained 327. If in 3 3 5, x 3 and y 3 5 , then its rationalising
is : (Punjab 2012) factor is : (Madhya Pradesh 2012)
(1) S + 20 n (2) 5S + 80 n (1) x + y
(3) 5S + 4 n (4) 5S (2) x – y
319. The centre of a clock is taken as origin. At 4.30 pm, the (3) x5 + x4y + x3y2 + x2y3 + xy4 + y5
equation of line along minute hand is x = 0. Therefore at this (4) x5 – x4y + x3y2 + x2y3 + xy4 + y5
instant the equation of the line along the hour hand will be: 328. A farmer divides his herd of x cows among his 4 sons so,
(Punjab 2012) that first son gets-one-half of the herd, the second son gets
(1) x + y = 0 (2) x – y = 0 one-fourth, the third son gets one-fifth, and the fourth son
x gets 7 cows, then the value of x is :
(3) y = 2x (4) y (Madhya Pradesh 2012)
2
(1) 100 (2) 140 (3) 160 (4) 180
320. A conical vessel of radius 6 cm and height 8 cm is completely 329. If A merchant purchases 9 pens and sells 8 pens at the cost
filled with water. A metal sphere is now lowered into the price of 9 pens, then his profit percent is :
water. The size of the sphere is such that when it touches (Madhya Pradesh 2012)
the inner surface, it just gets immersed. The fraction of water
that overflows from the conical vessel is : (Punjab 2012) 15 2 1 1
(1) 5 (2) 8 (3) 12 (4) 11
3 5 7 5 17 3 2 9
(1) (2) (3) (4) 330. Marked price of a Saree is ` 600 and is available on ` 450.
8 8 8 16
Rate of discount is : (Madhya Pradesh 2012)
321. If the eight digit number 2575d568 is divisible by 54 and 87, (1) 25% (2) 30% (3) 15% (4) 40%
the value of the digit 'd' is : (Punjab 2012)
(1) 4 (2) 7 (3) 0 (4) 8
EBD_7332
66
B- NTSE Stage 1 Question Bank

331. PQRS is a parallelogram and M, N are the mid-points of PQ 339. The highest power of 2 by which the product of first 100
and RS respectively. Which of the following is not true ? counting numbers can be divided without any remainder is
(Madhya Pradesh 2012) _________. (Odisha 2012)
P (1) 97 (2) 96 (3) 95 (4) 94
S
340. If x + y + z = 1, x² + y² + z² = 2 and x³ + y³ + z³ = 3 then the
value of xyz is __________. (Odisha 2012)
M N (1) 1/5 (2) 1/6 (3) 1/7 (4) 1/8
341. Four whole numbers added three at a time give sums
Q R 180, 197, 208 and 222 respectively. The largest of the four
(1) RM trisects QS (2) PN trisects QS numbers is ________. (Odisha 2012)
(3) PSN ~ QMR (4) MS is not parallel to QN (1) 87 (2) 88 (3) 89 (4) 90
332. For which values of ‘a’ and ‘b’ does the following pair of 2 f (n) 1
linear equations have an infinite number of solutions- 342. If f (n 1) , n 1, 2, ...... and f (1) = 2, then
2
2x + 3y = 7, (a – b) x + (a + b) y = 3a + b – 2
(Rajasthan 2012) f (101) = ____________. (Odisha 2012)
(1) a = 5, b = 1 (2) a = 4, b = 2 (1) 53 (2) 52 (3) 51 (4) 50
(3) a = 1, b = 5 (4) a = 2, b = 4 343. The value of sin 21° + sin22° + …………. + sin2 89° is
333. In a given figure in trapezium ABCD if AB CD then value __________. (Odisha 2012)
of x is (Rajasthan 2012) 1
(1) 1 (2) 44 (3) 44 (4) 45
A B 2
344. If A + B + C = and m C is obtuse then tan A. tan B is
2 x–2
O ____________. (Odisha 2012)
5 (1) > 1 (2) < 1 (3) = 1 (4) 1
2x+ 5 345. The area of a circle inscribed in an equilateral triangle is 48
D C square units. The perimeter of the triangle is __________
29 8 1 units. (Odisha 2012)
(1) (2) (3) 20 (4) (1) 24 (2) 32 (3) 36 (4) 72
8 29 20
346. The reminder of 20052002 + 20022005 when divided by 200 is
334. If the angle of elevation of sun increases from 0° to 90° then _________. (Odisha 2012)
the change in length of shadow of Tower will be-
(1) 0 (2) 1 (3) 2 (4) 3
(1) No change in length of shadow (Rajasthan 2012)
347. Let ABCD be a cyclic quadrilateral inscribed in a circle of
(2) Length of shadow increases
(3) Length of shadow decreases radius 1 unit. If AB.BC.CD.DA 4, then the quadrilateral
(4) Length of shadow will be zero ABCD is a _________. (Odisha 2012)
335. The median and mode of a frequency distribution are 525 (1) Parallelogram (2) Square
and 500 then mean of same frequency distribution is- (3) Rectangle (4) Rhombus
(1) 75 (2) 107.5 (Rajasthan 2012) 348. On increasing each of the radius of the base and the height
(3) 527.5 (4) 537.5 of a cone by 20% its volume will be increased by _________.
336. The number of straight line drawn from one point to any (Odisha 2012)
other point are- (Rajasthan 2012) (1) 20% (2) 40% (3) 72.2% (4) 72.8%
(1) 4 (2) 3 (3) 2 (4) 1 1 1
337. In a given figure PQ ST, PQR = 110°, RST = 130°, then 349. If, x 1 and y 1 , then the value of xyz is
value of QRS is- (Rajasthan 2012) y z
(West Bengal 2012)
S T
(1) 1 (2) – 1 (3) 0 (4) – 2
130° 350. The difference between a discount of 40% of Rs,1000 and
P two successive discount of 30% and 10% on the same
Q
amount is (West Bengal 2012)
110°
? ? (1) 0 (2) 20 (3) 30 (4) 40
R
(1) 20° (2) 50° (3) 60° (4) 70°
351. Find out the value of sec 2 cosec2
338. The bisectors of angles of a parallelogram makes a figure (West Bengal 2012)
which is- (Rajasthan 2012) (1) tan + cot (2) sec + cosec
(1) Rectangle (2) Circle 2
sec
(3) Pentagon (4) Octagon (3) (4) tan – sec
cosec 2
Solved Questions B-67

352. A copper wire of certain length is turned into a square first 362. In the adjacent figure, if AOC =110°, then the value of D
and then into a circle. Which one of them is larger? and B respectively (Andhra Pradesh 2012)
(West Bengal 2012) D
(1) square
(2) circle
O
(3) both of them are equal
(4) none of the above A C
353. The greatest angle of a cyclic quadrilateral ABCD in
B
which A = (2x – 1)°, B = (y + 5)°, C = (2y + 15)° and
D = (4x – 7)° is (West Bengal 2012) (1) 55°, 125° (2) 55°, 110°
(1) 115° (2) 120° (3) 125° (4) 130° (3) 110°, 25° (4) 125°, 55°
354. A person borrows a certain amount of money at 5% simple 363. In a XYZ, if the internal bisector of X meets YZ at ‘P’,
interest and he lends that amount to another person at 10% then …. (Andhra Pradesh 2012)
compound interest, after three years his profit is ` 905. Find Z
out the amount ` he borrowed. (West Bengal 2012)
(1) ` 2000 (2) ` 3000
P
(3) ` 5000 (4) ` 8000
x sec 2 81 X Y
355. If, tan 9 then, the value of is
y 1 cot 2 81 XY XZ YZ XY XZ
(West Bengal 2012) (1) (2)
XZ PZ PZ YP
x3 x4 x5 y2 XY PZ XZ YP
(1) (2) (3) (4) (3) (4)
y3 y4 y5 x2 XZ YP XY YZ
364. Two poles of height 6 m and 11 m stand vertically upright on
356. If x 2 y 2 10 (2 2 x 4 2 y ) then the value of (x + y) a plane ground. If the distance between their feet is 12m, the
is (West Bengal 2012) distance between their tops is…….
(1) 4 2 (2) 3 2 (3) 6 2 (4) 9 2 (Andhra Pradesh 2012)
357. A greedy businessman cheats both the seller and buyer by (1) 12 m (2) 14 m (3) 13 m (4) 11 m
20%. His real profit is (West Bengal 2012) 4 4
(1) 43% (2) 45% (3) 44% (4) 40% 365. The value of
3 6
a9
6 3
a9 is ….
358. If sum of the digits of any integer in between 100 and 1000 is
subtracted from that integer, then the result is always (Andhra Pradesh 2012)
divisible by (West Bengal 2012)
(1) a 16 (2) a 12 (3) a 8 (4) a 4
(1) 7 (2) 6 (3) 5 (4) 9
366. If the ratio of the legs of a right-angled triangle is 1:2, then
359. By eliminating from x = a cos3 , y = a sin3 the expression
is (West Bengal 2012) the ratio of the corresponding segments of the hypotenuse
made by a perpendicular upon it from the vertex will be
(1) x y a (2) x2 + y2 = a2 (Andhra Pradesh 2012)
2/3
(3) x + y = a2/3 2/3 (4) none of the above (1) 4 : 1 (2) 1: 2 (3) 1 : 2 (4) 1: 5
360. In the figure, ABCD is a square whose sides are of length
2 cm. The midpoints of sides AB, BC, CD and DA are 367. If f : R R; g : R R are functions defined by f (x) = 3x – 1;
respectively P,Q,R and S. Four arcs are drawn with A,B,C g(x) = x 6 , then the value of (gof–1) (2009) is
and D as centres and AP as radius. Area of shaded portion (Andhra Pradesh 2012)
is (West Bengal 2012) (1) 26 (2) 29 (3) 16 (4) 15
368. In the diagram, a square ABCD has a side with a length of 6
(1) 4 sq. cm A P B cm. Circular arcs of radius 6 cm are drawn with centres B and
3
D. What is the area of the shaded region in sq. cm. ?
(2) (4 – ) sq.cm (Andhra Pradesh 2012)
S Q
A B
(3) 2 sq. cm
2
D R C
(4) 3 sq. cm
7 D C
361. If A = sin2 + cos4 , then for all real values of (1) 18 (2) 36
(West Bengal 2012) (3) 18 – 24 (4) 18 – 36
3 369. How many numbers between 3000 and 4000 can be formed
(1) 1 A 2 (2) £ A £1 from the digits 3, 4, 5, 6, 7 and 8; no digits being repeated in
4 any number (Andhra Pradesh 2012)
13 3 13 (1) 20 Nos. (2) 15 Nos.
(3) A 1 (4) A (3) 60 Nos. (4) 120 Nos.
16 4 16
EBD_7332
68
B- NTSE Stage 1 Question Bank

370. If log10 2 = 0.3010, then the number of digits in 25650 is or ( AOB) = 3 sq. cm
(Andhra Pradesh 2012) or ( COD) = 12 sq. cm
(1) 120 (2) 121 (3) 256 (4) 50 or ( ABCD) = ................
371. If sin A, cos A and tan A are in Geometric Progression, then (1) 27 sq. cm (2) 45 sq. cm
cot6 A – cot2 A is (Andhra Pradesh 2012) (3) 36 sq. cm (4) 18 sq. cm
(1) 2 (2) 4 (3) 3 (4) 1 381. In the right-angled ABC, cot A · cot B · cot C = ................
372. A chess-board contains 64 equal squares and the area of (Maharashtra 2012)
each square is 6.25 cm2. A border (inside) round the board is (1) 1 (2) 0
2 cm. wide. The length of the chess-board is …. (3) 2 (4) None of these
(Andhra Pradesh 2012) 382. In the figure given below, radius of a greater circle is r cm.
(1) 8 cm (2) 24 cm (3) 12 cm (4) 16 cm Find the area of non-shaded portion :(Maharashtra 2012)

373. The value of log . 2 2 2 2 2 is….


2
(Andhra Pradesh 2012)
O
31 31 31
(1) (2) (3) (4) None
32 4 8
374. For simultaneous equations 3x + ay = 21 and 2x + 2y = 12; Dx
= – 6; then what is the value of a? (Maharashtra 2012)
(1) 2 (2) 3 (3) 4 (4) 1
375. A bus takes 5 hours more than a train to cover the distance r2 3 r2
of 900 km from Vardha to Pune. If speed of the train is (1) sq. cm (2) sq. cm
4 4
15 km/hr more than that of the bus, then what is the speed of
bus per hour? (Maharashtra 2012) 4 r2
(1) 60 km/hr (2) 75 km/hr (3) 2 r2 sq. cm (4)
sq. cm
3
(3) 55 km/hr (4) 45 km/hr 383. In the figure given below, points P and Q are mid points on
376. The premodal class and postmodal class rectangles have the sides AC and BP respectively. Area of each part is
same height. If the modal class is 40–60, then what is mode? shown in the figure, then find the value of x + y.
(Maharashtra 2012) (Maharashtra 2012)
(1) 40 (2) 60 (3) 100 (4) 50 A
377. If P(A) + P(B) = 1; then which of the following option explains
the event A and B correctly? (Maharashtra 2012)
(1) Event A and B are mutually exclusive, exhaustive and
complementary events. y P
(2) Event A and B are mutually exclusive and exhaustive R
Q
events. 3 x
(3) Event A and B are mutually exclusive and 7
complementary events.
(4) Event A and B are exhaustive and complementary B C
events. (1) 11 (2) 4
378. Which of the following is a false statement? (3) 7 (4) 18
(Maharashtra 2012) 384. ‘O’ is a centre of a circle, AC is a tangent to a circle at point
(1) Median can be determined graphically. A. If OAC is an isosceles triangle, then find the measure of
(2) Mean can be determined from the graph. OCA. (Maharashtra 2012)
(3) Mean cannot be determined by inspection. (1) 30° (2) 60°
(4) Mode can be determined graphically. (3) 90° (4) 45°
379. If sin2 + cosec2 = 6, then sin + cosec = ............... . 385. A rectangular paper of dimensions 6 cm and 3 cm is rolled to
(Maharashtra 2012) form a cylinder with height equal to the width of the paper.
(1) 3 2 (2) 2 2 (3) 4 2 (4) Its radius of the base is (Karnataka 2012)
2
380. ABCD is a trapezium, AB || DC. Diagonals of trapezium 3 6
(1) (2)
intersect to each other at point O: (Maharashtra 2012) 2
A 10 cm B 9 3
(3) (4)
2
O 386. A right circular cone is cut by two planes parallel to the
base and trisecting the altitude What is the ratio of the
volumes of the three parts : top, middle, bottom respectively:
(1) 1 : 7 : 19 (2) 1 : 2 : 3 (Karnataka 2012)
(3) 1 : 8 : 27 (4) 1 : 7 : 18
C
387. X is a matrix then X – X is : (Karnataka 2012)
D 20 cm
(1) symmetric matrix (2) skew symmetric matrix
(3) zero matrix (4) identity matrix
Solved Questions B-69

SECTION 5. HISTORY
1. Who was the ruler of France during the French revolution? (Andhra Pradesh-2020-21)
(Delhi 2020-21) (A) The first printing press was made by Johannes Gutenberg.
(1) Louis XV (2) Louis XIV (B) Paper and printing with blocks were first developed
(3) Louis XX (4) Louis XVI by the Chinese.
2. Hitler was born in (Delhi 2020-21) (C) The first printed book in the press was “The Bible”
(1) France (2) Austria Codes:
(3) Germany (4) Poland (A) (A), (B) and (C) are correct
3. Who published ‘The folklores of southern India’? (2) (A) and (B) are correct
(Delhi 2020-21) (3) (B) and (C) are correct
(1) Rabindra Nath Tagore (2) Natesa Sastri (4) (A) and (C) are correct
(3) Mahatma Gandhi (4) Pattabhi Srimalu 12. Oceans in descending order in terms of their size:
4. Who formed the secret society “young Italy”? (Andhra Pradesh-2020-21)
(Delhi 2020-21) (1) Pacific, Atlantic, Indian, Antarctic, Arctic
(1) Otto Van Bismarck (2) General Wan Run (2) Pacific, Atlantic, Indian, Arctic, Antarctic
(3) Giuseppe Mazzini (4) King Victor Emmanual II (3) Pacific, Indian, Atlantic, Arctic, Antarctic
5. Which of the following was not related to the Gandhi Irwin (4) Pacific, Arctic, Atlantic, Indian, Antarctic
Pact 1931? (Delhi 2020-21) 13. Who among the following proposed a social contract
(1) Government agreed to release the Political prisoners. between people and their representatives?
(2) Gandhi Ji consented to participate in the Second Round (Andhra Pradesh-2020-21)
Table Conference. (1) Rousseau (2) John Locke
(3) Formation of Simon Commission. (3) Thomas Jefferson (4) Montesquieu
(4) Gandhi Ji called off Civil Disobedience Movement. 14. “God save our Noble King” was/is the National Anthem of
6. In Swaraj Flag (1921), Self Help was represented by (Andhra Pradesh-2020-21)
(Delhi 2020-21) (1) Britain (2) Italy
(1) Flower (2) Crescent Moon (3) Germany (4) France
(3) Two Ox (4) Spinning wheel 15. The first language state was formed in Independent India:
7. Which one of the following pairs is NOT correctly matched? (West Bengal-2020-21)
(Andhra Pradesh-2020-21) (1) Andhra Pradesh (2) West Bengal
(1) Michelangelo – Statue of David (3) Tamil Nadu (4) Gujarat
(2) Leonardo da Vinci – Mona Lisa 16. The writer of the book named “Chhere Asha Gram” was
(3) Filippo Brunelleschi – The Last Supper (West Bengal-2020-21)
(4) Albrecht Durer – Praying Hands (1) Manikuntala Sen (2) Dakshina Ranjan Basu
8. Match List-I with List-II and select the correct answer using (3) Sankha Ghosh (4) Selina Hossain
the codes given below. (Andhra Pradesh-2020-21) 17. Who was known as the “Tsar the Liberator”?
List-I (Author) List-II (Book) (West Bengal-2020-21)
A. Machiavelli (i) Praise of Folly (1) Tsar Nicholas I (2) Tsar Nicholas II
B. Montesquieu (ii) Utopia (3) Tsar Alexander I (4) Tsar Alexander II
C. Thomas More (iii) The spirit of Laws 18. The day 24th October 1929 was marked as ‘Black Thursday’
D. Erasmus (iv) The prince in the USA because (West Bengal-2020-21)
Codes: (1) Terrorist Attack
(1) A - i, B - ii, C - iv, D - iii (2) A - i, B - ii, C - iii, D - iv (2) Natural Calamity
(3) A - iv, B - iii, C - ii, D - i (4) A - iv, B - ii, C - i, D – iii (3) The Great Economic Depression
9. Bastille prison was destroyed in France on: (4) Change of Political Background
(Andhra Pradesh-2020-21) 19. France was named as “a museum of economic errors” by
(1) June 20th 1778 (2) May 5th 1786 (West Bengal-2020-21)
(3) July 14th 1789 (4) July 4th 1798 (1) Rousseau (2) Adam Smith
10. This national leader fasted on the First Independence Day (3) Montesquieu (4) Quesnay
of the Nation. He said, “If I am to die by the bullet of a mad 20. Jagannath Singh Dhol was the leader of
man, I must do so smiling.” Who was he? (A) Kol Rebellion (West Bengal-2020-21)
(1) Jawaharlal Nehru (Andhra Pradesh-2020-21) (B) Santhal Rebellion
(2) Gandhiji (C) Munda Rebellion
(3) Subhash Chandra Bose (D) Chuar Rebellion Ans
(4) Sardar Vallabhbhai Patel The incident of Chauri Choura took place in
11. Consider the following statements and select the CORRECT (1) 1919 AD (2) 1920 AD
answer using the codes given below. (3) 1922 AD (4) 1925 AD
EBD_7332
70
B- NTSE Stage 1 Question Bank

21. Which period is known as “Arab spring”? 34. Who published the newspaper “Som Prakash”?
(Rajasthan-2020-21) (1) Ishwar Chandra Vidyasagar (Bihar-2020-21)
(1) 2010-2013 (2) 2007-2010 (2) Bal GangadharTilak
(3) 2006-2016 (4) 2012-2015 (3) Ram Mohan Roy
22. Arrange the following events is chronological order and (4) M. G Ranade
select the correct answer from the codes given below: 35. Where is Sanchi stupa situated? (Uttar Pradesh-2020-21)
(Rajasthan-2020-21) (1) Gaya (2) Lumbini
(A) Korea War (B) Swej Crisis (3) Samath (4) Bhopal
(C) Congo Crisis (D) Somalia Crisis 36. Which relegion did Ashoka adopt?
Codes: (Uttar Pradesh-2020-21)
(1) A, C, B, D (2) A, B, C, D (1) Buddhism (2) Hinduism
(3) C, B, D, A (4) B, A, D, C (3) Jainism (4) Shaivism
23. Which one of the following was not included among the 37. Which is the oldest language of South India?
Axis nations during the First World War? (Uttar Pradesh-2020-21)
(Rajasthan-2020-21) (1) Telugu (2) Kannada
(1) Italy (2) Germany (3) Tamil (4) Malayalam
38. Who among the following had introduced market control
(3) Hungary (4) Turkey
policy? (Uttar Pradesh-2020-21)
24. Who was the writer of “Ananda Math”?
(1) Balban (2) Alauddin Khilji
(1) Bal Gangadhar Tilak (Rajasthan-2020-21)
(3) Muhammad Bin Tuglaq (4) Jalaluddin Khilji
(2) Bankim Chandra Chatterjee
39. Who among the following founded the Vijay Nagar empire?
(3) Dayanand Saraswati (Uttar Pradesh-2020-21)
(4) Raja Rammohan Roy (1) Vijay Rai (2) Harihar and Bukka
25. Dhapi Dadi is related to which village of Sikar peasant (3) Pushyamitra (4) Rana Sanga
movement? (Rajasthan-2020-21) 40. Which of the following cities was built by Akbar?
(1) Kudan (2) Palthana (Uttar Pradesh-2020-21)
(3) Katrathal (4) Ghassu (1) Daulatabad (2) Fatehpur Sikri
26. Which newspaper compared the Neemuchana massacre to (3) Agra (4) Delhi
the Jallianwala Bagh massacre? (Rajasthan-2020-21) 41. In which year Vasco da Gama came to India?
(1) Rajputana (2) Riyasat (Uttar Pradesh-2020-21)
(3) Rajasthan (4) Kisan (1) 1350 AD (2) 1450 AD
27. Which of the following was first invented? (3) 1498 AD (4) 1598 AD
(Rajasthan-2020-21) 42. Who founded the Indian National Congress?
(1) Flying Shuttle Loom (2) Water Frame (Uttar Pradesh-2020-21)
(3) Spinning Jenny (4) Powerloom (1) Mahatma Gandhi (2) Queen Victoria
28. Which dynasty succeeded the Bourbon dynasty after the (3) Sardar Patel (4) A. O. Hume
1830 revolution? (Bihar-2020-21) 43. Which among the following is the autobiography of
(1) Habsburg (2) Orleans Gandhiji? (Uttar Pradesh-2020-21)
(3) Tsardom (4) None of these (1) India Divided (2) Nation in Making
29. Charles Albert was the King of which country? (3) Neel Darpan (4) MyExperiments with Truth
(Bihar-2020-21) 44. Which one of the following is the oldest mountain system?
(1) Naples (2) Parma (Uttar Pradesh-2020-21)
(3) Modena (4) Sardinia (1) Nilgiri (2) Aravali
30. Who was the author of ‘War and Peace”? (Bihar-2020-21) (3) Satpura (4) Vindhya
(1) Tolstoy (2) Karl Marx 45. Who among the following was the first Peshwa of Shahu?
(3) Lenin (4) St. Simon (Tamil Nadu-2020-21)
31. Who built the Angkorwat Temple? (Bihar-2020-21) (1) Balaji Vishwanath (2) Baji Rao
(1) Jayavarman (2) Suryavarman II (3) Balaji Baji Rao (4) Madhava Rao
(3) Mahendravarman (4) Rudravarman 46. Before French Revolution the political body of France was
32. In 1878 which Viceroy passed the “Vernacular Press Act”? called as ___________ (Tamil Nadu-2020-21)
(Bihar-2020-21) (1) Duma (2) Reich Stag
(1) Lord Ripon (2) Lord Lytton (3) Lok Sabha (4) Estates General
(3) Lord Curzon (4) Lord Chelmsford 47. The war between Britain and France had centred on the
33. After which incident Rabindranath Tagore surrendered the control of colonies in North America
(Tamil Nadu-2020-21)
title “Knight”? (Bihar-2020-21)
(1) The American war of Independence
(1) Rowlatt Act
(2) Seven years war
(2) Khilafat Movement
(3) The Crusades
(3) Jallianwala Bagh Massacre
(4) Battle of Bunker Hill
(4) Coming of Simon Commission
Solved Questions B-71
48. Arrange the following events in the chronological order (1) dice (yet) (2) horse and the chariot races
(A) Vietnam Nationalist Party (Tamil Nadu-2020-21) (3) boxing (4) Chess
(B) Fascist Party 59. Identify the well-known play of Shakespeare on which
(C) The Union of South Africa Marathi play ‘Natsamrat’ was styled?
(D) French Communist Party (Maharashtra-2020-21)
(1) (A), (B), (C), (D) (2) (A), (D), (C), (B) (1) Hamlet (2) King Lear
(3) (C), (B), (D), (A) (4) (C), (B), (A), (D) (3) Julius Ceaser (4) Romeo-Juliet
49. Match the following (Tamil Nadu-2020-21) 60. Emperor Menelik was the ruler of which African Country?
COLUMN – I COLUMN – II (Maharashtra-2020-21)
(A) Hundred Days (i) 1832 (1) Ethiopia (2) Cape Colony
of Reform (3) Kenya (4) Uganda
(B) Russian (ii) 1898 61. The great military genius of the Mongols
Revolution (Maharashtra-2020-21)
(C) First Opium War (iii) 1911 (1) Napoleon Bonaparte (2) Jan Zizka
(D) The Chinese (iv) 1917 (3) Alexander (4) Genghis Khan
Revolution 62. Who is known as ‘Waterman of India’?
(1) (A) – (iv), (B) – (iii), (C) – (ii), (D) – (i) (1) Shree Anna Hajare (Maharashtra-2020-21)
(2) (A) – (iv), (B) – (i), (C) – (ii), (D) – (iii) (2) Sardar Vallabhbhai Patel
(3) (A) – (ii), (B) – (iv), (C) – (i), (D) – (iii) (3) Dr. Rajendra Singh Rana
(4) (A) – (ii), (B) – (iii), (C) – (i), (D) – (iv) (4) Shree Sundarlal Bahuguna
50. Liberty, Equality and Fraternity were the important slogans 63. Choose the correct statements related to the Ryotwari
during the ______ (Tamil Nadu-2020-21) system : (Karnataka-2020-21)
(1) American Revolution (2) French Revolution (A) The charge of collecting the revenue and paying it to
(3) Russian Revolution (4) Chinese Revolution the company was given to the village headman.
51. __________ is not a part of the world network biosphere (B) Alexander Reed introduced it in Baramahal Region.
reserves of UNESCO. (Tamil Nadu-2020-21) (C) Farmer had to pay 50% of the produce as revenue to
(1) Nilgiri (2) Agasthiyamalai the Government.
(3) Great Nicobar (4) Kachch (1) an only (2) B and C only
52. Who argued that the prevailing practices of arranging (3) A and B only (4) A, B and C only
historical events in chronological order are not right? 64. In List - A Mahajanapadas and in List - B their Capitals are
(Maharashtra-2020-21) given. Write the correct option by matching them :
(1) Michel Foucault (2) Karl Marx (Karnataka-2020-21)
(3) Leopold Von Ranke (4) Friedrich Hegel List - I List - II
53. Ishwardas Nagar, Bhimsen Saxena were the histories of A. Anga i. Saketha
_________ ‘s times. (Maharashtra-2020-21) B. Kashi ii. Kushinagara
(1) Jahangir (2) Aurangzeb C. Kosala iii. Champa
(3) Shahjahan (4) Akbar D. Malla iv. Varanasi
54. Identify the style of painting that influenced Maratha style v. Kausambi
of painting. (Maharashtra-2020-21) (1) A - ii B- i C - iv D - iii (2) A - iii B- iv C - i D - ii
(1) Art style (2) Varli Kala (3) A-v B-i C - ii D - iv (4) A - ii B- iii C - iv D - v
(3) Rajput Kala (4) Classical Art 65. Select the correct chronological order of wars :
55. Newspaper ________ began to publish articles about the (Karnataka-2020-21)
nationwide situations, books in the native languages and (1) The first Anglo-Sikh war, The second Anglo-Marata war,
the politics in other countries. (Maharashtra-2020-21) The second Anglo-Mysore war, The third Carnatic war.
(1) Maratha (2) Kesari (2) The first Anglo-Sikh war, The second Anglo-Mysore
(3) Deenbandhu (4) Indu Prakash war, The third Carnatic war, The second Anglo-Marata
56. Who was the editor of the periodical name ‘Pragati’ (1929)? war.
(1) Pandit Narendra Sharma (Maharashtra-2020-21) (3) The second Anglo-Marata war, The third Carnatic war,
(2) Balshastri Jambhekar The first Anglo-Sikh war, The second Anglo - Mysore
(3) Traymbak Shankar Shejwalkar war.
(4) Gopal Hari Deshmukh (4) The third Carnatic war, The second Anglo-Mysore war,
57. Identify the wrong pair related to the movie and its The second Anglo-Marata war, The first Anglo Sikh
directions. (Maharashtra-2020-21) war.
(1) Bal Shivaji – Prabhakar Pendharkar 66. Identify the statement that is NOT related to the Bahamani
(2) Dhanyate Santaji Dhanaji – Diankar D. Patil Kingdom : (Karnataka-2020-21)
(3) Bajirao Mastani – Bhalji Pendharkar (1) Asar Mahal was one of the important monuments.
(4) Baji Prabhu Deshpande – Dadasaheb Torane (2) Mir Syed Ali and Abdul Samad were the artists.
58. Which game was not mentioned in ancient Indian literature (3) Kitab-E- Navras, a literary work was written.
and the EPICS? (Maharashtra-2020-21) (4) Shahabuddin Ahmed shifted his capital.
EBD_7332
72
B- NTSE Stage 1 Question Bank

67. Read the statements and find out to who do they belong: (III) Controlled Capitalism
(Karnataka-2020-21) (IV) War Communism
i. He took Deeksha from Govindabhatta. (1) I, II and IV (2) I, III, and IV
ii. His Tatvapadas include Kalaga pada, Alaavi pada and (3) III and IV (4) II and III
Sawal Jawab Pada. 76. Hind Swaraj was written by which writer? (Bihar 2019)
(1) Chaithanya (2) Kanakadasa (1) Vallabh Bhai Patel (2) M.K. Gandhi
(3) Shishunal Shariffa (4) Guru Nanak (3) Raja Gopalachari (4) Tej Bahadur Sapru
68. The List - A contains the names of poets and List - B with 77. Mahatma Gandhi determined to stop which movement after
their works. Select the correct answer from the code given Chauri Chaura incident? (Bihar 2019)
below. (Karnataka-2020-21) (1) Champaran Satyagrah
List-A List - B (2) Khilafat Movement
A. Sree Purusha i. Gadya Chintamani (3) Non Co-operation Movement
B. Vadiba Simha ii. Raghava (4) Civil Disobedience Movement
Pandaviya 78. Which statement is incorrect with regard to “Tebhaga”
C. Durvinita iii. Gajashastra Movement? (Andhra Pradesh)
D. Hemasena iv. Gajastaka (1) This agitation was started in Bengal.
v. Shabdavathara (2) Bigger Landlords participated in this movement.
Codes: (3) This movement was about tenancy reforms.
A B C D (4) This movement was led by Provincial Kissan Sabha.
(1) i iii v iv 79. Arrange the following events in correct chronological order with
(2) ii v iv iii regard to Indian National Movement: (Andhra Pradesh)
(3) iv i ii v A. Quit India Movement
(4) iii i v ii B. Three member Cabinet Mission came to India
69. Read the given statements and identify the correct C. Direct Action Day
sociologist related to all of them : (Karnataka-2020-21) D. Cripps Mission came to India
A. He/she worked as the UGC national fellow and as Head (1) A, B, C, D (2) D, A, B, C
of the Department of Sociology of Bombay University. (3) A, D, B, C (4) D, C, B, A
B. As he/she was following the Marxist ideology in his/ 80. The “Zollverein” is known as (Andhra Pradesh)
her sociological studies, he/she was identified as a (1) Tax (2) Customs Union
Marxist Sociologist. (3) Administrative Union (4) Religious Union
C. His/her ‘The Sociological Background of Indian 81. The following regional military and strategic alliances given
Nationalism’ is the best work. below, is not related to U.S.A. (Andhra Pradesh)
D. Indian Rural sociology, slums and Urbanisation are (1) SEATO (2) CENTO
his/her major works. (3) WARSAW (4) NATO
(1) Iravati Karve (2) C. Parvathamma 82. Identify the given regions in the map with their
(3) M.N. Srinivas (4) A.R. Desai corresponding European settlements in India and select
70. The First Literary Source is - (Uttar Pradesh 2019) the correct option using the codes given below:
(1) Rigveda (2) Samveda (Karnataka 2019)
(3) Yajurveda (4) Atharvaveda
71. During whose reign Megasthenes visited to India?
(Uttar Pradesh 2019)
(1) Ashoka (2) Harsh Vardhan
(3) Chandragupta Maurya (4) Kumar Gupta
72. Which dynasty was ruling over North India at the time of
Alexander’s invasion? (Uttar Pradesh 2019)
(1) Nanda (2) Maurya
(3) Shunga (4) Kanva
73. The Name of Shershah in childhood was-
(Uttar Pradesh 2019) A D
(1) Hasan (2) Farid
(3) Sher Khan (4) None of the above B
74. Mazzini, the founder of young Italy, conducted the slogan
(Bihar 2019)
(I) God (II) The Revolutionaries
C
(III) People (IV) Italy
(1) I, II and III (2) I, II, and IV
(3) I, III and IV (4) II, III and IV
75. Lenin finally stressed upon (Bihar 2019)
(I) Nationalisation of industries
(II) Collective farms
Solved Questions B-73
(1) A - Ahmedabad, B - Bharuch, C -Machilipatnam, 86. Match the column ‘A’ with the column ‘B’ and choose the
D - Chandranagar correct answer: (Karnataka 2019)
(2) A - Chandranagar, B - Ahmedabad, C - Bharuch, List-A List-B
D - Machilipatnam A. Veer Savarkar i. Nagpur
(3) A - Bharuch, B - Chandranagar, C - Machilipatnam, International Airport
D - Ahmedabad B. Lal Bahadur Shashtri ii. Bhuvaneshwar
(4) A - Machilipatnam, B - Bharuch, C - Ahmedabad, International Airport
D - Chandrangar C. Babasaheb Ambedkar iii. Port Blair
83. Identify the correct statements with regard to the Solanki International Airport
dynasty: (Karnataka 2019) D. Sardar Vallabhbhai iv. Ahmedabad
A. Chandradeva was the founder of this dynasty. International Airport
B. Moolaraja II defeated Mohammed Ghazni near Mount v. Varanasi
Abu. (1) A-ii, B-v, C-iv, D-i (2) A-iii, B-v, C-i, D-iv
C. During the rule of this dynasty, the famous Jain (3) A-iv, B-iii, C-i, D-ii (4) A-iii, B-i, C-iv, D-ii
Scholar Hemachandra compiled Prakruth dictionary 87. Characteristics of culture is given below. Find the one which
‘Deshimala’. is not a characteristic of culture: (Karnataka 2019)
D. Ulugh Khan and Nusrat Khan, the military generals of (1) Culture is learnt
Allauddin Khilji defeated Karnadeva (2) Culture is transmissive
(1) Only A, B and C are correct (3) Culture is a static Phenomena
(2) Only B, C and D are correct (4) Culture is continuous and cumulative
(3) Only A, C and D are correct 88. Where we can see the Murals of Maratha style in the old
(4) Only C and D are correct wadas in Maharashtra? (Maharshtra 2019)
84. Observe the following pictures of social reformers and (1) Pune (2) Satara
identify the order of the statements correctly related to (3) Solapur (4) Kolhapur
them: (Karnataka 2019) 89. Identify the name of the gentleman, who started The First
A. Dedicated her life for the betterment of women of India English Newspaper of India. (Maharshtra 2019)
and established ‘Mukti Mission’. (1) Balshashtri Jambhekar (2) Bhau Mahajan
B. She opposed the exploitation of women in her book (3) James Augustus Hickey (4) Sir John Marshal.
‘Stri Purusha Tulana’. 90. Colour Television was introduced on _________ in India.
C. While treating the patients of plague along with her (Maharshtra 2019)
son, she died of it. (1) 23 July, 1927 (2) 15 September, 1959
D. Called for universal brotherhood without distinction (3) 1 May, 1972 (4) 15 August, 1982
of race, creed, sex, caste and colour. 91. Who of the following was not the courtier of Kanishka?
(Rajasthan 2019)
(1) Charaka (2) Megasthenes
(3) Nagarjuna (4) Ashwaghosha
92. Who was the writer of ‘Mudrarakshasa’?
(Rajasthan 2019)
(1) Kalidasa (2) Vishakhadatta
(3) Amar Singh (4) Sudraka
Annie Besant Savitri Bai Phule Tarabai Shinde Pandita Rama Bai 93. The fourth Buddhist conference was organized during the
reign of which ruler? (Rajasthan 2019)
i ii iii iv (1) Kanishka (2) Rudradaman
(1) B C D A (3) Ashoka (4) Chandragupta Maurya
(2) A B D C 94. Where is the ‘Jantar-Mantar’ situated? (Rajasthan 2019)
(3) C A B D (1) Sikar (2) Ajmer
(4) D C B A (3) Jaipur (4) Bikaner
85. Choose the correct sequence to indicate the given 95. Sui Munda was the leader of (West Bengal 2019)
statement of the results of the Battle of Plassey as True (1) The Munda Rebellion (2) The Kol Rebellion
(T) or False (F): (Karnataka 2019) (3) The Chuarh Rebellion (4) The Santhal Rebellion
A. The Battle of Plassey brought out the immorality, lack 96. The editor of the ‘Bengal Gazette’ was (West Bengal 2019)
of unit among the Indians and the greed of Indian (1) Marshman
businessmen. (2) Surendranath Bandyopadhyay
B. After the Battle of Plassey, Siraj-ud-daulah became the (3) James Augustus Hickey
Nawab of Bengal. (4) William Carrey
C. The company gained exclusive rights to do business 97. The first woman graduate of Calcutta University was
in Bengal after the Battle of Plassey. (1) Kadambini Ganguly
D. Mir Jaffar had to pay huge amount as a war relief. (2) Sarala Devi Chaudhurani
(1) T T T T (2) T F T F (3) Swarna Kumari Devi
(3) T F F T (4) T F T T (4) Kalpana Dutta
EBD_7332
74
B- NTSE Stage 1 Question Bank

98. Mr. Allan Octavian Hume, who was the founder of the (B) Shifting agriculture in Sri Lanka was called ‘Chena’.
Indian National Congress was a (West Bengal 2019) (C) The people of forest communities benefited in many
(1) Journalist (2) Civil Servant ways after the forest department took control of the
(3) Politician (4) Police forests.
99. What was ‘Livre’? (Delhi 2018) Which of the above statements is/are correct?
(1) Currency of France (2) Newspaper of France (1) B only (2) A and C
(3) Magazine of France (4) Flag of France (3) A and B (4) A, B and C
100. The first Modern Novel published in Malayalam in the year 110. In 1868 England was producing about 80 percent of the
1889 was (Delhi 2018) food it consumed. The increase in food-grain production
(1) Indulekha (2) Rajasekhara Caritamu was made possible mainly by : (Andhra Pradesh 2018)
(3) Manju Ghose (4) Pariksha Guru (1) the use of bio-fertilizers only
101. The painting ‘Damayanti’ was made by (Delhi 2018) (2) bringing new lands under cultivation
(1) Abanindranath Tagore (2) William Jones (3) extensive use of chemical fertilizers
(3) Raja Ravi Verma (4) Rabindranath Tagore (4) radical innovations in agricultural technology
102. ‘Rinderpest’ is a term used for (Delhi 2018) 111. Famous Enabling Act was passed in Germany in 1933. With
(1) A cattle disease this Hitler : (Andhra Pradesh 2018)
(2) Missing of cattle (1) restored the dignity of Germany
(3) Indentured Labourer (2) established socialism in Germany
(4) Mass production in a factory (3) became the chancellor of Germany
103. Gudem Rebellion was led by (Delhi 2018) (4) became the dictator of Germany
(1) Baba Ramchandra (2) Jawaharlal Nehru 112. Who was the first Secretary of the Indian National
(3) Alluri Sitaram Raju (4) Mahatma Gandhi Congress established in 1885? (Goa 2018)
104. The principle of the ‘Garden City’ was developed by (1) Kashinath Trimbak Telang
(Delhi 2018) (2) Allan Octavian Hume
(1) Raymond Unwin (2) Barry Parker (3) Peter Alvares
(3) Ebenezer Howard (4) Herbert Baker (4) Dr. Ram Hegde
105. Which of the following features was NOT related to Stalin? 113. The revolutionaries in Bengal raided the Chittagong
(Andhra Pradesh 2018) Armouries under the leadership of ______. (Goa 2018)
(1) Collectivization of agriculture (1) Surya Sen (2) Khan Abdul Gaffar Khan
(2) Rapid industrialization (3) Abbas Tyabji (4) Sukhdev
(3) Announcement of “The New Deal” 114. Who was the “Father of Goa’s freedom movement”?
(4) Introducing five year plans (1) Purushottam Kakodkar (Goa 2018)
106. Which one of the following statements is correct? (2) Luis de Menezes Braganca
(Andhra Pradesh 2018) (3) P. P. Shirodkar
(1) William-I was proclaimed king of united Italy in 1861. (4) Dr. T.B. Cunha
(2) Victor Emmanuel-II was proclaimed German Emperor in 115. Why did feeding the cattle became a persistent problem for
1861. the Massais? (Goa 2018)
(3) Victor Emmanuel-II was proclaimed king of united Italy (1) Because continuous grazing in small area deteriorated
in 1871. the quality of pasture.
(4) William-I was proclaimed German Emperor in 1871. (2) Because Massais were confined to a small area.
107. Give the correct chronological order. (3) Because Massais lived in an arid zone without any
A. Simon Commission (Andhra Pradesh 2018) grass.
B. First round table conference (4) Because Massais began to cultivate pastoral land.
C. Gandhi- Irwin Pact 116. Dhangars were an important pastoral community in ______
D. Re-launch of Civil Disobedience Movement (Goa 2018)
(1) A, B, C, D (2) D, C, A, B (1) Uttar Pradesh (2) Maharashtra
(3) C, B, A, D (4) A, C, B, D (3) Himachal Pradesh (4) None of these
108. A sanyasi, who had earlier been to Fiji as an indentured 117. Who has written the book ‘Bij Ganit? (Gujarat 2018)
labourer, led a peasant movement. He used to recite verses (1) Shankaracharya (2) Bhaskaracharya
from Tulsidas Ramanyana to rural audience. ‘Who was he’ (3) Vatsayayan (4) Maharshi Patanjali
referred to here? (Andhra Pradesh 2018) 118. Name the two imperialist countries against which the
(1) Jhinguri Singh (2) Jadunandan Sharma nationalist Vietnamese fought? (Chandigarh 2018)
(3) Baba Ram Chandra (4) Sahajanada Saraswati (1) France and Britain (2) Britain and Japan
109. Consider the following statements regarding the forest (3) Japan and France (4) France and Germany
policies implemented under the British rule. 119. Who formed the Swaraj Party? (Chandigarh 2018)
(Andhra Pradesh 2018) (1) Jawaharlal Nehru and Mahatma Gandhi
(A) The first inspector General of Forests in India was a (2) Jawaharlal Nehru and Subhash Chandra Bose
French expert appointed by the British government. (3) Jawaharlal Nehru and Motilal Nehru
(4) Motilal Nehru and C.R. Das
Solved Questions B-75
120. Which two new colonial powers joined European powers in A. Dandi Satyagraha took place
the process of carving up of Africa between themselves at B. The Moplah uprising against Bristish
Berlin in 1885? (Chandigarh 2018) C. The Declaration of Poorna Swaraj
(1) Britain and France (2) Italy and France D. The Quit India Movement was launched
(3) Belgium and Germany (4) Britain and Italy (1) i-C, ii-A, iii-D, iv-B (2) i-B, ii-D, iii-A, iv-C
121. What does Proto-industrialisation mean? (3) i-D, ii-C, iii-B, iv-A (4) i-A, ii-B, iii-C, iv-D
(Chandigarh 2018) 126. The chronological order of the incidents of the first war of
(1) The first, and early form of industrialisation
Indian Independence is (Karnataka 2018)
(2) Industrialisation after 1800 C.E.
(3) Industrialisation after 1900 C.E. A. The Queen of Britain passed a Declaration assuring a
(4) Industrialisation after 2000 C.E. stable Government for Indians.
122. The location shown on the map of India was a French Colony B. A group of soldiers declared Bahadurshah Zafar as the
(Karnataka 2018) emperor of India.
C. The Sepoys of Meerut revolted against their British
Officers.
D. Tantia Tope was executed by British at Shivapuri.
(1) D C B A (2) B A C D
(3) A D B C (4) C B A D
127. The correct statement related to Satyashodak Samaj is
(West Bengal 2018)
A. The samaj started the Cow Protection Association.
B. The samaj opened the schools for shudras and girls.
C. The samaj advocated polytheism.
D. The samaj encouraged vedic education.
(1) Chandernagore (2) Goa (1) A (2) B (3) C (4) D
(3) Mahe (4) Pondicherry
128. ‘A Train to Pakistan’ was written by (West Bengal 2018)
123. The correct provision passed by the British Government in
India in their chronological order is (Karnataka 2018) (1) Khushwant Singh (2) Sadat Hasan Manto
A. Supreme Court of Judicature was established at (3) Md. Ali Jinnah (4) M.K. Gandhi
Calcutta. 129. Which the following statements are correct regarding
B. A new institution named ‘Board of Controllers’ liberal Nationalism in 19th century Europe? (Haryana 2018)
consisting of six Commissioners was started. (I) Right of Liberty and Equality
C. The post of Governor General was changed to (II) Formation of people’s govt.
Viceroy. (III) Ownership of private property
D. Representation of the Regional Council was allowed (IV) Complete control of govt. on all public and private
to Indians through election based on religion. properties
(1) D C A B (2) A B C D (1) I, II, III (2) IV, III, I
(3) B A D C (4) C D B A (3) III, IV, II (4) I, II, IV
124. The correct chronological order of the treaties signed 130 Arrange the following historical developments in a
between British and Native States is (Karnataka 2018) chronological sequence. (Haryana 2018)
A. The treaty of Salbai, the treaty of Srirangapatna, the (I) Unification of Germany
treaty of Amritsar and the treaty of Mangalore.
(II) Unification of Italy
B. The treaty of Salbai, the treaty of Mangalore, the treaty
of Srirangapatna and the treaty of Amritsar. (III) The French Revolution
C. The treaty of Amritsar, the treaty of Mangalore, the (IV) Treaty of Vienna
treaty of Salbai and the treaty of Srirangapatna. (1) I, II, III, IV (2) III, IV, II, I
D. The treaty of Amritsar, the treaty of Salbai, the treaty of (3) II, IV, III, I (4) IV, I, III, II
Srirangapatna and the treaty of Mangalore. 131. Arrange the following historical developments in a
(1) A (2) B chronological sequence. (Haryana 2018)
(3) C (4) D (I) Poona Act
125. Select the correct order of events related to freedom struggle (II) Lahore congress : demand of ‘Purna Swaraj’
starting from North to South as shown in the map. (III) Establishment of Oppressed Class Association by
(Karnataka 2018) B. R. Ambedkar
(IV) Second round table conference
N (1) I, II, III, IV (2) III, IV, II, I
i (3) II, III, IV, I (4) IV, II, III, I
132. What does the novel ‘Sevasadan’ by Munshi Premchand
A mainly relate to? (Haryana 2018)
ii (1) Atrocities under colonial rule
iii (2) Social problems like child marriage and dowry
(3) Life of an orphan
iv (4) Miseries of a poor peasant
EBD_7332
76
B- NTSE Stage 1 Question Bank

133. Which of the following statements are correct on Gandhiji’s 141. Who discovered the spinning jenny ? (Rajasthan 2017)
breaking the salt law to start civil disobedience movement? (1) John Ke (2) T.E. Nicholson
(Haryana 2018) (3) Raphael Samuel (4) James Hargreaves.
(i) Salt was the need of both rich and poor. 142. The year of the Partition of Bengal was(Rajasthan 2017)
(ii) British govt. levied tax on the salt. (1) 1903 (2) 1905
(iii) Only govt. agencies were allowed to make salt. (3) 1907 (4) 1909
(iv) Lord lrwin abolished tax on salt. 143. Which one of the following countries was not among the
(1) I, II, III (2) II, III, IV Allied Powers ? (Rajasthan 2017)
(3) III, IV, I (4) I, III, IV (1) England (2) France
134. Statement I : Railways were essential for colonial trade and (3) Russia (4) Germany
for the movement of imperial troops. (Haryana 2018) 144. When was the publication of Bengal Gazette initiated ?
Statement II : The forests around the railway tracks started (Rajasthan 2017)
disappearing fast. (1) 1750 (2) 1780
Read the statements and select the correct answer from the (3) 1850 (4) 1880
options given below. 145. Consider the following Points : (Rajasthan 2017)
(1) Statement I is true. (A) Mahatma Gandhi started salt March with his 78
Statement II is false. confidential volunteers.
(2) Statement I is false. (B) Mahatma Gandhi violated the salt law at Dandi on
Statement II is true. April 20th, 1930.
(3) Both statements are true and statement II provides ex- Choose the correct answer from the codes given below.
planation to statement I. (1) only (A)
(4) Both statements are true but statement II does not pro- (2) only (B)
vide explanation of statement I. (3) both (A) and (B)
135. Kheda Satyagrah was related to (Delhi 2017) (4) None of these
(1) Against the oppressive plantation system 146. After which war the British rule was founded in India?
(2) Movement of cotton mill workers (Rajasthan 2017)
(3) Relaxation in revenue collection (1) Battle of Sabrao
(4) None of the above (2) Battle of Panipat
136. What do you mean by "Hind Swaraj"? (Delhi 2017) (3) Battle of Plassey
(1) Political Party of Tilak (4) Second Anglo Mysore war
(2) Book of Mahatma Gandhi 147. When was the Great Economic Depression between the
(3) Symbol of Indian National congress two World Wars held ? (Rajasthan 2017)
(4) Political Party of Mahatma Gandhi (1) 1921 (2) 1929
137.The first Historical novel written in Bengal was (3) 1935 (4) 1939
(1) Chemmin (2) Anguriya Binimoy 148. Who composed Ananda Math ? (Rajasthan 2017)
(3) Chomna Didi (4) Anandmath (1) Rabindranath Tagore
138. Match List I with List II correctly and choose the correct (2) Munsi Premchand
code from the following : (Rajasthan 2017) (3) Mahatma Gandhi
List-I (4) Bankim Chandra Chattopadhyay.
(A) Meeting of the Estates General 149."Imperialism: The Highest stage of Capitalism" was written
(B) Bastille was destroyed on by (West-Bengal 2017)
(C) Abolishment of feudal system in France (1) Lenin (2) Stalin
(D) Swore of Tennis Court (3) Karl Marx (4) Rousseau
List II 150.24th October, 1929 was marked as “Black Thursday” in U.S.A.
(i) 20th June, 1789 (ii) 4th August, 1789 because (West-Bengal 2017)
(iii) 14th July, 1789 (iv) 5th May, 1789. (1) Terrorist Attack (2) Natural Calamity
A B C D (3) Great Depression (4) Change in Political aspect
(1) i ii iii iv 151.During the period of Russian Revolution the Russian ruler
(2) iv iii ii i was (West-Bengal 2017)
(3) iv i ii iii (1) Czar Alexander–I (2) Czar Alexander–II
(4) i iv iii ii (3) Czar Nicholas–I (4) Czar Nicholas–II
139. The state of India where the Jallianwalla Bagh is situated, 152.“Flying Shuttle” was invented by (West-Bengal 2017)
is (Rajasthan 2017) (1) James Hargraves (2) Edmund Cartwright
(1) Haryana (2) Uttar Pradesh (3) James Watt (4) John Kay
(3) Punjab (4) Rajasthan. 153.“Mein Kampf” was written by (West-Bengal 2017)
140. The German King in 1871 was (Rajasthan 2017) (1) Hitler (2) Mussolini
(1) William I (2) Nepoleon III (3) Lenin (4) Stalin
(3) Frederik William IV (4) Emmanuel II.
Solved Questions B-77
154.The country which did not join the League of Nations: a. Madras b. Bengal
(West-Bengal 2017) c. Sindh d. Assam
(1) America (2) France (1) a and b (2) a, b and c
(3) Italy (4) Japan (3) b, c and d (4) a, c and d
155.Present name of General Assembly’s Institution is 166. Identify the correct statements related to French
(West-Bengal 2017) Revolution (Karnataka 2017)
(1) Hindu School (2) Scottish Church College a. Priests and rich class people are exempted from
(3) Loreto House (4) St. Xavier’s College paying taxes and led a luxurious life.
156.The first Chancellor of Calcutta University was b. Agricultural and industrial developments were
(West-Bengal 2017) stumbled.
(1) Lord Canning (2) Lord Dalhousie c. Count Cavour made a secret pact with France.
(3) James William Colvile (4) Sir Ashutosh Mukherjee d. Louis XVI of Bourbon dynasty led a luxurious and
157.Sardar Vallabhbhai Patel was known as extravagant life.
(West-Bengal 2017) (1) a, b and c (2) a, c and d
(1) Saviour of India (2) Modern Man of India (3) b, c and d (4) a, b and d
(3) Iron Man of India (4) Machiavelli of India 167. Choose the group of nations which belong to Triple
158.‘Communal Awards’ (1932) in India was declared by Alliance during First World War (Karnataka 2017)
(West-Bengal 2017) a. Germany b. Russia
(1) Lord Irwin (2) Ramsay Macdonald c. Austria d. Italy
(3) Md. Ali Zinnah (4) Lord Mountbatten (1) a and b (2) a, b and c
159.Pahartali European Club was attacked in 1932 by (3) a, c and d (4) a, b, c and d
(West-Bengal 2017) 168. Metternich remarked "When France Sneezes, the rest of
(1) Kalpona Dutta (2) Bina Das Europe catches cold". Identify the most appropriate
(3) Pritilata Waddedar (4) Lila Nag justification for this statement (Karnataka 2017)
160.“All India Trade Union Congress” (AITUC) was formed in (1) French revolution sparked nationalism
(West-Bengal 2017) (2) French revolution inspired world nations
(1) 1915 AD (2) 1920 AD (3) The development in France ignited uprising in Belgium
(3) 1922 AD (4) 1928 AD and breaking away from UK
161. Bhagath Singh, Jathindas, Batukeshwar and others founded (4) France became the leader of Europe
a Revolutionary Association called (Karnataka 2017) 169. The Kannada speaking region which came under the rule
(1) Hindusthan Army for Independence of Madras province in 19th century is (Karnataka 2017)
(2) Hindusthan Socialist Republic Army (1) Raichur (2) Bellari
(3) Hindusthan Socialist Revolutionary Army (3) Bijapura (4) Gulbarga
(4) Hindusthan Kissan Movement Sabha 170. Match personals in List-A with appropriate positions in
162. Choose the group of correct statements related to II Carnatic List-B (Karnataka 2017)
War (Karnataka 2017) List-A List-B
a. Battle between Nasir Jung and Muzaffar Jung for the a. Shaikh Abdulla I. State Reorganisation Com
throne of Hydrabad Nizam. mission
b. Britishers helped Anwaruddin. b. Fazal Ali II. Chief of Razakars
c. French helped Chandra Saheb under the leadership of c. Sardar Vallabhbhai III. King of Jammu and
Dupleix. Patel Kashmir
d. Britishers defeated Nasir Jung in a battle d. Qasim Razvi IV. First Home Minister of
(1) a, b and c (2) a and d India
(3) a, b and d (4) a, c and d e. Raja Harisingh V. Founder of National
163. The idea of Indian National Army (INA) was conceived by Conference
(1) Subhaschandra Bose (Karnataka 2017) a b c d e
(2) Ross Behari Bose (1) I V IV III II
(3) Mohan Singh (2) V I IV III II
(4) Aravind Ghosh (3) V I II IV III
164. Choose the correct of the incidents of Indian freedom (4) V I IV II III
movement in a chronological order: (Karnataka 2017) 171. The correct statement that is related to French East India
a. Poona Agreement Company is that it was, (Karnataka 2017)
b. Direct Action Day (1) Private Company
c. August Offer (2) Private Company controlled by the French Government
d. Establishment of Forward Block Party (3) Officially a State owned comapny
(1) a, d, c, b (2) b, a, d, c (4) Controlled by the French Merchant
(3) a, d, b, c (4) d, a, b, c
165. Choose the correct group of areas in which Britishers
introduced the Ryotwari system. (Karnataka 2017)
EBD_7332
78
B- NTSE Stage 1 Question Bank

172. “For this earth is not allotted to anyone nor is it presented 182. The Ryotwari settlement was introduced by the British in
to anyone as a gift. It is awarded by providence to people the ... (Andhra Pradesh 2017)
who in their hearts have the courage to conquer it, the (1) Madras Presidency. (2) Bengal Presidency.
strength to preserve it and the industry to put it to the (3) Central Presidency. (4) Assam Presidency.
plough.” Whose ideology is this? 183. The famous Quit India Resolution was passed on .....
(Andhra Pradesh 2017) (Andhra Pradesh 2017)
(1) Benito Mussolini (2) Adolf Hitler (1) August 18, 1942 (2) April 4, 1942
(3) Ho Chi Minh (4) Stalin (3) April 14, 1942 (4) August 8, 1942
173. According to the census of 1921, 12 to 13 million people 184.In 1920 Millitant Guerilla Movement in Guden Hills of Andhra
perished as a result of (Andhra Pradesh 2017) Pradesh (India) was led by: (Haryana 2016)
(1) First World War (2) Epidemics (1) Alluri Sitaram Raju (2) Baba Ramchandra
(3) Famines (4) All the above. (3) Birsa Munda (4) Diku and Kanu
174. Find out the wrong statement about Giuseppe Mazzini ? 185.Who founded the Communist Party in Vietnam?
(Andhra Pradesh 2017) (Haryana 2016)
(1) He was a member of the secret society of the Carbonari. (1) Huyunh Phu So (2) Phan Boi
(2) He believed “The God had intended nations to be the (3) Ha Tinh (4) Ho chi Minh
natural units of mankind.” 186.When did Muslim League celebrate the ‘Deliverance Day’?
(3) He was the founder of young Europe. (Haryana 2016)
(4) None of the above. (1) 23 March, 1940 (2) 22 December, 1939
175. Who wrote the book “The History of the loss of Vietnam”? (3) 24 October, 1939 (4) 16 March, 1940
(Andhra Pradesh 2017) 187. In which state of India are Gujjar Bakarwals largely found?
(1) Phan Boi Chau (2) Ho Chi Minh (Haryana 2016)
(3) Huynh Phu So (4) Phan Chu Trinh (1) Rajasthan (2) Gujarat
176. Compulsory Elementary Education Act was made in England (3) Jammu and Kashmir (4) Maharashtra
in the year.... (Andhra Pradesh 2017) 188. The Duma were council assemblies which were created by:
(1) 1829 (2) 1849 (Haryana 2016)
(3) 1860 (4) 1870 (1) Lenin (2) Karl Marx
177. Who developed the concept of “The principle of the Garden (3) Trotsky (4) Tsar of Russia
Citv”? (Andhra Pradesh 2017) 189. Arthsastra is written by whom? (Haryana 2016)
(1) Andrew Means (2) Henry Mayhew (1) Kautilya (2) Machiavelli
(3) Ebenezer Howard (4) Haussman (3) Chandragupta Maurya (4) Samudragupta
178. Who wrote “Ninety five Theses” criticising many of the 190. Ibrahim Lodhi was defeated (Uttar Pradesh 2016)
practices and rituals of the Roman Catholic Church ? (1) In the first battle of Panipat
(Andhra Pradesh 2017) (2) In the second battle of Panipat
(1) Martin Luther (2) Thomas Pain (3) In the first battle of Talikota
(3) J.V. Schley (4) Richard M. Hoe (4) In the first battle of Tarain
179. Kashi baba, a kanpur mill worker wrote and published 191. Who led the revolt of 1857 in Bihar (Uttar Pradesh 2016)
“Chhote Aur Bade Ka Sawal” in 1938 to show the links (1) Khan Bhadur Khan (2) Tatiya Tope
between .... (Andhra Pradesh 2017) (3) Kunwar Singh (4) Mangal Pandey
(1) Caste and Class exploitation. 192. Who is famous as Deshbandhu (Uttar Pradesh 2016)
(2) Caste and Religion relation. (1) Chandrashekhar (2) A. O. Hume
(3) Income and Untouchability. (3) Chittranjan Das (4) Veer Savarkar
(4) Industrialists and Politicians. 193.Who was the Governor General of India in 1857?
180. “Only a decade ago, they were as illiterate, helpess and (Jharkhand 2015)
hungry as our own masses, who could be more astonished (1) Wellesley (2) Dalhousie
than an unfortunate Indian like myself to see how they had (3) Canning (4) Minto
removed the mountains of ignorance and help lessness in 194.Who was the First Indian Woman President in the Indian
these few years.” Name the Indian, who quoted this Russian National congress ? (Jharkhand 2015)
revolution ? (Andhra Pradesh 2017) (1) Mrs. Annie Besant (2) Sucheta Kripalani
(1) M N Roy. (2) Rabindranath Tagore. (3) Sarojini Naidu (4) Indira Gandhi
(3) Mahatma Gandhi. (4) Jawaharlal Nehru. 195.The First English factory in India was established at
181. Find out the wrong statement related to Franklin Roosevelt. (Jharkhand 2015)
(Andhra Pradesh 2017) (1) Bombay (2) Hooghly
(1) Announced New Deal Policy to eradicate economic (3) Surat (4) Calcutta
depression. 196.The Asiatic Society of Bengal was founded by
(2) Introduced the much needed Social Security system. (1) Raja Ram Mohan Roy (Jharkhand 2015)
(3) President of America during Second World War. (2) Sir William Jones
(4) None of the above. (3) W.W. Hunter
(4) William Bentinck
Solved Questions B-79
197.Who is the first woman Prime Minister of India ? 213.Who was Viceroy of India during Civil Disobedience
(Jharkhand 2015) Movement? (Maharashtra 2014)
(1) Meira Kumar (2) Sonia Gandhi (1) Lord Curzon (2) Lord Minto
(3) Indira Gandhi (4) Annie Besant (3) Lord Canning (4) Lord Irwin
198.The Ajanta caves were built during the period of the 214.Suffrage Movement was started for: (Maharashtra 2014)
(West Bengal 2015) (1) The Right to Vote for Women
(1) Mauryas (2) Guptas (2) Right to Education
(3) Kushanas (4) Chalukyas (3) Food for Poors
199.Hieun-Tsang came to India during the reign of (4) Peasants Right
(West Bengal 2015) 215.Jacobin Club was the most powerful political organisation in
(1) Harshavardhana (2) Kanishka (Maharashtra 2014)
(3) Chandragupta Maurya (4) Samudragupta (1) Germany (2) Russia
200.Who built the Red Fort? (West Bengal 2015) (3) France (4) Britain
(1) Sher Shah (2) Aurangzeb 216.What was “Dawes Plan”? (Maharashtra 2014)
(3) Akbar (4) Shahjahan (1) Nazi Plan to supress the Jews
201. The first Governor General of India was (2) American plan to control the Bolsevik
(1) Lord Clive (West Bengal 2015) (3) Stalin plan of collectivisation
(2) Lord Wellesley (4) American plan to take out Germany from the financial
(3) Lord William Bentinck crisis (great depression)
(4) Warren Hastings 217.To establish French supremacy in India, the Governor General
202. Who was called the ‘Frontier Gandhi’? appointed in 1742 AD was (Karnataka 2014)
(West Bengal 2015) (1) Dalhousie (2) Hastings
(1) Mahatma Gandhi (2) Pattavi Sitaramaiya (3) Canning (4) Dupleix
(3) Vallabhbhai Patel (4) Khan Abdul Gaffar Khan 218.Nawab Anwaruddin of Carnatic region was used as a pawn
203.The Autobiography of Hitler is (Delhi 2015) by (Karnataka 2014)
(1) Mein Kampf (2) The Secret Book (1) French and Spanish (2) French and British
(3) French and Portuguese (4) British and Dutch
(3) Nazi Leaflet (4) Gypsy
219.The organization formed by Balgangadhar Tilak in 1916
204.Gandhiji’s “Champaran Satyagrah” was associated with
(Karnataka 2014)
(Delhi 2015)
(1) Home Rule League (2) Arya Samaja
(1) Rubber Farming (2) Cotton Farming (3) Swaraj Party (4) Prarthana Samaja
(3) Tea Farming (4) Indigo Farming 220.Jacobin Club was the most powerful political organisation
205.First Newspaper published in India was- in: (Delhi 2014)
(Chhattisgarh 2015) (1) Germany (2) Russia
(1) Om Prakash (2) Sambaddha kaumudi (3) France (4) Britain
(3) Rast-Goftar (4) Bengal Gazzet 221.What was “Dawes Plan”? (Delhi 2014)
206.The Founder of Judaism was- (Chhattisgarh 2015) (1) Nazi Plan to supress the Jews
(1) Abraham (2) Moses (2) American plan to control the Bolsevik
(3) Zolwastra (4) Augustus (3) Stalin plan of collectivisaion
207. Kheda Movement (1917) was started for (4) American plan to take out Germany from the financial
(Maharashtra 2014) crisis (great depression)
(1) Farmers (2) Labourers 222.The Tripartite Pact (1940) was signed by:
(3) Mill Owners (4) Soldiers (Chhattisgarh 2014)
208.Bharat Mata, Painting was drawn by (Maharashtra 2014) (1) Britain, France and Germany
(1) Rabindranath Tagore (2) Nand Lal Basu (2) Germany, Italy and Japan
(3) Amrita Shergil (4) Abanindra Nath Tagore (3) Japan, Britain and Russia
209.Gutenberg was associated with (Maharashtra 2014) (4) Russia, Britain and USA
(1) Powerloom (2) Rail Engine 223.Philanthropsis mainly work for: (Chhattisgarh 2014)
(3) Computer (4) Printing Press (1) industrial workers
210.‘Depressed Class Association’ was formed by (2) peasants
(Maharashtra 2014) (3) social and religios reforms
(1) Jyoti Ba Phule (2) Mahatma Gandhi (4) social unliftment and charity
(3) B. R. Ambedkar (4) Shahuji Maharaj 224.Which was the capital of Mahajanpad Vatsa in 6th century
211. Who were the ‘Kulak’ in soviet society? B.C. ? (Madhya Pradesh 2013)
(Maharashtra 2014) (1) Champa (2) Kaushambi
(1) Small Farmers (2) Big Farmers (3) Varanasi (4) Mathura
(3) Shop Keepers (4) Lawyers 225.How was Jamil related to Swami Mahavir ?
212.Reichstag was: (Maharashtra 2014) (Madhya Pradesh 2013)
(1) French Parliament (2) Russian Parliament (1) Son (2) Friend
(3) German Parliament (4) Japan Parliament (3) Son-in-law (4) Father
EBD_7332
80
B- NTSE Stage 1 Question Bank

226.Who is regarded as Light of Asia ? 241.Which battle established the British supremacy in India?
(Madhya Pradesh 2013) (Punjab 2012)
(1) Gautam Buddha (2) Gandhiji (1) The battle of Panipat (2) The battle of Plassey
(2) Swami Mahavir (4) Mao-Tse Tung (3) The battle of Buxar (4) The battle of Mysore
227.God Rudra mentioned in Rigveda is : 242.By selling which of the items to china, did the British regulary
(Madhya Pradesh 2013) collect money for purchasing tea from China?
(1) Brahma (2) Vishnu (Punjab 2012)
(3) Mahesh (4) Yamraj (1) Opium (2) Jute
228.When was Hajrat Mohammad born ? (3) Cotton (4) Sugarcane
(Madhya Pradesh 2013) 243.Who said, "I am the state, and my words are law" ?
(1) 550 A.D. (2) 560 A.D. (Madhya Pradesh 2012)
(3) 570 A.D. (4) 580 A.D. (1) Louis XIV (2) Louis XV
229.Who was the author of “Geet Govind” ? (Rajasthan 2013) (3) Louis XVI (4) Rousseau
(1) Sarangdhar (2) Jaidev 244.Who is regarded as father of Italian unification ?
(3) Madhodas (4) Bihari (Madhya Pradesh 2012)
230.Who was mainly responsible for “Bang-Bhang” ? (1) Mazzini (2) Cavour
(Rajasthan 2013) (3) Garibaldi (4) None of these
(1) Lord Lytton (2) Lord Rippon 245.Who led Russian revolution of 1917 ?
(3) Lord Macaulay (4) Lord Curzon (Madhya Pradesh 2012)
231.In which country first time the cultivation of opium began? (1) Stalin (2) Brezhnev
(Rajasthan 2013) (3) Lenin (4) Karl Marx
(1) Portugal (2) India 246.Who built Jantar-Mantar ? (Rajasthan 2012)
(3) China (4) Britain (1) Sawai Jaisingh (2) Mirza Raja Jaisingh
232.Which treaty was imposed on Germany after first world (3) Raja Mansingh (4) Pratap Singh
war ? (Rajasthan 2013) 247.Who among the following was the Congress President when
(1) Versailles (2) Paris Congress declared complete independence as its goal?
(3) Vienna (4) London (1) Jawaharlal Nehru (Odisha 2012)
233.Who was confucius? (West Bengal 2013) (2) Subhas Chadra Bose
(1) A Chinese Philosopher (3) Sardar Vallabh Bhai Patel
(2) King of Tibet (4) Maulana Abul Kalam Azad
(3) Religious leader of Japan 248.Who was the last Tsar of Russia? (Odisha 2012)
(4) Disciple of Dalai Lama (1) Nicholas I (2) Nicholas II
234.What was Barbarossa Operation? (West Bengal 2013) (3) Alexander I (4) Alexander II
(1) It was a plan to stop World war-II 249.Who among the following leaders was not a pioneer of
(2) Name of Germany’s invasioin of Russia during World Non-alignment movement. (Odisha 2012)
War-II (1) J. Nehru (2) Nasser
(3) Name of treaty between Germany and Russia (3) Woodrow Wilson (4) Tito
(4) A secret meeting of central forces 250.Which of the following features stood for Nazism?
235.‘When France Sneezes, the rest of Europe catches cold?. (1) Anti-semitic Policy (Odisha 2012)
This was the remark of ? (Delhi 2012) (2) Pro-socialistic policy
(1) Bismarck (2) Metternich (3) Pro-democratic Policy
(3) Mazzini (4) Napoleon (4) Pro-communist Policy
236.In 1860 the famous expedition of the thousand to South Italy 251.The name of first newspaper published in India –
was led by: (Delhi 2012) (West Bengal 2012)
(1) Mazzini (2) Garibaldi (1) Digdarshan (2) Samachar Darpan
(3) Victor Emmanuel II (4) Count Cavour (3) Bengal Gazette (4) Sambad Koumudi
237.The Dutch started ‘Scientific forestry’ at: (Delhi 2012) 252.The first Vice-Chancellor of Calcutta University was
(1) Sumatra (2) Kalimantan (West Bengal 2012)
(3) West Irian (4) Java (1) Michael Sadlar
238.In 1940 Vietnam was occupied by (Delhi 2012) (2) Sir Ashutosh Mukhopadhyay
(1) China (2) France (3) Sir James William Colville
(3) Japan (4) Germany (4) Sir Charles Wood
239.In Which “Congress Session” the resolution on Poorna 253.‘Reichstag’ is a Parliamentary House of a State mentioned
Swaraj was passed? (Punjab 2012) below (West Bengal 2012)
(1) Calcutta Session (2) Karachi Session (1) Japan (2) Russia
(3) Lahore Session (4) Tripura Session (3) Switzerland (4) Germany
240.The “Great Depression was a period of (Punjab 2012) 254.In which session of the Muslim League was the resolution
(1) Political crisis (2) Global crisis for a Separate Muslim State adopted?
(3) Social crisis (4) Economic crisis (West Bengal 2012)
Solved Questions B-81
(1) Delhi Session (2) Karachi Session 260.Which event led to the end of feudalism?
(3) Lahore Session (4) Lucknow Session (Maharashtra 2012)
255.He was known as the father of history writing (1) Constantinople’s fall
(Andhra Pradesh 2012) (2) First Religious War
(1) Thucidydes (2) Plutarch (3) French Revolution
(3) Herodotus (4) Democritis (4) American War of Independence
256.“Boston tea party” means (Andhra Pradesh 2012) 261.Which statement is not applicable for capitalism?
(1) A party unloading a ship containing tea at Boston (Maharashtra 2012)
(2) The event where the ship containing tea was un- (1) Economic system based on mass production
loaded on the orders of Governor of Boston duly (2) Means of production are Government owned
(3) Economic system based on mass distribution
throwing the crates of tea in sea
(4) The capitalist aims at maximizing profits.
(3) A tea party arranged by a group of people dressed as
262.‘Father Gapon’ is related to which event?
Red Indians at Boston (Maharashtra 2012)
(4) A tea party arranged by the Governor of Boston in (1) Leader of Petrograd Workers
honour of Red Indians. (2) Confiscation of landed estates
257.This was not one of the factors in the rise of Imperialism (3) Experimentation of collective farming
(1) Political unrest (Andhra Pradesh 2012) (4) The Bloody Sunday
(2) Search for Raw material 263.The dictatorship of the proletariat as the only solution for all
(3) Search for markets feudal problems was advocated by: (Karnataka 2011)
(4) National pride (1) Czar Nicholas II (2) Karl Marx
258.Europeans were attracted to Indonesia for this reason (3) Vladimir Lenin (4) Joseph Stalin
(Andhra Pradesh 2012) 264.Which one of the following groups is in the chronological
(1) It is a small country order? (Karnataka 2011)
(2) It has rich spice products (1) Home Rule League, Swaraj Party, Simon Commission,
(3) Majority of the population are muslims Poona Pact, Purna Swaraj, Cripp’s Mission.
(4) It has a large number of ethnic groups (2) Home Rule League, Purna Swaraj, Simon Commission,
259.Through whose writings did the people find an expression to Poona Pact, Cripp’s Mission, Swaraj Party
their feelings in France? (Maharashtra 2012) (3) Home Rule League, Swaraj Party, Simon Commission,
(1) Montesquieu (2) Voltaire Purna Swaraj, Poona Pact, Cripp’s Mission
(3) Necker (4) Calon (4) Home Rule League, Swaraj Party, Cripp’s Mission,
Poona Pact, Simon Commission, Purna Swaraj

SECTION 6. CIVICS
Civics 5. Main recommendation of the Mandal Commission was:
(Delhi 2020-21)
1. Who was the chairman of the drafting committee of the (1) Reservation for Socially and Educationally Backward
Indian Constituent Assembly? (Delhi 2020-21) Classes.
(1) Dr. Rajendra Prasad (2) Dr. B.R. Verma (2) Reservation for schedule caste.
(3) Dr. B.R. Menon (4) Dr. B.R. Ambedkar (3) Reservation for schedule Tribes.
2. The movement that seeks equality in the personal and family (4) Reservation for Minorities.
life of women is known as – (Delhi 2020-21) 6. What is Mid-Term Election? (Delhi 2020-21)
(1) Narivadi Andolan (1) Election for the seat to be vacated due to some reason.
(2) Nari Sashaktikaran Andolan (2) Election in the event of death of one member.
(3) Mahila Shakti Andolan (3) Election to be held in whole country or state before
(4) Mahila Adhikar Andolan the scheduled time.
3. In case of conflict between the central government and the (4) Completion of 5 years election.
State Government over the subject in the Concurrent list: 7. Who becomes the members of Gram Sabha?
(Delhi 2020-21) (Delhi 2020-21)
(1) Supreme Court decides which of the two should be (1) Only elderly people
considered. (2) Only elected members of Gram Panchayat
(2) The state government should be obeyed. (3) All the voters of the village
(3) The central government should be obeyed. (4) Only the youth of the village
(4) Both governments should be obeyed. 8. A person who is not a member of Parliament is appointed
4. In India, seats are reserved for women in which of the on the minister, he/she has to get elected to one of the
following bodies:- (Delhi 2020-21) houses of Parliament within: (Delhi 2020-21)
(1) In Lok Sabha (2) In State Legislatures (1) A month
(3) In Rajya Sabha (4) In Panchayati Raj (2) Three months
(3) Six months
(4) Selected time is fixed by the President
EBD_7332
82
B- NTSE Stage 1 Question Bank

9. Respect for the National Flag and the National Anthem is: 19. The minimum age for the citizen to exercise their right to
(Andhra Pradesh-2021) vote has been reduced to 18 years from 21 years through
(1) A fundamental right of every citizen the (West Bengal-2021)
(2) A fundamental duty of every citizen (1) 42nd Amendment Act (2) 44nd Amendment Act
(3) A directive principle of state policy (3) 61nd Amendment Act (4) 74nd Amendment Act
(4) An ordinary duty of every citizen 20. The Panch-Sheel Agreement was signed between
10. Which one among the following was not an aim of the 42nd (West Bengal-2021)
Constitutional amendment? (Andhra Pradesh-2021) (1) India and China (2) India and Nepal
(1) Excluding the courts from election disputes (3) India and Pakistan (4) Pakistan and China
(2) Strengthening the central government 21. When was the unification of Rajasthan completed?
(3) Abolition of bonded labour (Rajasthan-2021)
(4) Making the judiciary subservient to parliament (1) 30th March, 1949 (2) 15th May, 1948
11. The Supreme Court judgement popularly known as the (3) 26th January, 1950 (4) 1st November, 1956
‘Keshavananda Bharathi case’ says that: (Andhra Pradesh-2021) 22. From which district of Rajasthan, the modern three-tier of
(1) Certain basic provisions in the Constitution cannot Panchayati Raj system was lunched on October 2nd, 1959?
be changed under any circumstances (Rajasthan-2021)
(2) The word ‘secular’ is to be added to the preamble of (1) Ajmer (2) Nagour
the constitution. (3) Jaipur (4) Bikaner
(3) ‘Right to Property’ should be removed from fundamental 23. When was the constitution of India adopted, enacted and
rights. spirited? (Rajasthan-2021)
(4) Laws should not be amended (1) 26th November, 1949 (2) 26th October, 1948
12. Constitution Day is celebrated in India every year on which (3) 26th January, 1950 (4) 26th January, 1949
day? Why? (Andhra Pradesh-2021) 24. Where did the 18th summit of ‘SAARC’ take place in
(1) 26th November – Constituent Assembly met for the November, 2014? (Rajasthan-2021)
first time on that day. (1) Nepal (2) New Delhi
(2) 26th January – Constitution came into force on that day. (3) Bhutan (4) Maldiv
25. What can be maximum number of members of Rajya Sabha
(3) 26th November – Constitution was finally adopted on
as per constitution provision? (Rajasthan-2021)
that day.
(1) R (2) A
(4) 26thJanuary – Constitution was amended for the first
(3) M (4) Q
time on that day. 26. Till what age can the Judges of high court hold his post?
13. The sequence in which the given terms are mentioned in (Rajasthan-2021)
the constitutional preamble is : (Andhra Pradesh-2021) (1) R (2) A
(1) Sovereign, Socialist, Secular, Democratic, Republic (3) M (4) Q
(2) Sovereign, Socialist, Secular, Republic, Democratic 27. Which one of the following terms is not included in the
(3) Sovereign, Socialist, Democratic, Secular, Republic preamble to the Indian Constitution?
(4) Sovereign, Socialist, Republic, Secular, Democratic (1) Republic (2) Justice (Bihar-2021)
14. Minimum educational qualification required for a candidate to (3) Monarchy (4) Equality
contest in Parliamentary elections are: (Andhra Pradesh-2021) 28. Which of the following institutions can amend the
(1) SSC or equivalent constitution of India? (Bihar-2021)
(2) Intermediate or equivalent (1) The Parliament (2) The Cabinet
(3) Graduation (3) The Prime Minister (4) The President
(4) No educational qualification is required 29. Which of the following are the features of the Federal
15. “MONEY BILL is first introduced in the Government? (Bihar-2021)
(West Bengal-2021) I. Unwritten Constitution II. Division of Powers
(1) Lok Sabha (2) Rajya Sabha III. Single Citizenship IV. Independent Judiciary
(3) Supreme Court (4) High Court (1) I and II (2) II and III
16. The number of judges of the International Court of Justice (3) I and IV (4) II and IV
is (West Bengal-2021) 30. The first speaker of Lok Sabha was- (Uttar Pradesh-2021)
(1) 9 (2) 10 (1) Ganesh Vasudev Mavalankar
(3) 15 (4) 16 (2) Pt. Govind Vallabh Pant
17. The age of retirement of the judges of the High Court is (3) Ananthasayanam Ayyangar
(West Bengal-2021) (4) C. Subramanian
(1) 65 years (2) 60 years 31. Right to Education Act came into effect on-
(3) 62 years (4) 70 years (Uttar Pradesh-2021)
18. The headquarter of the World Health Organization is (1) 2005 (2) 2010
(West Bengal-2021) (3) 2008 (4) 2012
(1) London (2) Manchester 32. ‘Forward Bloc’ is a regional party of- (Uttar Pradesh-2021)
(3) Geneva (4) Paris (1) Odisha (2) Jharkhand
(3) West Bengal (4) Chhatisgarh
Solved Questions B-83
33. ‘National Democratic Alliance’ was founded in- C. It takes up crimes like murder, robbery, dacoity etc.
(Uttar Pradesh-2021) D. It is also known as the District Magistrate court.
(1) May 1998 (2) June 1996 (1) Civil Court (2) Commissioner’s Court
(3) May 1999 (4) June 1997 (3) Tahashildar Court (4) Criminal Court
34. A communication issued by an appropriate authority stating 45. The Supreme Court declared as a fundamental right to
the policy of the Government is known as provide free and compulsory education to all children
(Tamil Nadu-2021) between the age of 6 to 14 years under article 21A in :
(1) Office Memorandum (2) Legislative Procedure (Karnataka-2021)
(3) Government Order (4) Circular (1) Golaknath v/s Punjab Government
35. Which party always won elections in Mexico from its (2) Keshawananda Bharathi v/s Kerala Government
independence in 1930 until 2000? (Tamil Nadu-2021) (3) Unnikrishnan v/s Andrapradesh Government
(1) Revolutionary Party (4) Minerva Mills v/s Union of India
(2) Mexican Revolutionary Party 46. In which year “The Right to Information Act” was passed
(3) Institutional Revolutionary Party (Uttar Pradesh 2019)
(4) Institutional Party (1) 2001 (2) 2003
36. The Government of India Act came into function in the year (3) 2005 (4) 2007
(Tamil Nadu-2021) 47. The Constitution of India primarily did not include in its
(1) 1930 (2) 1932 preamble (Uttar Pradesh 2019)
(3) 1935 (4) 1938 (1) Sovereign (2) Socialist
37. The State in which assembly election held in 2020-21 (3) Democratic (4) Republic
(Tamil Nadu-2021) 48. Article-370 was associated with (Uttar Pradesh 2019)
(1) Odisha (2) Maharashtra (1) Uttar Pradesh (2) Nagaland
(3) Karnataka (4) Bihar (3) Jammu & Kashmir (4) Telangana
38. _______ is an informal body of Senior Ministers. 49. The first female speaker of Lok Sabha is
(Tamil Nadu-2021) (Uttar Pradesh 2019)
(1) Council of Ministers (2) Cabinet Ministers (1) Vijay Laxmi Pandit (2) Sucheta Kriplani
(3) Ministers of State (4) Cabinet (3) Tarkeshwari Sinha (4) Meira Kumar
39. The Legislative Council still in existence in _______ 50. The state in which Panchayati Raj was introduced first-
(Tamil Nadu-2021) (Uttar Pradesh 2019)
(1) Maharashtra (2) Tamilnadu (1) Uttar Pradesh (2) Bihar
(3) Odisha (4) Kerala (3) Rajasthan (4) Gujarat
40. The Elected Representatives in turn elect their representative 51. Who was the first Muslim President in India?
is known as _____ election. (Tamil Nadu-2021) (Uttar Pradesh 2019)
(1) District (2) Indirect (1) Fakhruddin Ali Ahmed (2) Dr. Zakir Hussain
(3) National Level (4) State Level (3) Salman Khursheid (4) Dr. Abdul Kalam Azad
41. Which organ of the United Nation passes the annual budget 52. Which of the following statement is incorrect regarding
of the United Nation as in function? (Maharashtra-2021) with first general elections of India? (Andhra Pradesh 2019)
(1) General Assembly (1) Only 10% of the population could vote in that elections.
(2) Security Council (2) Massive campaign to encourage the voters.
(3) Economic and Social Council (3) Separate ballot boxes for each candidate.
(4) Trusteeship Council (4) Symbols were introduced.
42. Choose the percentage which has been raised to increase the 53. With reference to the Fundamental Rights, consider the
representation of women in politics. (Maharashtra-2021) following statements: (Andhra Pradesh)
(1) 33% (2) 50% (A) Indian Constitution guarantees Fundamental Rights to
(3) 60% (4) 58% its citizen.
43. The correct group of statements related to fundamental (B) Fundamental Rights are absolute and never suspended.
duties is: (Karnataka-2021) Which of the statement/s given above is/are correct?
A. Fundamental duties are delineated in article 51 ‘A’. (1) Both (A) & (B) (2) Neither (A) Nor (B)
B. 11 fundamental duties have been delineated in the 3rd (3) (A) only (4) (B) only
chapter of our constitution. 54. Match Column – I with Column – II and select the correct
C. 10 fundamental duties were included as per the 42nd answer using the codes given below the columns.
amendment in the year 1976. (Andhra Pradesh 2019)
D. Through the 86th amendment to the constitution in Column – I Column – II
2002, the 11th fundamental duty was included Political Party State
(1) Only A, B and D (2) Only A, C and D A. SAD 1. Uttar Pradesh
(3) Only A, B and C (4) Only C and D B. DMK 2. Assam
44. Read the following statements and write the correct option C. AGP 3. Tamil Nadu
with which all those links. D. BLD 4. Punjab
A. It came into existence on 1st April 1974. (1) A-4, B-3, C-2, D-1 (2) A-4, B-2, C-3, D-1
B. It functions under the High court. (3) A-1, B-2, C-3, D-4 (4) A-1, B-3, C-2, D-4
EBD_7332
84
B- NTSE Stage 1 Question Bank

55. Which of the following statement is correct regarding with A. Centralization of power and responsibilities of village
“Coalition Government”? (Andhra Pradesh 2019) administration.
(1) Power shared by two or more political parties. B. Abolition of special powers of village administration.
(2) Power shared among Governments at different levels. C. Decentralize and confer constitutional status to the
(3) Power shared among different organisations of panchayat institutions.
Government. D. To bring village administration under the direct control
(4) Power shared by different social groups. of state.
56. The list ‘A’ contains fundamental rights and the list ‘B’ (1) A (2) B
their article numbers. Choose the correct option that (3) C (4) D
matches them: (Karnataka 2019) 62. Among the following who is a supporter of the Pluralistic
List-A List-B Theory of Democracy? (Rajasthan 2019)
A. Right to Freedom i. 23 and 24 articles (1) J.S. Mill (2) T.H. Green
B. Right to freedom of ii. 14 to 18 articles (3) Hobbes (4) H.J. Laski
religion 63. Who decides whether a bill is a money bill or not?
C. Right against iii. 29 and 30 articles (Rajasthan 2019)
exploitation (1) Prime Minister (2) President
D. Cultural and iv. 19 to 22 articles (3) Speaker of Lok Sabha (4) Vice-President
educational rights 64. ‘Fundamental Duties’ of the citizen of India are described in
v. 25 to 28 articles the Constitution of India under chapter (West Bengal 2019)
(1) A-ii, B-iii, C-i, D-iv (2) A-v, B-i, C-ii, D-iii (1) III (2) IV
(3) A-iv, B-v, C-i, D-iii (4) A-i, B-iv, C-iii, D-ii (3) V (4) VI
57. The process of creation of an act in the state’s legislature 65. How many members of the Rajya Sabha can be nominated
is jumbled up. Find the correct option that shows proper by the President of India ? (West Bengal 2019)
order of process of creation of an act: (Karnataka 2019) (1) 2 (2) 4
A. A discussion takes place on the bill and sent to (3) 6 (4) 12
respective house committee. 66. The President of India can Proclaim ‘National Emergency’
B. The house approves the bill with 2/3 majority. according to Article _________. (West Bengal 2019)
C. The person who tables the bill reads out the text loudly. (1) 350 (2) 352
D. The bill is sent to the Governor’s office for approval. (3) 356 (4) 360
E. The house committee deliberates on the bill and 67. The ‘Joint Session’ of the Parliament in India is presided
submits report to the legislative house. over by the ________. (West Bengal 2019)
(1) A C D B E (2) C A E B D (1) Vice-President (2) Speaker of the Lok Sabha
(3) E A C B D (4) C E B D A (3) Governor (4) President
58. He discussed the origin, development and functions of 68. Features of good governance are given below. Identify the
state in his book ‘Politics’. Identify the person: wrong option out of it. (Maharshtra 2019)
(Karnataka 2019) (1) Transparency in working of the government
(1) Plato (2) Aristotle (2) Responsive government.
(3) Socrates (4) Herodotus (3) Just and all inclusive development.
59. One of the following is not a function of Karnataka public (4) Unaccountable government.
service commission: (Karnataka 2019) 69. Choose the option, which is not only a political form of
A. Appointing the gazetted and non-gazetted officers. democracy? (Maharshtra 2019)
B. Conducting interviews for direct recruitment of (1) Elections
candidates. (2) Voting
C. Conducting departmental exams for state government (3) Governmental structure
employees. (4) Protection of Human values.
D. Train the teachers on modern skill of teaching. 70. Which of the following countries has community government?
(1) A (2) B (Bihar 2019)
(3) C (4) D (1) Spain (2) Sri Lanka
60. ‘Jharhkand Mukti Morcha’ Organization came in to being (3) Belgium (4) France
as a result of: (Karnataka 2019) 71. The administrative head of Municipal Corporation is called
(1) Long standing demands of people for the separate state. (Bihar 2019)
(2) Movement against atrocities of police force against (1) Mayor (2) Deputy-Mayor
innocents. (3) Municipal Commissioner (4) Sarpanch
(3) Actions of companies that rendered thousands of tribal 72. In democracy, power is finally concentrated in the hands
displacements due to mining. of (Bihar 2019)
(4) Movement for reservation of Schedule caste and (1) Bureaucrats (2) Parliament
Schedule tribe. (3) Electorate (4) Council of Ministers
61. The main aim of 73rd amendment of the constitution in
1993 with respect to Panchayatraj system is:
(Karnataka 2019)
Solved Questions 85
B-

73. Who among the following was not a member of the 84. Who Presides the Joint session of Parliament?
constituent assembly? (Delhi 2018) (Haryana 2018)
(1) Mahatma Gandhi (2) Jawahar Lal Nehru (1) President (2) Vice President
(3) Dr. Rajendra Prasad (4) Dr. B. R. Ambedkar (3) Speaker of Lok Sabha (4) Prime Minister
74. Which of the following Secretary General said that “US war 85. In the year 1971, Bangladesh become and independent and
on Iraq was not legal” (Delhi 2018) sovereign country as earlier it was the part of which country?
(1) Kofi Annan (2) B. B. Ghali (Gujarat 2018)
(3) U Thant (4) Ban Ki Moon (1) India (2) Afghanistan
75. President can declare emergency when- (Delhi 2018) (3) China (4) Pakistan
(1) Prime minister advises him to do so 86. Who was appointed as the chairman of state reorganisation
commision by Jawaharlal Nehru in 1953? (Gujarat 2018)
(2) Parliament advises
(1) Mr. Hridaynath kunzu
(3) The council of ministers, in writing, advises him to do so
(2) Mr. K. M. Panikar
(4) Home minister ask him to do so (3) Justice Fazal Ali
76. ‘End of Racial Discrimination’ is a part of which fundamental (4) Dr. Sarvapalli Radhakrishnan
right? (Delhi 2018) 87. Which word was inserted in the constitution by 42nd
(1) Right of Freedom Amendment 1976? (Gujarat 2018)
(2) Right of equality (1) Social (2) Political
(3) Right against exploitation (3) Sovereign (4) Secular
(4) Right to education and culture 88. In which year Union Summit was held at Kaula Lumpur?
77. The movement for the individual and family right of women (1) 2001 (2) 2003
is known as- (Delhi 2018) (3) 2004 (4) 2009
(1) Mahila Adhikar Aandolan 89. The Judge who handles the criminal cases is called?
(2) Mahila Shakti Aandolan (Gujarat 2018)
(3) Narivadi Aandolan (1) District Judge (2) Munsif Magistrate
(4) Nari Shasktikaran Aandolan (3) Session Judge (4) Supreme Court Judge
78. What is the meaning of ‘Transparency’? (Delhi 2018) 90. The name of the autobiography written by Nelson Mandela
(1) When decision is taken by the ruler is (Chandigarh 2018)
(2) When decision are made through leader’s conclusion (1) Robben island (2) The long walk to Freedom
(3) When decision are made for individual greeds (3) Blacks (4) Aparthied
(4) When decision are taken with honesty and proper 91. The ________ legislates on residuary subjects.
follow of rules (Chandigarh 2018)
79. The international organisation that works for human rights (1) Union Government (2) State Government
(3) Local Government (4) President
is- (Delhi 2018)
92. Which of these is not a good reason to say that Indian
(1) Amety International (2) Amnesty International
election is democratic? (Goa 2018)
(3) Asnesty International (4) Afnesty International (1) India has the largest number of voters in the world
80. India is a _______ (West Bengal 2018) (2) Indian Election Commission is very powerful.
I. Socialist II. Secular (3) In India, everyone above the age of 18 has a right to vote.
III. Sovereign IV. Democratic (4) In India, the losing parties accept the electoral verdict.
V. Republic 93. Union council of Ministers include following types on
Identify the correct sequence as maintained in the Preamble Minister. Choose the correct option. (Goa 2018)
of the Indian Constitution (i) Cabinet of Ministers.
(1) III, I, II, IV, V (2) II, III, V, I, IV (ii) Chief Minister
(3) III, V, I, IV, II (4) III, V, I, IV, II (iii) Minister of State with independent charges.
81. Parliament of India consists of (West Bengal 2018) (iv) Minister of State
(1) Rajya Sabha and Lok Sabha (1) (i) and (ii) (2) (ii)
(2) Election Commission, Rajya Sabha and Lok Sabha (3) (iii) (4) (i) (iii) and (iv)
(3) President, Election Commission, Rajya Sabha and Lok 94. The President of India may declare ‘National Emergency’
Sabha (Karnataka 2018)
(4) President, Rajya Sabha and Lok Sabha a. External aggression b. Internal disturbances
82. Which article is related to Equality before the Law in Indian c. Natural disasters d. Financial crises
Constitution? (West Bengal 2018) (1) a and b only (2) b and c only
(1) Art, 13 (2) Art, 14 (3) c and d only (4) a and c only
(3) Art, 15 (4) Art, 16 95. Identify the correct chronological sequence in which the
83. Choose the odd pair of personalities from the following- following became Secretary Generals of UNO.
(Karnataka 2018)
(Haryana 2018)
i. Antonio Guterres ii. U. Thant
(1) Mrs. Indira Gandhi and Narendra Modi
iii. Kofi Annan iv. Boutros Ghalli
(2) Balram Jakhar and Shivraj Patil (1) i, iii, ii, iv (2) ii, iv, iii, i
(3) Narendra Modi and Sumitra Mahajan (3) iii, iv, i, ii (4) iv, ii, i, iii
(4) Dr. Rajendra Prasad and Dr. V.V. Giri
EBD_7332
86
B- NTSE Stage 1 Question Bank

96. Read the following statements and select the correct (C) Rajendra Prasad (iii) Chairman of the
option. (Karnataka 2018) Drafting Committee
Assertion (A) : Directive principles are enshrined in the (D) T.T. Krishnamachari (iv) Legal Advisor
Constitution for the Government Administration. (1) (A) - iv, (B) - iii, (c) - i, (D) - ii
Reason (R) : People can question in the cout for not imple- (2) (A) - iv, (B) - ii, (c) - i, (D) - iii
menting Directive Principles. (3) (A)- i, (B)- iii, (c)- iv, (D)- ii
(1) A is false but R is true (4) (A) - iii, (B) - iv, (c) - i, (D) - ii.
(2) R is fasle but A is true 105. Choose the correct statement describing the word ‘code of
(3) A and R are true and R is correct explanation of A conduct’ : (Rajasthan 2017)
(4) A and R are true but R is not correct explanation of A (A) A set of norms and guidelines to be followed by
97. By whom the "Right to Constitutional Remedies" was Political Parties
considered as the soul and heart of Indian constitution? (B) A set of norms and guidelines to be followed by
(Delhi 2017) candidates in Election
(1) Mahatma Gandhi (2) Dr. Rajendra Prasad (C) Guidelines for Election Commission
(3) B. R. Ambedkar (4) Jawahar Lal Nehru (D) Compulsory voting for voters.
98. The distinguish feature of a federal government is (1) A, B, C (2) A, B
(Delhi 2017) (3) B, C (4) C, D
(1) National government gives some powers to the 106. According to the Constitution of India, how many maximum
provincial government no. of judges can be appointed in Supreme Court ?
(2) Power is distributed among the legislature, executive (Rajasthan 2017)
and judiciary (1) 29 + 1 (2) 30 + 1
(3) Elected officials exercise supreme power in the (3) 28 + 1 (4) 31 + 1
government 107. How many members will be nominated in Legislative
(4) Governmental power is divided between different level Council ? (Rajasthan 2017)
of government (1) 1/3 (2) 1/2
99. Following is a minority community in Belgium (3) 1/6 (4) 1/4
(Delhi 2017) 108. By which Article of the Constitution of India is the Prime
(1) Italian - speaking (2) French - speaking Minister appointed ? (Rajasthan 2017)
(3) Dutch - speaking (4) English - speaking (1) 74th (2) 75th
100. In which House is the finance bill presented first ? (3) 52nd (4) 61st
(Rajasthan 2017) 109. The Vice-President of India is elected by (Rajasthan 2017)
(1) Rajya Sabha (1) elected members of Lok Sabha
(2) Lok Sabha (2) all members of Rajya Sabha
(3) Both Lok Sabha and Rajya Sabha anywhere (3) elected members of Lok Sabha & Rajya Sabha
(4) Reserve Bank of India (4) all members of Lok sabha, Rajya sabha and all state
101. Who among the following is a part of the political executive? legislative assemblies
(Rajasthan 2017) 110. Match List - I and List - II and choose the correct code from
(1) District Collector the given codes : (Rajasthan 2017)
(2) Secretary of the Ministry of Home Affairs List - I List - II
(3) Home Minister (A) Union list (i) Computer Software
(4) Director General of Police' (B) State list (ii) Communications
102. Which of the following institutions can make changes to (C) Concurrent list (iii) Police
an existing law of our country ? (Rajasthan 2017) (D) Residuary powers (iv) Forests.
(1) Supreme Court of India A B C D
(2) International Court of Justice (1) iii ii i iv
(3) Prime Minister (2) ii iii iv i
(4) Parliament. (3) ii iv i iii
103. Which one of the following is considered as a fundamental (4) iv iii ii i
right according to the Constitution of India ? 111. “Political Science begins and ends with the State”, is stated
(1) Right to work (Rajasthan 2017) by (West-Bengal 2017)
(2) Right to adequate livelihood (1) Gettel (2) Garner
(3) Right to protect one's culture (3) Seeley (4) Aristotle
(4) Right to get higher education 112. How many members in the Lok Sabha can be nominated by
104. Match the following in reference to constitution making the President of India? (West-Bengal 2017)
Process : (Rajasthan 2017) (1) 2 (2) 3
(A) B.N. Rao (i) President of the (3) 4 (4) 5
Constituent Assembly 113. Joint Session of the Indian Parliament is presided over by the
(B) B.R. Ambedkar (ii) Member of the (West-Bengal 2017)
Drafting Committee (1) Vice-President (2) Speaker
(3) Governor (4) President
Solved Questions 87
B-

114. In Indian Parliamentary system of government the Council of C. Prime Minister III. Appointment of
Ministers is responsible to (West-Bengal 2017) Governors of State
(1) President (2) Prime Minister D. Finance Minister IV. Presentation of
(3) Parliament (4) Supreme Court Union Budget
115. In modern times Direct Democracy is existed in V. Act as Ex-Officio
(West-Bengal 2017) Chairperson of
(1) India (2) Britain Rajya Sabha
(3) France (4) Switzerland Codes:
116. The World Trade Organisation was established in the year (1) A-i, B-v C-iv D-ii
(West-Bengal 2017) (2) A-ii B-iii C-iv D-i
(1) 1990 (2) 1991 (3) A-iii B-v C-i D-iv
(3) 1994 (4) 1995 (4) A-iv B-ii C-iii D-i
117. The number of permanent members of the Security Council 122. List-A is the list of Regional Parties, List-B is their existence
of United Nations are (West-Bengal 2017) in the State and List -C is their symbol. (Karnataka 2017)
(1) 5 (2) 7 List-A List-B List-C
(3) 8 (4) 10 (Regional (State their (Symbol)
118. The United Nations Organisation was established in Parties) existence)
(West-Bengal 2017) A. Shivasena E. Tamil nadu I. Arrow
(1) 1945 (2) 1941 B. AIADMK F. Maharashtra J. Two leaves
(3) 1947 (4) 1950 C. Peoples G. Bihar K.Ink pot
119. Section 25 of Indian Constitution provides following right Democratic Party and pen
to Citizens of India (Karnataka 2017) D. JD (U) H. Jammu & L. Bow and
(1) Guarantees educational facilities to all Kashmir arrow
(2) Guarantees reservation in political field to scheduled The correctly matched set related to the above table is
castes and tribes (1) AFL, BEJ, CHK, DGI
(3) Guarantees reservation in the employment sector to (2) AFK, BGI, CEL, DHJ
scheduled castes and tribes (3) AEL, BGK, CHJ, DFL
(4) Guarantees free entry to all people to social and religious (4) AHI, BGJ, CFL, DEK
places 123. First Indian Women President is coded as (P), Prime Minis-
120. Match Column 'A' with Column 'B' and choose correct an- ter as (Q), Governor as (R) and Chief (Karnataka 2017)
swer. (Karnataka 2017) Minister as (S) and their names are given below.
Column-A Column-B a. Indira Gandhi b. Suchetha Kripalani
a. group of peopleI. Movement c. Prathiba Patil d. Sarojini Naidu
pelting stones Which one of the following choices represents PQRS order?
b. people assembled II. Campaign (1) c a b d (2) d c b a
near a theatre to (3) d a b c (4) c a d b
buy tickets 124. Regional Co-operation Organisations of different nations
c. A group protesting III. Riot are given below. (Karnataka 2017)
against a project I. SAARC II. ASEAN
in a peaceful way III. European Union IV. African Unity
d. A group of people IV. Mob Identify the correct choice of these organisations in the
involved in bringing chronological order of establishment.
awareness (1) I III IV II
a b c d (2) IV II I III
(1) III IV I II (3) III I II IV
(2) I II III IV (4) II IV III I
(3) III I IV II 125. Read the following statements and select the correct option.
(4) IV II I III Assertion (A): The Governor of a State cannot be dismissed
121. Match list 01 of Heads/Ministers of Executive of Indian by the Chief Minister. (Karnataka 2017)
Government with List 02 of their “functions” and select the Reason (R): The Governor of a State is not elected.
correct answer using the codes given below. (1) Both ‘A’ and ‘R’ are true and ‘R’ is the correct
(Karnataka 2017) explanation of ‘A’
List-01 List-02 (2) Both ‘A’ and ‘R’ are true and ‘R’ is not the correct
(Heads of Executive (Functions) explanation of ‘A’
of Indian Government) (3) ‘A’ is true but ‘R’ is false
A. President I. Chairperson of the (4) ‘R’ is true but ‘A’ is false
NITI Ayoga 126. Which of the following statement/statements about Anti
B. Vice President II. Appointment of Defection Law is/are correct? (Karnataka 2017)
Chief Minister to A. The Constitution Act of 1985 is popularly known as
States the Anti Defection Law.
EBD_7332
88
B- NTSE Stage 1 Question Bank

B. Independently elected member of Parliament or a State 127. Consider the following two statements on power sharing
Legislature will not be disqualified if he/she join any and select the answer using the codes given below.
political party after elections. (Andhra Pradesh 2017)
Choices: (a) Power sharing is good for democracy.
(1) Only A is correct (b) It helps to reduce the possibility of conflicts between
(2) Only B is correct social groups.
(3) Both A and B are correct Which of these statements are true and false ?
(4) Both A and B are incorrect (1) Both a and b are true. (2) Both a and b are false.
(3) a is true but b is false. (4) a is false but b is true.
128. Match the ministry with the news that the ministry may have released. (Andhra Pradesh 2017)

A B
(a) A new policy is being made to increase the jute (i) Ministry of Defence.
exports from the country. (ii) Ministry of Health.
(b) Telephone services will be made more (iii) Ministry of Agriculture, Food and Public
accessible to rural areas. Distribution.
(c) The price of rice and wheat sold under the (iv) Ministry of Commerce and Industry.
Public Distribution system will go down. (v) Ministry of Communications and Information
(d) A pulse polio campaign will be launched. Technology.
(e) The allowances of the soldiers posted on high
altitudes will be increased.

Codes :
(a) (b) (c) (d) (e)
(1) I iii ii iv V
(2) iv v iii ii i
(3) iii v ii i iv
(4) ii v iii iv i
129. Find out the right which is not under the Indian 132. Find out the wrong statement about National Human Rights
Constitution? (Andhra Pradesh 2017) Commission. (Andhra Pradesh 2017)
(1) Freedom of Speech and Expression. (1) This is an independent Commission established by
(2) Move freely through the Country. law in 1993.
(3) Practise any profession. (2) Present Chairman for National Human Rights
(4) None of the above. Commission is Justice Jeevan Reddy.
130. Match the following countries and the path democracy has (3) Like National Human Rights Commission, there are
taken in that country. (Andhra Pradesh 2017) State Human Rights Commissions in 14 states of the
Country Path to Democracy country.
(A)Nepal (i) End of One party Rule. (4) There is no fee or any formal procedure to approach
(B) Chile (ii) King agreed to give up his powers. the National Human Rights Commission.
(C) Ghana (iii) End of Military Dictatorship. 133. Find out the subject which is under concurrent list ?
(D Poland (iv) Freedom from British Colonial Rule. (Andhra Pradesh 2017)
Codes: (1) Police (2) Communication
A B C D (3) Marriages and Divorce (4) None of the above.
(1) (i) (iii) (iv) (iii) 134. A struggle known as “Bolivia’s water war” took place in …
(2) (ii) (iii) (iv) (i) city. (Andhra Pradesh 2017)
(3) (iii) (ii) (i) (iv) (1) Cochabamba (2) Lapaz
(4) (iv) (i) (iii) (ii) (3) Trinidad (4) Montero
131. Consider the following statements about pressure groups 135. Which of the following does not figure in preamble Indian
and parties. (Andhra Pradesh 2017) Constitution? (Haryana 2016)
(a) Pressure groups are the organised expression of the (1) Democratic (2) Secular
interests and views of specific social sections. (3) Republic (4) Religious Justice
(b) Pressure groups take positions on political issiiesi 136. In which game P.V. Sindhu won the silver medal for India in
(c) All pressure groups are political parties. Rio Olympics? (Haryana 2016)
Which of the statements given above are correct ? (1) Lawn Tenis (2) Badminton
(1) a, b and c (2) a and b (3) Wrestling (4) Archery
(3) b and c (4) a and c
Solved Questions B-89
137. Which of the following statements properly define the (3) The Parliamentary Select Committee
'Constitution'? (Rajasthan 2016) (4) The Speaker of the Lok Sabha
(A) Constitution protects the rights of citizens 149. In contemporary world, which quality makes democracy
(B) It determines the functioning of governments superior to any other form of Government:
(C) It determines the process of legislation (Maharashtra 2014)
(D) It decides the name of person who is going to be the (1) Efficiency (2) Responsiveness
President. (3) Transparency (4) Legitimacy governance
(1) A, B, D (2) B, C 150. Gender division refers to (Maharashtra 2014)
(3) A, B, C (4) B, C, D (1) Unequal child sex ratio
138. Which one of the following is the Institution, functioning (2) Absence of voting rights
for International law, Security, Social equality and World (3) Biological differences
peace? (Rajasthan 2016) (4) Unequal roles assigned to men and women
(1) International Monetary Fund 151. When was the “Kittiko-Hachchiko” movement started?
(2) United Nations Organisation (Maharashtra 2014)
(3) World Bank (1) 1984 (2) 1986
(4) None of these (3) 1987 (4) 1983
139. Who among the following was the pioneer, to abolish caste 152. Who is the Political Head of Municipal Corporation?
inequality and establish social harmony? (Rajasthan 2016) (Maharashtra 2014)
(1) Sir Sayed Ahmed Khan (2) Dadabhai Naoroji (1) Collector (2) Pradhan
(3) W. C. Bonerjee (4) Dr. B. R. Ambedkar (3) Mayor (4) Sarpanch
140. President of India is elected by (Delhi 2015) 153. Which one of the following parties grew out of a movement?
(1) By the people (Maharashtra 2014)
(2) By all the elected members of Parliament and State (1) Communist Party of India
Legislature (2) Bahujan Samaj Party
(3) By the members of Lok Sabha and Vidhan Sabha (3) Assam Gana Parishad
(4) By the members of Rajya Sabha (4) Assam United Democratic Front
141. ‘End of Racial Discrimination’ this fundamental right is a 154. Amnesty International is an international organisation which
part of (Delhi 2015) works for (Maharashtra 2014)
(1) Right to equality (1) World Peace (2) Human Rights
(2) Right to Freedom (3) Justice (4) Restoration of Democracy
(3) Right to Education and Culture 155. Name the Law which empowers the people to carry on
(4) Right against exploitation democratic reforms: (Maharashtra 2014)
142. The Headquarter of world Bank is- (Chhattisgarh 2015) (1) Company Act (2) M. R. T. P.
(1) Geneva (2) Newyork (3) Right to Information (4) Right to Vote
(3) London (4) Washington D.C. 156. Which of the following is the main feature of Pressure group?
143. When was Universal Human Rights day declared- (Maharashtra 2014)
(Delhi 2015) (1) Direct control on political power
(1) 1st October 2001 (2) 10th December 1948 (2) Try to influence the politics of Government
(3) 1st March 1950 (4) 1st April 2000 (3) Lax Organisation
144. The Constitution of India was adopted on (4) Direct participation in Political Power
(West Bengal 2015) 157. In case of conflict between the centre and the State
(1) 9 December, 1946 (2) 14 August, 1947 Government over a subject in the concurrent list:
(3) 26 November, 1949 (4) 26 January, 1950 (1) The state has the upper hand (Maharashtra 2014)
145. The Chairman of the Drafting Committee of the Constituent (2) The centre has the upper hand
Assembly of India was (West Bengal 2015) (3) Both are equally applicable
(1) B. R. Ambedkar (2) Jawaharlal Nehru (4) Supreme Court decides whether the State Law should
(3) Sardar Vallabhbhai Patel (4) Dr. Rajendra Prasad prevail or Central Law
146. ‘Democracy is a government of the people, for the people, 158. Total languages in the Eighth schedule of Indian
by the people’ was said by (West Bengal 2015) Constitution (Karnataka 2014)
(1) George Washington (2) Woodrow Wilson (1) 18 (2) 19
(3) Abraham Lincoln (4) Roosevelt (3) 21 (4) 22
147. The Chief Justice of the Supreme Court is appointed by the 159. Which one of the following is an anti-national and economic
(West Bengal 2015) crime? (Karnataka 2014)
(1) President (2) Vice-President (1) Smuggling (2) Communalism
(3) Prime Minister (4) Attorney General (3) Corruption (4) Economic inequality
148. When a bill presented at the table of Lok Sabha who decides 160. In 1946, the features of Indian foreign policy was explained
whether a bill is Money Bill or not? (West Bengal 2015) by (Karnataka 2014)
(1) The Prime Minister (1) Indira Gandhi (2) Dr. B. R. Ambedkar
(2) The President (3) Pandit Jawaharlal Nehru (4) Lal Bahaddur Shasthri
EBD_7332
90
B- NTSE Stage 1 Question Bank

161. ‘The Untouchability Crime Act’ was implemented in 173. “Code of conduct” is related to ? (Rajasthan 2013)
(Karnataka 2014) (1) War (2) Transportation
(1) 1948 (2) 1950 (3) Cereals (4) Election
(3) 1956 (4) 1955 174. Which of the following country has adopted the one (single)
162. Name the Law which empowers the people to carry on Party System? (Rajasthan 2013)
democratic reforms: (Delhi 2014) (1) India (2) America
(1) Company Act (2) M. R. T. P. (3) Japan (4) China
(3) Right to Information (4) Right to Vote
175. Which institution of the following institutions reserves seat
163. Which of the following is the main feature of Pressure group?
for women? (Delhi 2012)
(1) Direct control on political power (Delhi 2014)
(2) Try to influence the politics of Government (1) Lok Sabha (2) Rajya Sabha
(3) Lax Organisation (3) Vidhan Sabha (4) Panchayat
(4) Direct participation in Political Power 176. What do you understand by the term ‘defection’?
164. Which of the following locations is not correct? (Delhi 2012)
(Chandigarh 2014) (1) Change of party allegiance from the party on which a
Name of the organization Location of Headquarter person got elected to a legislation body to a different
(1) UNESCO Paris party
(2) ILO Geneva (2) Change of the preference of the voters
(3) FAO Brussels (3) Internal politics of Party
(4) International Court of the Hague (4) Change of the party before election
Justice 177. Which article of the Indian Constitution is regarded as the
165. Which of the following statements about the international Soul of Indian Constitution? (Delhi 2012)
Court of justice is not correct? (Chandigarh 2014) (1) Art. 21 (2) Art. 31
(1) It is the principla judicial organ of the United Nations (3) Art. 19 (4) Art. 32
(2) Its judges are elected
178. How many members are there in the security council of
(3) It s decisions are not enforceable
United Nation? (Punjab 2012)
(4) Nations aggrieved by the non-implementation of the
(1) 15 (2) 20 (3) 17 (4) 22
decision have no further redress
166. Who was the King of Nepal in 2006 during the ‘second 179. What is ‘Zero Hour?’ (Punjab 2012)
movement for democracy’? (Delhi 2014) (1) When the proposals of the opposition are considered.
(1) King Virendra (2) King Gyanendra (2) When the matters of utmost importance are raised.
(3) King Vijendra (4) King Tejendra (3) When money bill is introduced in the Lok Sabha.
167. Which of the following people movement later converted (4) Interval between the morning and the evening session.
into a political party? (Delhi 2013) 180. Who among the following is a part of Political Executive?
(1) Assam Movement (1) District collector (Punjab 2012)
(2) Chipko Movement (2) Secretary of the ministry of Home Affairs
(3) Narmada Bachao Aandolan (3) Home Minister
(4) All of the above (4) Director General of Police
168. By which Act, Communal Electoral System was introduced 181. When was Universal Adult Franchise granted in India?
in India ? (Madhya Pradesh 2013) (1) 1948 (2) 1950 (Punjab 2012)
(1) Indian Councils Act, 1892 (3) 1952 (4) 1954
(2) Indian Councils Act, 1909 182. By which country India was inspired to include Directive
(3) Indian Councils Act, 1919
Principles of State Policy in the Indian Constitution?
(4) Indian independence Act, 1947
(1) Britain (Madhya Pradesh 2012)
169. In which year the first meeting of Constituent Assembly
took place ? (Madhya Pradesh 2013) (2) United States of America
(1) 1945 (2) 1946 (3) Russia (4) Ireland
(3) 1947 (4) 1948 183. In which year IX Schedule was included in the Indian
170. By which Five Year Plan, Community Development Programme Constitution ? (Madhya Pradesh 2012)
was launched in India? (Madhya Pradesh 2013) (1) 1950 (2) 1951
(1) First (2) Second (3) 1952 (4) 1953
(3) Third (4) Fourth 184. Which part became 22nd State of India on 26th April, 1975 ?
171. Who is the Chairman of Planning Commission in India ? (Madhya Pradesh 2012)
(Madhya Pradesh 2013) (1) Nagaland (2) Tripura
(1) President (2) Prime Minister (3) Himachal Pradesh (4) Sikkim
(3) Planning Minister (4) Vice-President 185. “The long walk to freedom” is autobiography of ?
172. Who is the first Person of India ? (Rajasthan 2013) (Rajasthan 2012)
(1) Prime Minister (2) President (1) Nelson Mandela (2) Mahatma Gandhi
(3) Governor (4) Chief Minister (3) Barak Obama (4) Amitabh Bachchan
Solved Questions B-91
186. Who can seek information from government under “right to 196. Rule of laws means (Andhra Pradesh 2012)
information Act”? (Rajasthan 2012) (1) Law spelt out in terms of rules and regulation
(1) Any Citizen (2) Equal law related to the whole country
(2) Only government officials (3) Law related to judiciary
(3) Only elected members of Loksabha (4) Law that does not recognized any special privilege
(4) Only elected members of Vidhan Sabha based on birth or wealth
187. The first nation of the world which provided adult franchise 197. In India, the first general elections were held in this year
is- (Rajasthan 2012) (Andhra Pradesh 2012)
(1) America (2) India (1) 1951 (2) 1952
(3) Brazil (4) Newzealand (3) 1950 (4) 1947
188. Which of the following is not a feature of Fundamental 198. The international court of Justice is located in this place
Rights in India? (Odisha 2012) (Andhra Pradesh 2012)
(1) They are enumerated in Part III of our constitution (1) New York (2) Paris
(2) They are absolute in nature (3) Hague (4) Yugoslavia
(3) Some of them are positive and some are negative in 199. This was not the reason for Indo-China war in 1962
nature (Andhra Pradesh 2012)
(4) They are Judiciable in the Court of law (1) Border dispute between two countries
189. Who of the following does prorogue the Houses of the (2) Chinese regarded India as their main rival in Asia
Parliament in India? (Odisha 2012) (3) Growing friendship between India and former Soviet
(1) President (2) Vice President Union
(3) Speaker of Lok Sabha (4) Prime Minister (4) Growing friendship between India and U.S.A.
190. The voting age in India was lowered from 21 years to 18 200. Which one of the following is a wrong statement?
years by which Amendment Act of the constitution? (Maharashtra 2012)
(1) 24th Amendment Act, 1971 (Odisha 2012) (1) The Supreme Court is the highest court of record.
(2) 42nd Amendment Act, 1976 (2) The Supreme Court reconsiders the decisions.
(3) 44th Amendment Act, 1978 (3) The Supreme Court is the guardian of the Constitution.
(4) 61st Amendment Act, 1989 (4) The Supreme Court decisions are not final and binding
191. Which of the following is not a challenge to the National on all other courts.
Integration in India? (Odisha 2012) 201. Which of the following is not adopted in Indian Federal
(1) Regionalism (2) Casteism System? (Maharashtra 2012)
(3) Secularism (4) Communalism (1) India is a country with an extensive territory.
192. Who is the Supreme Commander of the Indian Armed (2) Indian Society is multi-lingual.
Forces? (West Bengal 2012) (3) Indian Society is multi-religious.
(1) The President of India (4) Indian Society has only one political family.
(2) The Prime Minister of India 202. Which person is appointed by the Indian Administrative
(3) The Defence Minister of India Service for Local Government? (Maharashtra 2012)
(4) The Parliament of India (1) Corporation (2) Municipal Corporation
193. Who presides over the joint session of the Indian (3) Panchayat Samiti (4) Gram Panchayat
parliament? (West Bengal 2012) 203. Which one of the following is included in the Concurrent
(1) The Prime Minister of India List? (Maharashtra 2012)
(2) The President of India (1) Environment (2) Defence
(3) The Chairman of the Rajya Sabha (3) Post and Telegraph (4) Railway
(4) The Speaker of the Lok Sabha 204. The highest body set up in India to protect Human Rights
194. The words ‘socialist and ‘secular’ were included in the is: (Karnataka 2012)
preamble to the Indian Constitution by which Amendment (1) Human Rights Committee
Act (West Bengal 2012) (2) Human Rights Commission
(1) 42nd (2) 44th (3) National Human Rights Committee
(3) 62nd (4) 73rd (4) National Human Rights Commission
195. The Government of which country is Unitary in form? 205. ‘SAPTA’ is an agreement on Trade among the SAARC
(1) The United States of America (West Bengal 2012) National. The expansion of SAPTA is one of the following:
(2) Britain (Karnataka 2012)
(3) India (1) South Asian Protection of Trade Agreement
(4) Switzerland (2) South Asian Partial Trade Agreement
(3) South Asian Preferential Trade Agreement
(4) South Asian Pacific Trade Agreement
EBD_7332
92
B- NTSE Stage 1 Question Bank

SECTION 7. ECONOMICS
Economics 12. In underdeveloped countries most of the labour force is
generally engaged (West Bengal-2020-21)
1. Golden Revolution in India is related to: (Delhi 2020-21)
(1) Industrial sector (2) Service sector
(1) Jewelry Export
(3) Agricultural sector (4) Banking sector
(2) Gold mines
13. The nation having a mixed economy is
(3) Honey and Horticulture
(Rajasthan-2020-21)
(4) Electronic Goods
(1) China (2) India
2. When an able person is willingly unemployed while there
(3) Japan (4) France
is an opportunity to work is known as (Delhi 2020-21)
14. The Central bank of India is (Rajasthan-2020-21)
(1) Disguised unemployment
(1) State Bank
(2) Voluntary unemployment
(2) Indian Industrial Development Bank
(3) Seasonal unemployment
(3) Export-Import Bank
(4) Educated unemployment
(4) Reserve Bank of India
3. Which of the following prepares the Human Development
15. The process which increases the role of private sector in
Report? (Delhi 2020-21)
economy, is called (Rajasthan-2020-21)
(1) Planning Commission of India
(1) Liberalization (2) Privatization
(2) International Monetary Fund
(3) Globalization (4) Swadeshi
(3) World Health Organisation
16. At present which form of money is increasingly used apart
(4) United Nation Development Programme
from paper money? (Bihar-2020-21)
4. In which city of Haryana is the automobile industry
(1) Commodity money (2) Metallic money
situated? (Delhi 2020-21)
(3) Plastic money (4) All of the above
(1) Faridabad (2) Gurgaon
17. Mixed economy means an economy where there is
(3) Panipat (4) Sonipat
(Bihar-2020-21)
5. Consider the following taxes: (Andhra Pradesh-2020-21)
(1) Existence of Capitalism
(A) Corporate tax (B) Property tax
(2) Privatization, Liberalization and globalization
(C) Income tax (D) Service tax
(3) Existence of both public and private sectors
Which of the above is/are NOT included in direct tax?
(4) Growing crop along with rearing animals
(1) A only (2) C only
18. __________ scheme was launched in 1970-1971 in
(3) A, B and C only (4) D only
Maharashtra. (Maharashtra-2020-21)
6. In our country as per Indian National Co-operative Union
(1) Rural Water Supply
guidelines all the states conduct cooperative week
(2) Nutritious diet
celebrations: (Andhra Pradesh-2020-21)
(3) To supply more electricity for pumps
(1) From November 14th to 20th
(4) Pulse Polio Vaccination
(2) From January 10th to 16th
19. Paper Money Spread to Europe through the travels of
(3) From April 10th to 16th
________ (Maharashtra-2020-21)
(4) From July 1st to 7th
(1) Marcopolo (2) Columbus
7. The World Trade Organization was founded in __________.
(3) Magellan (4) Stanley
(West Bengal-2020-21)
20. _______ is an innovator of new ideas and business
(1) 1990 (2) 1995
processes (Maharashtra-2020-21)
(3) 2000 (4) 2005
(1) Entrepreneur (2) Professionals
8. In which economy is the policy of Laissez-faire adopted?
(3) Businesspeople (4) Organizer
(West Bengal-2020-21)
21. The List -A contains the laws and the List-B with their year
(1) Capitalist Economy (2) Socialist Economy
of implementation. Choose the correct option that matches
(3) Mixed Economy (4) Any Economy
exactly. (Karnataka-2020-21)
9. Exclusion principle is not applicable in the case of.
List - I List - II
(West Bengal-2020-21)
A. Minimum wages act i. 1956
(1) Capital goods (2) Consumer goods
B. Prevention of unethical ii. 1993
(3) Public goods (4) Private goods
activities act
10. Railway in India highlighted by which of the following market
C. Prohibition of bonded iii. 2005
form? (West Bengal-2020-21)
labour act.
(1) Perfect competition (2) Monopolistic competition
D. Human Rights iv. 1976
(3) Monopoly (4) Oligopoly
protection act
11. Which of the following taxes follows the ability to pay
v. 1948
principle? (West Bengal-2020-21)
(1) A - i, B - ii, C - v, D - iii (2) A - ii, B - iii, C - i, D - iv
(1) Wealth Tax (2) Entertainment Tax
(3) A - v, B - i, C - iv, D - ii (4) A - iv, B - v, C - ii, D - i
(3) Goods and Services Tax (4) Excise Duty
Solved Questions 93
B-

22. Read the given statements and select the correct answer : (2) Controlling monetary policy.
(Karnataka-2020-21) (3) Controlling government spending.
Assertion (a): Journal is called the book of original entry. (4) Acting as a banker’s bank.
Reason (r): The full particulars of the transactions are 32. Which indicators are used in the Human Development Index
recorded first in the Journal. (HDI) ? (Bihar 2019)
(1) Both ‘a’ and ‘r is true and ‘R is the correct explanation I. Standard of living II. Education
of ‘a’ III. Life expectancy rate IV. Condition of environment
(2) Both ‘a’ and ‘Eis true and ‘R is not the correct (1) Only I, II and IV (2) Only I, II and III
explanation of ‘a’ (3) Only I and II (4) All of the above
(3) ‘a’ is true and ‘R is false 33. Which of the following statements are true about
(4) ‘a’ is false and ‘R is true globalisation? (Bihar 2019)
23. One of the statements related to Labourers is wrong. I. Developed countries have always been more benefitted
Identify it: (Karnataka-2020-21) from globalisation.
(1) Organised labourers come under the gratuity Act of II. Globalisation has led to improvement in living condition
1971. of workers in the developing countries.
(2) Unorganised sector labourers are responsible for child III. Globalisation is the process of rapid integration or
labour. interconnection between countries.
(3) A large section of farmers are unorganised labourers. IV. Indian cement industries have been hit hard by
(4) Migration is the major challenge faced by organised globalisation.
sector labourers. V. To achieve the goal of fair globalisation, major roles
24. Choose the correct group of Direct tax: can be played by MNCs.
(Karnataka-2020-21) (1) Only I, II, III and V (2) Only I, II, III and IV
(1) Corporate tax, Wealth tax, Central Excise tax, Service Tax. (3) Only III, IV and V (4) Only I, II and III
(2) Income Tax, Stamp Duty, Service Tax, Export Tax. 34. In a SHG (self help group) most of the decisions regarding
(3) Value Added Tax, Service Tax, Corporate Tax, Export Tax. saving and loan activities are taken by- (Bihar 2019)
(4) Income Tax, Corporate Tax, Wealth Tax, Stamp Duty. (1) Bank (2) Members
25. Identify the correct statements related to Antyodaya Anna (3) State Government (4) Chairperson of SHG
Yojana. (Karnataka-2020-21) 35. Which of the following are correct regarding WTO?
i. It was launched in July 2001. (i) Its main aim is to liberalise international trade.
ii. Under this scheme one crore of the poorest among the (ii) It was started at the initiative of the developed countries.
BPL families covered under the targeted public (iii) The rules of WTO are framed to favour the developing
distribution system were identified. countries.
iii. Poor families were identified by the respective state (iv) It establishes rules regarding international trade.
rural development departments through a BPL survey. (1) Only (ii) and (iii) (2) Only (i), (ii) and (iv)
iv. Fifty kilograms of food grains were made available to (3) Only (iii) and (iv) (4) All of these
each eligible family. 36. Terms of credit does not include
(1) Only i and ii are correct (2) Only ii and iii are correct (1) Interest rate (2) Collateral
(3) Only iii and iv are correct (4) Only i and iv are correct (3) Cheque (4) Mode of repayment
26. The Blue Revolution is related with (Uttar Pradesh 2019) 37. In the rural areas, the unorganised sector mostly comprises of
(1) Food Grain Production (2) Fish Production (i) Landless agricultural labourer.
(3) Milk Production (4) Oil seed Production (ii) Garment makers.
27. The Oil and Natural Gas Commission (ONGC) was set up (iii) Street vendors.
in (Uttar Pradesh 2019) (iv) Sharecroppers and artisans.
(1) 1956 (2) 1957 (1) (iii) and (iv) (2) (i) and (iv)
(3) 1959 (4) 1961 (3) (i) and (ii) (4) (ii) and (iii)
28. I.M.F was established by the recommendations of which 38. Which of the following methods can be used by the
committee? government for a fair globalisation?
(1) Bretton Woods committee (2) Goswami committee (i) impose trade barriers.
(3) Narsingham committee (4) None of them (ii) negotiate at the WTO for fairer rules.
29. Where is situated the Headquarters of World Bank? (iii) align with other developing countries.
(Uttar Pradesh 2019) (iv) close its market for foreign trade.
(1) Texas (2) Canada (1) Only (i) and (ii) (2) Only (ii) and (iv)
(3) Washington (4) Geneva (3) All of these (4) Only (i), (ii) and (iii)
30. Who has first developed the theory of Rent? 39. The purpose of Pradhan Mantri Mudra Yojana is:
(Uttar Pradesh 2019) (Karnataka 2019)
(1) Ricardo (2) Adam Smith A. To ensure access to financial services in nationalised
(3) Marshall (4) None of them banks.
31. Which of the following is not a function of the central B. To help every rural citizen to open bank account.
bank in an economy ? (Bihar 2019) C. To provide loan for small businessmen/startups from
(1) Dealing with foreign exchange. 50,000 to 10 Lakh.
EBD_7332
94
B- NTSE Stage 1 Question Bank

D. To provide life insurance to all citizens between the (3) Availability and accessibility of food to all at all the time
age of 18 to 50 years. (4) Availability, accessibility and affordability of food to
(1) A (2) B all at all the time
(3) C (4) D 53. Per capita income is calculated by : (Tamil Nadu 2018)
40. Assertion (A): The private sector consists of business TotalPopulation Total Population
owned by individuals or a group of individuals. (1) (2)
Reason (R): The private sector may either be partly or GrossDomesticProduct National Income
wholly owned by central or state government as per 1991 National Income Gross Domestic Product
industrial policy. (Karnataka 2019) (3) (4)
Total Population Total Population
(1) ‘A’ is right, ‘R’ is the correct explanation of ‘A’
(2) ‘A’ is right, ‘R’ is not the correct explanation of ‘A’ 54. Which is the tenth largest stock exchange in the world and
(3) Both ‘A’ and ‘R’ are wrong oldest stock exchange in South Asia? (Tamil Nadu 2018)
(4) Only ‘A’ is wrong (1) National Stock Exchange (2) Madras Stock Exchange
41. The White Revolution is related to (Rajasthan 2019) (3) Bombay Stock Exchange (4) Calcutta Stock Exchange
(1) Production of eggs (2) Production of milk 55. Find out the wrong statement. (Telangana 2018)
(3) Production of sugar (4) Production of rice (1) Employment in the service sector has not increased to
42. The World Trade Organization was established on the same extent as production.
(Rajasthan 2019) (2) Most of the workers in the organised sector enjoy the
(1) 1st January, 1935 (2) 1st April, 1935 job security.
(3) 1st January, 1995 (4) 1st April, 1995 (3) The activities in primary, secondary and tertiary
43. The reason of inflation in India is (Rajasthan 2019) sectors are interdependent.
(1) Rapid growth in agricultural production (4) None of the above
(2) Rapid growth in industrial production 56. International Co-operative day is celebrated every year on
(3) Low level of public expenditure the (Andhra Pradesh 2018)
(4) High level of public expenditure (1) First Saturday of July
(2) First Saturday of August
44. India’s first petro-chemical industry is (West Bengal 2019)
(3) Second Saturday of July
(1) UCIL (2) HPL
(4) Second Saturday of August
(3) IPL (4) NOCIL
57. “We have not inherited the world from our forefathers - we
45. Diamond Quadrilateral related to (West Bengal 2019)
have borrowed it from our children”. (Andhra Pradesh 2018)
(1) Metro Rail (2) High Speed Railways
- This quote expects us :
(3) Roadways (4) Waterways
(1) to use non-renewable resource extensively.
46. Which one of the following is not a characteristic of a (2) to extract more ground water.
Capitalist Economy ? (West Bengal 2019) (3) to prefer sustainability of development.
(1) Private Ownership of resources (4) to prefer rapid industrialisation.
(2) Freedom of enterprise 58. The policy of the government which works through the
(3) Consumer sovereignty budget is known as (West Bengal 2018)
(4) Existence of Central Planning Authority (1) monetary policy (2) income policy
47. Human Development Index measures _________ of an (3) commercial policy (4) fiscal policy
economy. (West Bengal 2019) 59. Those workers whose marginal productivity is _______
(1) Birth rate (2) Death rate are called disguisedly employed (West Bengal 2018)
(3) Quality of education (4) Quality of life (1) maximum (2) minimum
48. Which of the following organisations looks after the credit (3) zero (4) positive
needs of agriculture and rural development in India? 60. Indicators involved in Human Development Index (HDI)
(Delhi 2018) (Karnataka 2018)
(1) FCI (2) IDBI (3) NABARD (4) SBI (1) National Income, Employment Rate and Sex Ratio
49. How many phases are there in circular flow of income? (2) Per Capita Income, Life Expectancy and Literacy Rate
(Delhi 2018) of women
(1) 2 (2) 3 (3) 6 (4) 5 (3) Life Expectancy, Literacy, Attainment and Purchasing
50. In which year was the Integrated Child Development Power of People
Service (ICDS) Introduced (Delhi 2018) (4) National Income, Purchasing power of People and Sex
(1) 1965 (2) 1975 Ratio
(3) 1985 (4) 1995 61. Aspects of Money supply concepts are given below.
51. Globalization was not stimulated by - (Bihar 2018) A. Currency Notes (Karnataka 2018)
I. Money II. Transport B. Coins
III. Population IV. Computer C. Savings deposits in Post Office
(1) Only I is correct (2) Only I and II are correct D. Time/term deposits of Commerical Banks
(3) I, III and IV are correct (4) I, II, III and IV are correct The Group which classifies the above aspects as ‘Narrow’
52. What does food security mean? (Bihar 2018) and ‘Broad’ Money respectively are
(1) Availability of food (1) A, B and C, D (2) A, C and B, D
(2) Accessibility of food (3) A, D and B, C (4) B, D and A, C
Solved Questions 95
B-

62. Removing barriers or restriction set by the government is (1) Money (2) Land
called (Chandigarh 2018) (3) Labour (4) Capital
(1) Liberalization (2) Investment 76. The main aim of ______ programme was to provide
(3) Favourable trade (4) Free trade employment of 100 days per year to one member of rural
63. Which of the following is considered as a component of family (West-Bengal 2017)
social infrastructure? (Delhi 2017) (1) TRYSEM (2) IRDP
(1) Transport (2) Education (3) NREGS (4) JGSY
(3) Communication (4) Energy 77. The expenditure of government for payment of government
64. Which bank first introduced credit card in India employees is expenditure on _____ account
(1) Central Bank ofIndia (2) State Bank of India (West-Bengal 2017)
(3) ICICI Bank (4) HDFC Bank (1) revenue (2) capital
65. In which five year plan, Mahalanobis Model was adopted (3) development (4) investment
in India? (Delhi 2017) 78. Which of the following is not a public goods?
(1) Fifth (2) First (West-Bengal 2017)
(3) Second (4) Third (1) Roads and bridges (2) Administration
66. Rise in Green Net National Income implies higher sustainable (3) Food products (4) Defence
development in an economy. Green Net National Income is 79. If the value of domestic currency falls in terms of foreign
the difference between. (Delhi 2017) currency then (West-Bengal 2017)
(1) Difference between Net National Income and gross (1) import payment will increase and export earnings will
savings. also increase
(2) Difference between Net National Income and (2) import payment will fall and export earnings will also fall
depreciation of natural capital (3) import payment will increase and export earnings will fall
(3) Difference between Net National Income and (4) import payment will fall and export earnings will increase
depreciation of man-made capital 80. The principle based on which the insurer is liable only for
(4) Both (2) and (3) those losses which have been insured against is
67. In India the currency note is issued by (Rajasthan 2017) (1) Principle of indemnity (Karnataka 2017)
(1) Reserve Bank of India (2) State Bank of India (2) Principle of contributions
(3) NABARD (4) Bank of India (3) Principle of loss minimization
68. The source of institutional credit is (Rajasthan 2017) (4) Principle of proximate cause
(1) Money lender (2) Landlord 81. Match the items in List-A Entrepreneurs with List-B
(3) Bank (4) Relatives. Entrepreneurship (Karnataka 2017)
69. The example of tertiary sector is (Rajasthan 2017) a. Naresh Goyal I. Infosys
(1) Agriculture b. Kiran Mazumdar II. Reliance Shah
(2) Fisheries c. Narayan Murthy III. Balaji Telefilms
(3) making sugar from sugarcane d. Dhirubai Ambani IV. Biocan
(4) Banking services V. Jet Airways
70. The Government of India enacted the law of "Right to a b c d
Information" Act in. (Rajasthan 2017) (1) V IV I II
(1) October, 2005 (2) November, 2006 (2) IV V I II
(3) December, 2007 (4) January, 2008 (3) III V I II
71. The Multinational Company of India is (Rajasthan 2017) (4) V III IV II
(1) Infosys (2) Asian Paints 82. Identify the group of Financial Institutions with gives loan
(3) Tata Motors (4) All of these. to small scale industries. (Karnataka 2017)
72. If national income increases at a higher rate than population (1) IDBI, IFCI, SFC, SIDBI
the per capita income (West-Bengal 2017) (2) RBI, NABARD, IFCI, IDBI
(1) increases (2) decreases (3) NABARD, IDBI, RBI, SFC
(3) remains same (4) may increase or decrease (4) SIDBI, RBI, NABARD, IDBI
73. To control the situation of deflation it is necessary to 83. Identify the correct group of Nationalised Banks
(West-Bengal 2017) (Karnataka 2017)
(1) increase the demand for bank loan (1) Canara Bank, State Bank of India, Vijaya Bank, Dena
(2) decrease the demand for bank loan Bank, IDBI Bank
(3) decrease the purchasing power of the people (2) IDBI Bank, ICICI Bank, Karnataka Bank, Dena Bank,
(4) increase national saving Canara Bank
74. Economic rent is the price paid for the use of (3) IDBI Bank, Vijaya Bank, ICICI Bank, Karnataka Bank,
(West-Bengal 2017) Canara Bank
(1) land only (2) scarce resources (4) Vijaya Bank, Canara Bank, HDFC Bank, ICICI Bank,
(3) machinery only (4) building only Dena Bank
75. Which of the following is not a factor of production?
(West-Bengal 2017)
EBD_7332
96
B- NTSE Stage 1 Question Bank

84. Calculate the Male Literacy Rate from given data and choose (3) Wheat is supplied at the rate of Rs.6 and rice at the
the correct answer from the given choices. (Karnataka 2017) rate of Rs.7 under this scheme.
Gender Total Persons Literate persons (4) None of the above.
Male 1350 981 90. Find out the correct one related to under employment.
Female 785 435 (1) They do not want to work. (Andhra Pradesh 2017)
Total 2135 1416 (2) They work in a lazy manner.
(1) 78.5% (2) 66.3% (3) They work less than what they are capable, of doing.
(3) 55.4% (4) 72.6% (4) They are not paid for their work.
85. The following are the major changes that occured in agri- 91. Find out the wrong one about Secondary sector.
culture as a result of Green Revolution in India. (Andhra Pradesh 2017)
(Karnataka 2017) (1) Secondary sector is also called as industrial sector.
a. Use of high yield variety seeds in wheat grains only (2) Manufacturing of bricks and sugar come under
b. The poor and marginal small farmers avail the benefits secondary sector.
of improved technology (3) The share of secondary sector is more in current GDP
c. Construction of granaries, cold storages warehouses of India.
to store the surplus produce (4) None of the above.
d. The improved technology of Green Revolution was 92. Which among the following is money function ?
restricted and practices in northern states of India i.e., (Andhra Pradesh 2017)
Pubjab, Haryana, Rajasthan States. Which of the (1) Medium of exchange. (2) Unit of account.
above signifies post harvest technology of Green (3) Store of value. (4) All the above.
Revolution? 93. Consider the following statements about Globalisation.
(1) a and b (2) b and c (Andhra Pradesh 2017)
(3) c and d (3) d and a (1) The most common route for investment by MNC’S in
86. Whom would you consider as unemployed related to the countries around the world is to buy existing local
following situations? (Karnataka 2017) companies.
A. A farmer in farm produces 100 kg paddy in one acre of (2) Investment made by Multinational companies is called
land during every season. In addition one year his foreign investment.
adult son Raju joined him in farming. But output (3) Cargill Foods, an American company purchased an
remained the same. Indian company called Parakh Foods.
B. Raghu has income by his huge property to lead (4) Ford Motors is one of the biggest German Automobile
comfort life. So he does not work. manufacturer.
(1) Raju is unemployed Which of the given statements are True?
(2) Raghu is unmeployed (1) a, c, d (2) a, b,c
(3) Both Raju and Raghu are unemployed (3) b, c, d (4) a, b, c, d
(4) Both Raju and Raghu are not unemployed 94. In which year, did the Bengal Famine occur, which was
87. Indicators of development are given below. responsible for the death of 30 lakh people in Bengal
(Karnataka 2017) Province ? (Andhra Pradesh 2017)
a. National Income b. Per capita Income (1) 1933 (2) 1943
c. Life expectancy d. Educational Achievement (3) 1953 (4) 1963
e. Rate of women’s job participation 95. Find out the wrong one related to Annapurna Scheme (APS).
f. Standard of living (1) Introduced in the year 2000. (Andhra Pradesh 2017)
Which of the above indicators are measured in Human (2) A scheme meant for indigent senior citizens.
Development Index? (3) 10 kg of food grains are supplied freely under the scheme.
(1) a, b, c (2) c, d, f (4) None of the above.
(3) a, b, e (4) d, e, f 96. Match List-I with List-II and select answer using the appropriate
88. Consider the following statements. (Andhra Pradesh 2017) code from among the following alternatives. (Odisha 2016)
(i) Equitable allocation of resources. List-I List-II
(ii) Generation of employment. A. Disparities in income in (I) Trickle Down theory
(iii) Tax concession to big corporates. a developing economy
(iv) Universalisation of public distribution. B. Economic development (II) Change in
Which of the factors given above can bring inclusive benefits the poor occupational structure
growth in our country ? C. Shifting of labour from (III) Less redistribution of
(1) (i), (ii), (iii) (2) (i), (ii), (iv) agricultural to non income in favour of
(3) (i), (iii), (iv) (4) (ii), (Hi), (iv) agricultural sector poor
89. Which of the following is wrong related to Antyodaya Anna D. Increase in the (IV) Human development
Yojana ? (Andhra Pradesh 2017) capabilities of people
(1) Antyodaya Anna Yojana was launched in December 2000. (1) A - II, B - III, C - I, D - IV (2) A - I, B - II, C - III, D - IV
(2) 2 crore families have been covered under the (3) A - III, B - I, C - II, D - IV (4) A - IV, B - II, C - I, D - III
Antyodaya Anna Yojana.
Solved Questions B-97
97. If Life Expectancy Index for a country is 0.53, Educational (1) Jim Yong Kim (2) Lewis Preston
Attainment Index is 0.67 and Per Capital Real GDP Index is (3) Barber Conable (4) None of these
0.42, then HDI for the country will be (Odisha 2016) 109. When was the National Development Council formed?
(1) 0.93 (2) 0.70 (Jharkhand 2015)
(3) 0.54 (4) 0.68 (1) 26th January, 1950 (2) 15th March, 1950
98. If cash reserve ratio of banks is 20% and currency reserve in (3) 6th August, 1951 (4) 6th August, 1952
the banking system amount to 50 million rupees, the 110. Creation of utility for exchange is called
maximum amount of demand deposits which can be created (West Bengal 2015)
by the banks is (Odisha 2016) (1) Consumption (2) investment
(1) 200 million rupees (2) 250 million rupees (3) Production (4) Savings
(3) 500 million rupees (4) 1000 million rupees 111. In which of the following situation total demand for a
99. The Government of India supplies food grains and other necessary commodity will rise? (West Bengal 2015)
essential commodities of BPL households through fair price (1) Rise in price level (2) Rise in population size
shops. Name of the programme is (Odisha 2016) (3) Rise in production (4) Rise in the rate of interest
(1) ICDS (2) MDM 112. Inflation indicates a situation of (West Bengal 2015)
(3) PDS (4) Antodaya (1) very high prices (2) continuous fall in prices
100. Identify incorrect sentence related to Asian continent: (3) continuous rise in prices (4) very low prices
(Maharashtra 2016) 113. An economy is said to be on the path of growth when
(1) This continent is the biggest of all from the (1) total production increases (West Bengal 2015)
perspectives of area and population (2) per capita income increases
(2) The continent got the name from the word 'Aasu' (3) inequality in the distribution of income falls
(3) The renaissance era was started from this continent (4) total population increases
(4) The emergence of old religion and culture this 114. Which organization in india is called the “Lender of the last
continent resort” for the commercial banks? (West Bengal 2015)
101. ‘NABARD’ is a __________ (Delhi 2015) (1) ICICI (2) IDBI
(1) Poverty alleviation Programme (3) NABARD (4) RBI
(2) Bank 115. Who is known as the father of Green Revolution?
(3) Social Security Scheme (Maharashtra 2014)
(4) Agriculture Marketing agency (1) Verghese Kurien (2) Babu Jagjivan Ram
102. What was the aim of ‘Antyodaya Programme’? (Delhi 2015) (3) M. S. Swaminathan (4) M. S. Subhramanyam
(1) Helping the poorest of poor 116. An example of Indirect Taxes is: (Maharashtra 2014)
(2) Upliftment of SC/ST (1) Income Tax (2) Corporate Tax
(3) Women empowerment (3) Estate Tax (4) Entertainment Tax
(4) Children welfare 117. Which academy supports food security programme?
103. ‘MNREGA’ is reffered to- (Chhattisgarh 2015) (Maharashtra 2014)
(1) Right to information (2) Right to Education (1) Academy of Development Science
(3) Right to work (4) Right to protection of life (2) Academy of Crop Science
104. Which will be a feature of the developing country? (3) Academy of Food Security
(Chhattisgarh 2015) (4) Academy of Grain Science
(1) Agriculture as the main occupation 118. Which summit of BRICS held in 2014?(Maharashtra 2014)
(2) Mass poverty (1) 5th (2) 6th (3) 4th (4) 7th
(3) Mass illiltracy 119. COPRA is related to: (Maharashtra 2014)
(4) High technological development (1) Trader (2) Environment
105. The most appropriate measure of a country's economic (3) Agriculture (4) Consumer
growth is (Jharkhand 2015) 120. Where is the headquarters of Life Insurance Corporation
(1) Gross Domestic Product (2) Net Domestic Product situated ? (Madhya Pradesh 2013)
(3) Net National Product (4) Per Capita Product (1) Kolkata (2) Chennai
106. Finance is distributed between the centre and the states on (3) Mumbai (4) New Delhi
the recommendations of which of the following ? 121. What is GDP? (Rajasthan 2013)
(1) Planning Commission (Jharkhand 2015) (1) Gross Daily Production
(2) Public Accounts Committee (2) Gross Domestic Production
(3) Finance commission (3) Gross Domestic Power
(4) National Development Council (4) Gross Development Production
107. The first Agricultural University of the country is 122. According to Census-2011 the literacy rate of Rajasthan?
(Jharkhand 2015) (Rajasthan 2013)
(1) J.N.K.V. Jabalpur (2) G..B.P.A.U. Pant Nagar (1) 48.34% (2) 54.90%
(3) P.A.U. Ludhiana (4) R.A.U. Bikaner (3) 67.06% (4) 74.04%
108. Who amongst the following is the current President of the
World Bank (Jharkhand 2015)
EBD_7332
98
B- NTSE Stage 1 Question Bank

123. In which year did the Economic Liberalisation start in India? (1) increases (2) decreases
(Rajasthan 2013) (3) remains unchanged (4) none of the above
(1) 1991 (2) 1996 137. The Consumer Protection Act, 1986 ensures which of the
(3) 1999 (4) 2004 following as rights which every consumer in India should
124. Which one of the following is the main cause of land posses (West Bengal 2012)
degradation in Punjab? (Delhi 2012) (1) right to choice (2) right to information
(1) Deforestation (2) Over Irrigation (3) right to redressal (4) all of the above
(3) Over grazing (4) Intensive cultivation 138. Micro Economics is also known as
125. Which of the following agencies markets steel for the public (Andhra Pradesh 2012)
sector plants? (Delhi 2012) (1) Income theory (2) Price theory
(1) HAIL (2) SAIL (3) Expenditure theory (4) Savings theory
(3) Tata Steel (4) MMTC 139. This is not a social and demographic indicator under
126. Which of the following is a non-farm activity? Economic development (Andhra Pradesh 2012)
(Punjab 2012) (1) Literacy rate
(1) Multiple cropping (2) Crop rotation (2) Urbanisation
(3) Dairy farming (4) Modern farming (3) Increase in employment
127. Which one of the following organization prepares ‘Human (4) Life expectancy
Development report’? (Punjab 2012) 140. This is an example for direct taxation
(1) UNO (2) WHO (1) Customs - Imports (Andhra Pradesh 2012)
(3) IMF (4) UNDP (2) Wealth – Profits
128. The National Rural Employment Guarantee Act enacted by (3) Excise – Manufacturing
legislation on : (Punjab 2012) (4) Turn-over - Sale
(1) July 20th 2006 (2) August 25th 2005 141. The factor which does not cause any increase in demand
(3) August 25 th 2004 (4) July 20th 2000 for Goods and Services is : (Maharashtra 2012)
129. In which year Reserve Bank was set up ? (1) Reduction in taxation
(Madhya Pradesh 2012) (2) Increase in exports
(1) 1930 (2) 1935 (3) 1940 (4) 1945 (3) Increase in population
130. Which Bank was merged with Punjab National Bank in 1993 ? (4) Hoarding/Malpractices of goods
(Madhya Pradesh 2012) 142. The term ................. means to increase the value of domestic
(1) New Bank of India (2) Bank of Maharashtra currency compared to foreign currency. (Maharashtra 2012)
(3) Kashi Nath Bank (4) lndus Bank (1) Over valuation
131. Which of the following is a demerit of globalisation? (2) Increase in Price index
(1) Consumerism (Odisha 2012) (3) Monetary measures
(2) Employment opportunity (4) Bloatedness/Inflated currency
(3) Proper use of capital and resources 143. A sole objective of GATT set up is : (Maharashtra 2012)
(4) Cultural Osmosis (1) creating a global environment of free trade
132. Density of Population indicates the _____.(Odisha 2012) (2) to create international peace
(1) Capital-Land Ratio (2) Land-Output Ratio (3) to develop underdeveloped/developing countries
(3) Land-Labour Ratio (4) Man-Land Ratio (4) to boost Tariff and non-Tariff Trade
133. ‘Jago Grahak Jago’ slogan is meant for _______. 144. Which of the following statements are associated with co-
(Odisha 2012) operative farming? (Karnataka 2012)
(1) Consumers (2) Producers A. Lands of all the members (farmers) are pooled and cultivated.
(3) Labourers (4) Distributors B. The members lose ownership rights of their lands.
134. What type of unemployment is found in Indian agricultural C. A major portion of the yield is shared among farmers in
sector? (Odisha 2012) proportion to their size of lands.
(1) Seasonal unemployment D. Management of all the activities, is under the control
(2) Cyclical unemployment of the Government.
(3) Voluntary unemployment (1) A and C only (2) B and D only
(4) Disguised unemployment (3) B, D and C only (4) A, B and C only
135. Human Development Index (HDI) is prepared on the basis 145. The purpose of devaluation of Indian rupee is :
of (West Bengal 2012) (Karnataka 2012)
(1) 2 basic indicators (2) 3 basic indicators (1) To reduce imports and to increase exports.
(3) 4 basic indicators (4) 5 basic indicators (2) To increase imports and to reduce exports.
136. When market supply remains unchanged and demand (3) To reduce differences between value of goods and
increases, then equilibrium price (West Bengal 2012) value of services imported.
(4) To maintain favourable balance of payments.
Solved Questions B-99

SECTION 8. GEOGRAPHY

Geography Codes:
(1) (i), (ii), (iii) all are correct (2) (i), (ii) are correct
1. Coffee cultivation was first introduced in (3) (ii), (iii) are correct (4) (i), (iii) are correct
(Delhi 2020-21) 12. Baltic sea has very low salinity. Which of the following factors
(1) Himalayas (2) Aravali Hills is mainly responsible for this? (Andhra Pradesh-2020-21)
(3) Garo Hills (4) Baba Budan Hills (1) Evaporation (2) Precipitation
2. Which one of the following describes a system of agriculture (3) Ocean currents (4) Fresh water flow from rivers
where a single crop is grown on a large area? 13. Match List-I with List-II and select the correct answer using
(Delhi 2020-21) the codes given below. (Andhra Pradesh-2020-21)
(1) Shifting Agriculture (2) Plantation Agriculture List - I (Coast) List-II (State)
(3) Horticulture (4) Intensive Agriculture (A) Coromandel (i) Karnataka
3. Neyveli lignite mines are located in the state of (B) Circar (ii) Kerala
(Delhi 2020-21) (C) Malabar (iii) Andhra Pradesh
(1) Kerala (2) Tamil Nadu (D) Canara (iv) Tamil Naidu
(3) Karnataka (4) Andhra Pradesh Codes:
4. Hirakund Dam is constructed on the river – (1) A – i, B – ii, C – iv, D – iii (2) A – i, B – ii, C – iii, D – iv
(Delhi 2020-21) (3) A – iv, B – iii, C – ii, D – i (4) A – iv, B – ii, C – i, D – iii
(1) Ganga (2) Manjira 14. Assertion (A): Coromandel coast remains mostly dry during
(3) Manas (4) Mahanadi the South-West monsoon season.
5. When and where first cement plant was set up? Reason (R): It is in the rain shadow area of the Arabian sea
(Delhi 2020-21) branch and is parallel to the Bay of Bengal branch.
(1) Chennai, 1905 (2) Chennai, 1904 (Andhra Pradesh-2020-21)
(3) Kolkata, 1905 (4) Kolkata, 1904 (1) Both ‘A’ and ‘R’ are true and ‘R’ is the correct
6. Who wrote the book “Small is Beautiful”? explanation of ‘A’.
(Delhi 2020-21) (2) Both ‘A’ and ‘R’ are true but ‘R’ is not the correct
(1) Gandhi Ji (2) Brundtland explanation of ‘A’.
(3) Schumacher (4) Annie Besant (3) ‘A’ is true but ‘R’ is false.
7. According to the main role industry can be divided into (4) ‘A’ is false but ‘R’ is true.
(1) Agro-based and mineral-based (Delhi 2020-21) 15. Consider the following statements.
(2) Key and consumer industries (Andhra Pradesh-2020-21)
(3) Public, private, and Joint sector (i) India is the largest producer of raw jute and jute goods
(4) Heavy and light industries in the world.
8. Which of the following is not a renewable resource? (ii) The first cement plant in India was setup in Jamshedpur
(Delhi 2020-21) in 1904.
(1) Forest (2) Animals Which of the above statements is/are CORRECT?
(3) Water (4) Petroleum (1) (i) only (2) (ii) only
9. Consider the following statements. (3) Both (i) and (ii) (4) Neither (i) nor (ii)
(Andhra Pradesh-2020-21) 16. Consider the following statements
(i) Laying ‘Pakka’ roads were devised by McAdam (Andhra Pradesh-2020-21)
(ii) The steam locomotive made by Stephenson was (i) Infant mortality rate is the number of children who die
named ‘Rocket’. within one year, our of 100 live children born.
Which of the above statements is/are CORRECT? (ii) Literacy rate is the percentage of the literate population
(1) (i) only (2) (ii) only in the 6 and above age group.
(3) Both (i) and (ii) (4) Neither (i), nor (ii) Which of the above statements is/are CORRECT?
10. Thick barks and wax-coated leaves is an important feature (1) (i) only (2) (ii) only
of: (Andhra Pradesh-2020-21) (3) Both (i) and (ii) (4) Neither (i) nor (ii)
(1) Tropical evergreen forests 17. Which one of the following groups of state has the largest
(2) Temperate evergreen number of cotton textile centres?
(3) Temperate deciduous forests (Andhra Pradesh-2020-21)
(4) Mediterranean vegetation (1) Gujarat and Maharashtra
11. Consider the following statements and select the correct answer (2) Karnataka and Tamil Nadu
using the codes given below. (Andhra Pradesh-2020-21) (3) Maharashtra and Madhya Pradesh
(4) Uttar Pradesh and Gujarat
(i) India is the second-largest producer of paddy in the
18. Which of the following regions is suitable for constructing
World after China.
railway lines? (Andhra Pradesh-2020-21)
(ii) India is the largest producer as well as the consumer
(1) Himalaya Mountains Range
of pulses in the world.
(2) Garo, Khasi and Jaintia Hills
(iii) India is the largest producer of sugarcane in the world.
(3) The northern Plains
(4) Rajasthan desert
EBD_7332
100
B- NTSE Stage 1 Question Bank

19. The time difference between Greenwich Mean Time and (1) West Bengal (2) Maharashtra
the Indian Standard Time is (West Bengal-2020-21) (3) Kerala (4) Punjab
(1) 6 hours (2) 5 hours 30 minutes 35. Where is ropeway in Bihar? (Bihar-2020-21)
(3) 5 hours 15 minutes (4) 5 hours (1) Bihar Shariff (2) Rajgir
20. The climate which is found in the Cape Town of South (3) Gaya (4) Munger
Africa is (West Bengal-2020-21) 36. Select the correct statements – (Bihar-2020-21)
(1) Equatorial climate (2) Tropical Monsoon climate (A) Koshi river is the sorrow of Bihar
(3) Mediterranean climate (4) Hot Desert climate (B) Parrot is the national bird of India
21. Which of the following, parallel of latitude, passes through (C) Maruti Industry is situated in Delhi
the middle of India? (West Bengal-2020-21) (D) Varanasi is situated on the bank of river Ganges
(1) Equator (2) Tropic of Capricon (1) A and D (2) B and C
(3) Prime Meridian (4) Tropic of Cancer (3) B, C and D (4) A, C and D
22. The lake located, in between the deltas of the Godavari 37. Which name is correct for Patna Airport?
river and the Krishna river is (West Bengal-2020-21) (Bihar-2020-21)
(1) Kolleru (2) Pulicat (1) Jai Prakash Narayan International Airport
(3) Chilka (4) Vembnad (2) Patna Airport
23. Diamond Quadrilateral project is related to (3) Rajendra Prasad International Airport
(West Bengal-2020-21) (4) Bihar Airport
(1) Air Transport (2) Rail Transport 38. Which statement is not correct? (Bihar-2020-21)
(3) Road Transport (4) Water Transport (A) Medha Patekar is related to Narmada Bachao Andolan.
24. The colour used for drawing contour lines in a topographical (B) New alluvial soil is termed as Bangar
map is (West Bengal-2020-21) (C) Mangrove Forest found in the coastal area in India.
(1) Black (2) Brown (D) Plantation agriculture is one of the types of commercial
(3) Red (4) Blue farming.
25. Which lake is located in Rajsamand district? (1) A (2) B
(Rajasthan-2020-21) (3) B, C, and D (4) C and D
(1) Tal chhapar (2) Nauchouki 39. The Golden Quadrilateral Super highway connected with
(3) Tordisagar (4) Navlekha the following – (Bihar-2020-21)
26. Which one of the following is a food crop? (1) Jammu, Bikaner, Jodhpur, Rajkot
(Rajasthan-2020-21) (2) Porbandar, Bikaner, Amritsar, Srinagar
(1) Sugarcane (2) Tobacco (3) Delhi, Mumbai, Chennai, Kolkata
(3) Barley (4) Tea (4) Sikkim, Siliguri, Jorhat, Agartala
27. Which district in Rajasthan is called as cement city? 40. The Ragur Soil is also known as:
(Rajasthan-2020-21) (Uttar Pradesh-2020-21)
(1) Chittorgarh (2) Kota (1) Red Soil (2) Yellow Soil
(3) Bundi (4) Pratapgarh (3) Black Soil (4) Alluvial Soil
28. Which of the following is the highest Literacy-rate district 41. Which of the following is the longest river of the world?
of Rajasthan? (Rajasthan-2020-21) (Uttar Pradesh-2020-21)
(1) Ajmer (2) Jaipur (1) Amazon river (2) Yangtze river
(3) Kota (4) Udaipur (3) Ganga river (4) Nile river
29. Which of the following is ferrous metallic mineral? 42. The Toda tribes are the original inhabitants of:
(Rajasthan-2020-21) (Uttar Pradesh-2020-21)
(1) Tin (2) Cobalt (1) Aravalli hills (2) Nilgiri hills
(3) Gold (4) Bauxite (3) Satpura hills (4) Guru Shikhar
30. Which is the major source of rainwater storage in the 43. The city where the first Earth summit was organized.
Shekhawati region? (Rajasthan-2020-21) (Uttar Pradesh-2020-21)
(1) Khadin (2) Tanka (1) Rio de Janeiro (2) Shangai
(3) Nadi (4) Johad (3) Tokyo (4) Manila
31. Who coined the slogan “Jo sir sante Roonkh rahe to bhi 44. Manrega was implemented from the year?
sasto Jaan”? (Rajasthan-2020-21) (Uttar Pradesh-2020-21)
(1) Smt. Amrita Devi (2) Smt. Jaana Bai (1) 2005 (2) 2006
(3) Smt. Sushila Devi (4) Smt. Harsha Devi (3) 2007 (4) 2008
32. The Green revolution started in (Rajasthan-2020-21) 45. Where is the headquarter of Tea Board located?
(1) 1966-67 (2) 1970-71 (Uttar Pradesh-2020-21)
(3) 1988-89 (4) 2001-02 (1) Darjeeling (2) Bengaluru
33. Bharatpur Bird sanctuary is situated in – (3) Kolkata (4) Mumbai
(Bihar-2020-21) 46. Which wind is referred to as “roaring forties” at 400 ‘furious
(1) Gujarat (2) Rajasthan fifties’ at 500 and ‘screaming =sixties’ at 600 latitudes?
(3) Assam (4) Bihar (Tamil Nadu-2020-21)
34. The highest Literacy rate in India is in ……..? (1) Trade winds (2) Easterlies
(Bihar-2020-21) (3) Westerlies (4) Cyclones
Solved Questions B-101
47. Pulicat lake lies in the ________ district 61. Identify the wrong pair: (Maharashtra-2020-21)
(Tamil Nadu-2020-21) River Tributary
(1) Kanchipuram (2) Chennai (1) Ganga – Yamuna
(3) Tiruvalluvar (4) Vellore (2) Sindhu – Satluj
48. Match the following: (Tamil Nadu-2020-21) (3) Krishna – Tungabhadra
COLUMN – I COLUMN – II (4) Tapi – Bhima
(A) Tehri Dam (i) Sutlej 62. The name Gaja (Cyclone) was named by _______
(B) Hirakud Dam (ii) Bhagirathi (Maharashtra-2020-21)
(C) Nagarjuna Sagar Dam (iii) Mahanadi (1) Bangladesh (2) Sri Lanka
(D) Indira Gandhi Canal (iv) Krishna (3) Pakistan (4) Oman
(1) (A) – (ii), (B) – (iii), (C) – (iv), (D) – (i) 63. Match Column-A with Column-B and choose the correct
(2) (A) – (iii), (B) – (iv), (C) – (i), (D) – (ii) answer. (Karnataka-2020-21)
(3) (A) – (ii), (B) – (iv), (C) – (iii), (D) – (i) Column – A Column – B
(4) (A) – (iv), (B) – (iii), (C) – (ii), (D) – (i) (Ports) (States)
49. ______ is not a part of the world network biosphere A. Nhava Sheva i. West Bengal
reserves of UNESCO. (Tamil Nadu-2020-21) B. Tuticorin ii. Tamilnadu
(1) Nilgiri (2) Agasthiyam C. Paradip iii. Odisha
(3) Great Nicobar (4) Kachch D. Haldia iv. Andhra Pradesh
50. ______ is the most common but powerful means of v. Maharashtra
communication. (Tamil Nadu-2020-21) (1) A - ii, B - iv, C - i, D - iii (2) A - v, B - ii, C - iii, D - i
(1) Internet (2) Social Media (3) A - iii, B - v, C - iv, D - ii (4) A - i, B - iii, C - ii, D - v
(3) Print Media (4) Television 64. Read the following statements and select the correct answer:
51. The name Gaja (Cyclone) was named by _______ (Karnataka-2020-21)
(Tamil Nadu-2020-21) (i) It forms from the weathering of Granite and gneiss
(1) Bangladesh (2) Sri Lanka rocks.
(3) Pakistan (4) Oman (ii) It has a more sandy and less clayey composition.
52. The full form of HCR (Tamil Nadu-2020-21) (iii) It does not retain moisture.
(1) Head Count Ratio (2) Human Center Ratio (iv) Total area covered in India about 5.2 lakh sq. km.
(3) Head Calculating Rate (4) Head Continue Ratio (1) Red soil (2) Black soil
53. Identify the first nuclear reactor of India functioning on (3) Laterite soil (4) Alluvial soil
atomic energy. (Maharashtra-2020-21) 65. Match List-I with List-II. Select the correct answer from the
(1) Dhruv (2) Apsara code given below: (Karnataka-2020-21)
(3) Tarapur (4) Zarina List I List - II
54. _________ from the Western Rajasthan is considered as A. Vulnerable species i. Nicobar Pigeon
the driest part in India. (Maharashtra-2020-21) B. Rare species ii. Blue sheep
(1) Jaisalmer (2) Ajmer C. Endemic species iii. Asiatic cheetah
(3) Jodhpur (4) Jaipur D. Extinct species iv. Wild Asiatic buffalo
55. In India about _______ present of passengers are carried (1) A - i, B - ii, C - iii, D - iv (2) A - iii, B - i, C - iv, D - ii
by road. (Maharashtra-2020-21) (3) A - ii, B - iv, C - i, D - iii (4) A - iv, B - iii, C - ii, D - i
(1) 75% (2) 65% 66. Read the description and choose the correct answer:
(3) 85% (4) 80% (Karnataka-2020-21)
56. In which direction of Brazil, there is no sea coast? i. It is a joint venture of Punjab, Haryana and Rajasthan.
(Maharashtra-2020-21) ii. It is built across River Sutlej.
(1) East (2) West iii. It is 226 metres high and is the highest straight gravity
(3) South (4) North dam in India.
57. Which one of the following is not a major trading partner of iv. It has four power projects (houses).
Brazil? (Maharashtra-2020-21) (1) Hirakud project (2) Bhakra Nangal project
(1) Pakistan (2) Germany (3) Rihand Valley project (4) Damodar Valley project
(3) India (4) Canada 67. Read the following statements: (Karnataka-2020-21)
58. What type of tourism is developing in Brazil? (i) Delhi, Kolkata, Chennai and Mumbai cities are linked
(Maharashtra-2020-21) by six-lane highways.
(1) Historical (2) Cultural (ii) Srinagar to Kanyakumari linking North to South.
(3) Eco-tourism (4) Professional (iii) Silchar and Porabander connect East to West.
59. India is located in the _________ hemispheres of the earth. (iv) The major objective is to reduce the time and distance
(Maharashtra-2020-21) between the megacities.
(1) Southern and Eastern (2) Northern and Eastern The above statements refer to :
(3) Northern and Western (4) Southern and Western (1) National Highways
60. ___________ is the most urbanized state in India. (2) Border Roads
(Maharashtra-2020-21) (3) State Highways
(1) Maharashtra (2) Gujarat (4) Golden Quadrilateral superhighways
(3) Goa (4) Kerala
EBD_7332
102
B- NTSE Stage 1 Question Bank

68. The Himalayan mountain range is an example of (4) Mudumalai – Tamilnadu


(Uttar Pradesh 2019) Periyar – Kerala
(1) Block mountain (2) Folding mountain Bharathpur – Gujarat
(3) Volcanic mountain (4) Residual mountain Jaladapara - Assam
69. The forest of Ganga-Brahmputra-delta is known as 80. Read the following statements and choose the correct
(Uttar Pradesh 2019) answer: (Karnataka 2019)
(1) Evergreen Forest (2) Monsoon Forest Assertion (A): Laterite soil is not that much useful for
(3) Sundar Ban (4) Deciduous Forest agriculture.
70. How many districts are there in Uttar Pradesh? Reason (R): Laterite soil undergoes more leaching process.
(Uttar Pradesh 2019) (1) Both (A) and (R) are correct and (R) is the correct
(1) 70 (2) 75 explanation of (A)
(3) 80 (4) 85 (2) Both (A) and (R) are correct but (R) is not the correct
71. In which continent the Sahara desert is situated- explanation of (A)
(Uttar Pradesh 2019) (3) (A) is correct, (R) is incorrect
(1) South America (2) Africa (4) (A) is incorrect, (R) is correct
(3) Asia (4) North America 81. Match the marked multipurpose river valley projects on
72. Identify the crop which is cultivated in the tropical the map of India (i, ii, iii, iv) with their respective names:
highlands of India and grows well particularly on the laterite (Karnataka 2019)
soils of Karnataka and Tamilnadu. (Bihar 2019)
(1) Groundnut (2) Cotton
(3) Coffee (4) None of these
73. Agricultural forestry is mainly practised in which state of
India? (Bihar 2019) b
(1) Jharkhand (2) Haryana
(3) Rajasthan (4) Uttrakhand
74. Shimoga mines is famous for (Bihar 2019) c
(1) Iron ore (2) Gold
(3) Manganese (4) Petroleum a
75. Which of the following statements is not true regarding
India’s climate? (Andhra Pradesh 2019)
(1) The climate of India is described as the monsoon type d
(2) The North-East monsoons are responsible for most of
the rainfall in India
(3) The climate of India is strongly influenced by monsoon
winds
(4) India’s climate has characteristics of tropical as well i. Bhakra Nangal ii. Nagarjuna Sagar
as subtropical climate iii. Kosi iv. Hirakud
76. Which of the following lakes is a fresh water lake? (1) a-ii, b-iii, c-i, d-iv (2) a-iv, b-i, c-iii, d-ii
(Andhra Pradesh 2019) (3) a-i, b-iv, c-ii, d-iii (4) a-iii, b-i, c-iv, d-ii
(1) Sambhar (2) Dal 82. The type of vegetation shown in the picture is found in
(3) Pulicat (4) Chilika this type of forests in India: (Karnataka 2019)
77. Kudremukh is an important Iron ore mine of
(Andhra Pradesh 2019)
(1) Andhra Pradesh (2) Karnataka
(3) Madhya Pradesh (4) Kerala
78. The ocean beds are rich in (Andhra Pradesh 2019)
(1) Copper (2) Iron
(3) Manganese (4) Gold
79. The group of wild life sanctuaries and the states in which
they are located is given below. Identify the correctly
matched group: (Karnataka 2019)
(1) Manas - Meghalaya
Bhadra - Karnataka
Annamalai – Tamilnadu
Mudumalai – Kerala (1) Mangrove Forest (2) Mountain Forest
(2) Biligiriranga Hills - Karnataka (3) Monsoon Forest (4) Evergreen Forest
Ranthambore – Gujarat 83. Settlements become sparse as we move in the central part
Nagarjuna Sagar – Andhra Pradesh of Brazil because: (Maharshtra 2019)
Jaldapara - Kerala (1) This area has favourable climate and an ideal for human
(3) Nagarjuna Sagar – Telangana settlements.
Bharathpura – Rajasthan (2) Area is covered by thick dense equatorial rainforests.
Manas – Assam (3) Area has good transportation system.
Dandeli – Karnataka (4) Fertile soil (rich soil) has been found in this area.
Solved Questions 103
B-

84. Which of the following is not the tributary of Sindhu river? List-I List-II
(Maharshtra 2019) (Iron and Steel Industries) (State)
(1) Chenab (2) Satluj (A) Durgapur (i) Jharkhand
(3) Betva (4) Ravi (B) Rourkela (ii) Chattisgarh
85. Observe the outline map of Brazil and identify the forest (3) Bhilai (iii) Orissa
type shown by shaded part. (Maharshtra 2019) (D) Bokaro (iv) West Bengal
Codes:
A B C D
(1) (iv) (iii) (ii) (i)
(2) (iv) (iii) (i) (ii)
(3) (i) (ii) (iii) (iv)
(4) (ii) (i) (iii) (iv)
96. “New Mangalore” seaport is located in which state of
India? (Rajasthan 2019)
(1) Karnataka (2) Tamil Nadu
(3) West Bengal (4) Maharashtra
97. Which of the following rivers falls in the Arabian Sea?
(Rajasthan 2019)
(1) Tapti (2) Krishna
(1) Swampy lands (2) Thorny shrubs (3) Kaveri (4) Mahanadi
(3) Equatorial forests (4) Hot Deciduous forests 98. Limestone is an example of (West Bengal 2019)
86. Identify the tributary of river Sindhu which originates near (1) Igneous rock
Mansarovar and flows west-ward: (Maharshtra 2019) (2) Sedimentary rock
(1) Jehlum (2) Ravi (3) Metamorphic rock
(3) Chenab (4) Satluj (4) None of the above
87. Which of the following option indicates sparse density of 99. Which of the following is not a right bank tributary of the
population distribution? (Maharshtra 2019) Ganga river ? (West Bengal 2019)
(1) Mountainous hilly regions – dry desert – dense forests (1) Yamuna (2) Son
(2) Hilly region – dense forests – industries (3) Damodar (4) Gomti
(3) Dry desert – plain lands – fertile lands 100. Crops grown during April, May and June are known as
(4) Availability of water – mountainous regions – plain lands (West Bengal 2019)
88. Which one is not the Fold Mountain?(Maharshtra 2019) (1) Zayed crops (2) Kharif crops
(1) The Himalayas (2) The Black Forest (3) Rabi crops (4) Spring crops
(3) The Rockies (4) The Aravalis 101. Which mineral has excellent dielectric strength, insulating
89. Identify the state of the Brazil which does not has coastline, properties, low power loss factor and resistance to high
(Maharshtra 2019) voltage? (Delhi 2018)
(1) Rio de Janeiro (2) Sao Paulo (1) Aluminium (2) Lime stone
(3) Goias (4) Bahia (3) Copper (4) Mica
90. Identify the correct option which shows right order of 102. Which of the following lake lies on the Equator?
neighbouring countries lies from south to north direction. (Delhi 2018)
(Maharshtra 2019) (1) Lake Victoria (2) Lake Malavi
A. Argentina B. Peru (3) Lake Nasser (4) None of these
C. Uruguay D. Bolivia 103. The longitudinal valleys lying between lesser Himalayas
(1) C, A, D, B (2) B, A, D, C and Shivaliks are known as (Delhi 2018)
(3) D, C, B, A (4) A, B, D, C (1) Valleys (2) Coast
91. Identify the bird from the Brazil which is huge in size and (3) Passes (4) Duns
fly high in the sky? (Maharshtra 2019) 104. In winters, the western cyclonic disturbances originate from
(1) Condor (2) Macaws which sea? (Delhi 2018)
(3) Piranhas (4) Puma (1) Caspian sea (2) Black sea
92. Which place in western Rajasthan is driest part of India? (3) Mediterranean sea (4) Baltic sea
(Maharshtra 2019) 105. Balancing the need to use resources and also conserve them
(1) Mounsinram (2) Cherapunji for future is called (Delhi 2018)
(3) Jodhpur (4) Jaisalmer (1) Resource development
93. Rabi crop is (Rajasthan 2019) (2) Resource conservation
(1) Rice (2) Gram (3) Sustainable development
(3) Maize (4) Soyabean (4) Human Resource Development
94. Which one of the following is the copper mine situated in 106. Consider the following statements: (Andhra Pradesh 2018)
Rajasthan? (Rajasthan 2019) A. lgnesous rocks are responsible for the formation of
(1) Morija-Banol (2) Degana-Bhakri black soil.
(3) Zawar (4) Khetri-Singhana B. Terai is a narrow belt of pebbles.
95. Match List-I with List-II and select the correct answer using C. The newer alluvial deposits of the northern plains are
the codes given below: (Rajasthan 2019) called khadar.
EBD_7332
104
B- NTSE Stage 1 Question Bank

Which of the above statements are correct? (3) Low pressure develops in the northern part of the
(1) A, B and C (2) A and B country
(3) B and C (4) A and C (4) Clear skies and rise in temperature
107. Consider the following countries : (Andhra Pradesh 2018) 119. Which one of the following minerals is formed by the
A. USA B. Egypt decomposition of rocks leaving a residual mass of weathered
C. Brazil D. Mongolia material? (Goa 2018)
E. Canada F. Uzbekistan (1) Coal (2) Bauxite
Which of the above countries are smaller than India with (3) Gold (4) Zinc
respected to area? 120. Which of the following is a new arrival on the transportational
(1) C and D only (2) A, B and F only map of India? (Goa 2018)
(3) B, D and F only (4) C and F only (1) Pipeline tansportation network
108. The Godavari is known as the ‘Dakshin Ganga’ because : (2) Waterways transportation network
(Andhra Pradesh 2018)
(3) Railways transportation network
(1) of its origin in Western Ghats
(4) Airlines transportation network
(2) of its making of waterfalls
(3) of its drainage into Bay of Bengal 121. Which of the following ports is an inland riverine tidal port?
(4) of its length and the area it covers. (Goa 2018)
109. Out of the following states, which one receives the South- (1) Kolkata (2) Vishakhapatnam
West monsoon lately? (Andhra Pradesh 2018) (3) Kandla (4) Kochi
(1) Karnataka (2) Kerala 122. Match the column ‘A’ with Column ‘B’ and choose the
(3) Maharashtra (4) Gujarat correct answer (Karnataka 2018)
110. Per capita consumption of which energy source is considered Column -A Column-B
as an index of development ? (Andhra Pradesh 2018) a. Yamunotri i. Valley
(1) Petroleum (2) Electricity b. Armakonda ii. Hill station
(3) Natural gas (4) Solar energy c. Kulu iii. Glacier
111. Arrange these hills from west to east. (Chandigarh 2018) d. Ranikhet iv. Peak
A. Khasi B. Garo v. Ground water
C. Naga D. Jaintia (1) a-iii, b-iv, c-i, d-v (2) a-ii, b-iv, c-i, d-iii
(1) C, A, B, D (2) D, B, A, C (3) a-iv, b-iii, c-ii, d-i (4) a-iii, b-iv, c-i, d-ii
(3) A, B, C, D (4) B, A, D, C 123. The Himalayan yew species is in danger because
112. Which of the following two extreme locations are connecting (Karnataka 2018)
the east west corridor? (Chandigarh 2018) (1) a chemical compound called ‘Taxol’ is extracted from
(1) Mumbai and Nagpur (2) Ahmedabad and Kolkata the bark to cure cancer.
(3) Silchar and Porbandar (4) Nagpur and Siliguri (2) the hide of the animal is extracted for producing
113. Which of the following statements is true? percussive instruments.
(Chandigarh 2018) (3) the birds feathers are colourful and they are collected
(1) El Nino is a Greek word meaning the child. by killing.
(2) Presence of the El Nino leads to decrease in Sea-Surface (4) the insecticides have brought adverse effects on them.
temperatures. 124. Read the following statements and write the correct option
(3) EI Nino is a name given to the periodic development of with which all those links. (Karnataka 2018)
warm ocean current along the coast of Peru.
A. This soil is suitable for cultivation of coffee with
(4) I TCZ is a broad trough of high pressure in equatorial
latitudes. adequate doses of manures and fertilizers
114. What causes rainfall on the coastal area of Tamil Nadu in B. This soil is mainly found in Karnataka, Kerala,
the beginning of winters? (Chandigarh 2018) Tamilnadu, Madhya Pradesh and the hilly areas of
(1) South West monsoon (2) Temperature cyclones Odisha and Assam.
(3) North East monsoon (4) Local Air circulation C. Humus content in this soil is low.
115. The Red soil develop a reddish colour due to D. It is found in areas with high temperature and heavy
(Chandigarh 2018) rainfall?
(1) deforestation and over grazing (1) Alluvial soil (2) Black soil
(2) the presence of potash and magnesium (3) Red soil (4) Laterite soil
(3) diffusion of iron in crystalline and metamorphic rocks 125. Choose the correct type of soil with reference to the shaded
(4) formation from the lave flows area in the given map. (Karnataka 2018)
116. A ‘mushroom rock’ is a landform caused by the action of
_____ (Goa 2018)
(1) sea waves (2) moving ice
(3) wind erosion (4) running water
117. Hanging valleys are carved out by the action of (Goa 2018)
(1) rivers (2) glaciers
(3) wind (4) ocean Waters
118. Which one of the following is the characteristic of cold
weather season? (Goa 2018)
(1) North East trade winds prevail over the country
(2) South West trade winds prevail over the country (1) Mountain Soils (2) Black Soils
(3) Laterite Soils (4) Red and Yellow soils
Solved Questions B-105
126. In our routine life, about how many types of minerals are (3) the vast northern plain
used directly or indirectly? (Gujarat 2018) (4) the peninsular plateau.
(1) 100 types (2) 200 types 137. The rising place of the largest river of peninsular plateau is
(3) 300 types (4) 400 types (Rajasthan 2017)
127. Other than current fallow land is known as land (1) Betul (2) Nasik
(Haryana 2018) (3) Jabalpur (4) Cuddalore
(1) Left without cultivation for one or less than one 138. The quantity of rainfall received on the Western Ghats by
agricultural year south-west monsoon is (Rajasthan 2017)
(2) Left uncultivated for the past 1 to 5 agricultural year. (1) 100 - 150 cm (2) 150 - 200 cm
(3) Area so more than once in an agricultural year. (3) 200 - 250 cm (4) above 250 cm'
(4) None of the above 139. In which Indian forest are silver, fir and pine trees found ?
128. In India this primitive form of cultivation is called by different (1) Tropical deciduous forest (Rajasthan 2017)
names. Select the correct answer using the code given (2) Montane forest
below- (Haryana 2018) (3) Mangrove forest
A. Madhya Pradesh (i) Pama Dabi or Koman (4) Tropical evergreen rain forest.
B. Odisha (ii) Bewar of Dahiya 140. Match List - I and List II and choose the correct code from
C. In Western Ghats (iii) Jhumming the following: (Rajasthan 2017)
D. North-East region (iv) Kumari List - I List - II
(1) A-ii, B-i, C-iv, D-iii (2) A-i, B-ii, C-iii, D-iv (A) Northern end (i) 8° 4' N
(3) A-ii, B-iii, C-iv, D-i (4) A-iii, B-ii, C-i, D-iv (B) Southern end (ii) 37° 6' N
129. Certain minerals may occur as alluvial deposits in sands of (C) Eastern end (iii) 68° 7' N
valley floors and the base of hills. By which name these are (D) Western end (iv) 97° 25' N
known? (Haryana 2018) A B C D
(1) ii iii iv i
(1) Placer deposits (2) Manganese nodules
(2) i ii iv iii
(3) Bromine (4) Malleable
(3) ii i iv iii
130. Which one of the following statement is not true?
(4) iii ii i iv
(Haryana 2018)
141. Which of the following is the major sugarcane producing
(1) Coal that has been burried deep and subjected to state ? (Rajasthan 2017)
increase temperatures is bituminous coal. (1) Uttar Pradesh (2) Rajasthan
(2) Large reserves of natural gas have been discovered in (3) West Bengal (4) Madhya Pradesh
the krishana Godavri basin. 142. Important deposits of which mineral are found in Koraput
(3) The monazite sands of Tamil Nadu is also rich in Thorium. in Odisha ? (Rajasthan 2017)
(4) Photovoltaic technology converts sunlight directly into (1) Iron ore (2) Coal
electricity. (3) Copper (4) Bauxite
131. The following waterways have been declared as the National 143. In which year was the first successful cotton textile mill
waterways by the Govt. Select the Correct answer using established in India ? (Rajasthan 2017)
the code given below- (Haryana 2018) (1) 1853 (2) 1854
A. Allahabad and Haldia (i) National Waterways No. 3 (3) 1855 (4) 1856
B. Kottapuram-Kollam (ii) N.W. No. –4 144. Indian population policy 2000 not includes (Rajasthan 2017)
C. Kakinada Puducherry (iii) N.W. No. –2 (1) free education
stretch of canals (2) free from diseases
D. Sadiya and Dhubri (iv) N.W. No. 1 (3) reducing infant mortality rate below 30
waterways (4) increase the employment opportunities.
(1) A-iv, B-i, C-ii, D-iii (2) A-i, B-ii, C-iii, D-iv 145. Gas transportation pipeline which passes through Kota in
(3) A-ii, B-iii, C-iv, D-i (4) A-iii, B-iv, C-ii, D-i Rajasthan is (Rajasthan 2017)
132. ‘Rainbow revolution’ is related to (West Bengal 2018) (1) Guwahati - Barauni - Allahabad - Kanpur
(1) New Agricultural Policy (2) Barauni - Rajbandh - Haldia
(2) Egg Production (3) Hazira - Vijaipur - Jagdishpur
(3) Artificial Rain Making (4) Salaya - Viramgam - Mathura - Delhi.
(4) Non-conventional energy 146. In which state of India is red and yellow soil found ?
133. Which crop takes almost a year to grow? (Delhi 2017) (Rajasthan 2017)
(1) Cotton (2) Jute (1) Chhattisgarh (2) Rajasthan
(3) Rice (4) Sugarcane (3) Jammu and Kashmir (4) None of these.
134. The coriolis force is caused due is (Delhi 2017) 147. The example of capital is (Rajasthan 2017)
(1) Wind movement (2) Earth rotation (1) Water (2) Forest
(3) Cyclonic depression (4) Jet stream (3) Climate (4) Machine
135. S.T.P. is the abbreviation of (Delhi 2017) 148. The rabi crop is (Rajasthan 2017)
(1) System Tech Park (2) Software Technology Park (1) Jowar (2) Bajra (Millet)
(3) State Thermal Plant (4) Software Tech Picket (3) Maize (4) Wheat.
136. 'Khadar' is found in (Rajasthan 2017) 149.Augite metamorphosed to (West-Bengal 2017)
(1) the northem mountain region (1) Horn blande (2) Pyroclastic
(2) Thar desert (3) Brecia (4) Pegmatite
EBD_7332
106
B- NTSE Stage 1 Question Bank

150.‘Busket of Egg topography’ is a common feature of b. Sundarbans II. West Bengal


(West-Bengal 2017) c. Gir National park III. Gujarat
(1) River deposition (2) Wind deposition d. Tandova National IV. Maharashtra
(3) Glacial erosion (4) Glacial deposition Park
151.Widest waterfall of world is (West-Bengal 2017) a b c d
(1) Khone waterfall of Laos (2) Salto Angel of Veneguela (1) I II III IV
(3) Niagra of U.S.A (4) Stanley waterfall of Congo (2) I III II IV
152.‘Cyclone’ or ‘Anti-cyclone’ is a (West-Bengal 2017) (3) I IV III II
(1) Trade wind (2) Periodical wind (4) IV II I III
(3) Sudden wind (4) Local wind 164. Assertion (A): Increase in population, urbanisation,
153. Benguela Current flows along the coast of Industrialisation lead to the increased use of fossil fuels.
(West-Bengal 2017) Reason (R): Over use of conventional energy resources
(1) California (2) South-West Africa has resulted in the phenomenon of green house effect.
(3) Peru (4) East Greenland Select the correct option from the given alternatives.
154.Coromandel coastal plain is located at (West-Bengal 2017) (Karnataka 2017)
(1) Kerala state (2) Karnataka state (1) A is true, R is false
(3) Tamilnadu state (4) Maharashtra state (2) A is false, R is true
155.Among these regions _____is under Tropical Monsoon (3) Both A and R are true, but R is not correct explanation
climate. (West-Bengal 2017) of A
(1) Canada (2) India (4) Both A and R are true and R is the correct explanation
(3) Guinea (4) Argentina of A
156.UNESCO has registered Sundarban as ‘World Heritage Site’ 165. Choose the group of statements, which are correct with
in the year (West-Bengal 2017) respect to "Golden Quadrilateral and Corridor Project".
(Karnataka 2017)
(1) 1978 (2) 1979
a. This project covers National Highways with the length
(3) 1986 (4) 1987
of 15,000 kms.
157.Which state of India ranks first as per hectare production?
b. The project was started in the year 2001
(West-Bengal 2017)
c. Total cost of the proposed project was Rs. 54,000 crores
(1) Punjab (2) West Bengal d. This is the largest project taken up and no country in
(3) Uttar Pradesh (4) Andhra Pradesh the world has taken up such project
158.‘White Revolution’ is related with (West-Bengal 2017) (1) a, b and c
(1) Milk production (2) b, c and d
(2) Paper production (3) a, b, c and d
(3) Egg production (4) a, c and d
(4) Non-Conventional energy sources 166. Assertion (A): Majority of paper industries are found in
159.In India the Metro Rail starts for the first time in West Bengal. (Karnataka 2017)
(West-Bengal 2017) Reason (R): News print paper is being imported from
(1) Delhi (2) Mumbai Norway, Sweden, Canada and the U.S.A.
(3) Kolkata (4) Bengaluru Select correct option from the given alternatives.
160.Which of the following satellites are launched from India? (1) A true, R false
(West-Bengal 2017) (2) A false, R true
(1) LANDSAT (2) SPOT (3) Both A and R are true, but 'R' is not the correct
(3) GOMs (4) IRS explanation of 'A'
161. The correct decreasing order of the neighbouring countries (4) Both 'A' and 'R' are true, but 'R' is the correct explanation
based on the length of the border that India shares is of 'A'
(Karnataka 2017) 167. The total population of an area divided by its geographical
(1) China, Bangladesh, Nepal, Pakistan area gives (Karnataka 2017)
(2) Bangladesh, Pakistan, China, Nepal (1) Population index
(3) China, Bangladesh, Pakistan, Nepal (2) Distribution of population
(4) Bangladesh, China, Pakistan, Nepal (3) Population Density
162. Choose the right arrangement of mountain ranges of India (4) Population growth rate
from South to North. (Karnataka 2017) 168. Match Column 'A' with Column 'B' and choose correct answer.
(1) Western Ghats, Vindhya Ranges, Satpura Hills, Aravali Hills (Karnataka 2017)
(2) Western Ghats, Aravali Hills, Satpura Hills, Vindhya Column-A Column-B
Ranges a. Rawat Bata I. Thermal Electricity
(3) Western Ghats, Satpura Hills, Vindhya Ranges, Aravali Hills b. Barauni II. Wind Energy
(4) Vindhya Ranges, Western Ghats, Aravali Hills, Satpura c. Nagarcoil III. Automic Energy
Hills d. Baramar IV. Solar Energy
163. Match Column 'A' with Column 'B' and choose correct a b c d
answer. (Karnataka 2017) (1) III I II IV
Column-A Column-B (2) I III II IV
a. Kaziranga National I. Assam (3) III I IV II
Park (4) IV III II I
Solved Questions 107
B-

169. The main cause for Land degradation in Punjab, Haryana 177. Match list A with B and select the correct answer using the
and Western part of Uttar Pradesh (Karnataka 2017) codes given below the list. (Andhra Pradesh 2017)
(1) Intensive Agriculture (2) Over Irrigation List - A List - B
(3) Deforestation (4) Mining
(a) Hyderabad is warmer (i) Altitude
170. Choose the group of correct answer with respect to the state-
than Mumbai. (ii) Mango
ments about South West Monsoon (Karnataka 2017)
a. Trade winds transform as South West Monsoon after (b) Snowfall in showers.
they cross Equator Himalayas. (iii) Distance from
b. These enter India in two branches (c) North western plain sea.
c. Most of the country's region receive rain during South gets rainfall in winter. (iv) Western
West Monsoon except Tamil Nadu (d) Rainfall in summer. Depression
d. The western region of the Western Ghats is called a b c d
rain-shadow region (1) iii ii iv i
(1) b and c only (2) a, b and c (2) ii i ii iv
(3) b, c and d (4) a, b and d (3) iii i iv ii
171. Match the nations and the respective names of cyclones (4) iv ii i iii
and choose correct answer. (Karnataka 2017) 178. Which one of the following bioreserves of India is not
A B
a. India I. Thane included in the world network of bio-reserve ?
b. Bangladesh II. Giri (Andhra Pradesh 2017)
c. Myanmar III. Mujan (1) Sunderbhan (2) Gulf of Mannar
d. Oman IV. Jal (3) Nanda Devi (4) Silent Valley
a b c d 179. Highest Annual Growth Rate in India was recorded in these
(1) IV II I III decades. (Andhra Pradesh 2017)
(2) II III I IV (1) 1981, 1971, 1991 (2) 1991, 2001, 1971
(3) II III IV I (3) 1971, 2001, 1991 (4) 1961, 1971, 1981
(4) IV III II I 180. Which of these is not related to Conservation of Resources?
172. Match the following and choose correct answer. (Andhra Pradesh 2017)
(Karnataka 2017) (1) The club of Rome advocated resources conservation
Animals/Birds Catagory of Existence
species for the first time in a more systematic way in 1968.
a. Indian Rhino I. Rare species (2) Brundtland Commission Report, 1987 introduced the
b. Hornbill II. Extinct species concept of “Sustainable Development”.
c. Asiatic cheetah III. Vulnerable species (3) E.F Schumacher is the author of the book “Small is
d. Gangtic Dolphin IV. Endangered species Beautiful”.
a b c d (4) Earth Summit was held in New York in 1997.
(1) IV II I III 181. With reference to Indian agriculture, which of the following
(2) IV I II III statements is not correct?(Andhra Pradesh 2017)
(3) III II IV I (1) India is the largest producer as well as the consumer
(4) III II I IV of pulses in the world.
173. Arrange the area covered by types of soil in India, in in-
(2) India is the second largest producer of rice in the world
creasing order (Karnataka 2017)
(1) Laterite soil, Red soil, Black soil, Alluvial soil after China.
(2) Laterite soil, Black soil, Red soil, Alluvial Soil (3) Tea is an important beverage crop introduced in India
(3) Alluvial soil, Black soil, Red soil, Laterite soil initially by the Persians.
(4) Alluvial soil, Red soil, Black soil, Laterite soil. (4) Groundnut is a kharif crop and accounts for about half
174. Sikkim, West Bengal, Assam and Arunachal Pradesh have of the major oil seeds produced in the country.
common frontiers with (Andhra Pradesh 2017) 182. In which of these following industries, limestone is not
(1) China (2) Bhutan used? (Andhra Pradesh 2017)
(3) Bangladesh (4) Mayanmar (1) Cement industry. (2) Iron and Steel industry.
175. Which of these is hot a Himachal Range ? (3) Oil Refinery industry. (4) None of the above.
(Andhra Pradesh 2017) 183. Find the wrongly matched. (Andhra Pradesh 2017)
(1) Dhaula Dhar (2) Pirpanjal Range
(1) Ferrous mineral - Iron ore.
(3) Kailash Range (4) Mahabharat Range
176. The Himalayas is divided into four major Geological (2) Non-ferrous – Mica. mineral
sections. Choose among the following which is not one of (3) Non-Metallic - Limestone. mineral
them. (Andhra Pradesh 2017) (4) Fuel minerals - Coal.
(1) Nepal Himalayas – Between Kali and Teesta. 184. Identify the non - fibre crop? (Andhra Pradesh 2017)
(2) Mahabharat Himalayas - Between Indus and Gilgit. (1) Hemp (2) Cotton
(3) Kumaon Himalayas - Between Sutlej and Teesta. (3) Natural Silk (4) Rubber
(4) Assam Himalayas – Between Teesta and Dihang.
EBD_7332
108
B- NTSE Stage 1 Question Bank

185. The South-east Trade winds are attracted towards the 199. Nanda Devi bio-reserve is located in- (Chhattisgarh 2015)
Indian sub continent in the month of June due to … (1) Uttaranchal (2) Delhi
(Andhra Pradesh 2017) (3) Punjab (4) Orissa
(1) the effect of the westerlies. 200. Rainfall occurs during winter in North-Western part of India
(2) the effect of Somaliya current. due to- (Chhattisgarh 2015)
(1) Cyclonic Depression (2) Western
(3) the presence of low atmospheric pressure over North-
disturbance
west India. (3) Retreating Monsoon (4) South west
(4) None of the above. Monsoon
186. In which of the following states Koyana multipurpose project 201. Which continent of today is not a part of Gondwana land:
is set up? (Haryana 2016) (Maharashtra 2014)
(1) Karnataka (2) Maharashtra (1) North America (2) South America
(3) Andhra Pradesh (4) Tamil Nadu (3) Africa (4) Australia
187. In which country the ‘slash and burn’ agriculture is known 202. Which of the following rivers of India flow through a rift
as ‘Roca’? (Haryana 2016) valley? (Maharashtra 2014)
(1) Mexico (2) Vietnam (1) Tapi River (2) Ganga River
(3) Brazil (4) Indonesia (3) Mahanadi River (4) Yamuna River
188. Which is the first expressway of India (Jharkhand 2015) 203. Rainfall in the Northern and North-Western parts of India in
(1) Delhi – Kolkata (2) Mumbai – Pune the month of October-November is caused mainly due to
(3) Pune – Chennai (4) Delhi – Mumbai (Maharashtra 2014)
189. Capital of Lakshdweep is (Jharkhand 2015) (1) Western disturbances (2) Jet streams
(1) Kavaratti (2) Daman (3) Advancing Monsoon (4) Upper Air Circulation
(3) Silvassa (4) Port Blair 204. The slash and burn agriculture is known as Milpa in which
190. Which mine is found in Khetri ? (Jharkhand 2015) of the following countries? (Maharashtra 2014)
(1) Copper Mines (2) Lignite Mines (1) Indonesia (2) Mexico
(3) Bauxite Mines (4) Iron Ore Mines (2) Vietnam (3) Sudan
191. What is the longitudinal extent of India ? 205. Which of the following is a non-renewable as well as non-
(1) 67°10' East to 96°27' East (Jharkhand 2015) recyclable resources? (Maharashtra 2014)
(2) 67°05' East to 96°22' East (1) Iron (2) Coal
(3) 69°05' East to 98°22' East (3) Water (4) Forests
(4) 68°08' East to 97°25' East 206. Which of the following is not a nuclear power station?
192. Which of the following is the youngest structure of India ? (Maharashtra 2014)
(Jharkhand 2015) (1) Kaiga (2) Narora
(1) Great Himalayas (2) Shiwalik Range (3) Korba (4) Kakrapara
(3) Middle Himalayas (4) Peninsular India 207. Which of the following is the working age group of the
193. Rusting of rock is caused by the process of population? (Maharashtra 2014)
(West Bengal 2015) (1) 15-59 years (2) 20-60 years
(1) Carbonation (2) Hydration (3) 20-65 years (4) 18-60 years
(3) Oxidation (4) Solution 208. According to Census of India 2001 a literate person is the
194. Roches Montanees are the hillocks of rocks. that are formed one who: (Maharashtra 2014)
by (West Bengal 2015) (1) can read and write his/her name
(1) Glacial abrasion (2) can read and write any language
(2) Glacial abrasion and Plucking (3) is 7 years old and can read and write any language with
(3) Glacial deposition understanding
(4) None of the above (4) knows 3 R’s (reading, writing, arithmetic)
195. Canaries current flows along the coast of 209. National Waterways No. 1 of India connects which two cit-
(West Bengal 2015) ies of India? (Maharashtra 2014)
(1) Portugal (2) Peru (1) Sadia – Dubri (2) Allahabad – Haldia
(3) Japan (4) India (3) Kottapuram – Kollam (4) Delhi – Kanpur
196. In India the Tropic of Cancer touches how many states? 210. Which state of India has made rooftop rain water harvesting
(West Bengal 2015) compulsory to all houses? (Maharashtra 2014)
(1) 6 states (2) 7 states (1) Andhra Pradesh (2) Tamil Nadu
(3) 8 states (4) 9 states (3) Kerala (4) Karnataka
197. Which of the following lakes has formed as a result of 211. Which one of the following groups, represents
tectonic activity? (Delhi 2015) neighbouring countries of India? (Karnataka 2014)
(1) Wular Lake (2) Chilika Lake (1) Afghanistan, Pakistan, Bangladesh, Srilanka
(3) Pulkit Lake (4) Kolleru Lake (2) Persia, Pakistan, Nepal, Maldives
198. What is the name of the Indian Research centre located in (3) Burma, Bhutan, Tazikistan, China
Antarctica? (Delhi 2015) (4) Malasia, Burma, Nepal, Srilanka
(1) Dakshin Gangotri (2) Shwet Baharti 212. Which of the following is/are the method/s of minimizing
(3) Him Putra (4) Shivalik Mani soil erosion? (Karnataka 2014)
Solved Questions 109
B-

A. Shifting cultivation 222. Which is the nearest planet of sun ?


B. Construction of check-dams (Madhya Pradesh 2013)
C. Contour ploughing (1) Venus (2) Jupiter
D. Afforestation (3) Mercury (4) Mars
(1) A only (2) B, C and D 223. Which country of Europe is called 'Playground of Europe' ?
(3) B and D (4) D only (Madhya Pradesh 2013)
213. Match the National parks indicated on the map of India, (I, II, (1) England (2) Holland
III and IV) with their respective names. (Karnataka 2014) (3) Switzerland (4) Belgium
224. Areawise what is the position of India in the world ?
(Madhya Pradesh 2013)
(1) Third (2) Fourth
IV (3) Sixth (4) Seventh
III 225. Rajsamand lake is in the Indian province of :
(Madhya Pradesh 2013)
II (1) Chhattisgarh (2) Jharkhand
(3) Rajasthan (4) Uttarakhand
226. What is “Barkan” ? (Rajasthan 2013)
I (1) Name of Sand dune (2) Name of Village
(3) Name of Tree (4) Name of Mountain
227. Which state coast line is called the Malabar?
A. Gir B. Nagarhole (Rajasthan 2013)
C. Jim Corbett D. Kaziranga (1) Gujrat (2) Kerala
(1) I – C II – D III – A IV – B (3) Rajasthan (4) West Bengal
(2) I – A II – B III – C IV – D 228. The ore of iron is (Rajasthan 2013)
(3) I – B II – A III – D IV – C (1) Haematite (2) Uranium
(4) I – D II – B III – C IV – A (3) Bauxite (4) Lignite
214. The slash and bum agriculture is known as Milpa in Which 229. Bhakra-Nangal Project is situated on the River?
of the following countries? (Delhi 2014) (Rajasthan 2013)
(1) Indonesia (2) Mexico (1) Satluj (2) Tungbhadra
(3) Vietnam (4) Sudan (3) Damodar (4) Mahi
215. Which of the following is a non-renewable as well as non- 230. Salty Water lake is : (Rajasthan 2013)
recyblable resources? (Delhi 2014) (1) Jaisamand lake (2) Rajsamand lake
(1) Iron (2) Coal (3) Didwana lake (4) Gapsagar lake
(3) Water (4) Forests 231. Which one of the following is a leguminous crop?
216. Why is there scanty rainfall in the Deccan plateau? (1) Jowar (2) Pulses (Delhi 2012)
(1) It is far away from the sea (Chandigarh 2014) (3) Millets (4) Sesamum
(2) It is near the sea 232. Which of the following rivers flows through a rift valley?
(3) It is in the rain shown region (1) Mahanadi (2) Tungabhadra (Delhi 2012)
(4) None of these (3) Krishna (4) Narmada
217. Flood occur frequently in the northern plains of India be- 233. Which one of the following is a non-metallic mineral?
(1) Lead (2) Copper (Delhi 2012)
cause of: (Chandigarh 2014)
(3) Tin (4) Limestone
(1) Long spells of rainy weather
234. Which of the following is an Inland Riverine Port?
(2) the presence of many large river courses
(1) Kandla (2) Kolkata (Delhi 2012)
(3) Fluctuation of the level of the understand water table (3) Mumbai (4) Tuticorin
(4) Uncertain and uneven occurrence of rains in the plains 235. Which one of the following causes rainfall during winter in
218. Which type of drainage pattern is formed, when river and the north western part of India? (Punjab 2012)
its tributaries resemble the branches of a tree. (Delhi 2013) (1) Cyclonic depression (2) Western disturbances
(1) Dendritic (2) Radial (3) Retreating monsoon (4) South west monsoon
(3) Trellis (4) Rectangular 236. On which of the following rivers Sardar Sarovar Dam is built?
219. Which one of the following types of vegetation does ‘rub- (1) Kaveri (2) Krishna (Punjab 2012)
ber ’ belong to (Delhi 2013) (3) Narmada (4) Satluj
(1) Tundra (2) Tidal 237. Majuli, the largest inhabited riverine island is found in the
(3) Himalayan (4) Tropical Evergreen ____________ river. (Punjab 2012)
1 (1) Ganga (2) Brahmaputra
220. Tropic of cancer 23 N does not pass through which (3) Satluj (4) Yamuna
2
238. El Nino are the ____________ . (Punjab 2012)
state of India (Delhi 2013) (1) cold ocean current (2) warm ocean current
(1) Rajasthan (2) Chattisgarh (3) trade winds (4) north east winds
(3) Odisha (4) Tripura 239. In which continent, there is no active volcano ?
221. Who is regarded chief of solar system ? (Madhya Pradesh 2012)
(Madhya Pradesh 2013) (1) Asia (2) Africa
(1) Sun (2) Moon (3) Europe (4) Australia
(3) Earth (4) Sky
EBD_7332
110
B- NTSE Stage 1 Question Bank

240. Which State of India does not have common boundary with 252. The largest producer of Sulphur in the World
Myanmar ? (Madhya Pradesh 2012) (Andhra Pradesh 2012)
(1) Arunachal Pradesh (2) Tripura (1) Indonesia (2) Mexico
(3) Nagaland (4) Manipur (3) Malaysia (4) Brazil
241. Which State has Satpuda hills? (Madhya Pradesh 2012) 253. In Indo-Gangetic plains, the older alluvium of flood plain
(1) Uttar Pradesh (2) Bihar is called as …… (Andhra Pradesh 2012)
(3) Andhra Pradesh (4) Madhya Pradesh (1) Bhanger (2) Khadar
242. When was “Indian wildlife Protection Act” implemented? (3) Terai (4) Babar
(1) 1970 (2) 1972 (Rajasthan 2012) 254. The non-metallic mineral among the following
(3) 1974 (4) 1976 (Andhra Pradesh 2012)
243. Of which river the Chambal is a tributary? (Odisha 2012) (1) Graphite (2) Chromite
(1) The Narmada (2) The Yamuna (3) Bauxite (4) Tungston
(3) The Godavari (4) The Tapti 255. In India which irrigation sources has the maximum area?
244. What type of forest is found in the region where the annual (Maharashtra 2012)
rainfall is more than 200 cms? (Odisha 2012) (1) Rivers (2) Dams
(1) Alpine Forest (2) Evergreen Forest (3) Wells and tubewells (4) Canals
(3) Deciduous Forest (4) Monsoon Forest 256. Which station is not situated on the Mumbai-Delhi railroute
245. Which of the National Highways of India connects Kolkata on western railway? (Maharashtra 2012)
with Chennai? (Odisha 2012) (1) Jhansi (2) Surat
(1) No.5 (2) No.6 (3) Vadodara (4) Ratlam
(3) No.23 (4) No.42 257. Backwater inland waterway is in : (Maharashtra 2012)
246. From which of the ores Thorium is extracted? (1) Gujarat (2) Kerala
(1) Ilmenite (2) Haematite (Odisha 2012) (3) Maharashtra (4) West Bengal
(3) Monazite (4) Dolomite 258. Which cotton textile centre is not in Tamil Nadu?
247. The earth’s crust is mainly composed of (West Bengal 2012) (Maharashtra 2012)
(1) granite and conglomerate rocks (1) Tutikorin (2) Coimbatore
(2) mudstone and sandstone rocks (3) Salem (4) Trichur
(3) gneiss and marble rocks 259. Which of the following statements represent characteristics
(4) granite and basalt rocks of black soil? (Karnataka 2012)
248. The process of removal of rocks on the earth surface is A. It develops cracks during hot weather.
called (West Bengal 2012) B. Iron content present in this soil turns into iron oxide
(1) gradation (2) denudation due to diffusion.
(3) weathering (4) erosion C. It’s made of fine clay particles.
249. ‘Eutrofication’ is related to (West Bengal 2012) D. It retains moisture for long.
(1) water pollution (2) air pollution (1) A and C only (2) A, B and C only
(3) land pollution (4) cultural pollution (3) C and D only (4) A, C and D only
250. Cold wall is (West Bengal 2012) 260. The following is the list of iron and steel plants located in
(1) a line between warm cold ocean current various parts of India. (Karnataka 2012)
(2) a two dimensional plane between cold and warm ocean A. Vishakhapatnam B. Bhadravati
current C. Jamshedpur D. Salem
(3) a three dimensional plane between warm and cold ocean E. Rourkela F. Bokaro
current G. Vijayanagar H. Durgapur
(4) multi dimensional plane between warm and cold ocean Which one of the following represents iron & steel industries
current located outside the Chotanagpur region?
251. The ‘Monsoons’ provide the best example for (1) A. D. G. H (2) B. C. F. G
(Andhra Pradesh 2012) (3) A. B. D. G (4) B. E. G. H
(1) The trade winds (2) The westerlies 261.Massai Mara national park is located in- (Rajasthan 2012)
(3) The seasonal winds (4) The local winds (1) India (2) Pakistan
(3) Sudan (4) Kenya
HINTS & SOLUTIONS
SECTION 1. PHYSICS P1 2m1K1 2m1K1
1. (3) On hills or higher altitudes - lower atmospheric pressure P2 2m2 K 2 2m2 K 2
and thus lower boiling point, so food needs to be
cooked for a longer time. Hence cooking food is difficult 2g 1
on hills.
2. (3) When a plane mirror moves towards the stationary 4g 2
object with speed u, the image will move with speed 2u
in same direction as that of mirror. P2 : P1 2 :1
3. (2) From momentum, 13. (3) The quality or timbre of sound enables us to distinguish
between two sounds having same pitch and loudness.
P 2mk P2 2 mk
energy mgh
P 2 (3000) 2 14. (1) Power, P
Kinetic energy, k = 90,000 J time t
2m 2 50 m 30
4. (2) Power of the lens to be used, Given, 30 Kg/min Kg/s, h = 10 m and g is
t 60
1 1 9.8 m/s2
P 2 D.
f (in metre) 50
30
100 P 9.8 10 49 J/s.
5. (2) Power of the lens in spectacles is –2D i.e., –ve hence 60
person suffers from myopia or short-sightedness i.e., 5
can see near object clearly but cannot see far off objects R R
R R 6 5
clearly. 15. (1) R AB R || R
6. (2) Acceleration of the system, 3 2 5 11
R R
m2 m1 12 8 g 6
a g g
m1 m2 12 8 5 R 4
R
7. (3) In circular motion, due to change in direction - R R 2 3 4
R AC || R R
acceleration and velocity change but speed being 2 3 R 4 11
scalar does not depend on the direction hence it R
2 3
remains constant.
R 3
total distance R
8. (3) Average speed R R3 2 3
time taken RBC || R R
3 R 3
2 11
x R
3 2
x x Hence equivalent resistance between A and B is
v v
2 1 2 2
5
x x maximum i.e., R AB R.
11
x x 3x 2 x 16. (2) F Kx ma Kx
2 20 2 30 120
Kx 15 20 10 2
x 120 a 10 m/s 2
24 m/s. m 0.3
5x 17. (4) In part AB acceleration is decreasing as line bending
9. 2
(4) Given : u = 20 m/s, a = 4 m/s , t = 2s towards time axis.
By using first equation of motion, 18. (2) To decelerate and bring at rest force F < 100 N.
V = u + a = 20 + 4 × 2 = 20 + 8 = 28 m/s
W
W 9.8 19. (2) From formula, V
10. (4) Weight, W = mg m 1 kg. q
g 9.8
11. (3) Kinetic energy due to its speed and potential energy W 18 18
q 0.2 C
due to its height or configuration. V 100 10 90
12. (2) Relation between momentum P, mass m and kinetic 20. (4) Magnetic field lines form closed loop. Electric field lines
energy K. never form closed loop.
Given : K1 K 2 , m1 2 g, m2 4g total distance travelled
21. (1) Average speed
P1 2m1 K1 P2 2m2 K 2 total time taken
EBD_7332
112
B- NTSE Stage 1 Question Bank

x 5v1v2 1 1 1
x /5 4x or,
v2 4v1 f 20
v1 5v2 20
f 10 cm
distance 2
thickness
30. (4) As the image is formed behind the retina, so person is
22. (2) Time taken suffering from hypermetropia or long sightedness. The
Vmed lens used to correct hypermetropia is convex lens.
31. (3) Power of combination of lenses,
t c
Vmed Peq P1 P2 P3 3.5 2.5 1 7D
c n
n 32. (2) Petrol is volatile liquid. So particles of petrol evaporate
by using energy from palm and causes cooling effect.
nt
Time taken
c
v
23. (3) Current, I 33. (2)
R
V 2.2 3 6.6
I max 0.33 A Atomic number of oxygen = 8
( Req ) 20 20
Electronic configuration of oxygen = 2, 6.
V 2.2 2.2 34. (4) Alcohol is organic compound which does not ionise in
I min 0.06 A water. Therefore alcohol does not conduct electricity.
Req R0 6.66 30 36.66 35. (4) When light ray travels from rarer to denser medium, it
bends towards the normal and vice-versa.
24. (1) Using v 2 u 2 2 gh we get VA VB VC 36. (3) Swayam is a 1-U Picosatellite placed in low earth orbit
(LEO) around the earth at a height of 515 Km.
v h1
25. (3) From magnification, m 37. (4) Earth revolves round the Sun due to gravitational force.
u h0 Tension provides, required centripetal force due to
which stoned tied to a string whirled in a circle. Electron
v 5 12 5 around the nucleus revolves due to nuclear force and
v 30 cm
12 2 2 due to electro-magnative force motion of blades of fan
1 1 1 1 1 1 occurs.
Now from,
v u f 30 ( 12) f 2GM E
38. (1) Escape velocity VE RE
11.2 Km / s
1 1 1 2 5
Given :
f 30 12 60
Mass of planet, M P 25M E (ME = Mass of earth)
60 Radius of planet, R P 125R E (RE = Radius of earth)
f 8.6 cm
7
2G 25M E 1
thrust VP VE
26. (2) Pressure , placing a round piece of cloth 125R E 5
area
increases the surface area, hence, decreasing the 39. (2) Velocity of light in sapphire,
pressure.
C 3 108
total distance travelled Vsap 1.67 108
27. (1) Average speed n 1.8
total time
C 3 108 velocity of light in vacuum of air
260 300 560
14 m/s % decrease in velocity of light
20 20 40
28. (4) Because of gravitational force which is the force 3 108 1.67 108
100
between any two masses, the moon is binded to earth, 3 108
we are binded to earth and tides are formed also
because of gravitational pull of moon. 1.33
100 45%.
29. (2) From the figure, u = –20 cm, v = 20 cm, f = ? 3
1 1 1 40. (4) Vector quantities have both magnitude as well as
Using lens formula, direction and obey law of vectors addition.
v u f 41. (4) An object is said to have uniform velocity, if magnitude
1 1 1 1 1 1 as well as direction of its velocity remains the same.
20 ( 20) f 20 20 f AC Force×displacement
42. (3)
B momentum
Solved Questions 113
B-

Change in momentum displacement 53. (3) is temperature coefficient of resistance and its S.I.
unit is /°C.
time momentum
F
displacement 54. (2) Pressure, P F = mg
= = Velocity. A
time Given,
43. (1) Mass is a quantity of matter contained in a body. It is F 40 10 = 400 N and
independent of acceleration due to gravity (g).
Gm1m2 A = 80 cm 2 = 80 10 4
m2
44. (3) From, F =
d2 F 400 4
P= = = 5 10 Pa
A 80 10 4
Gm1m2
F'= = 4F . [ d = halved] 55. (2) Given, emf v = 1.5 V, Charge, C = 0.5 C, energy provided
2
d by the cell, W = ?
÷ Using, W qv W 0.5 1.5 = 0.75 J
2
56. (1) Force in west direction,
45. (2) From v f
FW = 4N + 3N = 7N is cancelled by force equal but
250m/sv in opposite direction east, Fe = 7 N.
f = =
50M Force in south direction, FS = 4 N and in opposite
100 direction.
Frequency f = 500 Hz. Force FN = 5 N.
Hence, net force of 5 N – 4 N = 1 N in north direction.
46. (2) From, F = ma 57. (3) Magnetic lines of force start from North pole and end
10 at South pole.
m = 10g = kg, a = 10 m/s2
1000 distance travelled
58. (3) Velocity or speed =
10 time
Force, F 10 = 0.1N 59. (1) Far point is infinity and near point 25 cm for normal
1000
human eyes. The far point of the eye is the maximum
1 1 1 distance to which the eye can see the objects clearly.
47. (3) =
v u f 60. (2) Refractive index of water w = speed of light in air or
vacuum/speed of light in water
v = image distance
or, w = 3 × 108/2.25 × 108 = 4/3 = 1.33
u = object distance and
61. (3) Power of lens P = 1/focal length (f) of lens in metres
f = focal length of lens
Given, f = 20 cm = (20/100) metres = (1/5) metres
1 1 Power of lens = 1/(1/5) = +5D (+ve as convex lens)
48. (2) Power of lens, P = = =2D
f (in metre) 50 62. (2) A voltmeter is a high resistance and is always connected
100 in parallel in the circuit. It is used to measure potential
difference between any two points of the circuit.
R Ammeter is always connected in series in the circuit.
49. (3) F R = 2F
2 63. (3) 1 H.P. = 746 watt
50. (4) Angle of refraction for critical angle C = 90°. 64. (4) A magnet attracts magnetic materials like iron, cobalt,
Rarer medium nickel, etc.
65. (1) Mass m, linear momentum p and kinetic energy related
as p = 2mE
66. (4) From equation,
90°
ft
u = 0, t = 4s, a = g = 32
s2
C
1 2
using, s = ut + at
2
Denser medium 1 32 16
=0×4+ × 32 × (4)2 s= = 256 ft
2 2
V2 V2 67. (1) Volume, V = L3
51. (2) Power, P R=
R P
V 3 L
220 220 V
=
L
= = 4840
10
y
energy mgh 100 10 5 y = 3x x=
52. (1) Power = = = = 500 Watt 3
time t 10
EBD_7332
114
B- NTSE Stage 1 Question Bank

68. (1) Using, refractive index, GM G V G 4 3


g 2 2 2
R
1 1 1 R R R 3
sin i, sin 45 1/ 2 4
g G R
Refractive index of the medium with respect to air. 3
69. (4) Unit of heat and work is same i.e., joule 4
70. (3) Gamma ray ( ray) is an electromagnetic wave. g1 3 G r1 r1
71. (3) The minimum distance between a real object and its g2 4 r2
G r2
real image formed by convex lens is 4f i.e. four times of 3
focal length. 77. (4) We know, Lorentz Force
V 2
(200) 2
40000 F = q v B sin
72. (1) Power consumed, P = where is angle between the direction s of v and B.
R 50 50
= 800w = 0.8 kW Case-I, If = 0°
F=0
73. (1) The lens formula Case-II, If = 90°
1 1 1 F=qvB 0
f u v 78. (3) Momentum = mass velocity = kg-m/s
f = focal length 79. (4) In simple harmonic motion, total energy always
remains constant and it is equal to sum of potential
u = object distance energy and kinetic energy.
v = image distance 80. (3) The critical angle C is given by
1 –1 1
f 40 sinC =
g w
f = – 40 cm
100 100 – 5 sinC = w w
P= – g w
f 40 2 g g
P = –2.5D
4
74. (2) Gravitational force is required for convection only.
75. (1) I = 2 A 3
sinC =
3 3
R 2
8
6 C = sin–1
9
81. (2) Acceleration due to gravity at the surface of the earth
is
6V
GM
A g=
R2
Two resistance are in parallel, where, R = radius of earth
1 1 1 2 1 3 1 M = mass of earth
R1 3 6 6 6 2 G = Gravitational constant
The radius of the earth decreases from equator to the
R1 = 2 ...(i)
Pole. Hence, value of g is maximum at poles.
Voltage 82. (3) Biomass energy source, is any organic matter - wood,
Req =
Current crops, animal wastes that can be used as energy
6V source. As gobar gas is a mixture of gases produced
Req = 3 ...(ii) by break down of organic matter, so it is a biomass
2A energy source.
where unknown resistance R, from (i) and (ii) 83. (3) All the resistors are connected in parallel
R=3 –2
1 1 1 1 1 1
R=1
76. (1) We know, Two planets having density 1 and but R eq R R R R R
2
both are same, so R
R eq =
1 = 2 = 5
Solved Questions 115
B-

84. (3) For = 45° 88. (4) Given,


2 2 2 Power of bulb P, = 100 W
u sin 45° u 1
Height, H = = sin 45° = Voltage, V = 220 V
2g 4g 2 Power of bulb Q = 60 W
u 2 sin 2 u 2 sin 90 u2 Voltage = 110 V
Horizontal range, R =
g g g V2
Power, P =
R u2 4g R
= =4
H g u2 V2
Resistance of bulb, R
R = 4H P
85. (4) Time period of simple pendulum, Rp VP 2 PQ 12
RQ 2 PP 5
l VQ
T= 2
g eff 89. (4) When an object moves in a circle, it is constantly
changing its direction. Velocity is the speed in a
where, l = length of simple pendulum specified direction and here the direction of speed
geff = effective value of acceleration due to gravity changes continuously. Therefore, velocity of the
On earth’s surface time period particle vary even when its speed is constant.
90. (1) Given,
l Object distance, u = 30cm
T1 = 2
g When a lens is cut along the principle axis into two
Acceleration due to gravity at height h above the equal parts focal length remains same for each part.
surface of earth is Focal length, f = 20cm
2
Using lens formula
R
g' = g ÷ 1 1 1
R+h
f v u
At h = R
2 1 1 1 1
R g
g' = g ÷ = v 20 30 60
R+ R 4 v = 60 cm
Time period at height R, 91. (4) We know
l 4l 1
T2 = 2 2 Frequency (f)
g g wavelength( )
4 As, Red > Yellow > Green > Blue
fRed < fYellow < fGreen < fBlue
92. (3) In graph 3, the slope is increasing. So acceleration is
4l
2 increasing. Thus it represents non-uniform
T2 2 g acceleration.
= =
T1 l 1 93. (4) Distance of person from the wall, d = 55m
2
g Speed of sound, v = 330 m/s
86. (2) Time period of charged particle of mass m and charge d d
q moving in a circle under the influence of magnetic Echo time, t =
field B is given by v v
2d 2 55
2 m t=
Time period = v 330
qB
t = 0.33 s
Clearly time period is independent of R and v. 94. (4) 1 kWh is the quantity of electrical energy which is
87. (4) Given, dissipated in one hour in an electrical circuit when
Mass of the bullet, m = 40 g the electrical power in the circuit is 1 kW.
Speed of the bullet, v = 1200 ms–1 1 kWh = 1000 W × 3600 s
Let he can fire n bullets per second. 1 kWh = 3.6 × 106 J
Force = m × v × n 95. (1) Here two 4 are in parallel and in series with three 3
in parallel. So, the given circuit is equivalent to
40
144 = 1200 n
1000
n=3
EBD_7332
116
B- NTSE Stage 1 Question Bank

3 From 1st equation of motion,


4 v = u + gt
v2 = u2 + g × td
v2 = 0 + 10 × 4
3 = 40 ms–1
(From 2nd equation of motion)

4 v22
u22 160 0
h= = 80 m
3 2g 2 10
4 4 100. (2) Cone cells are responsible for colour: The second
R1 2 ...(1) type of photo receptor cell in the retina, cone cells
4 4 are responsible for the vision in bright light and
colours vision.
1 1 1 101. (3) From the equation
R2 1 ...(2)
3 3 3 u = mgh or
From (i) and (2) GMm
u=
R = R1 + R2 r
the units of gravitational potential energy must be,
=2 +1 kg. (m/s2) . m = (kg. m/s2) . m = N.m
102. (2) According to snell’s law,
R=3
c = speed of light in vaccum 3 108 m/s
96. (3) When the ball starts falling from rest i = angle of incidence
Initial velocity, u = 0 = 45°
1 2 r = angle of refractive
Using, h = ut + gt
2 sin i c
=
1 2 1 2 sin r v
=0 t+ gt = gt
2 2 sin r sin 30
v= c 3 108
2h sin i sin 45
t= it is independent of mass,
g 1
2 2 1.414
Hence all three will fall together. = 3 108 3 108 =3 108
1 2 2
97. (1) Given,
Power of bulb, P = 60W 2
Voltage, V = 220V = 3 0.707 108
Using, P = VI = 2.121 108 m/s
60 = 220 × I 103. (4) Given, m = 250 kg
g = 10 m/s2
60
I= for single bulb h = 150 m
220
Let n bulbs can be run from the main supply Work
Power =
Time
60
5= n m g h 250 10 50
220 =
n = 18 t 60 sec
1 375000
98. (4) Magnetic field, B =
r 60
rC > rA > rB
6250
Therefore, BB > BA > BC = = 8.37 HP
Stronger at B than A, weaker at C than A. 746
104. (2) Platinum is inert metal therefore occurs in free state.
99. (4) Let, ta and td be the time of ascent and time of descend
respectively, 105. (4) Given that, initial velocity of a particle, u = 10 m/s
acceleration, a = 4 m/s2
ta + td = 8
From motion 2nd Equation
and ta = td
2ta = 8 1 2
s = ut + at
ta = 4 = td 2
Solved Questions B- 117

1 115. (2) The primary reason why the colour red is used for traffic
s = 10 × 2 + × 4 × (2)2 signals is that red light is scattered the least by air
2 molecules. So, the red light is able to travel the longest
1 distance.
= 20 + ×4×4
2 v2 v1
1 116. (1) Average acceleration, at time t, angle 45° and
= 20 + × 16 t2 t1
2 at time 1s, angle 60° with time axis
s = 28m
106. (2) Speed of wave, v = 350 m/s v tan 60 tan 45
aar = 3 1
wavelength, = 100 cm t 1
=1 m 117. (2) Given,
F = 20 N
v 350
v= = 350 Hz m1 = 4kg
1
107. (2) Longitudinal Wave: The waves in which the individual m2 = 2kg
particles of the medium oscillate along the direction F
of wave propagation. ac =
m1 m2
108. (3) Focal length of a lens, F = 25 cm
f = 0.25 m 20 10 10 20
ac = m/s 2 F42 2 N
1 1 6 3 3 3
P= = 4D 118. (4) We know,
f 0.25
109. (2) Magnetic Flux, = BA Y = a diabatic constant
SI unit of flux is Wb (weber) R = gas constant
110. (1) A lactometer is used to check purity of milk. Milk is M = molecula mass of gas
poured in and allowed to stand until the cream deposit T = absolute temperature
in degrees determines the quality of milk. If the milk
sample is pure, the lactometer floats, if it is impure, the RT VO 2 1
Vsound =
M VH 32 4
lactometer sinks.
111. (3) Mass of the bike, m = 250 kg 119. (3) As we know that, Negative sign indicate decrease.
Speed of the circus man on the circular path, v = 20 m/s
V 1 T 1
Diameter of the circle, = 100 m V T 1 0.5%
V 2 T 2
Radius of the circle, r = 50 m
120. (3) Given that, incident angle of light = 60°
Centripetal acceleration is given by
According to Bhewster’s law, = tan ip
v 2 20 20 = tan 60°
a= = 8 m/s2
r 50 = 3
112. (3) The critical angle for the diamond-air interface is very 121. (3) Distance travelled in first 10 seconds
small, so when light enters a diamond, it will probably be
completely reflected internally. 1 2
x1 ut at
113. (1) Given, current through the heater, I = 5 A 2
Heat produced, H = 54000 J 1
x1 0 + a (10)2
Time, T = 6 min 2
= 6 60 = 360 sec x1 = 50 a ...(1)
Work (w) Distance travelled in 20 seconds,
Power (P) =
Time (t ) 1
x1 x2 ut a (20)2
and H = I2 R t 2
H 1
R= x1 x2 0 a 400
I 2
t 2
x1 x2 200 a
54000 54000
=
(5)2 360 25 360 x2 200a 50a

54000 x2 150a ...(2)


= =6
9000 Distance travelled in 30 seconds
R=6 1
114. (4) Neptune’s moon-Triton. Triton is the largest natural x1 + x2 + x3 = 0 + a (30)2
satellite of the planet Neptune. 2
EBD_7332
118
B- NTSE Stage 1 Question Bank

x1+ x2 + x3 = 450a 127. (2) Let the acceleration due to gravity on earth be g
x3 = 450a – 50a – 150a = 9.8 m/s2
x3 = 250a ...(3)
From (1), (2) and (3) GM
g
x1 : x2 : x3 = 1 : 3 : 5 R2
122. (1) Applying law of conservation of momentum, g
0 = m × 16 + 2m × 1
g
2m = – 16m R2
= – 8 m/s G (2M) 2G M
g = =
1 2 1 (3R) 2 9R 2
Total kinetic energy released = m (16) m (8) 2
2 2
2 2
=128 m + 64 m g = g = 9.8 2.177 m/s2
9 9
=192 mJ
The weight of the body on planet is W = mg
n1 2
123. (2) Given n12 = 5×2.1777
3 n2
Thus W = 10.88N
Applying Snell’s law
n1 sin i = n2 sin r 128. (4) Inertia of a body depends on mass, higher the mass higher
will be inertia.
n1
sin x sin 45º 129. (2) Area of velocity time graph will give the distance travelled,
n2
so area of velocity time graph in 3 sec
2 1 = Distance(s) = Area of triangle
sin x
3 2
1
3 = 3 30
sin x 2
2 = 3×15 = 45 meter
x = 60º
130. (3) m1 m2
124. (4) The power and focal length of concave lens is r
negative. Now,
1 m1 m2
P
f r/2
As the distance between two masses is halved,
1
4 gravitational force is quadrupled.
f
G m1 m2
f = – 0.25 m F=
125. (4) New value of current I = 1.5I r2
Now,
Power dissipated, P = I2 R ...(1) r = r/2
New power dissipated P = I 2 R G m1 m2
F =
P = (1.5 I)2 R (r ) 2
= 2.25 I2 R G m1 m2
P = 2.25P F =
2
Percentage increase in power dissipation r
2
P P 1.25P F' 4F
= 100 = 100
P P
Percentage increase in Power dissipation = 125% 131. (1) Wave velocity ( ) = 350 m/s
126. (2) The distance between two consecutive compressions
Wave length ( ) = 70 cm = .7m
or two consecutive rarefactions is called wavelength ( )
As compression and rarefaction occurs alternatively, Wave frequency (f) = ?
therefore distance between a consecutive compression
and rarefaction is half of the wavelength ( /2).
Solved Questions 119
B-

f
3 108
wave velocity = m/s
1.5
= wave length
= 2 ×108 m/s
350
= 140. (2) Cataract is the situation in which a milky and cloudy
0.7 layer is formed on the eye lens of old age person due to
which they lose their vision partially or completely.
= 500 Hz
141. (1) 3
P Q
132. (2) Slip rings are not part of direct current generator, these
3
are used to generate alternating current.
3 3
X
1 1 1 1
133. (3)
Req 20 10 20 Y
Resistors between P and Q and Q and Y are in series so,
1 1 2 1 their effective resistance R1 = 3 + 3 = 6
Req 20 R1 and 3 resistance between P and Y are in parallel.
1 4 So, their effective resistance R2 is given by
Req 20 1 1 1
Req = 5 R2 3 6

l 1 2 1
134. (2) R = R2 6
A
l2 R2 = 2
R= R2 and resistance between P and X are in series.
Hence
m
d= Rnet = 3 + 2 = 5
v
142. (1) Resistivity of the certain material drops to zero at low
m
v= temp. is called superconductor.
d
i.e, =0
84 g
= 1
10.5 g /cm3
840
= cm3
105 1
= 8 cm3 0
Side of square = 2 cm
where = resistivity & conductivity
4
1.6 10 4 143. (2) Refraction of light through prism causes refraction at
=
8 two refracting surfaces.
4
R 0.8 10
A
135. (3) option (3) is correct because in doing so focal length of
eye lens effectively decreases and we know this person 1 2 where A = angle of prism
requires a high power of the eye lens.
136. (4) When a concave mirror is rotated, image does not rotate.
144. (4) With reference to the wavelength, violet is at the lower
137. (3) Metal dishes used for receiving TV signals from distant end of visible spectrum.
communication satellites are concave reflectors.
1 1 1
138. (3) When a convex lens is cut into two parts along principal 145. (4)
Req 6 6
axis, focal length of each part does not change.
139. (1) Refractive index of glass is 1.5 6
Req
2
speed of light in vacuum =3
speed of light in glass =
Refractive index of glass
EBD_7332
120
B- NTSE Stage 1 Question Bank

146. (4) u = 25 cm, = –100 cm


V2 V2 V2
1 1 1 1 1 1 Initial power = Final power = 2
R R/2 R
; Heat generated is doubled.
f u f 100 25
156. (3) Force acting at different parts of
100 cm 1 the magnet is different. So both
f = m
3 3 torque and force will be acting. N S
1 1 157. (2) Power, P = 2 × 103 watt
P= ; P= 3D Energy in 1 min = 2 × 103 × 60 J.
f (m) 1
3 2 × 103 × 60 = m × 10 × 10
m = 1200 kg ; Volume = 1200 litre.
Farsightedness, i.e, Hypermetropia 158. (3) Initial velocity = V
corrective lens convex lens of P = +3D Final velocity = V
147. (2) The displacement at the end of 2 minutes 20 seconds, i.e, 1 1
140s will be the diameter because the athlete will complete mv 2 4 m(v') 2
3 and half rounds. 2 2
V = 2 V.
Therefore, for complete 3 rounds, displacement is zero.
Initial momentum = mv
For half round, the displacement will be the diameter. Final momentum = 2mv.
Therefore displacement = 2R Momentum is doubled.
148. (4) 48×103J in 1 minute 159. (3) Total work done by gravity is zero.
2
48 103 Mm Re
In 1s 160. (2) gm g e = 0.49 m/s2
60 Me Rm
= 8×102 = 800J of energy will be released. 161. (2) P = P1 + P2

800 1 1 f1 f 2
P P
Number of fussion taking place = f1 f2 f1f 2
3.2 10 11
= 250×1011 sin i sin 30o V
162. (1) V' 3V
= 2.5×1013 sin r sin 60o V '
149. (3) Speed of sound depends upon density of medium; so 1 1
speed of sound in air is independent of pressure. 163. (2) g(t)2 g(t 2) 2 40 5t 2 5(t 2)2 40
2 2
2 (r h) t 2 (t 2) 2 8
150. (2) V=
T
1 2 1
2 3.14 (6400 3600) Height gt 10 9 45m.
3.14 = 2 2
T
164. (1) T 2h
T = 2×10000
= 20000s g
151. (3) Electric motor converts electrical energy into mechanical Ta : Tb = a : b
energy. Deflection of reading, pointer in voltmeter and 165. (1) u = V
ammeter also changes with electricity. v = 3V
152. (1) At 4ºC density of water is maximum due to anomalous a=g
expansion of water. Below 4ºC its density will start 4V 2
decreasing and hence volume of water at 4ºC will be less v2 – u2 9V 2 – V 2
S= ; S S=
than the volume of water at 0ºC. 2a 2g g
166. (4) The air bubble has less density than water so it will be
displacement 1 collected at the neck.
153. (4) Velocity =
time tan 60º 167. (3) V3 = V1 + V2 (as V3 is connected in parallel with V1 and V2)
168. (4)
disp 1 VA + 3 – 7 × 3 = VB
=cot 60º =
time 3 VA – VB = 18 V
154. (1) Minimum number of forces F1
V2 V2
required is 3. 169. (4) P1 10 watt; P2
A 3R R/3
V2
155. (1) Resistance of the heater be R. F2 P2 = 3 90 watt
New resistance of heater is R/2. R
Solved Questions 121
B-

V = –30 cm
0i 0 ( ni ) 2 0i
170. (3) B = B = B = n B = n 2B v 30
2R 2R / n 2R m 2
1 1 1 u 15
171. (3) m= 2
V u f 182. (1) Resistivity do not depend on shape and size of material.
1 1 1 183. (2)
f 2 = qp 1 2
f p f q f
1 3 2
f f
172. (3) –m = –u f ; m –u2 f 1 2
1
–f f
f – u1 f – u2 1/3 3 2/3
A B
u1 u2 R eq=4
f= A
2
173. (3) P (V)3 1 2
If velocity is doubled power will become 8 times. Req 3 4
174. (4) (A) Slope of distance time graph is constant for uniform 3 3
speed 60 km/h 40 km/h
(D) Slope of velocity time graph is zero for uniform 184. (3)
A B C
speed 30 km 30 km
175. (3) mb = 50 gm 30 30 1 3 5
vb = 100 m/s T = tAB + tAC = h
mg = 10 kg 60 40 2 4 4
vg = ? = 75 min
185. (4)
50
mbvb 100 1
Vg 1000 Vg m/s
mg 10 2 u
176. (4) 2 2
v=0 v – u = 2g (100)
o – u2 = –2g (100) h
u2 = 2g (100) ... (i)
and when initial velocity is doubled
100 m (2u)2 = 2gH ... (ii) 3u
2
(ii) 2gH 4u (–3u)2 = (u)2 + 2(–g)(–h)
H = 400 m 9u2 = u2 + 2gh
(i) 2g(100) u2
8u2 = 2gh
GM u g M
177. (2) g 4u 2
R2 G R2 h
g
1
178. (2) f 186. (2) Let v2 = 2v1
T
so, unit of frequency is (second)–1. k1 1/ 2m1v12 v1
179. (1) v = f p1 mv1 2
v = 10 × 1000 × 3 × 10–3
v = 30 m/s k2 1/ 2mv22 2v1 v1 k1
Now, 2 2
distance 3 1 p2 mv2 2 2 p1
t t 0.1sec.
speed 30 10
3 108
187. (3) V=v = 0.3 m = 30 cm
109
188. (2)
180. (3) F = 16 N
C F P

8 cm
181. (4) f = +30 cm, u = –15 cm
m

1 1 1 1 1 1
8c

8 cm
f v u v 30 15
EBD_7332
122
B- NTSE Stage 1 Question Bank

F 16 5 C F 32
P –4
0.25 104 Pa 199. (2) C (F 32)
A 8 8 10 9 100 180
[P = 2500 Pa] 9 5
189. (4) W = Fs cos C F 32 C (F 32)
5 9
when = 0°, cos 0° = 1(maximum)
200. (2) Due to refraction when a light ray passes from dense
W = Fs is maximum medium is rarer medium it bends away from normal.
F 201. (1) 4 Meter. Image formed by the plane mirror is at equal
s is correct distance as that of the object from plane mirror.
190. (1) 202. (1) According to the law of floation weight of a floating
10 10 10 body is equal to the weight of displaced liquid.
Ans (1) wt. of displaced liquid.
203. (1) Req = R1 + R2
20 = (2 + 2) = 4
A B
1 1 1
R eq R1 R2
20 20
RAB = ? 1 1 2
1
1 1 1 1 2 2 2
So, Req= 4 , 1
R AB 30 20 40 204. (2) no matching option.
1 4 6 3 13 120 (i). n = 2, (ii) n = 3, (iii) n = 5
RAB = = 9.23 1
R AB 120 120 13 S= u a (2n 1)
2
L (i). 2n – 1 = 2 × 2 – 1 = 3
191. (2) R1 = ...(1)
4A (ii). 2n – 1 = 2 × 3 – 1 = 5
2L
R2 = ...(2)
A (iii). 2n – 1 = 2 × 5 – 1 = 9 Ratio 3: 5 :9
R1 L A
R2 4 A 2L
v2 u2
R1 1 205. (4) a=
1: 8 2l
R2 8
192. (1) On increasing the length of the conductor by strecthing, v2 u2 l
V2 = u2 2
the new resistance is given by 2l 2
R = (n)2R Where n is no. of times of stretching
R = (3)2R [n = 3] u 2 v2
V=
R = 9R 2
193. (1) Our solar system lies in“Milky way” Galaxy
194. (3) Resistance is obstruction to the flow of electric current.
195. (3) Radiation 206. (3)
Conduction and convention needs medium to travel, r
while radiation does not need medium to travel.
196. (4) 1000 cubic centimeter
1 litre = 10–3m3 2 r 4 r 6 r
1 meter = 100 cm = 3 r
2 2
1 litre = 10–3 (100 cm)3 = 10–3 × 106 cm3
litre = 103 cm3 Distance in 45 second 3 r 3
197. (4) Irregular & non - periodic vibration =
Displacement in 45 second 2r 2
Music is produced by regular and periodic vibrations x
Noise is produced by irregular & non - periodic vibrations. 207. (3) P x
198. (3) x x

x
O
1 1 1 1 3 1 3
= =
R X X X x R X
Ray which passes through optical centre remains
undeviated.
Solved Questions 123
B-

X 4 X 220. (2) In electromagnetic wave, the direction of electric and


R= X X R= magnetic field are always perpendicular to each other.
3 3 3 221. (2) Vm–1 is the unit of electric field intensity.
3 1 7 1
= 2
4X X 4X 222. (3) K.E1 = M1V ...(1)
2
1 7 4X 1
= R= 2
R 4X 7 K.E2 = M 2 V ...(2)
2
According to Question, The velocity of both the body is same as they have
4X 21 equal momentun.
=3 X= Dividing eq (2) by (1)
7 4
208. (4) By the rule of conservation of momentum K.E 2 M 2 2
M1V1 = M2V2 K.E1 M1 3
3 × V1 = 6 × V2 ...(1)
Thus, the ratio is M1 : M2 = 3 : 2
1 223. (3) 224. (2) 225. (2) 226. (3)
K= 3 V12 216
2 227. (3) Joule is a unit of heat, it is not the unit of force.
228. (4) Holland is also known as country of wind mills.
216 229. (1) The first president of atomic energy commission in
V1 = 2 12 m / s
3 India was Homi Jahangir Bhabha.
from eq. (1) 230. (1) Convex lens is used in camera.
3 × V1 = 6 × V2 231. (3) Uri Gagarin was the first astronaut who went to space.
3 × 12 = 6 × V2 232. (3) Supply voltage = V1 (through resistor of 2 k )
V2 = 6 m/s + V2 (through volt meter)
209. (3) 210. (4) 211. (2) 212. (1) 10 V = V1 + 4V
213. (1) A real and enlarged image can be formed by using a V1 = 10 V – 4 V = 6 V
convex mirror. V=RI
214. (4) There are 8 planets in the solar system. (Pluto is not 6 = I × 2000
considered as a planet). 6
215. (4) In house electrical circuits the fuse wire for safety I= = 3 mA
should be of high resistance and low melting point. 2000
216. (2) An the three resistors are connected in parallel Power developed across R, P = V2 = 4×3 mW = 12 mW
233. (4) During landing force exerted on parachutist by the
1 1 1 1 ground,
Re q R R R N = weight + mass of parachutist × upward acceleration
= mg + m × 3g = 4 mg = 4 W
1 3 234. (2) Vair = 340 m/s, Vwater =1440 m/s
Re q R Echo is the sound that we hear after reflection of sound
wave from the reflecting surface.
R i.e. total distance travel by sound wave = 2d
Re q
3 distance
217. (1) For an incident ray directed towards centre of Velocity =
time
curvature of a spherical mirror the reflected ray retraces
its path. 2d 1440 × 1.5
1440 = d= = 1080.0 m = 1.08 kms
L 1.5 2
218. (4) Resistance (R) = 235. (2) Potential of one small drop of mercury,
A
Length is stretched to double kq
V= 20V
L = 2L r
A
Volume of big drop = Volume of 8 small drops
Area A = 4 3 4 3
2 R 8 r R 2r
3 3
2L
R = A Q = 8q
Potential of big drop,
2
kQ K 8q 4kq
L V= 4 20 80V
R = 4 R 2r r
A 236. (3) The relation between momentum P and Kinetic energy K
R = 4R
R= 1 New Resistance, R = 4 P = 2 Km P m
219. (2) Alternating current supply has two polarity, positive P1 m1 1 1
and negative polarity. Thus, for 50Hz frequency, the
polarity of an A.C. supply would change 100 times. P2 m2 4 2
EBD_7332
124
B- NTSE Stage 1 Question Bank

237. (1) Focal length of lens to be used must be equal to 248. (4) As we go from solid to gaseous state, velocity of sound
distance of far point. decreases. Velocity of sound in iron > oil > water > air.
So, f = – 40 cm 249. (1) Given : V = 220 volt
(As myopic person needs concave lens so f negative) P = 60 W
1 R= ?
1
Power of lens, P = =- = -2.5D I= ?
f (in metre) 0.40
238. (2) Ring will be at rest as flux (f = BA cos q) is zero v
2
(60)2
As, R = = = 806.6652
(Q q = 90° and cos 90° = 0). p 220
239. (4) One AC cycle P 60
+ Max And I == = 0.27 ampere
V 220
0 250. (3) The maximum attraction in a magnet is on the poles.
0 0 m1u1 + m 2 u 2 m ´ 3v + 3m ´ v
251. (2) Velocity, V = =
– Max m1 + m 2 m + 3m -
2 3mv + 3mv 6mv 3
u
240. (1) H = [For maximum height], Kinetic energy is zero V= = = v
2g 4m - 4m 2
and potential energy is maximum = mgH at highest point. 252. (2) Charge on electron, | e | = 1.6 × 10–19 C
Let at the height + h Current through wire = 0.32A = 0.32 C/s
Kinetic energy of the ball is reduced to 70% 0.32
= e / s = 2 × 1018 e/s
u 2 v2 u2 1.6 ´ 10-19
h= - = 0.3 = 30 m No. of ‘e’ pass through wire in one hour
2g 2 g 2g
= 2 × 1018 × 3600 e/hr = 7.2 × 1021 e/hr = 72 × 1020 e/hr
(Q u2 = H 2g and H = 100 m)
241. (1) Resistivity is the reciprocal of conductivity R
253. (3) Let resistance of each part be ‘r’, r =
1 5
i.e. r = r C
s D
Lowest Resistivity = Highest Conductivity r r
242. (4) By Right Hand Thumb rule : According to this rule, if r
we grasp the conductor in the palm of the right hand so that A B
r
the thumb points in the direction of the flow of current.
Then the direction in which fingers curl gives the direction
V
of magnetic field.
As circuit is forming wheat stone bridge, so
Req = [RAD + RDC] | | [RAB + RBC]
éR Rù é R R ù é 2R ù é 2R ù
243. (2) Displacement = ê + ú ê5 + 5ú = ê 5 ú ê 5 ú
ë5 5û ë û ë û ë û
2R R
O time As both resistance in parallel. So Req will be equal to .
5 5
Displacement 254. (2) Magnetic field at the centre of circular coil is directly
Velocity = =O proportional to the number of turns
Time
Q (From graph, displacement) \ B = 10 B0
255. (3) A virtual & enlarged image is produced by concave
Density of body 7100 kg m -3 mirror
244. (3) R.D. = = = 7.1
Density of water 1000 kg m -3 256. (1) f = –15 cm V = –10 cm
As we know,
(Q Water density in SI system = 1000 kg m–3)
245. (3) K = 273 + C Þ DK = DC 1 1 1 1 1 1 1 æ 1ö
i.e. the difference in temperature in °C = temperature - = = - = - - ç- ÷
v u f u v f 10 è 15 ø
difference in Kelvin scale.
246. (4) As l¢ = l0 (1 + a Dt) i.e., the expansion of a solid depends 1 1 3+ 2 1
= - + =- =- u = –30 cm
on both nature (1) and temperature (Dt) of the solid. 10 15 30 30
247. (4) The particles of the medium in the transverse wave 257. (None) [Power] = [force × velocity]
move perpendicular i.e., in the form of crests and troughs = [MLT–2] [LT–1] = [ML2T–3]
perpendicular to the direction of wave motion. So, none of the given option is correct.
Crest 258. (4) According to law of inertia or Newton’s first law of
motion, a body in state of rest or of uniform motion remains
in state of rest or of uniform motion unless it is acted upon
by an external force. Hence a body in uniform motion required
no force to keep them in uniform motion.
Trough
Solved Questions B-125

F Energy consumed by 10 HP water pump in 30 minutes


259. (3) a , Force between the blocks = Power × time = 10 × 746 × 30 × 60 J
M1 +M 2 ( 1 HP = 746 W and 1 min = 60 sec)
F = 13.428 × 106 J
= M2
M1 + M 2 SECTION 2. CHEMISTRY
F 32 C
260. (3) As C O2 CO2
9 5 1. (3)
12 g 22.4 L at STP
And from question, F = 2C
12 g of C is completely burnt in air to give 22.4 L CO2 at
2C 32 C STP.
9 5 22.4
9C = 10C – 32 × 5 4g of C will give 4 7.46 L
C = 160 12
2. (1) Molar mass of CuSO4 = 159.609 g mol–1
261. (3) Mass of Cu present in 159.609 g (1 mol) CuSO4 = 63.5 g
I Mass of Cu present in 50 g CuSO4
O 63.5
50 19.89 19.90 g
2f 2f 159.609
3. 23
(3) 6.022 × 10 number of nitrogen atoms = 14 g
i.e. distance between object and image = 2f + 2f = 4f
3.022 × 1023 number of nitrogen atoms
kQ 5Q
262. (4) E E1 at 5 cm = k 14 3.022 1023
2
r 52 7 g.
6.022 1023
k 5Q E1 4. (2) Electronic configuration of potassium (K)
and E2 at 10 cm = 4 :1 K[19] : 1s2 2s2 2p6 3s2 3p6 4s1
2
(10) If we observe above electronic configuration, 19th
E2
263. (1) Solar energy is due to the nuclear fusion reactions. electron is present in 4s orbital.
264. (2) Resistors B and C are in parallel connection n 4
Effective Resistance of B and C is l = 0 (for s orbital)
m1 = 0
1 1 1 1 1
1
R B C 3 2 ms
2
B C 3 2 6 5. (2) At anode O2 and at cathode H2 will be obtained.
R
B C 3 2 5
Resistor A is in series connection with B and C Na 2SO 4 2Na SO 24
Total effective resistance = A + R
6 36 H 2O H OH
6 , , = 7.2
5 5 At cathode : 2H 2e H2
265. (4) As magnetic lines of forces are closed continuous
curves, so it is incorrect that they start from the north 1
pole and end on the south pole rather, outside the body At anode : 2OH O2 H 2 O 2e
2
of the magnet. The direction of magnetic field lines is
H2SO4 Acidified
from north pole to south pole and from south pole to 6. (3) X Y (gas) Q (green solution)
north pole inside the magnet. dichromate
solution
266. (2) Rainbow is a seven colours band VIBGYOR that we Baryta
see often in rainy season. It is due to dispersion, Ba(OH)2
reflection and refraction of light.
267. (3) (Turbidity)
Total distance travel SO 2 gas changes dichromate (orange) to green
268. (1) Average speed = substance.
Total time taken
So, SO2 can be produced if substance containing
720m (SO32–) reacts with dil. H2SO4.
= 36 ms 1
20s Alcoholic
269. (2) We get a p-type of semiconductor when semiconductor 7. (3) C2H5I C2H4 + KI + H2O
KOH
(intrinisic) is doped with trivalent material like Indium. Ethyl Iodide
270. (3) Escape velcoity Alkaline
KMnO4 (purple)
Ve = 2g Re where Re = redius of the earth
Work done or Energy (Colourless)
271. (4) Power P = substance
Time
EBD_7332
126
B- NTSE Stage 1 Question Bank

H 2S HCl Electrolysis
8. (1) X FeS FeCl2 H 2S 17. (3) NaCl(aq) NaOH(aq) Cl 2 H2
(Green (Black ppt.) Brine (Anode) (Cathode)
solution solution
contains
Fe2 ) 18. (1) 4Al 3O 2 2Al2 O3

9. (4) CH3 – CH – CH3 Acidified


CH3 – C – CH3
I2
CHI3
Reactant Product
K2 Cr2O 7 Na2CO3
Al 4×1 2×2
OH O
(X) Y (gas) O 3×2 2×3
x = 4, y = 3 and z = 2
10. (3) H 2SO 4 Na 2 CO3 Na 2SO 4 CO 2 H 2O So, coefficient of Al2O3 = 2.
100 ml Excess
Sunlight
0.1 M 19. (3) CH 3 Cl Cl 2 CH 2 Cl 2 HCl
Moles of H2SO4 = M × V It is a substitution reaction.
= 0.1 × 100 mL 20. (1) Collenchyma is the living mechanical tissue as it
= 10 millimole = 0.01 moles provides flexibility and elasticity to the plant. The plant
bends without breakage due to collenchyma. The cells
1 Mole of H2SO4 Produces 1 mole of CO2 are elongated and have differential thickening at the
corners of the cell wall due to the deposition pectin,
0.01 moles of H2SO4 Produces 0.01 moles of CO2 cellulose and hemicellulose.
1 mole of CO2 molecules occupies 22.4 L. Collenchymatous tissue provide mechanical support
0.1 moles of CO2 occupy = 0.01 × 22.4 L = 0.224 L to the growing parts of the plant such as young stem
11. (2) Molar mass of NH3 = 17 g and petiole of a leaf.
Molar mass of HCl = 36.5 g H
|
1
Rate of diffusion 21. (1) H C C C H
Molar mass |
H
NH3 gas will diffuse at faster rate than HCl. So, white
ammonium chloride ring will get formed near HCl bottle. Covalent bonds = Total no. of 6 bonds
+ Total no. of bonds
NH 3 HCl NH 4 Cl = 6 + 2 = 8.
Heat [O]
12. (2) 2Pb(NO3 ) 2PbO 4NO 2 O2 22. (3) CH3OH HCOOH
Oxidation Formic acid
Lead nitrate Nitrogen Methyl alcohol
dioxide 23. (3) Germanium (Ge) is the element of 14th group and IVth
period. Antimony (Sb) is the element of 15th group and
13. (3) Pb(NO3 ) 2 2KI PbI 2 KNO3 Vth period.
(Yellow 24. (4) Total no. of atoms in 1 molecule of Ca(HCO3)2 = 11
ppt)
Total no. of atoms in 1 mole of Ca(HCO3)2
14. (1) CH3CH 2OH
Conc. H 2SO 4
CH 2 CH 2 H 2O = 11 × 6.02 × 1023.
443 K 25. (1) Carbon dating is a technique used to determine the
Ethyl alcohol Ethene Water
15. (2) CuO is insoluble in water. approximate age of once living materials. It is based on
the decay rate of radioactive carbon isotope 14C.
16. (1) Number of atoms = moles × NA
26. (3) 1 mol. of H2O (water) contains = 6.022 × 1023 molecules
Given mass 0.25 moles of H2O will contain
Moles of an atom
Molar mass = 6.022 × 1023 × 0.25 = 1.5055 × 1023 molecules.
27. (1) 18 g (1 mol) of water contains 6.022 × 1023 molecules.
100g 9 g of water contain 3.011 × 1023 molecules.
Atoms of He NA 25N A
4g (1) Mass of 1 mol of Mg = 24 g
24 g of Mg contains = 6.022 × 1023 molecules
100g
Atoms of Na NA 4.347N A 12 g of Mg contains = 3.011 × 1023 molecules
23g (2) Mass of 1 mol of C = 12 g
100g 12 g of Carbon contains = 6.022 × 1023 atoms
Atoms of Li N A 14.49N A (3) Mass of 1 mol of NH3 = 17 g
6.9g
17 g of NH3 contains = 6.022 × 1023 molecules
100 g (4) Mass of 1 mol. of CO2 = 44 g
Atoms of Al N A 3.7N A
27 g 44 g of CO2 contains = 6.022 × 1023 molecules
Hence, Helium will have largest number of atoms. 11 g of CO2 contains = 1.505 × 1023 molecules
28. (1) Molecular formula of chloride = MCl2
Valency of metal M = 2
Solved Questions 127
B-

Molecular formula of its oxide will be 4. He, Ne, Ar- Noble gases (Group 18)
M O 42. (3) Soaps are sodium or potassium salt of fatty acids, made
by hydrolysis of fats and oils with bases. This process
yields soap as a product and glycerol as by-product.
Valency 2 2 O

Formula M2O2 = MO CH2 – O – C


29. (1) Conjugate Acid H+ + Conjugate base R1
+
O CH2 – OH R1 – COONa
1. HCO3 H CO 32
+
CH – O – C + 3 NaOH CH – OH + R2 – COONa
2. NH 4 H NH 3 R2 CH2 – OH R3 – COONa
+

3. O
H 2O H OH
CH2 – O – C
4. HNO3 H NO3
R3
HNO 2 H NO 2 Triglyceride Sodium Glycerol 3soap molecules
30. (3) HCl gas is used for precipitation of pure common salt (fat or oil) hydroxide
(NaCl) from saturated solution of common salt. When 43. (2) The structure of proypyne is
HCl gas is passed, the concentration of Cl– increases
H
and value of ionic product of [Na+] and [Cl] becomes
greater than Ksp of NaCl. H–C C–C–H
Hence, pure NaCl is precipitated.
31. (2) The correct increasing order of reactivity of elements H
is There are 6 bonds and 2 bonds. Hence ratio of and
Au < Cu < H < K bonds in propyne is 6 : 2 or 3 : 1
32. (2) X has 8 electrons in M shell when gains one electron so 44. (2) At a given temperature
K L M P1V1 = P2V2
= P1 = P, P2 = ?
2 8 8 V1 = V, Volume is decreased by 5%
K L M V2 = 0.95 V
2, 8, 7 Hence, P × V = P2 × 0.95 V
3rd period, 17th group P2 = 1.0526 P
So, valency = 1 increase in pressure = 1.0526 P – P
Hence, (ii) and (iii) are incorrect. = 0.0526
33. (1) Kossel and Lewis proposed the octet rule.
34. (2) Compound (b) is ether and name of this compound is % increase = 5.26%
methoxy methane. Hence there will be 5.26% increase in pressure of the
35. (2) Bond will be Ionic gas.
EN 1.7
EN 1.7 Bond will be covalent 45. (4) Pb(NO3 ) 2 H 2S PbS 2HNO 3
(Black)
EN 1.7 50% Ionic and 50% covalent.
36. (3) Conc. HNO3 being a strong oxidising agent deposits a ZnCl 2 H 2S ZnS 2HCl
(white)
thin layer of iron oxide making it passive.
37. (3) Rain water is slight acidic having pH range 5 – 5.5. 46. (3) In both the structure, functional groups are same, only
pH pOH 14 there positions are different. Hence, they are positional
isomers.
pOH = 14 – 5.5 = 8.5.
47. (2) Egg albumin in water forms a colloid.
38. (3) LPG is composed of hydrocarbons containing three or
four carbon atoms. The normal components of LPG are 48. (2) 4NH 3 5O 2 4NO 6H 2 O
propane (C3H8) and butane (C4H10) 2NO O 2 2NO 2
39. (3) When chlorine is passed over dry slaked lime at room
temperature, the main reaction product is CaOCl2 4NO2 O2 2H 2 O 4HNO3
(bleaching powder). The reaction is as follows :
Ca(OH)2 + Cl2 CaOCl2 + H2O 49. (4) Electrolysis of brine (aqueous solution of NaCl)
produces sodium hydroxide, hydrogen gas and
40. (1) Magnalium : Al + Mg chlorine gas.
Bronze : Cu + Sn
Electricity
Brass : Cu + Zn NaCl(aq) NaOH Cl 2 (g) H 2 (g)
Alkali Chlorine gas Hydrogen gas
German silver : Cu + Ni + Zn (Sodium hydroxide)
41. (3) 1. Li, Na, K-Alkali metals (Group 1)
2. Be, Mg, Ca-Alkaline earth metals (Group 2)
3. N, O, F- (Period 2)
EBD_7332
128
B- NTSE Stage 1 Question Bank

50. (2) The structure of Benzene (C6H6) is A B


H A2B3
3 2
H C H 64. (4) Zn and Al are more reactive than iron, therefore they
C C will displace iron from its salt solution giving black
residue, while Cu being less reactive than iron will
C C not able to displace iron from its salt solution.
H C H FeSO + 2Al Al2(SO4)3 + 3Fe
FeSO4 + Zn ZnSO4 + Fe
H FeSO + Cu No reaction
Total covalent bonds in benzene are 15 FeSO + Fe No reaction
51. (1) 2CsO2 + 2H2O 2CsOH + H2O2 + O2 65. (2) Electro-negativity increases on moving left to right in
CsO2 is the oxide of alkali metal. It is a basic oxide. the periodic table, while it decreases on moving down
Due to formation of CsOH its aqueous solution is the group.
basic. The correct order of E.N. is
52. (1) When universal indicator is added to a neutral Si < P < C < N
solution its colour changes to green hence pH should (1.8) (2.1) (2.5) (3.0)
be 7. 66. (3) 2Na + 2CH3 – CH2 – OH 2CH3CH2ONa + H2
So, the nearest option 1 i.e. pH value 7 – 9. Sodium ethoxide
53. (2) Sublimation is a process in which solid directly Hot
changes to gaseous state without undergoing liquid CH3CH2OH CH2 = CH2 + H2O
Conc. H 2SO 4
state.
Alcohols with sodium metal produce sodium alkoxide.
(s) (g) Alcohols also undergo dehydration in presence of
54. (1) pH = –log[H+] conc. H2SO4 to produce corresponding alkene.
[H+] = 2 10–8 67. (1) MgSO4.7H2O commonly known as epsom salt
pH = –log (2 10–8) CuSO4.5H2O commonly known as blue vitriol
pH = 8 log 10 – log (2) FeSO4.7H2O commonly known as green vitriol
pH = 8 – 0.301 CaSO4.2H2O commonly known as gypsum
pH = 7.699 68. (2) one mole of any gas occupies 22.4 L of volume at
55. (4) SO2 has both covalent as well as coordinate bond. STP
56. (2) Sulphide ores are concentrated by froth flotation 22.4
process. 0.5 mol of gas occupies L volume
2
57. (3) Al2O3 is amphoteric oxide. It can behave as acidic as = 11.2 L
well as basic oxide. Na2O and CaO are basic oxides Reduction
while SO2 is acidic oxide.
+4 0 0 +3
e 69. (1) 3MnO2(s) + 4Al(s) 3Mn + 2Al2O3 + Heat
58. (4) Among the isoelectronic species greater the ratio
p Oxidation
larger will be the ionic radii, hence the order is MnO2 acts as an oxidising agent and itself gets
S2– > Cl– > K+ > Ca2+ reduced, while aluminium undergoes oxidation process
59. (4) Propyne gives white precipitate on reaction with and act as a reducing agent.
ammonical AgNO 3 due to presence of acidic C Si, Ge Sn, Pb
hydrogen. 70. (3)
Nonmetal Metalloids Metals
60. (2) Carboxylic acid molecule has higher extent of H- Within a group from top to bottom in the periodic
bonding than alcohol, therefore order of b.p. is table metallic character increases. Carbon is non metal,
CH3CH2CH2COOH > CH3CH2CH2CH2OH> CH3CH2CH2CHO Si and Ge are metalloids and Sn, Pb are metals.
71. (3) The general molecular formula of alkyne is Cn H2n – 2.
61. (4) For n = 2, molecular formula is C2H2
CH3CH2CH–CH3 CH3CH2CH=CH2 + CH3CH=CHCH3 72. (3) Ammonium hydroxide is a weak base as it ionises
1-butene (Minor) 2-butene (Major)
partially in its aqueous solution, while NaOH and KOH
Br are strong bases.
62. (3) Isotopes are the atoms of same element having same
atomic number but different mass numbers. They 73. (2) Zn + CuSO4 ZnSO4 + Cu
(blue) colourless
possess an equal number of protons and electrons.
Zinc displace copper from CuSO4 solution, so blue
63. (1) ‘A’ belongs to III B / 13th group, therefore valency of A colour of the solution will disappear.
is 3, ‘B’ belongs to VI B / 16th group, so it shows
valency 2 Mass 0.36
74. (4) Number of Moles = = 0.02
So, chemical formula of the compound is Molecular mass 18
Solved Questions 129
B-

75. (2) Cobalt-60 is radioactive isotope of cobalt which is 3 = n +1


used in cancer treatment.
n=2
76. (1) OH– is conjugate base of H2O and Cl– is conjugate
base of HCl. O
77. (1) Plaster of paris is calcium sulphate hemihydrate. 94. (2) H– N
..
O
78. (4) Loss of electron is oxidation process.
HNO3 consists of co–ordiante bond between N & O
Fe+2 Fe+3 + e–
Oxidation 95. (3) To decrease the acidity of stomach, we can use
+2 +4
Mg(OH)2, Al(OH)3, NaHCO3 compounds
0 0
79. (2) 2PbO + C 2Pb + CO2 96. (4) If gypsum is heated above 100º C then
Reduction
100º C
So, the given reaction is an example of redox reaction. CaSO 4 CaSO 4
80. (1) Carbon monoxide (CO) is highly toxic and inflammable dead burnt plaster
gas. It combines with blood haemoglobin to form
carboxyhaemoglobin. 97. (1) Order of atomic radius Li > Be > B > C
81. (1) Copper sulphate penta hydrate is commonly known as 1
blue vitriol. Effective nuclear charge
atomic radius
82. (2) H2S2O7 2SO3 + H2O
Pyrosulphuric acid 98. (2) Fe O
83. (3) NH4NO2 N2 + 2H2O
Ammonium nitrite produces nitrogen gas on heating. 3 2
84. (1) (CH3)2CHCH3 and (CH3)2CHCH2CH3 have different Fe2O3
molecular formula, therefore they are not isomer to
each other. 99. (4) Acid +Alcohol Conc H2SO4 Ester + H2O
85. (4) CH3COOH + NaHCO3 CH3COONa + CO2 + H2O
C2H5OH + NaHCO3 no reaction CH3COOH + C2H5OH Conc H2SO4 CH3COOC2H5 + H2O
Acetic acid shows effervescence on reaction with Acetic acid Ethyl alcohol
NaHCO3 solution due to the formation of CO2 but 100. (3) Nitrous oxide, commonly known as laughing gas. It is a
with ethanol no effervescence is produced. colourless non-flammable gas with slight metallic taste.
86. (2) Li, Na, K all have one election in outermost s-orbital 101. (4) Silver jewellery turns black when kept in the air because
while Be has fully filled outermost s-orbital with two silver reacts with sulphur compounds such as hydrogen
electrons, therefore Be has highest first ionizotion sulphide (H2S) present in air. The phenomenon is called
energy. corrosion.
87. (2) HCl is an acid and methyl orange turns orange to red in 102. (2) Glass is fairly inert to the oxidising power of nitric acid
acidic solution. (constituent of aqua regia).
88. (3) NH4Cl is a salt made by NH4OH and HCl. HCl being 103. (3) Marble is chemically calcium carbonate.
strong acid will give acidic salt and thus acidic solution. CaCO3 + 2HCl CaCl2 + CO2 + H2O
104. (4) Rancidity is the complete or incomplete oxidation of fats
89. (1) pH = – log [H3O+] and oils when exposed to air, light or moisture or by
90. (4) Methyl orange changes its colour from orange to yellow bacterial action, resulting in unpleasant taste and odor.
in basic solution, as 8.3 to 10.0 pH is for basic solution, 105. (4) Milk of magnesia is used as an antacid.
thus the colour changes to yellow. 106. (1) Bronze has the property of being stronger and more
91. (1) Aqueous solution of SO2 is acidic. corrosion resistant. So it is used in the preparation of
SO2 (g) + H2O(l) H2SO4 (aq) statues, coins and medals.
sulphur water sulfuric acid 107. (3) Concentration of ore, reduction and roasting are step of
metallurgy but not corrosion.
108. (1) Formula of carbon – tetrachloride is CCl4
dioxide
109. (2) Electrons present in Cl– = 18
92. (4) Sublimation is the transition of a substance directly from
Electronic configuration of Cl– = 2, 8, 8
solid to the gas phase without passing through
intermediate liquid phase. As it contains 8 electrons in outermost shell so, valency
The substance showing sublimation property is camphor electron in Cl– ion is 8
110. (4) As at 0ºC (freezing point) both slate exist, i.e, solid &
93. (3) Tritium = 1H3 liquid.
Atomic mass = no. of neutrons + no. of protons 111. (1) Isoelectronic species are elements or ions that have the
3=n+p same or equal number of electrons.
EBD_7332
130
B- NTSE Stage 1 Question Bank

Na+, Mg2+, H3+ = 10e– 121. (3) C2H5OH + NaHCO3 No reaction


Ca2+ = 18e– CH3COOH + NaHCO3 CH3COONa + CO2 (
112. (4) K > Mg > Al > B (Reactivity series) In case of ethanoic acid, brisk effervescence of carbon
Reactivity of metals decreases as we move from left to dioxide is evolved.
right in a period. 122. (1) s, p, d, f subshells are present in atom
123. (1) Boron and cadmium used in atomic reactors to
113. (1) Oxalie acid COOH is solid at room temperature. control speed of neutron.
| 124. (2) 125. (3)
COOH 126. (1) Cr = [Ar] 4s1 3d 5
Oxidation Half filled configuration of d – subshell is more
114. (3) symmetrical
127. (3) Amphoteric. Al2O3 can react with acid and base both.
O +8/3 128. (3) As solution is acidic so pH < 7
+1 O 129. (3) In basic solution phenolphthalein shows pink colour.
3Fe(s) + 4H2O(g) Fe3O4(s) + 4H2(g) 130. (1) 4s, 4p, 4d, 4f
Reduction 4s = one orbital, 4d = five orbital
Total = 7 + 5 + 3 + 1 = 16
(Oxidising agent) 4p = three orbital 4f = seven orbital
The increase in oxidation state of an atom, through a So, total no. of electron = 32
chemical reaction, is known as an oxidation, a decrease 131. (1) CO2, CH4, N2O and O3 are act as green green house
in oxidation state is known as a reduction. gases.
115. (1) Formula of red oxide is Fe2O3 132. (2) Due to Ca(NO3)2 and CaSO4. As oxides of nitrogen &
116. (3) d - block elements are known as transition elements. sulphur reacts with limestones on Taj Mahal to form
nitrates and sulphates of calcium.
117. (2) Isotopes of carbon – C –12, C –14 133. (4) Cheese is gel.
Isotopes of chlorine – Cl –35, Cl –37 134. (4) CaO + H2O Ca(OH)2
Isotopes of iodine – I –129, I –131 Ca(OH)2 + 2NH4Cl CaCl2 + 2NH3 + 2H2O
But Neon has no isotopes. 135. (1) Cu + SO42– Cu2+ + SO2
118. (1) Silver nitrate is used in voting ink. Choking smell
LiAlH 4
119. (4) Balanced chemical equation: 136. (1) CH3 COOH CH3 CH2 OH
137. (2) Sodium is present in first group of s block.
NaOH (aq) + HCl (aq) NaCl (aq) + H2O(l)
138. (4) Baking powder is a mixture of sodium hydrogen
15 ml of NaOH neutralizes 10 ml of HCl from reaction, it carbonate and tartaric acid.
is clear that 1mol. of NaOH reacts with 1 mol of HCl 139. (1) CO2 is a gas and SiO2 is solid due to polymeric
structure because it does not form Si = O.
This means 140. (1) 46 g Na atom contains = 46/23 = 2 mole = 2NA atom
nNaOH = nHCl 8 g O2 = 8/32 = 0.25 mole = 0.5 NA atom
0.1 mole C= 0.1 NA atom
Let the concentration and volume of NaOH and HCl be 28 g N2 = 28/28 = 1.0 mole = 1.0 NA atom
C1 V1 and C2 H2 respectively. 0.1 mole carbon atom = 0.1 Na atom
28 g N2 molecules = NA molecules
C1 V1 = C2 H2 141. (3) Valency of X = 2, So correct formula is MgX.
C1×15 = C2×10 H+
142. (3) MnO 4 +Cl Mn 2+ + Cl 2
C1 15 143. (4) Carbon has small size and maximum catenation property.
C2 =
10 144. (3) Fe2O3 + 2Al Al2O3 + 2Fe (Molten)
145. (2) NH4Cl (sublime); NaCl (not sublime)
3
C2 = C1 H+
2 146. (3) HCl Cl
Acid Conjugate base
3
So C1×30 = C1 V2 373K
2 147. (3) CaSO4 . 2H2O CaSO4 . 1/2 H2O + 3/2H2O
V2 = 20 ml Gypsum Plaster of paris
148. (2) 149. (4)
Hence we require 20 ml of HCl of the same concentration
3 2 1
to neutralize the 30 ml of NaOH. 150. (2) H3 C C == CH2
|
120. (1) CO + ZnO Cr2 O3 CH OH CH3
3 2-methylprop-1-ene
300atm Methanol
151. (1) S K L M
16 2 8 6
Solved Questions 131
B-

152. (3) Fe Fe2+, Fe3+ 188. (4) Gold number is defined as the minimum amount of
153. (1) protective colloid in milligram which prevents the
154. (1) Freon – 112 is C2F2Cl4 change of colour.
155. (2) 14 g N2 contains 14/28 = 0.5 moles (if molecular 189. (3) Hydrogen : Oxygen 1 : 4 = WH2 :WO2
weight of N2 is = 28)
Number of molecules in 0.5 moles of nH 2 WH2 M O2
N2 = 0.502×6.022×1023 = 3.011×1023 1 32 4
nO2 = M W = = 4:1
156. (1) H2 O2 4 2 1
Mg Cl2 MgCl 2 (high m.p. / b.p.) Mg(OH) 2 I & IV = 2
MgCl2 is ionic compound Water soluble 190. (4) Ca = 2 8 8 2
I II III IV II & III = 8
(ionic compound have high
melting and boiling point)
The nature of CaO is basic.
O O 191. (1) Alitame is an artificial sweetner. It is nearly two
157. (4) nHO – C – (CH2)4 – C – OH + nH2N – (CH2)6 – NH2 thousand time & sweet in comparision to sugar.
Adipic acid Hexamethylene diamine
192. (4) Turmeric is an example of natural pH indicator. It can
be used to determine a substance's pH.
O O 193. (3) Generally metals are ductile to drawn out into a thin
wire.
–– C – (CH2)4 – C – NH(CH2)6 – NH ––n + nH2O 194. (4) Coal is called fossil fuel because they have been formed
158. (2) from fossilized remains of prehistoric organisms as long
159. (3) Ethylene has the formula C2H4 or H2C = CH2. as millions and millions years ago.
160. (1) All non-metal oxide on reacting with water gives acidic 195. (1) Functional group of butanal is aldehyde showing
solution. structure – CHO. e.g.
161. (3) Anthracite has maximum percentage of carbon. H H H
162. (1) O
163. (1) Zn + 2HCl ZnCl2 + H2 H–C C C C
H
164. (2) Bromine liquid at room temperature. H H H
165. (1) Na reacts with cold water to give H2 gas Butanal
166. (3)
167. (4) Flint glass 196. (2) NH 4 Cl
168. (3) Controlled nuclear fission required in nuclear reactor. H
169. (4) CaC2 + H2O C2H2 + Ca(OH)2 + – –
170. (2) Stainless steel contains Iron, Chromium and Carbon. NH 4 Cl N+ Cl
H H
171. (4) H
172. (1) Na2CO3 is formed from NaOH and H2CO3 i.e, strong NH4Cl shows ionic bond between NH4+ and Cl– ions
base and weak acid. and three covalent bonds are formed between nitrogen
173. (3) Ionisation energy decreases down the group and and three hydrogen atoms and one coordinate bond is
increases across the period. Hence K formed between nitrogen and one hydrogen atom.
174. (4) K+ = 19 – 1 = 18 hot, conc.
197. (3) CH 3 CH 2 OH CH 2 CH 2 H 2 O
Cl– = 17 + 1 = 18 H 2SO 4
(Ethene)
Ca2+ = 20 – 2 = 18
Hence only K 198. (1) 11 Na – 2, 8, 1
175. (3) Na2SO4 + BaCl2 BaSO4 + 2NaCl
19 Na – 2, 8, 8, 1
176. (2) CuSO4 . 5H2O CuSO4 Na, and K have same electronic configuration of
Blue white anhydrous CuSO 4 valance shell.
177. (4) Ant sting contains methanoic acid also called formic 199. (3) Methanol (CH3OH) is added into ethanol (C2H5OH)
acid. to make it unfit for drinking.
178. (2) HCl. Rest all are ionic. 200. (4) Ultra violet radiation is absorbed by CO2 molecules in
179. (1) Copper lies below H in reactivity series. the atmosphere.
180. (2) CH3COOH and C2H5COOH a difference of –CH2 201. (3) Kerosene oil is extracted from fractional distillation of
181. (2) CH3 – CH = CH2 petroleum. Petroleum has carbon atoms between
C11 – C16
182. (1) Stainless steel is a mixture of iron, nickel, chromium 202. (2) Chemical formula –
and carbon. CuSO4.5H2O (Blue vitriol)
183. (1) Non-metals form acidic oxides. Hence atomic no. 7 NaHCO3 (Baking soda)
184. (2) CnH2n+1 is alkyl group formula. Na2CO3.10H2O (Washing soda)
185. (4) CO2 and water (CO2 is reduced to C6H12O6 and H2O CaSO4.2H2O (Gypsum)
is oxidized to oxygen). Thus baking soda's chemical formula does not show
186. (2) 114 (N2O remains in the atmosphere for 114 years) the presence of water (H2O)
187. (2) The German silver, an alloy, shows composition, O
||
Cu = 60%, Ni = 20% Zn = 20%. 203. (3) H3C CH 2 C OH (Propanoic acid)
EBD_7332
132
B- NTSE Stage 1 Question Bank

204. (2) C2H4O2 (CH3COOH) is ethanoic acid.


CH3COOH + NaHCO3 CH3COONa + H2O + CO2 227. (3) CH 4 O 2 CO 2 H 2 O
205. (1) After combustion, methane forms carbon dioxide and
water.
X Y 228. (3) By applying a coat of zinc, iron frying pan should be
Proton 8 8 prevented from rusting.
206. (3) 229. (3) Rusting takes place in ordinary water as well as distilled
Neutron 8 10 water. It occurs in the presence of water and oxygen.
Atom X and Y show same number of protons and 230. (2) Mg + 2H2O Mg (OH)2 + H2 + Heat
different number of neutrons. On the basis of this When Mg get dissolve in hot water, it forms Mg (OH)2
property, they will show different physical properties. magnesium hydroxide.
207. (1) Atomic number = 20 231. (2) Due to incomplete burning, the vessel gets blackened
from outside.
K L M N 232. (1) Phenophthalein and methyl orange are synthetic
20 = 2, 8, 8, 2 indicator. NaCl solution is a basic solution so, after
208. (1) When the rice straw is burnt then carbon dioxide, addition of phenolphthalein. One part turned pink and
methane and sulphur dioxide gases are produced which another part turned red by addition of methyl orange.
are green-house gases. 233. (2) The cations which have more electronegativity, have
209. (2) A – (iv); B – (i); C – (ii); D – (iii) more covalent character.
210. (1) In the solution of hydrogen fluoride and acetic acid, RbCl < LiCl < MgCl2 < BeCl2
an immersed glass exhibits apparent disappearance. 234. (2) 10 g of H2 contains 5 moles of Hydrogen gas.
211. (2) Increasing order of pH value 64 g of O2 contains 2 moles of Oxygen gas.
Tooth paste blood saliva (after meal) coffee 2H2 + O2 2H2O
tomato Juice If 1 mole of oxygen gives 2 moles of water then, 2 mole
212. (3) 16 g of CH4 = 1 mole of CH4 of O2 give 4 moles of water.
1 CH4 molecule have = 10 e–'s 235. (4) Butane and Isobutane have same chemical formula but
1 mole of CH4 have = 10 6.022 1023 e–'s different arrangement of atoms and have different
= 60.22 1023 e–'s structure.
213. (3) H H H H H H H
Oxidation
H–C–C–C–C–H , H–C–C–C–H
H H H H H H
(+6) (–2) (Butane) H – C – H
(–2) (–1)
PbS + 4 H2O2 PbSO4 + 4 H2O H (Isobutane)
236. (3) Alum contains potassium and aluminium as a two
cations and a sulphate ions. It is obtained by the
Reduction combination of two different salts, which crystallizes
214. (3) As the temperature of water increase, the water to form a single substance.
dissociation constant (Kw) is increases. Hence pH of 237. (1) H – C C – H
the water decreases but water remains neutral. Triple bond is shorter in length than double bond and
215. (2) Oxides of metals are basic and that of non-metals are single bond.
acidic. 238. (4) Aldehydes and ketones are functional isomers of each
Na2O < CaO < ZnO < CO2 other belonging to family of carbonyl compounds with
Strongly Basic Amphoteric Acidic
general molecular fomula CnH2nO.
Basic
239. (1) Other solutions will result in pH greater than 7.
216. (1) CH3COOH is acid whereas HCOOCH3 is an ester. NaCl solution – neutral.
Both have different boiling points. 240. (1) B(OH)3 is boric acid. Also referred as H3BO3
217. (2) 218. (1) 219.(1) 220.(1) 241. (2) It is due to greater screening effect in aluminium, its
221. (1) In homologous series, the members are same in radius is greater than gallium.
chemical properties. 242. (3) Sample contains 3.01 × 1023 MgCO3 molecules.
222. (4) Fe2O3 + 2Al Al2O3 + 2Fe Therefore, 6.022 × 1023 molecules weigh 84 gms
This reaction is an example of displacement reaction.
223. (1) NaHCO3, sodium bicarbonate is commonly called as 84
3.01 × 1023 molecules weigh g
baking soda. 2
224. (3) Antacids like Mg(OH)2 Milk of magnesia are used for = 42 g
treating indigestion. = 0.042 kg
225. (2) Element M have valency 3 and its cations is M3+, thus 243. (2) Na = 2, 8, 1 Na+ = 2, 8
the formula of its nitrate will be M(NO3)3. Cl = 2, 8, 7 Cl– = 2, 8, 8
226. (1) CFC react with ozone (O3) to form dioxygen molecule 244. (1) Other solutions will result in pH greater than 7.
and thus deplete the ozone layer. NaCl solution – neutral.
hv 245. (3) 246. (4)
O3 ¾¾® O2 + O
nascent
oxygen
247. (2) Na 2CO3 (aq) NaOH + H 2 CO3
Strongly Weakly
Basic Acidic
Solved Questions B-133
248. (3) 249. (1)
250. (2) Graphite is used as a lubricant due to layer structure. 1´ 0.0821 L atm mol-1K-1 ´ 298 K
V=
251. (3) H3PO2 is a monobasic acid due to it has only one 0.987 atm
replaceable hydrogen. {1 bar = 0.987 atm}
O V = 24.8 L
265. (2) Methyl isocynate gas (CH3NCO) was leaked from the
union carbide plant of Bhopal (M.P.)
P 266. (1) Oxidation involve the loss of electron(s) and reduction
H involve gain of electron(s)
H OH 267. (1) Ammonium chloride is a sublime substrance hence the
252. (4) Atomic mass of gold (Au) = 197 g (100%) mixture can be separated by sublimation method.
24 carat gold –– 197 g (100%) 268. (3)
20 carat gold –– ? 269. (1) Bromine is found in liquid state.
20 ´197 270. (3) The atomic no. of Al is 2, 8, 3 hence its valency is 3.
mass of gold (Au) = = 164.16 g 271. (1)
24
164.16 wt of metal 60
% of gold present in 20 carat gold = 100 272. (2) Given :
197 wt of metal oxide 100
= 83.33% Let the atomic weight of metal be ‘x’
253. (2) Aldehydes with no -hydrogen atom will undergo x 60
"cannizaro reaction". then,
x 16 100
CHO x = 24, therefore the metal is Magnesium (Mg)
no –hydrogen
Molecular weight of metal oxide
Equivalent weight =
C6H5CHO = Charge on cation
So, Equivalent weight of Magnesium Oxide
254. (1) Atomic Nom – 19 No. of electrons = 19 Molecular weight
No. of protons = 19 Mass No. – 39 =
No. of neutrons = Mass No. – Atomic No. 2
= 39 – 19 = 20 273. (3) Ionization energy decreases from top to bottom in a
255. (1) Mass % of Nitrogen in N2O3 is group due to increase in the size of atom. So increasing
order will be
28 Po < Te < Se < S < O
= 100 = 36.84%
76 274. (3) 236 238 238
90 A, 92 B, 90 C
256. (3) The chloride of the metal M is MCl3 valency of the
metal is 3 A and C are isotopes (As they have same atomic number
Atomic weight = valency Equivalent weight but different atomic weight)
= 3 9 = 27 B and C are isobars (As they have same mass number
257. (3) Atomic weight = valency Equivalent weight but different atomic number)
275. (4) Al2O3 is an amphoteric oxide. Rest of them are basic.
At. wt. 30 276. (2) Na+ = 11 – 1 = 10e– ; Mg2+ = 12 – 2 = 10e–
Valency = = =3 Hence Na+ and Mg2+ are isoelectronic.
Eq. wt. 10
258. (2) According to Dulong and Petit & Law 277. (2) Atomic mass of Mg
At. weight specific heat = 6.4 23.98 78.6 24.98 10.11 25.98 11.29
At. wt. = 6.4/specific heat =
100
6.4 = 18.85 + 2.53 + 2.93 = 24.31
= = 64 278. (3) Salt of a strong acid and a weak base are acidic with pH
0.1
259. (4) Due to highest effective nuclear charge of sodium ion. value less than 7 and salt of a weak acid and strong
260. (2) Ethanethiol or ethylmercaptan base are basic with pH value more than 7.
261. (3) Given 279. (2) Only unsaturated hydrocarbons undergo addition
3 positive charge reaction. Among the given compounds only C3H6
27 mass number (Propene) and C 2 H2 (acetylene) are unsaturated
14 neutron hydrocarbons.
Mass number of any atom 280. (3) Vinegar-acetic acid, orange-citric acid, tamarind-tartaric
= No. of protons + No. of neutrons acid tomato-oxalic acid.
27 = No. of protons + 14 281. (4) The increase in size among F, O and C is due to decrease
No. of protons = 13 nuclear charge without addition of a new shell. Hence
No. of protons in atom = No. of electrons in atom = 13 the correct order is F < O < C.
No. of electrons = 13 – 3 = 10 Further the size of Cl is highest from the given, elements
262. (2) Ethene is used as natural fruit ripening hormone because one extra shell is added to it. Hence the correct
263. (4) Silver combines with SO2 in air to give silver sulphide order is F < O < C < Cl.
2Ag + SO2 Ag2S + O2 282. (3) Mass of a proton is about 1835 times mass of an
264. (3) PV = nRT electron.
EBD_7332
134
B- NTSE Stage 1 Question Bank

me = 9.1 × 10–31 kg ìCH3CH 2 CH 2 CHCH 3 2 - methyl pentane ï


ü
mp = 1.67 × 10–27 kg ï
ï ï
ï
ï | ï
ï
283. (3) Element ‘C’ is fluorine which is the most electronegative ï
ï CH ï
ï
ï 3 ï
element. It form electrovalent bond with metal, by íCH CH CHCH CH 3 - methyl pentane ý
gaining one electron to complete its octet. ï
ï 3 2 2 3 ï
ï
ï
ï | ï
ï
284. (3) Given : m = 0.5 g = 5 × 10–4 kg ï ï
c = 3 × 108 m/s ï
ï
î
CH 3 ï
ï
þ
From Einstein’s mass-energy equivalence relation,
E = mc2 299. (3) Oxygen is highly reactive hence on reaction with
E = 5 × 10–4 × (3 × 108 )2 J = 45 × 1012 J = 4.5 × 1010 kJ oxygen they form their oxide and cannot be used to
285. (2) Na2CO3 + CO2 + H2O 2NaHCO3 extract a metal from its ore.
286. (1) Based on the above 300. (4) Radioactivity is a process in which nuclei of certain
Characteristic given the metal is Cu elements undergo spontaneous disintegration
301. (3) Acetylene reacts with ammoniacal cuprous chloride
2Cu O2 CuO while ethylene has no effect on it.
(X) (Y) 302. (3) Acetylene gas is used as light producing gas in lamps.
Brown and Black
bright An acetylene gas powered lamp produces a
CuO + H2 Cu + H2O surprisingly bright, broad light.
287. (2) A basic solution turns red litmus to blue hence the 303. (4) C6H6 belongs to aromatic hydrocarbons having General
correct answer is 9. Formula Cn H2n–6y (where y: no. of benzene rings, n 6
288. (3) Milk is not considered a pure substance because it )
contains a mixture of different ingrediants. Milk is If n = 6, y = 1 then CnH2n–6y become C6H6
composed of water along with fat and sugar. C2H2 belongs to alkynes having general formula CnH2n – 2
289. (3) Halogens have highest electron gain enthalpy (electron C3H8 belongs to alkanes having general formula CnH2n + 2
affinity). Normally electron affinity decrease as we move C5H12 belongs to alkanes having general formula CnH2n + 2
down the group. However fluorine has lower value than 304. (1) Fluorine is the most electronegative element because
chlorine which is due to its small size. Hence chlorine it contains 5 electrons in its 2p shell. The optimal
has highest electron gain enthalpy. electron configuration of the 2p orbital contains 6
290. (1) Ni(CO)4 has sp3 hybridisation and tetrahedral shape. electrons. So since fluorine is so close to inert gas
291. (2) Graphite is used as electrode. configuration the electron are held lightly to the nucleus.
292. (2) Copper occur in uncombined form as a natural mineral. 305. (4) The electronic configuration of oxygen is 1s22s22p4,
293. (2) Zinc sulphate hepta hydrate is also known as white hence two oxygen atoms can achieve stable structure
vitriol. by sharing two pairs of electrons.
294. (2) H2 + Cl2 2HCl So O2 contains a double bond.
2 1.2 306. (2) Stearic acid acts as surfactant that help to cleans the
0.8 0 2.4 skin.
Here Cl2 is A limiting reagent. After the reaction the 307. (1) The juice obtained from sugar cane is slightly acidic &
composition of the resulting mixture will be 0.8 litre of contain some impurities.
H2 and 2.24 litre of HCl. Ca(OH)2 is added to precipitate the impurities as well
295. (3) On moving along a period acidic strength increase, as to neutralise the juice.
hence the correct order is Al2O3 < SiO2 < P2O3 < SO2 308. (3) Given pH of solution is 8
pH = – log10 [H+] = 8
Mass of metal log10 [H+] = – 8
296. (2) eq.wt.of metal = 8
Mass of oxygen [H+] = 10–8 mol/L
309. (1) On moving left to right in a period, atomic size
1.35 decreases. Thus the increasing order of atomic size for
8 20.32
1.88 1.35 given elements is Na > Mg > Al > Si
Valency of metal in if oxide = 2 310. (1) A redox reaction is one in which simultaneous oxidation
Atomic weight of x = 20.32 × 2 = 40.64 and reduction takes place. In following reaction Cu is
297. (4) Methyl orange show colour change only in acidic reduced and H2 is oxidised.
medium. Since the electrolysis of NaCl produces a basic
solution hence no colour change will observe. Cu 2 O 2 H 02 Cu 0 H 2 2 O 2
298. (3) There are 3 chain and 2 position isomers are possible 311. (2) 3CaO · Al2O3 · 6H2O is called tricalcium aluminate it is
for hexane. one of the components of portland cement. Other major
components of portland cement are calcium silicates,
gypsum etc.
CH3 – CH2 – CH2 – CH2 – CH2 – CH3 n-hexane 312. (2) Rock salt or halite is a mineral form of sodium chloride.
CH3 – CH2 – CH2 – CH – CH3 iso hexane Majorly it is colourless or white in colour.
| 313. (4) Methyl alcohol (CH3 OH) contain hydroxyl (–OH)
CH3 CH3 group as a functional group while acetic acid
| (CH3COOH) contain carboxyl (–COOH) group as a
functional group. Thus due to presence of different
CH3 – CH2 – C – CH3 neo hexane functional groups they possess different physical and
| chemical properties.
CH3
Solved Questions 135
B-

314. (4) TT tt
×
315. (4) In the extraction of iron, the roasted ore is mixed with Parents (Pure Tall) (Pure Dwarf)
with coke and lime stone (flux) and is then heated in
the presence of air at high temperature in a furance. Gametes T T t t
Impurity of SiO2 is removed as calcium silicate (CaSiO3
slag). Cross breeding

CaCO3 CaO CO 2 F1 Generation Tt


Hybrid Tall
CaO SiO2 CaSiO3 Self polination
impurity slag Gametes T t
316. (4) Ammonia shows intermolecular hydrogen bonding.
H
+
H
+
H
+ T t
TT Tt Pure Tall 1
– – – F2 Generation T Pure Tall Hybrid Tall
N H N H N H Tt tt Hybrid Tall 2
t Hybrid Tall Pure Dwarf Pure Dwarf 1

+H H + +H
F2 generation ratios : –1 : 2 : 1
317. (3) Colloidal solutions are heterogeneous system
consisting of two phases. So, A is dwarf pea plant while B is tall pea plant.
11. (1) Good ozone is found in the upper part of the atmosphere
SECTION 3. BIOLOGY called stratosphere and it acts as a shield absorbing
1. (1) Thyroid Stimulating Hormone (TSH), is produced in ultraviolet radiation from the Sun.
the anterior pituitary gland that promotes bone growth. 12. (4)
Ovaries produce and release egg in the female 13. (4) Opening of guard cells is facilitated by the entry of
reproductive tract at the mid (14th day) of every water inside guard cells. This makes the guard cell
menstrual cycle. Adrenal gland causes secure high turgid. Whereas closing of guard cells is facilitated by
blood pressure results in increase heart rate. Testis is water coming out of guard cells. This will make the
responsible for development of male sex organs. guard cells flaccid.
2. (1) Abscisic acid helps in ripening of fruits and its 14. (2)
development. 15. (1) Diaphragms, cerevical caps and vaults are barrier
3. (1) Pyrethroid is an organic compound which are produced method of contraception that are made of rubber and
by the flowers of pyrethrums (chrysanthemum are inserted into the female reproductive tract to cover
cinerariaefolium and chrysanthemum coccineum). the cervix during coitus.
4. (2) Plasma membrane of a cell is made up of protein and 16. (1) Sperms are produced in the seminiferous tubules. As,
lipids. It acts as a semipermeable membrane that allows seminiferous tubules is lined on inside by two types of
the exchange of materials between cells and its cells called male germ cells and sertoli cells. The male
surrounding. germ cell undergo meiotic divisions finally leading to
5. (3) When the blood flows through a wound, the platelets sperm formation, while sertoli cells provide nutrition to
release an enzyme called Thrombokinase to stop the the germ cell.
blood flow. This enzyme facilitates the conversion of 17. (4) The villi and microvilli present in the small intestine
prothrombin to thrombin results is coagulation of increase the surface area for the absorption of digested
blood. food.
6. (4) Tuberculosis is caused by bacterium. Mycobacterium 18. (1) During excessive and rigorous exercise, the muscle
tuberculosis that affects the lungs. cells shifts from aerobic respiration to lactic acid
7. (4) While auxin, florigen and cytokinin are plant hormones, fermentation due to limited supply of oxygen. In lactic
Oxytocin is an animal hormone and act as neuro- acid fermentation, the pyruvate produced during
transmitter that is involved in child birth and breast glycolysis is converted into lactic acid with the help of
feeding. the enzyme lactate dehydrogenase.
8. (2) Carbohydrates and fats are high energy rich food. The 19. (3) The adrenal gland produces the hormone adrenalin and
glucose from carbohydrates is converted into energy similarly pancreas produces the hormone insulin.
which is required by brain and muscles. 20. (2) Amoeba undergoes binary fission, yeast undergoes
Fat is required by the body for development of brain, budding, Leishmania undergoes longitudinal binary
controlling inflammation and blood clotting. fusion and Bryophyllum undergoes vegetative
9. (3) Electricity is produced by humans, so it is not a natural propagation by leaves.
resource. 21. (1) Octopus is most clever animal among all non-chardates
10. (3) Pure breed plant A × Pure breed plant B and which can change its colour. Due to the presence
F1 Generation : Only B type plant of chromatophare cell, they are able to change its skin
colour.
Means A is recessive trait, which re-emerges in F2
generation. 22. (4) Rifamycin is used for the treatment of tuberculosis as
it is effective antibiotics against tuberculosis.
23. (4) Glucose and fructose syrup can be obtained from
carnflour by action of enzymes obtained from Bacillus
and Streptomyces.
EBD_7332
136
B- NTSE Stage 1 Question Bank

24. (3) Consumption of tobacco product lead to cancer of 38. (3) Cnidocytes, also known as stinging cells, are specialized
lungs by damaging the cells that lines the lungs. neural cells that typify the phylum cnidaria (sea anemones,
25. (2) Pneumonia is a communicable disease which is caused eorals, hydroids, and jellyfish) which often contains
by bacteria called streptococcus pneumoniae and poison, is to ward off enemies or to capture prey.
Mycoplasma pneumoniae. 39. (1) Liver is the largest gland of body which produces bile
26. (2) Chitineous cell wall is found in fungi. Fungi are juice which is stored into the gallbladder for
eukaryotes that lack chlorophyll and are heterotropic. concentration, storage, or transport into the first region
The cell wall of fungi is rigid as it is composed of of the small intestine, the duodenum. Bile juice is
nitrogen - containing polysaccharide called chitin. It is responsible for emulsification of fat.
also composed of branched and filamentous hyphae, 40. (3) Glycolysis is the first stage of aerobic and anaerobic
which form a net like structure known as mycelium. respiration where one molecule of glucose is converted
27. (1) Lack of oxygen occurs when algal bloom is formed in a into two molecules of pyruvate in the cytoplasm of cell.
water body. Algal bloom is formed because of the 41. (1) Auxin is a plant hormone produced in the stem tip that
presence of large amounts of nutrients in water results promotes cell elongation. Auxine promote stem
in excessive growth of planktonic or free floating algae. elongation inhibit growth of lateral buds & maintains
It causes deterioration of the water quality and fish apical dominance.
mortality. 42. (2) Cholera is an acute diarrheal infection caused by
28. (2) Lichens are symbiotic association between algae and ingestion of food or water contaminated with the
fungi. The algal component is known as phycobiont bacterium Vibrio cholerae.
and fungal component as mycobiont, that are 43. (4) Blind spot of eye lacks receptor cells (rod & cones)
autotrophic and heterotrophic. Lichens are very good because the optic nerves leave the eye ball from here,
pollution indicators as they do not grow in polluted so no image is formed in this part of the eye.
areas. 44. (4) Tetanus is acquired through infection of a cut or wound
29. (2) Earthworm is not a member of phylum Arthropod as it with the spores of bacterium Clostridiumtetani. This
belongs phylum Annelida. disease affects the nervous system results in painful
30. (4) DNA synthesise takes place in S-phase or synthesise muscle contraction.
phase. In this phase of cell cycle, DNA replication 45. (2) Lysosome stores large number of hydrolytic enzymes,
begins in the nucleus and the centriole duplicates in which are also known as acid hydrolases they are also
the cytoplasm. known as suicide bags of the cell because they have
31. (3) Fats are broken down by lipases with the help of bile capability of digesting cells and unwanted materials.
into di- and monoglycerides. 46. (3) Cardiac muscles are striated as they have light & dark
32. (1) Julius Van Sachs was a German Botanist, for the first bands. These muscles are involuntary in nature and
time plants could be grown to maturity in a defined are located in the wall of the heart.
nutrient solution in the complete absence of soil. This 47. (2) Oesophagus is the part of human alimentary canal where
technique of growing plants in a nutrient solution is digestive glands are absent, only mucus glands are
known as Hydroponics. present.
33. (3) Stele is the central part of the stem inner to endodermis. 48. (1) lgG is the only antibody that significantly crosses the
It consist pericycle, vascular bundle, pith. Cambium is human placenta.
a tissue layer that provides partially undifferentiated
cells for plant growth and is found in the area between 49. (3) The phenomenon of accumulation of non-
xylem and phloem. biodegradable chemicals, e.g., DDT, in a food chain
34. (2) Skeletal muscle tissue is attached to bones and its at each trophic level is called biomagnifaction.
contraction results in the locomotion, facial expressions, 50. (4) The pathway involved in reflex action from stimulus
posture, and other voluntary movements of the body. to response is called reflex arc. The stimulus is
35. (3) Statements (a) and (b) are correct. received by a receptor and passed on to the sensory
Statements (c) and (d) are false. neuron and then to relay neuron. The impulse is
As dominant factor is expressed only in F1 generation transmitted to motor neuron which carries it to the
of a monohybrid cross. effector organ.
Gregor Mendel conducted his experiment on pea plant. 51. (1) Tricuspid valve is present between right atria and
36. (1) The RBC are devoid of nucleus, mitochondria ribosome right ventricle.
and endoplasmic reticulum. The absence of these
organelles accommodates more haemoglobin thereby 52. (2) During night, root pressure is effectively involved in
maximising the oxygen carrying capacity of the cell. transport of water through xylem. During day, due to
Biconcave shape increase the surface area for oxygen the opening of stomata, transpiration pull becomes
binding, loss of mitochondria allow the RBC to the major factor for transporting water in xylem.
transport all the oxygen to tissues and loss of 53. (3) Respiration involves
endoplasmic reticulum allows more flexibility for RBC (a) Oxidation of glucose
to move through the narrow capillaries. (b) Reduction of O2
37. (1) Mitochondria are membrane-bond cell organelles found 54. (4) This is the case of anaerobic respiration, which takes
in the cytoplasm of all eukaryotic cell that generate the place during vigorous muscular activity. During this
cells primary energy molecule ATP Mitochondria are process lactic acid is produced which gets
known as the power house of the cell because it is accumuleted in muscles & leads to cramps.
responsible for extracting energy from food through
cellular respiration.
Solved Questions B-137
55. (1) Complete separation of oxygenated and deoxygenated 77. (1) Insulin helps in maintaining blood glucose level by
blood is a feature of warm-blooded animals; their heart promoting glucose utilization by the body cells.
has 4 chambers which ensure the effective transfer of Thyroxine play a key role in regulation of the basal
oxygen in the body and also meet the high energy metobolic rate. ADH, also known as vasopressin,
requir ement to maintain their constant body stimulates the reabsorption of water and electrolytes
temperature. by kidney. Oxytocin involves in the secretion of milk
56. (4) 57. (1) 58. (1) 59. (2) from mammary glands.
60. (2) 61. (4) 62. (2) 78. (4) The sperm releases from the testis, enters into
epididymis which leads to vas deferens. Then sperms
63. (4) Vas deferens is a part of male reproductive system. are transferred into the urethra.
64. (3) Kala azar (Black fever) is caused by protozoan 79. (1) Coralloid roots contain nitrogen fixing symbiotic
Leishmania genus. cyanobacteria. They are present in Cycas and helps
65. (4) The biotic components of ecosystem includes in fixing nitrogen.
producers, consumers and decomposers. 80. (1) Chipko movement was an environmental movement
66. (4) Lysosomes contains various types of hydrolytic aimed at protection and conservation of trees. It was
enzymes which are capable of digesting started from Jodhpur in Rajasthan.
carbohydrates, proteins, nucleic acids and lipids. 81. (2) The large intestine consists of three parts namely
Therefore, they are known as the ‘suicide bags’ of caecum, colon and rectum.
the cell. 82. (1) Vitamin K plays a key role in the clotting of blood
67. (4) Plasma membrane, cytoplasm and ribosome are and thus prevents excessive bleeding. Its deficiency
present in a plant cell, animal cell and bacterial cell. cause haemorrhage.
68. (1) The swim bladder or air bladder is a gas - filled 83. (1) Saliva in our mouth secretes a carbohydrate splitting
internal organ which regulates buoyancy and prevents enzyme, salivary amylase. Rice is a rich source of
the sinking of most fishes. complex carbohydrates such as starch which breaks
69. (3) down by the hydrolytic action of salivary amylase
and thus aids in digestion.
Parents: XhY × XX
Starch Salivary amylase Maltose
Haemophilic man Normal female 84. (3) Organs which are similar in function but differ in origin
and also have no common ancestry are known as
Xh Y analogous organs, for example, wings of butterfly and
birds perform similar functions but their origin is
X XhX XY different.
X XhX XY 85. (4) When preformed antibodies are directly given to
provide immediate protection and quick immune
The probability of their son being haemophilic is nil. response against foreign agents, it is called passive
70. (4) Decomposers are present at the final level in a food immunity.
web. They breakdown dead and decaying organic 86. (1) Pseudopodia are cytoplasmic projections found in
matter (plants and animals) and convert into nutrients protozoans like Amoeba. They are able to change their
in the soil. They naturally increase the decomposition form and thus help in movement of protozoans.
process and therefore used in natural biocomposting.
71. (1) If chest cavity is considered as a room, then 87. (4) Leishmania is a protozoan and it has flagella.
diaphragm is the floor. Lungs are the windows that 88. (1) Since the wings of housefly and wings of sparrow have
will receive the air we breathe in, clavicle is the roof same function but dissimilar structure, they would be
and ribs are the walls, surrounding the chest cavity. analogus organs.
72. (4) A biennial plant completes its life cycle over two
89. (4) Cytokinin is a growth promoter hormone which promotes
growing seasons. Pea plant is an annual plant.
73. (1) Each scientific name has two components-the generic division of cell in plants.
name, represents genus and the specific epithet, 90. (1) Adrenaline is an emergency hormone which is secreted
represents species. In Homo sapiens, ‘sapiens’ from adrenal gland during a fight or flight.
represents species as it is the second component. 91. (1) ATP is formed by addition of phosphate to ADP in the
74. (3) Saliva and gums are the secretory materials of animals
and plants respectively. Tears are excreted by animals presence of light during photosynthesis. This ATP
while fallings of leaves are the excretory product of formation by a photosynthesizing plant cell would be
plants. known as photophosphorylation.
75. (3) The wall of arteries is thick and muscular. 92. (4) Medulla oblonata is the part of human brain responsible
Veins collapse when they are empty.
Hence, statements (A) and (D) are incorrect and for controlling the rate of breathing in humans.
statements (B) and (C) are correct. 93. (2) Pollen tube carries two male gamete inside ovule.
76. (4) Auxins are synthesized in the growing tips of stems 94. (3) Tuberculosis is a bacterial disease caused by
and roots. When light is coming from one side of the Mycobacterium tuberculosis
plant, auxin flow towards the shady side. The
increased concentration of auxin allows the cells to 95. (3) Ovary is a primary reproductive organ.
grow longer on the shaded side. Hence, statements 96. (2) Cardiac (heart) muscles are cylindrical and inter
(B) and (D) are related to the picture. connected, smooth muscles are spindle in shape, whereas
skeletal muscles are cylindrical with no branching.
EBD_7332
138
B- NTSE Stage 1 Question Bank

97. (2) Potassium h ydroxide absorbes CO 2 . Therefore, 117. (1) Radish is actually root part of plant which
photosynthesis will not occur due to absence of CO2. stores food-Tomato is fruit part-Potato is underground
98. (4) Sex determination in humans is due to the two types of stem which stores food.
chromosomes (X & Y) in males. 118. (4) Acid is produced in stomach for digestion of food.
99. (2) Lizard belongs to class Reptilia and the respiratory organ Sometimes extra amount of acid is produced which causes
in reptiles is lungs. burning sensation. Baking soda is basic in nature
100. (4) The whole process of anaerobic respiration takes place neutralizes acid on contact so it would help to relieve
in cytoplasm, i.e, glycolysis and fermentation (Lactic acid burning sensation due to acidity.
fermentation or alcoholic fermentation). 119. (2) Chromosomes carry genes, which are the hereditary
101. (2) In the villi of small intestine, lymph capillaries (Lacteals) units, which carry hereditary characters to the off-
absorbs the lipids (fats) and conduct it to the blood spring.
circulation. 120. (3) MItochondria is called the power house of the cell.
102. (3) Excessives secretion of growth hormone in adults causes 121. (4) Plasma membrane is made up of both protein and lipid.
enlargement of the body and give gorilla like appearance. 122. (2) Oviduct is the site of fertilization in humans.
In children, the condition is called gigantism. In adult, it 123. (1) Heart never takes rest as it has cardiac muscles which
is called acromegaly. never gets fatigue.
103. (2) Virus: As they donot have their own cellular apparatus, 124. (1) Lacteal present in the villi of the small intestine help to
they don’t have a metabolism on their own, they can’t absorb fatty acids and glycerol.
125. (1) The experiment ‘origin of primitive life on Earth’ was
reproduce without a host cell.
performed by Urey and Miller.
104. (2) Symbiotic association of algae and fungi is lichen
126. (2) Bicuspid valve is present in the human heart in be-
105. (3) ATP ADP iP
tween left atrium and left ventricle.
106. (4) The recessive character in pea plant is green seed. Rest
127. (3) During the light reaction NADPH and ATP are synthe-
of them are dominant character.
sized which are utillzed in dark reaction.
107. (4) Haemophilia is more common in males because it is a 128. (4) Grafting in monocot plants is not possible because
recessive trait carried by X chromosome. they have scattered Vascular Bundles.
108. (4) When haemoglobin combines with carbon monoxide it 129. (1) Haemophilia disease is linked with sex chromosome.
forms carboxy haemoglobin and when haemoglobin 130. (3) The primary building blocks of DNA Nitrogenous base,
combines with oxygen it forms oxyhaemoglobin. phosphorus and deoxyribose.
109. (1) Transcription is the first step of gene expression, in which 131. (1) Islets of Langerhans helps in formation of insulin
a particular segment of DNA is copied into RNA 132. (3) Sclerenchyma is simple permanent tissue in which lig-
(especially mRNA) by the enzyme RNA polymerase. nin is present which makes it dead & it provides me-
110. (1) Outermost whorls is calyx, followed by corolla, chanical strength as in the fibrous covering of coco-
androecium and gynoecium is at centre. nut.
111. (1) Carbohydrates are broken by glycolysis into pyruvic 133. (3) Nucleus is the headquarter of the cell which control
acid which converts into Acetyl COA which then enters cell and discovered by Robert Brown.
krebs cycle to produce CO2 and H2O and energy. 134. (3) Cytokinin is a plant hormone which mainly control cell
112. (2) Water is the source of oxygen during photosynthesis division & promote growth in plants.
process. Chlorophyll is the pigment which absorbs far 135. (2) As Lichens are SO2 sensititive and can’t grow in the
light. This solar energy is used to break water molecule area where sulphur dioxide pollution is present.
into oxygen and H+. This process is called as photolysis 136. (3) Cycas & Pinus are gymnosperms which are perennial,
which occurs in light reaction of photosynthesis. evergreen & woody.
113. (2) Hydathodes are modified pores, especially on a leaf, 137. (1) Vacuole maintains the osmotic pressure in plants &
which exudes drops of water. maintain turgidity.
138. (3) Sunlight, chlorophyll, CO2 are the main elements es-
114. (2) Concentration of acid is very high in digestive fluids in
sential for photosynthesis.
stomach therefore, approximate pH is 2.
139. (3) The nature of nerve impulse is both electrical & chemi-
115. (4) Tendons & ligaments are made up of white and yellow
cal, so eletrochemical.
fibres.
140. (4)
116. (3) Diabetes is caused due to hyposecretion of insulin
141. (4) Monohybrid Cross
hormone. Arthritis is caused by normal wear and tear of
joints. Cataract is clouding of the clear lens of the eye
with aging. Jaundice is caused by infection.
Solved Questions B-139
158. (2) Estrogen is secreted by ovary
Parents - Tall dwarf 159. (4) 1, 2 and 3 are pteridophytes and 4 is bryophyte
TT tt 160. (4) Psammophytes is a plant that grows in shifting sands,
primarily in deserts. They are marked by a number of
Gametes - T T t t adaptations which enable them to exist on wind-blown
sands for e.g. Haloxylon persicum, Calligonum,
Ammondendron, Eremosparton, Smirnowi.
° 161. (2) In the human skin, melanogenesis is initiated by
° T T exposure to UV radiation, causing the skin to darken.
t Tt Tt Melanin is an effective absorbent of light due to which
t Tt Tt it is thought to protect skin cells from UVB radiation
damage i.e. reducing the risk of cancer.
F1 generation - Tt (tall) 162. (2) Binomial nomenclature is a system of providing a name
with two components. This naming system given by
selfing Carolus Linnaeus is being practised by biologists all
over the world. The first name represents the genus
Tt Tt while second name represents the species name. For
Gametes - T t T t e.g. the scientific name of mango is written as Mangifera
indica.
163. (3) Ozone is located in upper part of the atmosphere called
° T t stratosphere and it acts as shield absorbing UV radia-
°
T TT Tt tion from sun. Ozone-depleting substances: CFCs,
HCFCs, halons, etc result in depletion of ozone & for-
t Tt tt mation of a large area of thinned ozone layer, com-
monly called as ozone hole.
164. (2) The process of copying genetic information from one
F2 generation-Phenotypic ratio - 3 : 1 strand of the DNA into RNA is termed as transcription.
tall : dwarf It takes place in nucleus with the help of RNA poly-
Genotypic ratio - 1 : 2 : 1 merase.
TT : Tt : tt 165. (2)
142. (1) Cartilage is skeletal connective tissue which forms en- 166. (2) Insectivorous plants are plants that derive some or
doskeleton of human body. most of their nutrients from trapping and consuming
143. (4) Echidna is a mammal & lay eggs & act as a connecting animals or protozoans, typically insects and other
link between reptiles & mammals. arthropods. They have adapted to grow in places where
144. (1) the soil is thin or poor in nutrients, especially nitrogen.
145. (3) 167. (2) Spleen is a secondary lymphoid organ where prolifera-
146. (4) CO2 and water (CO2 is reduced to C6H12O6 and H2O tion of lymphocytes takes place.
is oxidized to oxygen) 168. (4) Tobacco Mosaic Virus (TMV) is single stranded RNA
147. (4) Testis is male reproductive organ while others are fe- virus.
male. 169. (2) RNA was the first genetic material. There is now enough
148. (1) 50 decibel (in day time, 40 decibel at night in silent evidence to suggest that essential life processes (such
zone) as metabolism, translation, splicing, etc.) evolved
149. (2) 114 (N2O remains in the atmosphere for 114 years) around RNA. RNA used to act as a genetic material as
150. (3) Two (Deoxygenated blood enters the heart and then well as a catalyst. But, RNA being a catalyst was reac-
lungs, oxygenated blood enters the heart and pumped tive and hence unstable. Therefore, DNA has evolved
to other parts of the body. from RNA with chemical modifications that make it more
151. (4) Seismonastic movement (Nastic movement, stable.
nondirectional movement towards direction of touch) 170. (3) One of the two male gametes fuses with two polar nu-
152. (2) Regeneration is not truly a reproductive process, rather clei located in the central cell to produce a triploid pri-
it’s a process of renewal, restoration and growth in mary endosperm nucleus (PEN). PEN becomes primary
organisms. endosperm cell (PEC) & develops into endosperm.
153. (2) 23 pairs of chromosomes (22 pairs of autosome and 171. (4)
one pair of allosome) 172. (4) The noosphere is the sphere of human thought. The
154. (1) Darwin explained natural selection, Lamarck explained word derives from the Greek words (nous "mind") and
inheritance of acquired characters and Mendel is (sphaira "sphere"), in lexical analogy to "atmosphere"
known for pioneering work in inheritance. and "biosphere".
155. (4) 1, 2 and 3 belongs to Thallophyta and 4 belongs to 173. (3) The excretory organs of earthworm are minute, coiled
Bryophyta tube like structures called as nephridia.
156. (2) Raphide crystals are sharp needle like crystals of 174. (2) Plasmid” is made up only of one type of macro
calcium oxalate that dart and cause discomfort to molecule.
throat, activates inflammatory reaction by 175. (2) The relative energy yield in Kcal/gm is best represented
production of histamines. in first ATP, followed Lipid and protein.
157. (3) Area A is for perception of touch, pain etc. 176. (4) The sub units of ribosomes in the cells of nephron of
Area B is for perception of sound mouse is 60s & 40s.
Area C is occipital lobe of brain for visual perception 177. (3) Involuntary muscles are not found in “tongue”
Area D is for thinking, intelligence etc.
EBD_7332
140
B- NTSE Stage 1 Question Bank

178. (1) Only “denitrifying bacteria” work strictly under anaero- 9


bic conditions. 320 = 180 (yellow & round seeds)
179. (1) The correct sequence of the given figure is (iii), (ii), 16
(iv), (i) 205. (4) Earthworm (Lumbricus terrestris) is a beneficial
180. (2) Mylein sheath is not present in grey neurons. (non organism for farmers. It belongs to phylum-Annelida.
myleinated neuron) 206. (1) Epithelial tissues are associated with secretion,
181. (3) The incorrect statement is they convert water and CO2 selective absorption, protection, transcellular
into carbohydrate only in the absence of light)” transport and sensing activities.
182. (1) The correct pathway of blood in circulatory system is 207. (3) Colour blindness is more common in males than
Atria Ventricles Artery Vein females, the reason is that the genes for red and green
183. (4) “Iodine” is essential for formation of thryoxine n thy- colour receptor, are situated on the X chromosomes,
roid gland. males have one x and females have two x.
184. (2) According to Lindemauns’ 10% low of energy transfer, 208. (1) Black gram requires minimum quantity of urea for its
only 10% of the energy is transferred from one tropic growth. The reason is that leguminous plants have
level to heat. root nodules providing shelter for symbiotic nitrogen-
185. (3) Both similarities and variation are transmitted from par- fixing bacteria which convert nitrogen gas from the
ents to offspring during reproduction. atmosphere into NH3 doing formation of amino acids
186. (3) During excess of physical exercise, lactic acid gets ac- and nucleotides.
cumulated in the muscles which causes pain. 209. (2)
187. (2) A good food chain includes Grass, Goat, Lion 210. (3) Enzymes are proteins having linear chain of amino
188. (1) The Oxygen rich blood from lungs comes to the heart acids.
in left atrium, the left chamber of the heart. 211. (2) Pepsin converts proteins into proteoses & peptones
189. (4) Growth of pollen tube in the style towards the ovule in (peptides). Rennin helps in the digestion of milk
plants is an example of chemotropism which is a proteins in infants. Later, pancreatic juices (trypsin
movement of plant part in response to a chemical stimulus. chymotrypsin & carboxypeptidase) digest proteins,
190. (4) Urethra in human male, is a common passage of urine peptones & proteoses into dipeptides.
and sperm. The urethra is held closed by the urethral 212. (3) The proteins are synthesized with the help of ribosome
sphincter, admuscular structure that helps keep urine on RER and lipids are synthesized by SER. Post
in the bladder. translational modifications take place in golgi body
191. (1) Pepsin gets active in acidic medium created by which are then sent to their respective destinations
hydrochloric acid (HCl). Pepsin is a conversion of with the help of vesicles.
pepsinogen which digests proteins breaking down into 213. (1) The cerebellum located at the back of the head
peptides. coordinates voluntary muscle movements and helps
192. (2) Bowman capsule is found in kidneys. It takes part in in maintaining posture, balance, and equilibrium.
filtration of blood. It is a cup like structure. 214. (1) Organisms which excrete uric acid as the main
193. (4) DNA is found in both mitochondria and nucleus. excretory product are termed as uricotelic for e.g.
194. (3) The plants of Bryophyta are called amphibious plants reptiles, birds, land snails and insects.
because their habitats are concerned with aquatic and 215. (1) Mushroom exhibits saprophytic mode of nutrition
terrestrial region. while others (ticks, tape worms, Cuscuta exhibit
195. (2) Protein is digested with the help of enzyme pepsin. parasitic mode of nutrition).
This process occurs in stomach. It happens in acidic 216. (3)
medium. 217. (1) Auxins promotes the stem elongation in plants.
196. (1) Gaseous hormone, Ethylene (C2H4) is found in plants. 218. (1) Ginger is a root, which is used as spice.
It causes fruit ripening. 219. (2) Food is stored in the form of glycogen in liver.
197. (4) Calciferol (Vitamin D) is helpful in the development of 220. (2) Cytokinine is a plant hormone, which initiates organ
bone. formation in variety of tissue culture.
198. (1) Bile does not contain any digestive enzyme. It has 221. (2) Olfactory lobes have speciallized sensory cells of the
bile pigments - bilirubin and biliverdin. It makes the nasal cavity in vertebrates.
medium alkaline through NaHCO3. 222. (3) Plastid is a double-membrane organelle found in plants,
199. (1) Spongy parenchyma is responsible for storage of algae, and some other eukaryotic organisms. They are
starch in plant leaves. The spongy parenchyma has of different types such as chloroplasts, chromoplasts,
many spaces between cells to facilitate the exchange gerontoplasts, & leucoplasts (used for storing starch
of gases. and fats).
200. (3) Sclerenchyma contains stone cells having hardness. It 223. (1) The term 'auxin' is applied to the indole-3-acetic acid
protects the plant from outside. (IAA), and to other natural and synthetic compounds
201. (4) Homo sapiens is the scientific name of man. This is a having certain growth regulating properties. It is
modern man. produced by the growing apices of the stems & roots,
202. (4) Ammonia (NH3), urea, uric acid are harmful products from where they migrate to the regions of their action.
of bio-chemical reactions but lymph is not produced 224. (3) Agricultural & industrial wastes are rich in nitrates &
biochemically. phosphates which over-stimulate the growth of algae
203. (1) Lungfish is an organism which is a connecting link (algal bloom) & other plants, causing unsightly scum
between pisces and amphibian. and unpleasant odors, and robbing the water of
204. (2) The ratio of F2 generation = 9 : 3 : 3 : 1 dissolved oxygen vital to other aquatic life.
Solved Questions B-141
225. (4) It is a liver disease caused by hepatitis E virus. It is 246. (1) A nerve impulse is transmitted from one neuron to
mainly transmitted through drinking water another through junctions called synapses. A synapse
contaminated with faecal matter. Symptoms include is formed by the membranes of pre-synaptic neuron &
jaundice, lack of appetite & nausea. Hepatitis E usually post-synaptic neuron, which may or may not be
resolves on its own within 4-6 weeks. Treatment focuses separated by a gap called synaptic cleft. Chemicals
on supportive care, rehydration and rest. called neurotransmitters transmit from axonal end of
226. (1) When a plant cell is placed in hypertonic solution, one (pre-synaptic) neuron to the dendritic end of other
exosmosis will take place that will result in plasmolysis. (post-synaptic) neuron.
227. (1) Phototropic and geotropic movements of a plant 247. (2) Ribosomes are also known as ‘Protein Factories of Cell’
depend upon different concentrations of auxin. Mitochondria is responsible for Respiration.
228. (4) Pulmonary Vein carries oxygenated blood from the Chloroplast helps in photosynthesis
lungs to the left auricle of the heart. Smooth Endoplasmic Reticulum helps in fats synthesis
229. (3) In animal cells like muscles during exercise, when 248. (4) Carolus Linnaeus is also known as ‘Father of Binomial
oxygen is inadequate for cellular respiration pyruvic Nomenclature’.
acid does not enter the Krebs cycle and gets reduced The term ‘species’ was coined by John Ray.
to lactic acid by lactate dehydrogenase in cytoplasm. Father of Biology is Aristotle.
230. (1) Cutin, a waxy layer prevents transpiration from aerial A.P. Decandolle introduced the term ‘taxonomy’.
parts of xerophytes. 249. (2) The O2 present in CO2 molecule is used in the
231. (3) Malaria is caused by Plasmodium sp.
formation of glucose molecule during calvin Benson
232. (4) The increase in concentration of non-biodegradable
substance with subsequent trophic level of a food cycle of photosynthesis
chain is called bio-magnification. 250. (3) The percentage of oxygen in air is about 21% by volume.
233. (4) Kidneys are not involved in secretion of antibodies. 251. (2) Animal cells lacks plastid but it is present in plant cell.
234. (1) Phototropic and geotropic movements of a plant 252. (4) Blood is a type of fluid connective tissue, which
depend upon different concentrations of auxin. contains plasma and blood corpuscles (RBC, WBC and
235. (2) Lower invertebrates (such as sponges, flatworms, etc) platelets).
exchange gases by simple diffusion over their entire 253. (4) Bile juice are stored in gall bladder, which is secreted
body surface. Earthworms use moist cuticle & insects by liver during digestion.
have network of tubes to transport atmospheric air 254. (1) Sodium bicarbonate is taken as medicine, when acidity
within the body. Aquatic arthropods, mollusks & fishes increases in stomach.
use gills Reptiles, birds & mammals respire through 255. (2) Planaria belongs to phylum platyhelminthus.
lungs. Amphibians like frogs can respire through their 256. (1) Mosses, liverwort and hornwort are bryophytes.
moist skin also. 257. (2) Aves are oviparous whereas mammals are mostly
236. (2) Photosynthesis is an enzyme regulated anabolic viviparous. But both consists 4-chambered heart,
process of manufacture of organic compounds inside generally warm blooded and also respire through
the chlorophyll containing cells (chloroplast) from CO2 lungs.
& H2O with the help of sunlight. It can be divided in 258. (4) CO2, H2O, chlorophyll and light are necessary for the
two phases: photochemical phase & biosynthetic process of photosynthesis which occurs in
phase. autotrophs.
237. (1) 259. (3) Sphygmomanometer is an instrument, which measures
238. (3) Uracil is present in RNA. blood pressure.
239. (2) Igneous rocks form from molten rock, and rarely have 260. (4) Analogous organs are similar in function but are
fossils in them. Metamorphic rocks have been put under anatomically different and unrelated. Hence, Analogous
great pressure, heated, squashed or stretched, and structures are a result of convergent evolution different
fossils do not usually survive these extreme conditions. structures evolving for the same function and hence
Generally it is only sedimentary rocks that contain having similarity.
fossils. 261. (3) A nerve impulse is transmitted from one neuron to
240. (1) Changes in blood volume, body fluid volume and ionic another through junctions called synapses.
concentration activate osmoreceptors which stimulate 262. (3) Build up of lactic acid (CH3CHOH. COOH) in muscle
the hypothalamus to release antidiuretic hormone cells when oxygen utilisation is faster than its
(ADH) or vasopressin from the pituitary gland. They availability as during vigorous exercise leads to fatigue
regulate water re-absorption to maintain blood volume and results in cramps.
etc. 263. (2) Pepsin is major enzyme of gastric juice secreted from
241. (4) Calvin cycle uses NADPH not NADH. the lining of stomach and functions in acidic medium
242. (4) Lysosome carries the hydrolytic enzymes. (pH 2.0 – 3.5).
243. (2) Insulin regulates the blood sugar level by converting 264. (3) Malaria is a communicable (protozoan) disease caused
glucose into glycogen. It is secreted by pancreas and by Plasmodium vivax while others are congenital
its deficiency leads to diabetes. (genetic) disorders. Plasmodium enters the human
244. (4) Dog fish (Soliodon) belongs to class pisces. Silverfish body as sporozoites (infections form) through the bite
belongs to phylum arthropoda, Jellyfish or sea jelly of infected female Anopheles mosquito.
belongs to phylum cnidaria, and Starfish or sea stars 265. (3) Arthropods have body cavities (coelomates), with main
belongs to phylum echinodermata. internal cavity called haemocoel which accomodates
245. (4) Nitrogen fixing bacteria converts atmospheric N2 into their internal organs and through which blood
nitrite and nitrate and makes it available for the plants circulates. They have open circulatory system.
EBD_7332
142
B- NTSE Stage 1 Question Bank

266. (1) The endoplasmic reticulum bearing ribosomes on their 283. (1) Sahiwal is not a foreign breed cow. It is originated from
surface called rough endoplasmic reticlum (RER) which the Sahiwal district of Pakistan, a region in Punjab
are actively involved in protein synthesis. The smooth provience of Pakistan.
endoplasmic retriculum (SER) is the major site for 284. (3) Stomata facilitate the process of transpiration by which
synthesis of lipid. In animal cells, steroidal hormones water evaporates from the surface of the plant. This
syntherized in SER. helps the plant to get cool and also helps in the transfer
267. (3) The amount of the filtrate formed by the kidney per of minerals. As the plant takes water from the soil, the
minute is called glomerular filtration rate (GFR). In a openings absorb other minerals.
healthy individual it is approximately 125 ml/minute i.e., 285. (3) Chromosome consists of DNA, some basic proteins
180 litres per day. called histones, some non-histone proteins and also
268. (1) In Mammals, the heart is always 4-chambered (2 atria + RNA.
2 ventricle). Right atrium receives deoxygenated blood 286. (3) Abscisic acid is anti-transpirant hormone. It stimulates
through anterior vena cava and posterior vena cava. the closure of stomata in the epidermis and increases
269. (3) Auxin responsible for initiating and promoting cell the tolerance of plants to various kind of stresses. It is
division in certain tissues. The most striking effect of also called stress hormone.
gibberellin is on the elongation of stem. Cytokinins 287. (3) Ammonia is highly toxic. It must be removed
promotes cell division. Abscisic acid acts as growth immediately from the body. Excretion of ammonia takes
inhibitor in plants. place with the help of water. Kidneys do not play any
270. (3) Stanley L.Miller and Harold C. Urey performed significant role in elimination of ammonia.
experiment by using ‘Spark discharge apparatus’. They 288. (4) The yellow fever virus is transmitted by the bite of
used the mixture of water vapours (H2O), hydrogen female mosquitoes (the yellow fever mosquito, Aedes
(H2), ammonia (NH3) and methane (CH4), at 800ºC. They aegypti).
observed the formation of amino acids. 289. (2) Proteins are polypeptides. They are linear chains of
271. (3) The scrotum (a pouch where testes are situated) helps amino acids linked by peptide bonds.
in maintaining the low temperature of the testes (2-2.5°C 290. (2) Phloem is a vascular tissue helps in transportation of
lower than the normal internal body temperature) food. The end product of photosynthesis is glucose
necessary for spermatogenesis. [C6H12O6] synthesized in leaves and transported to
272. (2) Typhoid is a bacterial disease caused by Salmonella the various storage organs like flowers, fruits, seeds
typhi. Streptococcus sp. are responsible for meningitis, and tubers etc. through phloem tissue.
pneumonia, endocarditis etc. Giardia is an anaerobic 291. (3) In the Aerobic respiration the complete oxidation of
flagellate protozoans causes giardiasis. Myco- glucose takes place in mitochondria and energy is
bacterium sp. causes tuberculosis and leprosy. stored in the form of ATP. Mitochondria are also known
273. (2) The Bhopal gas tragedy was an industrial catastrophe as the power house of cell.
that occurred in 1984 at the Union Carbide India Limited 292. (1) Cerebrum forms the major part of the human brain. In
(UCIL) pesticide plant in Bhopal, Madhya Pradesh. cerebral cortex, there are motor areas, sensory areas
Methyl isocynate gas (CH3NCO) was leaked from the and association areas. Association areas are
plant. responsible for complex functions like intersensory
274. (2) Emblica officinalis - Amla associations, memory and communication.
Zingiber officinale - Ginger 293. (3) The major two essential fatty acids are linolic acid and
Ocimum sanctum - Tulsi linolenic acid which are not synthesized in our body,
Medicago sativa - Alfalfa commonly taken through our diet.
275. (4) Florigen is also known as flowering hormone. It is 294. (2) Meiosis (or) Reduction division commonly takes place
produced in the leaves and acts in the shoot apical in reproductive cells. This is specialised kind of cell
meristem of buds and growing tips. division that reduces the chromosome number by half
276. (2) When a cell placed in hypertonic solution (has more results in the production of haploid daughter cells.
solutes) the cell will shrink. Water moves out, firstly 295. (3) In glycolysis, 1 molecule of glucose (6C-compound) is
from the cytoplasm and then from the vacuole. converted into 2 molecules of pyruvic acid (3C-
277. (1) Prokaryotes are organisms unicellular having primitive compound).
type of nucleus but lacking nuclear membrane and 296. (2) Large intestine specialised to absorb water from slurry
membrane bound cell organelles. of the undigested matter to form semi-solid faecal matter.
278. (4) Bryophyllum vegetative propagated through leaves Bacteria present in large intestine produce vitamin B
while sugarcane and rose propagated through stem and K which are also absorbed here.
cuttings (artifical method of vegetative propagation). 297. (2) Bile is alkaline due to presence of alkaline inorganic
279. (4) The pituitary gland is located in a bony cavity and is salts (mostly bicarbonates of sodium and potassium).
attached to hypothalamus by a stalk. The pars distalis It makes the food alkaline for action of pancreatic guice.
region of pituitary, called anterior pituitary, produces 298. (1) Estrogen is essential for development of secondary
growth hormone (GH). sex organs and external female sex characters like
280. (1) Stigma is the receptive part of carpel (pistil). It provides breasts, high pitch voice at puberty.
substratum for pollen grains to germinate. 299. (2) There are 22 pairs of autosomes and 1 pair of sex
281. (2) Asterias (commonly called star fish) and Antedon chromosomes (XY in males and XX in females) are seen
(commonly called sea-lily) belong to the same phylum, in human beings.
Echinodermata. 300. (2) Speciation is the first step of organic evolution by which
282. (4) The characteristic features of Aves (birds) are the formation of new species occur. It develops due to the
presence of feathers. They are oviparous (egg-laying). reproductive isolation which includes geographical
Solved Questions 143
B-

isolation (e.g., river, mountain), spatial isolation (e.g., 6. (4) Number of cards = 48
changes in structure, timing and season of breeding) Number of cards divisible by 5 = 10
and genetic variation (e.g., mutation). Number of prime cards = 12
301. (2) Potassium hydroxide (KOH) is a strong base while CO2 Number of cards divisible by 19 = 3
shows acidic property. KOH reacts with the CO2 and Number of composite number cards = 36
neutralise its effects. So that, CO2 is not available to (i) - (r), (ii) - (p), (iii) - (s), (iv) - (q).
the plants and the process of photosynthesis will not 7. (4) Let three prime number are a, b, c
occur. This shows that CO 2 is required for A.T.Q.
photosynthesis. 437 a b 19 23 ...(i)
302. (3) Acquired Immuno Deficiency Syndrome (AIDS) is an and ...(ii)
infectious disease caused by Human Immuno 551 b c 19 29
deficiency Virus( HIV), a type of retrovirus. For the From (i) and (ii), we get
detection of HIV, a technique, Enzyme linked immuno b 19, a 23, c 29
sorbent assay (ELISA) is used. a b c 19 23 29 71.
SECTION 4. MATHEMATICS 8. (4) Let p( x ) x 3
8x 2
3kx 7 divided by x – 1
1. (2) Let `x, `y and `z money invest by P, R and S Remainder p(1) 1 8 3k 7 16 3k
respectively.
Time duration of P = 60 – 26 = 34 years and q( x) 2 x3 kx 2 5x 6 also divide by x – 1
Time duration of R = 60 – 27 = 33 years
Time duration of S = 60 – 28 = 32 years Remainder q(1) 2 k 5 6 3 k
34 33 32 A.T.Q.
1 1 1 p(1) q(1) 9
x 1 y 1 z 1
10 10 10
(16 3k ) (3 k ) 9 10 2k k 5.
121 11
x y z 9. (4) Since x 2 bx a 0 and x 2 ax b 0 have equal
100 10
roots.
x y z For equal roots, we have
100 110 121 D = 0 and D' = 0
x : y : z 100 :110 :121. b2 4a (1) 0 and a 2 4(b)(1) 0
28 34 41 23 45 18 21 210 Now, b 2 4a a2 4b 0
2. (1) Mean
7 7 2 2
b a 4a 4b
x 30
(b a)(b a ) 4(b a)
2 log x 2 log x 3 log x 5 2 log x (2 3 5)
a b 4 ( a b)
2 log x 30 2 log30 30 2 1 1.
10. (2) A
3 3
3. (3) Given that x 5 5 x and x 2 3 5 x.
x x
3 H
x2 P
x 2 (5 x ) x2 x 3 x 5x x
x2 x 3 x 2 (5 x) 5x x 3
x2
x
3x 6 x 1 5 B B C
2 .
6 x 3x 2 2 A.T.Q.
4. (2) Let number of boys be x, girls be y. H–B=2 B=H–2 ...(i)
( x y )15.8 16.4 x 15.4 y H –1
and H = 2P + 1 P= ...(ii)
15.8 x 15.8 y 16.4 x 15.4 y 2
In ABC,
0.6 x 0.4 y
H2 P2 B2 (By pythagoras theorem)
x 0.4 2 2
. H 1
y 0.6 3 H2 (H 2)2
2
x y x
5. (1) a (By using equation (i) & (ii))
2x y 2 x 2y 2 x y 2z
H 2 18 H 17 0
x y z
a ( H 17)( H 1) 0
2x y z x 2y z x y 2z
H 17, H 1 (not possible)
x y z 1 Base = B = H – 2 = 17 – 2 = 15.
a .
3( x y z ) 3
EBD_7332
144
B- NTSE Stage 1 Question Bank

11. (3) A.T.Q. Since X is mid point of BC.


1 BX = CX = 3 cm
Tp a ( p 1)d ...(i) BC 3 3 6 cm
q
AX is also angle bisector of A , we have
1
Tq a (q 1)d ...(ii) AB BX 12 3
p AC 12 cm
Subtract eqn. (ii) from (i), we get AC XC AC 3
1 1 p q 1 12 6 12 30
( p q)d d Now, S AB BC AC 15
q p pq pq 2 2
Put in eq. (i) Area of ABC 15(15 12)(15 12)(15 6)
1 1 1 15 3 3 9 9 15 cm 2
a ( p 1) a 15. (3) Let radius of circle is r cm
pq q pq
Circumferance = 2 r
Now, Since square enclose the circle.
1 1 Length of square = 2r
T pq a ( pq 1)d ( pq 1)
pq pq Now, perimeter of square 4(2r ) 8r
1 1 P
1 1 A.T.Q., 2 r 8r P r
pq pq 2 8
12. (4) Given that,
16. (4) Since x + k is a common factor of x 2 px q and
sin 2 cos 2
x 2 , y 2
x2 lx m
We have A.M. G.M.
2
x y x y k pk q k 2 lk m 0
sin 2 cos 2
So, x y 2 2
2 2 lk pk m q
2 2 m q
2sin 2cos k
21 ( cos 2 sin 2 1) l p
2
17. (3) Let original price of rice be Rs. x / Kg.
sin 2 2
2 2 cos 2 2. 1400
13. (3) In equilateral triangle centroid and circumcentre are Total quantity
coincide x
C 40 7x
New price x 1
100 5
(–2, 2) 1400 5 1000
O Total quantity
7x x
A.T.Q.
30° 1400 1000
A x B 10
x x x
(2, 2)
400 10 x x 40.
18. (1) Total possible outcomes = 4 × 4 = 16
OA (2 2)2 (2 2)2 4 The product of the numbers is more than 14
x 3 x = {(3, 6), (3, 8), (5, 4), (5, 6), (5, 8), (7, 4), (7, 6), (7, 8)}
cos 30 Number of favourable outcomes = 8
OA 2 4
( AOB = 120° and OAB = OBA) 8 1
P .
x 2 3 16 2
Side 2 x 4 3 19. (2) Given that, sin A sin 2 A 1

14. (4) A sin A 1 sin 2 A cos 2 A


cos 2 A cos 4 A cos 2 A (cos 2 A) 2

12 cm cos2 A sin 2 A 1
20. (3) Since both less than and more than ogive intersect at
N
(40, 30) = Median,
2
B C Median = 40.
3 cm X
Solved Questions 145
B-

21. (2) BC 1 13 6
And cos 45
a/2 AC 2 AC
AC 13 6 2 26 3
a/2 Total height of tree
AB AC 13 6 26 3 13 3(2 2) m
Since diameter of circle = a units 3
a 26. (3) sin cos
radius 2
2 Squaring both side
So, side of inner square = a. 9
Area of shaded region (sin cos )2
4
1 a2 a2
a2 4 (4 ) 9
4 4 4 sin 2 cos2 2 sin cos
22. (4) Since equations has infinitely many solutions. 4
a1 b1 c1 9 5
1 2 sin cos sin cos
a2 b2 c2 4 8
27. (1) Let first term of an A.P. be a and common difference be d.
P 3 ( P 3) A.T.Q.
P 6.
12 P P a3 a8 7
23. (2) a10 a 9d 16 ...(i)
a 2d a 7 d 7
and a16 a 15d 10 ...(ii) ...(i)
2a 9d 7
On solving (i) and (ii), we get
a = 25 and d = –1 And, a7 a14 3
a26 = a + 25d = 25 – 25 = 0. a 6d a 13d 3
24. (2) A 2 a 19 d 3 ...(ii)
30°
45° 2 a 9 d
_ _ _ 7

80 m 10d 10
d 1 a 8
30° 45° a10 a 9d 8 9 1.
D C B 28. (1) Let number of blue balls be x.
In ABC, Number of red balls be 5.
AB 80 Total number of balls = 5 + x
tan 45 1 A.T.Q.
BC BC P (Blue) = 2P (Red)
BC 80 m x 5
In ABD, 2
5 x 5 x
AB 1 80
tan 30 x 10
BD 3 BD Total balls = 10 + 5 = 15.
BD 80 3 29. (3) a 2 b2 (a b)2 2ab (2 3)2 2 3
CD 80 3 80 80( 3 1)m. = 12 – 6 = 6.

C1 a 4 b4 (a 2 b 2 )2 2a 2b2 (6) 2 2 (3) 2


= 36 – 18 = 18.
30. (4) Here PQ QR RS PS 7 3 cm
A Area of square = (side)2 (7 3)2 147 cm 2
25. (2) 1 2
Area of sector RQAS r
45° 4
C B 1 22
13 6 m 7 3 7 3
4 7
In ABC, = 115.5 cm 2
AB AB Area of shaded portion = 147 – 115.5 = 31.5 cm2.
tan 45 1 31. (3) Let denominator of fraction is x
BC 13 6
then numerator of fraction is x – 3.
AB 13 6m
EBD_7332
146
B- NTSE Stage 1 Question Bank

x 3 Given that,
Fraction AB || CD EDC A 28 (Alternate angles)
x
New numerator = x – 3 + 4 = x + 1 In CED,
New denominator = x – 2 y 40 28 180 (Angle sum properties)
A.T.Q.
y 112 .
x 1 2( x 2) x 5 39. (1) Since, total surface area of cube = 6a2 = 96
2 a 4
Fraction . Volume of cube = a3 = 64 cm3.
5
32. (4) Let ratio be x 40. (3) Given word : EDUCATION
Angles of triangle are 3x, 4x and 5x. Vowels = E = {E, U, A, I, O}
n(E) = 5
3x 4 x 5x 180 (Angle sum property)
x 15 5
P( E )
Greatest angle 5 x 5 15 75 . 9
33. (1) Since area of each face of the cube = l2 A
41. (4)
144 l 2 l 12 cm
Total surface of two cuboid
= Area of cube + 2 × area of one face 5m E
6l 2 2l 2 8l 2 8 144 1152 cm 2 . 100 cm
34. (2) cosecA 2 A 30
B y C x ? D
cos A cos 30
tan A tan 30 BC = Distance walked by girl = speed × time y = 1.9 × 4
1 sin A 1 sin 30 y = 7.6 m
1 3/2 1 3 2 ABD ~ ECD (AA similarity)
3 1 3 2 3 AB BD
1
2 EC CD
1 1 2 5 7.6 x
5 x 7.6 x
1 x
3 3 3
35. (1) Ascending order of given data : 4 x 7.6
2, 5, 6, 7, 8, 9, 10, 14, 15, 17, 18, 19, 24, 27, 28 x 1.9 m 190 cm
Number of observations (N) = 15 (odd)
th th 42. (3)
N 1 15+1
Median Obs.= Obs.
2 2
= 8th Obs. = 14. h = 21 cm
36. (2) Here a 3 3, b 7, c 3
Discriminant (D) b2 4ac (7)2 4 3 3 3
= 49 – 36 = 13.
Total = 100 Circumference of base of cone = 2 r 220
37. (4)
60 40 22
? 2 r 220
7
V C r 35
Volume of cone
10 1 22
35 35 21 26,950 cm 3
Here, n(V) = 60 and n(C) = 40 3 7
n(V C ) 100 10 90 43. (1) ax = b ...(i)
y
b =c ...(ii)
Now, n(V C ) 60 40 90 10 cz = a ...(iii)
n (Only C) = 40 – 10 = 30. putting the value of b from eqn.(i) to eqn. (ii), we get
A B (ax)y = c axy = c ...(iv)
38. (3) 28° Putting the value of c from eqn (iv) to eqn (iii)
E (axy)z = a axyz = a1
xyz = 1
y 3 1
44. (1) Given that, a b 3
40° 3 1
C D
Solved Questions 147
B-

Rationalise the demonimator, we get On squaring both side, we get


2
3 1 3 1 x 1 1 x 1
a b 3
3 1 3 1
2 x 1 1 x 1 2 x 1
( 3 1)
a b 3 1 2 x 1
( 3)2 12
1
3 1 2 3 x 1
a b 3 2
3 1 On squaring both side, we get
4 2 3 1
a b 3 x 1
2 2 4
2 3 a b 3 1 5
On comparing both side, we get x 1
a = 2, b = –1 4 4
2 2
45. (1) In a leap year, total 366 days in which 52 weeks and and 49. (1) Given that 3 2 x
2.3x x 2 32( x 2)
0
2 day remaining 2 2
( x 2) 2
Possible outcomes = (Sun, Mon), (Mon, Tues), (32 x ) 2.3 x .3 x 2
3 0
(Tues, Wed), (Wed, Thurs), (Thurs, fri), (Fri, Sat), (Sat, Sun)
(a b)2 a2 2 ab b 2
2
Favourable outcomes = (Sun, Mon), (Sat, Sun) [3 x 3 x 2 ]2 0
Probability of getting 53 Sundays 2
No. of favourable outcomes 2 3x 3x 2

= x2 = x + 2
Total no. of outcomes 7 x2 = x + 2 (By comparing the powers)
1 x2 – x – 2 = 0
46. (3) sin cos 2 sin cos x2 – 2x + x – 2 = 0
2 x (x – 2) + 1 (x – 2) = 0 x(x – 2) (x + 1) = 0
1 x = 2, –1
sin 2 [ sin 2 2sin .cos ]
2 x {–1, 2}
We have, –1 sin 2 1 50. (4) 3x y 81 34
Min. value of sin 2 is –1 x+y=4 ...(i)
1 1 and 81x–y = 3
so, min. value of sin cos 1= 34(x–y) = 34
2 2
47. (2) Let radius and height of right circular cylinder be r and 1
h respectively x y ...(ii)
4
volume of cylinder = r2h from (i) & (ii)
When radius is increased by 10%
17 15
10 11r so, x and y
New radius = r 1 8 8
100 10 51. (1) Volume of larger cube = Volume of 125 cubes
Let the decreased in height is x%, a3 = 125 b3
x a = 5b
New height = h 1 S.A. of larger cube = 6a2 = 6 (5b)2 = 150 b2
100
A.T.Q
Original volume = New volume
2 125 cubes
11r x
r 2h h 1 Length of cuboid = 125b
10 100
Breadth = b
121 x Height = b
r 2h r2h 1
100 100 S.A. of 125 smaller cubes forming a cuboid
= 2 [125 × b + 125b × b + b × b] = 502b2
121 x 100 x % of increase in S.A.
1 1 1
100 100 121 100 502b2 150b2
100
x 100 x 21 150b2
1
100 121 100 121
352b 2
21 = 100
x 100 17.355 17.36% approx. 150b2
121
704 2
48. (1) x 1 x 1 1 = 234 %
3 3
x 1 1 x 1
EBD_7332
148
B- NTSE Stage 1 Question Bank

3 5 7 19 In DOP,
52. (1) .... DO = 3 5
1.4 4.9 9.16 81.100
4 1 9 4 16 9 100 81 In BOQ, BO = 5 5
....
1.4 4.9 9.16 81.100 BD = 8 5
1 1 1 1 1 1 1 In ADB
1 .... (AD)2 + (AB)2 = (BD)2
4 4 9 9 16 81 100 As, ABCD is square
1 99 AD = AB
1 BD2 = 2AB2
100 100 320 = 2 × AB2
53. (4) Given that sin + cos = x 160 m2 = AB2
1 1 Hence, area of square = 160 m2
2 sin cos x 56. (4) From the given equn,
2 2 (2x – 4)3 + (22x – 2)3 = (22x + 2x – 6)3
(2x – 4)3 + (22x – 2)3 – (22x + 2x – 6)3 = 0
2 sin .cos cos .sin x 3(2x – 4)(22x – 2) (–22x – 2x + 6) = 0
4 4
2x = 4 x=2
2 sin x 1
4 22x = 2 x=
2
sin A.cos B cos A.sin B sin( A B) 2x + 3 = 0, it is not possible
As, 0 < < 90° 2x – 2 = 0 x = 1
1 x 2 1
Sum of all possible values of x = 2 1
54. (1) Let the ratio is k : 1, using section formula, we get 2
= 3.5
3k 2 7k 4 57. (4) Let p = 2019x and q = 2019–x
x ;y
k 1 k 1
p6 – q6
Now, (p q ) [( p 2 q 2 )2 – p 2q 2 ]
p–q
K 1
A B 3[(( p q )2 – 2 pq ) 2 – p 2 q 2 ]
(2, –4) P (3, 7)
(x, y) 3 48
Since P (x, y) lies on 9x – 3y – 14 = 0 = 12
58. (1) Let r be the radius of the circle, then r2 = (p – 1)2 + (p –
3k 2 7k 4 4)2
9 3 14 0
k 1 k 1 r2 = 2p2 – 10p + 17
27k + 18 – 21k + 12 – 14k – 14 = 0 = 2(p2 – 5p) + 17
16 = 8k As, p is root of equn x2 – 5x + 7 = 0
k=2 p2 – 5p + 7 = 0 p2 – 5p = – 7
Ratio = 2 : 1 =3
55. (1) From the Figure, Now, Area of circle = r2
= 3 sq. units
D C
1 1 1
Q 59. (4) x y x y
6
m

1 1 1
P –
10 m x y x y
x–x– y 1
x( x y) y
–y2 = x2 + xy
A B (x – y) (x2 + xy + y2) = 0
DPO ~ BQO x3 – y3 = 0
DP PO x3 = y3
6 3
BQ QO x x
2
6 x y y
10 8– x 60. (4) Let x1, x2, x3, ..., x15 be the no. of observations
x1 + x2 + x3 + x4 + ... +x8 = 384
x = 3m x8 + x9 + x10 + ... + x15 = 424
Solved Questions 149
B-

x1 + x2 + x3 + ... + x15 = 750 64. (3) It is given that


x1 + x2 + x3 + x4 + x5 + x6 + x7 + x8 ...+ x15 = 750 x
384 – x8 + 424 = 750 cos 43° =
Hence, eighth observation is 58 x2 y2
61. (1)
x
sin (90° – 43°) =
2
x y2
x
sin 47° =
2
x y2
y y1
Slope = 2 A
x2 x1
For the points (a, 0), (0, b) and (1, 1),

x2 y2
x

47°
b 0 1 b C B
y
Slope =
0 a 1 x
b = ab – a tan 47° =
a + b = ab y
65. (3) (i) For equal roots,
a b 1 1 D=0 b2 – 4ac = 0
= 1, =1
ab ab b a 16 – 8 (a + 5) = 0
62. (3) Let a1 and a2 be the sides of two cubes and S1 and a+5=2
S2 be the surface area of cubes a = –3
a13 27 (ii) For distinct real roots,
= D>0 b2 – 4ac > 0
a23 64 2
a + 12b(a + 4) > 0
9 + 12b > 0
a1 3 S 6a12 9
= , 1 9
a2 4 S2 6a22 16 b>–
12
63. (4)
A 3
b>
4
66. (1) Here, O is the centre of circle.

B C
p
1
Area of = ph
2
3p 7p Let r be the radius of circle
p = p
10 10 2 2
r= 25 24
h 5h
h = h
4 4 = 625 – 576
1 7p 5h = 49 = 7 cm
=
2 10 4 67. (4) For given numbers,
(55)725, unit digit = 5
35 (73)5810, unit digit = 9
= ph
80 (22)853, unit digit = 2
Unit digit in the expression
1 35 55725 + 735810 + 22853 is 6
ah ah 100
2 80 68. (3) Given series can be written like
Percentage decrease = = 12.5
1 3 5 7 17 19
ah ...
2 1 4 4 9 9 16 64 81 81 100
EBD_7332
150
B- NTSE Stage 1 Question Bank

1 1 1 1 1 1 1 1 1 ( )2 ( )2
=1 + ... + =
4 4 9 9 16 64 81 81 100 ( )2 4 ( )2 4
1 99 2
=1 = = 0.99 b2 q
100 100
69. (4) f(x) = (x – 1)2 + (x – 2)2 + (x – 3)2 + (x – 4)2 a2 p2 b2 q2
= =
2 b2 4c q2 r b2 4ac q2 4 pr
5 4.
= 4 x ÷ +5 a2 a p2 p
2
5 b2 q2 b2 D
f(x) is minimum at x = = 2.5 = 2
= 1
2 D1 D2 q D2
70. (2) n(S) = [1, 2, 3, ..., 100] = 100 74. (*) Radius of sphere = R C
1 ABC ~ ODC
x+ >2
x AB AC h
=
x2 + 1 > 2x OD OC
x2 – 2x + 1 > 0
(x – 1)2 > 0 r r 2 + h2 O R D
x = [2, 3, ... ,100] =
R h R R

( )
n(E) = [2, 3, 4, ..., 100] = 99
99 r(h – R) = R r 2 + h2 B r A
P(E) = = 0.99
100 r2(R2 + h2 – 2hR) = R2(r2 + h2)
71. (1) According to question r2h2 + r2R2 – 2hRr2 = R2r2 + R2h2
2
r22 r2h2 – 2hRr2 = R2h2
A2 = r1
h(r2 – R2) = 2Rr2
A2 = r12 r22 (1)
2Rr 2
A2 = (r1 – r2)2 + 2r1r2 r1 – r2 = d h=
= d2 + 2r1r2 r 2 R2
A 2 > d2 Portion of water which comes out of the cone
A>d 1 2 4 3
72. (4) Area grazed by horses = r h R
3 3
7 7 7 7 7 7 1
A+ B+ C÷ = (r 2 h R 3 )
360° 360° 360° 3
(using sector formula) Answer will be in terms of multiple of
A Hence, no option is correct.
7 7 75. (*) A
= ( A + B + C)
360°
34
m

22
42

7 7
m

= 7 180
360°
= 77 m2 y
B C
Area of ABC 20 m
= 48(48 20)(48 34)(48 42)
= 48 28 14 6
= 336 m2
Area ungrazed by horses x x x
= 336 – 77 B D E C
= 259 m2 In AEB,
73. (2) Let , and , are roots of equation AE2 = y2 + 4x2
ax2 + bx2 + c = 0 and px2 + qx + r = 0 In ABC,
AC2 = y2 +(3x)2
respectively, Given = = y2 + 9 x2
3AC2 + 5AB2 = 3y2 + 27x2 + 5y2
Applying componendo & dividendo rule = 8y2 + 27x2 = 8y2 + 32x2 – 5x2
= 8(y2 + 4x2) – 5x2
= = 8AE2 – 5BD2
3AC2 + 5AB2 = 8AE2 – 5BD2
( )2 ( )2 No option is correct
=
( )2 ( )2
Solved Questions 151
B-

76. (1) A b a
cosec + cot =
2 2 2
b a b a2
30° 30° b a
=
b a
80. (3)
C
60° 60°
B C 2+x 2+x
D

12
cm
3 F E

cm
14
we know that height of an equilateral triangle a,where
2 12 – x 10 – x
2 3 2 3
a is the side of equilateral triangle AD a BC 2 A B
4 4 x D 10 – x
77. (3) (1) tan + cot = 5
Squaring both sides, (tan + cot = 25 10 cm
tan2 + cot2 = 23 Let AD = x
Option (1) is correct So, BD = BE = 10 – x
(2) Given equations has infinitely many solution. CE = CF = 2 + x
4 5 q AF = 12 – x
So, As, AF = AD
8 k 18 12 – x = x
K = 10 x=6
Option (2) is correct
(3) cosec60 x + cos45 y = 4 81. (2) A
2 1 1
x y 4
3 2
D E
Put (tan60 , sec 45 ) = 3, 2
2 1 2
3 2 = 21 + 1 = 3 4
3 2 B C
Option (3) is not correct 6 cm
(4) + = 2; =1 AD 1
3 + 3 = ( + )3 –3 ( + )
3 + 3 = (2)3 –3(1) (2) = 2 BD 2
Option (4) is correct DE BC
78. (*) It is given that –2 is a root of x2 – px + 6 = 0 AD DE
4 + 2p + 6 = 0
p = –5 AB BC
x2 – 5x – k = 0 has equal roots 1 DE
b2 – 4ac = 0
3 6
25 + 4k = 0 DE = 2 cm
25 82. (4) Given series is 5, 12, 19.....,173
k=
4 Here, a = 5, d = 7, l = 173
Hence, no option is correct
nth term from the end = l – (n –1)d
79. (2) A = 173 – (9) (7) = 110
83. (2) Let zeros of the polynomial be a – d, a, a + d
A.T.Q.
a–d+a+a+d=3
b 3a = 3
b2 a2 a=1
Putting value of a in given function,
1–3+m=0
m=2
B a C 84. (3) Let a, b be the two numbers
a Base A.T.Q.
Since, cos a.b = 168 × 12
b Hypotenuse a + b = 108
Difference between numbers is,
AB = b2 a 2 (using Pythagoras The.)
(a – b) = (a b)2 4ab
EBD_7332
152
B- NTSE Stage 1 Question Bank

= 108 108 4 168 12 vs v 1 2 144


= r h 1
= 11664 8064 v 3 125
= 3600 = 60 M 1 r 2 h 19
85. (3) Let MN = 2x = 100
x 3 v 125
In MNO, A
P Percentage change in volume
MA MN = MP2 0.152 100 = 15.2%
3 2x = x2
x
x=6 90. (2)
Given, MN MO
MO = 2x = 12 N O
P
86. (4) y

x
Q R
As, PSQ ~ PQR, QSR ~ PQR
and PSQ ~ QSR h h
In PSQ & QSR
PS QS 2
= QS = PS SR
QS SR
In PQR & QSR x
In figure,
PQ QS y
= PQ QR = QS PR tan and=
PR QR x
87. (3) In given series, h
tan =
60 58 56 x
a1 = 20 = , a2 = , a3 = tan y
3 3 3 =
tan h
25 60 2
S25 = 2 24 = 25(20 – 8) = 300 tan
2 3 3 h+y= h 1
tan
36 60 2 25
S36 = 2 35 36 300 91. (4) Let edge = a and r be the radius of vessel
2 3 3 3 2r = 30 r = 15
88. (2) Area of shaded region = area of semi-circle – area of a3 = r2h
triangle 22 140
a3 = 15 15 1000
r2 7 99
r 2 = 126 a = 10 cm
2
r2 ( – 2) = 126 2 92. (1) S 3 R
8
r2 = 2 126 h
7
2r = 7 3 2 21 2 cm O
89. (1) Let v be the original volume and vs be the changed 2h
volume.
1 2
v= r h P 6 Q
3
In trapezium PQRS,
6r 4h
Now, rs = , hs Ar ( POQ ) 1/2 6 2h 4
5 s = =
1 2
Ar ( ROS ) 1/2 3 h 1
vs = (rs ) h5 93. (1) From given condition,
3
20
1 36r 2 4h 1 2 144 Dividend = 100 160 3200
vs = r h 100
3 25 5 3 125
3200
Rate of return = 100 16.67%
120 160
Solved Questions 153
B-

94. (2) Let speed of boat in still water be x km/hr 103. (1) Area of quad. ABCD = area of ABD + area of
and speed of stream be y km/hr CBD.
30 1 3
=3 x + y = 10 …(1) = 5 12 (13) 2
x y 2 4
30 169 3 120 169 3
=5 x–y=6 …(2) = 30 + sq.cm
x y 4 4
From solving equations (1) and (2)
x + y = 10 fi xi f i xi
104. (4) weighted mean 13.5 P
–x–y=6 fi fi
+ –
2y = 4 y = 2 km/hr. and 216
13.5 P
x = 8 km/hr 16
95. (2) Let x be the Selling Price (S.P.) of the bicycle 13.5 = 13.5 + P
12 P=0
x+ x = 10304 105. (3) a : b = 2 : 3
100
x : y= 3 : 4
112
x = 10304 x = 9200 2ax 25by a 2, b 3
100 for , put x 3, y 4
GST = 10304 – 9200 3ay 4bx
GST = Rs. 1104 2 2 3 25 3 4 12 300 24
SGST = Rs. 552
96. (3) Volume of cuboid = l.b.h = 3000 cm3 3 2 4 4 3 3 24 36 5
Area of base = l.b = 150 cm2
Volume 106. (4)
= Height of cuboid C1
Area
3000
Height = = 20 cm
150
180
97. (2) As 1 radian = 1 degree
Sq2
2 2 180
radian =
3 3 C2
120 Sq1
Time = = 20 min.
6
98. (4) Area of shaded portion = area of square – area of 4 radius of c1 = a
circle 2a
(28)2 – 4 (7)2 side of sq1 2a
784 – 616 = 168 cm2 2
99. (2) 460[4 + 42 + 43 + 44 ] = 460[4 + 16 + 64 + 256] side of sq1 = (radius of c2) × 2
460 340 a
100. (2) sin + sin3 = cos2 radius of c 2
sin (1 + 1 – cos2 ) = cos2 2
sin2 (2 – cos2 )2 = cos4 a a 2
(1 – cos2 )(4 + cos4 – 4cos2 ) = cos4 side of sq2 2 a
4 + cos4 – 4cos2 – 4cos2 – cos6 + 4 cos4 2 2 2
= cos4 107. (2) a cos b sin c & a sin b cos p
cos6 – 4 cos4 + 8 cos2 = 4 On squaring and adding both equations
101. (3) We know, AM GM
a 2 cos2 sin 2 b2 sin 2 cos 2 c2 p2
x y
xy a2 + b2 = c2 + p2
2
a2 + b2 – c2 = p2
12
xy x y 12 xy 36 p a 2 b2 c 2
2
108. (3) x2 – 3x + 2 is factor
102. (3) It is given that p = l 2 b2 h 2 (x –1) (x –2) are also factors.
q = 2(lb + bh + lh) By remainder theorem,
r=l+b+h (1)4 – p(1)2 + q = 0
p2 + q = (l + b + h)2 1–p=q ... (1)
l+b+h= p2 q and 24 – p(2)2 + q = 0
16 – 4p + q = 0 ... (2)
r= p2 q From (1) and (2)
p = 5, q = 4.
EBD_7332
154
B- NTSE Stage 1 Question Bank

109. (1) Let common difference of AP = d 2


2 4 6
1 1 1 1 S cos cos cos
14 14 14
x1 x2 x1 x2 d
6 8
cos cos
1 1 1 1 14 14
x2 x3 x2 x3 d 2 4 8
2
S cos cos cos
1 1 1 1 14 14 14
xn 1 xn xn 1 xn d S cos cos 2 cos 4
1 1 1 2
16
x1x2 x2 x3 xn 1xn sin
14 1
S
1 1 1 1 1 3 2 64
2 sin
d x1 x2 x2 xn 14

1 1 1 112. (4) S P 1 Q.
Sum 2
d x1 xn S1 = P = T1
S2 = 2 P + Q
1 xn x1 S2 – S1 = P + Q
Sum d x1.xn S2 – S1 = (a + d + a) – (a) = a + d
T2 = a + d = P + Q and T1 = P
1 x1 n 1 d x1 d=Q
Sum d x1.xn 113. (4) Sum of 25 observations = 25 × 36 = 900
Sum of first 13 observations = 13 × 32 = 416 ... (1)
n 1 Sum of last 13 observations = 13 × 39 = 507 ... (2)
Sum S26 sum of 25 observations + 13th observation
x1.xn
900 + 13th observation = 416 + 507
1 13th observation = 23
110. (2) Consider : b a 114. (2) Let ‘b’ boys and ‘g’ girls are there.
1 x xc a
50b 40 g
1 46
b g
xa a
xb a
xc a
4 b = 6g
1
b 3
xa xb xc desired ratio
g 2
xa xa xa 115. (3) Container which can measure exactly the two
xa containers
= HCF of (volume 1, volume 2)
x a xb xc = HCF (240, 112)
Similarly on solving other two parts we get = 16 litres
xa xb xc = 16000 cm3
116. (1) 2x–1 + 2x+1 = 2560
xa xb xc xa xb xc xa xb xc
2x 1
1 22 2560
xa xb xc
1
xa xb xc 2560
2x 1 512
sin 2 r 5
111. (2) cos 2 r x–1
2 = 512 = 2 9
2 r sin x = 10
Converting whole series in terms of cos 117. Product of 2 numbers = LCM × HCF
13 11 3 9 5 N1 = 36x and N2 = 36y
sin sin ; sin sin ; sin sin 36x . 36y = 12960
14 4 14 4 14 14
xy = 10
3 5 For x = 1, y = 10
S sin sin sin 1 x = 2, y = 5
14 14 14
x = 5, y = 2
6 3 4 5 2 x = 10, y = 1
also, sin cos ,sin cos ;sin cos 4 such pairs are possible
14 14 14 14 14 14
Solved Questions B-155

118. (3) A 2 2
123. (1) x 1
x 1 x 1
10 cm x=1
6 cm On keeping x = 1,
2
term becomes indeterminant.
x 1
B C no real roots exist.
x D 12-x
12 cm 1
124. (1) For x ; minimum positive value is 2.
As AD is angle bisector x
By Angle Bisector theorem and maximum negative value is –2.
we can say.
AB AC
BD CD 1 1
2 x and 2 x
x x
10 6
and 1 sin 1
x 12–x Option 1 is not possible.
3x = 60 – 5x 125. (2) P = cos 15° cos 7.5° sin 7.5°
x = 7.5 cm.
119. (4) Sum becomes 6 times, at 5 % per annum rate. 2 cos 7.5 sin 7.5 cos15
P
S I = 5 × Principle 2
P 5 T cos15 sin 15 2 cos15 sin15
5P P P
100 2 4
T = 100 sin 30 1
For sum to become 12 times at same time P
4 8
P R 100 126. (2) P(Hindi) = 0.6
11P
100 P(English) = 0.4
R = 11%
P(English and Hindi) = P E H 0.2
120. (2)
C P (English or Hindi) = P H P E P E H
r2 = 0.6 + 0.4 – 0.2
P R
r1 r3 P E H = .8
Probabily of people who does not read neither
A B Hindi = 1 – 0.8
Q 1
AB = AQ + QB = 5 0.2
5
BC = BR + RC = 7
CA = AP + PC = 6 127. (2) Centroid divides the line joining
From above equations circumcentre and orthocentre in 2 : 1 ratio
r1 + r3 = 5 ... (1) 2:1
r2 + r3 = 7 ... (2) H O
G
r1 + r2 = 6 ... (3) (x,y) (2,4) (1,7)
On solving (1), (2) and (3), we get
2 x 14 y
r1 = 2 cm 2 and 4
r2 = 4 cm 3 3
r3 = 3 cm x = 4 and y = – 2
121. (2) p, q, r are roots H (x, y) = (4, –2)
p+q+r=–5 128. (2)
pq + qr + rq = 16
pqr = – 48
p(qr + q + r) + qr
= pqr + (pq + pr + qr) B
(o,b) 2
= – 48 + 16
= – 32 R (h,k)
122. (2) f(x) = 2x +1
f(2x) = 2(2x) + 1 = 4x + 1 1
f(4x) = 2(4x) + 1 = 8x + 1
For f(x), f(2x), f(4x) to be in GP A
(4x + 1)2 = (2x + 1) (Bx + 1) (a,o)
16x2 + 8x + 1 = 16x2 + 10x + 1 x = 0
Only 1 real value of x exists.
EBD_7332
156
B- NTSE Stage 1 Question Bank

2a b 2 2 2 1
R h, k ,
3 3 1 3 5–5 10 5
3 x 5
2a b x
h and k
3 3 134. (4) sin 3 cos3 0
3h
a and b = 3k sin cos sin 2 cos 2 sin cos 0
2
x y sin cos 1 sin cos 0
Equation of AB is : 1
a b Either sin cos or 1 sin cos
x y tan 1 or 2 = sin 2 , which is not possible
1 –45
3h 3k
2 –45
2 kx + hy = 3hk 135. (4) log 4 7 x
129. (3) Given : x3 + 4x = 8 x3 = (8 – 4x)
On squaring 1 1 2
log 7 16
x6 = 64 + 16x2 – 64 x log16 7 1 log 4 7
log 4 7
(x6 + 64x) = 64 + 16x2 2
(x6 + 64x) = 16 (x2 + 4)
2
Multiply by ‘x’ on both sides
(x7 + 64x2) = 16(x3 + 4x) (x7 + 64x2) = 16 × 8 x
(x7 + 64x2) = 128 136. (2) Sum of numbers divisible by 2
130. (1) 102017 – 2017 S1 2 1 2 .... 50
10.........00000
Now, 102017 = 144244 3 50 51
2017zeros 2 2550
2
102017 999.........97983
1442443 Sum of numbers divisible by 5
2013 9 S2 = 5 (1 + 2 + ... + 20)
Sum of digits = 9 × 2013 + (7 + 9 + 8 + 3) 21
= 18144 5 20 1050
131. (2) P(1) = 0 x = 1 make p(x) zero 2
(x – 1) is factor of p(x) Sum of numbers divisible by 10
Similarly (x –2) (x – 3) are also factors of p(x) S3 = 10 (1 + 2 + ... + 10)
p(x) = R (x – ) (x – 1) (x –2) (x – 3) + R. 11
p(x) = x4 + ax3 + bx2 + cx + d has highest power S3 10 10 550
2
coefficient = 1, Required sum = S1 + S2 – S3
k = 1, also (x – 1) is factor, R = 0 = 2550 + 1050 – 550
p(x) = (x – ) (x – 1) (x – 2) (x – 3) = 3050.
p(0) = (– ) (–1) (–2) (–3) = 6 137. (3) a, b, c are in AP.
p(4) = (4 – ) (3) (2) (1) a + c = 2b
= 24 – 6 a – 2b + c = 0
p(0) + p(4) = 24 On comparing with ax + by + c = 0
132. (2) p = 39 + 312 + 315 + 3n we get x = 1; y = –2
p = 39 (1 + 33 + 36 + 3n–4) 138. (4)
A
3 6 2 2
If we have 1 3 3 3 1 3 32 1

then it is of the perfect cube form F E


2 6 3 5 60º
1 3 1 3 3 3
For p, if we keep n = 14, B C
P D
We get p as perfect cube. BFED is a parallelogram
3 1
i.e. p 33 1 32 FE = BD BC
2
FBP = FED = 50°
1
133. (2) x 5 F is mid point of hypoteomuse AB.
x FP = BF = AF
2x 2 FBP = FPB = 50°
For FPC = 180° – 50° = 130°
3x 2
5x 3 3
3x 5
x
Solved Questions 157
B-

139. (1) x < 1 and y < –1 145. (1) Since its a right with sides (6, 8, 10)
let x = 0.5 and y = – 2
(x –1, y –3) = (0.5–1, –2 –3) 1
6 8
= (– 0.5, –5) area of triangle 2 24
= Third quadrant Inradius 2
S 6 8 10 12
4 2
140. (2) 3 sin x cos x 6 sin x cos x 4 sin 6 x cos6 x 2
146. (2) A
2 3 2
3 1 sin 2 x 6 1 sin 2 x 4 1 sin 2 x E
4 Q F
2 2
3 1 sin 2 x 2sin 2 x 6 6sin 2 x 4 – 3sin 2 x D
= 3 + 6 + 4 = 13 B C
P
x
a
141. (3) bc ax abc ... (1)
a
AB||EP and BC||QF
by y ABC = QEP (opposite angles of parallelogram)
ac b abc ... (2)
b ABC + DEF = QEP + DEF
cz = 180º
ab c3 abc ... (3) 147. (3)
c A
(1) × (2) × (3) 40º
3
ax b y cz abc
x = 3, y = 3, z = 3
1 1 1 1 D x E
3 1
x y z 3 O
142. (4) For distinct roots, D > 0
B C
2
5 k 4 k 3 0 In ABC, 40º + 2 + 2 = 180º
= 70º
25 k 2 10 k 4k 12 0
In BOC, BOC + = 180º
2
k 13k k 13 0 BOC = 110º = x
k 1 k 13 0 148. (1) A

kt ,1 13,
i.e. k < 1 and k > 13 h

143. (3) ax by z
60º 30º
B D
y z C
10
a b x h
BC = ( tan 60º = 13 )
Taking log on both sides, with base b 3
also BD = h 3 ( tan 30º = )
y z 3
log b a log x b x h
h 3 – 3 = 10
y z
logb a h = 5 3 m.
x
log a y z x 3 x 5
149. (1) and
log b x y 5 z 7
144. (3) For equal roots, D = 0 Let x = 15, (LCM of Numerators in both expressions)
2 y = 25 and z = 21
2b a c 4 a 2 b2 b2 c2 0
y z 25 21 4 2
2 Now:
b2 a c (a 2 b2 ) b2 c2 0 y z 25 21 46 23
150. (4) For diagonal SQ,
b2 a 2 b2 c2 2b 2 ac a 2 b 2 a 2 c2 b4 b2 c2 0 5 15 5 n
2b2 ac = a2c2 +b4 Midpoint = ,
(b2 – ac)2 = 0 2 2
b2 = ac. For diagonal PR.
EBD_7332
158
B- NTSE Stage 1 Question Bank

m 9 15 and also, ( ) ( +1) = q


Midpoint = , 2+
2 2 =q ...(2)
2
P – 4q
m 9 15 5 n
10 = and = (2 +1)2 – 4( 2 + )
2 2 2
m= 1 and n = 10 = 4 2+1+4 –4 2–4
m–n=1 =1
151. (4) Largest possible triangle in semicircle is the one whose 157. (1) In chess board total 32 Black and 32 White squares are
height is equal to radius of circle and base is equal to there.
diameter of circle. 1 Total possible solutions = 64 C3
r r
area of triangle = 2 and favourible solutions = (2 Black + 1 White)
2 16 16 or
r2 (2 White + 1 Black)
=
256 32 32
152. (3) Given tan5 tan4 =1 C2 C1
1
tan5 = 32 32
tan 4 C2 C1 2 16
tan5 = cot4 P E
64 21
C3
tan5 = tan(90° 4 )
5 = 90° 4 5 7
3 2 5 3 7 3
= 10º 158. (2) 3 3 3 2 2 4
153. (1) sin2 A + sin2 B + sin2 C = ? ...(1) 4 2
In a right ABC, 5 21 1
let A = 90º =
2 8 8 y2 y1
C = 90º – B 159. (4) A: slope of a line is, obtained by
Eqn (1) can be rewritten as x2 x1
formula
sin2 90º + sin2 B + sin2 (90° – B) B: slope of a line parallel to y–axis is infinite
1 + sin2 B cos2 B C: The tangent ratio of an angle made by line and
1 + 1= 2 positive X axis gives the slope of a line.
154. (3) Sn = 3n + 2n 2 D: Slope of line which makes acute angle lies
S1= T1 = 3 + 2 = 5 ...(1) between (0, ) and slope of line which makes
S2 = T2+T1 = 3(2) + 2(2)2 obtuse angle lies between (– , 0)
= 14 Hence all statements given are wrong.
...(2) P Q R
160. (4)
From eqn. (1) and (2)
T1 = 5, T2 = 9
For common difference = T2 – T1= 9 – 5 = 4 A 5 B 13 C 17 D
155. (4) y Angles in semi circle is 90º
x–y =2 APB = BQC = CRD = 90°
(0, 3)
By using Pythagorean tripletes
In APB, (3, 4, 5) triplet
(2, 0)
x
In BQC, (5, 12, 13) triplet
3 In CRD, (8, 15, 17) triplet
,0
k kx + y = 3 (AP + PB) + (BQ + QC) + (LR + RD)
= (3 + 4) + (5 + 12) + (8 + 15)
= 47 Units
For intersection in 1st quadrant 161. (4) Let the sides of squares be x and y.
3 3 4x + 4y = 120 and x2 + y2 = 468
2(has to be ahead of (2,0)) x + y = 30
k k
x2 + y2 + 2xy = 900
3
k>O( will be positive) and R 0 468 + 2xy = 900
3k 2xy = 432
O< k<
2 For (x–y);
156. (4) Let , 1 be the two consecutive integers
(x–y)2 = x2 + y2 – 2xy
+ 1 = – (–P)
= 468 – 432 = 36 differnce = 6
2 +1= P ...(1)
Solved Questions 159
B-

162. (4) 5x+1 + 52–x = 126 D=0 [ roots are equal]


52
5.5x + x = 126 p2 – 4 × 2 × 8 = 0
5
Let 5x = t 2 p2 = 8× 8 p=8
5
5t + = 126 5t2 + 25 = 126t 2
p(x + x) + k = 0
t
5t2 – 126t + 25 = 0 5t2 – 125t – t + 25 = 0 8x2 + 8x + k = 0 [ p = 8]
1
(t–25) (5t–1) = 0 t = 25, t =
5 Again D= 0
x = 2 or x = –1
64 – 4 × 8 × k = 0 64 – 34k = 0
163. (3) Q
64
M k=
32 = 2.
O
167. (4) x2 – p(x + 1) – k = 0
60º
R x2 – px – p – k = 0
P
T x2 – px – (p + k) = 0 ...(1)
By Alternate Segment theorem, Since, and are zeroes of polynomial.
We know that QPT = PMQ = 60º Then, + = p [ equation (1) is quadratic equation]
PMQR is cyclic quadrilateral = – (p + k)
PMQ + PRQ = 180º PRQ = 120º Now ( + 1) ( + 1) = 6
164. (4) C +( + )+1 = 6
– (p + k) + p + 1 = 6
–k+1 = 6
k = – 5.
y 2 6 3 4
168. (3) 6 = 3 , 6 = 216, 6 = 1296, , ...
52 = 25, 53 = 125, 54 = 625, ...
We notice for all powers of 6 and 5 the units digit is 6 and 5
B A
respectively.
E D
Unit digit of 611 – 510 = 6 – 5 = 1.
x x x
169. (3) PQS
AC2 EC 2
For P
DC2 BC2
20°
y2 (3 x )2 y2 x2 40+x
T
= O
y2 (2 x )2 y2
x
y2 9 x2 y 2 x 2 8 x2 40° 50°
= 2 Q R
y 2 4 x2 y 2 4x2 S
PQS = QPS
3 2 3 3 3 [ Angles opposite to equal sides]
165. (3)
3 3 3 3 QPS = 40 + x
(3 2 3)(3 3) 9 6 3 3 3 6 Now in PQT,
= = 40 + x + + x = 90
9 3 6
60 + 2x = 90
15 9 3 x = 15°.
=
6
1 1
15 9 170. (3) 20, 19 , 18 , ...
= 3 [ a, b are rational number] 4 2
6 6 3
5 3 a = 20, d =
3 4
=
2 2 3
5 3 T n = 20 + (n – 1) <0
a 3b 3 4
2 2 3
5 3 = 20 + (1 – n) < 0
4
a = , b a b 2
or 80 + 3 – 3n < 0
2 2
166. (2) 2x2 + px + 8 = 0 83
n > n = 28
3
EBD_7332
160
B- NTSE Stage 1 Question Bank

171. (2) C A

12 12
10 – x
y
12 m
B C
y

10 x
A 90 – B 10
16 x
O
12
tan =
16
12
cot = [ tan (90 – ) = cot ] 175. (4) p + q = – r [ p + q + r = 0]
x
squaring both sides
x = 12 tan r × (p2 + q2 + 2pq) = (r2) × r
12 [multiply by r on both sides]
= 12 × =9 x = 9 m.
16 r p2 q2 2pqr r3 ...(i)
cos A cos A Similarly
172. (2) m = ;n =
cos B sin B
p q2 r2 2pqr p3 ...(ii)
2 2
cos A cos A &
(m2 + n2)cos2 B= cos 2 B
cos 2 B sin 2 B
q p2 r2 2pqr q3 ...(iii)
2 1 From equation (i) & (ii) & (iii)
= cos A 2 2
cos 2 B
2
cos B sin B p q r q2 p r r2 p q 6pqr p3 q3 r3
cos 2 A ...(iv)
= n2 Also if p + q + r = 0
sin 2 B
then p3 q 3 r 3 3pqr ...(v)
173. (4) Let ABC and PQR be two triangles with height BD and
From (iv) and (v)
PS respectively. p2 (q + r) + q2 (p + r) + r2 (p +q) = – 3pqr
2
Now, ar (ABC) BD 2p 2 q r 2q 2 p r 2r 2 p q
ar (PQR) QS =–6
pqr
4
2
16 176. (3)
=
9 81
B Q 2.5

2.5
Radius of inner circle (r 1) =
2
Diameter of outercircle = diagonal of square
A C P R
D S 2.5 2
174. (3) (10 – x)2 + y2 = 122 ...(1) Radius of outer circle (r2) =
2
x2 + y2 = 102 ...(2)
r22
From equation (1) and (2) we get Hence, ratio of areas =
r12
x = 2.8 cm. ; y = 9.6 cm.
BC = 2y = 19.2 cm.
Solved Questions B- 161

2 183. (3)
2.5 2 A
2 a a
2 2 60° 2
2
2 :1
2.5 1 a
2 2
60° 60°
B a a C
1 3 1 3 1 2 2
177. (1) x = [Rationalizing]
3 –1 3 1 2
3 2
2x – 1 = 3 Since, area of equilateral triangle = a
4
Squaring on both sides
4x2 – 4x + 1 = 3 4x2 – 4x – 2 = 0 3 2
a = 49 3
2x2 – 2x – 1 = 0 4
Since, the given expression is, a = 14 cm
4x3 + 2x2 – 8x – 3
The above expression can also be written as: Radius of circles = 7 cm
4x3 – 4x2 – 2x + 6x2 – 6x – 3 60 22 1
= 2x (2x2 – 2x – 1) + 3 (2x2 – 2x – 1) Area of each sector = r2 × = × 72 ×
360 7 6
= 2x × 0 + 3 × 0 = 0 [ 2x2 – 2x – 1 = 0]
178. (1) Distance covered in 1 revolution = 2 r 77
Let number of revolutions be k =
3
Distance covered in k revolutions = 2 rk
According to questions 77
Area of all 3 circular sectors = 3 × = 77
2 rk = 100 × 2r 3
100 Shaded area = 49 3 – 77
k=
= 49 × 1.73 – 77 = 7.77 cm2
179. (2) a + (p – 1) d = q ...(1) 184. (2) A + B = 90°
a + (q – 1) d = p ...(2)
From eqn (1) & (2) tan A tan B tan A cot B sin 2 B
(p – q) d = q – p d = – 1
Now eqn (1) becomes
sin A.sec B cos2 A
a + (p – 1) (–1) = q cot B tan B cot Bcot B cos2 A éê B = 90°- A ùú
cos2 A êë\ sin B = cos Aúû
=
a p q –1 sin A cosecA
mth them is a + (m – 1) d = (p + q – 1) + (–1) (m – 1) 1 cot 2 B
=p+q–m = 1 = cot2 B
180. (2) Sum of numbers possible are 2, 3, 4, ...., 12 1
Perfect square numbers are 4, 9 y 4 x4 y3
Possible cases for 4 = (1, 3), (3, 1), (2, 2)
Possible cases for 9 = (3, 6), (6, 3), (4, 5), (5, 4) 185. (3) x(x y) x
Favourable outcomes = 7 y2 xy x 2
Total number of outcomes = 6 × 6 = 36
7 (y2 x 2 )(y x) y3
Hence probabilitiy = =
36 x(y2 xy x 2 )
181. (3) A x( y 2 xy x 2 )
= =–1
x( y 2 xy x 2 )
h
x 2 bx m 1
186. (1)
ax c m 1
B C D In standard form, given equation is (m + 1)x2 – x(bm + b +
ma – a) + cm – c = 0
Since, BC = h cot
Since roots are equal in magnitude but opposite
Also BD = h cot in signs
Distance between objects = hcot – hcot Sum of zeros = 0
= h (cot – cot ) or bm + b + ma – a = 0
182. (3) By distance formula a b
m
(4 – 1)2 + (q – 0)2 = 52 a b
32 + q2 = 52 q2 = 16 187. (2) 10a + b = (a + b) 4 + 3
10a + b = 3ab + 5
q= 4
6a = 3b + 3
EBD_7332
162
B- NTSE Stage 1 Question Bank

b 1 x2 + y2 + 2xy + x + y = 30
5 (b 1) b 3 b 5
2 (x + y) (x + y + 1) = 30
3 2 3 Let x+ y= t
5b 5 b b b 5 t (t + 1) = 30
2 2
3 2 9 t2 + t – 30 = 0
b b 0 (t – 5) (t + 6) = 0
2 2
3b 1 t = 5 or t = – 6
(b 3) 0 b 0 as a x + y = 5 or x + y = – 6
2 2
b= 3 193. (2) 217x + 131y = 913
a= 2 131x + 217y = 827
Number is 23. Odd prime. From eqn (1) and (2),
d
188. (1) a + b + c + d = 125 a 4 b 4 4c t
4 x = 3 and y = 2
t 4 t 4 (t / 4) 4t 125
x+y= 5
25t
125 t = 20 1
4 194. (1) x =
1
a = 24, b = 16 2
c = 5, d = 80 1
2
189. (2) Let the three digit number be ‘x’ 2 x
So when 34369 and 31513 is divided by x it leaves 1
remainder ‘r’ and quotient p, q respectively x= (2 x)
2
34369 = x . p + r ...(1) 2 (2 x) 1
31513 = x . q + r ...(2) 1
Subtracting equation (2) from equation (1) we get x=
(2 x)
2
x (p – q) = 2856 3 2x
x (p – q) = 119 × 24 3 2x
[ 2856 = 2 × 2 × 2 × 3 × 119] x=
2 (3 2x) (2 x)
Since x is three digit number, let x = 119
3 2x
So on dividing 34369 and 31513 by 119 remainder = 97 x= x2 – 2x + 1 = 0
4 3x
190. (1) Let the greatest four digit number be 9999 (x – 1)2 = 0
LCM of 3, 5, 7, 9 = 315 x= 1
When 9999 is divided by 315, it leaves remainder 234 195. (1) Since, x1 + x7 + x10 = – 6
hence 9999 – 234 = 9765 which is exactly divisible by a + a + 6d + a + 9d = – 6
315 which means it is also divisible by 3, 5, 7, 9 [ x1, x2, x3, ... are in A.P.]
exactly but it should leave remainder 1, 3, 5, 7 when 3a + 15d = – 6
divided by 3, 5, 7, 9 difference between remainder = 2 a + 5d = – 2 ...(1)
required number = 9765 – 2 = 9763 And x3 + x8 + x12 = – 11
191. (2) According to question a + 2d + a + 7d + a + 11d = – 11
ef gh ef gh 3a + 20d = – 11 ...(2)
=
100 2 From equation (1) and (2), we get a = 3, d = – 1
From the given options 4950 satisfies the condition Now
4950 49 50 x3 + x8 + x22 = a + 2d + a + 7d + a + 21d = 3a + 30d
=
100 2 x3 + x8 + x22 = – 21 [ a = 3, d = – 1]
99 196. (2) Since, 2, 5, 8 ... is in A.P. where a1 = 2, d1 = 3
49.5 =
2 and 7, 11, 15 ... is also in A.P. where b1 = 7, d2 = 4
49.5 = 49.5 2 5 8 ...n.terms 23
192. (1) x2 + xy + x = 12 ...(1) 7 11 15 ...n.terms 35
y2 + xy + y = 18 ...(2)
Adding equations (1) and (2) we get
Solved Questions 163
B-

n/2 [2(2) (n 1) 3] 23 Since x2 + 3x + m = 0 has equal roots


b2 – 4ac = 0 9 – 4m = 0
n/2 [2(7) (n 1) 4] 35 m = 9/4
3n 1 23 201. (2) 5 24 = x y
n = 15
4n 10 35
197. (4) In ABC, D, E and F are the midpoints of side BC, CA 5 2 6 = x y
and AB respectively.
5 2 3 2 = x y
Centroid of ABC = Centroid of DEF A
2 2
1 2 3 1 3 4 3 2 3 2 2 = x y
= ,
3 3
2
F E 3 2 = x y
2
= 2, 3 2= x y
3
B D C x = 3 and y = 2
198. (3) x + y = 5 & xy = 6
R Statement-1 is false; Statement-II is true.
202. (3) (cos2 5 + cos2 85) + (cos2 10 + cos2 80)
r + .... (cos2 40 + cos250) + cos2 45 = (cos2 5 + sin2 5) + ....
+ (cos2 40 + sin2 40) + cos2 45
1 1 1
= (1 + 1+ ... + 1) + =8 =8
2 2 2
203. (1) x = – a is a root of x2 + px + q & x2 + mx + n
R2 = 1386 – a 2 – pa q a 2 – ma n
R = 21 (m – p) a = n – q
r2 = 962.5
r = 17.5 n–q
a
Width = R – r = 21 – 17.5 = 3.5 cm m–p
1 2 204. (3) LCM = 14 GCD
199. (4) Volume of cone, V = r h [Given]
3 LCM + GCD = 600
20 6 [Given]
New radius = r + r = r 15GCD = 600
100 5
GCD = 40
6 LCM = 14 × 40 = 560
Similary new height = h
5 LCM × GCD = a.b
2
[Since product of LCM & GCD of two integers is equal
1 6 6h to the product of the integers]
New volume of cone = r 560 × 40 = 80 × b
3 5 5
b = 280
1 36 2 6h 205. (3) (P Q ) = P Q
= r
3 25 5 206. (1) A
216 1
= r2 h
125 3
1 2 N M
= (1.728) r h = 1.728V
3
% Increase in Volume
B C
L
New volume – old volume ALB CNB
=
old volume AB . BN
AL BL AB
1.728V – V BL = ...(1)
CN BN BC BC
= × 100 = 72.8%
V BMC ALC
200. (*) Given options are wrong BM CM BC BC . CL
‘– 4’ is a root of equation x2 + px – 4 = 0 CM = ...(2)
16 – 4p – 4 = 0 AL CL AC AC
4p = 12 p = 3 BMA CNA
EBD_7332
164
B- NTSE Stage 1 Question Bank

BM AM AB AM . AC 1 1
AN = ...(3) 215. (4) = logx (z – y) + logx (z + y)
CN AN AC AB log z y x log z y x
On mulitiplying equation (1), (2) & (3) we get
AL × BL × CM = BN × LC × AM 1
log b a
207. (2) F A T E log a b
A – 3! = 6 First place is finced] = logx (z2 – y2) ...(1)
Since x2 + y2 = z2
E – 3! = 6 [again first place is finced]
z2 – y2 = x2
F A E T – = 1 [first word for letter F] Equation (1) becomes
Rank = 6 + 6 + 1 = 13 logx (z2 – y2) = logx x2 = 2 logx x = 2.
208. (4) Since expression ax2 + bx + c is factorised into two 216. (1) Let f (x) = 2x3 + ax2 + bx + 10
Now since x + 2 is the factor of f (x)
equal real linear factors a (x – p) (x – p) i.e. a (x – p)2 f (– 2) = 0
Hence b2 – 4ac = 0 b2 = 4ac. 2 (– 2)3 + a (– 2)2 + b (– 2) + 10 = 0
209. (2) The above equation is only valid when (x + 4) 0 2 a – b =3 ...(1)
Also (2x – 1) is the factor of f (x)
x – 4.
210. (1) 3x2 + 4 = 0 f (1/2) = 0
2 (1/2)3 + a (1/2)2 + b (1/2) + 10 = 0
4 4
x2 x a + 2b = – 41 ...(2)
3 3
from equation (1) and (2)
which is not possible because square of any quantity
cannot be negative (i.e. always equal or greater than a = – 7, b = – 17
zero). a2 + b2 = (– 7)2 + (– 17)2 = 338.
Hence the given equation has no real roots. 217. (3) a + b= 2c = c + c
211. (1) 9x + 6x = 2.4x ...(1) a – c= c – b a – c = – (b – c)
Let 3x = p and 2x = q a b a b
equation (1) becomes =
a c b c b c b c
p2 + pq = 2q2 p2 + 2 pq – pq – 2q2 = 0
(p – q) (p + 2q) = 0 p–q=0
[ p – 2q as 3x – 2 (2x) b c b (2c b) 2(b c)
2.
3x – 2x = 0 3x = 2x x=0 =
b c b c (b c)
212. (4) f (x) = 2x3 – 3x + 4
f (– x) = – 2x3 + 3x + 4 r
t

f (x) + f (– x) = 4 + 4 = 8. 218. (1) CI = p 1 p


100
c b a
213. (2) Let k = 8
2
2c z 2b y 2a x CI = 7500 1 7500
100
cz by ax
k = ...(1) CI = 8748 – 7500
2cz z 2 2by y2 2ax x 2
CI = Rs. 1248.
x2 y2 z2 219. (3) Let cost price = x
Now; 2 [Given]
by cz cz ax ax by After increase of 40%
x2 = 2by + 2cz ; y2 = 2cz + 2ax and z2 = 2ax + 2by
140 x 7x
From equation (1) selling price =
100 5
cz by ax After the discount of 20%
k=
2ax 2by 2cz 2ax 2by 2cz 2ax 2by 2cz 7x 7x
selling price = 20% of
cz by ax (ax by cz) 1 5 5
k= .
2ax 2by 2cz 2(ax by cz) 2 28x
=
214. (3) Since, 25
log4[log4{log4(log4 x)}] = 0 Since, Gain = 48
log4{log4(log4 x)} = 40 = 1 28x
x = 48
log4 (log4 x) = 41 = 4 25
log4 x = 44 = 256 48
x= 25 x = Rs. 400.
x = 4256 = 2512. 3
Solved Questions B-165

220. (1) Curved surface area of right circular cylinder Again ABC ~ DAC
= 2 rh AB AC
DC 2AA ...(2)
Also curved surface area of sphere = 4 r2 AD DC
According to the question, From (1) and (2)
2 rh = 4 r2 5 AD
BC = = 5 BD.
h = 2r 2
225. (2) We know that;
Volume of cylinder r 2h
r 2 (2r) 3
Now, 2. fi x i
Volume of sphere 4 r3 4
3 3 mean =
fi
221. (2) Let l, b, h be the length, breadth and height of a
rectangular parallelopiped respectively. 12 (x 2) 15x 20 (x 1)
mean = 3x 1
l + b + h = 25 (Given)
Also2 (lb + bh + lh) = 264(Given) (whole surface area) 29 12 (x 2) 15x 20 (x 1)
Since l + b + h = 25 2 3x 1
(l + b + h)2 = 252 x = 3.
l2 + b2 + h2 + 2(lb + bh + lh) = 625 226. (3) Let the third angle be x
l2 + b2 + h2 + 264 = 625 x = 180° – (87° 24 54° + 32° 321 6 )
l2 + b2 + h2 = 361 x = 180° – (119° 56 0 )
Now, length of diagonal = l2 b2 h2 x = 60° 4
= /3 (approx.)
2
227. (2) Since, x sin – y cos = 0
Area = l2 b2 h2
x sin = y cos ...(1)
= l2 + b2 + h2 = 361. Also x sin 3 3
+ y cos = sin cos ...(2)
222. (2) From (1) and (2) we get
P S y cos sin2 + y cos3 = sin cos
Q y cos (sin2 + cos2 ) = sin cos
y = sin
A B x = cos (from (1))
x + y = sin2 + cos2 = 1.
2 2
228. (1) Four positive integers be a, b, c, and d respectively.
In PSQ:
d
PS2 = PQ2 + SQ2 then, a + b + c + d = 125. Let a + 4 = b – 4 = 4c = t
4
( SQ = PA – BQ = r 1 – r2 and PS = AB) t
AB = PQ + (r1 – r2)2
2 2 t – 4 + t + 4 + + 4t = 125
4
PQ = AB 2 (r1 r1 ) 2 PQ = 169 25 25t
125 ; t = 20
4
PQ = 12 cm. a = 20 + 4 = 24
223. (2) b = 20 – 4 = 16
20
P Q c= 5
4
O d = 20 × 4 = 80
229. (3) Required number of squares on a chessboard =
S 88 1 2 8 1
R 12+ 22 + 32 + 42 + 52 + 62 + 72 + 82
We know that OQ × OP = OS × OR 6
8 × 14 = 7 × OR 8 9 17
204.
OR = 16 cm. 6
230. (2) Here, area of square ABCD = 4cm2
RS = OR – OS = 16 – 7 = 9. According to question,
224. (3) Since, ABC ~ DBA Area of ADE = Area of BEC = 1
AB AC AD 4
BD ...(1) Area of DEC = 2
BO AD 2 2
EBD_7332
166
B- NTSE Stage 1 Question Bank

A E B 1 1 3
x3 y3
F
G 1 1
H 3
K
So, cube root is x y3
D C

2 239. (2) 0.23 0.23


Area of DFC = 1
2 0.2323 2323...... + 0.2333333
Area of DGC = 1/2 0.465656565
Area of DHC = 1/4 0.465
1 1 240. (1) Kx 2 2x 1
Area of KDC = = cm2
4 2 8 x 2 0
231. (3) According to question, K( 2 )2 2 2 1 0
xy = z, yz = x. x z = y x 2
x2y2z2 = xyz 2K + 2 + 1 = 0
xyz = 1 2K + 3 = 0
z2 = 1 z = 1, x = 1, y = 1 2K = – 3
xy + zy + zx = 3. 3
232. (1) Volume of cylinder = r2h K=
Total surface area of cylinder = 2 rh + 2 r2 2
Here, height of cylinder =1 unit 3x 2y 13xy ...(i) 5
Then, 241. (3)
4x 5y 2xy ...(ii) 2
____________________
V r2 1 r 1 1
1 15x 10y 65xy
S 2 r 2 r2 2 1 r 2 r 1
233. (3) 101 x 2 + 123 x + 123 = 0 8x 10y 4xy
________________
1 1 123 / 101 23x 69xy
1
123 / 101 23x = 69 xy
234. (4) If x3 + y = 2249, where x and y are natural numbers and x = 3xy
HCF of x and y is not 1.
Here, we put x = 13, 133 = 2197 1
1 = 3y y=
y = 2249 – 2197 = 52 3
HCF ( 13, 52) = 13 1 1
x + y = 13 + 52 = 65 In equation (i) y
3
235. (1) a 6 11 and b 6 11 3x + 2y = 13xy

ab 6 11 6 11 25 5 1 1
3x 2 13x
3 3
a2 b2 6 11 6 11 12 2 13x
3x
2 2 2 3 3
a b a b 2ab 12 10 22
2 13x 13x 9x 4x
a b 22 3x = =
3 3 3 3
236. (3) If 2a2 – 2ab + b2 + 6a + 9 = 0
a2 + 6a + 9 + a2 –2ab + b2 = 0 2 4x
(a + 3)2 + (a – b)2 = 0, a = b = –3 3 3 1
a + b + ab = – 6 + 9 = 3 4x = 2 x
237. (2) The sum of all the terms common to the arithmatic 2
2 1 1
progression 1, 3, 5.......1991 and 1, 6, 11,..... 1991 x= y
LCM ( 2, 5) = 10 4 2 3
1+ (n –1) 10 = 1991 n = 200 h h
242. (1) tan , tan(90 )
200 9 16
sum 1 1991 100 (1992) 199200.
2 h
tan ...(i)
1 1 1 1 9
238. (2) x y 3x 3 y 3 x3 y3 h
cot ...(ii)
16
Solved Questions 167
B-

A C = 9 cm
CF = 4 cm
In CDE ~ CAB
h EC 4 n
...(2)
BC 9
From (1) and (2)
90°–
4 4 n
7m 9m (4 + n)2 = 4 × 9
Eq. (i) × (ii) 4 n 9
n2 + 8n + 16 = 36
h2 n2 + 8n = 36 – 16 = 20
1 n2 + 8n – 20
16 9
2
h = 16 × 9 n2 + 10n – 2n – 20
n (n + 10) – 2(n + 10)
h = 16 9 = 4 × 3 = 12 mt. n–2 n=2
243. (3) log5 [log 2 (log3 x)] 0 4 + n = 4 + 2 = 6 cm
1 = log2 (log3x) 3 15
21 = log3x 249. (1) a :b= =
5 25
32 = x x=9
244. (2) x = – 6, y = 3 5 15
Angle between them is 90° a:c= =
7 21
y (b – c) : (b + c)
(25 – 21) : (25 + 21)
y=3 4 : 46
1
250. (1) mth term of an AP =
n
x 1
x=–6 O nth term of an AP =
m
mnth term = ?
245. (2) 6, 8, 2x – 5, 2x – 1, 15, 17, 20, 22 1
2x 1 15 a + (m – 1)d =
n
= 14
2 1
2x + 14 = 14 × 2 = 28 a + (n – 1) d =
2x = 28 – 14 = 14 m
14 1
x= 7 a=d=
2 mn
246. (3) A' = {1, 2, 7, 8} 1
B' = {2, 4, 6, 8} mnth term =
mn
A' – B' = {1, 7} 251. (2) According to question
1 2 6 3
247. (3) c 2c 9 m 4n4 m2 n 2 m 2n 2 (mn)k
3
(mn)2 × (mn)1/3 × (mn)2/3 = (mn)k
1 2 1 2 1 2
= c 6c 27 = (c 9 c 3c 27) 2 k
3 3 mn 3 3 mn
1 1 3 k
= c(c 9) 3(c 9) = (c 3)(c 9) mn mn
3 3
k=3
1 252. (3) Let total students in a group = x
= c 3 (c 3) < 20 = 10% of total students
3
248. (2) In CFE ~ CDB 20 – 40 = 20% of total students
40 – 60 = 35% of total students
EC 4 60 – 80 = 20% of total students
...(1)
BC 4 n 80 – 100 = 30% of total students = (100 – 85) % =15%
x 15
A 30
100
9 cm x = 200 students.
D
So, (40 – 60) will have higher no. of students.
F
4c
m

B E C
EBD_7332
168
B- NTSE Stage 1 Question Bank

253. (4) According to question. Hecne, the variance of the resulting observations is =
1/20. 400
One root of quadratic equation = 3 2
= 20. = 22.5
another will be = 3 2 258. (1) According to question,
sum of roots = 6 P ( –1) = (–1 –1) = –2
products of roots = 7 P ( –1) = (1)2 +1 = 2
x2 – (sum of the roots) x + product of the roots = 0 R1 + R2 = 0
x2 – 6x + 7 = 0 –2 + 2 = 0. A
254. (2) According to question 259. (4) Here,
A
ar.AXY 1
4
ar . ABC 2
D X Y
4 5 2
AX 1
90° AB 2
C
B AX 1 B C
4 5 4 AB 2
AC 4 5
AC = 20 AX BX 2
BC = 8 5 AX 1
Area of ABC = 80 sq.m BX 2
1
3
2 AX 1
4 a 4a 2
2 BX 2 1
255. (1) Required percentage = 100
2 AX 1
4a
= 125%. AX 1
D (300 100) BX 2 –1
256. (1) T =
S 25
= 2 1
400 260. (2) According to question,
= 16 seconds
25 1
257. (3) Let the given observations be x1, x2, ....... x20 and xM 5 2 6
be their mean. 5 2 6
Given that n = 20 and variance in 5. 5–2 6 5 2 6
20 10
1 2
So, x xM 5
20 i 1 1 a4
10
20
2 a2 2 B
x xM 100.(1) 261. (4) According to question, D
i 1
If each observation is multiplied by 2, then the new b a
AD2 = AB2 –22
observations are yi = 2xi, i = 1, 2, .....20. AD2 = 42 – CD2
So, xi = yi/2 AB2 – 22 = 42 – CD2 C
4 A
1 20 a2 + b2 = 20.
Hence, ym = 20 yi
i 1 5 x2 5 k
262. (4) =– x x2 2
1 20 20 2 2 2 2
2 xi 1
20 i 1 ym = 2 xi
20 i 1 k
This gives us xM = 1/2 yM =
2
Substituting the values of xi and xM in (1) we have Now, ( + )2 = 2 + 2 + 2
20 2
1 1 25
yi yM 100 =( 2+ 2+ )+
i 1 2 2 4
20 2 25 21 k
1 =
yi yM 100 4 4 2
i 14 k=2
20 2 263. (4) Let three consequtive terms of the A. P. be (a – d), a
yi yM 400 and (a + d).
i 1
a – d + a + a + d = 21
Solved Questions B- 169
3a = 21 a = 7 3p 4q 3r 4s
Hence middle term = 7
264. (2) Sp = Sq 3p 4q 3r 4s
p q 6p 6r
[2a + (p –1)d] = [2a + (q – 1 ) d)] [By componendo and Dividendo]
2 2 8q 8s
2 2
2a (p – q) + d [(p – q ) – (p – q)] = 0 ps = qr, ps : qr = 1 : 1
(p – q) [2a + d (p + q – 1)] = 0 270. (2) x2 – 4 0 x2 4
p q , 2a + d (p + q – 1) = 0 –2 x 2
p q 271. (1) x2 x 2 x=x–2
Now Sp+q = [2a + (p + q – 1)d] = 0
2 0 = – 2, Which is not true
p q Hence no solution.
Sp+q = 0 =0 272. (4) 1 + 5 + 3 = 180° ...(i) (Angle Sum oroperty)
2 2 + 6 + 4 = 180° ...(ii) (Angle Sum oroperty)
1 1 Adding (i) and (ii), we get
265. (1) 1 + 2 + 3 + 4 + 5 + 6 = 180° + 180° = 360°
1 cot 2 1 tan 2
273. (2) Mean of 100 observations = 24
1 1 Then sum of 100 observations = 24 × 100 = 2400
= 2 After adding 4 to each of the observations
cos sin 2
1 1 New sum = 2400 + 4(100) = 2800
sin 2 cos 2 After multiply 2.5 to all the observations.
New sum = 2.5 × 2800 ( 2.5 will be common to all observations)
sin 2 cos 2 New sum = 8000
=
sin 2 cos 2 cos 2 sin 2 8000
New mean = 80 .
sin 2 cos 2 100
= =1 m
274. (2) 2 – 2 m – 1 =4
sin 2 cos 2
2m
x 2m 4 2 2m 2m 4 2
266. (1) cos43° = 2 2
= sin47° 2 1
x y
[ cos(90° – 47°) = sin 47°] 2m [2 1] 4 2 2m 8
Now, cos247° = 1 – sin2 47°
2m 23 m 3
x2 y2 m 3
cos247° = 1 m 3 27
x2 y2 x2 y2 3x 1
275. (1) 2 64
y
cos 47° = 23x (26 ) 1
x2 y2
23x 2 6
x 3x 6
sin 47 x2 y2 x x 2
Now, tan 47° = 50x 100
cos 47 y y
and 10y = 0.01
x 2 y2 1
267. (4) f (11) = (11)6 – 10 × (11)5 – 10 × (11)4 – 10 × (11)3 – 10 × 10y =
100
(11)2 – 10 × (11) + 10
1
= 115 (11 – 10) – 10 × 114 – 10 × 113 – 10 × 112 – 10 × 11 1
+ 10 (50x)–1 × (10y)–1 = (–100)–1 ×
100
= 114 (11 – 10) – 10 × 113 – 10 × 112 – 10 × 11 + 10
= 113 (11 – 10) – 10 × 112 – 10 × 11 + 10 1 100
1 =
= 112 (11 – 10) – 10 × 11 + 10 100 1
= 11 (11 – 10) + 10 276. (4) Let d1 be the common difference of 1st A.P. and d2 be
= 11 + 10 = 21 the common difference of 2nd A.P.
268. (4) a3 + b3 – c3 + 3abc
y x 3d1 x 4d 2
= (23)3 + (27)3 – (50)3 + 3 × 23 × 27 × 50
= (23)3 + (27)3 + (–50)3 – 3 × 23 × 27 × (–50) 3d1 4d 2
Now, 23 + 27 + (– 50) = 0
(23)3 + (27)3 + (– 50)3 – 3 × 23 × 27 × (– 50) = 0 4
d1 d2
[Since if a + b + c = 0, then a3 + b3 + c3 – 3abc = 0] 3
3p 4q 3p – 4q 4
269. (2) (a 2 a1) = (b3 b2 )
3r 4s 3r 4s 3
EBD_7332
170
B- NTSE Stage 1 Question Bank

4 1 1
(a 2 a1) = (b2 b3 ) sin 2 cos 2 2sin 2 1
3 2 2
a 2 a1 4 3 3 3
2 sin 2 sin 2 sin
b 2 b3 3 2 4 2
277. (2) 4x – 3(2x + 3) + 128 = 0 60o
(2x)2 – 24.2x + 128 = 0
y2 – 24y + 128 = 0 (a, b) (c, d)
282. (2)
y2 – 8y – 16y + 128 = 0 A B
y(y – 8) – 16(y – 8) = 0
(y – 8)(y – 16) = 0
y = 8 or y = 16
Now y = 8 2x = 23 x=3
y = 16 2x = 24 x=4
sum of roots = 3 + 4 = 7
278. (4) sinx + cosecx = 2 O
1 Let A = (a, b)
sinx + =2 B = (c, d)
sin x
In right AOB,
sin 2x + 1 = 2sinx sin 2x – 2sinx + 1 = 0 OA2 + OB2 = AB2
(sinx – 1)2 = 0 a2 + b2 + c2 + d2 = (a – c)2 + (b – d)2
sinx = 1 cosecx = 1
a2 + b2 + c2 + d2 = a2 + c2 – 2ca + b2 + d2 – 2bd
sin10x + cosec10x = 2
2 (ac + bd) = 0
ac + bd = 0
279. (2) r
s a1 b1 c1
Here = Area of the triangle and 25 = perimeter of the 283. (1) a 2 b2 c2
triangle
a.b 1 1 1 2 3
1 ab 284. (2) x=
2s 2 2 3 2 3 2 3
a + b + 10 = a.b
2 3
a + b = ab – 10 .........(i) x= 2 3
(a + b)2 = (ab –10)2 4 3
a2 + b2 + 2ab 1 1 2 3 2 3
= a2b2 –20ab + 100 now,
a2 + b2 = 100 10 x 2 3 2 3 4 3
a
2ab = a2b2 – 20ab r 1
a2b2 – 22ab = 0 2 3
ab(ab – 22) = 0 x
ab 0, ab = 22 1
perimeter = a + b + 10 = ab = 22 b [Using (i)] now x2 (2 3)2 (2 3)2
2
x
7 3 126 39
280. (4) CP 18 13 = 4 3 4 3 (4 3 4 3)
10 10 10
=7 4 3 7 4 3
165
= ` 16.5 =4 3 4 3 8 3
10
SP = ` 18.15 285. (3) P
Gain = ` 1.65
Gain 1.65
% gain 100 100 = 10%
CP 16.5
sin cos
tan cot cos sin
281. (1) 2 2
tan cot sin cos M N
cos sin
sin 2 cos 2
Q R
sin .cos 1
2 Since MN||QR
sin 2 cos 2 sin 2 cos2 PMN = PQR [Corr. Angles]
sin .cos Hence PMN ~ PQR [By AA]
Solved Questions 171
B-

291. (3) (a + b)3 – (a – b)3


PM 2 Area of PMN 1 a3 + b3 + 3a2b + 3ab2 – (a3 – b3 + 3ab2 – 3a2b)
PQ 2 Area of PQR 2 2b3 + 6a2b = 2b(b2 + 3a2)
292. (1) + = – b/a = c/a
PM 1
1 1 a
PQ 2
c
PM 1
1– 1 b a
PQ 2 a2 bc
a c
PQ PM 2 1
a 2 bc 0
PQ 2
AB BC AC
QM 2 1 293. (4)
DE DC CE
PQ 2 4 BC AC
286. (4) Let a and b are two given number and b > a
3 4.8 4.2
a b BC = 6.4 AC=5.6
then = 2 a + b = 4 ...(i)
2 1 cos 2 1 cos 2 sin 2
G.M. = a(b 1) = 2 294. (1) 1
sin 2 sin 2 sin 2 sin 2
a (b + 1) = 4 2
295. (4) ax + bx + c = 0
(4 – b) (b + 1) = 4
4b + 4 – b2 – b = 4 b b 2 4ac b b 2 4ac
3b – b2 = 0 b (3 – b) = 0 root form =
2a 2a 2a
b 0, b= 3 296. (3) a + 2d + a + 6d = 6
a=1 2a + 8d = 6
2ab 2 1 3 3 a + 4d = 3 …(1)
Now, H.M. = (a + 2d)(a + 6d) = 8
a b 1 3 2 (3 – 2d)(3 + 2d) = 8 from (1)
x a x b x c 9 – 4d2 = 8
287. (3) 1 1 1 0 1 = 4d2
b c c a a b
x a b c x a b c x a b c 1
0 d=
b c c a a b 2
297. (3) AOB ~ COD
1 1 1
(x a b c) 0 AO OB AB
b c c a a b
x+a+b+c=0 OC OD CD
x = –(a + b + c) 2 x 2
288. (4) sin x + cosec x = 2 , x 20
5 2x 5
1 298. (2) Equation of line passing (0, y) and (x, 0) will be
sin x 2
sin x X Y
2
sin x + 1 = 2 sinx 1 it passes (1, 1) so.
sin2x – 2 sin x + 1 = 0 x y
(sin x – 1)2 = 0 1 1
sin x = 1 1 x + y = xy
x y
cosec x = 1
sin10x + cosec10 x = 2 (0, y)
289. (2) Since, BOD is a straight line.
Then,
BOC + COD = 180° (1, 1)
COD = 180° – BOC = 180° – 115° = 65°
In DOC, (x, 0)
OCD + ODC + COD = 180°
OCD + 80° + 65° = 180°
OCD + 145° = 180°
OCD = 180° – 145° = 35° 3
Since, ODC OBA 299. (1) sin (A + B) = A + B = 60°
Then, OAB = OCD = 35° 2
n(E) 3
290. (1) P(E) n(S) 4 / 6 2 / 3 cos (A – B) = A – B = 30°
2
So, A = 45°, B = 15°.
EBD_7332
172
B- NTSE Stage 1 Question Bank

300. (2) Diagonal = 306. (3) D ABC ~ D AEF


2 side
ar.(ΧAEF) æç AE ö÷
2
a + b = 2 side 1
<ç ÷ <
a+b ar.(ΧABC) çè AB ø÷ 4
or side = Þ Area of quadrilatral BEFC = ar. (D ABC) – Ar. (D AEF)
2
2 1 3
æa+bö (a + b) 2 = ar. (D ABC) = ar. (D ABC) = ar. (D ABC)
Area = side2 = ç ÷ = 2 4 4
è 2 ø 307. (4) Ð PSQ = ÐQPT = 70°
301. (4) Let CP of 1 gm = ` 1 (Angles in alternate segment of circle are equal)
So CP of 800 gm = ` 800 S
SP of 800 gm = CP of 1000 gm = 1000 Rs. Q
70°
Profit = 1000 – 800 = 200 O 110°
200 R
Profit % = × 100% = 25%
800 P 70° T
2
P´r´2 \ Ð PRQ = 180° – Ð PSQ = 180° – 70° = 110°
æ r ö
302. (3) P ç1 + ÷ –P = 618 and = 600 308. (3) r2 = x2 + 25 A 10 B
è 100 ø 100 r2 = (17 – x)2 + 144 x
r
é r2 2r ù 600 ´100 Now x2 + 25 = (17 – x )2 + 144 0
P ê1 + + –1ú = 618 and P = Þ x = 12 r
ë 100 ´ 100 100 û 2´ r 2
r = 144 + 25
17–x
B 24 D
600 ´ 100 é r 2 ù r = 13 cm
\ ´r ê + úû = 618 309. (2) Length of box = 25 cm 5 5
2´r ë 100 ´ 100 100 5 5
Breadth of box = 15 cm 25
æ r ö Height of box = 5 cm 5 5
Þ 300 çè 100 + 2 ÷ø = 618, Þ 3r = 18% Þ r = 6% Volume of box = 15 × 25 × 5 = 1875 cm3 5 5
303. (1) log12 27 = a 2 x 2 -11x +30
35
310. (1) (x –7x + 11) =1
log 27 log 33 2 2
If x – 7x + 11 = 1 or x – 11x + 30 = 0
=a Þ =a
log12 log 3 + log 4 Þ x2 – 7x + 10 = 0
= 3log3 = a[2log2 + log3] = 3log3 – alog3 = 2a log2 x = 2, 5 or Þ x = 5, 6
311. (3) Consider tan2 a tan2 b + tan2 b tan2 g + tan2 g tan2 a
2a log 2 + 2tan2 a tan2 b tan2 g = 1
= log3 = ......(1)
3– a
sin 2 a sin 2 b sin 2 b sin 2 g sin 2 g sin 2 a
log16 Þ ´ + ´ + ´
4log 2 cos 2 a cos 2 b cos 2 b cos 2 g cos 2 g cos 2 a
Now log616 = = ......(2)
log 6 log 2 + log 3
sin 2 a sin 2 b sin 2 g
+2 . . =1
4 log 2 4(3 – a) cos 2 a cos 2 b cos 2 g
= [using (1)] =
log 2 +
2a log 2 3+ a Þ sin2 a sin2 b cos2 g + cos2 a sin2 b sin2 g + sin2 a cos2
3–a b sin2 g + 2sin2 a sin2 b sin2 g
= cos2 a cos2 b cos2 g
304. (2) 7 7 Þ sin 2 a sin 2 b (1 – sin 2g) + (1 – sin 2a) sin 2b sin 2 g
7 7 + sin2a (1 – sin2b) sin2g = 2 sin2a sin2b sin2g
(Q sin2q + cos2q = 1)
7 7 Þ (1 – sin2a) (1 – sin2b) (1 – sin2g)
Þ sin2a sin2b – sin2a sin2b sin2g + sin2b sin2g – sin2a sin2b
7 7 sin2g sin2a sin2g – sin2a sin2b sin2g + 2 sin2a sin2b sin2g
Area of space enclosed by the circles = 1 – sin2g – sin 2b + sin 2b . sin2g – sin 2a + sin 2a sin 2g +
= Area of square of side 14 cm sin 2a sin 2b – sin2a sin 2b sin 2g
– 4 (Area of quardrant of radius 7 cm) Þ sin2a + sin2b + sin2g = 1
312. (2) 3 sin q + 5 cos q = 5
1 22 (3 sin q + 5 cos q)2 = 25
= (14)2 – 4× ´ ´ 7 ´ 7 = 196 – 154 = 42 cm2 9 sin2 q + 25 cos2 q + 30 sin q cos q = 25
4 7
9(1 – cos2 q) + 25(1 – sin 2 q) + 30 sin q cos q = 25
16 æç AB ö÷ æ BC ö÷2
2
<ç ÷÷ < çç ÷
9 cos2 q + 25 sin2 q – 30 sinq cosq = 9
305. (2)
9 çè PQ ø èç QR ø÷ (5 sin q – 3 cos q)2 = 9
5 sin q – 3 cos q = ± 3
16 æç AB ö÷ 16 æ 12 ö÷
2 2
Þ < çç ÷÷ and < çç ÷÷ 313. (2) 2010
2 7 - 3 3 ´ 4020 55 + 12 21
9 è 18 ø 9 çè QR ø
2010
4 AB 4 12 = 2 7 - 3 3 ´ 4020 (2 7 + 3 3) 2
Þ 3 < 18 and 3 < QR
Þ AB = 24 cm, QR = 9 cm
Solved Questions 173
B-

2010
319. (1) If the centre of the clock is origin and x = 0 or y-axis is
= 2 7 3 3 2010 (2 7 3 3) along minute hand at 4 : 30 pm then hour hand can have
equation.
= 2010 (2
7 3 3)(2 7 3 3) 1 There are two cases for x and y-axis and hence for angle of
314. (4) On dividing x4 – 11x3 + 44x2 – 76x + 48 by x2 – 7x + 12 elevation of minute hand as shown in the fig. (i) and (ii). But
we obtain the quotient in both the cases = 135°
x2 – 4x + 4, On comparing, we get Ax2 + Bx + C = x2 – 4x + 4 Hence required equation,
A = 1, B = – 4, C = 4 y = (tan 135°) x y = – x x + y = 0
Hence, Required descending order is C, A, B
315. (1) ADE is isosceles (as AD = AE given) Y Y
So ADE = AED A 11 12 1 11 12 1
180° – ADE = 180° – AED 10 2
10 = 135° 2
ADB = AEC
Now in ADB and AEC X 9 X X 9 X
2y+3 43 O 3 O 3
= 135°
8 4 8 4
BAD = EAC (given)
AD = AE (given) 7 6 5 7 6 5
10 x–1
ADB = AEC (proved) B D E C Y Y
ADB AEC (ASA congruence)
Fig (i) Fig (ii)
So AB = AC and BD = CE (cpct)
or 2y + 3 = 43 and x – 1 = 10 8 r r
so y = 20, x = 11. 320. (1) From the similarity of triangles
10 6
316. (2) O is the centre of circle. K and L are mid points of 48 – 6r = 10r
Chords AB and CD respectively. r = 3.
B 6 cm
K Fraction of water overflows
A r cm
OK AB and OL CD
As AB = CD Volume of sphere

(8-r)cm
O = Volume of cone
OK = OL. (equal chords are 10 cm
equidistant from centre) C 4 8 cm
So, OKL is an isosceles. L
D (3)3 3
3
OKL = OLK = 25° (given) = =
1 8
Therefore LKB = OKL + OKB = 25° + 90° = 115° (6) 2 (8)
3
4 321. (2) So that 2575 d 568 may be divisible by 54 and 87 it
317. (3) Given the length of tangent L = r, where r is the
3 should be divisible by 2, 27 and 29. The number is always
radius. divisible by 2. So as to make it divisible by 27, it must be
B divisible by 3 at least. So d = 1, 4 or 7. Hence d = 7
L 3L/4 2
3cos 43 cos37 cosec53
3L x 322. (4)
or r = . A sin 47 tan 5 .tan 25 .tan 45 .tan 65 .tan 85
4 C 3L/4 O
2
3sin 47 cos 37 1
=
sin 47 sin 53 tan 5 tan 25 (1) cot 25 cot 5
2 2
3L 3L
From the figure L2 x = 32
sin 53 1
=9–1=8
4 4 sin 53 tan 5 tan 25
2 2 tan 5 tan 25
3L 3L 3L 323. (4) By property of similar triangle, we have
L2 x2 2x
4 4 4 AB BC AC 4 BC AC
= = = =
3L DE DC CE 3 4.8 4.2
x2 2 x L2 0 BC = 6.4 and AC = 5.6
4
324. (4) E H
2x2 + 3Lx – 2L2 = 0 or (x + 2L)(2x – L) = 0
L A B
x = – 2L or
2
L O1 O2
We reject x = – 2L. Hence x = .
2
C D
318. (2) Let there be n numbers in a set.
They are x1, x2, ............, xn. G F
So, x1 + x2 + ... + xn = S. There are four common tangents, AB, CD, EF and GH.
According to question new sum is
{5(x1 + 20) – 20} + {5(x2 + 20) – 20} + ... + {5(xn + 20) – 20}
= 5(x1 + x2 + ... + xn) + 80 + 80 + ... 80 (n times ) = 5S + 80n
EBD_7332
174
B- NTSE Stage 1 Question Bank

1 335. (4) Median = 525, Mode = 500


1 x2 +
x+ We have, Mode = 3 median – 2 mean
325. (3) Given : x =M To find : x2 Þ 500 = 3(525) – 2 mean
2 2 1075
1 1 Þ 2 mean = 1575 – 500 Þ mean = = 537.5 .
Þ x+ = 2M Þ x 2 + 2 + 2 = 4M 2 (on squaring) 2
x x 336. (4) Only one straight line can be drawn from one point to
any other point.
1
Þ x2 + = 4M 2 - 2 S T
x2 P Q 130°
337. (3)
1 50°
x2 + 2
110°
x 2 = 4M - 2 = 2M 2 - 1
\ Mean = R
2 2 ÐQRS = 110° – (50°) = 60°
326. (2) 5 × 10 × 25 × 40 × 50 × 55 × 65 × 125 × 80 338. (1) The bisectors of angles of a parallelogram makes a
= 5 × 2 × 5 × 52 × 23 × 5 × 2 × 52 × 11 × 5 × 13 × 5 × 53 figure which is a Rectangle.
× 24 × 5 = 29 × 513 × 11 × 13 = (2 × 5)9 × 54 × 11 × 13 100 100 100
As we know that zeroes are formed by the product of a 2 339. (1) = 50, 2 = 25, 3 = 12.5 » 12
2 2 2
and a 5 i.e. 2 × 5. Hence number of zeroes depends on the (By taking only integer part)
number of pairs of 2’s and 5’s that can be formed in the
given product. Since 9 pairs of 2’s and 5’s are formed in the 100 100
= 6.25 » 6, » 3.125, » 3
given product, hence there will be 9 zeroes in the given 24 25
product. 100
327. (4) Only option ‘d’ is correct. Because when we multiply = 1.5625 » 1
26
with x + y, x – y and x5 + x4y + x3y2 + x2y3 + xy4 + y5 Now, take only integer parts and add them
We will not be able to remove square or cube root. So, we get 50 + 25 + 12 + 6 + 3 + 1 = 97
328. (2) According to the question, we have 340. (2) Let x + y + z = 1 ……(i)
x x x x2 + y2 + z2 = 2 ……(ii)
A= ,B= ,C= ,D=7 Þ (x + y + z)2 – 2(xy + yz + zx) = 2
2 4 5
Total no. of cows = x Þ 1 – 2 (xy + yz + zx) = 2
x x x 10 x + 5 x + 4 x –1
\ + + +7=x Þ = x-7 Þ xy + yz + zx = ……(iii)
2 4 5 20 2
Þ 19x = 20x – 140 Þ 140 = x Also, given
329. (3) Let CP of 1 pen = x x3 + y3 + z3 = 3
CP of 9 pen = 9x and CP of 8 pen = 8x Þ x + y3 + z3 – 3xyz = 3 – 3xyz
3

According to the question, SP of 8 pens = 9x Þ (x + y + z) (x2 + y2 + z2 – xy – yz – zx) = 3 – 3xyz


Þ 2 – (xy + yz + zx) = 3 – 3xyz
9x - 8x 100 1
% profit = ´ 100 = = 12 1 5
8x 8 2 Þ 2 + = 3 – 3xyz Þ – 3 = – 3xyz
330. (1) Given, MP = 600 and SP = 450 2 2
1 1
M.P. - S.P. 150 Þ – = – 3xyz Þ xyz =
Discount = ´ 100 = ´ 100 = 25% 2 6
M.P 600 341. (3) Let whole numbers be a, b, c & d
331. (4) MS is parallel to QN Given a + b + c = 180 ……(1)
\ (4) is not true b + c + d = 197 ……(2)
332. (1) Given equations are c + d + a = 208 ……(3)
2x + 3y = 7 and (a – b)x + (a + b)y = 3a + b – 2 d + a + b = 222 ……(4)
For infinite solution (4) – (1) = d – c = 42 ……(5)
a1 b1 c1 2 3 7 (4) – (3) = b – c = 14 ……(6)
= = Þ = = (4) – (2) = a – c = 25 ……(7)
a2 b2 c2 a - b a + b 3a + b - 2
(3) – (2) = a – b = 11 ……(8)
2 3 If a = x, b = x – 11 (from 8)
Þ = Þ 2a + 2b = 3a – 3b Þ a = 5b
a -b a +b c = x – 25 (from 7)
3 7 d = c + 42
= = x – 25 + 42 = x + 17 (from 5 & 7) b + c + d = 197
a + b 3a + b - 2
= x – 11 + x – 25 + x + 17 = 3x – 19
Þ 9a + 3b – 6 = 7a + 7b Þ 2a – 4b = 6
3x = 197 + 19 = 216 Þ x = 72
Þ 2(5b) – 4b = 6 [Q a = 5b]
a = 72, b = 61, c = 47, d = 89
Þ 10b – 4b = 6 Þ 6b = 6 Þ b = 1 Þ a = 5
333. (3) By using property of trapezium, 2f ( n ) + 1
342. (2) Given f (n + 1) =
OA OB 2 x-2 2
= Þ = Þ 4x + 10 = 5x – 10 Þ x = 20 and f (1) = 2 then f (101) = 2
OC OD 5 2x + 5
334. (3) Length of shadow decreases and will be zero. 2f (1) + 1 5
for n = 1, f (2) = =
2 2
Solved Questions 175
B-

2f ( 2 ) + 1 347. (3) Centre of a cyclic quadrilateral coincides with that of


for n = 2, f (3) = =3 circle and diameter is the diagonal so, angles are all right
2 angle.
2f ( 3 ) + 1 6 +1 7 So, it is either rectangle or square
f (4) = = = a2 + b2 = 22 = 4
2 2 2 AB · BC · CD · AD = a.b.a.b = a2b2 A B
5 7 If a ¹ b, let b = 2a
So, f (1), f (2), f (3), f (4), ……, 2, ,3,
2 2 4 b
So, a2 + (2a)2 = 4 Þ a2 =
3+ n 3 + 101 5
f(n) = f(101) = = 52 AB · BC · CD · AD = a2b2 = a2 (2a)2 = 4a4
2 2 a
343. (3) Consider, 2 D C
sin2 1° + sin2 2° + …… + sin2 89° æ 4ö 64
4(a2)2 Þ 4 ç ÷ = >4
= sin2 1° + sin 2 2° + …… + [sin (90 – 2)]2 5
è ø 25
+ [sin (90 – 1)]2
( 2)
4
= sin2 1° + sin2 2° + …… + cos2 2° + cos2 1° If a = b, then 2a2 = 4 Þ a = 2 and then a4 = =4
= [(sin2 1° + cos2 1°) + (sin 2 2° + cos2 2°) + …… So, AB · BC · CD · DA ³ 4
44 terms] + sin2 45° It is rectangle
1 1 1 348. (4) Let r be the radius and h be the height of the cone.
= (1 + 1 + …… 44 terms) + = 44 + = 44
2 2 2 1 2
344. (2) Since A + B = p – C we have \ Volume of the cone = pr h
3
tan (A + B) = tan (p – C)
r ´ 20 r+r 6r
tan A + tan B Now, New radius = r + = =
Þ = – tan C > 0 ……(i) 100 5 5
1 - tan A tan B
(Q C is obtuse angle Þ tan C < 0) h + h ´ 20 6h
New height = =
p 100 5
Since, A and B are each less than , it follows that 2
2 1 æ 6r ö 6h
tan A + tan B > 0 New volume = pç ÷ ´
3 è 5ø 5
Hence (i) will hold if 1 – tan A tan B > 0
Þ tan A · tan B < 1 é 1 æ 36r 2 ö 6h 1 2 ù
ê pç ÷´ - pr h ú
345. (3) A êë 3 è 5 ø 5 3 úû
% increased = ´ 100
1 2
pr h
3
a a
43 43 1 2 é 216 ù
pr h ê - 1 ´ 100
3 ë 125 úû 91
43 = = ´ 100 = 72.8%
1 2 125
B C pr h
a 3
Given, area of circle = 48p
1
Þ pr2 = 48p Þ r2 = 48 Þ r = 4 3 units 349. (1) Let x + =1 ...(1) and y - 1 = 1 ...(2)
y z
We know, distance of centroid on median from any vertex is
From (1) we have, xy + 1 = y
2 a Put value of y in (2),
of height which is equal to .
3 3 1
xy + 1 – = 1
a a z
Also =rÞ = 4 3 Þ a = 12 xyz = 1
3 3
Hence, perimeter of triangle = 3 × a = 36 350. (3) 40% discount on ` 1000 = ` 400 ...(1)
346. (1) Consider (2005)2002 + (2002)2005 Equivalent discount of successive discounts of 30% and
5 æ 30 ´ 10 ö
When we divide 2005 by 200 we get remainder as 10% is ç 30 + 10 - ÷ % = 37%
200 è 100 ø
Now, 37% discount on ` 1000 = ` 370 ...(2)
5 Required difference = 400 – 370 = 30
Now, Remainder = <1
200
and (Remainder)0 tends to 0 (Q Remainder is small) 351. (1) Consider sec 2 q + cosec2 q
Similarly, when we divide 2002 by 200 we get remainder as
= sec 2 q + cosec 2 q + 2 - 2
2
200 = sec 2 q - 1 + cosec 2 q - 1 + 2 = tan 2 q + cot 2 q + 2
Again, (Remainder)0 tends to 0.
\ Required answer = 0 = (tan q + cot q)2 = tan q + cot q
EBD_7332
176
B- NTSE Stage 1 Question Bank

352. (2) Let length of wire be = l 359. (3) Let x = a cos3 q and y = a sin3 q
4a = l 1/ 3 1/ 3
æ xö æ yö
l l l Þ cos q = ç ÷ and sin q = ç ÷
a = ; 2pr = l, 2r = = èaø èaø
4 p 3.14 We know,
Since diameter of the circle is longer than length of a side of cos2 q + sin2 q = 1
the square, therefore circle is larger. 2 2
353. (3) In cyclic quadrilateral sum of opposite angles = 180° éæ x ö1/ 3 ù éæ y ö1/ 3 ù 2 2 2
Þ ÐA + ÐC = 180° Þ 2x – 1 + 2y + 15 = 180° êç ÷ ú + êç ÷ ú = 1 \ x 3 + y 3 = a 3
ëè a ø û ëè a ø û
Þ x + y = 83 ...(i)
and ÐB + ÐD = 180° 360. (2) ar (square ABCD) = 2 × 2 = 4 sq. cm
A P B
Þ 4x + y = 182 ¼ 1 2
x = 33 (By using (i)) ar (sector APS) = ´ p ´ 1
4
y = 50 S
Thus, ÐA = 65°; ÐB = 55° ; ÐC = 115°; ÐD = 125° p Q
=
So, greatest angle = 125° 4
354. (3) S.I. , C.I. ¼
ar (sector APS) = ar D R C
r = 5%, r = 10%
n = 3, n = 3 ¼ = ar (sector CQR)
(sector BPQ) ¼
3
P ´ n ´ r 3P æ 10 ö
S.I. = = , C.I. = P ç 1 + ÷ -P ¼ =p
= ar (sector DRS)
100 20 è 100 ø 4
Given, C.I. – S.I. = 905.
p
éæ 11 ö3 Area of non-shaded region = 4 ´ = p sq. cm
é 11 3 ù 3P 3ù
Þ P êæç ö÷ - 1ú - = 905 Þ P êç ÷ - 1 - ú = 905
4
ëè 10 ø û 20 ëè 10 ø 20 û \ Area of the shaded region
= Area of square ABCD – area of the non-shaded region
é1331 - 1150 ù 905 ´ 1000 = (4 – p) sq. cm
Þ Pê = 905 Þ P = = ` 5000
ë 1000 úû 181 361. (2) A = sin2 q + cos4 q = sin2 q + cos2 q . cos2 q
= sin2 q + cos2 q (1 – sin2 q) = sin2 q + cos2 q – sin2 q cos2 q
x
355. (4) Given tan 9° = 1 2 1
y = 1 - (2 sin q cos q) = 1 - sin 2 2q
4 4
sec 2 81° sec 2 81° Min. (sin 2 2q) = 0
= Max. (sin2 2q) = 1
1 + cot 2 81° cos ec 2 81°
= tan2 81°= [tan(90° – 9°)]2 = (cot 9°)2 1 1 3
Min (A) = 1 - . Max (sin 2 2q) = 1 - .1 =
4 4 4
1 y2
= cot2 9° = = 1 1
2
tan 9° x2 Max. (A) = 1 - . Min. (sin 2 2q) = 1 - . 0 = 1
4 4
356. (2) Given : x2 + y2 + 10 = 2 2 x + 4 2 y 3
\ £ A £1
Þ x 2 - 2 2 x + y 2 - 4 2 y + 10 = 0 4
362. (1) Given Ð AOC By property of circles, we have
Þ x 2 - 2·x· 2 + ( 2) 2 + y 2 - 2·y·2 2 + (2 2) 2 = 0 Ð AOC = 2Ð ADC Þ Ð ADC = 110° ¸ 2 = 55º
Þ ( x - 2)2 + ( y - 2 2) 2 = 0 Þ ÐADC = 55° (cyclic quadrilateral)
\ Ð B + Ð D =180°
Þ x - 2 = 0 and y - 2 2 = 0 Þ Ð B =125°
363. (1) The internal bisector of ÐX meets YZ at P
Þ x = 2 and y = 2 2 \ x + y = 3 2
357. (4) Let actual printed C.P. = ` 125 XY YP
=
Price paid by the business man = 80% of 125 XZ PZ
Profit for him at the time of buying = 20% of 125 = ` 25 Add 1 on both the sides
S.P. of the businessman = 120% of ` 125 XY YP XY + XZ YP + PZ YZ
Profit for him at the time of selling = 20% of 125 = ` 25 Þ +1 = +1 Þ = =
Total profit = ` 50 XZ PZ XZ PZ PZ
364. (3) Let AB and CD be two poles such that AB = 6m,
50 CD = 11m Given AC = 12 m, To find: BD.
Profit % = ´ 100 = 40%
125 ABEC is a rectangle, CE = 6m, ED = 5m and BE = 12m
358. (4) Let ‘abc’ be a 3 digit number.
\ abc = 100 a + 10 b + c
when (a + b + c) is subtracted from (100a + 10b + c), then
the result = (100 a + 10b + c) – (a + b + c) = 99a + 9b
= 9(11a + b) which is divisible by ‘9’.
\ The result is divisible by ‘9’.
Solved Questions 177
B-

Since, D BED is right angle triangle, 371. (4) Given sin A, cos A and tan A are in G.P.
\ BD2 = BE2 + ED2 sin A
Þ BD2 = 122 + 52 Þ BD = 13 m Þ cos2 A = sin A× tan A Þ cos2A = sin A.
cos A
4 4 Þ cos3 A = sin2 A Þ cot3 A = cosecA
æ ö æ ö
365. (4) Consider ç 3 6 a9 ÷ ç 6 3 a 9 ÷ Squaring on both sides
è ø è ø cot6 A = cosec2 A Þ cot6 A =1+ cot2A Þ cot6 A– cot2
4 4 4 4 A =1
é 91ù é 91ù
= éê 3 9 6 ùú éê6 9 3 ùú = ê 6.3 ú ê 3. 6 ú 372. (2) Area of each square is 6.25 cm 2
ë a û ë a û ëa û ëa û Let ‘x’ be the side of each square
= a2. a2 = a4
366. (1) Let ABC be a right angled triangle. 2 625
\ x =
AB 1 100 2
Given = 5
BC 2 5x
Þ x= 2
Let AB = x, BC = 2x 2
Since, DABC rigth angle x Length of the chess board
\ AC = 5x and BD ^ AC. h = 2 × border round wide + 8 × each square length
To find :- CD : AD 5
Let CD = a, AD = b and BD = h = 2 × 2 + 8 × cm = 4 + 20 cm = 24 cm
x 2
a + b = 5x ... (1) 1
1–
D BDC is right angle, h2 = 4x2 – a2 n
D BDA is right angle, h2 = x2 – b2 373. (3) We know, 2 2 2...n times = 2 2
\ 4x2 – a2 = x2 – b2 \ Given expression can be written as
Þ 3x2 = (a – b) (a + b) Þ 3x2 = (a – b) 5x æ 1– 1 ö
çè 5÷
3x log 2 2 ø = log (231/ 32 )
Þ (a – b) = 21/ 4 21/ 4
5
æ 31 ö
x a 4 4x 1 ç ÷ 31 31
= 1 log 2 2è ø = 4 ´ =
from (1) & (2), b = ,a = Þ = 32
5 5 b 1 32 8
367. (1) Given f (x) = 3x – 1, g (x) = x + 6 4
Let y = f (x) 374. (3) Given system of equations is
3x + ay = 21
y +1 2x + 2y = 12
Þ y = 3x – 1 Þ y + 1 = 3x Þ x =
3 21 a
–1 y + 1 Now, Dx = = 42 - 12a
Now, f ( y ) = x = 12 2
3 But Dx = – 6 (given)
–1 x + 1 48
Þ f ( x) = and g ( x) = x + 6 \ 42 – 12a = –6 Þ =aÞa=4
3 12
Consider, (gof –1) (2009) = g {f –1(2009)} 375. (4) Let speed of bus = x km/hr
æ 2009 + 1ö æ 2010 ö Let time taken by train = t hr
= g çè ÷ = g çè ÷ According to question,
3 ø 3 ø
Distance travelled by bus = Speed of bus × time taken by
2010 2028 bus
= +6 = = 676 = 26
3 3 900 = x (t + 5) ...(1)
368. (4) Area of sector BAC = 9p Distance travelled by train = Speed of train × time taken by
Area of sector DAC = 9p train
Required area = sum of area of two sectors – area of square 900 = (x + 15)t ...(2)
= (9p + 9p ) – 36 = 18p – 36 From eqs. (1) and (2)
x (t + 5) = (x + 15) t
369. (3) x t + 5x = xt + 15t Þ x = 3t
The first place filled with 3 - 1 way Putting this in eq. (1)
Second place - 5 ways 900 = 3t (t + 5) Þ 3t2 + 15t – 900 = 0 Þ t2 + 5t – 300 = 0
Third place - 4 ways (t + 20) (t – 15) = 0, t = – 20, t = 15.
Fourth place - 3 ways ‘t’ cannot be negative
Total numbers = 1×5× 4×3 = 60 \ t = 15
370. (2) Let x = 25650 900
By taking log on both the saides 900 = (x) (15 + 5) Þ x = = 45 km/hr..
log x = 50 log10 256 20
Þ log x = 50 log10 28 = 400log102 = 400×(0.3010) 376. (4) Let f be the frequency of the modal class.
log x =120.4 Let frequency of premodal class
Number of digits in 25650 is 120 + 1=121. = frequency of postmodal class = x
EBD_7332
178
B- NTSE Stage 1 Question Bank

Given, modal class = 40 – 60 \ l = lower limit = 40 1


h = width = 60 – 40 = 20 ar (DPQC) = ar (DBAC)
4
f -x
Now, mode = 40 + ´ 20 1
2f -x-x Þ x= (10 + x + y ) Þ 3x = 10 + y ...(1)
4
f -x Also, ar (DBPC) = ar (DBAP) (Q BP is median)
= 40 + ´ 20 = 40 + 10 = 50
2 f - 2x Þ7 +x= 3+ yÞ4 =y–x ...(2)
377. (1) Since P(A) + P(B) = 1 From equation (1) and (2),
Therefore A Ç B = 0. Thus, event A and B are mutually 3x = 10 + (4 + x)
exclusive, exhaustive and complementary events. Þ 3x = 14 + x Þ x = 7 and y = 11
378. (2) Statement given in option (2) is false. Thus, x + y = 7 + 11 = 18
“Mean cannot be determined from the graph” is the correct 384. (4) Let AC be the tangent to circle of centre O.
statement. OA ^ AC A C
379. (2) Let sin2 q + cosec2 q = 6
Þ (sin q + cosec q)2 – 2 sin q cosec q = 6 (Q Radius always ^ to tangent)
Þ (sin q + cosec q)2 = 6 + 2 sin q cosec q \ ÐOAC = 90° and DOAC is isosceles
triangle O
Þ (sin q + cosec q)2 = 6 + 2 × 1 = 8
\ ÐO = ÐC = x
Þ (sin q + cosec q) = 8 = 2 2 Now, by angle sum property of D. We have
1 90° + x + x = 180° Þ x = 45°
380. (1) Area of DAOB = ´ 10 ´ h1 Hence, ÐOCA = 45°
2
1 6 3 385. (4) 6 cm

h = 3 cm
3 = ´ 10 ´ h1 Þ h1 = =
2 10 5
3 cm
1
Area of DCOD = ´ 20 ´ h2 r
2
1 12 6 Circumference of the cylinder = length of the rectangular
12 = ´ 20 ´ h2 Þ h2 = = paper
2 10 5
Now, height of trapezium ABCD = h1 + h2 3
Þ 2pr = 6 Þ r =
3 6 9 p
= + = 386. (1) DABC ~ D ADE ~ DAFG
5 5 5 \ h1 : h2 : h3 = r1 : r2 : r3
1
\ Area of trapezium = ´ [ (h1 + h2 ) ´ (10 + 20) ]
h1 : h2 : h3 = 1 : 2 : 3
2 \ r 1 : r2 : r 3 = 1 : 2 : 3
1 9 54 Required ratio
= ´ ´ 30 = = 27 sq. cm. A = [(1)2 ×1] : [(2)2 × 2 – (1)2 × 1] : [(3)2 × 3 – (2)2 × 2]
2 5 2 = 1 : 7 : 19
381. (2) Let ABC be a right angled triangle. A
Since, cot B = cot 90° = 0
Then, h1
AB BC
cot A · cot B · cot C = ´0´ =0 B r1 C h2
BC AB
382. (2) Required area B C h3
= Area of 2 small semi-circles D r
2
E
+ Area of big semi circle
2 2
æ rö æ rö
pç ÷ pç ÷ 2
F r G
è 2ø è 2ø pr 2 æ rö pr 2 3

Þ + + Þ pç ÷ +
2 2 2 è 2ø 2
éa bù
pr 2 pr 2 3 pr 2 387. (2) Let X = ê
Þ + = sq.cm. ëc d úû
4 2 4 éa cù
383. (4) Since, in DABC P is the mid-point of AC and Q is the Then X¢ = ê
mid-point of BP then ëb d úû
A b - cù é 0 b - cù
é 0
\ X – X¢ = ê =
ëc - b 0 úû êë - (b - c) 0 úû
é 0 c - bù
R P = -ê = – (X - X¢)¢
Q ëb - c 0 úû
Hence (X – X¢) is skew symmetric matrix.

B C
Solved Questions B-179

SECTION 5. HISTORY 10. (2) Gandhi Ji, the father of the nation, said the statement,
If I am to die by the bullet of a mad man, I must do so
History smiling on 28 January 1948.He said so due to the disturbance
created by a section of people in his secular prayer meetings
1. (4) The period that started in 1789 and ended in November a couple of times.
1799 with the formation of the French Consulate is referred 11. (1) In 1439, the first printing press was made by Johannes
to as the French Revolution. During this period, Louis XVI, Gutenberg and his associates. The Bible was the first book
who belonged to the ‘Bourbon’ dynasty, was the king of that was printed in the press in Europe. During the Eastern
France.He the last ruler of France before the monarchy came Han period (25–220 CE), China developed paper and
to an end during the French Revolution. printing with blocks. Paper and ink were two earlier Chinese
2. (2) Hitler was born in Austria. He was born on 20 April inventions thatledto blocking printing.
1889 in Braunau am Inn. He was an Austrian-born German 12. (1) Oceans cover 71 per cent of the earth’s surface. There
politician who dictated Germany from 1933 to 1945. He are five oceans in the world. These five oceans are in
became the leader of the Nazi Party. During his tenure, he descending order by area, the Pacific, Atlantic, Indian,
invaded Poland on September 1, 1939 ad, therefore, initiated Southern (Antarctic), and Arctic Oceans. Therefore, the
World War II in Europe. Pacific Ocean is the largest ocean and the smallest ocean is
3. (2) The book, The Folklore of Southern India was the Arctic Ocean in the world.
published by Natesa Sastri. The book is a collection of 13. (2) Government (1690) by Locke and The Social Contract
Tamil folktales and was published in four volumes. He (176 (2) by Jean-Jacques Rousseau (1712–78) proposed
considered folklore as national literature. According to him, justifications of political association grounded in the newer
real thought and characteristics could be shown and political requirements of the age.
projected by folktales only. Natesa was a noted supporter 14. (1) The symbols of New Britain – the British flag (Union
of Harikatha. Jack), the national anthem (God save our Noble King) and
4. (3) The secret society or a political society ‘young Italy’ the English language were actively promoted and the older
was formed by Giuseppe Mazzini in the year 1831. The nations survived only as subordinate partners in this union.
movement aimed to unite people as a republican Italian 15. (1) Andhra Pradesh emerged as the first linguistic state on
nation. It was the movement that was founded for the Nov. 1, 1956. In Oct. 1953, the government of India was forced
independence of Italy. He had a belief this movement could to create the first linguist state known as Andhra Pradesh by
lead to the unification of people and would contribute to a separating the Telugu speaking areas from Madras state.
European-wide revolutionary movement. 16. (2) He was one of the leaders of the Young Bengal group in
5. (4) Gandhi Irwin Pact was an agreement that was signed 19th-century India. As an orator, editor of several periodicals,
by Mahatma Gandhi and Lord Irwin in 1931. According to and a social reformer, he donated land for the Bethune School
the pact, Gandhi agreed to participate in the Second Round 17. (4) Alexander II became known as Tsar the Liberator able
Table Conference. He also promised to withdraw the civil to implement the most challenging reforms undertaken in
disobedience movement. The agreement also included the Russia since the reign of Peter the Great.
withdrawal of all prisoners with the expectation of who 18. (3) Black Thursday is the name given to Thursday, October
were involved in severe violent crimes. 24, 1929, when panicked investors sent the Dow Jones
6. (4) Gandhiji had designed the Swaraj flag by the year of Industrial Average plunging 11% at the open in very heavy
1921. The flag was a tricolour,i.e., red, green and white and volume. Black Thursday began the Wall Street crash of 1929,
had a spinning wheel in the centre. The wheel represented which lasted until October 29, 1929.
the idea of Gandhian of self-help. At the time of struggle, 19. (2) France was named “a museum of economic errors” by
freedom fighters used to hold the flag, representing a Adam Smith. He is considered as the “Father of Economics”
symbol of opposition. and known for his magnum opus, The Wealth of Nations.
7. (3) Michelangelo made the statue of David. It was a 20. (3) On 4th February 1922, a group of protestors
masterpiece made in the period between 1501 and 1504. An participating in the Non-Cooperation Movement clashed
Italian artist, Leonardo da Vinci made the best known, the with the police who opened fire. The angry protestors in
most visited and the most painting, and the most parodied turn set the police station on fire, killing all its occupants.
work of art in the world the Mona Lisa. It is a half-length This took place in Chauri Chaura, in Gorakhpur district of
portrait of the Italian Renaissance. Leonardo da Vinci made erstwhile United Provinces. On seeing the movement turn
the famous painting, the last supper. It depicts the last meal violent, Mahatma Gandhi put a halt to the Non-Cooperation
Jesus having with his apostles. Albrecht Durer made the Movement in the subcontinent.
painting, Praying Hands. It is a famous ink and pencil sketch 21. (1) It was started on 17 December 2010. It was a series of
drawing that was created in the early 16th century. anti-government protests, uprisings, and armed rebellions
8. (3) Machiavelli wrote the book, The Prince in 1513. that spread across much of the Arab world in the early
Montesquieu wrote the book, The spirit of Laws in 1748. 2010s. It began in response to oppressive regimes and a
Thomas More wrote the book, Utopia in 1516. Erasmus low standard of living, starting with protests in Tunisia.
wrote the book, Praise of Folly in 1509 but was printed in 22. (2) The Korean War began on June 25, 1950. Suez Crisis,
1511. This book was a satirical attack on several traditions (1956), an international crisis in the Middle East, precipitated
and superstitions. on July 26, 1956, when the Egyptian president, Gamal Abdel
9. (3) There was an attack on the Bastille prison on July 14, Nasser, nationalized the Suez Canal. The Congo Crisis
1789, in Paris. On the day, the violent attack took place on (French: Crise congolaise) was a period of political upheaval
the government by French residents as the onset of the and conflict in the Republic of the Congo (today the
French Revolution. People attacked the prison in search of Democratic Republic of the Congo) between 1960 and 1965.
arms and ammunitions as they had a belief that they had From 1988 to 1990, the Somali Armed Forces began engaging
stored them at the fort. in combat against various armed rebel groups.
EBD_7332
180
B- NTSE Stage 1 Question Bank

23. (1) Although nominally allied with the German Empire and 35. (4) Sanchi is a Buddhist complex, famous for its Great
the Empire of Austria-Hungary in the Triple Alliance, the Stupa, on a hilltop at Sanchi Town in Raisen District of the
Kingdom of Italy did not join the Central Powers, Germany State of Madhya Pradesh, India. It is located 46 kilometres
and Austria–Hungary had taken the offensive while the northeast of Bhopal, the capital of Madhya Pradesh.
Triple Alliance was supposed to be a defensive alliance. 36. (2) After Ashoka’s successful but devastating conquest
24. (2) Anandamath is a Bengali novel, written by Bankim of Kalinga early in his rule, he converted to Buddhism and
Chandra Chatterjee and published in 1882. Set in the was inspired by its doctrine of dharma. Thereafter, he ruled
background of the Sannyasi Rebellion in the late 18th his empire through peace and tolerance and focused on
century, it is considered one of the most important novels public works and building up the empire rather than
in the history of Bengali and Indian literature. expanding it.
25. (1) Dhapi Dadi from village Kudan (Sikar, Rajasthan) was 37. (3) Tamil was the oldest language of South India. Of the
a brave widow, a leading Freedom Fighter who took part in four literary languages in the Dravidian family, Tamil is the
the Shekhawati farmers movement in Rajasthan. She was oldest, with examples dating to the early Common Era. In
of Fandan Gotra from village Raseedpura. the early 21st century, Tamil was spoken by more than 66
26. (2) Riyasat was the name of the newspaper. The main million people, mostly residing in India, northern Sri Lanka,
objective of General Dyer behind the Jallianwala Bagh Malaysia, Singapore, Mauritius, Fiji, and Myanmar (Burma).
massacre was to ‘produce a moral effect’, to create in the 38. (2) In the early 14th century, the Delhi Sultanate
minds of satyagrahis a feeling of terror and awe. ruler Alauddin Khalji (r. 1296-1316) instituted price controls
27. (1) Flying shuttle, Machine that represented an important and related reforms in his empire.
step toward automatic weaving. It was invented by John 39. (2) It was established in 1336 by the brothers Harihara I
Kay in 1733. In previous looms, the shuttle was thrown, or and Bukka Raya I of the Sangama dynasty, members of a
passed, through the threads by hand, and wide fabrics pastoralist cowherd community that claimed Yadava
required two weavers seated side by side passing the lineage. The empire rose to prominence as a culmination of
shuttle between them. attempts by the southern powers to ward off Islamic
28. (2) In 1589, the French throne was taken by the Bourbon invasions by the end of the 13th century.
dynasty. It took place after the killing of the last Valois 40. (2) The royal city at Fatehpur Sikri, situated 26 miles west
king, the childless Henry III. After the fall of Napoleon, the of Agra, Uttar Pradesh, was built under the orders of the
dynasty returned to the throne In 1814. However, the last great Mughal Emperor Akbar. In honour of Saint Shaikh
Bourbon king was absconded after the Revolution of 1830 Salim Chisti, Akbar founded a magnificent city on Sikri
and was replaced by a cousin from the Orleans dynasty. ridge.
29. (4) The King of Sardinia named Charles Albert was on the 41. (3) Vasco da Gama was best known for being the first to
throne for the period from April 27 1831 to March 23, 1849. sail from Europe to India by rounding Africa’s Cape of Good
The first Italian constitution, the Albertine Statute, and the Hope. Throughout two voyages, beginning in 1497 and
First Italian War of Independence are associated with him. 1502, da Gama landed and traded in locales along the coast
He stayed in France during the Napoleonic period. There of southern Africa before reaching India on May 20, 1498.
he received a liberal education. 42. (4) Retired British Indian Civil Service (ICS) officer Allan
30. (1) A Russian author, Leo Tolstoy wrote a book, War and Octavian Hume founded the Indian National Congress (A
Peace in 1869.This book focuses on the invasion of Russia political party of India (British India to Free India)) to form
in 1812. It projects the Russian society against the invasion. a platform for civil and political dialogue among educated
It has spirituality and happiness as the ultimate reward for Indians.
spiritual sufferings. 43. (4) The Story of My Experiments with Truth is
31. (2) King Suryavarman II was the successor of Cholas. the autobiography of Mohandas K. Gandhi, covering
Hebuilt the Angkorwat temple in the 12th century. The his life from early childhood through to 1921.
temple has a huge complex. The complex consists of a 44. (2) The Aravalli Range, an eroded stub of ancient mountains,
thousand buildings. At the onset of the 12th century, it is the oldest range of fold mountains in India. The natural
was converted into a Buddhist temple. history of the Aravalli Range dates back to times when the
32. (2) The “Vernacular Press Act was passed by Lord Lytton Indian Plate was separated from the Eurasian Plate by an ocean.
in 1878. He was the then viceroy of India (governed 1876– 45. (1) The first Peshwa of Shahu was Balaji Vishwanath. He
80). In British India, the act aimed to restrict the freedom of was appointed as Peshwa by Sahu on November 16, 1713.
the Indian press and prevent the expression of criticism He helped a young Maratha emperorto get his kingdom
toward British policies. This criticism was the outcome of back in a better state as it was damaged due to regular
the beginning of the Second Anglo-Afghan War (1878–80). attacks by the Mughals under the rule of Aurangzeb.
33. (3) The Jallianwala Bagh Massacre took place on April 46. (4) Before French Revolution, the political body of France
13, 1919. The moment Rabindranath Tagore got to know was called Estates General. In this period, France was
about the Jallianwala Bagh Massacre. He returned the title, divided into three parts or states, namely First Estate that
‘the sir’ (Knight hood) to the British government as a sign was related to clergy, the Second Estate was related to
of protest. In this event, there were 379 people died at the nobility and the Third Estate was related to commoners.
hands of the British army. The king was above or beyond all these three estates.
34. (1) Dwarakanath Vidyabhusan started the newspaper, Som 47. (2) From 1756 to 1763, the Seven Years War was a global
Prakash. It was a weekly newspaper which was started on conflict that took place. It pitted a coalition of Great Britain
15 November 1858. However, the original idea or plan for and its allies against a coalition of France and its allies. The
the newspaper was given by Ishwar Chandra Vidyasagar. war escalated from a regional conflict between Great Britain
The aim was to provide the opportunity of employment for and France in North America, known today as the French
a deaf scholar. and Indian War.
Solved Questions B-181
48. (3) Vietnam National Party was formed in the year of 1939. 61. (4) Genghis Khan was one of the most feared and admired
The Fascist Party was formed in 1921 by Benito Mussolini. leaders of his time. He created a Mongol military might that
The union of South Africa came into existence in 1910. It was superior to any other he faced. His campaigns were
included the territories that were originally a part of the brilliant, and showcased his military genius and established
South African Republic and the Orange Free State. French standards for manoeuvre warfare, brutality, and human
communist party formed in the year 1920. endurance never before seen.
49. (3) Hundred Days of Reform took place in 1898. It was an 62. (3)
attempt to renovate the social and other conditions of China. 63. (2) The Ryotwari System was first implemented in the
Russian Revolution began in 1917 and ended in 1923. In Baramahal region by Alexander Reed in 1792. This system
this revolt, peasants And the workingclass started the was later implemented in Madras and Mysore region by
protest against the government of Tsar Nicholas II. First Thomas Monroe in 1801. In the Ryotwari system, there were
Opium War was started in 1839. It was fought between no middlemen to collect the revenue. The East India
China and Britain. The Chinese Revolution took place in Company and the farmers were directly linked.
1911. It leads to the end of the Chinese dynasty. 64. (2) Mahajanapadas were sixteen kingdoms that existed in
50. (2) The meaning of this phrase is that if one does not northern ancient India from the sixth to fourth centuries
grant liberty, equality, or fraternity to others—one does not BCE. The capital of Anga was located on the bank of this
treat others like they would treat their brother—one will river and was also named Champa. Varanasi was the capital
meet death. The three ideals, namely freedom, equality and of Kashi. Saketha was the capital of Kosala. Kushinagara
brotherhood were the foundation of the ‘new France’ that was the capital of Malla.
the revolutionaries sought. 65. (4) The third Carnatic war took place in 1756. The second
51. (4) Kachch is not recognised by UNESCO. There are 11 Anglo-Mysore war took place in 1780. The second Anglo-
biosphere reserves of India that are now considered as Marata war took place in 1803. The first Anglos Sikh war
UNESCO Protected Biosphere Reserve. took place in 1845.
52. (1) Michel Foucault was the French philosopher who 66. (2) Allauddin Hasan Gangu was the founder of the Bahamani
wrote the book Archaeology of Knowledge in which he Kingdom. The Iranian painters were brought by Humayun were
argued that the prevailing practice of arranging historical Mir Sayyid Ali and Abdus Samad. Both the painters were the
events in chronological order is not right. founders of an independent branch of Persian art popularly
53. (2) Ishwardas Nagar, Bhimsen Saxena, Khafi Khan and known as the Mughal school of miniature painting.
Niccolo Manucci were among the historians of Aurangzeb’s 67. (3) An Indian social reformer, Shishunal Shariffa believed
times whose accounts are important sources of Mughal in the philosophies given in different religions. He took
history. Many British officials in India wrote about Indian Deeksha from Govindabhatta. He wrote several Tatvapadas
history. Their writings display the strong influence of the which means moral poems. The objective of the poems was
colonial policies of the British. to bring social transformation.
54. (3) This style consists of coloured paintings. - It occurs 68. (4) Sripurusha was the king of the Western Ganga dynasty.
as murals and also miniatures used in manuscripts. - The He is the author of the book “Gajasastra” which was written
murals are seen in old wadas, temples, ceilings etc. - Maratha in Sanskrit.
style was influenced by the Rajput and European painting. This book is considered western Ganga literature and also
55. (2) Kesari began to publish articles about the nationwide a very sacred text.
situations, books in the native languages and the politics Gadya Chintamani was written byVadiba Simha.. Shabdavathara
in England was written by Durvinita.Raghava Pandaviya was written by
56. (3) Shejwalkar primarily wrote in the Marathi language Hemasena.This book tells the stories of Pandavas and Rama.
and was the founder-editor of the now-defunct Marathi 69. (4) Akshay Ramanlal Desai was a renowned Indian
periodical Pragati (1929–1932). sociologist. He was influenced by Leon Trotsky and therefore
57. (4) Baji Prabhu Deshpande (c. 1615-1660) was a was a Marxist and a social activist. He worked as the UGC
commander of Maratha king Chatrapati Shivaji Maharaj, national fellow and as the Head of the Department of Sociology
the founder of the Maratha empire. The legend of Baji at Bombay University. One of his best works was “The
Prabhu is linked with an important rear guard battle enabling Sociological Background of Indian Nationalism’’. His other major
Chatrapati Shivaji Maharaj’s escape from Panhala fort; he works include Indian Rural Sociology, Slums and Urbanisation.
was the hero who sacrificed his life for his king. 70. (1) Rigveda: All written records in the form of texts,
58. (3) Games such as Chess, Snakes and Ladders, Playing essays or descriptions are taken under literary sources.
Cards, Polo, the martial arts of Judo and Karate had originated This includes religious texts of Hindus, Buddhists and
in India and it was from here that these games were transmitted Jains along with some secular sources. The first literary
to foreign countries, where they were further developed. sources of Hindus are ‘Samhitas’ which includes four Vedas.
59. (2) King Lear is a tragedy written by William Shakespeare. The Rig-Veda which is seem to be the first literary sources
There are two versions, but modern editors usually conflate provides us information about the civilization of the early
these to produce a single play. Both versions are based on Vedic Age while the rest of the three Vedas are useful to
the mythological Leir of Britain. King Lear relinquishes his know about the civilization of the later Vedic age.
power and land to two of his daughters. 71. (3) Chandragupta Maurya: Megasthenes was a Greek
60. (1) Menelik II was crowned King of Kings and Emperor historian, diplomat and author who visited India in the
of Ethiopia on November 3, 1889, with the additional royal reign of Chandragupta Maurya. In the 4th century B.C. he
sobriquet of “the Conquering Lion of the Tribe of Judah.” was sent by the Hellenistic king, Seleucus I on embassies
The coronation, which took place in the great Entotto to the Mauryan emperor Chandragupta. Megasthenes
Mariam Church in Addis Ababa, was captured for posterity remained in Pataliputra, the capital of the Mauryan empire
by the Italian artist Pio Joris (1843-1921). and wrote ‘Indica’ during his stay. In the book, he records
his impressions of India of the Mauryan period.
EBD_7332
182
B- NTSE Stage 1 Question Bank

72. (1) At the time when Alexander came to invade India in State for India, Sir Stafford Cripps, President of the Board
326 B.C., the most powerful kingdom was Magadha, under of Trade, and A. V. Alexander, the First Lord of the Admiralty
the rule of Nanda dynasty. The then Nanda king was reached Delhi on 24th March 1946. On the 16th of August,
Dhananana. He maintained a huge force of 200,000 infantry, 1946, the Muslim League called for a “Direct Action Day”.
80,000 cavalry, 8000 chariots and 6000 war elephants. This 80. (2) Zollverein was a German customs union established
was the largest and the most powerful standing army in in 1834 under Prussian leadership. It created a free-trade
the world. After hearing the greatness of Magadha’s forces, area throughout much of Germany and is often seen as an
the forces of Alexander lost all courage and mutinied and important step in German reunification.
refused to attack over Magadh. 81. (3) Warsaw, is the capital city of Poland located in the
73. (2) Farid Sher Shah Suri was born in 1486. His original east-central part of the country, Warsaw is also the capital
name was Farid. He left his home at the age of 15 and of Mazowieckie województwo (province). Poland is a
went to Jaunpur. He had very good administrative skills. country located in Central Europe.
In 1522, he joined the service of Bahar Khan, governor of 82. (1) A - Ahmedabad, B - Bharuch, C - Machilipatnam,
Bihar at that time. He was given the title of Sher Khan by D - Chandranagar;
Bahar Khan, for the courage and gallantry shown by him The European settlements were in coastal zones for trading
in killing a tiger single-handedly. Later Bahar Khan and shipment. These were in Gujrat, Andhra Pradesh and
appointed him as a deputy governor and tutor of his son Bengal.
Jalal Khan. Later on Sher Khan became the virtual ruler of 83. (2) A is wrong, only B, C, D are correct.
Bihar as Jalal Khan was a minor. In 1531, he asserted his Solanki Dynasty was established by Mulraj that ruled the
freedom from the Mughal ruler Humayun. western and central India during the 10th century to the
74. (4) Giuseppe Mazzini (1805–1872) was an Italian 13th century AD.
politician, journalist, activist for the unification of Italy, Acharya Hemachandra (1089 – 1172) was the great Jain
and spearhead of the Italian revolutionary movement. His scholar, poet, and polymath, trained in religious discourse,
efforts helped bring about the independent and unified philosophy, logic and grammar. He was ordained as the
Italy in place of the several separate states. He also helped acharya of the Svetambara sect of Jainism.
define the modern European movement for popular In 1297, Ala-ud-Din conquered Gujarat under the command
democracy in a republican state. He conducted a slogan of Ulugh Khan and Nusrat Khan. Karan Dev, the ruler of
for the country including revolutionaries, people and Italy. Gujarat, was defeated by them.
75. (4) Lenin (1870–1924) was a Russian revolutionary, 84. (4) It is related to Social reformation chapter, especially
politician, and political theorist. He served as a head of achievement with women welfare, option 4 is correct response.
government of Soviet Russia from 1917 to 1924 and of the Pandita Rama Bai dedicated her life for the betterment
Soviet Union from 1922 to 1924. He stressed upon to collect of women of India and established Mukti Mission.
the farms to reconstruct agriculture on a socialist basis Tara Bai Shinde wrote a book – ‘Stri Purusha Tulana’.
applying all the achievements of science and technology Savitribai Phule (1831–1897) was an Indian social
to educate the working masses in the spirit of socialism reformer, educationalist, and poet from Maharashtra.
with eradication of capitalism. She is regarded as the first female teacher of India.
76. (2) Hind Swaraj (Indian Home Rule) is a book written by She died while treating a plague patient.
Mahatma Gandhi in 1909. In it he expresses his views on Annie Besant (1847–1933) was a British socialist,
Swaraj. The book was banned in 1910 by the British theosophist, women’s rights activist, writer, orator,
government in India as a seditious text. In the book Gandhi educationist, and philanthropist. She propagated
gives a diagnosis for the problems of humanity in modern universal brotherhood regardless race, creed, gender,
times, the causes and his remedy. caste and colour.
77. (3) The Chauri Chaura incident took place in Gorakhpur 85. (4) In 1757, Plassey war option b is wrong remaining
district of the United Province, (Uttar Pradesh) on 4 options are correct.
February 1922. In this incident, a large group of protesters, The Battle of Plassey was fought on 23 June, 1757 between
taking part in the Non-cooperation Movement, clashed Robert Clive and Siraj-ud-Daulah. The battle helped the
with police resulting in fire. In retaliation the demonstrators Company seize control of Bengal.The battle took place at
attacked and set fire to a police station, killing all of its Plassey on the bank of the Hooghly River. Mir Jafar was
occupants. The incident led to the deaths of three civilians crowned as the new Nawab of Bengal.
and 22 policemen. 86. (2) Map related question on International Airports in
78. (2) Tebhaga Movement was the sharecroppers’ movement India, option 2 is the correct answer.
demanding two thirds of the produce from land for Veer Sawarkar International Airport— Port Blair
themselves and one third for the landlords. Tebhaga Lal Bahadur Shashtri International Airport—Varanasi
movement was organised mainly by the communist cadres Babasahib Ambedkar International Airport—Nagpur
of the Bengal Provincial Kisaan Sabha. Under their Sardar Ballabhbhaipatel International Airport—Ahmedabad
leadership the barga (sharecropping) peasants were 87. (3) Related to characteristics of culture option 3 is incorrect.
mobilised against the landlord class. Tebhaga movement Culture is used in a special sense in anthropology and
spread out to nineteen districts of Bengal. sociology. It refers to the sum of human beings’ life ways,
79. (4) The Cripps Mission was sent by the British government their behaviour, beliefs, feelings, thoughts. It indicates
to India in March 1942 to obtain Indian cooperation for the everything that is acquired by them as social beings.
British war efforts in the 2nd World War. On 8 August 1942 Culture is not a static phenomenon, it is learnt,
at the All-India Congress Committee session in Bombay, transmissive and a continuous process.
Mohandas Karamchand Gandhi launched the ‘Quit 88. (2) Murals of Maratha style is an example of art style. The
India’ movement. A high-powered mission of three British style known as Maratha paintings began to develop in the
Cabinet members- Lord Pethick-Lawrence, the Secretary of latter half of the 17th century C.E. This style includes
Solved Questions B-183
coloured paintings and they occur as murals and also representational government and was one of the conveners
miniatures used in manuscripts Murals of Maratha style of the first session of the Indian National Congress, held
can be seen in the old wadas at places like Wai, Menavali at Bombay (Mumbai) in 1885. He served as a general
and Satara in Maharashtra. secretary of the Congress for its first 22 years.
89. (3) James Augustus Hicky was an Irishman. He launched 99. (1) The livre was the currency of France. It was a unit of
first printed newspaper of India, in January 1780, with the currency formerly used in France.
name Bengal Gazette. It was known as Hicky’s Bengal 100. (1) The delightful novel called Indulekha is written by
Gazette. He is also known as the ‘father of Indian Press’ O. Chandu Menon. It was published in 1889, and was the
by the British. first modern novel in Malayalam.
90. (4) In the year 1982 , the colour television was introduced 101. (3) The painting Damayanti was made by the great Indian
in India .It was the time for the Asian Games, a colourful painter Raja Ravi Verma.
spectacle. But most of the country was in the danger of 102. (1) In Africa, a fast-spreading disease of cattle plague or
seeing it in shades of black and white. rinderpest had a terrifying impact on people’s livelihood
91. (2) Megasthenes visited India between 302 and 288 BC, and the local economy. It arrived in Africa in 1880s.
during the reign of Chandragupta Maurya. He visited the 103. (3) The Gudem rebellion spread in response to the non-
Maurya capital Pataliputra. He passed through the Punjab cooperation movement in 1921. In the Gudem hills of Andhra
region in north-western India, he provided a detailed Pradesh, the militant guerrilla movement spread under the
account of the rivers in this area. He traveled to Pataliputra leadership of Alluri Sitaram Raju. It was against forest laws.
along the Yamuna and the Ganga rivers. 104. (3) Architect and planner Ebenezer Howard developed the
92. (2) The Mudrarakshasa is a historical play in Sanskrit by principle of the Garden City. The Garden city was a pleasant
Vishakhadatta (4th century CE) narrates the ascent of the space full of plants and trees where people lived and worked.
king Chandragupta Maurya to power in Northern India 105. (4) Introducing five year plan was not related to Stalin.
with the help of Chanakya. 106. (4) In January 1871, Prussian King William I was proclaimed
93. (1) Fourth Buddhist Council was held at Kundalvana German Emperor in a ceremony held at Versailles.
(Kashmir) in 72 AD under the patronage of Kushan king 107. (1) In Britain, the new Tory government constituted a
Kanishka and the president of this council was Vasumitra, statutory commission under Sir John Simon. It was
with Asehvaghosa as his deputy. This council distinctly appointed in 1927. The first round table conference was
divided the Buddhism into two sects- Mahayan and held between November 1930 and January 1931 in London.
Hinayan. Gandhi-Irwin pact was a political agreement signed by
94. (3) The Jantar Mantar is a collection of nineteen Mahatma Gandhi and Lord Irwin, the then Viceroy of India
architectural astronomical instruments built by the Maratha on 5 March, 1931. Re-launch of Civil Disobedience movement
Rajput king Sawai Jai Singh II, the founder of Jaipur in 1932 by Mahatma Gandhi after the negotiations in second
(Rajasthan). The monument was completed in 1734. It round table conference was broke down.
features the world’s largest stone sundial, and is a UNESCO 108. (3) In Awadh, peasants were led by Baba Ramchandra–a
World Heritage site. Sanyasi. The movement here was against talukdars and
95. (2) Sui Munda was the leader of Kol Rebellion. The Kol landlords who demanded from peasants exorbitantly high
rebellion was a revolt of the indigenous Kol people of rents and variety of other cesses.
Chota Nagpur between1829-1839 as a retort to the unfair 109. (1) One of the major impacts of European Colonialism was
treatment brought on by the systems of land tenure and on the practice of shifting cultivation or swidden agriculture.
administration which was initiated by British powers in In Sri Lanka shifting agriculture was known as Chena.
those areas. The British’s oppressive tactics, revenue and 110. (2) The increase in foodgrain production was made
judicial policies, and high revenue rates devastated the possible not by any radical innovations in agricultural
conventional social structure of the Kols. The Kols technology but by bringing new land under cultivation.
gathered under Jhindrai Manki, Joa Bhagat, Bir Budhu 111. (4) The famous Enabling Act was passed in March 1933.
Bhagat, and Sui Munda, who were the leaders of the Kol This Act established dictatorship in Germany. It gave Hitler
rebellion. all powers to sideline Parliament and rule by decree.
96. (3) The Bengal Gazette is the first newspaper published 112. (2) A. O. Hum was the first secretary of the INC which was
from Calcutta. Its founding editor was James Augustus Hicky. established in 1885.
The paper was first published in 1780 with the motto to 113. (1) Surya Sen, an Indian revolutionary who was influential
report objectively and impartially the affairs of the country. in the Indian Independence movement against British rule
97. (1) Kadambini became the women graduate from Bethune in India Chittagong armoury which is also known as
College of Calcutta University in 1882. She became two of Chittagong uprising which started on 18th April 1930.
the first female graduates (another is Chandramukhi) in the 114. (4) Dr. T. B Cunha was the father of Goa’s freedom
entire British Empire when they received their degrees in movement. Cunha was the first goan to be tried by the
1883. Kadambini Ganguly was the very first Indian female Portuguese military Court.
doctor who practiced with a degree in western medicine 115. (1) Due to continuous grazing within a small area inevitably
and she was also the first female speaker at the Indian meant a deterioration of the quality of pastures. The best
National Congress. grazing land was taken over for white settlements and
98. (2) Allan Octavian Hume, was a member of the Imperial Massai were pushed into a small area.
Civil Service (later the Indian Civil Service). He entered 116. (2) Dhangars, an important pastoral community is in
the Indian civil service in Bengal in 1849. In 1870–79 he Maharashtra.
worked for the central government of India as secretary in 117. (2) Bhaskaracharya wrote Bij Ganit.
the revenue and agriculture department. On his retirement 118. (3) Nationalist Vietnames fought against Japan and France.
from the civil service in 1882, he involved himself in political 119. (4) CR Das and Motilal Nehru formed the Swaraj party
activities aimed at giving Indians a more democratic, within the congress to argue for a return to council politics.
EBD_7332
184
B- NTSE Stage 1 Question Bank

120. (3) Belgium and Germany became new colonial powers who 131. (3) In December, 1929 under the presidency of Jawaharlal
joined European powers in the process of carving Africa. Nehru, the Lahore congress formalised the demand for Purna
US also became colonial power in the late 1890s. Swaraj. Dr. B. R. Ambedkar organised the Dalits into the
121. (1) In England and Europe, there was large scale industrial depressed classed Association in 1930 clashed at Mahatma
production for an international market. This was not based Gandhi at the second Round Table conference. Ambedkar
on factories. Many historians refer to this phase of accepted Gandhiji’s position and the result was Poona Pact,
industrialisation as proto industrialisation. 1932.
122. (3) Mahe was the French colony until 1954 when it was 132. (2) Munshi Prem Chand’s Novel Sevasadan, was novel
integrated into the Union of India. originally written in Urdu. The novel deals with poor
123. (2) The correct pr ovision passed by the British condition of women in society, child marriage and dowry.
Government in India was: Supreme Court of Judicature was The novel was a shift from fantasy to serious reflection of
established at Calcutta. A new institution named Board of life.
controllers consisting of six Commissioners was started. 133. (1) Mahatma Gandhi found in salt law a powerful symbol
The post of Governor General was changed to viceroy. that could unite the nation. It was the most essential items
Representation of regional council was allowed to Indians of food.
through election based on religion. 134. (3) The spread of railways from 1850 created a new demand.
124. (2) The treaty of Salbai was signed on 17th May, 1782 by Railways were essential for colonial trade and movement of
the representatives of Maratha Empire and the British East imperial troops. The forests around the railway tracks fast
India Company. The treaty of Mangalore was signed started disappearing due to reckless cutting of trees by
between Tipu Sultan and the British East India Company on contractors.
11th March, 1784. The treaty of Srirangapatna signed on 18th 135. (3) It was organised to support the peasants of Kheda
March, 1792 ended the Third Anglo Mysore War. The treaty district. Due to crop failure and plague epidemic, the
of Amritsar, 1809 was an agreement between the British East presents could not pay the revenue.
India Company and Ranjit Singh. 136. (2)
125. (1) The declaration of Poorna Swaraj in December, 1929 137. (2) Anguriya Binimoy was the first historical novel written
under the Presidency of Jawaharlal Nehru, the Lahore in 1857.
congress formalised the demand for full independence for 138. (2) On 5th May, 1789, Louis XVI called together an assembly
India. Dandi Satyagrahs was an act of non-violent Civil of Estate General to pass proposal for new taxes. Bastille,
Disobedience in colonial India led by Gandhji to produce was destroyed on 14th July 1789. A group of several hundred
salt From sea water. The Quit–India movement started in people marched towards the eastern part of the city and
August 1942. Mahatma Gandhi decided to initiate a new stormed the fortres prison, the Bastille. On the night of 4
phase of movement against the British in the middle of the August, 1789, the Assembly passed a decree abolishing the
second World War. The moplah uprising against British feudal system. On 20 June 1789, the members of the Third
spread like Wildfire and took the Character of a mass Estate, who began to call themselves, the National Assembly
rebellion. took the Tennis court oath.
126. (4) Some sepoys of the regiment at Meerut refused to do 139. (3) Jallianwalla Bagh massacre also known as Amritsar
the army drill using new catridges. The sepoys of Meerut Massacre took place on 13 April 1919 in the State of Punjab.
rode all night of 10 May to reach Delhi. Soldiers persisted and On that day large crowd gathered in the enclosed ground of
proclaimed Bahadur Shah Zafar as their leader. Thereafter, Jallianwalla Bagh. General Dyer, was responsible for
the queen of Britain passed a declaraction assuring stable Jallianwalla Bagh Massacre in Amritsar.
govt. for Indians. Tantia Tope was killed in 1859. 140. (1) In January 1871, the Prussian king William-I was
127. (2) Sathya Shodak Samaj opened the schools for Shudras proclaimed German Emperor in a ceremony held at Versailles.
and girls. It is a social reform society founded by Jyotiba 141. (4) Spinning Jenny was devised by James Hargreaves in
Phule in Pune on 24th September, 1873. 1764, this machine speeded up the spinning process and
128. (1) Train to Pakistan is a historical novel written by reduced labour demand.
Khushwant Singh which was published in 1956. It recounts 142. (2) The partition of Bengal took place on 16 October 1905,
the partition of India in August 1947. and separated the largely Muslim eastern areas from the
129. (1) Liberal nationalism developed in Europe in 19th C. Liberal largely Hindu Western areas. The partition of Bengal was
nationalism means individual freedom, equality before law, announced by the Viceroy of India Lord Curzon.
government by consent, freedom of markets and abolition 143. (4) Germany Axis Power.
of state imposed restrictions on the movement of goods 144. (2) Bengal Gazette was an English newspaper published
and services. from Calcutta (Kolkata). It was the first major newspaper in
130. (2) French Revolution in 1789 was a period of far reaching India, started in 1780.
social and political upheaval in France and its colonies 145. (1) On 6 April he reached Dandi and Ceremonially Violated
beginning in 1789. the salt law manufactured salt by boiling water. Ma-
Treaty of Vienna of 1815 was brought up with the object of hatma Gandhi was accompanied by his 78 trusted volun-
undoing most of the changes that had come about in Europe teers. The march was other 240 miles.
during the Napoleonic wars. 146. (3) The battle of Plassey (1757) laid foundation for British
Unification of Italy: The Italian unification or the Risorgimento rule in India.
was the political and social movement that consolidated 147. (2) The Great Economic Depression began around 1929
different states of Italian peninsula into a single state of the and lasted till the mid 1930s. During this period most parts
kingdom of Italy. It had long history of political fragmentation. of the world experience is great decline in production, em-
Unification of Germany into a politically and administratively ployment, incomes and trade.
integrated nation state occured in January, 1871 in the Hall of 148. (4) Anandmath, a Bengali fiction written by Bankim
mirrors at the palace of Versailles Chandra Chattopadhyay and publised in 1882.
Solved Questions B-185
149. (1) Lenin’s Imperalism the Highest stage of capitalism Razvi organised a private militia the Razakars Raja Harsingh
describes the function of financial capital in generating was the last ruling Maharaja of the princely state of Jammu
profits from imperalists colonialism as the final stage of and Kashmir in India.
capitalist development to ensure profit. 171. (3) French East India Company was commercial enterprise
150. (3) The Wall Street Crash began on October 24th 1929, it founded in 1664. It officially state owned company.
marked the beginning of widespread and long-lasting 172. (2) The above is said by Hitler, Secret Book ed. Telford Taylor.
consequences for the united states. 173. (4)
151. (4) Tsar Nicholas II ruled Russia and its empire. 174. (4) Giuseppe Mazzini, an Italian revolutionary he became
152. (4) John Kay was the inventor of flying shuttle. Flying a member of the secret society of the carbonari. Founded
shuttle was the important development in the industrialisation young Europe in Berne. He also believed that God had
of weaving during early industrial revolution. intended nations to be natural units of mankind.
153. (1) Mein Kempf written by Hitler an autobiographical book. 175. (1) The Book “The History of Loss of Vietnam” was written
It was originally published on 18 July, 1925. by Phan boi chan the most influential book. It was written
154. (1) United states never joined the League of Nations, under the strong influence and advice of Qichao.
despite formulating the concept and signing the covenant. 176. (4) The Elementary Education Act 1870, commonly known
155. (2) Scottish church college is the oldest running christian as Forster’s Education Act set the framework for schooling
liberal arts and science college in India. It was founded in 1830. of all children between the ages of 5 and 12 in England.
156. (1) The first Chancellor of Calcutta was Lord canning. 177. (3) Ebenezer Howard, architect and planner developed the
157. (3) Sardar Vallabhbhai Patel is known as the Iron man of principle of the Garden City, a pleasant space full of plants
India. He was the first Deputy Prime Minister of India. and trees where people would both live and work.
158. (2) The communal Award 1932 was declared by the 178. (1) It was in 1517, the religious reformer Martin Luther
Ramsay MacDonald granting separate electorate in India. wrote Ninety Five Thesis.
159. (3) Pralilata Waddedar, was Bengali revolutionary 179. (1) Kashibaba, a Kanpur Mill Worker in 1938 to show the
nationalist attacked the Pahartati European club which had links between caste and class exploitation wrote the book
a sign board that real ‘Dogs and Indians not allowed’. chhote Aur Bade Ka Sawal.
160. (2) AITUC, was founded in 31st October 1920 in Bombay 180. (2)
by Lala Lajpat Rai, Joseph Baptista, N.M. Joshi, Chaman Lal 181. (2) Franklen Rooserelt announced New Deal Policy. He
and others. was the president of America during Second World War.
161. (2) Hindustan Socialist Republican Association was a 182. (1) The Ryotwan Settlement was introduced in the Madras
revolutionary organisation also known as Hindustan Presidency. It was introduced by Thomas Munno.
Socialist Republican Army was established in 1924 at Feroz 183. (4) On August 8, 1942, the Quit India Resolution was
Shah Kotla, Delhi. passed at the Bombay (Mumbai) Session of All India
162. (1) Congress Committee.
163. (3) The idea of Indian National Army (INA) was conceived 184. (1) Alluri Sitaram Raju led a millitant guerrilla movement in
in Malaya by Mohan Singh, an Indian Officer of the British Guden Hills of Andhra Pradesh in 1920. It was protest against
Indian Army Madras Forest Act. He was inspired by revolutionaries in Bengal.
164. (1) Poona Agreement between Dr. Baba Saheb Ambedkar 185. (4) Ho Chi Minh founded the Communist Party in Vietnam
and Mahatma Gandhi on September 24, 1932 Forward Bloc in 1930. He led the Vietnamese National Movement for three
was formed in 1939 by Subhash Chandra Bose. August offer decades. He fought against Japanese then French
was the proposal made by British Government in 1940, colonization and then US.
thereby promised the expansion of the Executive Council of 186. (2) Muslim League celebrated the Deliverance Day on 22
the Viceroy of India to include more Indians. Direct Action December, 1939 during the Indian Independence Movement.
Day, 16 August 1946 also known as the Great Calcutta Killings It was led by Muhammad Ali Jinnah.
was a day of undespread riot and manslaughter between 187. (3) Gujjar Bakarwals are largely found in Jammu and
Hindus and Muslims in the city of Calcutta. Kashmir. They are a unique ethnic group migrating in
165. (4) 166. (4) Western Himalayas. They rear sheep and goats.
167. (3) The three nation belonged to Triple Alliance during 188. (4) Duma is a Russian Assembly with advisory or
the First World War were Germany, Austria and Italy. It was legislative functions. It was introduced into the Russian
a secret agreement between the three countries. It was formed Empire by Tsar Nicholas II in 1906.
on 20 may 1882. 189. (1) Arthastra is written by Kautilya who was chief advisor
168. (3) The July revolution sparked an uprisong in Brussels of the Indian Emperor Chandra Gupta Maurya (350-275 BC).
which led to Belgium breaking away from the United He was a stateman and philosopher.
Kingdom of the Netherlands. 190. (1) In the battle of Panipat, Ibrahim Lodi was defeated by
169. (2) In the 19th C the Kannada-speaking regions of Belari Babur. It established the root of Mughal empire in India.
came under Madras Province. 191. (3) Kunwar Singh from Jagdishpur (Bihar) led the revolt of 1857.
170. (4) Shaikh Abdullah, known as Sher-e-Kashmir played a In the age of eighty years, he gave good fight to British forces.
central role in the politics of Jammu and Kashmir. He founded 192. (3) Chittaranjan Das was a founder member of the Swaraj
National Conference. Fazal Ali was an Indian judge the Party in Bengal in British India. He was a famous politician
governor of two Indian states. He was the head of the states called as Deshbandhu.
Reorganisation Commission which determined the 193. (3) Canning was the Governor – General of India during
boundaries of several Indian states in the 1950. Sardar the Indian Rebellion of 1857.
Vallabhbhai Patel the first deputy prime minister and also 194. (3) Sarojini Naidu, popularly known as 'Nightingale of India'
India’s first Home minister. He was instremental in uniting was the first Indian woman to become congress President.
the coutry through merger of small princely states. Qasim She presided the Kanpur session in 1925.
EBD_7332
186
B- NTSE Stage 1 Question Bank

195. (3) The first English factory East India Company arrived 235. (2) Metternich said this and he meant that whatever
first at Surat, India in 1608 in the ship Hector commanded by happens in France, affects the rest of Europe. Since the 1st
William Hawkins and within a few years established a French revolution of 1789 and the theories of French
permanent factory there. philosophers sparked off nationalist sentiments throughout
196. (2) The Asiatic Society was founded by Sir William Jones Europe bringing about a greater revolution for liberalism.
in 1784. In 1832 the name was changed to "The Asiatic 236. (2) The Expedition of the Thousand was an event of the
Society of Bengal". Italian unification which took place in 1860. A corps of
197. (3) Indira Gandhi, daughter of Jawaharlal Nehru, was India's volunteers led by Giuseppe Garibaldi landed in Sicily in order
third and first woman Prime Minister of India. to conquer the Kingdom of the Two Sicilies, ruled by the
198. (2) The Ajanta caves were built during the peirod of Gupta Bourbons.
Dynasty. 237. (4) In the nineteenth century, it became important to
199. (1) Hieun–Tsang came to India during the reign of control territory and not only people, Dutch enacted scientific
Harshvardhana. forestry in Java. With this law common people are restricted
200. (4) Red fort was built by Shah Jahan in 1648. to enter the forests.
201. (4) Warren Hastings was the first Governor General of India. 238. (3) The Japanese occupation of Vietnam began in
202. (4) Khan Abdul Ghaffar Khan was called as "Frontier Gandhi". September 1940 and continued for the duration of World
203. (1) 204. (4) 205. (4) 206. (3) War II. The catalyst for this invasion was Japan's ongoing
207. (1) Kheda Movement was a no-tax peasant struggle, war with China, which began in 1937.
launched in 1917 under the leadership of Gandhiji, Sardar 239. (3) "Poorna Swaraj" or The Declaration of Independence
Vallabhbhai Patel, Indulal Yagnik and several others. was officially promulgated on January 26, 1930 during Lahore
208. (4) Bharat Mata is an epic painting by Abanindranath session. The flag of India had been hoisted by Congress
Tagore. President Jawaharlal Nehru on December 31, 1929, on the
209. (4) German inventor Johannes Gutenberg invented Print- banks of the Ravi river in Lahore.
ing press in 1430s. 240. (4) The Great Depression was a period of severe economic
210. (3) Dr. B.R. Ambedkar founded Depr essed class crisis which started during 1930's and remained till the
Association in 1930. beginning of World War II. The Great Depression had
211. (2) Kulaks were the rich peasant. devastating effects in countries rich and poor. Personal
212. (3) Reichstag is a German word which means parliament. income, tax revenue, profits and prices dropped, while
213. (4) Civil Disobedience Movement was one of the most international trade plunged by more than 50%.
important events launched during the reign of Lord Irwin, 241. (3) The first major conflict of the British against an Indian
the then viceroy of India. power was in Bengal. The history of Bengal from 1757 to
214. (1) Suffrage Movement is for the right of women to vote in 1765 is the history of gradual transfer of the power from the
elections. nawabs to the British. During this short period of eight years
215. (3) Jacobin Club was the most powerful political three nawabs, Siraj-ud- Daula, Mir Jafar and Mir Qasim ruled
organisation formed during french revolution in France. over Bengal but they failed to uphold the sovereignty of
216. (4) “Dawes Plan” was introduced by the USA to resolve a the nawab and ultimately the reign of control passed into
serious international crisis due to World War I. Dawes Plan the hands of the British. Finally Battle of Bauxar established
provided short term economic benefits to the German the British Supremacy in India.
economy and softened the burden of war reparations. 242. (1) The English East India Company used to buy tea and
217. (4) In 1742, Duplex was appointed as governor-general of silk from China for sale in England. For buying tea and silk
the company with authority over French investment in India. they used to trade opium.
218. (2) Anwaruddin was appointed by Nizam as the Nawab of 243. (3) Louis XVI
the Carnatic in July 1744 who maintained a cordial 244. (3) Garibaldi is usually considered the father of the Italian
relationship with East India Company and French but with unification. Between 1850 and 1870 (roughly), he lead the
his death the relationship came to an end in 1748. three wars of Independence and marched to Rome.
219. (1) Bal Gangadhar Tilak formed the All India Home Rule 245. (3) Vladimir Lenin and his Bolshevik Party led the Russian
League in 1916–18. revolution of 1917. The Bolshevik party took total control
220. (3) Jacobin Club was the most powerful political of the government, and introduced a new form of absolutism.
organisation in France The Bolsheviks, who came to be known as the Communists,
221. (4) “Dawes Plan” was introduced by the USA to take renamed the country from The Russian Empire to the Union
out Germany from the financial crisis. of Soviet Socialist Republics (USSR). (Since the collapse of
222. (3) 223. (4) 224. (2) 225. (3) the USSR in 1991, the remaining "republics" of the country
226. (1) 227. (3) 228. (3) called themselves the Russian Federation.)
229. (2) The Gita-Govind was composed by Jayadeva who was 246. (1) The Jantar Mantar is a collection of architectural
a poet of 12th century AD. astronomical instruments, built by Sawai Jai Singh who was
230. (4) Bang bhang or the Partition of Bengal was announced a Mughal Commander and served Emperor Aurangzeb and
by the Viceroy of India, Lord Curzon in 1905. later Mughals. The observatory consists of fourteen major
231. (2) Opium was cultivated in India for the first time in 16th geometric devices for measuring time, predicting eclipses,
century during the Mughal period. tracking stars' location as the earth orbits around the sun.
232. (1) The treaty of Versailles was imposed in 1919 by Germany 247. (1) Jawaharlal Nehru was the Congress President when
at the end of World War I. To establish peace among Congress declared complete independence as its goal.
Germany and its Allies. Declaration of the Independence of India was promulgated by
233. (4) the Indian National Congress on January 26, 1930, resolving
234. (2) Barbarossa operation was the name of Germany’s in- the Congress and Indian nationalists to fight for Purna Swaraj.
vasion on Russia during World War - II
Solved Questions B-187
248. (2) The last Tsar of Russia was Tsar Nicholas II. He was 261. (2) Means of production are owned by the capitalist
the last Emperor of Russia, Grand Duke of Finland, and titular entrepreneurs.
King of Poland. Nicholas II ruled from 1 November 1894 262. (4) On January 22, 1905, the day after a general strike burst
until his enforced abdication on 2 March 1917. out in St. Petersburg, Father Gapon organized a workers'
249. (3) Non-alignment movement is a group of countries who procession to present a petition to the Tsar, which ended
are not associated or aligned with or against any powerful tragically (Bloody Sunday 1905). Gapon's life was saved by
bloc. The organization was founded in Belgrade in 1961, and Pinchas Rutenberg, who took him away from the gunfire. He
was largely conceived by India's first prime minister, Jawaharlal then became the guest of Maxim Gorky.
Nehru; Indonesia’s first president, Sukarno; Egypt’s second 263. (2) The dictatorship of the proletariat as the only solution
president, Gamal Abdel Nasser; Ghana's first president Kwame for all feudal problems was advocated by Karl Marx.
Nkrumah; and Yugoslavia’s president, Josip Broz Tito. 264. (3) The correct chronological order is Home Rule League,
250. (1) The Nazi Party used populist anti-semitic views to gain votes. swaraj Party, Simon commission, Purna Swaraj, Poona Pact,
251. (3) The first major newspaper in India–The Bengal Gazette- Cripp’s Mission.
was started in 1780 under the British Raj by James Augustus
Hickey. Other newspapers such as The India Gazette, The SECTION 6. CIVICS
Calcutta Gazette, The Madras Courier (1785), The Bombay 1. (4) Dr Babasaheb Ambedkar was the chairman of the
Herald (1789), etc., soon followed. drafting Committee. The final draft of the Indian Constitution
252. (3) The first Vice-chancellor of Calcutta university was Sir was presented by him to Dr Rajendra Prasad on 25 November
James William Colvile and the first Indian Vice-chancellor 1949. He was known as the father of the Indian Constitution.
was justice Gooroodas Banerjee.. The University of Kolkata Based on the seniority level, Sachchidananda Sinha was the
in West Bengal was instituted on 24th January, 1857. The temporary President of the Constituent Assembly.
education in the university reached to its greatest height 2. (1) Narivadi Andolan was the movement that aimed to
during the vice-chancellorship of Sir Asutosh Mukherjee. fight for equal rights for women. It aimed to establish equal
253. (4) The Reichstag was a legislative body of Germany from political, economic, and social rights and opportunities for
1919, when it succeeded the Weimar National Assembly, women in India. Many Indian feminists also have struggled
until the Nazi tooke over in 1933. against issues related to our culture, and patriarchal society
254. (3) In Lahore session of the Muslim League the resolution in our country, such as inheritance laws.
for a Separate Muslim State was adopted. Lahore Resolution 3. (3) A concurrent list is a list in which both Centre and
had been the pioneering step of the Muslim League towards state can make laws on subjects mentioned in it. On the
the creation of Pakistan. Also known as Pakistan Resolution, other hand, if there is a situation of a conflict between the
the Muslim League adopted a formal resolution in the three Centre and state over the law exist in the concurrent list,
day session held from 22-24th March 1940 regarding their the law of the central government is obeyed.
claim for a separate state with Muslim majority. 4. (4) In India, seats are reserved for women in Panchayat
255. (3) Herodotus was known as the father of history because Raj bodies which are local self-governing bodies.
he was the first historian known to collect all the sources According to the 73rd amendment, one-third of the total
and materials systematically, also tested their accuracy to number of seats to be filled by direct elections present in
give a proper explanation. local bodies, such as municipalities and panchayats are
256. (2) The Boston Tea Party was an important event in the reserved for women. According to the data, more than ten
growth of the American Revolution. It was the culmination lakh women have been elected in rural and urban areas.
of a resistance movement throughout British America against 5. (1) On 1 January 1979, the Janata Party government under
the Tea Act, which had been passed by the British Parliament Prime Minister Morarji Desai established the Mandal
in 1773. On December 16, 1773, officials in Boston refused Commission, or the Socially and Educationally Backward Classes
to return three shiploads of taxed tea to Britain; a group of Commission (SEBC). It was led by an Indian parliamentarian,
colonists boarded the ships and destroyed the tea by the late B.P. Mandal. The commission aimed to identify the
throwing it into Boston Harbor. socially or educationally backward classes of India.
257. (1) European nations had a variety of motives for 6. (3) Apart from general elections and by-elections, midterm
imperializing other countries. During the growth of election refers to a type of election where the people can elect
Imperialism, there were many factors that helped its their representatives and other subnational officeholders in
expansion such as technology, economics, trade and power. the middle of the term of the executive.
Political unrest was not the factor, it must be political factor. 7. (3) All registered voters in the electoral roll of the Panchayat
258. (2) Several European countries were attracted to Indonesia in the village are members of Gram Sabha. The council leader
because of its exotic resources and its prime location for is named Sarpanch in Hindi, and each of the five members is
trade. Indonesia had hotly coveted resources, such as a Gram Panchayat Sadasya or Panch. In such a system, each
spices, cloves, nutmeg, and sugar. villager can voice his opinion in the governance of his village.
259. (1) Montesquieu, was a French social commentator and 8. (3) If a person who is not a member of parliament is
political thinker who lived during the Age of Enlightenment. appointed as a minister he has to get elected to either house
He is famous for his articulation of the theory of separation of parliament within six months. After six months he ceases
of powers, which is implemented in many constitutions to be a minister. In other words, a non-member may be
throughout the world. appointed as a minister, but he/she will have to get elected
260. (3) On August 4, 1789, a body known as the National to either house within six months.
Assembly announced the end of feudalism in France. This 9. (2) Part IVA of the constitution stipulates fundamental duties
stopped the Church from collecting taxes and most of Indian citizens which includes to abide by the constitution
importantly, stopped the forced labor from the poor peasants. and respect its ideals and institutions, the National Flag and
This also allowed positions in Church, government, and the National Anthem. This Act stipulates the punishment to
army to be open to all citizens of France. be awarded for violating Article 51Aa of the constitution.
EBD_7332
188
B- NTSE Stage 1 Question Bank

10. (3) The 42nd Constitutional amendment took place in 1976. 23. (3) The Republic is governed in terms of the Constitution
It was called Mini Constitution as it brought many of India which was adopted by the Constituent Assembly on 26th
amendments to the Constitution. The main aim of the November 1949 and came into force on 26th January 1950.
amendment is to reduce the power of the Supreme Court 24. (1) The eighteenth summit of ‘South Asian Association
and High courts and strengthening the central government. of Regional Cooperation’ (SAARC) was held in Kathmandu,
It had no amendment related to bonded labour. the capital of the Federal Democratic Republic of
11. (1) The Kesavananda judgment also defined the extent to Nepal during 26–27 November 2014.
which Parliament could restrict property rights, in pursuit 25. (3) The Rajya Sabha should consist of not more than 250
of land reform and the redistribution of large landholdings members - 238 members representing the States and Union
to cultivators, overruling previous decisions that suggested Territories, and 12 members nominated by the President.
that the property right could not be restricted. Rajya Sabha is a permanent body and is not subject to
12. (3) Constitution Day or Samvidhan Divas is also known as dissolution
National Law Day. It is celebrated in India every year on the 26. (1) “Clause (1) of Article 217 of the Constitution of India
day the Constitution of India was adopted, i.e., 26 November allows every Judge of a High Court to hold office until he
1949 by the Constituent Assembly of India. However, the attains the age of sixty-two years.
Constitution came into effect on 26 January 1950. 27. (3) The preamble is the summary of the Constitution. It
13. (1) The preamble of the Indian Constitution is:WE, THE highlights the main points and basic structure of the
PEOPLE OF INDIA, having solemnly resolved to constitute Constitution. It was adopted by the constituent assembly
India into a SOVEREIGN SOCIALIST SECULAR on November 26, the preamble aims to secure justice and
DEMOCRATIC REPUBLIC and to secure to all its citizens. liberty, provide equality to all citizens and maintain unity
Therefore, we can say that the correct sequence of the and integrity of the nation.
terms in the Constitution is Sovereign, Socialist, Secular, 28. (1) The Parliament has the power to amend the
Democratic, Republic. Constitution, though it has limited power. It cannot bring
14. (4) To be chosen a member of Parliament, a person must any change into the basic structure of the Constitution.
be a citizen of India and not less than 30 years of age in the Under Article 368 the parliament has no power to change
case of Rajya Sabha and not less than 25 years of age in the regarding the Amendment in Part III of the Constitution.
case of Lok Sabha. Also, the Parliament does not have the power to change or
15. (1) Money Bills can be introduced only in Lok Sabha (the increase its power to amend by amending Article 368.
directly ele cted ‘people’s house’ of the Indian Parliament). 29. (4) A system of government, such as in the United States,
Money bills passed by the Lok Sabha are sent to the Rajya that divides power and responsibilities between the two
Sabha (the upper house of parliament, elected by the state levels of government, i.e., national and state is known as the
and territorial legislatures or appointed by the president). federal system. In the legal context, Independence of the
16. (3) The International Court of Justice is composed of 15 Judiciary can be defined as the power to uphold without fear
judges elected to nine-year terms of office by the United or favour, the Rule of Law, personal freedom and liberty, equality
Nations General Assembly and the Security Council. before the law and impartial and effective judicial control over
17. (1) Originally the age of the retirement of the judges of administrative and executive actions of the Government. They
the High Courts was fixed at 60 but it was raised to 62 in must be free to judicial functions.
1963 according to the 15th amendment of the Constitution. 30. (1) Ganesh Vasudev Mavalankar (27 November 1888 – 27
Supreme court judges retire at the age of 65. February 1956) popularly known as Dadasaheb was an
18. (3) The World Health Organ ization (WHO/OMS) independent activist, the President (from 1946 to 1947) of
headquarters is situated on the outskirts of Geneva at the the Central Legislative Assembly, then-Speaker of the
end of Avenue Appia, approximately three kilometres from Constituent Assembly of India, and later the first Speaker
the centre of town, and about one and a half kilometres from of the Lok Sabha.
the Palais des Nations (UN). 31. (2) The Right of Children to Free and Compulsory Education
19. (3) The sixty-first amendment of the Constitution of India, Act or Right to Education Act (RTE) is an Act of the
officially known as The Constitution (Sixty-first Parliament of India enacted on 4 August 2009, which
Amendment) Act, 1988, lowered the age for voting of describes the modalities of the importance of free and
elections to the Lok Sabha and the legislative assemblies of compulsory education for children between the age of 6 to
States from 21 years to 18 years. This was done by amending 14 years in India under Article 21A of the constitution.
Article 326 of the Constitution, which concerns elections 32. (3) The All India Forward Bloc (AIFB) is a left-wing
to the Lok Sabha and the Assemblies. nationalist political party in India. It emerged as a faction
20. (1) The Panchsheel, or Five Principles of Peaceful within the Indian National Congress in 1939, led by Subhas
Coexistence, was first formally signed on April 29, 1954, Chandra Bose. The party re-established as an independent
between India and the Tibet region of China. The agreement political party after the independence of India. It has its
was signed between then Prime Minister Jawaharlal Nehru and main stronghold in West Bengal.
China’s first Premier (Prime Minister) Chou En-Lai. 33. (1) The National Democratic Alliance is an Indian political
21. (1) Rajasthan was formed on 30 March 1949. When India alliance made up of predominantly centre-right political
became independent, 23 princely states were consolidated parties and led by the Bhartiya Janata Party. It was founded
to form the State of Rajasthan, “home of rajas”. in 1998 and currently controls the Indian union government
22. (2) There are significant difference as well as the government of 18 Indian states.
between the traditional Panchayati Raj system, which was 34. (1) A communication allotted by a proper authority stating
envisioned by Gandhi, and the system formalized in India the policy or decision of the government is referred to as
in 1992. The Panchayat Raj system was first adopted an office memorandum. The chief purpose that a
by the state of Rajasthan in Nagaur district on 2 Oct 1959 memorandum justifies is that it targets to record and relay
but first started by Andhra Pradesh. information and to make brief requests.
Solved Questions B-189
35. (3) The Institutional Revolutionary Party (Spanish: Partido three new words Socialist, Secular, and Integrity were
Revolucionario Institucional, PRI) is a Mexican political added. The term secular in the Constitution of India means
party founded in 1929 that held uninterrupted power in the that all the religions in India get equal respect, protection
country for 71 years from 1929 to 2000, first as the National and support from the state. Articles 25 to 28 in Part III of
Revolutionary Party (Spanish: Partido Nacional the Constitution guarantee Freedom of Religion as a
Revolucionario, PNR). Fundamental Right.
36. (3) The Government of India Act, 1935 was passed by 48. (3) Jammu & Kashmir: The Article 370 of Indian
British Parliament in August 1935. With 321 sections and Constitution, which came into power in 1952, had
10 schedules, this was the longest act passed by British “temporary provisions with respect to the State of Jammu
Parliament so far and was later split into two parts. The act and Kashmir” which gave special powers to the state
helped in the reorganization of the states. allowing it to have its own Constitution. It also empowered
37. (4) The elections to the 243-seat Bihar Assembly took Jammu and Kashmir to be a near autonomous state since
place in three phases the first for 71 seats on 28 October it limited the Centre’s authority to adjust external affairs,
2020, the second for 94 seats on 3 November 2020, and the defence, finance and communication.
third for the remaining 78 seats on 7 November 2020. These 49. (4) Meira Kumar
were the first big elections to take place amid the Covid-19 50. (3) Rajasthan
pandemic outbreak. 51. (2) Dr. Zakir Hussain
38. (4) A cabinet is a body of high-ranking state officials, 52. (1) The first general elections were held between October
typically consisting of the executive branch’s top leaders. 25, 1951 and March 27, 1952. The voter turnout for the
Members of a cabinet are usually called cabinet ministers elections was 45.7 %. The Congress won 364 of 489 Lok
or secretaries. Cabinet committees are extra-constitutional Sabha seats and Nehru became the first prime minister of
bodies. However, they derive their legality from India. The results of the first general elections of India
the constitution. In constitutional parlance, a government were announced on February 10, 1952. The day is also
is called an executive, which also includes the President of celebrated as India’s first democratic leap as these were
India. the first general elections in India.
39. (1) Its establishment is defined in Article 169 of the 53. (3) Article 12 to 35 contained in Part III of the Constitution
Constitution of India. As of Jan 2020, 6 out of 28 states deal with Fundamental Rights. These are guaranteed in the
have a State Legislative Council. These are Andhra Pradesh, Constitution in the form of six broad categories of Fundamental
Karnataka, Telangana, Maharashtra, Bihar, and Uttar Rights, which are justiciable. Initially, the constitution of India
Pradesh The latest state to have a council is Telangana. had 7 Fundamental Rights that are borrowed from the
40. (2) An indirect election is an election in which voters do Constitution of the USA. But later on, Right to property was
not choose between candidates for an office, but elect abolished and now there are just 6 Fundamental Rights in
people who then choose. In an indirect election, the voters force. The fundamental rights are not absolute because they
elect a body which in turn elects the officeholder in can be controlled and are subject to reasonable restrictions
question. It is also used for some supranational legislatures. for the protection of general welfare.
41. (1) When the General Assembly decides on important 54. (1) The Shiromani Akali Dal (SAD) is a state political
questions such as those on peace and security, admission party in Punjab, India. Dravida Munnetra Kazhagam
of new members and budgetary matters, a two-thirds (DMK) is a political party in the state of Tamil Nadu and
majority of those present and voting is required. Union Territory of Puducherry. Asom Gana Parishad (AGP) is a
42. (1) The INC has increased women’s participation by state political party in Assam. Bharatiya Lok Dal was a political
instituting a 33% quota for women at all levels of the party. party in India. The BLD was formed at the end of 1974
43. (2) There were ten fundamental duties. Then, due to the through the fusion of seven parties opposed to the rule
86th Amendment in the Constitution in the year 2002, 11th of Indira Gandhi, including the Swatantra Party, the Utkal
duty was added.11 Fundamental duties have been Congress, the Bharatiya Kranti Dal, and the Socialist
delineated in the 4th chapter of the Indian constitution. Party. The leader of the BLD was Charan Singh.
44. (4) 55. (1) A coalition government is one in which multiple
45. (3) Unnikrishnan v/s Andhra Pradesh Government,1993 political parties come together and often reduce the
was a watershed judgement in the history of education in dominance of the party that has won the highest number of
India. In this case, the Supreme Court declared the right to seats. Coalition governments are formed as seldom does a
education as a fundamental right. It also mentioned that it political party win the necessary number of seats to form
was the duty of the state to provide free and compulsory the government on its own. In India, the first coalition
education to all children between the age of 6 to 14 years government to complete its full-term was the Atal Bihari
under Article 21A of the Indian Constitution. Vajpayee-led National Democratic Alliance from 1999 to 2004.
46. (3) 2005: The Government of India has enacted “The 56. (3) According to the constitution articles option 3 is the
Right to Information Act 2005” to set out the practical correct response.
regime of Right to Information (RTI) for citizens to secure 57. (2) Related to the process of creation of an act in the
access to information under the control of public state’s legislature option 2 is correct.
authorities to promote transparency and accountability in The proper order of process of creation of an act is as follows—
the working of any public authority. The application/ The person who tables the bill reads out the text loudly.
request for information must be disposed off either by A discussion takes place on the bil and sent to
providing the information or rejecting the request, within respective house committee.
a period of 30 days. The house approves the bill with 2/3 majority.
47. (2) Socialist: The Preamble is called the introduction letter The bill is sent to the Governor’s office for approval.
of the Indian Constitution. The preamble was amended by 58. (2) Aristotle was declared as a father of Political science,
the 42nd Constitutional Amendment Act in 1976, in which he wrote the famous book ‘Politics’.
EBD_7332
190
B- NTSE Stage 1 Question Bank

Aristotle was a Greek philosopher, logician, and scientist. presided over by the Speaker or, in his absence, by the
Aristotle is generally regarded as one of the most influential Deputy Speaker of the Lok Sabha or in his absence, the
ancient thinkers in a number of philosophical fields, Deputy-Chairman of the Rajya Sabha. The Chairman
including political theory. He has discussed the origin, doesn’t preside over the joint session at any means/cost.
development and functions of state in his book - Politics. 68. (4) A good governance usually has the following features:
59. (4) Related to Karnataka Public Service Commission 1. Participation 2. Rule of Law 3. Transparency
functions- Train the teachers on modern skills of teaching 4. Responsiveness 5. Consensus 6. Orientation 7. Equity
is wrong. 8. Effectiveness an d Efficiency 9. Accountability
The function of Karnataka Public Service Commission— 10. Strategic Vision
to appoint the gazette and non-gazette officers Thus we find that the accountability is essential to
to conduct interviews for direct recruitment of candidates establish good governance.
to conduct departmental exams for state government 69. (4) In a democracy, the people of a country are involved in
employees. choosing its leader or head. The people are involved in the
Karnataka-SSA training recognizes the critical and central process of forming a government. They have the free will and
role of teachers and gives a focus on their academic, right to vote for a party to come into power. The right to vote
professional and technical needs. is not determined by wealth or class or race. In a democracy,
60. (3) Jharkhand Mukti Morcha came into being as a result there are different parties that have a manifesto or an idea
of action of companies that rendered thousands of tribal about how a country should be governed. The protection of
displacements due to mining. human values in a democratic state is not compulsory.
Jharkhand Mukti Morcha organization is a regional political 70. (3) The Community government in Belgium is elected by
party mainly centered in Jharkhand state. The Jharkhand people belonging to one language community— Dutch,
Mukti Morcha was organized to oppose the companies that French and German speaking, no matter where they live.
rendered thousands of tribal displacements due to mining. This government has the power relating to culture,
61. (3) The main aim of 73rd amendment of the constitution in education and language issues.
1993 with respect to Panchayatraj system is to decentralize 71. (3) A municipal corporation is administratively headed
and confer constitutional status to the Panchayat institutions. by a municipal commissioner, who is the head of the
62. (4) H.J. Laski: Harold Joseph Laski (1893–1950) was a municipal corporation. It is usually granted to a city of
British political scientist, educator, and prominent member more than one million in population. While a mayor is
of the British Labour Party who turned to Marxism in his elected to serve as the nominal head of a municipal
effort to interpret the “crisis in democracy” in Britain corporation. A municipal commissioner is appointed by
during the economic depression of the 1930s. the state government from the Indian Administrative
Pluralism states that politics and decision making are located Service and mostly from State Civil Services to supervise
mostly in the framework of government, but that many non- the administrative staff of the municipal corporation.
governmental groups use their resources to exert influence. 72. (3) In a democracy, the power is concentrated in the hands
63. (3) Speaker of Lok Sabha: The Speaker of the Lok Sabha of electorates. An election is a way for citizens (electorates)
conducts the proceedings in house, and decides whether a in a democracy to communicate with representatives.
bill is a money bill or not. He maintains discipline and decorum Different rules about elections and concentration of
in the house and can punish a member for unruly behaviour assembly power can create either majoritarian or
by suspending. He also permits the moving of various kinds proportional systems of democracy.
of motions and resolutions such as a motion of no confidence, 73. (1) Mahatma Gandhi was not the member of the
motion of adjournment, motion of censure and calling Constituent Assembly, yet there were many members who
attention notice as per the rules. The Speaker decides on the followed his vision.
agenda to be taken up for discussion during the meeting. 74. (1) The United Nation Secretary General Kofi Annan
64. (2) Part IV-A of the Indian Constitution deals with declared explicitly for the first time that the US led war on
Fundamental Duties. Originally, the Constitution of India Iraq was not legal. The war against Iraq was not authorised
did not contain these duties. Fundamental duties were by the UN Security Council.
added by 42nd Constitutional Amendment acts of 1976. 75. (3) The President can declare emergency on the basis of
Presently we have 11 fundamental duties in our the written request by the cabinet headed by the Prime
constitution under article 51-A, which are statutory duties. Minister.
65. (4) Under the Article 80 of the Constitution, the 76. (2) Right to equality is a part of fundamental right which is
presidentmay ominate 12 members to the Rajya Sabha from related to end racial discrimination.
people who have special knowledge or practical experience 77. (4) The movement for the individual and family right of
in art, literature, science and social service. Rajya Sabha, woman is known as Nari Shasktikaran Andolan.
at present, has 245 seats. Of these, 233 members repre- 78. (4) Transparency means that the decisions are taken with
sent the States and the Union Territories, and 12 members honesty and rules are properly followed. In decision-mak-
are nominated by the President. ing process of the government, people are concerned about
66. (2) Emergency provisions in India are borrowed from what changes in laws are made.
Weimar Constitution of Germany. The Indian Constitution 79. (2) Amnesty International is an international human rights
gives President the authority to declare three types of organisation. It is a London-based non-governmental
emergencies: national emergency (Under Article 352), state organisation. The organisation claims to have more than
emergency (Under Article 356), and financial emergency seven million members and supporters around the world.
(Under Article 360). 80. (1) India is a Sovereign, Socialist, Secular, Democratic
67. (2) Article 108 of the Indian Constitution of India republic.
provides for Joint sitting of both the Houses. The joint 81. (4) The Parliament of India consists of the President, Rajya
sitting of the Parliament is called by the President and is Sabha and Lok Sabha.
Solved Questions B-191
82. (2) Article 14 of the Indian Constitution provides that the 115. (4) The pure form of direct democracy exists only in the
state shall not deny to any person equality before the law or swiss cantons. Switzerland is a rare example of a country
the equal protection of laws within the territory of India. with instruments of direct democracy.
83. (3) Option three is odd pair of persondities the others are 116. (4) The World Trade Organisation was extablished on 1
similiar, i.e., 1, 2 and 4. January, 1995. It is an intergovernmental organisation that
84. (3) The speaker of the Lok Sabha presides the joint ses- regulates international trade.
sion of Parliament. 117. (4) The Permanent Member of United Nation security
85. (4) Bangladesh became an independent and sovereign Councial are five : China, France, Russia, United Kingdom
Country in 1971, earlier it was part of Pakistan. and the United States.
86. (3) The commission was headed by Fazal Ali, the retired 118. (1) The United Nation Organisation was established on
Chief Justice of the supreme court. 24 october 1945, San Francisco, Californai United States. It
87. (4) The word secular was inserted in the Constitution by was established after world war II with the aim of preventing
the 42nd amendment 1976. another such conflict.
88. (3) The summit was held at Kaula Lumpur. 119. (4) Section 25 says that all persons are equally entitled to
89. (3) Session Judge handles criminal cases. freely profens practice and propagate religion. It guarantees
90. (2) The autobiography written by Nelson Mandela is the free entry to all people to social and religious places.
Long Walk to Freedom. It was published in 1994. 120. (1)
91. (1) The Union government legislates on residuary subjects. 121. (3) The Governors of the state are appointed by the
92. (1) India has the largest number of voters in the world. president of India for a period of five years Vice President
93. (4) Union Council of ministers include cabinet ministers acts as the ex-officio chairman of Rajya Sabha. In this
who are usually top level leaders of the ruling party. Minis- capacity, his powers and functions are similar to those of
ters of state with independent charge are usually in-charge the speaker of Lok Sabha. On 1 January 2015 a cabinet
of small Ministries, Ministers of state are attached to and resolution was passed to replace the Planning Commission
required to assist Cabinet Ministers. with the newly formed NITI Ayog (National Institution for
94. (1) The President of India may declare National Emergency Transforming India) Its chairperson is Shri Narendra Modi,
in case of external aggression and internal disturbances. Hon’ble Prime Minister Finance Minister presents Union
95. (2) The secretary General of the UNO– Kofi Annan, Budget in the Parliament.
Boutros Ghali, U-Thant, Antonio Gutrres. 122. (1) Shive sena a regional party of Maharashtra formed in
96. (2) Directive Principles enshrined in the Constitution of 1966 by Bala Sahab Thackeray had political symbol bow
India for the Government administration. People cannot and arrow AIADMK (All India Anna Dravida Munnnetra
question in the court for not implementing Directive Kazhagaon) founded in 1972 by M.G. Rama Chandran. Its
principles. People can move to court if their fundamental symbol is two leaves peoples Democratic Party in Jammu
rights are violated. and Kashmir founded in 1999 by Mufti. Mohammed Sayeed.
97. (3) 98. (2) 99. (2) Its symbol is Inkpot and Pen JD(U) Janta Dal (United) was
100. (2) The finance bill is first presentated in the Lok Shabha. formed in 2003. Its election symbol is arrow.
Lok Sabha exercises more powers in money matters. 123. (4)
101. (3) 124. (2) The organization of African Unity was established on
102. (4) Parliament is the final authority for making laws in the 25 May 1963 in Addis Ababa the Association of Southeast
country Parliament all over the World can make new laws, Asian Nations (ASEAN) is a regional gouping founded on
charge existing laws or abolish existing laws and make new 8 August, 1967. The South Asian Association for Regional
one is their place. cooperation (SAARC) was founded in Dhaka on 8 December,
103. (3) Right to protect one’s culture is considered as the fun- 1985. European Union was created by the maastricht Treaty
damental Right. It means any section of the citizen with a in November 1993.
distinct language or culture have a right to conserve. 125. (2) Governor of the state is appointed by the president.
104. (1) Governor can be dismissed by the president.
105. (2) At the time of election, the candiate has to follow the 126. (1) The anti-defection law was passed by parliament in 1985,
norms and guidlines. Even the political parties too have to it sets the provision for disqualification of elected members
follow set of norms and guidelines. on the grounds of defection to another political parts.
106. (2) The maximum number of Judges that can be appointed 127. (1) Both the statements are true power sharing is the very
in Supreme Court is 30 + 1. spirit of democracy. It also helps reduce the possibility of
107. (3) conflict between social groups. It ensures stability of
108. (2) According to Article 75(1) of the Indian constitution political order.
the Prime Minister shall be appointed by the President. 128. (2)
109. (3) The Vice-President of India is elected indirectly by an 129. (4) All the rights are included under the Indian Constitution.
electoral colllege consisting of both the houses of Parliament. 130. (2)
110. (4) 131. (2) A pressure group tries to influence and pressurise the
111. (2) Garner, An American prosessor of political science state government in order to fulfil the interests of its members or
the statement the group interest in general. Unlike political parties.
112. (1) In the Lok Sabha two members of the Anglo-Indian are 132. (2) Mr. Justice H.L. Datta former Chief Justice of India is
community nominated by the president of India. the chairperson of National Human Rights Commission.
113. (2) Joint Session of the Indian Parliamen tis presided by 133. (3) Concurrent list includes subject of common interest to
speaker. In his absence by the Deputy speaker of Lok Sabha. both the union government and the state government such
114. (3) The councial of Ministers of responsible to the as marriage adoption etc.
Parliament, in Indian Parliamentary system of governemnt. 134. (1) Cochabamba
EBD_7332
192
B- NTSE Stage 1 Question Bank

135. (4) Religious Justice does not exist in preamble of Indian 174. (4) China has adopted the one single party system and it
constitution. It provides to its citizens social, economical and is known as Chinese Communist Party.
political justice. 175. (4) The Indian government has raised reservations for
136. (2) P.V. Sindhu won the silver medal for India in badminton women in all tiers of the panchayati raj system from 33 to at
in Rio Olympics 2016. least 50%. The Union Cabinet, presided over by Prime Minister
137. (3) Manmohan Singh, cleared a proposal to amend Article 243(4)
138. (2) United Nations Organisation is the institution which of the Constitution for this purpose on August 27, 2009.
functions for international law, security, social equality and 176. (1) A defector is basically a person in politics who gives
world peace. up his own political party and join other alliance for benefit.
139. (4) Dr. B.R. Ambedkar was the pioneer to abolish caste This term is also applied, often pejoratively, to anyone who
inequality and establish social harmony. switches loyalty to another religion, sports team, political
140. (2) 141. (1) 142. (4) 143. (2) party, or other rival faction. In that sense, the defector is
144. (4) The constitution of India was adopted on 26 January, 1950. often considered a traitor by his original side.
145. (1) The chairman of the drafting committee of the 177. (4) In the Indian constitution, there are 5 writs according
Constituent Assembly of India was B.R. Ambedkar. to Article 32 that empower the Supreme court to enforce the
146. (3) "Democracy is a government of the people, for the Fundamental right of an individual. Dr. B R Ambedkar, the
people, by the people was said by Abraham Lincoln. chairman of the Drafting committee called Art. 32 the
147. (1) The Chief Justice of Supreme Court is appointed by Fundamental Right to Constitutional Remedies as the heart
the President. and soul of the Indian constitution. According to this right,
148. (4) When a bill is presented at the table of Lok Sabha, the a person can move the Supreme Court in case of violation of
speaker of the Lok Sabha decides whether a bill is Money their fundamental rights.
Bill or not. 178. (1) There are 15 members of the Security Council. This
149. (4) In contemporary world legitimacy governance makes includes five veto-wielding permanent members-China,
democracy superior to any other form of government that France, Russia, the United Kingdom, and the United States-
ensures appropriate use of power by a legally constituted based on the great powers that were the victors of World
government. War II.
150. (4) Gender division refers to the difference between the 179. (2) ZERO HOUR denotes the time immediately following
tasks assigned to men and women. the Question Hour in both Houses of Parliament. This is
151. (3) The “Kittiko-Hachchiko” movement was started in about 12 noon which is why it is called Zero Hour. During
Karnataka in 1987. this hour members can raise matters of great importance
152. (3) Mayor is the political head of Municipal Corporation, without prior notice. However, the duration of the Zero Hour
who is elected by the members of the corporation. has varied over the years. It is not possible to predict what
153. (3) The Assam Gana Parishad is a state political party in kind of matters might be raised during Zero Hour as there is
Assam grew out of a student movement. no mention of any Zero Hour in the rules of the Parliament.
154. ( 2) Amnesty International was founded in London in 1961 180. (3) Home minister is a part of Political Executive. The
is an international organisation which works for Human Rights. Ministry of Home Affairs (MHA) or Home Ministry is a
155. (3) Right to Information Act is the law which empowers ministry of the Government of India. It is mainly responsible
the people to carry on democratic reforms. It mandates timely for the maintenance of internal security and domestic policy.
response to citizen requests for government information. The Home Ministry is headed by Union Minister of Home
156. (2) Pressure group is an organised group that controlled Affairs.
directly the political power but try to influence the politics 181. (2) Universal Adult Franchise means the right to vote to
of government. every adult of the country. The right to vote is not restricted
157. (2) In case of conflict between the centre and the State by race, sex, belief, wealth, or social status. In India it was
Government over a subject in the concurrent list, the centre granted in 1950.
has the upper hand. The constitution provides primacy to 182. (4) India was inspired to include Directive Principles of
parliament on concurrent list items; if there is a conflict, a State Policy in the Indian Constitution from Ireland. The
central law will override a state law. Constitution lays down certain Directive Principles of State
158. (4) There are total 22 languages which got recognition in Policy which though not justiceable, are 'fundamental in
the eighth schedule of Indian constitution. governance of the country' and it is the duty of the State to
159. (3) Corruption is considered as anti–national and economic apply these principles in making laws.
crime. 183. (2) Article 31-B was inserted by the First Constitutional
160. (3) In 1946 the features of Indian Foreign Policy was (Amendment) Act 1951. Article 31-B of the Constitution of
explained by Pandit Jawaharlal Nehru. India ensured that any law in the Ninth Schedule could not
161. (4) The untouchability crime act was implemented in 1955. be challenged in courts and Government can rationalize its
162. (3) Right to Information Act is the law which empowers programme of social engineering by reforming land and
the people to carry on democratic reforms. agrarian laws.
163. (2) 164. (3) 165. (3) 184. (4) Sikkim became the 22nd Indian State on April 26, 1975.
166. (2) King Gyanendra was the King of Nepal in 2006 during On May 16, 1975, Sikkim officially became a state of the
the second movement of democracy. Indian Union and Lhendup Dorji became head of State (chief
167. (1) Assam movement later converted into a political party. minister).
168. (2) 169. (2) 170. (1) 171. (2) 185. (1) Long Walk to Freedom is an autobiographical work
172. (2) The President is the first citizen of any country. written by South African President Nelson Mandela, and
173. (4) Code of conducts are the specific set of issue for political published in 1994 by Little Brown & Co. The book profiles
parties in election. his early life, coming of age, education and 27 years in prison.
Under the apartheid government, Mandela was regarded as
Solved Questions B-193
a terrorist and jailed on the infamous Robben Island for his 204. (4) The National Human Rights Commission (NHRC) of
role as a leader of the then-outlawed ANC. He has since India was setup in India to protect Human Rights on 12
achieved international recognition for his leadership as October 1993. It is responsible for the protection and
president in rebuilding the country's once segregated promotion of human rights, defined by the act as “rights
society. relating to life, liberty, equality and dignity of the individual.”
186. (1) The Right to Information Act (RTI) is an Act of the 205. (3) South Asian Preferential Trade Agreement.
Parliament of India "to provide for setting out the practical
regime of right to information for citizens" and replaces the SECTION 7. ECONOMICS
erstwhile Freedom of Information Act, 2002. The Act applies 1. (3) In India, the Golden revolution referred to the period
to all States and Union Territories of India except the State between 1991 to 2003. This revolution is associated with
of Jammu and Kashmir. the production of honey and horticulture. It is one of the
187. (4) Newzealand is the first nation of the world that provided important agricultural revolutions of India. It led to an
adult franchise. increase in the production of fruits, vegetables, flowers,
188. (2) They are not absolute in nature. They are subject to aromatic plants, spices, etc. Nirpakh Tutaj is known as the
reasonable restrictions. They strike a balance between father of the golden revolution.
individual liberty and social Security. But the reasonable 2. (2) Voluntary unemployment can be defined as the
restrictions are subject to Judicial review. situation when the workers intentionally and consciously
189. (1) The President has the power to summon and prorogue choose to leave due to certain reasons such as low salary
either House of Parliament or to dissolve Lok Sabha. package, lack of opportunity to work. Under this type of
190. (4) The Constitution (61st Amendment) Act, 1989: It employment, the worker remains unemployed even if the
lowered the voting age from 21 to 18. jobs are available in the market.
191. (3) National integration means creating a mental outlook 3. (4) The Human Development Report (HDR) is an
which will promote and inspire every person to place loyalty annual report published by the Human Development Report
to the country above group loyalties and the welfare of the Office of the United Nations Development Programme
country above narrow sectarian interest that is what (UNDP). The first HDR was launched in 1990 by the
secularism does. Pakistani economist Mahbub ul Haq and Indian Nobel
192. (1) The president is supreme commander of armed forces, laureate Amartya Sen.
but the functioning of armed forces is controlled by ministry 4. (2) Gurgaon is the most developed commercial and business
a panel of high ranked officers, during war time also the centre of Haryana, located on the outskirts of Delhi. It is spread
parliament and ministry decides what should armed force over 2,766 sq km and has a population of over 600,000. The
do. industrial areas around Gurgaon house most of the automobile
193. (4) Joint session of Parliament can be ordered by President. and auto component manufacturers in the state.
Whereas, it presided over by the speaker of Lok Sabha or in 5. (4) A tax that is paid by a person or organisation directly
his absence by Deputy Speaker, or in his absence by the to the entity that imposed it. For example, a person pays
deputy chairman of Rajya Sabha. direct taxes to the government for various purposes,
194. (1) The words 'socialist and 'secular' were included in the including income tax, property tax, corporate tax, or taxes
preamble to the Indian Constitution by 42nd Amendment on assets. Direct taxes are dependent upon the income of
Act. an individual.
195. (4) The Government of Switzerland is Unitary in form. 6. (1) The National Cooperative Union of India celebrated
196. (4) The rule of law means that the law cannot discriminate the 67th All India Cooperative Week from 14th to 20th
between persons on the basis of their religion, caste or November 2020. This celebration took place for a week for
gender. various purposes. Here, cooperatives have the opportunity
197. (2) The Constitution of India came into force on January to share their success stories, and achievements with
26, 1950. The first general elections under the new others. They also develop future action plans for their
Constitution were held during the year 1951-52 and the first organisations.
elected Parliament came into being in April 1952. 7. (2) The WTO began life on 1 January 1995, but its trading
198. (3) The international court of Justice is located in Hague system is half a century older. Since 1948, the General
in Netherland. It is the primary judicial branch of the United Agreement on Tariffs and Trade (GATT) had provided the
Nations. Its main functions are to settle legal disputes rules for the system. (The second WTO ministerial meeting,
submitted to it by states and to provide advisory opinions held in Geneva in May 1998, included a celebration of the
on legal questions submitted to it by duly authorized 50th anniversary of the system.)
international branches, agencies, and the UN General 8. (1) Laissez-faire economics is a key part of free-
Assembly. market capitalism. The policy of laissez-faire received
199. (4) The main reason for Indo-China war in 1962 was border strong support in classical economics as it developed in
dispute between two countries, growing friendship between Great Britain under the influence of the philosopher and
India and former Soviet Union, and china saw India as their economist Adam Smith.
main rival but growing friendship between India and U.S.A 9. (3) In economics, the exclusion principle states “the
was not a reason. owner of a private good may exclude others from use unless
200. (4) The decision of the Supreme Court is final and binding they pay.”; it excludes those who are unwilling or unable to
on all other courts. pay for the private good, but does not apply to public
201. (4) Indian Society has a multi party system. goods that are known to be indivisible: s
202. (4) The Block Development Officer of the Gram Panchayat 10. (3) Indian railways in an example of monopoly as there is
is appointed by the Indian Administrative Services. so close substitute for Indian railways which provide
203. (1) environment transportation at such nominal rates.
EBD_7332
194
B- NTSE Stage 1 Question Bank

11. (1) The ability to pay is another criterion of equity or are hired as cheap and fast workers. There is no security
fairness in taxation. This theory requires that individuals for the employers, due to the involvement of highly
should be asked to pay taxes according to their ability to unskilled children. This sector also faces the challenges of
pay. The rich have a greater ability to pay, therefore they migration.
should pay more tax to the Government than the poor. 24. (4) A direct tax is a tax that is paid directly to an imposing
12. (3) In underdeveloped countries, most of the labour force entity, usually the government, by an individual or an
is generally engaged in the agricultural sector. People are organisation. This type of tax cannot be transferred to
involved in agriculture as there is a fewer number of another individual or entity. Income Tax, Corporate Tax,
employment opportunities. Wealth Tax, Stamp Duty are direct taxes. The collection
13. (2) India too is a mixed economy and it has adopted this and levying of direct taxes are regulated by the Central
approach post-independence. The industrial policies Board of Direct Taxes.
implemented in the year 1948 and 1956 have helped the 25. (2) Antyodaya Anna Yojana was launched in 2000 to
private and the public sector to co-exist. provide foodgrains, namely, rice and wheat to extremely
14. (4) The Reserve Bank of India (RBI) is the central bank of poor families in rural India. Under this scheme, one crore
India, which was established on Apr. 1, 1935, under the Reserve BPL (Below Poverty Line) families covered under the
Bank of India Act. The Reserve Bank of India uses monetary targeted public distribution system were identified. Poor
policy to create financial stability in India, and it is charged families were identified by the respective state rural
with regulating the country’s currency and credit systems. development departments through a BPL survey.
15. (2) Privatization can be defined as the process by which It was first implemented in Rajasthan. Each of the identified
something can be moved from the public sector to the families was issued 25 kg foodgrains (rice and wheat)
private sector. This process increases the role of the private initially, but this was later increased to 35 kg per family with
sector in the economy. Nowadays privation of the different effect from 1st April 2002.
sectors is taking place. 26. (2) Fish production: The rapid increase in the production
16. (2) The money which is issued by the central bank of a of fish and marine products through a package program is
country in form of metals that acts as an unlimited legal known as the Blue Revolution.
tender in the economy is known as metallic money. This 27. (1) 1956: Oil and Natural Gas Corporation (ONGC) is an
kind of money is made of precious metals such as gold and Indian Multinational Crude Oil and Gas Corporation. Its
silver. The face value of the money can be determined by registered office is at New Delhi. It is a state-owned enterprise
the value inscribed on the coins. of the Government of India under the administrative control
17. (3) A mixed economy is can be defined as when both of the Ministry of Petroleum and Natural Gas.
public and private enterprises exist together. Along with ONGC was founded on 14 August, 1956 by Government
there is a level of freedom to utilize capital to earn profit. of India. It is involved in exploring and exploiting
However, the interference of the government aims to benefit hydrocarbons.
society. Therefore we can say that a mixed economy is where 28. (1) Bretton Woods Committee: International Monetary
both capitalism and socialism exist at the same time. Fund (IMF) was formed in 1944 at the Bretton Woods
18. (1) In Maharashtra, the Employment Guarantee Scheme Conference primarily by the ideas of Harry Dexter White
(EGS) evolved from the relief employment programs that and John Maynard Keynes. It came into formal existence
were undertaken during a severe drought in the state from in 1945 with 29 member countries and the goal of
1970 to 1973. The droughtaffected 15 to 30 million rural reconstructing the international payment system.
people, constituting from 43 percent to 86 percent of the 29. (3) Washington: The World Bank is based in Washington,
state’s rural population. D.C. The World Bank is an international organization
19. (1) It was certainly a shock to Marco Polo, who wrote dedicated to providing financing, advice and research to
about it in his travels, saying: With these pieces of paper they developing nations to aid their economic advancement.
can buy anything and pay for anything. And I can tell you 30. (1) Ricardo
that the papers that reckon as ten bezants do not weigh one. 31. (3) A central bank is an independent national authority
20. (1) The entrepreneur is commonly seen as a business leader that controls monetary policy, acts as a banker’s bank,
and innovator of new ideas and business processes. and provide financial services. It is not related with
21. (3) controlling government spending.
(1) Minimum wages Act came into existence in 1948. 32. (2) The Human Development Index (HDI) is an index of
(2) Prevention of unethical activities Act came into life expectancy, education, and per capita income. A country
enforcement in 1956. scores a higher HDI when the lifespan is higher, the
(3) Prohibition of bonded labour Act came into education level is higher, and the gross national income
enforcement in 1975. per capita is higher. Environmental condition does not
(4) Human rights protection Act came into enforcement determine the HDI.
in 1993. 33. (1) Globalization is the process of interaction and
22. (1) In the journal, business transactions are initially integration among people, companies, and governments
recorded and therefore it is called the book of original entry. worldwide. It has benefitted to developed countries than
It is known so because in this book. All business developing countries. It has promoted the living conditions
transactions, their details and descriptions are first recorded of workers in developing countries. There is no negative
in the book of original entry. The full particulars of the effect over cement companies.
transactions are recorded first in the Journal. 34. (2) A Self Help Group is generally composed of over 20 to
23. (4) Organised labour in India is covered under the 50 members comprising of both male and female from a certain
Gratuity Act of 1971. In India, agriculture is one of the fields community. This committee is usually found and prevalent
that give employment to unorganised labourers. Child in rural areas. The group members themselves are responsible
labour mainly comes under the unorganised sector. Children for taking decision about savings and loan activities.
Solved Questions B-195
35. (2) The World Trade Organization (WTO) created in 1995 studies for high speed rail connectivity viz: Delhi-Mumbai,
is the only global international organization dealing with Mumbai-Chennai, Chennai-Kolkata, Kolkata-Delhi, Delhi-
the rules of trade between nations. It superseded the Chennai and Mumbai-Kolkata routes.
1947 General Agreement on Tariffs and Trade 46. (4) Capitalism is an economic system based on the private
(GATT) created in the wake of World War II. The primary ownership of the means of production and their operation
purpose of the WTO is to open trade for the benefit of all. for profit. Characteristics of a Capitalist Economy include
36. (3) Credit means a loan, an agreement in which the lender Right to Private Property, Price Mechanism, Profit Motive
(creditor) supplies the borrower with money, goods or and Freedom of Enterprise.
services which is to be returned in future. Terms of credit 47. (4) The Human Development Index (HDI) is a statistical
apart from the rate of interest, collateral also includes tool used to measure a country’s overall achievement in
documentation, mode of repayment. Cheque does not its social and economic dimensions. The social and
include in terms of Credit. economic dimensions of a country are based on the health
37. (2) In the rural areas, the unorganised sector mostly of people, their level of education attainment and their
comprises of landless agricultural labourers, small and standard of living. This index can also be used to examine
marginal farmers, share croppers and artisans (such as the various policy choice of nations.
weavers, blacksmiths, carpenters and goldsmiths). 48. (3) National Bank for Agriculture and Rural Development
38. (4) Globalisation means unification on integration of the (NABARD) is an apex development financial institution in
domestic economy with the world economy through trade, India. It looks after the credit needs of agriculture and rural
capital and technology flows. Government can ensure fair development in India.
globalisation to its people by 49. (2) There are three different phases in circular flow of
(i) ensuring that the labour laws of the country are income. These are production phase, income phase and
followed so that workers are not exploited. expenditure phase.
(ii) supporting small producers to improve their performance 50. (2) The Integrated Child Development Services (ICDs) was
so that they remain viable in the face of competition forum. introduced in 1975. The scheme is a government initiative for
(iii) rules aligning with other developing countries for the all round development, ie., health, nutrition and educa-
better trade terms. tion of children under 6.
(iv) MNCs negotiating with WTO for fairer rules. 51. (1) Globalisation was not stimulated by money.
39. (3) The purpose of Pradhan Mantri Mudra Yojna is to 52. (4) Food security means availability, accessibility and
provide loan for small businessmen from 50000 to 10 lakhs. affordability of food to all at all the time.
40. (2) Option 2 is the correct answer. National income
The private sector is the part of the economy referred to 53. (3) Per capita income is calculated by
as the citizen sector, which is owned by private individuals Total Population
or groups, usually as a means of enterprise for profit, which 54. (3) Bombay stock exchange is the tenth largest stock exchange
is not owned by central or state government. in the world and oldest stock exchange in South Asia.
41. (2) Production of Milk: White Revolution is associated 55. (4) All of the above statements are correct.
with an increase in the milk production. The White 56. (1) International Co-operative Day is celebrated on the
Revolution is also known as Operation Flood which was first Saturday of July. Celebrations of the day are held at
launched in 1970s to make India self dependent in milk international, national and local levels.
production. Dr. Verghese Kurien is known as the father of 57. (3) To prefer sustainability of development
the White Revolution in India. 58. (4) fiscal policy
42. (3) 1st January, 1995: The World Trade Organization 59. (3) The marginal productivity of the disguised unemployed
(WTO) is an international organization. It is concerned is zero. It is unemployment that does not affect aggregate
with the regulation of international trade between nations. output.
The WTO officially commenced on 1 January, 1995 under 60. (3) Indicators involved in HDI are life expectancy, literacy
the Marrakesh Agreement, signed by 123 nations on 15 attainment and purchasing power of people.
April, 1994, replacing the General Agreement on Tariffs 61. (1) The group which classifies the above aspects are
and Trade (GATT). It came into existence in 1948. It is the narrow and broad money.
largest international economic organization in the world. 62. (1) Removing barriers or restrictions set by the govern-
43. (4) High Level of Public expenditure In India, the reason ment is called liberalisation.
of increasing inflation is the expenditure on public welfare. 63. (1) 64. (1) 65. (4)
The other causes are - Demand-pull and Cost-push. They 66. (2) Green Net National income is the diffrenet between
are responsible for a general rise in prices in an economy. net national income and depreciation of natural capital.
But they work differently. Demand-pull conditions occur 67. (1) In India, the Reserve Bank of India issues currency notes
when demand from consumers pulls prices up. Cost-push on behalf of the central government. As per Indian law no
occurs when supply cost force prices higher. other individual or organisation is allowed to issue currency.
44. (4) India’s first petro-chemical industry is ‘The National 68. (3) 69. (4)
Organic Chemicals Industries Limited (NOCIL)’. It was 70. (1) The Righ to Information Act (2005) was passed by the
established in private sector in 1961 in Mumbai. The plants Government of India. The Act ensures its citizens all the
located at Mumbai, Barauni, Mettur, Pimpri and Rishra are information about the functions of government department.
the major producers of plastic materials. 71. (4) The multinational companies owns or control produc-
45. (2) Diamond Quadrilateral is related to High Speed tion in more than one nation.
Railways. At present there are six corridors on Diamond In India the big companies like Infosys, Tata motors, Asian
Quadrilateral connecting metropolitan cities and growth paints etc., are the companies which are spreading their
centers of the country have been identified for feasibility operations worldwide.
72. (1)
EBD_7332
196
B- NTSE Stage 1 Question Bank

73. (1) To control situation of deflation it is essential to 106. (3) The Indian Constitution provides for a Finance
increase the demand for bank loan. Commission to advice the President on distribution of
74. (1) Economic rent is the price paid for the use of land or financial resources between the union and the states.
natural resources inhose supply is perfectly inalastic. 107. (2) G.B.P.A.U. Pant Nagar is the first Agricultural University
75. (1) Money is not the factor of production because it is a of India established in 1960.
resource used to acquire resources that go into producing 108. (1) Jim Yong Kim, a South Korean, is the current President
goods. of the World Bank.
76. (3) 77. (1) 78. (3) 79. (3) 109. (4) NDC was set up on 6th August, 1952 is the apex body
80. (4) Proximate cause is a key principle of insurance and is for decision making and deliberations on developmental
concerned with how the loss or damage actually occured matters in India.
and whether it is indeed as a result of an insured peril. 110. (3) Creation of Utility for exchange is called production.
81. (1) Naresh Goyal is an Indian businessman and founder 111. (2) Rise in population size will increase the demand for a
chairman of Jet Airways. He started operating Jet Airways necessary commodity.
in 1993. Kiran Mazumdar Shaw an Indian billionaire 112. (3) Inflation indicates a situation of continuous rise in prices.
entreprenuer. She is the chairperson and managing director 113. (2) An economy is said to be on the path of growth when
of Biocon Ltd, a biotechnology company based in Banglore. in equality in the distribution of income falls.
Narayan Murthy, is an Indian IT industrialist and the co- 114. (4) RBI is known as the Lender of Last Resort because,
founder of Indosys Dhirubhai Ambani, a business tycoon banks are supposed to meet their short falls of cash from
who founded Reliance industries in Bombay. other sources and if the other sources don't meet demand,
82. (1) 83. (1) 84. (4) 85. (2) then they approach RBI.
86. (3) 87. (2) 88. (2) 115. (3) M. S. Swaminathan is known as the father of Green
89. (3) Under Antyoday Anna Yojana the wheat is supplied at Revolution in India for his contributions to the development
the rate of `2 per kg and `3 per kg for rice. of high-yielding varieties of wheat.
90. (3) 91. (3) 92. (4) 116. (4) Entertainment Tax is an example of Indirect taxes. Which
93. (2) Ford motors, an American company, is one of the are not directly levied on the income of an individual but is
world’s largest automobile manufacturers. indirectly levied on the expense incurred by the individual.
94. (2) 117. (1) The Academy of development Science supports food
95. (4) All the statements given about Annapurna Scheme are security programme in Maharashtra by setting up Grains
correct. Banks.
96. (3) A. D is parities in income in a developing economy — 118. (2) The 6th summit of BRICS hosted by Brazil was held in
Less redistribution of income in favour of poor. July 2014.
B. Economic development benefits the poor-Trickle down 119. (4) COPRA is the Consumer Protection Act 1986 to protect
theory interests of consumers in India.
C. Shifting of labour from agricultural to non agricultural 120. (3)
sector. 121. (2) GDP is known as the Gross Domestic Production which
Change in occupational structure implies the total value of goods produced and services
D. Increase in the capability of people–Human development provided in a country during a year.
1 1 1 122. (3) Litracy rate fo Rajasthan is 67.07% according to 2011.
97. (3) HDI (LEI) (EI) (GDP Index) 123. (1) The economic liberalisation in India refers to on going
3 3 3 economic reforms in India that started on 24 July, 1991.
1 1 124. (4) Intensive cultivation enhanced by excessive use of
0.53 0.67 0.42 1.62 0.54 chemical fertilizers has caused land degradation in Punjab.
3 3
98. (2) Cash reserve ratio = 20% Also irrational use of surface and ground water for irrigation.
Banking system amount = 50 million 125. (2) SAIL is a public sector company and is the largest
Maximum amount of demand deposit = integrated iron and steel producer in India. It is responsible
for the marketing of steel for the public sectors in India.
1 126. (3) Dairy farming is a non farm activity. Dairy farming is a
Banking system amount
Re serve ratio class of enterprise, for long-term production of milk, usually
from dairy cows but also from goats, sheep and camels,
1 5 which may be either processed on-site or transported to a
= 50 million 100 = 250 million
20 dairy factory for processing and eventual retail sale.
99. (3) The Govt of India supplies food grain eg. rice, wheat 127. (4) UNDP prepares annual publication 'Human Development
sugar, etc. and other essential commodities to below poverty Report'. The report was first launched in 1990 by the Pakistani
line house holds through Public distribution system. PDS is economist Mahbub ul Haq and Indian Nobel laureate Amartya
an Indian food security system. Sen. Its goal was to place people at the center of the development
100. (3) 'Renaissance' word is used for the phase in European process in terms of economic debate, policy and advocacy.
history for many changes between 14th and16th centuries 128. (2) The National Rural Employment Guarantee Act was
in the field of classical art and intellect. enacted by legislation on August 25th 2005. The Act
101. (2) 102. (1) 103. (3) 104. (1) provides 100 days assured employment every year to every
105. (1) GDP is the logical extension of measuring economic rural household in 200 districts.
growth in terms of monetary expenditures 129. (2) The Reserve Bank of India was set up on the basis of
OR the recommendations of the Hilton Young Commission. The
Economic growth is the increase in the market value of goods Reserve Bank of India Act, 1934 (II of 1934) provides the
and services produced by an economy over time. It is statutory basis of the functioning of the Bank, which
measured as the percent rate of increase in real GDP. commenced operations on April 1, 1935.
Solved Questions B-197
130. (1) In 1993 Punjab National Bank acquired New Bank of of the common examples for this crop are rubber, tea, coffee
India, which the Government of India had nationalized in 1980. and sugarcane. This method is best known as the meeting
131. (1) Globalization produces consumerism. People being point for agriculture and industry.
attracted by attractive goods and advertisements, want to 3. (2) Neyveli lignite mines are located in the Cuddalore
buy these goods. They would not hesitate to earn money district state of Tamil Nadu. These mines come under the
for this by unfair means. This has resulted in vast increase public sector. These mines own reserves of about 34,764
in corruption and other social evils. million tonnes. Lignite is low-grade brown coal. It has a
132. (4) Population density is the number of people living in a high content of moisture and therefore has a soft texture.
unit area of the earth's surface. It is normally expressed as 4. (4) The longest earthen dam in the world is the Hirakud
per square km. The average density of population in the dam. This dam is situated don the great river, Mahanadi in
whole world is 51 persons per square km. the Sambalpur region of Odisha about 15km. Post-
133. (1) ‘Jago Grahak Jago’ slogan is a awareness slogan for Independence, it was the first major multipurpose river
consumers. valley project in India. This dam aimed to control flood,
134. (4) If a person does not contribute anything in the supply water for irrigation purpose and generate electricity.
production process or in other words, if he can be removed 5. (2) In 1904, the first cement plant was set up in India in
from the work without affecting the productivity adversely, Chennai. This plant was set up in Porbandar, Gujarat but
he will be treated as disguisedly unemployed. The marginal the production in the plant was started in Chennai. It was
productivity of such unemployed person is zero. Agriculture set up by India Cement Company Limited. The capacity of
sectors of underdeveloped/developing economics posses the plant was 10,000 tonnes.
this type of unemployment at a large scale. 6. (3) Ernst Friedrich Schumacher was born on August 16,
135. (2) Human Development Index (HDI) is prepared on the 1911 in Bonn, Germany. He died on September 4, 1977, in
basis of 3 basic indicators health, education and income. Romont, Switzerland.He was a German-born British
136. (1) When market supply remains unchanged and demand economist who developed the concepts of intermediate
increases, then equilibrium price increases. technology. He wrote the book, Small is Beautiful in 1974.
137. (4) The Consumer Protection Act, 1986 ensures right to The book was based upon Gandhian philosophy.
choice, right to information, right to redressal as rights which 7. (2) Industries can be classified into two categories on the
every consumer in India should posses. basis of their main role, namely key and consumer
138. (2) Land, labour, capital and entrepreneur, all factors industries. The industries that supply their product or raw
contribute in production process. So they get rewards in materials to manufacture other goods are known as basic
the form of rent, wages, interest and profit respectively. Micro industries, for example, iron and steel, copper smelting and
economics deals with determination of such rewards i.e. aluminium smelting. The industries that sell their products
factor prices. So micro economics is also called as 'Price to the consumer directly are known as consumer industries,
Theory' or 'Value Theory'. for example, automobile.
139. (3) It is economic indicator not a social and demographic 8. (4) Non-renewable energy comes from sources that will
indicator. run out or will not be replenished for thousands or even
140. (2) A government levy on the income, property, or wealth millions of years. Most sources of non-renewable energy
of people or companies. A direct tax is borne entirely by the are fossil fuels. ... Most non-renewable energy sources are
entity that pays it, and cannot be passed on to another fossil fuels, such as coal, petroleum, and natural gas.
entity. The rest all are indirect taxes. 9. (3) McAdam devised the method of creating pakka roads.
141. (2) Increase in exports does not cause any increase in This method was invented by John Loudon McAdam in
demands of goods and services. 1820 for the construction of roads. In this method, layers of
142. (4) Inflated currency- currency which is too high in relation single-sized crushed stone are placed in shallow lifts and
to other currencies. compacted thoroughly. In 1829, Robert Stephenson
143. (1) Expansion of international trade; Increase of world production developed a steam locomotive named rocket. It was the
by ensuring full employment in the participating nations; only locomotive that completed the trials with a maximum
Development and full utilisation of world resources; and Raising speed of 30 mph.
standard of living of the world community as a whole. 10. (4) Mediterranean vegetation includes trees with thick
144. (4) Co-operative farming a voluntary organisation in which barks and wax-coated leaves. These trees help them reduce
the farmers pool resources in order to carry out the various transpiration. This adaptation helps them in their survival,
agricultural operations with each other’s for the protection especially in dry summer. These trees are known as the
of their common economic cause and the adoption of new orchards of the world for their fruit cultivation. It consists
skills and techniques. of trees with a height of 2.5 m or 8 feet.
145. (2) The purpose of devaluation of Indian rupee is to 11. (2) In the 2018/2019 crop year, around 148.5 million metric
increase imports and to reduce exports. tons of milled rice was produced by China. This amount is a
higher volume than any other country. India produced 116.42
SECTION 8. GEOGRAPHY
million metric tons of milled rice in that crop year and became
1. (4) Coffee was brought to India before the existence of the second-largest producer in the world. India accounts for
the East India Company. In India, the coffee plant was 25% of global production is the largest producer in the world.
introduced in 1670. An IndianSufi saint, Baba Budan Its consumption accounts for 27% of world consumption
broughtcoffee beans to India from Yemen during the later and 14 % of the total import of pulses in the world.
17th century. In India,coffee cultivation wasintroduced to 12. (4) The seawater is diluted due to the addition of
the hills of Chikmagalur of Karnataka. freshwater from many rivers to it. It reduces the content of
2. (2) A type of commercial farming in which a single crop is salt and therefore reduces the density of the seawater. The
grown for the entire year is known as plantation agriculture. Baltic Sea is almost enclosed by northern Europe and
It is both intensive and capital intensive agriculture. Some Scandinavia. The sea has a very low salinity of about 10 ppt.
EBD_7332
198
B- NTSE Stage 1 Question Bank

13. (3) Coromandel Coast is situated on the coastal plain in temperate crop, barley is also grown in many tropical countries,
eastern Tamil Nadu state. It spreads over an area of about typically by poor farmers in hostile, dry, cool environments.
8,800 square miles The Northern Circars is situated on the 27. (1) Chittorgarh is the cement city of Rajasthan.
western coast of the Bay of Bengal in the state of Andhra 28. (3) Literacy in India is a key for socio-economic progress.
Pradesh. Malabar is situated on the coast of the Arabian Despite government programmes, India’s literacy rate
Sea in Kerala. Canara is situated on the coast of the Arabian increased only “sluggishly”. The 2011 census, indicated a
Sea, western Karnataka state, India 2001–2011 decadal literacy growth of 9.2%, which is slower
14. (1) The south-west monsoon winds are not very strong and than the growth seen during the previous decade
therefore start withdrawing gradually. By the onset of October, 29. (2) Cobalt is a ferrous metallic mineral. The metallic
the monsoon withdraws from the northern plains. This minerals that contain iron are known as ferrous minerals. In
withdrawal of the monsoon is shown by clear skies and an the total production of metallic minerals, 75% is constituted
increase in temperature. On the other hand, Tamil Nadu receives by ferrous metallic minerals. Its common examples include
scanty rainfall as it is in the rain shadow area of the Arabian iron ore, manganese, chromite and nickel
sea branch and is parallel to the Bay of Bengal branch. 30. (4) The people in the region depend on rainwater
15. (1) The leading world’s jute producing countries are India, harvesting. The harvested rainwater from the monsoon
Bangladesh, China and Thailand. India is the world’s largest season (during July and August) is stored in pucca tanks
producer of raw jute and jute goods, contributing to over and used throughout the year for drinking purposes.
50 percent and 40 percent respectively of global production. 31. (1) The given quote was given by Smt. Amrita Devi. This
The first cement plant was set up in the year 1904, in quote was given to save trees. She fought against tree cutting.
Chennai (Tamil Nadu). 32. (1) The Green Revolution in India started in the late 1960s
16. (4) Infant mortality rate (IMR) is the number of children and with its success, India attained food self-sufficiency
who die within one year, out of 1000 live children born within a decade. In India, it was first introduced in Punjab in
under one year of age. The literacy rate is defined as the the late 1960s as part of a development program issued by
percentage of the population belongs to the age group of 7 international donor agencies and the Government of India.
years or above. It includes the population that can both During the British Raj, India’s grain economy hinged on a
read and write. They should also be able to comprehend a unilateral relation of exploitation.
short simple statement he/she uses regularly. 33. (2) Bharatpur Bird sanctuary is also known as Keoladeo
17. (1) Gujarat and Maharashtra have the largest number of National Park. It is situated in Bharatpur, Rajasthan. It is a
cotton textile centres. Although cotton textile mills are man-made sanctuary. In the sanctuary, more than350 species
located in over 80 towns and cities of India, yet its larger of birds can be found including herons, cormorants and eagles.
concentration is found in Maharashtra, Gujarat, West Here, the birds can be seen mostly in the winter season.
Bengal and Uttar Pradesh. 34. (3) In India, the literacy rate is 74.04 per cent, 82.14 for
18. (3) The northern plains are suitable for constructing males and 65.46 for females. The state of Kerala has the highest
railway lines. The plains give the platform for the tracks. literacy rate, 93.91%. After this, the literacy rate in Laksha
19. (?) Greenwich Mean Time (GMT) is the mean solar time at the dweep is 92.28 per cent and in Mizoram, it is 91.58 per cent.
Royal Observatory in Greenwich, London which is measured at 35. (2) Rajgir ropeway is the oldest ropeway situated in the
midnight. In English speaking nations, GMT is often used state of Bihar, India. It is a ropeway that takes towards the
synonymously with Universal Time Coordinate (UTC). hills of Ratnagiri where the famous Vishwa Shanti Stupa is
20. (3) The city has a Mediterranean climate, with rainy situated, also known as the Peace Pagoda. It goes to a
winters and dry summers. height of over 1000 feet from the ground level.
21. (4) Tropic of Cancer is an imaginary line, at an angle of 36. (4) The Kosi River is known as the “Sorrow of Bihar”.The
23.50 degrees North from the Equator, that passes through river got its name since it is affected a huge area of
the middle of India agricultural land, 21,000 km2 since it ia flood-prone area.
22. (1) Kolleru is located between Krishna and Godavari This leads to the disturbance in the rural economy. The
deltas. Kolleru spans into two districts - Krishna and West headquarters of Maruti Industries situated in Delhi, India.
God kavari. The lake is fed directly by water from the Varanasi is located on the left bank of the GangaRiver and
seasonal Budameru and Tammileru streams and is is one of the seven sacred cities of Hinduism.
connected to the Krishna and Godavari irrigation systems 37. (1) Patna airport is also known as Jay Prakash Narayan
by over 67 major and minor irrigation canals. Airport. It is situated in Patna, the state capital of Bihar in
23. (2) The Diamond Quadrilateral is a project of the Indian India. It is a custom airport. It is named after independence
railways to establish a high-speed rail network in India. activist, Jayaprakash Narayan. It has a short runway and
The Diamond Quadrilateral will connect the four mega cities therefore it is known as a restricted international airport.
in India, Delhi, Mumbai, Kolkata and Chennai, similar to 38. (2) Medha Patkar with other is allies of NAPM were the
the Golden Quadrilateral expressway system. founder of Narmada Bachao Andolan (NBA) and Ghar
24. (2) The colour brown is used to denote most contour lines Bachao Ghar Banao Andolan. She started NBA in three states,
on a map, which are relief features and elevations. Madhya Pradesh, Maharashtra and Gujarat. Mangrove forest
Topographic maps use green to denote vegetation such as is found in the coastal area in India. A type of farming in
woods, while blue is used to denote water features like which only one crop is grown is known as plantation
lakes, swamps, rivers, and drainage agriculture. It is also a type of commercial farming.
25. (2) Rajsamand Lake is a lake near the town of Rajsamand in 39. (3) A national highway network that connects most of
the Indian state of Rajasthan. Builtin 1660 by Rana Raj Singh, the major industrial, agricultural and cultural centres of
it is approximately 1.75 miles wide, 4 miles long and 60 feet India is known as the Golden Quadrilateral Super highway.
deep. It was built across the Gomati, Kelwa, and Tali rivers. It forms a quadrilateral connecting the four major metro cities
26. (3) Barley is the fourth most important cereal crop in the of India, i.e., Delhi (north), Kolkata (East), Mumbai (west)
world after wheat, maize, and rice. Although generally a and Chennai (south).
Solved Questions B-199
40. (3) Black soil is also called Regur soil. It is black in colour 53. (2) India’s and Asia’s first nuclear reactor was the Apsara
and ideal for growing cotton. This type of soil is typical of research reactor at Mumbai. The domestic uranium reserve in
the Deccan trap (Basalt) region spread over the North-West India is small and the country is dependent on uranium imports
Deccan plateau and is made up of lava flows. from other countries to provide fuel to its nuclear power industry.
41. (4) The Nile River, located in Africa, is listed as being 6,853 54. (1) Jaisalmer is called the driest place in India because,
kilometres (4,258 miles) long and is hence commonly considered considering the past weather records, this district receives
to be the longest river in the world. On the other hand, The the lowest annual rainfall in India. Ruyli located in the
Amazon is considered the world’s largest river by volume, Jaisalmer district get the least amount of rainfall – 8.3 cms
42. (2) Toda people are a Dravidian ethnic group who live in 55. (3) As per the National Highways Authority of India, about
the Nilgiri Mountains of the Indian state of Tamil Nadu. Their 65% of freight and 80% of passenger traffic is carried by the roads.
economy was pastoral, based on the buffalo, whose dairy products 56. (2) The Brazilian coast with its 8000 km reaches from north
they traded with neighbouring peoples of the Nilgiri Hills. of the equator to the far south, the border to Argentina. A
43. (1) In 1992, more than 100 heads of state met in Rio de Janeiro, big part is in the tropic regions, the southeast is sub-tropical
Brazil for the first international Earth Summit convened to (with Rio and Sao Paulo) and further in the south the climate
address urgent problems of environmental protection and is about as in the Mediterranean
socio-economic development. 57. (1) Pakistan is not a major trading partner of Brazil. The
44. (2) MGNREGA was implemented in 200 districts in the first major countries are the US, China, India, Canada, Argentina
phase with effect from February 2006 and extended and Nether land. Various products that Brazil import are
subsequently, to additional 113 and 17 districts with iron ore, crude petroleum, soybeans and raw sugar.
effect from April 1st 2007 and May 15th 2007, respectively. 58. (3) Brazil ishome toone ofthe most popular ecotourism destinations
The remaining districts were included under the Act with on Earth: the Amazon. The river itself and the surrounding
effect from April 1st, 2008. jungle are a major draw for thousands of tourists every year.
45. (3) The Tea Board of India is a state agency of Everything from structured day hikes from jungle lodges
the Government of India established to promote the cultivation, to extended treks is very popular.
processing, and domestic trade as well as export of tea from 59. (2) India is a vast country. Lying entirely in the
India. It was established by the enactment of the Tea Act in Northern hemisphere (Figure 1.1) the mainland extends
1953 with its headquarters in Kolkata (formerly Calcutta). between latitudes 8°4’N and 37°6’N and longitudes 68°7’E
46. (3) Westerlies wind is known with different names such and 97°25’E.
as“roaring forties” at 400 ‘furious fifties’ at 500 and ‘screaming 60. (3) Goa is the most urbanized state in India. Among the
sixties’. These winds blow in the Southern Hemisphere. Their States, Goa is now the most urbanised State with a 62.2
names are given based on the latitudes on which they occur percent urban population, a significant increase since 2001
in the Southern Hemisphere and their intensity. when the urban population of Goa was 49.8%. Another
47. (3) The city, Pulicat is located in the district of Tiruvallur significant instance of rapid urbanisation is that of Kerala,
in the state of Tamil Nadu. It is situated almost 60kms north its urban population is now 47.7 per cent, while a decade
of Chennai city particularly on the seaward side of Sri ago it was just 25.9 percent.
Harikota island blockade that segregates Pulicat Lake from 61. (4) Bhima River, a major tributary of the Krishna River,
the Bay of Bengal. It is a saltwater lagoon. flowing through Maharashtra and Karnataka states,
48. (1) Tehri Dam is situated on the Bhagirathi river. Hirakud western India. It rises in the Bhimashankar heights of
Dam is situated on the great river, Mahanadi. Nagarjuna the Western Ghats and flows southeastward for 450 miles
Sagar Dam is situated on the Krishna river. Indira Gandhi (725 km) in Maharashtra to join the Krishna in Karnataka.
Canal is situated on the river Sutlej. 62. (2) Sri Lanka gave the name Gaja to the cyclone. It came
49. (4) Kachchi is not a part of the world network biosphere on November 11 and was the deep depression that
reserves of UNESCO. The World Network of Biosphere strengthened into a cyclonic storm.
Reserves of the MAB Programme consists of a dynamic 63. (2)
and interactive network of sites of excellence. 157 sites in A. Nhava Sheva is situated in Maharashtra.
24 countries in Asia and the Pacific. 302 sites in 38 countries B. Tuticorin is situated in Tamil Nadu.
in Europe and North America. 130 sites in 21 countries in C. Paradip is situated in Odisha.
Latin America and the Caribbean. D. Haldia is situated in West Bengal.
50. (3) Print Media is the most common but powerful means 64. (1) When granite is subjected to intense heat and pressure,
of communication. It creates credibility. Investing in printed it changes into a metamorphic rock called gneiss. It has a
material signals to customers that you are serious about more sandy and less clayey composition. It does not retain
your business and that you offer a worthwhile product or moisture. Total area covered in India about 5.2 lakh sq. km.
service. Printed materials can also create engagement 65. (3) A Blue Sheep is a vulnerable species. Wild Asiatic
between the customer and the brand. buffalo is a rare species. The Nicobar region is an endemic
51. (2) Hurricane Committee determines a pre-designated list species. Asiatic cheetah is an extinct species.
of Hurricane names southern India, on 16. Cyclone 66. (2) Bhakra Nangal project is a joint venture of the states
Gaja is named Sri Lanka and Cyclone title was named of Punjab, Haryana and Rajasthan. It is situated on the
Gaja which means elephant in language!, 2020 each country river Sutlej. It was a multipurpose project. It produces
the deep depression strengthened into a cyclonic storm hydroelectric power.
52. (1) The full form of HCR is Head Count Ratio. The head 67. (4) A national highway network that connects most of the
count ratio (HCR) is the population proportion that exists, or major industrial, agricultural and cultural centres of India is
lives, below the poverty threshold. One of the undesirable known as the Golden Quadrilateral Superhighway. It forms
features of the head count ratio is that it ignores the depth a quadrilateral connecting the four major metro cities of
of poverty; if the poor become poorer, the head count index India, i.e., Delhi (north), Kolkata (East), Mumbai (west) and
does not change. Chennai (south).
EBD_7332
200
B- NTSE Stage 1 Question Bank

68. (2) Folding mountain: Himalayas are a type of fold Bharatpura - Rajasthan
mountains which are formed due to collision of tectonic Manas - Assam
plates. When two tectonic plates collide they get Dandeli - Karnataka
compressed resulting in upliftment of Earth’s crust. The 80. (1) Related to Laterite soil, option 1 is the right answer.
process through which Fold Mountains are created is Laterite soil is not suitable for agriculture because of its
called orogeny. high content of acidity and it cannot retain moisture.
69. (3) Sundar Ban: The forests in the Ganga-Brahmaputra 81. (2) On Multipurpose river valley project, option 2 is the
delta are known as Sunderbans. Sunderbans are named correct one remaining all mismatched.
after the ‘sundari’ trees that are found there in large a - Hirakund (Odisha)
amount. It is also named as Ganges Delta, or Bengal Delta. b - Bhakharanangal (Punjab)
70. (2) 75: Uttar Pradesh is divided into 75 Districts and 18 c - Kosi (Bihar)
Divisions. The district is further divided into sub-divisions; d - Nagarjuna Sagar (Andhra Pradesh)
Sub-divisions are divided into blocks, which comprise 82. (1) It is picture identification question in that option 1 is
Panchayats and town Municipalities. Correct, it is also called Mangrove or Tidal forest.
71. (2) Africa: The Sahara Desert is located in the northern A mangrove is a shrub or small tree that grows in coastal
part of Africa. The desert covers nearly 10% of the African saline or brackish water. Mangroves are salt-tolerant trees,
continent. It has a surface area of approximately 3, 320, called as halophytes, and are adapted to life in harsh
000 square miles (8.6 million square kilometers) making it coastal conditions. They contain a complex salt filtration
the world’s largest hot desert with one of the harshest system and complex root system to cope with salt water
environments on earth. immersion and wave action.
72. (3) Coffee is grown in three regions of India— Karnataka, 83. (2) The central part of Brazil is sparsely populated
Kerala and Tamil Nadu in well-drained soil conditions with because it’s dense rain forest. Rain forests aren’t a very
rich organic matter. Ideal climatic conditions to grow coffee friendly environment for human habitation, showing lack
are related to temperature and rainfall; temperatures in the of fresh water, dangerous animals, lack of solid ground,
range of 23°C and 28°C with rainfall incidence in the range lack of dry shelter, limited fuel and food sources, and
of 1.5–2.0 m. Coffee is cultivated under a well-defined two- difficult transportation.
tier mixed shade canopy. 84. (3) The Sindhu river (Indus) rises in the south-western
73. (2) Haryana state has both wet and arid regions so the Tibet at an elevation of about 18,000 feet. For about 320
rain-fed and irrigated agriculture is practiced by the farmers. km, it flows north-west, crossing the south- eastern of the
Consequently, many suitable agro-forestry models have Kashmir region at about 15,000 feet. The Chenab , Sutluj
been developed by the forest department, while some other and Ravi are tributary rivers for it . The Betwa is a river in
have been evolved by farmers through practical method. Northern India, and a tributary of the Yamuna. It rises
74. (2) Shimoga (Karnataka) mines is famous for gold ores. from Hoshangabad in Madhya Pradesh.
It lies on the banks of the Tunga River. 85. (2) The north-east part of Brazil is the driest part of the
75. (2) The period of June to September is referred to as the country. This region also constitutes the hottest part of Brazil,
‘Southwest Monsoon’ period. The whole country receives where during the dry season between May and November,
nearly 75% of its rainfall during this period. Southwest Monsoon temperatures of more than 38 °C have been recorded. Most
period is the principal rainy season for the Indian subcontinent. of the centeral-west has 1,500 to 2,000 mm of rain per year.
This is the summer monsoon period where the southwest 86. (4) The Sutluj River is the eastern most tributary of the
monsoon holds away over the country. The north-east River. The source of the Sutluj is the west of catchment
monsoon, commonly known as winter monsoon blows from area of the Lake Rakshastal in Tibet. The river flows north-
land to sea and it gives the state 48% of its annual rainfall. west for about 260 km.
76. (2) Salt Water lakes and lagoons are different in category 87. (1) The most sparse area is :
but almost same in nature. Sambhar lake of Rajasthan is Mountainous hilly regions-dry desert – dense forest.
India’s largest inland salt water lake and Chilka Lake is Mountainous hilly, dry desert and dense forest area are
the India’s largest brackish water saline lake. Pulicat Lake is such type of regions where population is not dense due
a brackish saline water lagoon at the border of Andhra to unavailability of basic needs for development and human
Pradesh and Tamil Nadu. habitation.
Dal Lake is a fresh water lake situated in Jammu & 88. (2) Fold Mountains are formed as a result of the
Kashmir, India. compression of tectonic plates leading to the formation of
77. (2) Kudremukh is an iron ore mine situated at Chikkamagaluru large fold-like structures on the earth’s crust. Fold
district in Karnataka. The Kudremukh mine, one of the largest Mountains are found all over the world with most of the
iron ore mines in the world and was closed in 2006. world’s tallest peaks being fold mountains. The Himalayas
78. (3) Sea beds are rich in clay particles and micro skeletons of are example of Fold Mountains.
marine organisms, minerals, such as manganese nodules. The 89. (3) Rio de Janeiro, Paraná, São Paulo, Bahia are the states
sea bed, also known as the ocean floor, is composed of different of Brazil lying in coastal areas whereas Goias is situated
minerals than what is found in the uppermost section of earth’s inside land areas.
surface. The ocean floor itself is made of mafic rocks, the 90. (1) In South America, neighbouring countries lying from
crystallized matter from silicate magma. Minerals found under south to north direction are as follows:
the seabed include gabbro, basalt, serpentine, peridotite, olivine Uruguay – Argentina – Bolivia – Peru
and ore minerals from Volcanic Massive Sulfide. 91. (1) The condor is a bird found in Brazil (South America).
79. (3) It is concept based question in Geography, option 3 It is the largest flying bird in the world by combined
is the right answer. measurement of weight and wingspan. It has a maximum
Nagarjuna sagar- Telangana wingspan of 3.3 m.
Solved Questions B-201
92. (4) Western Rajasthan is the driest region in India. 106. (4) Igneous rocks have denuded overtime and are
Jaisalmer in western Rajasthan is the driest place which responsible for the formation of black soil. The new alluvial
receives the lowest rainfall (less than 10 cm). It is due to deposits of the northern plain are called khadar. They are
its distance from the monsoon winds of the Bay of Bengal renewed almost every year and are fertile.
and location in the sub-tropical high-pressure belt. 107. (3) The countries like Egypt, Mongolia and Uzbekistan
93. (2) Rabi Crops : The crops grown in the winter season are smaller than India with respect to area. Whereas Canda,
(October to March) are called Rabi crops. Examples of rabi USA and Brazil are bigger than India in terms of area.
crops are wheat, gram, pea, mustard and linseed. 108. (4) The Godavari is the largest peninsular river. The
94. (4) Khetri-Singhana: Khetri is situated at the foothills Godavari is joined by number of tributaries such as the
of the Aravalli Range, which possesses copper mines, Purna, the Pranhita, the Manjra, the Penganga, the Wenganga
giving rise to a 80 km long metallogenetic zone from and the Wardha. Godavari is also known as Dakshin Ganga
Singhana in the north to Raghunathgarh in the south, because of its length and the area it covers.
popularly known as Khetri Copper Belt. 109. (4) The state of Gujarat receives South-West monsoon
95. (1) lately.
(1) Durgapur (iv) West Bengal 110. (2) The per capita consumption of electricity is consid-
(2) Rourkela (iii) Orissa ered as an index of development.
(3) Bhilai (ii) Chhattisgarh 111. (4) Hills from west to east are Garo, Khasi, Jantia and Naga.
(4) Bokaro (i) Jharkhand 112. (3) The East-West corridor connects Silchar (Assam) and
96. (1) Karnataka: New Mangalore Port is a deep-water, all- Porbandar (Gujarat). The highway project are implemented
weather port at Panambur, Mangalore in Karnataka. It is by the National Highway Authority of India (NHAI).
the deepest inner harbour on the west coast. It is the only 113. (3) El-Nino is a Spanish word meaning child . The pres-
major port of Karnataka and is the seventh largest port in ence of El-Nino leads to increase in the sea-surface
India. This port is operated by New Mangalore Port Trust. temperatures and weakening of the trade winds in the re-
97. (1) Tapi River (length of 724 km) is one of only three rivers gion. The name is given to warm ocean currents along the
in peninsular India that run from east to west. The others coast of Peru as a temporary replacement of the cold Peru-
are the Narmada River and the Mahi River. The river rises in vian current.
the eastern Satpura Range of Madhya Pradesh, and flows 114. (3) During this season, the north-east trade winds prevail
towards west and falls in the Arabian sea. over the country. They blow from land to sea and hence for
98. (2) Limestone is an example of Sedimentary rocks. It is most parts of the country, it is a dry season. Some amounts
composed primarily of calcium carbonate (CaCO3) in the of rainfall occurs on the Tamil Nadu coasts from these winds
form of the mineral calcite. As a sedimentary rock, many as here they blow from sea to land.
different variations of limestone occur in nature because 115. (3) Red soil develops a reddish colour because of diffu-
of the different conditions that produce the rock. It takes sion of iron in crystalline and metamorphic rocks. It looks
millions of years for limestone to form from shell, sand yellow when it occurs in a hydrated form.
and mud deposits left by lakes and oceans. Some limestone 116. (3) A mushroom rock is a landform caused by the action
contains visible fossil fragments along with calcite and of wind erosion.
other aragonite materials. 117. (2) Hanging valleys are carved out by the action of glaciers.
99. (4) The major right-bank tributaries include Yamuna, Son, 118. (1) The cold weather season begins from mid November
Punpun and Damodar Rivers while the major left-bank in northern India and stays till February. During this season,
tributaries include Gomti, Ghaghara, Gandaki, and the north east trade winds prevail over the country. The
Kosi Rivers. The hydrology of the Ganges River is very weather is normally marked by clear sky, low temperature
complicated, especially in the Ganges Delta region. and low humidity and feeble variable winds.
100. (1) There is a short season between Kharif and Rabi 119. (2) Bauxite deposits are formed by a wide variety of rocks
season in the months of March to July, the crops that rich in aluminium silicates. Odisha is the largest bauxite pro-
grow in this season are Zaid crops. These crops are grown ducing state in India.
on irrigated lands and do not have to wait for monsoons. 120. (1) Pipline transport network is a new arrival on the
The crops grown in these seasons are related to the transportation map of India. Earlier these were used to trans-
availability of water and the amount of daylight. Some port water to cities. Now, these are used for transporting
examples of the Zaid crops include cucumber, watermelon, crude oil, petroleum products and natural gas.
muskmelon, bitter gourd, pumpkin, ridged gourd. 121. (1) Kolkata is an inland riverine port. This port serves a
101. (4) Mica is a mineral made up of series of plates or leaves. very large and rich hinterland of Ganga-Brahmaputra basin.
Mica can be clear, black, green, red, yellow or brown. Due to 122. (4) Yamunotri–glacier Armakonda-Peak, Kulu-valley,
its excellent di-electric strength, low power loss factor, Ranikhet-hill station.
imulating properties and resistance to high voltage, mica is 123. (1) The Himalayan yew is an endangered native high value
one of the most important minerals used in electric and medicinal plant of the Himalayan region. The several me-
electronic industries. dicinal properties of the bark and leaves of this species
102. (1) Lake Victoria lies on the equator. have increased its risk of extinction.
103. (4) The longitudinal valley lying between lesser Himalayas 124. (4) Laterite soil has been derived from the Latin word ‘later’
and the Shiwaliks are known as Duns. Dehra Dun, Kotli which means brick. Humus content of the soil is low be-
Dun and Patli Dun are few well known Duns. cause most of the microorganisms particularly, the decom-
104. (3) In winters, the western cyclonic disturbances posers like bacteria get destroyed due to high temperature.
originates from the Mediterranean Sea. It is very useful for growing tea and coffee.
105. (3) Sustainable Development is development that meets 125. (2) Black soil is ideal for growing cotton. This type of soil
the need of the present without comprising the ability of is typical of the Deccan trap plateau and is made up of lava
future generations to meet their needs. flows.
EBD_7332
202
B- NTSE Stage 1 Question Bank

126. (2) In our routine life we use 200 types of minerals either 156. (4) The Sundarbans Mangrove forest one of the largest
directly or indirectly. such forest in the world. It was inscribed on the UNESCO
127. (2) Other than current fallow land is known as the land left world Heritage list in 1987 as a natural property.
uncultivated for the past 1-5 agricultural year. 157. (1)
128. (1) In Madhya Pradesh it is known as Bewar or Dahiya. In 158. (1) White Revolution is related with milk production. While
Odisha it is known as Pama Dabe or Koman. In western Revolution was one of the biggest dairy development
Ghats it is called Kumari . In North-east region it is known movement by the Indian Governemnt in India 1970. Verghese
by the name Jhumming. Kurien is known as the Father of the white Revolution in India.
129. (1) These deposits are called placer deposits and gener- 159. (3) The Kolkata metro is a Mass rapid Transist Urban
ally contains minerals which are not eroded by water. Railway network in Kolkata. It was the first underground
130. (3) The monazite sands of Kerala is rich in thorium railway in India.
131. (1) Allahabad and Haldia – NW-1 160. (4) India’s Remote sensing programme under the Indian
Kottapuram Kolam – NW-3 space reserarch Organisation started off in 1988 with IRS-IA,
Kakinada Puducherry the first series of the indigenous state of ant operating remote
Stretch of Canals – NW-4 sensing saltellite. The others are not launched from India.
Sadiya and Dhubri 161. (4)
Waterways – NW-2 162. (3) The mountain ranged of India from South to North
132. (1) Rainbow revolution is related with new agricultural include Western Ghats, Satpura hills, Vindhya ranges and
policy. The various colours of rainbow revolution indicates Aravali Hills.
various farm practices such as green revolution (foodgrains), 163. (1) Kaziranga National Park is a national park in the
white revolution (milk), yellow revolution (oil seeds), Blue Golaghat and Nagaon districts of the state of Assam.
revolution (fisheries), Golden revolution (fruits), Silver revo- Sunderbans National Park is a national Park Tiger Reserve
lution (eggs), Pink revolution (meat), Grey revolution and a Bioshere reserve in West Bengal. It is located in the
(fertilisers). sunder bans delta of West Bengal. Gir National Park is a
133. (2) 134. (2) 135. (3) wildlife sanctuary in Gujarat western India. It is famous Asialic
136. (3) It is found in northern plain. The never younger de- lions. Tandova National park is a tiger reserve in Chandrapur
posits of the flood plains are called khadar. district of Maharashtra.
137. (2) 164. (4) Both the statements are true
138. (4) The windward side of the western Ghats receives very 165. (4) 166. (3)
heavy rainfall more than 250 cm. 167. (3) Population density is a measurement of population per
139. (2) Silver, Fir and Pine Trees are common trees of these unit area.
forest (montane). They are found in hilly areas. 168. (1) The Rajasthan Atomic Power station is located at
140. (3) Rawatbhata in the state of Rajasthan. Rawatbhata site
141. (1) Uttar Pradesh, is the major sugarcane producing state. consists of 6 units. Barauni thermal power station is located
The other states include are Maharashtra, Karnataka, Tamil at Begusarai district, Bihar. It was established which the
Nadu, Andhra Pradesh, Bihar, Punjab and Haryana. technical help of yugoslavia and Poland. The wind power
142. (4) plants located at Nagercoil in Tamil Nadu Barmar in Rajasthan
143. (2) The first successful cotton textile mill was established have the largest single location Solar plant.
in Mumbai in 1854. Cotton textile industry is one of the 169. (2) In the states of Punjab, Haryana and Western part of
largest single industries in India. Uttar Pradesh over-irrigation is responsible for land to
144. (4) The National Population Policy framework 2000 does increase in Salinity and Alkalinity in the soil.
not provide for increasing the employment opportunties. It is 170. (2)
applicable uniformly to the whole of country. 171. (1) Names of cylones of different countries such as in India
145. (3) it is called Jal. In Bangaladesh it is known as Giri. In myanmar
146. (1) Yellow and Red Soils are found in Chhattisgarh. It is it is known as Thane. In Oman it is called Mujan.
also found in parts of Odisha, Southern Parts of middle 172. (2)
Ganga plain and along the piedmont zone of the Western 173. (1) Laterite soil are mainly found in Karnataka, Kerala, Tamil
Ghats. Nadu, Madhya Pradesh and hilly areas of Odisha and Assam
147. (4) Machines, tools and buildings can be used in produc- Red Soil develops on crystalline igneous rocks in areas of
tion over many years and are called fixed capital. low rainfall in the eastern and Southern parts of Deccan
148. (4) Rabi crops are sown in winter from October-December plateau. Black soil region spread over north west Deccan
and harvested in summer from April to June. Besides wheat plateau. They cover the plateaus of Maharashtra, Saurashtra
other rabi crops are barley, peas, grams and mustard. Malwas Madhya Pradesh and Chhattisgarh and extend in
149. (1) the south-east direction along the Godavari and the Krishna
150. (4) The basket of egg topography is a glacial landform valleys. Alluvial soil is most widely spread. The entire
due to depositions of glacial sediments or till. nothern plains are made up of alluvial soil.
151. (3) 152. (3) 174. (2) These states such as Sikkim, West Bengal, Assam and
153. (2) Benguela current flows along the coast of South-West Arunachal Pradesh hare Common frontiers with Bhutan.
Africa nearly to the equation before merging the the 175. (3)
westward flowing Atlantic South equatorial current. 176. (3) The part of the Himalayas lying between Satluj and
154. (3) Coromandel coastal plain located in Estern Tamil Nadu Kali rivers is known as Kumaon Himalayas.
state. Southern India extending over an area of about 8,800 177. (3)
sq miles. 178. (4) Sundarbans (WB) Gulf of Mannar (T.N) and Nanda
155. (2) India has tropical monsoon climate, as its climate is Devi (Uttarakhand) have been included in the world network
influenced by the monsoon winds. of biosphere reserves.
Solved Questions B-203
179. (3) 212. (2) B, C and D
180. (4) The Earth Summit held in New York in 1997 is not related All the three methods such as construction of check dam,
with Conservation of Resources. contour plonghing, afforestation can be adopted to minimize
181. (3) Tea an important beverage crop introduced in India soil erosion.
inetially by the British. Tea cultivation in an example of 213. (3) (i) Nagarhole (2)
plantation agriculture. (ii) Gir (1)
182. (3) Limestone is not used in oil refinery industry. (iii) Kaziranga (4)
183. (2) (iv) Jim Corbett (3)
184. (4) Rubber is non-food crop, an important industrial raw 214. (2) The slash and bum agriculture is known as Milpa in Mexico.
material, Hemp, natural silk and cotton are fibre crops. 215. (2) Coal is non-renewable as well as non-recyclable resource.
185. (3) 216. (3) 217. (4)
186. (2) Koyana Multipurpose Project is set up on the Koyana 218. (1) A dendritic pattern is formed when river and its tribu-
river in Satara district (Maharashtra). On the basis of taries resemble the branches of a tree.
electricity generating potential, this river is called as the life 219. (4) Rubber belongs to tropical evergreen region
line of Maharashtra. 220. (3) 221. (1) 222. (3) 223. (3)
187. (3) 'Slash and Burn' agriculture is known as 'Roca' in Brazil. 224. (4) 225. (3)
It is a shifting cultivation in which field is cultivated for 226. (1) Barchans are the crescent shape sand dunes which
short period then left for recovering. can shift from one place to another and they are generally
188. (2) Mumbai–Pune Expressway is India's first six–lane high found in the deserts of Turkestan.
speed expressway and was made by Maharashtra State Road 227. (2) The long narrow coastline of Kerala is called as Malabar
Development Corporation. Coast.
189. (1) Kavaratti is the capital of the Union Territory Lakshdweep 228. (1) Haematite is a reddish-black mineral which consists of
and the region comes under the jurisdiction of Kerala High ferric oxide which is an important ore of iron.
Court. 229. (1) Bhakra-Nangal Project is situated on the Satluj River.
190. (1) Copper Mines are found in Khetri, a town in Rajasthan 230. (3) Didwana is a salt water lake situated nearly 180 km
State. west of Jaipur in Rajasthan.
191. (4) The Longitudinal extent of India is from 68°08' East to 231. (2) Pulses are an example of leguminous crops. Being legumes,
97°25' East these plants have the advantage of fixing atmospheric nitrogen
192. (1) The Greater Himalayas are known as the young fold for their own needs and for soil enrichment, thereby reducing
mountains. the cost of fertilizer inputs in crop farming.
193. (3) Rusting of rock is caused by the process of oxidation. 232. (4) Narmada is the only river in India that flows in a rift
194. (2) Roches Montanes are the hillocks of rocks that are valley, flowing west between the Satpura and Vindhya ranges.
formed by Glacial Abrassion and Plucking. 233. (4) Limestone is a non metallic mineral. Limestone is a
195. (1) Canaries current flows along the coast of Portugal. sedimentary rock composed largely of the minerals calcite
196. (3) Tropic of Cancer passes through eight state of India. and aragonite, which are different crystal forms of calcium
They are Mizoram, Tripura, West Bengal, Jharkhand, carbonate.
Chattisgarh, Madhya Pradesh, Rajasthan, Gujarat. 234. (2) Kolkata is an inland riverine port. It is the oldest
197. (1) 198. (1) 199. (1) 200. (2) operating port in India, and was constructed by the British
201. (1) Gondwana land is the name given to an ancient East India Company. This port serves a very large and rich
supercontinent that included India, Australia, South Africa and hinterland of Ganga-Brahmaputra basin.
South America. 235. (2) Western disturbances cause rainfall during winter in
202. (1) Tapi river run through rift valleys. the north western part of India. This is a pattern driven by
203. (1) Western Disturbances is an extratropical storm Westerlies. Western Disturbances are important to the
originating in the Mediterranean region that rainfall in the development of the Rabi crop in the northern subcontinent,
Northern and North-Western parts of India in the month of which includes the locally important staple wheat.
October-November. 236. (3) Out of 30 big dams proposed along the Narmada, Sardar
204. (2) The slash and burn agriculture is the primitive form of Sarovar Project (SSP) and Narmada Sagar Project (NSP) are
cultivation and known as Milpa in Mexico. the megadams. The Maheshwar and Omkareshwar dams
205. (2) Coal is non-renewable as well as non-recyclable along with SSP and NSP, are to form a complex which would
resource. Because it cannot be replenished. ultimately cater to the needs of SSP.
206. (3) Korba is a thermal power station located in Korba 237. (2) Majuli, the largest inhabited riverine island is found in the
district of Chhatisgarh. Brahmaputra River. Majuli had a total area of 1,250 square
207. (1) The working age population is defined as those aged kilometres (483 sq mi), but having lost significantly to erosion it
15 to 59. has an area of only 421.65 square kilometres (163 sq mi) in 2001.
208. (3) According to Census of India 2001 a person aged 238. (2) El Niño (Spanish name for the male child), initially
7 years and above who can read and write any language referred to a weak, warm current appearing annually around
with understanding has been taken as literate. Christmas time along the coast of Ecuador and Peru and
209. (2) Ganga – Bhagirathi – Hoogly river system from lasting only a few weeks to a month or more. Every three to
Allahabad to Haldia was declared as N.W. number 1. seven years, an El Niño event may last for many months.
210. (2) In the state of Tamil Nadu, rainwater harvesting was 239. (4) Australia is the only continent without any active volcanoes.
made compulsory for every building to avoid groundwater 240. (2) Tripura is a state in North East India. The third-smallest
depletion. state in the country, it covers 10,491 km2 (4,051 sq mi) and is
211. (1) Afghanistan, Pakistan, Bangladesh, Srilanka are the bordered by Bangladesh to the north, south, and west, and
neighbouring countries of India. the Indian states of Assam and Mizoram to the east. It does
not have common boundary with Myanmar.
EBD_7332
204
B- NTSE Stage 1 Question Bank

241. (4) Satpura hills are part of the Deccan plateau in Madhya 253. (1) The right answer is Bhangar
Pradesh. The hills stretch for some 560 miles (900 km) across Bhangar plains are upland, and consist of older alluvial soil.
the widest part of peninsular India, through Maharashtra Bhangar areas are less prone to flooding but are usually
and Madhya Pradesh states. The range, the name of which more sandy and less fertile as well.
means "Seven Folds," forms the watershed between the Khadar plains are low lying and consist of new alluvial
Narmada (north) and Tapti (south) rivers. deposits and are often very fertile.
241. (2) The Wildlife Protection Act, 1972 is a landmark in the The rivers, after descending from the mountains deposit
protection of wildlife in India especially after the Second pebbles in a narrow belt of about 8 to 16 km in width lying
World War when the struggle for freedom started taking its parallel to the slopes of the Shiwaliks. It is known as Bhabar.
shape and the wildlife was ignored to sustain at its own All the streams disappear in this bhabar belt.
risks. South of this belt, the streams and rivers re-emerge and
243. (2) River Chambal, a principal tributary of river Yamuna, create a wet, swampy and marshy region known as Terai.
originates in the Vindhyan ranges near Mhow in Indore 254. (1) Graphite in a non-metallic allotrope of carbon, whereas
District of Madhya Pradesh. the other options are metallic in nature.
244. (2) Tropical evergreen forests are usually found in areas 255. (3) Wells (including tube wells) account for more than 30
receiving more than 200 cm of rainfall. They are called million acres (more than 55 per cent of total irrigated area).
evergreen because these trees do not shed their leaves in a Well irrigation is most common in alluvial plain areas where
particular season, so they appear green always. the water table is fairly high. Owing to the soft nature of the
245. (2) National Highways 6 connects Kolkata with Chennai. soil, wells are easy to dig and the yield of crops from the
246. (3) The major commercial source of thorium is monazite, land after irrigation is rewarding.
an anhydrous rare earth phosphate with the chemical formula 256. (1) Jhansi lies in central India and does not come in that
(Ce,La,Nd,Th) PO4. Typically, 3 to 5 percent of the metal route.
content of monazite is thorium (in the form of thorium dioxide, 257. (2) The Kerala Backwaters are a network of interconnected
ThO2). canals, rivers, lakes and inlets, a labyrinthine system formed
247. (4) "The Earth's surface is composed mostly of water, by more than 900 km of waterways, and sometimes compared
basalt and granite. Oceans cover about 70% of Earth's to the American Bayou. In the midst of this landscape there
surface. These oceans are up to 3.7 km deep. The Earth's are a number of towns and cities, which serve as the starting
thin, rocky crust is composed of silicon, aluminum, calcium, and end points of backwater cruises. National Waterway
sodium and potassium." No. 3 from Kollam to Kottapuram, covers a distance of 205
248. (4) Natural loss of soil and rock debris, blown away by km and runs almost parallel to the coast line of southern
wind or washed away by running water, laying bare the Kerala facilitating both cargo movement and backwater
rock below. Over millions of years, denudation causes a tourism.
general lowering of the landscape. 258. (4) Trichur is in the state of Kerala.
249. (1) Eutrophication was recognized as a water pollution 259. (4) The black soils are made up of extremely fine clayey material.
problem. Eutrophication can be human-caused or natural. They are well known for their capacity to hold moisture. They
Untreated sewage effluent and agricultural run-off carrying are rich in soil nutrients such as calcium carbonate, potash, lime
fertilizers are examples of human-caused eutrophication. etc. They develop cracks during hot weather.
However, it also occurs naturally in situations where 260. (3) Vishakhapatnam, Bhadravati, Salem and Vijayanagar
nutrients accumulate. are located outside the Chotanagpur region. Chotanagpur
250. (1) When the Gulf stream encounters the cold water of the plateau region has the maximum concentration of iron and
Labrador current, principally in the vicinity of the Grand steel industries. It is because of low cost of iron ore, high
Banks, there is little mixing of the waters. Instead, the junction grade raw materials in proximity, cheap labour and vast
is marked by a sharp change in temperature. The line or growth potential in the home market.
surface along which this occurs is called the cold wall. 261. (4) Maasai Mara (Masai Mara) is known as one of Africa's
251. (3) The word monsoon is derived from an Arabic word Greatest Wildlife Reserves, situated in southwest Kenya
'mausim' which means seasonal wind. In this system the and is part of the northern section of the Serengeti National
direction of the winds reverses seasonally. Park. Famous for the abundance of the big cats, Lion,
252. (2) Mexico - Sulphur, Indonesia - Rubber, Malaysia - Tin, Leopard, Cheetah and the Great Wildebeast Migration and
Brazil - Coffee. the Maasai people, well known for their distinctive custom
and dress.
MAT EXEMPLER PRACTICE QUESTIONS
5 3 6
DIRECTIONS (Qs. 1 to 4) : The following questions are based on 2 4 2 3 4 7
number or letter. Find the correct alternative in each questions to
complete the series. 10. 584 694 ?
1. 1, 4, 2, 8, 6, 24, 22, 88, ? 2 2 3 2 1 2
(1) 90 (2) 176 1 2 1
(3) 352 (4) 86 (1) 937 (2) 824
2. KM1, IP3, GS6, EV11, ? (3) 769 (4) 678
(1) BX18 (2) BY16 11. If ‘+’ stands for multiplication, ‘–’ stands for division,
‘x’ stands for subtraction and ‘ ’ stands for addition then
(3) CY18 (4) CZ18
(36 4) 8 4
3. LXF, MTJ, NPN, OLR, ?
4 8 2 16
(1) PHV (2) PIU
(1) 0 (2) 8
(3) PKX (4) PJW
(3) 12 (4) 16
4. a b_ _ b c c b a _ b _
DIRECTIONS (Qs. 12 & 13) : The four figures marked as A, B, C
(1) c a c a (2) b b b c
& D in the following questions follow a series. Select a figure from
(3) a a a b (4) a a b c
the given alternatives marked as 1, 2, 3 and 4 which will continue
5. If the word GRANDEUR is coded as NARGRUED, which
the same series.
word will be coded as SERPEVRE?
(1) PERSERVE (2) PRESEVER 12.
(3) PERSEVER (4) PRESERVE
6. In a certain code language, the sentences written in Column
I are coded as sentences written in Column II, but the
sequence of words is different. Decode the sentences and A B C D
find which word in coded language mean sweet?
Column - I Column - II
Drink fruit juice tee see pee
Juice is sweet see kee lee
He is intelligent Lee ree mee (1) (2) (3) (4)
(1) pee (2) see
(3) tee (4) Kee 13.
7. Pointing to a man in a photograph, Reena said, “His brother’s
father is the only son of my grandfather.” How is Reena
related to the man in photograph? A B C D
(1) Mother (2) Grand Daughter
(3) Sister (4) Daughter
DIRECTIONS (Qs. 8 to 10) : Find the missing character/number
in each of the following questions:
8. 2 4 0 (1) (2) (3) (4)
1 2 4 DIRECTIONS (Qs. 14 & 15) : In each of the following questions,
3 1 3 a problem figure is given. Study the figure carefully and then
36 ? 91 answer the questions given under it.
(1) 25 (2) 48 14. How many squares are there?
(3) 59 (4) 73
2 4 8
9. 4 C 10 H70 5 J
26 3 6 90 ?
5 4 6
(1) 2 (2) 3 (1) 18 (2) 19
(3) 4 (4) 5 (3) 25 (4) 27
EBD_7332
M-2 NTSE Stage 1 Question Bank
15. How many squares are there? DIRECTIONS (Qs. 19 & 20) : Three positions of a cube are
shown in the figure. Observe them and answer the following
questions.

(1) 22 (2) 20
(3) 18 (4) 16
(1) (2) (3)
16. Shrikant from a certain place went 4 km to East. He turned
19. What sign will be on the surface opposite to surface having
left and went 1 km. Then he turned right and went 2 km.
Again he turned right and walked a distance of 9 km. Then at sign ?
what distance is he from his original position ?
(1) 4 km (2) 10 km (1) (2)
(3) 9 km (4) 6 km
17. In the figure given below, a transparent square shaped paper (3) (4)
is folded along the dotted lines. What figure will be obtained ?
Find the figure from the alternative figures given. 20. Which sign will be on opposite surface of figure (2) ?
Question figure : (1) (2)

(3) (4)

21.
F
A E
(1) (2)
B
C D

The above figure is folded to form a cube. Observe the cube.


(3) (4) Of the following cube figures, find the non-relevant figure.
F E
18. A folded piece of square paper is shown as question figure at C B
left side. The paper is unfolded. How will it look is shown in the (1) A (2) D
alternatives on right side. Select the correct alternative.
Question Figure : C B

(3) B A (4) E F

22. In the following question the given figure is incomplete.


Select the correct alternative which can complete the figure.

(1) (2)

(1) (2)

(3) (4)
(3) (4)
Exemplar Practice Questions M-3
23. Sohan was born on 25th February 2012. Hasan is younger to DIRECTIONS (Qs. 31 & 32) : Kareem travels 4 kms towards
Sohan by four days. If 26th January in that year falls on North on his bike from his house and reaches point A. Then travels
Thursday, then Hasan’s birthday falls on which day ? 5 km towards East and reaches point B. From B he travels 4 km in
(1) Saturday (2) Wednesday North direction to reach point C. From C he travels 5 kms in the
(3) Monday (4) Thursday west direction to reach the mosque.
DIRECTIONS (Qs. 24 & 25) : Complete the given analogy by 31. How far is Kareem's house from the Mosque?
selecting the correct answers from the alternatives. (1) 8 kms (2) 9 kms
24. 8 : 32 : : 27 : ? (3) 13 kms (4) 18 kms
(1) 36 (2) 81 32. In which direction is the Mosque from the Kareem's house?
(3) 189 (4) 243
(1) South (2) Southeast
25. W U S Q : D F H J : : ? : N P R T
(1) G I K M (2) Y W U S (3) North (4) Northeast
(3) T R P N (4) M K I G DIRECTIONS (Qs. 33 & 34) : Find the correct mirror images for
26. Choose and substitute the correct set of signs/signs and the following problem figures choosing from the alternatives.
number in place of* sequentially, selecting from the given
alternatives to make the equations meaningful.
10 * 10 * 10 * 100 * 10
(1) +, =, ÷, + (2) ×, ÷, =, ÷ 33.
(3) ×, +, =, × (4) =, ×, ÷, +
27. How many Parallelograms are there in the given figure?

(1) (2)

(1) 18 (2) 20
(3) 22 (4) 24
28. How many Semicircles are there in the given figure? (3) (4)

34. K OH L I

(1) (2)
(3) (4)
35. Rajan and Kamala lives with their two daughters and three
(1) 16 (2) 14 sons along with their families. Each of the sons have one
(3) 12 (4) 10 daughter and two sons. Each of the daughters have two
DIRECTIONS (Qs. 29 & 30) : In the questions below the numbers daughters only. Find the female members in the family.
in the figures are related. Identify their relationship and find the (1) 10 (2) 11
missing numbers in the given figures. (3) 12 (4) 13
29. 4 12 4 8 DIRECTIONS (Qs. 36 & 37) : The following intersecting circles
represents a group of 70 actors, who act in Cinema, Serial and Drama,
9 3 10 5 Study the intersecting circles and answer the given questions

9 ? 8 5 Cinema Serial
10 Actors
Actors 15
10
8 6 10 16 15 10
(1) 14 (2) 12 5 Drama
(3) 11 (4) 6 Actors
30. 36. How many are Cinema and Serial Actors only?
4 226 1 37 2 135 (1) 20 (2) 15
(3) 10 (4) 5
6 74 3 11 ? ? 37. Totaliy how many are serial actors?
(1) 40 (2) 35
(1) 8, 138 (2) 7, 108
(3) 30 (4) 25
(3) 6, 46 (4) 5, 18
EBD_7332
M-4 NTSE Stage 1 Question Bank
38. In a code if J = 17 and DOG = 47, then MICE can be coded as DIRECTIONS (Qs. 46 to 48) : In each of the questions, there is a
(1) 30 (2) 45 big figure at the top having a vacant chamber. Identify one
(3) 58 (4) 65 figure from the given choices which will fit into the vacant chamber.

DIRECTIONS: (Qs. 39 & 40) : Geetha is 2 years older than Seetha.


Seetha is 3 years older than Deepthi. Shankar is one year older
46.
than Deepthi.
39. Who is the youngest among all?
(1) Geetha (2) Shankar
(3) Seetha (4) Deepthi
40. It the age of Deepthi is 6 years, then what is the age of
Geetha? (1) (2) (3) (4)
(1) 17 years (2) 13 years
(3) 11 years (4) 8 years
47.
DIRECTIONS (Qs. 41 & 43) : Read the following information
and answer questions. In a certain code language, 481 means ‘sky
is blue’, 246 means ‘sea is deep’ and 698 means ‘sea looks blue’.
41. What number is the code for blue?
(1) 1 (2) 6
(3) 8 (4) 9 (1) (2) (3) (4)
42. What number is the code for sea?
(1) 4 (2) 6
(3) 8 (4) 9 48.
43. What number is the code for deep?
(1) 2 (2) 4
(3) 6 (4) 8
DIRECTIONS (Qs. 44 & 45) : In each of the questions, which
one of the four numbered figures will replace the question mark so (1) (2) (3) (4)
as to maintain the sequence?
49. HCF of three number is 12. If they be in the ratio of 1 : 2 : 3,
44. the numbers are
(1) 12,24,36 (2) 10,20,30
(3) 5, 10, 15 (4) 4,8, 12
50. The ages of Vivek and Sumit are in the ratio of 2 : 3. After 12
years, their ages will be in the ratio of 11 : 15. The age of
Sumit is
(1) 32 years (2) 42 years
(3) 48 years (4) 56 years
51. A, B and C invest ` 2000, ` 3000 and ` 4000 respectively in
(1) (2) (3) (4) a business. After one year, A removed his money but B and
C continued for one more year. If the net profit after 2 years
45. be ` 3200, then A’s share in the profit is
(1) ` 1000 (2) ` 600
(3) ` 800 (4) ` 400
52. When the price of a toy increased by 20% the number of
toys sold was decreased by 15%. The effect on the sales of
the shop is
(1) 4% increase (2) 4% decrease
(3) 2% increase (4) 2% decrease
53. A and B can together do a piece of work in 15 days. B alone
(1) (2) (3) (4) can do it in 20 days. A alone can do it in
(1) 30 days (2) 40 days
(3) 45 days (4) 60 days
Exemplar Practice Questions M-5
54. A circle and a square have same area. The ratio of the side of 61. Problem figures:
the square and the radius of the circle is
(1)
(3) l :
:1 (2) 1:
(4) :l ?
55. The average age of four members of a family is 20 years. (a) (b) (c) (d)
If the age of grandfather be included, the average age is Answer figures:
increased by 9 years. The age of grandfather is
(1) 48 years (2) 52 years
(3) 65 years (4) 72 years
DIRECTIONS (Qs. 56 & 57) : In each of the following questions,
arrange the given words in the sequence in which they occur in (1) (2) (3) (4)
the dictionary and then choose the correct sequence : 62. Problem figures:
56. (A) Preach (B) Praise
(C) Precinct
(E) Precede
(D) Precept
?
(1) B, A, E, D, C (2) B, A, C, D, E
(a) (b) (c) (d)
(3) B, E, A, D, C (4) A, B, E, D, C
Answer figures:
57. (A) Select (B) Seldom
(C) Send (D) Selfish
(E) Seller
(1) A, B, D, E, C (2) B, A, E, D, C
(3) B, A, D, E, C (4) B, E, D, A, C
(1) (2) (3) (4)
58. In a queue, Amrita is 10th from front while Mukul is 25th from
behind and Mamta is just in the middle of the two. If there be DIRECTIONS (Qs. 63 & 64): Out of nine cells of a square one
50 persons in queue, what position does Mamta occupy cell is left blank and in the rest of the cells numbers are wirtten
from front ? which follow some rule. Get the rule and find out the proper option
(1) 26th (2) 19th for the blank cell (?)
th
(3) 18 (4) 17th
63.
DIRECTIONS (Qs. 59 to 62) : In these questions, there are four 7 32 ?
problem figures (a), (b), (c) and (d). One of them is, however,
missing. You have to select one figure from the four answer figures, 31 8 25
(1), (2), (3) and (4) such that the series is completed. Indicate your
answer as per the “Instructions”.
11 24 9
59. Problem figures: (1) 50 (2) 48
(3) 47 (4) 51

? 64. 4 20 25
(a) (b) (c) (d)
27 81 9
Answer figures:
11 44 ?
(1) 4 (2) 16
(3) 30 (4) 55
(1) (2) (3) (4)
Directions (Qs. 65 & 66): Find out the correct alternative of the
60. Problem figures:
questions based on the Dice figures

+ +
+ ?
(a) (b) (c) (d)
65.
Answer figures:

+ The number opposite side the face having the no. 4 will be:–
+ + + (1) 1 (2) 2
(3) 5 (4) 6
(1) (2) (3) (4)
EBD_7332
M-6 NTSE Stage 1 Question Bank
71. What percentage of the total weight of the human body is
equivalent to the weight of the skin in the human body?
(1) 1.6 (2) 0.16
66. (3) 0.016 (4) Insufficient data
72. To show the distribution of proteins and other dry elements
in the human body, the arc of the circle should subtend at
(i) (ii) (iii)
the centre an angle of:
Three positions of a Dice are shown. In figure (iii) which
number will come in place of (?). (1) 120° (2) 54°
(1) 5 (2) 3
(3) 108° (4) 252°
(3) 6 (4) 1
67. In a code, 73. What is the quantity of water in the body of a person
A + B means A is the daughter of B; weighing 50 kg?
A – B means A is the husband of B;
(1) 35 kg (2) 120 kg
A × B means A is the brother of B.
If P + Q – R, which one of the following is true ? (3) 71.42 kg (4) 20 kg
(1) R is the mother of P
(2) R is the sister-in-law of P DIRECTIONS (Qs. 74 to 76) : In each of the following questions, a
(3) R is the aunt of P statement is given followed by two conclusions I and II. Give answer :
(4) R is the mother-in-law of P (1) if only conclusion I follows;
(2) if only conclusion II follows;
DIRECTIONS (Qs. 68 & 69): In the following questions some (3) if either I or II follows;
relations are written by particular indicators as shown below :– (4) if both I and II follow.
= Greater than 74. Statement : All the organised persons find time for rest.
+ = Equal to Sunita, inspite of her very busy schedule,
= Not equal to finds time for rest.
= Not greater than Conclusions : I. Sunita is an organised person.
× = Not less than
= Less than II. Sunita is an industrious person.
Find out the correct answer for each question. 75. Statement : Quality has a price tag. India is allocating
68. If p q O r, it is possible that :– lots of funds to education.
(1) p × q × r (2) p × q r Conclusions : I. Quality of education in India would
(3) p q r (4) p q r improve soon.
69. If p q r, it is not possible that :–
II. Funding alone can enhance quality of
(1) p q r (2) p q r education.
(3) p q r (4) p + q × r
76. Statement : Modern man influences his destiny by the
DIRECTIONS (Qs. 70 to 73) : Refer to the following pie charts choice he makes unlike in the past.
and answer.
Conclusions : I. Earlier there were less options available
to man.
Proteins elements
16% II. There was no desire in the past to
Muscles 14% influence the destiny.
Hormones 1/3
Enzymes Skin DIRECTIONS (Qs. 77 to 79) : Each of the questions below consists
& other 1/10 of a question and two statements numbered (I) and (II) given
proteins 1/6
Bones below it. You have to decide whether the data provided in the
Water statements are sufficient to answer the question. Read both the
14%
statements and give answer
(1) Statement (I) ALONE is sufficient, but statement (II) alone is
Distribution of protein Distribution of elements not sufficient.
in human body in human body (2) Statement (II) ALONE is sufficient, but statement (I) alone is
70. What is the ratio of the distribution of proteins in the muscles not sufficient
to that of the distribution of proteins in the bones? (3) BOTH statements TOGETHER are sufficient, but NEITHER
(1) 1 : 2 (2) 2 : 1 statement alone is sufficient.
(3) 1 : 18 (4) 18 : 1
(4) EACH statement ALONE is sufficient
Exemplar Practice Questions M-7
77. In ABC, find r if AB = 5 and q = 40º DIRECTIONS (Qs. 82 & 83) : Read the following information
carefully and answer the questions that follow :
B
A, B, C, D, E and F are seated in a circle facing the centre. D is
r° between F and B. A is second to the left of D and second to the
right of E.
82. Who is facing A?
(1) B (2) D
p° q° (3) F (4) Either F or B
A C 83. Who among the following is facing D?
(1) A (2) C
(I) BC = 5 (II) r > p
(3) E (4) Cannot be determined
M
78. Is the number an odd integer? (You may assume that DIRECTIONS (Qs. 84 & 85) : Find the correct option for the
3
water images for the following questions.:
M
is an integer)
3 absence
(I) M = 3K, where K is an integer. 84.
(II) M = 6J + 3, where J is an interger. ?
79. In the given figure P, Q and R are centres of three equal (1) (2)
circles. What is the area of the shaded portion in the figure (3) (4)
shown below ?
85.

P
Xx°
(1) (2)

Xx° Xx° (3) (4)


Q R
DIRECTIONS (Qs. 86 to 88) : In the following diagram, the circle
represents college Professors, the triangle stands for surgical
Specialists, and medical Specialists are represented by the
(I) QR is known. rectangle
(II) The triangle is an equilateral triangle.
A
DIRECTIONS (Qs. 80 & 81) : Read the following information to
answer the questions given below : D
X X
(i) Five boys are standing in a line facing the wall wearing red, Y
green, yellow, white and blue dress.
(ii) The yellow-dressed boy is not standing at any end of the B Z B
line
(iii) The red-dressed boy is not standing at any end of the line. C
80. The boy in the middle wears which coloured-dress?
86. College professors who are also surgical Specialists are
(1) Green represented by
(2) Blue (1) A (2) B
(3) Either Green or Blue (3) C (4) D
(4) Cannot be determined
87. Surgical Specialists who are also Medical Specialists but
81. Who is to the right of yellow-dressed boy?
not Professors are represented by
(1) Data inadequate (2) White
(3) Green (4) Blue (1) B (2) C
(3) X (4) Z
EBD_7332
M-8 NTSE Stage 1 Question Bank
88. C represents 94. Which two students, who are not wearing glasses, are short?
(1) Medical Specialists (1) A and F (2) C and E
(2) College Professors (3) B and E (4) E and F
(3) Surgical Specialists 95. Which short student of Gandhi House is not wearing
(4) Medical and surgical Specialists glasses?
89. On January 12, 1980, it was Saturday. The day of the week (1) F (2) E
on January 12, 1979 was – (3) B (4) A
(1) Saturday (2) Friday 96. Which tall student of Gandhi House is not wearing glasses?
(3) Sunday (4) Thursday (1) B (2) C
(3) E (4) F
90. Find the exact time between 7 am and 8 am when the two
hands of a watch meet ? 97. On a shelf are placed six volumes side-by-side labelled A, B,
C, D, E and F. B, C, E, F have green covers while others have
(1) 7 hrs 35 min (2) 7 hrs 36.99 min yellow covers. A, D, B are new volumes while the rest are old
(3) 7 hrs 38.18 min (4) 7 hrs 42.6 min volumes. A, C, B are law reports while the rest are medical
extracts. Which two volumes are old medical extracts and
DIRECTIONS (Qs. 91 to 93) : In the questions below, two have green covers?
statements are given followed by two conclusions I and II. You (1) B, C (2) C, D
have to consider both the statmeents to be true even if they seem (3) C, E (4) E, F
to be at variance with commonly known facts. You have to decide 98. In a group of six women, there are four dancers, four
which of the given conclusions is/are definitely drawn from the vocal musicians, one actress and three violinists. Girija
given statements. Give answer. and Vanaja are among the violinists while Jalaja and
Shailja do not know how to play on the violin. Shailja
(1) if only I follows
and Tanuja are among the dancers. Jalaja, Vanaja, Shailja
(2) if only II follows and Tanuja are all vocal musicians and two of them are
(3) if neither I nor II follows also violinist?
(4) if both I and II follows. (1) Jalaja (2) Shailja
(3) Tanuja (4) Pooja
91. Statements : When it rains, usually X does not go out. X 99. Sarita cuts a cake into two halves and cut one-half into
has gone out. smaller pieces of equal size. Each of the small pieces is twenty
Conclusions : I. It is not raining. five grams in weight. If she has eleven pieces of the cake in
all with her, how heavy was the original cake ?
II. X has some urgent business to transact.
(1) 700 gm (2) 400 gm
(1) 2 (2) 1 (3) 3 (4) 4 (3) 500 gm (4) 200 gm
92. Statements : In a Golf Club, all the members are not active 100. How many triangles are there in the following figure?
players of the game but all of them are rich.
Mrs. X is a member.
Conclusions : I. She is a golfer.
II. She is rich.
(1) 1 (2) 3 (3) 2 (4) 4
93. Statements : All the employees of company A have
identity cards. Ram is an employee of
company A.
Conclusions : I. Ram has an identity card.
II. Ram is the General Manager of the (1) 9 (2) 13
Company.
(3) 16 (4) 18
(1) 2 (2) 1 (3) 4 (4) 3 101. Four groups of three words each are given. Only one group
DIRECTIONS (Qs. 94 to 96): Read the following information and in each does not share the underlying principle that combines
answer the questions given below it. the words. Find that group:
(1) India, China, Japan
Six students A, B, C, D, E and F are sitting in the field. A and B are (2) Canada, Mexico, Australia
from Nehru House while the rest belong to Gandhi House. D and (3) South Africa, Kenya, Zimbabwe
F are tall while the others are short. A, C and D are wearing glasses (4) United kingdom, France, Spain
while the others are not.
Exemplar Practice Questions M-9
DIRECTIONS (Qs. 102 – 104) : Choose the odd one out: 113. Given set (8, 3, 2)
(1) (10, 6, 5) (2) (95, 24, 5)
102. (1) Seismograph (2) Earthquake (3) (63, 8, 3) (4) (168, 15, 4)
(3) Cyclone (4) Tsunami
103. (1) 145 (2) 120 DIRECTIONS (Qs. 114 – 115) : The following questions are based
(3) 90 (4) 105 on the given matrix. The value of each letter is the product of its
104. How many such pairs of letters are there in the word row and column numbers.
‘RAZORS’ each of which has as many letters between them Example: The value of letter “N” is 4 × 3 = 12
as in the English alphabet series: 114. As per directions given, find the group of letters with the
(1) None (2) One lowest total.
(3) Two (4) Three
DIRECTIONS (Qs. 105 – 106) : Read the data carefully and answer Columns
the questions.
2
LAP TUB CAR SON HID
(New words to be formed may or may not necessarily be meaningful 2 B M T
English words.)

Rows
st rd
105. If the positions of the 1 and the 3 alphabets of each of the 3 A E O
word are interchanged, which of the following would form
meaningful words with the new arrangement? 4 G N P
(1) HID (2) SON
(3) TUB (4) CAR
106. If the given words are arranged in the order as they would (1) BOAT (2) TAME (3) GATE (4) TENT
appear in a dictionary from the left to right, which of the 115. As per directions given, find the group of letters with the
following will be 4th from the left? highest total.
(1) LAP (2) BUT (1) ATPOG (2) BMNET
(3) CAR (4) SON (3) PEGNO (4) MAETB
116. Amit is now 6 times as old as his son. Four years from now,
DIRECTIONS (Qs. 107 – 109) : In each of the following the sum of their ages will be 43 years. Determine Amit’s
questions, find which word cannot be made from the letters of present age:
the given word ? (1) 30 years (2) 32 years
107. CARPENTER (3) 34 years (4) 28 years
(1) NECTAR (2) CARPET 117. Aman starts walking in south and walks for 7 km, then turns
left and walks for 2 km, Then, once again turns left and walks
(3) PAINTER (4) REPENT
for 12 km, turns left one more time and walks for 2 km. How
108. TRIBUNAL
much distance he has to cover to reach the starting point?
(1) LATIN (2) BRAIN
(1) 7 km (2) 12 km
(3) URBAN (4) TRIBLE
(3) 4 km (4) 5 km
109. ORGANISATION
118. A is 3 times more efficient than B. Hence, he takes 60 days
(1) NATION (2) GRANT
less in painting a room. In how many days, work will be
(3) RECOGNISE (4) SATAN
completed, if A and B both work together?
DIRECTIONS (Qs. 110 – 111) : In each of the following questions, (1) 30 days (2) 45 days
a group of three interrelated words is given. Choose a word from
1 1
the given alternative, that belongs to same group. (3) 22 days (4) 17 days
2 2
110. Potato : Carrot: Radish : ? 119. A train travels 92.4 km/hr. How many metres will it travel in
(1) Tomato (2) Spinach 10 minutes?
(3) Sesame (4) Groundnut (1) 15400 (2) 1540
111. Clutch : Brake : Horn : ? (3) 154 (4) 15.40
(1) Scooter (2) Steering 120. A monkey climbs 30 feet before at the beginning of each
(3) Car (4) Accident hour and rests for a while when he slips back 20 feet before
DIRECTIONS (Qs. 112 – 113) : In each of the questions choose he again starts climbing in the beginning of the next hour. IF
that set of numbers from the four alternative sets, which is similar he begins his ascent at 8.00 am, at what time will he first
to the given set. touch a flag at 120 feet from the ground?
(1) 4 pm (2) 5 pm
112. Given set (17, 15, 8)
(3) 6 pm (4) 3 pm
(1) (15, 12 10) (2) (13, 12, 5)
(3) (10, 8, 7) (4) (7, 5, 3)
EBD_7332
M-10 NTSE Stage 1 Question Bank
121. In a row of girls, Kashish is fifth from the left and Mona is DIRECTIONS (Qs. 129 – 130) : In place of (*) insert the correct
sixth from the right. When they interchange their places sequence of signs, choosing from the alternatives, to make a correct
among themselves, Kashish becomes thirteenth from the
equation.
left. Then what will be Mona’s position from the right?
(1) Fourth (2) Eighth 129. 40 * 30 * 50 * 20
(3) Fourteenth (4) Fifteenth (1) +, =, – (2) –, =, +
122. A clock is so placed that at 12 noon its minute hand points (3) =, +, – (4) +, –, =
towards North-East. In which direction does its hour point 130. 60 * 12 * 8 * 5 * 8
at 1.30 p.m. ?
(1) +, =, ×, – (2) –, =, ×, +
(1) North (2) South
(3) –, +, =, × (4) =, ×, +, –
(3) East (4) West
123. On what day of the week India will celebrate its Republic DIRECTIONS (Qs. 131 – 133) : The following questions are
Day on 26th January, 2015 ? based on addition, where each letter is given a number code.
(1) Sunday (2) Monday 131. Find the number which represents L E T choosing from the
(3) Tuesday (4) Wednesday alternatives.
LEM
DIRECTIONS (Qs.124 – 126) : are based on the following
information: Data on 450 candidates, who took on examination in +ME LT
Social Science, Mathematics and Science, are given below: 99 99
Passed in all Subjects ——— 167 (1) 450 (2) 940
Failed in all Subjects ——— 60 (3) 405 (4) 504
Failed in social science ——— 175 132. Find the number which represents S E T choosing from the
Failed in Mathematics ——— 199 alternatives.
Failed in Science ——— 191 ENT
Passed in Social Science only ——— 62 +T E N S
Passed in Mathematics only ——— 48 1867
Passed in Science only ——— 52 (1) 631 (2) 641
124. How many failed in Social Science only? (3) 436 (4) 384
(1) 15 (2) 21 133. Pankaj starts from point A and walks east to point B. From
(3) 30 (4) 42 point B he walks south to point C. From point C he walks
125. How many failed in one subject only? northeast to point D. From point D he walks north to
(1) 152 (2) 144 point E.
(3) 61 (4) 56 Which of the following diagrams represent Pankaj’s
126. How many passed in Mathematics and at least one more route?
subject?
(1) 210 (2) 203 B B
(3) 170 (4) 94
(1) E (2) E
DIRECTIONS (Qs.127 – 128) : Read the questions carefully and
C
give the answer.
C
127. If ‘–’ means ‘+’; ‘+’ means ‘–’; ‘×’ means ‘÷’ and ‘÷’ means
‘×’, then the value of 7 – 8 × 5 6 + 4is D
(1) 3 (2) 12
3 48
(3) 12 (4)
5 5
A B E
128. If ‘A’ means ‘–’ ; ‘B’ means ‘÷’; ‘C’ means ‘+’ and ‘D’ means D
‘×’, then 15 B 3 C 2 4 A 1 2 D 2 = ? (3) (4) B
(1) 5 (2) 2 D
5 4
(3) (4) 23 C C
9 9
E
Exemplar Practice Questions M-11
DIRECTIONS (Qs. 134 - 136) : Read the statements and choose 143. A + B means A is Father of B;
the letter of the region which correctly represents the statement. A – B means A is Wife of B;
A × B means A is brother of B;
and A B means A is Daughter of B;
If P + R Q, Which of the following is true?
a f b Science (1) P is the Brother of Q (2) P is the Son of Q
Maths g (3) P is the Husband of Q (4) P is the father of Q
e d 144. In certain code PAPER is written as SCTGW, how will
MOTHER written in that code ?
c History
(1) ORVLGW (2) PQVJGT
(3) PQXJJT (4) PQXKJV
145. If in certain code POPULAR is coded as QPQVMBS, which
134. Students who took science but opted neither maths nor
word would be coded as GBNPVT ?
history:
(1) FAMOSU (2) FAMOUS
(1) b (2) e
(3) FASOUM (4) FOSAUM
(3) d (4) g
146. If in certain code CALCUTTA is coded as GEPGYXXE, which
135. Students who took maths and history both:
word would be coded as FSQFCE ?
(1) c (2) a
(1) BOMBYA (2) BOMBAY
(3) e (4) g
(3) BOMYAB (4) BOBAYM
136. Pointing to a boy, Mamta said, “he is the only son of my
father-in-law’s only child.” How is the boy related to Mamta? DIRECTIONS (Qs. 147 - 149): These questions consists of a
(1) Brother (2) Daughter number series which contains a wrong term. This term is given as
(3) Son (4) Husband one of the four alternatives among the four numbers given below.
The wrong term is :–
DIRECTIONS (Qs. 137 - 139) : In the following questions,
147. 89, 78, 86, 80, 85, 82, 83
numbers are written in a sequence. Find the missing number, to
replace the question mark, from the given alternatives. (1) 83 (2) 82
(3) 86 (4) 78
137. 3, 12, 27, 48, 75, 108, ?
148. 1, 1, 3, 9, 6, 36, 10, 100, 16, 225
(1) 147 (2) 162
(1) 225 (2) 16
(3) 183 (4) 192
(3) 10 (4) 9
138. 1, 4, 2, 8, 6, 24, 22, 88, ?
149. 444, 300, 200, 136, 87, 84, 80
(1) 90 (2) 176
(1) 300 (2) 200
(3) 352 (4) 86
(3) 136 (4) 87
139. KM1, IP3, GS6, EV11, ?
(1) BX18 (2) BY16 DIRECTIONS (Qs. 150 - 151): Out of nine cells of a square one
cell is blank and in the rest of the cells numbers are wirtten which
(3) CY18 (4) CZ18
follow some rule. Get the rule and find out the proper option for
DIRECTIONS (Qs. 140 - 141) : The following questions are based the blank cell (?)
on letter sequence. In each sequence, some letters are missing.
Find the correct alternative in each question to fill the blanks. 150.
2 72 56
140. a b_ _ b c c b a _ b _
(1) c a c a (2) b b b c ? 0 42
(3) a a a b (4) a a b c 12 20 30
141. x _ y y y y z _ x x y _ z z x _ y y z z
(1) y x z y (2) x z y x (1) 4 (2) 6 (3) 8 (4) 10
(3) x y y x (4) z x y x
151.
142. If train is called bus, bus is called tractor, tractor is called 91 64 73
car, car is called scooter, scooter is called bicycle and
bicycle is called aeroplane then which is used to plough a 84 76 61
field?
25 60 ?
(1) Train (2) Bus
(3) Car (4) Tractor (1) 66 (2) 68 (3) 69 (4) 71
EBD_7332
M-12 NTSE Stage 1 Question Bank
DIRECTIONS (Qs. 152 - 154): The following question are based DIRECTIONS (Qs. 158 - 159): Following four diagrams marked
on the arrangement of numbers in the form of a pyramid. In each 1, 2, 3 and 4 are given as alternatives. choose the best suitable
question there is some relationship between the two numbers on alternative diagram marked 1, 2, 3 and 4, the one that represent the
the left of the (::). The same relationship exists between the two best relationship amongest the three given groups.
terms in the right of which one is missing. Find the missing are
from the given alternatives. (1) (2)
1
2 3 4
(3) (4)
9 8 7 6 5
10 11 12 13 14 15 16
25 24 23 22 21 20 19 18 17 158. Birds, Crows, Cats
26 27 28 29 30 31 32 33 34 35 36 159. Snakes, Land creatures, Water Creatures
49 48 47 46 45 44 43 42 41 40 39 38 37 DIRECTIONS (Qs. 160 - 162) : In each of the questions find the
152. 132220 : 211412 :: 222931 : (?) missing number from the given alternatives:
(1) 304345 (2) 302123
160. 15 22 13 11 21 23
(3) 442931 (4) 312022
36 16 ?
153. 2873 : 13212014 :: 10242311 : (?) 16 9 7 9 15 13
(1) 29282423 (2) 28274746
(1) 9 (2) 25 (3) 49 (4) 64
(3) 25272824 (4) 29454430
154. 241214 : 752119 :: 14163234 : (?) 161.
(1) 19174139 (2) 20184240
(3) 21194341 (4) 20224244
DIRECTIONS (Qs. 155 - 157): Words in capital letters in column-
I are written and in small letters in a code language in column-II.
Decode the Language and find out the correct alternative for the
given letters in each questions.
Column-I Column-II (1) 24 (2) 30 (3) 40 (4) 48
HERO tbfw
162.
JOIN bakp
LAZY nsvg
MINE pdkt
PART rwsx
SAURY wveos (1) 10 (2) 15 (3) 20 (4) 25
BLUE eglt DIRECTIONS (Qs. 163 - 166) : Study the following diagram and
CIGAR vsqwp answer questions.
WRIT wpxy
VIRUS pzwoe 1 Trained teachers
QUACK jqems Teachers
PIRL wprg working 3 7 5 4
in the
155. Code for letters in the word TOIL are : 6 Experienced
rural areas
2 teachers
(1) pxba (2) bpgn
(3) bpxg (4) mpxg 163. How many experience and trained teachers, who work in the
156. Code for letters in the word COST are : rural areas, are there?
(1) boqx (2) xqps (1) 5 (2) 4
(3) qost (4) xqnr (3) 6 (4) 2
164. How many experienced and trained teachers work in urban
157. Code for letters in the word ULCER are : areas?
(1) ggwmr (2) teqwp (1) 3 (2) 4
(3) ktegp (4) gteqw (3) 5 (4) 6
Exemplar Practice Questions M-13
165. If O = 16, FOR = 42, then what is FRONT equal to? DIRECTIONS (Qs. 176 - 178) : These questions are based on
(1) 61 (2) 65 statements. In each question there are two statements followed
(3) 73 (4) 78 by two conclusions numbered I and II. Read both the conclusions
166. If TERMINATION is coded as 12345671586, how will and give answer.
MOTION be coded? A = if only conclusion I follows
(1) 481586 (2) 451586
B = if only conclusion II follows
(3) 485186 (4) 438586
C = if either I or II follows
DIRECTIONS (Qs. 167 – 170) : Following questions are based D = if neither I not II follows
on letter series. In each series some letters are missing. The missing
176. Statements: Some clips are green.
letters are given in the proper sequence as one of the alternative.
Find the correct alternatives in each case and indicate your answer. All greens are red.
167. a — bba — aa — b — a Conclusions: I. All clips are red.
(1) baab (2) aaba (3) aaab (4) abba II. Some clips are red.
168. — aabb — abba — b (1) A (2) B (3) C (4) D
(1) bab (2) baa (3) aba (4) abb 177. Statements: Some papers are files.
169. aa — bb — aa — abbbb — a Some files are pens.
(1) bbaa (2) aabb (3) abab (4) baba Conclusions: I. Some files are not pens.
170. aa — baaab — ba — II. Some pens are papers.
(1) bba (2) bbb (3) abb (4) baa
(1) A (2) B (3) C (4) D
DIRECTIONS (Qs. 171 - 175) : In questions numbers are placed 178. Statements: Our investment carry market risks.
in figures on the basis of some rules. One place in the figure is Consult your investment agent before
indicated by the interogation sign (?). Find out the correct investing.
alternative to replace the question mark and indicate your answer.
Conclusions: I. One should not invest.
4 7 11 16 II. The investment agent calculates the market
171. risk with certainty.
7 13 21 ?
(1) A (2) B (3) C (4) D
(1) 27 (2) 29 (3) 31 (4) 33
DIRECTIONS (Qs. 179 - 183) : In the following Venn diagram, if
15 27 40 57 square represents students playing cricket, circle represents
172. students playing kabaddi and triangle represents students playing
36 48 61 ?
volleyball, then answer the following questions.
(1) 68 (2) 78 (3) 74 (4) 69

30 42 53 5
31 21 11
173.
3
9 7 ? 4
2
2
(1) 9 (2) 6 (3) 10 (4) 7 1 5
2 1
12
25 17 38 18 89 16
174.
179. How many students play only kabbadi ?
6 8 ?
(1) 1 (2) 3 (3) 4 (4) 14
180. How many students play cricket, kabaddi as well as
(1) 13 (2) 17 (3) 15 (4) 19 volleyball ?
(1) 2 (2) 6 (3) 4 (4) 37
9 6 8 12 181. In the given number series how many times is 9 preceded by
3 ? 6 and 6 is preceded by 9 ?
175.
36996869888963968969
18 24 (1) 0 (2) 1 (3) 2 (4) 3
(1) 10 (2) 4 (3) 8 (4) 6
EBD_7332
M-14 NTSE Stage 1 Question Bank
182. In the given alphabetic series how many times is k succeeded DIRECTIONS (Qs. 189 – 191) : Read the following passage
by l and h succeeded by k ? carefully and answer the questions.
dbhfklthfbhkltbdbhtdbbdhk
(1) 3 (2) 2 P, Q, R, S and T are five teachers. Each teacher teaches only one
subject. T is not Hindi teacher and R, S do not teach English. P, Q
(3) 1 (4) 0
and R are not related with History and Hindi. Neither-S nor T
183. A is a group of triangles
teaches Mathematics. Geography is not taught by P, R and T.
B is a group of acute angled triangles
189. The History teacher is
C is a group of isosceles triangles
D is a group of equilateral triangles (1) P (2) Q
Then, which of the following figure represents the group A, B, (3) R (4) T
C, D. 190. The Hindi teacher is
(1) P (2) Q
A (3) R (4) S
B A 191. The teaching subject of P is
D
(1) (2) C B (1) Hindi (2) English
C D (3) Mathematics (4) History
DIRECTIONS (Qs. 192 – 194) : A, B, C, D and E are students in a
college
I. A and B study both Physics and Chemistry
B A II. C and D study both Chemistry and Mathematics
B
C II. D and A study both Electronics and Physics
(3) A C D (4)
D IV. E and B study both Mathematics and Electronics
192. Which student studies the most number of subjects ?
(1) E (2) A
DIRECTIONS (Qs. 184 - 185) : In each of the following questions (3) B (4) C
write which number in sequence replaces the question mark ?
193. Common subject A, D and C study is :
184. ?, 17, 33, 51, 75 (1) Chemistry (2) Mathematics
(1) 9 (2) 13 (3) 18 (4) 11 (3) Physics (4) Electronics
185. 14, 17, 24, 35, ? 194. The pair of students who study only two subjects are:
(1) 49 (2) 38 (3) 50 (4) 46 (1) A, D (2) B, C
DIRECTIONS (Qs. 186 - 188) : Study the information given below (3) C, E (4) E, D
and answer the questions that follow: DIRECTIONS (Qs. 195 – 196) : A paper is folded and a cut is
Six lectures A, B, C, D, E and F are to be organized in a span of made. Select the alternative which correctly depicts how the paper
seven days-Sunday to Saturday, only one lecture on each day will appear when it is opened?
accordance with the following:
(i) A should not be organized on Thursday.
(ii) C should be organized immediately after F.
(iii) There should be a gap of two days 195.
(iv) One day there will be no lecture (Friday is not that day), just
before that day D will be organized.
(v) B should be organized on Tuesday and should not be
followed by D.
186. On which day there is not lecture?
(1) (2)
(1) Monday (2) Friday
(3) Sunday (4) None of these
187. How many Lectures are organized between C and D?
(1) None (2) Four
(3) Two (4) Three
188. Which day will the lecture F organized? (3) (4)
(1) Thursday (2) Friday
(3) Saturday (4) Sunday
Exemplar Practice Questions M-15
DIRECTIONS (Qs. 202 – 203): The following questions are
related to paper cutting. The questions that follow contain a
196. set of three figures X, Y and Z, showing a sequence of folding
of a piece of paper. Fig. (Z) shows the manner in which the
folded paper has been cut. These three figures are followed by
four answer figures 1, 2, 3 and 4 (IInd Set) from which you have
to choose a figure which would most closely resemble the
unfolded form of fig. (Z).
(1) (2) 202. Sequence of folding the paper

(3) (4) X Y
Answer-Figures

DIRECTIONS (Qs. 197 - 198) : In each of the following questions,


four pairs of figures are given in three of them; the first figure is
related to the second figure in the same manner. Find the odd pair
out which is different from others. (1) (2) (3) (4)
203. Sequence of folding the paper

197.
I II I II I II I II
(1) (2) (3) (4)

198.
I II I II I II I II
Answer-Figures
(1) (2) (3) (4)

DIRECTIONS (Qs. 199 – 201) : A wooden cube is painted Blue


on all four adjoining sides and Green on two opposite sides i.e.
top and bottom. It is then cut at equal distances at right angles
four times vertically (top to bottom) and two times horizontally as
shown in the figure where dotted lines represent the cuts made.
Study the diagram and answer the questions that follow. (1) (2) (3) (4)

DIRECTIONS (Qs. 204 – 205) : In each of the questions, there


is a big figure at the top having a vacant chamber. Identify one
figure from the given choices which will fit into the vacant chamber.

204.

199. How many cubes will have one face painted only in Blue?
(1) 1 (2) 2 (3) 3 (4) 4
200. How many cubes will have one face painted only in Green?
(1) 1 (2) 2 (3) 3 (4) 4 (1) (2) (3) (4)
201. How many cubes will have at least three sides painted?
(1) 8 (2) 6 (3) 3 (4) 2
EBD_7332
M-16 NTSE Stage 1 Question Bank

210. Problem figures:


205.
+ +
+ ?
(a) (b) (c) (d)
Answer figures:
(1) (2) (3) (4) Answer figures:

DIRECTIONS (Qs. 206 – 208) : In each question there is a figure + +


and answer the question from the figure. + + +
(1) (2) (3) (4) (5)

206.
211. Problem figures:

How many triangle are there in this figure?


(1) 6 (2) 7 (3) 8 (4) 9 ?
(a) (b) (c) (d)
207. Answer figures:

How many triangle are there in this figure?


(1) 8 (2) 9 (3) 7 (4) 11 (1) (2) (3) (4) (5)

208. DIRECTIONS (Qs. 212 - 213) : Find the odd figure out.

How many triangles and quadrilaterals are there in this


figure? 212. (1) (2)
(1) 7 and 6 (2) 6 and 7
(3) 6 and 8 (4) 6 and 9
DIRECTIONS (Qs. 209 – 211) : In these questions, there are four
problem figures (a), (b), (c) and (d). One of them is, however, (3) (4)
missing. You have to select one figure from the five answer figures,
(1), (2), (3), (4) and (5) such that the series is completed. Indicate
your answer as per the “Instructions”.

209. Problem figures:


213. (1) (2)

?
(a) (b) (c) (d)
Answer figures:

(3) (4)

(1) (2) (3) (4) (5)


Exemplar Practice Questions M-17
DIRECTIONS (Qs. 214 - 216) : In the following figure, the solid 224. B D G K ? V
cube is painted on all sides by a single colour. Observe the given (1) N (2) P
solid and choose the correct alternative for the following (3) Q (4) M
questions. 225. 41, 31, ?, 17, 11, 5
(1) 19 (2) 21
(3) 23 (4) 27
226. 8, 15, 28, 53, ?
(1) 106 (2) 98
(3) 100 (4) 102
227. 336, 210, 120, ?, 24, 6, 0
(1) 40 (2) 50
214. Find the total number of blocks whose three surfaces are (3) 60 (4) 70
coloured ? 228. 2, 9, 28, ?, 126, 217
(1) 8 (2) 12 (1) 36 (2) 42
(3) 18 (4) 24
(3) 56 (4) 65
215. Find total number of blocks whose two surfaces are coloured.
(1) 0 (2) 8 229. 2, 12, 36, 80, 150, ?
(3) 16 (4) 24 (1) 194 (2) 210
216. Find total number of blocks whose only one surface is coloured. (3) 252 (4) 258
(1) 8 (2) 12
(3) 16 (4) 24 DIRECTIONS (Qs. 230 - 233): In each of the following number
series, a wrong number is given. Find out that number.
DIRECTIONS (Q. 217) : Find the correct mirror is for the following
figures from the alternatives. 230. 5 10 17 27 37 50 65
(1) 10 (2) 17
217. D (3) 27 (4) 37
231. 108 54 36 18 9 6 4
(1) 54 (2) 36
(1) D (2) D (3) 18 (4) 9
232. 2 3 5 8 14 23 41 69
(3) D (4)
D (1) 5 (2) 8
DIRECTIONS (Qs. 218 - 229) : Complete the series. (3) 6 9 (4) 41
218. AZ, CX, FU, ....... 233. 0 1 9 36 99 225 441
(1) IR (2) IV (1) 9 (2) 36
(3) JQ (4) KP (3) 99 (4) 225
219. 3F, 6G, 11I, 18L, ....
DIRECTIONS (Qs. 234 - 239) : Complete the series
(1) 21O (2) 25N
(3) 27P (4) 27Q 234. ab – –a – dcacb – acd –
220. EJO, TYD, INS, XCH, ? (1) dcbdb (2) cdbbd
(1) NRW (2) MRW (3) dabdb (4) cdbdb
(3) MSX (4) NSX 235. 1, 1, 2, 4, 3, 27, 4 ––, —
221. J2Z, K4X, I7V, ?, H16R, M22P (1) 5, 64 (2) 64, 81
(1) I11T (2) L11S (3) 64, 5 (4) 16, 5
(3) L12T (4) L11T 236. m n o n o p q o p q r s –– –– –– –– ––
222. B Y C X D W E ? (1) mnopq (2) oqrst
(1) S (3) T (3) pqrst (4) qrstu
(2) U (4) V 237. 3, 5, 9, 16, 22, 30, 41, ?
223. 3, 8, 35, 48, ?, 120 (1) 49 (2) 50
(1) 64 (2) 72 (3) 51 (4) 52
(3) 80 (4) 99
EBD_7332
M-18 NTSE Stage 1 Question Bank
238. If the positions of the third and tenth letters of the word DIRECTIONS (Qs. 247 - 256) : In each of the following questions,
DOCUMENTATION are interchanged, and likewise the po- four terms/words have been given, out of which three are alike in
sitions of the fourth and seventh letters, the second and some manner and one is different. Choose the odd one out.
sixth letters is interchanged, which of the following will be
eleventh from the right end ? 247. (1) Atom : Electron (2) Train : Engine
(3) House : Room (4) Curd : Milk
(1) C (2) I
248. (1) Crime : Punishment (2) Judgment : Advocacy
(3) T (4) U (3) Enterprise : Success (4) Exercise : Health
239. Select the combination of numbers so that letters arranged 249. (1) Broad : Wide (2) Light : Heavy
accordingly will form a meaningful word. (3) Tiny : Small (4) Big : Large
R A C E T 250. (1) Day - Night (2) Clever - Foolish
1 2 3 4 5 (3) Clear - Blurred (4) Arrive - Come
(1) 1, 2, 3, 4, 5 (2) 3, 2, 1, 4, 5 251. (1) 001011 (2) 1101011
(3) 5, 2, 3, 4, 1 (4) 5, 1, 2, 3, 4 (3) 101101 (4) 100101
252. (1) 3,4, 8 (2) 6, 2, 9
DIRECTIONS (Qs. 240 - 241) : Arrange the given words in the (3) 1, 5, 7 (4) 2, 6, 9
sequence in which they occur in the dictionary and then choose 253. (1) 3, 7, 5 (2) 2, 8, 6
the correct sequence. (3) 7, 9, 5 (4) 3, 8, 2
240. 1. Page 2. Pagan 3. Palisade 4. Pageant 254. (1) 13, 50, 37 (2) 23, 39, 40
5. Palate (3) 18, 38, 44 (4) 74, 10, 16
255. (1) BCD (2) NPR
(1) 1, 4, 2, 3, 5 (2) 2, 4, 1, 3, 5
(3) KLM (4) RST
(3) 2, 1, 4, 5, 3 (4) 1, 4, 2, 5, 3
256. (1) PRT (2) MOQ
241. 1. Wrinkle 2. Wriggle 3. Writhe 4. Wretch (3) GEC (4) TVX
5. Wrath DIRECTIONS (Qs. 257 - 269) : Out of the four choices given for
(1) 4, 5, 1, 2, 3 (2) 5, 4, 2, 1, 3 each question, you have to select one that will maintain the
(3) 4, 2, 5, 1, 3 (4) 5, 2, 1, 3, 4 relationship on the two sides of the sign : : the same if it is
substituted for the question mark ‘?’
DIRECTIONS (Qs. 242 - 244) : Find which one word cannot be
made from the letters of the given word. 257. Food : Hungry :: ? : ?
(1) Thought : Politics
242. TEACHERS (2) Water : River
(1) REACH (2) CHAIR (3) Rest : Weary
(3) CHEER (4) SEARCH (4) Wine : Intoxication
243. CONTEMPORARY 258. Jute : Cotton : Wool : ?
(1) PARROT (2) COMPANY (1) Terylene (2) Silk
(3) CARPENTER (4) PRAYER (3) Rayon (4) Nylon
244. REFRIGERATE 259. Basic : Pascal : Fortran : ?
(1) REFER (2) REGRET (1) Cyclotrone (2) Computer
(3) REGENERATE (4) FREE (3) Cobol (4) Bhopal
DIRECTIONS (Qs. 245 - 246) : Each of the following questions 260. 6 : 18 :: 4 : ?
is based on the following alphabet series. (1) 2 (2) 6
(3) 8 (4) 16
A BCD E FGHI JKLM NOP QRST UVWX YZ
261. 583 : 293 :: 488 : ?
245. If the english alphabet are divided into two equal halves – (1) 291 (2) 378
from A to M and N to Z, which letter in the later half would be (3) 487 (4) 581
corresponding to letter J? 262. Neck is related to Tie in the same way as Waist is related to–
(1) Q (2) V (1) Watch (2) Belt
(3) X (4) W (3) Ribbon (4) Shirt
246. Which letter is midway between 22nd letter from the left and 263. ACFJ is related to ZXUQ in the same way as EGJN is re-
21st letter from the right? lated to –
(1) L (2) M (1) DBYU (2) VTQM
(3) O (4) N (3) VTRP (4) VUSQ
Exemplar Practice Questions M-19
264. ‘JKLM’ is related to ‘XYZA’ in the same way as ‘NOPQ’ is 273. Which set of numbers will represent the word “TIME” ?
related to (1) 23, 66, 01, 02 (2) 43, 11, 55, 66
(1) RSTU (2) YZAB (3) 44, 32, 61, 72 (4) 51, 63, 70, 90
(3) DEFG (4) BCDE 274. Which set of numbers will represent the word “ERODE”?
265. ‘Engineer’ is related to ‘Machine’ in the same way as (1) 66, 10, 76, 02, 85 (2) 22, 55, 41, 62, 21
‘Doctor’ is related to (3) 23, 01, 65, 10, 03 (4) 65, 11, 01, 76, 44
(1) Hospital (2) Body 275. In a class of 60, where girls are twice that of boys, Kamal
(3) Disease (4) Medicine ranked seventeenth from the top. If there are 9 girls ahead of
266. If VISHAL is coded as 22102111517, then what will be the Kamal, how many boys are after him in rank ?
code for SACHIN ? (1) 3 (2) 7
(1) 1925311191 (2) 1295111319 (3) 12 (4) 23
(3) 1925111319 (4) 1952111319 276. The positions of the first and the second digits in the number
267. In a certain code, 3456 is coded as ROPE, 15526 is coded as 94316875 are interchanged. Similarly, the positions of the
APPLE. Then how is 54613 coded as ? third and fourth digits are interchanged and so on. Which of
(1) POEAR (2) PROEA the following will be the third to the left of the seventh digit
from the left end after the rearrangement ?
(3) PEORA (4) RPOEA
268. In a certain code if FRIEND is written as DNEIRF. Then, (1) 1 (2) 4
what will be the code for DESERT ? (3) 6 (4) None of these
(1) TRESED (2) DSERET 277. How many 5s are there in the following sequence which are
immediately followed by 3 but not immediately preceded
(3) TRSEED (4) TESERD
by 7?
269. In a certain code, if AFFAIR is FAAFRI, then FERRARIS
89 5325 3855 6873 3577 5365 3357 38
is coded as ?
(1) One (2) Two
(1) EFRRARIS (2) EFRRRASI
(3) Three (4) Four
(3) EFRRRAIS (4) EFRRARSI
278. Anmol finds that he is twelfth from the right in a line of boys
DIRECTIONS (Qs. 270 - 272) : In a certain code, ‘il be pee’ means and fourth from the left, how many boys should be added to
‘roses are blue’, ‘silk hee’ means ‘red flowers’ and ‘pee mit hee’ the line such that there are 35 boys in the line?
means ‘flowers are vegetables. (1) 19 (2) 13
270. How is ‘red’ written in that code ? (3) 14 (4) 20
(1) hee (2) silk 279. If the day before yesterday was Thursday, when will be
(3) be (4) cannot be determined Sunday?
271. How is ‘roses’ written in that code ? (1) Today (2) Two days after today
(1) il (2) pee (3) Tomorrow (4) Day after tomorrow
(3) be (4) cannot be determined 280. Rakesh is on 9th position from upwards and on 38th position
272. How is ‘vegetables are red flowers’ written in this from downwards in a class. How many students are in class?
code ? (1) 47 (2) 45
(1) pee silk mit hee (2) silk pee hee be
(3) 46 (4) 48
(3) il silk mit hee (4) none
281. Raman is 7 ranks ahead of Suman in a class of 39. If Suman's
DIRECTIONS (Qs.273 - 274) : Given below are two matrices rank is seventeenth from the last. What is Raman's rank from
containing two classes of letters. The rows and columns of matrix the start ?
I are numbered from 0 to 4 and that of Matrix II from 5 to 9. A letter (1) 14 th (2) 15 th
from these matrices can be represented first by its row number (3) 16 th (4) 17 th
and next by its column number.
282. Deepak said to Nitin, " That boy playing football is the
younger of the two brothers of the daughter of my father's
0 1 2 3 4 5 6 7 8 9 wife " How is the boy playing football related to Deepak ?
0 T N D R I 5 M W O F E (1) Son (2) Brother
1 R I T N D 6 F E M W O (3) Cousin (4) Niece
2 N D R I T 7 W O F E M 283. Pointing out to a photograph, a man tells his friend, " she is
the daughter of the only son of my father's wife. How is the
3 I T N D R 8 E M W O F
girls related to the man in the photograph?
4 D R I T N 9 O F E M W
(1) Daughter (2) Cousin
Matix I Matrix II
(3) Mother (4) Sister
EBD_7332
M-20 NTSE Stage 1 Question Bank
284. If S – T means 'S' is the wife of 'T' is S + T means 'S' is the 292. If 3 + 5 = 16 ; 7 + 9 = 64 ; 10 + 12 = 121, then 11 + 3 = ?
daughter of 'T' and S T means 'S' is the son of 'T' What (1) 56 (2) 48
will M + J K means ?
(3) 49 (4) 196
(1) 'K' is the father of 'M'
(2) M' is the grand daughter of K 293. If 36 × 92 = 9623 ; 25 × 82 = 8522; 68 × 75 = 7856, then
(3) 'J' is wife of 'K' 47 × 52 = ?
(4) 'K' and 'M' are brothers (1) 5742 (2) 5274
285. E is the son of A. D is the son of B. E is married to C. C is B's (3) 7427 (4) 5724
daughter. How is D related to E? 294. If A + B = 2C and C + D = 2A, then
(1) Brother
(2) Uncle (1) A + C = B + D (2) A + C = 2D
(3) Father-in-law (3) A + D = B + C (4) A + C = 2B
(4) Brother-in-law 295. Raj travelled from a point X straight to Y at a distance of 80
286. A is the brother of B, C is the brother of A . To establish a metres. He turned right and walked 50 metres, then again
relationship between B & C, which of the following turned right and walked 70 metres. Finally, he turned right
information is required. and walked 50 metres. How far is he from the starting point
I Sex of C II. Sex of B (1) 10 metres (2) 20 metres
(1) Only I is required (3) 50 metres (4) 70 metres
(2) Only II is required
296. Sobha was facing East. She walked 20 metres. Turning left
(3) Both I and II are required she moved 15 metres and then turning right moved 25 metres.
(4) Neither required Finally, she turned right and moved 15 metres more. How far
DIRECTIONS (Qs.287 - 288) : Study the following information is she from her starting point?
and answer the questions given below. (1) 25 metres (2) 35 metres
A + B means ‘A is father of B’ (3) 50 metres (4) 45 metres
A – B means ‘A is wife of B’ 297. If a person is walking towards North, what direction should
A × B means ‘A is brother of B’ he follow so that he is walking towards West ?
A ÷ B means ‘ A is daughter of B’ (1) right, right, left (2) left, left, right
287. P R + S + Q, which of the following is true ? (3) left, right, left (4) left, left, left
(1) P is daughter of Q 298. Two friends start a race, and together they run for 50 mts.
(2) Q is aunt of P Jack turns right and runs 60 mts while Bunny turns left and
runs 40 mts. Then Jack turns left and runs 50 mts while
(3) P is aunt of Q
Bunny turns right and runs 50 mts. How far are the two
(4) P is mother of Q friends now from each other
288. If P – R + Q, which of the following is true (1) 60 mts (2) 20 mts
(1) P is mother of Q (3) 100 mts (4) 150 mts
(2) Q is daughter of P 299. From her home, Prerna wishes to go to school. From home,
(3) P is aunt of Q she goes towards North and then turns left and then turns
(4) P is sister of Q right, and finally she turns left and reaches school. In which
289. It being given that :> denotes +, < denotes –, + denotes ÷, – direction her school is situated with respect to her home?
denotes =, = denotes ‘less than’ and × denotes ‘greater than’ (1) North-East (2) North-West
, find which of the following is a correct statement: (3) South-East (4) South-West
(1) 3 + 2 > 4 = 9 + 3 < 1 300. One morning after sunrise, Reeta and Kavita were talking to
(2) 3 > 2 > 4 = 18 + 3 < 2 each other face to face at Tilak Square. If Kavita’s shadow was
(3) 3 > 2 < 4 × 8 + 4 < 2 exactly to the right to Reeta, which direction Kavita was facing?
(4) 3 + 2 < 4 × 9 + 3 < 3 (1) North (2) South
290. If 20 – 10 means 200, 8 ÷ 4 means 12, 6 × 2 means 4 then (3) East (4) Data inadequate
100 – 10 × 1000 ÷ 1000 + 100 × 10 = ? 301. A man leaves for his office from his house. He walks towards
(1) 0 (2) 20 East. After moving a distance of 20 m, he turns south and
(3) 1000 (4) 1900 walks 10 m. Then he walks 35 m towards the west and further
291. If × means ÷, – means × , ÷ means + and + means – then 5 m towards the North. He then turns towards east and walks
15 m. What is the straight distance (in metres) between his
(3 – 15 ÷ 19) × 8 + 6 = ?
initial and final positions ?
(1) – 1 (2) 2
(1) 0 (2) 5
(3) 4 (4) 8
(3) 10 (4) Cannot be determined
Exemplar Practice Questions M-21
313. A watch which gains uniformly, is 5 min. slow at 8 o’clock in
DIRECTIONS (Qs. 302 - 304) : Choose the Venn diagram which
the morning on Sunday and it is 5 min. 48 sec. fast at 8 p.m.
best illustrates the three given classes in each questions:
on following Sunday. When was it correct ?
(1) 7pm on Wednesday
(2) 20 min. past 7 pm on Wednesdays
(3) 15 min past 7 pm on Wednesdays
(1) (2) (3) (4) (5) (4) 8 pm on Wednesdays
314. A clock is set right at 1 p.m. If it gains one minute in an hour,
302. Citizens, Educated, Men then what is the true time when the clock indicates 6 p.m. the
303. Sun, Moon, Stars same day ?
304. Mercury, Mars, Planets
5
DIRECTIONS (Qs. 305 - 314) : In the following diagram, three (1) 55
minutes past 5
6
classes of population are represented by three figures. The triangle (2) 5 minutes past 6
represents the school teachers, the square represents the married (3) 5 minutes to 6
persons and the circle represents the persons living in joint families.
1
(4) 59 minutes past 5
64
F DIRECTIONS (Qs.315 - 324): What number/character/letter
should replace the question mark?
E
B C
315.
7 8 9 3 8 5
D A
36 17 ?
305. Married persons living in joint families but not working as 4 5 2 6 3
5
teachers are represented by
(1) C (2) F
(1) 18 (2) 22 (3) 36 (4) 19
(3) D (4) A
306. Persons who live in joint families, are unmarried and who do 316. 7 2 9 4
not work as teachers are represented by
(1) C (2) B 10 ?
(3) E (4) D
307. Married teachers living in joint families are represented by 9 4 3 6
(1) C (2) B
(1) 2 (2) 9 (3) 12 (4) 19
(3) D (4) A
308. Between 2 O’clock to 10 O’clock, how many times the hands 26 5 37
of a clock are at right angle ?
317. 35 ? 30 15 2 17 12 6 18
(1) 14 (2) 12
(3) 16 (4) 15 24 3 35
309. On January 12, 1980, it was Saturday. The day of the week (1) 4 (2) 5 (3) 6 (4) 7
on January 12, 1979 was – 318. 3 2 2
(1) Saturday (2) Friday
(3) Sunday (4) Thursday 7 84 8 5 121 11 5 ? 6
310. The number of odd days in a leap year is –
(1) 1 (2) 2 5 11 7
(3) 3 (4) 4 (1) 28 (2) 36 (3) 48 (4) 42
311. How many times in a day, the two hands of a clock 319.
coincide ? 3C 2B 4A
(1) 11 (2) 12
(3) 22 (4) 24 27A ? 64B
312. What will be the acute angle between hands of a clock
at 2 : 30? 9C 4A 16B
(1) 105° (2) 115°
(3) 95° (4) 135° (1) 8C (2) 12 C (3) 16 C (4) 18 C
EBD_7332
M-22 NTSE Stage 1 Question Bank
326. Problem Figures
18 32
320. 12
30 16 40

?
6 8
(A) (B) (C) (D)
Answer Figures
18
36 27

(1) (2) (3) (4)


? 327. Problem Figures
(1) 6 (2) 9 (3) 12 (4) 18

321.

49 26
36

64 81
9

21 25 64
25

? 144
?
(A) (B) (C) (D)
25 16 36
(1) 19 (2) 23 (3) 25 (4) 31
322. 101 43 48 34
38 ?
35 15 56 184 (1) (2) (3) (4) (5)
A B
(1) 127 (2) 142 (3) 158 (4) 198 DIRECTIONS (Qs.328 – 330) : Each of the following questions
consists of five problem figures marked A, B, C, D and E. From the
1 4 3 6
323. 9 5 5 8 3 four answer figures marked 1, 2, 3, and 4, select a figure which will
551 246 ? continue the series established by the five problem figures.
3 4 7 2 328. Problem Figures
6 8 9 4
1
A B C
(1) 262 (2) 622 (3) 631 (4) 824

324. Z ? S
R O ?
(A) (B) (C) (D) (E)
? G C
Answer Figures
(1) WJK (2) KWT (3) WKJ (4) JKW

DIRECTIONS (Qs.325 – 327) : In the Problem Figures, one figure


marked by ? is missing. There is a set of answer figures also in
which five alternatives are given. You have to find out the one
right answer from answer figures. (1) (2) (3) (4)
325. Problem Figures 329. Problem Figures

?
(A) (B) (C) (D) (E)
(A) (B) (C) (D)
Answer Figures
Answer Figures

(1) (2) (3) (4) (1) (2) (3) (4)


Exemplar Practice Questions M-23
330. Problem Figures 336. Problem Figures

(A) (B) (C) (D) (E)


?
(A) (B) (C) (D)
Answer Figures Answer Figures

(1) (2) (3) (4)


(1) (2) (3) (4)
DIRECTIONS (Qs.331 - 334): Choose the figure which is different
from the others. 337. Problem Figures

331. Problem figures

?
(A) (B) (C) (D)
(1) (2) (3) (4) Answer Figures
332. Problem figures

(1) (2) (3) (4)


(1) (2) (3) (4) 338. Problem Figures
333. Problem figures

(1) (2) (3) (4) (A) (B) (C) (D)


334. Problem figures Answer Figures

(1) (2) (3) (4)


DIRECTIONS (Qs.335 - 339) : Each of the following questions (1) (2) (3) (4)
consists of two sets of figures. Figures A, B, C and D constitute 339. Problem Figures
the problem set while figures 1, 2, 3 and 4 constitute the answer
set. There is a definite relationship between figures A and B.
Establish a similar relationship between figures C and D and
choose the one figure from answer set as alternative letter.
335. Problem Figures
(A) (B) (C) (D)

? Answer Figures

(A) (B) (C) (D)


Answer Figures

(1) (2) (3) (4) (5)


(1) (2) (3) (4)
EBD_7332
M-24 NTSE Stage 1 Question Bank
DIRECTIONS (Qs.340 - 342) : In each of the following questions, 345.
select a figure from the four alternatives, which when placed in the
space where the question mark is shown in figure (X) would
complete the figure.
?
340.
X (1) (2) (3) (4)
DIRECTIONS (Qs.346 – 350) : Find the correct option for the
mirror images for the following questions.

(X)
346. jealous ?

(1) (2)
(1) (2) (3) (4) (3) (4)
341.
347. Rotate the mirror image 90° anticlockwise.

72

(X)
(1) (2)

(3) (4)

(1) (2) (3) (4)


348. ?
342.

(1) (2)

(3) (4)
(X)
349. Rotate the mirror image 90° anticlockwise.

ANT
(1) (2) (3) (4)
DIRECTIONS (Qs.343 - 345) : In each of the following questions,
A

select a figure from amongst the four alternatives, which when (1) (2)
placed in the blank space of fig. (X) would complete the pattern.

343. (3) (4)

(X) (1) (2) (3) (4) 350. A N S 4 3 Q 1 2


(1) (2)
344.
(3) (4)
(X) (1) (2) (3) (4)
Exemplar Practice Questions M-25
DIRECTIONS (Qs. 351 - 355) : In each of the following questions, DIRECTIONS (Qs. 356 - 357) : A cube painted red on two adjacent
choose the correct mirror-image of the Fig. (X) from amongst the faces and black on the faces opposite to the red faces and green
four alternatives (1), (2), (3) and (4) given along with it. on the remaining faces is cut into sixty-four smaller cubes of equal
size.
351.
356. How many cubes are there which have no face painted –
(1) 0 (2) 4 (3) 8 (4) 16
357. How many cubes have only one face painted –
(1) 8 (2) 16 (3) 24 (4) 32
(X) DIRECTIONS (Qs. 358 - 359): Count the number of cubes in
each of the following figures:
358.

(1) (2) (3) (4)


352. Looking into a mirror, the clock shows 9 : 30 as the time. The
actual time is
(1) 2 : 30 (2) 3 : 30
(3) 4 : 30 (4) 6 : 30 (1) 64 (2) 66 (3) 68 (4) 70
353. Select from the alternative, the box that can be formed by
folding the sheet shown in figure (X) : 359.

+
(1) 57 (2) 58 (3) 60 (4) 62
(X) DIRECTIONS (Qs.360 - 361): In each one of the following
questions, find from amongst the four response figures, the one
+ which resembles the pattern formed when the transparent sheet,
+ carrying a design, is folded along the dotted line.
(1) (2) (3) (4)
360. Transparent Sheet
(1) A only (2) A, B, and C only
(3) B and C only (4) A, B, C and D
354. Count the number of cubes in the given figure.

Response Figures

(1) 14 (2) 12
(3) 10 (4) 8
355. Which number lies opposite the face 4, if the four different
positions of a dice are as shown in the figures given below.
(1) (2)
4 2 1 2

6 5 5 1 5 6 4 3

(i) (ii) (iii) (iv)


(1) 5 (2) 3 (3) (4)
(3) 2 (4) 1
EBD_7332
M-26 NTSE Stage 1 Question Bank
361. Transparent Sheet

(1) (2)
Response Figures

(3) (4)
(1) (2)
DIRECTIONS (Qs.364 - 365) : In each of the following questions
a set of three figures A, B and C showing a sequence of folding of
a piece of paper. Fig. (C) shows the manner in which the folded
paper has been cut. These three figures are followed by four
answer figures from which you have to choose a figure which
would most closely resemble the unfolded form of fig. (C).
(3) (4)
DIRECTIONS (Qs.362 - 363) : Consider the three figures, marked 364.
X, Y, and Z showing one fold in X, another in Y and the cut in Z.
From amongst the four alternative figures 1, 2, 3 and 4, select the
one showing the unfolded position of Z. (A) (B) (C)

362.

(1) (2) (3) (4)

365.
X Y Z B

(A) (B) (C)


B B

B B B B B
B B B

(1) (2)
B B B B
B
B
B B
(1) (2) (3) (4)

DIRECTIONS (Qs. 366 - 368) : In each of the following questions,


you are given a figure (X) followed by four alternative figures (1),
(2), (3) and (4) such that fig. (X) is embeded in one of them. Trace
out the alternative figure which contains fig. (X) as its part.
(3) (4)
366.
363.

X Y Z
(X)
Exemplar Practice Questions M-27
371. What is F’s profession?
(1) Doctor (2) Engineer
(3) Teacher (4) Data inadequate
DIRECTIONS (Qs.372 - 374) : Study the following information
carefully and answer the questions given below it.
In a group of five persons A, B, C, D and E :
(1) (2) (3) (4) 1. B and C are intelligent in Mathematics and Geography
367. 2. A and C are intelligent in Mathematics and History
3. B and D are intelligent in Political Science and Geography.
4. D and E are intelligent in Political Science and Biology.
5. E is intelligent in Biology. History and Political Science.
372. Who is intelligent in Political Science, Geography and
Biology ?
(X) (1) E (2) D
(3) C (4) B
373. Who is intelligent in Mathematics, Political Science and
Geography ?
(1) A (2) B
(3) C (4) D
374. Who is intelligent in Mathematics and History but not in
(1) (2) (3) (4) Geography ?
(1) C (2) E
368.
(3) A (4) B
DIRECTIONS (Qs.375 - 379): Read the following information
and answer the questions given below it.

Six students A, B, C, D, E and F are sitting in the field. A and B are


from Nehru House while the rest belong to Gandhi House. D and
(X) F are tall while the others are short. A, C and D are wearing glasses
while the others are not.
375. Which two students, who are not wearing glasses, are short?
(1) A and F (2) C and E
(3) B and E (4) E and F
376. Which short student of Gandhi House is not wearing
(1) (2) (3) (4) glasses?
(1) F (2) E
DIRECTIONS (Qs.369 - 371): Study the following information
(3) B (4) A
carefully and answer the questions given below.
377. Which tall student of Gandhi House is not wearing glasses?
A, B, C, D, E, F and G are travelling in three different vehicles. (1) B (2) C
There are at least two passengers in each vehicle– I, II and III and (3) E (4) F
each vehicle has passengers of both the sexes. There are two 378. What is the number of rectangles (excluding squares) in
engineers, two doctors and three teachers among them. C is a lady figure.
doctor and she does not travel with the pair of sisters A and F. B,
a male engineer, travels with only G, a teacher in vehicle I. D is a
male doctor. Two persons belonging to same profession do not
travel in the same vehicle. A is not an engineer and travels in
vehicle II.
369. In which vehicle does C travel?
(1) I (2) II
(3) III (4) II and III (1) 10
370. How many lady members are there among them? (2) 12
(1) Three (2) Four (3)
(3) Three or Four (4) Data inadequate (4) 22
EBD_7332
M-28 NTSE Stage 1 Question Bank
379. What is the number of triangles in figure ? DIRECTIONS (Qs.382 - 384) : In each of the following questions,
from amongst the figures marked (1), (2), (3) and (4), select the one
which satisfies the same conditions of placement of the dot as in
fig. (X).
382.

(1) 24 (2) 32
(3) 48 (4) 52
DIRECTIONS (Qs.380 - 381) : In each of the following questions
there is a diagram (X) in which one or more dots have been placed (X) (1) (2) (3) (4)
in certain positions. Examine the placement of these dots carefully.
From the four choices, select the one in which the placement of 383.
dots is similar to that in the diagram.

(X) (1) (2) (3) (4)


380.
384.

(X) (X) (1) (2) (3) (4)


DIRECTIONS (Qs.385 - 387): In the following addition each of
the letters denote a different integer. Each letter stands for the
same integer throughout where ‘P’ stands for 4.
(1) (2) M N O P
+ A Q R P
Z Z Z Z

385. If the value of N is 6 and M > N, then what is the value of A?


(1) 2 (2) 1
(3) 3 (4) None of these
(3) (4)
386.. If N = 6 and M > N, then which of the following numbers
stand for M A N ? If N = 6, M > N and O = 5, then
(1) 146 (2) 716
381. (3) 356 (4) 236
387. What is the value of POAR ?
(1) 4513 (2) 4514
(3) 4517 (4) 4615

DIRECTIONS (Qs.388 - 390):


Given

(X) A B C D
+ S R O P
C C C C

(1) (2) where C = 9.


388. Based on the data derived from Questions 8 & 9, what is the
possible value of CAR ?
(1) 342 (2) 375
(3) 923 (4) Cannot be determined
389. If A = 2 and R is a prime number smaller than 5, then find. The
(3) (4) value of SCAR, using all the data from above ?
(1) 4752 (2) 2973
Exemplar Practice Questions M-29
(3) 7021 (4) Cannot be determined (3) B C F (4) B C F
390. The value of SOAP, using all the data from above
DIRECTIONS (Qs.397 - 399) : In each of the following questions
&D=8?
below are given 3 statements followed by three conclusions
(1) 7321 (2) 7021
numbered I, II & III. You have to take the given statements to be
(3) 2379 (4) 8109
true even if they seem to be at variance from the commonly known
Common Data for (Qs.391 - 392): facts. Read all the conclusions and then decide which of the given
Given conclusions logically follow from the given statements
disregarding commonly known facts.
B A N A
N A N A 397. Statements: All tree are flowers. No flower is fruit. All
branches are fruit.
V L W L
Conclusions:
where A = 2 and N = 3. I. Some branches are trees
II. No fruit is tree.
391. What will be the value of B, if V is a prime number ?
III. No tree is branch
(1) 6 (2) 4
(1) None follows
(3) 5 (4) 7
(2) Only either I or III follows.
392. What is the probable value of V ?
(3) Only II follows
(1) 6 (2) 4
(4) Both II & III follow.
(3) 7 (4) 5
398. Statements: All tigers are jungles. No jungle is bird.
DIRECTIONS (Qs.393 - 396) : Each question given below Some birds are rains.
consists of 5 or 6 statements followed by options consisting of 3 Conclusions:
statements put together in a specific order. Choose the option I. No rain is jungle.
which indicates a valid argument containing logically related II. Some rains are jungle.
statements that is, where the third statement is a conclusion drawn III. No bird is tiger.
from the preceding two statements. (1) Only I & II follows.
(2) Only III follows
393. A. All balls are locks (3) Only either I or II, and III follows
B. All keys are locks (4) All follows
C. All keys are balls 399. Statements: Some mountains are hillocks.
D. Some keys are locks Some mountains are rivers.
E. Some locks are balls Some mountains are valleys.
F. No balls is lock Conclusion:
(1) AC B (2) B E F I. All mountains are either hillocks or rivers or valleys
(3) CD B (4) C E F II. No valley is river
394. A. All cushions are pillows. III. Some river are valleys
B. Some pillows are sheets. (1) None follows
C. Some pillows are cushions. (2) Only I follows
D. Some pillows are not cushions. (3) Only either II or III follows.
E. All pillows are sheets (4) Only III follows
F. No pillows are sheets.
(1) ABE (2) B C D DIRECTIONS (Qs. 400 - 402) : In each question below are given
(3) EAC (4) E D C two statements followed by two conclusions numbers I & II. You
395. A. All heroines are pretty. have to take the given two statements to be true even if they seem
B. Some heroines are popular girls. to be at variance from commonly known facts. Read the conclusions
C. Shweta is pretty. and then decide which of the given conclusions logically follows
D. Shweta is a popular heroine. from the two given statements, disregarding commonly known
E. Some popular girls are pretty. facts.
(1) ABE (2) A C D Give answer
(3) DCA (4) E D C (1) If only conclusion I follows
396. A. Some abra are dabra. (2) If only conclusion II follows
B. All abra are cobra (3) If either conclusion I or II follows
C. All dabra are abra (4) If both conclusions I and II follow
D. All dabra are not abra 400. Statements:
E. Some cobra are abra. All pens are roads.
F. Some cobra are dabra All roads are houses.
(1) ABD (2) A E F Conclusions:
I. All houses are pens.
EBD_7332
M-30 NTSE Stage 1 Question Bank
II Some houses are pens. DIRECTIONS (Qs.406 - 407): In each question below is given a
401. Statements: All huts are mansions. statement followed by two assumptions numbered I and II. You
All mansions are temples. have to consider the statement and the following assumptions
Conclusions: and decide which of the assumptions is implicit in the statement.
I. Some temples are huts
II. Some temples are mansions. 406. Statement: It is desirable to put the child in school at the
402. Statements: All pens are chalks. age of 5 or so.
All chalks are chairs. Assumptions:
Conclusions: I. At that age the child reaches appropriate level of
I. Some pens are chairs. development and is ready to learn.
II. Some chalks are pens. II. The schools do not admit children after six years of
age.
DIRECTIONS (Qs.403 - 405): The logic problems in this set (1) Only assumption I is implicit
present you with three true statements: Fact 1, Fact 2, and Fact 3. (2) Only assumption II is implicit
Then, you are given three more statements (labeled I, II, and III), (3) Either I or II is implicit
and you must determine which of these, if any, is also a fact. One (4) Neither I nor II is implicit
or two of the statements could be true; all of the statements could 407. Statement: “In order to bring punctuality in our office, we
be true; or none of the statements could be true. Choose your must provide conveyance allowance to our employees.” -
answer based solely on the information given in the first three In charge of a company tells Personnel Manager.
facts. Assumptions:
403. Fact 1: Jessica has four children I. Conveyance allowance will not help in bringing
Fact 2: Two of the children have blue eyes and two of the punctuality.
children have brown eyes. II. Discipline and reward should always go hand in
Fact 3: Half of the children are girls. hand.
If the first three statements are facts, which of the (1) Only assumption I is implicit
following statements must also be a fact? (2) Only assumption II is implicit
I: At least one girl has blue eyes. (3) Either I or II is implicit
II: Two of the children are boys. (4) Neither I nor II is implicit
III: The boys have brown eyes.
DIRECTIONS (Qs. 408 - 410) : Each of the questions below
(1) I only
consists of a question and two statements numbered (I) and (II)
(2) II only
given below it. You have to decide whether the data provided in
(3) II and III only
the statements are sufficient to answer the question. Read both
(4) None of the statements is a known fact.
the statements and give answer
404. Fact 1: All drink mixes are beverages.
Fact 2: All beverages are drinkable. (1) Statement (I) ALONE is sufficient, but statement (II) alone is
Fact 3: Some beverages are red. not sufficient.
If the first three statements are facts, which of the (2) Statement (II) ALONE is sufficient, but statement (I) alone is
following statements must also be a fact? not sufficient
I. Some drink mixes are red. (3) BOTH statements TOGETHER are sufficient, but NEITHER
II. All beverages are drink mixes. statement alone is sufficient.
III. All red drink mixes are drinkable. (4) EACH statement ALONE is sufficient
(1) I and II only 408. A, B and C are positive integers. Is their product an even
(2) II only number?
(3) I and III only (I) A is an even number.
(4) III only (II) The product of A and B is an even number and that of
405. Fact 1: All chickens are birds. A and C is also an even number.
Fact 2: Some chickens are hens. 409. By selling a product for ` 100 how much profit was earned?
Fact 3: Female birds lay eggs. (I) 20% profit would have been earned if it had been sold
If the first three statements are facts, which of the for ` 90.
following statements must also be a fact? (II) The profit was one–third of the cost price.
I. All birds lay eggs. 410. Who is paternal uncle of P ?
II. Some Hens are birds. (I) P is brother of L, who is daughter of Q, who is sister of
III. Some chickens are not hens. N, who is brother of S.
(1) I only (II) M is brother of K, who is husband of L, who is mother
(2) II only of G, who is sister of P.
(3) II and III only
(4) None of the statements is a known fact.
Exemplar Practice Questions M-31
DIRECTIONS (Qs.411 - 414) : In the chart given below , A, B, C,
D and E represent the costs of paper, printing, binding, officer II Steno
11% 10%
miscellaneous and the royalty, respectively in publishing a book.
officer I Assistant
8% 15%
E A clerk II
15% 20% 20 % supervisor
D 17%
10% clerk I
19 %

B Employee Out of these Percent of


25% Direct promotees
C
30%
Steno 30 70
Assistant 40 60
411. What is the angle of the pie chart showing the expenditure
Supervisor 50 50
incurred on royalty ?
(1) 15° (2) 24° Clerk I 90 10
(3) 48° (4) 54° Clerk II 30 70
412. The marked price of the book is 20% more than the CP. If the Officer I 90 10
marked price of the book is `30, then what is the cost of the Officer II 70 30
paper used in a single copy of the book ?
(1) `6 (2) `5
(3) `4.5 (4) `6.50 415. What is the difference in direct recruit and promotee
413. Which two expenditures together will form an angle of 108° assistants?
at the centre of the diagram ? (1) A and E (1) 210 (2) 280
(2) B and E (3) 180 (4) 110
(3) A and D (4) D and E 416. The promotee clerk I is approximately what per cent of that
414. If the difference between the two expenditures are represented of direct recruit clerk I?
by 18 degrees in the diagram, then these expenditures must (1) 10 (2) 9
be :
(3) 11 (4) 10.50
(1) B and E (2) A and C
(3) A and D (4) D and E 417. How many employees are supervisor?
(1) 1050 (2) 1019
DIRECTIONS (Qs.415 - 418) : Study the following information
to answer the given questions. (3) 1109 (4) 1190
418. How many total direct recruits among all types of employees
Percentage of Different Types of Employees are there?
in an Organization
(1) 4000 (2) 3885
Total Employees = 7000
(3) 3000 (4) 3115
EBD_7332
M-32 NTSE Stage 1 Question Bank

Hints & Solutions


Mental Ability Test (MAT)

1. (4) 1 7. (3) Grand Father


1×4=4 only son
4–2=2
2×4=8 Reena’s Father
8–2=6
6 × 4 = 24
24 – 2 = 22 Reena
22 × 4 = 88
88 – 2 = 86 Man’s Father
2 2 2 2
2. (3) Ist Letter : K I G E C
Man — Brother
2nd Letter :
From above two diagram, it is clearly under stood that
3 3 3 3
M P S V Y Reena’s father and Man’s father is the same. And Reena
is sister to that man.
2 3 5 7
Number : 1 3 6 11 18 8. (4) We have,
23 + 13 + 33 = 8 + 1 + 27
3. (1) Ist Letter:
= 36
L
1
M
1
N
1
O
1
P 03 + 43 + 33 = 0 + 64 + 27
= 91
2nd Letter : Therefore, 43 + 23 + 13
4 4 4 4 = 64 + 8 + 1
X T P L H
= 73.
3rd Letter : 9. (3) The number inside the circle is obtained by multiplying
F
4
J
4
N
4
R
4
V the sum of upper number, the lower number and the
number corresponding to the position of the letter in
4. (1) a b c / a b c / c b a / c b a English alphabet, by the number on the right and then
i.e, c a c a. subtracting the number on the left from the product.
Thus, we have (2 + C + 5) × 3 – 4 = (2 + 3 + 5) × 3 – 4 = 26;
5. (4) G R A N D E U R (4 + 4 + 4) × 5 – 10 = (4 + 8 + 4) × 5 – 10 = – 70
(8 + J + 6) × ? – 6 = 90
96
(8 + 10 + 6) × ? = 96 ? 4
N A R G R U E D 24
10. (4) In the first figure,
Similarly 5 = (5 × 1), 8 = (4 × 2), 4 = (2 × 2)
In the second figure,
S E R P E V R E 6 = (3 × 2), 9 = (3 × 3), 4 = (2 × 2)
Thus, the digits of missing figure are (6 × 1), (7 × 1), (4 × 2)
i.e. 6, 7, 8.
Missing number = 678
P R E S E R V E
36 4 8 4
11. (1)
6. (4) Drink fruit Juice tee see pee 4 8 2 16 1
Replacing sign according to question. We get,
Juice is Sweet see Kee lee 36 4 8 4 32 8 4 4 4
0
4 8 2 16 1 4 4 16 1 256 1
He is intelligent Lee ree mee
12. (4) In every step, opposite pair ( , 0) rotates 90° clockwise.
Hence, code for sweet is Kee. Hence, option (4) is correct.
Exemplar Practice Questions M-33
13. (3) Element ( , 0) shifts anticlockwise in each step. While There is only one square i.e. ABCD composed of eight
shaded square shifts clockwise in each step. Hence, components.
option (3) is correct. These is only one square EFGH composed of twelve
14. (4) The figure may be labelled as shown:- components.
Total number of squares = 4 + 4 + 4 + 4 + 1 + 1 = 18

A E F B 2 km

16. (2) Shrikant starting 4 km 1 km 2 km


position
M Q R N
Y Z S
L G
9 km
X 8 km
A1 B1
K H
W T
P V U O
Shrikant final
position
D J I C
Distance from original position = (6)2 (8)2
The simplest squares are EFRQ, MQYZ, RNSZ, XYA,W,
QRZY, LXWK, ZSTB1, SGHT, WAVP, A B UV, B1TOU = 36 64 100 10 km .
and VUIJ i.e. 13 in number. 17. (2)
The squares having two components each are KA JD, 18. (1)
B, HCI, FBGZ and AEYL i.e. 4 in number. 19. (1) signs , , , are adjacent to sign.
The squares having four components each are MRB
W, YSOV, XZUP and QNTA, i.e. 4 in number. Hence, sign will be opposite to surface having
The squares having seven components each are YGCJ,
sign.
LZID, EBHA, and AFB, K i.e; 4 in number.
There is only one square i.e. MNOP composed of nine 20. (1)
components.
There is only one square i.e. ABCD composed of
seventeen components.
There are (13 + 4 + 4 + 4 + 1 + 1) = 27 squares. 21. (4) A is opposite to D.
F is opposite to B.
15. (3) The figure may be labelled as shown:- E is opposite to C.
Therefore, option (4) pair is not relavant to the above
W X given conditions.
O P
E B F 22. (4)
V I J Y
NA M CQ
U R
L K Z
C1
H D G
T S
B1 A1
23. (2) Sohan was born on 25th Feb. 2012. That means Hasan
The squares composed of two components each are was born on 29th Feb 2012 ( 2012 is a leap year).
AIML, CKMJ, BJMI and DLMK i.e. 4 in number. If 26th January in that year falls on Thursday. Then
The squares composed three components each are from 26th Jan. to 29 Feb these is total 34 days.
AMDH, MCGD, BFCM and EBMA i.e. 4 in number.
34
The squares composed of four components each are 6 odd days
7
VWBA, XYCB, ZADC, and B C AD i.e. 4 in number.
Thursday + 6 = Wednesday
The square composed of seven components each are
Hence, Hasan’s birthday follow Wednesday.
NOJL, TUIK, RSLJ and PQKI i.e. 4 in number.
EBD_7332
M-34 NTSE Stage 1 Question Bank
: 32 : : 30. (4) As, crosswise –
24. (4) 8 27 : 243
43 + 10 = 74, 63 + 10 = 226
3 5 3 5
13 + 10 = 11, 33 + 10 = 37
2 2 3 3
25. (4) As Similarly, 23 + 10 = 18

23 21 19 17 4 6 8 10 2
W U S Q D F H J 5 10 = 135
opposite letter Sol. for (31 and 32):
opposite letter
opposite letter Mosque O
opposite letter C
4 km
Similarly,
A 5 km B
13 11 9 7 14 16 18 20
M K I G N P R T 4 km
opposite letter
opposite letter House
opposite letter
opposite letter 31. (1) Distance between house to mosque = 4 + 4 = 8 km.
32. (3) According to the above diagram mosque is to the north
26. (2) The appropriate set of signs which make the equation of kareem's house.
meaningful is in option (2) 33. (4)
i.e., 10 × 10 10 = 100 10 34. (2)
27. (4) The parallelograms are– 35. (1) From question.
IJQP, JKLQ, PQNO, QLMN,
ABHS, BCDH, SHFG, HDEF, 2 × 2 daughter
2 daughter
IKLP, PLMO, ACDS, SDEG, Rajan + Kamala 3 × 1 daughters
IJNO, JKMN, ABFG, BCEF, 3 sons
3 × 2 sons
AVTE, UISL, ISER, IKMO
ACEG, ISTV, ULTV, ULER Total female members = 1 + 2 + 4 + 3 = 10
Hence total number of parallelograms = 24. 36. (1) Total number of cinema and serial actors only
= 15 + 10 – 5 = 20
R F E
G 37. (2) Total serial actors
V D = 10 + 5 + 10 + 10 = 35
H T
C 38. (3) As
A B
10 4 15 7
10 + 7 , 4 + 7 + 15 + 7 + 7 + 7
J D O G
= 47
N M
O U Similarly,
P L
Q 13 9 3 5
K 13 + 7 + 9 + 7 + 3 + 7 + 5 + 7 = 58
I J S M I C E
28. (3) According to the figure there are 12 semicircles. 39. (4) From question.
Geetha > Seetha > Deepthi
(2 years) (3 years)
and Shankar > Deepthi
(1 year)
12 11
10 9 Geetha > Seetha > Shankar > Deepthi
Hence, Deepthi is the youngest among all.
1 35 7 40. (3) From question,
2 64 8
Deepthi is 5 years younger than Geetha
29. (2) In the squares on the corners– Age of Geetha = 6 + 4 = 11 years
Product of left two numbers = product of right two Sol. for (41 to 43) :
numbers. Let eq. (1) Sky is blue 48 1
i.e., 4 × 9 = 12 × 3, 8 × 5 = 4 × 10 eq. (2) Sea is deep 24 6
5 × 16 = 8 × 10 9×8=6×?
eq. (3) Sea looks blue 69 8
9 8
? 12. From eq. (1) and (2)
6
Word “is” is common and No. ‘4’ is common
Exemplar Practice Questions M-35
is = 4
1
From eq. (1) and (3) 53. (4) (A + B)’s 1 days’s work =
15
Word “blue” is common and No. “8” is common
blue = 8 1
B’s 1 day’s work =
From eq. (2) and (3) 20
Word “Sea” is common and No. “6” is common
1 1 1
Sea = 6 A’s 1 day’s work =
15 20 60
if is = 4, blue = 8, then sky = 1
So, A alone can do the work in 60 days.
if sea = 6, is = 4, then deep = 2
if sea = 6, blue = 8, then looks = 9 54. (1) Let the radius of circle = r
41. (3) 8 = Blue Let the side of square = a
42. (2) Sea = 6 Given, Area of circle = Area of square
43. (1) Deep = 2 r2 = a2
44. (4) After Rubbing some part of the figure in a sequence. a2
The base and direction will remain same as original. = a:r= :1
r2
45. (1) Figure stretch little bit into right.
55. (3) Age of grandfather = [29 ×5 – 20 × 4]
= 65 years
56. (1) The alphabetical order is–
Praise > Preach > Precede > Precept > Precinct.
46. (1) 57. (3) The alphabetical order is–
47. (4) Seldom > Select > Selfish > Seller > Send.
58. (3) Amrita is 10th from front and 41st from the behind.
48. (1)
Therefore, 15 persons are in between Mukul and Amrita
49. (1) Let the numbers be x, 2x and 3x.
and Mamta will occupy 33rd position from behind.
Then, their H.C.F. = x. So x = 12 Hence, Her front position is 18th.
The numbers are 12, 24 and 36. 59. (2)
50. (3) Let ages of Vivek and Sumit are 2x and 3x respectively. 60. (4) In each step, shaded circle shifts one side ACW while
According to question. After 12 years, their new ratio plus symbol shifts one side CW.
will be 61. (1) In each step, each bar as well as small dots increases
by 1.
2 x 12 11 62. (1)
3 x 12 15 63. (3) Sum of column is 49, 64, 81 respectively.
30x + 180 = 33x + 132 ? + 25 + 9 = 81
33x – 30x = 180 – 132 ? = 81– (25 + 9) = 47
So, missing term is 47.
3x = 48
x = 16 64. (2) 4 25 4 5 20
Hence, age of Sumit = 16 × 3 = 48 yrs. 27 9 27 3 81
51. (4) Ratio of their shares 11 ? 44
= 2000 × 1 : 3000 × 2 : 4000 × 2 = 1 : 3 : 4
44
? 4 ? = 16
1 11
A’s share = ` 3200 ` 400
8 65. (1) 2, 3, 5 and 6 are adjacent to 1. Therefore, number 1 is
52. (3) Short cut Method: opposite to no. 4.

xy 1
Net effect = x + y +
100
66. (4) 2 6 3
Net effect on the sales on the top
5
20 15)
= + 20 – 15 +
100 4
= 5 – 3 = 2% increase.
Therefore, 1 will come in place of question mark.
EBD_7332
M-36 NTSE Stage 1 Question Bank
67. (1) P Q P is the daughter of Q. Statement (II) alone is sufficient to answer the
question.
Q–R Q is the husband of R. 79. (1) Since, all three circles are equal.
Represents this in diagrammatic form. We get, PQ = QR = PR
Q R PQR is an equilateral triangle.
And area of shaded portion
= Area of PQR – 3 (area of each sector),
P (I) QR is known.
side of triangle and radius is known.
Then, R is the mother of P. Hence, statement (I) alone is sufficient to answer the
68. (1) p q or p q p < q or p > q ...(1) question.
q or q = r or r < q ...(2)
Now, since all three circles are equal, therefore, statement
From eqs. (1) and (2), we can say that option (1) is
(II) is restatement of the information given in question.
correct.
69. (4) 80. (4) Cannot be determined
81. (1) Data inadequate
1 1
70. (2) Reqd. Ratio = : 2 :1
3 6 82. (4) B F
71. (4) Insufficient data about weights to answer . E D E D
72. (3) Proteins & other elements account for 16 + 14 = 30% OR
C F C B
360 30 A A
= 108
100
So, either F or B is facing A.
73. (1) Human body constitutes 70% of water
83. (2) C is facing D.
70 84. (1) The water image of ‘a’ is ‘ ’, ‘b’ is ‘ ’, ‘S’ is ‘ ’, ‘e’ is
50 = 35 Kg
100 ‘ ’, ‘n’ is ‘ ’, ‘c’ is ‘ ’ and ‘e’ is ‘ ’.
74. (4) Sunita has a very busy schedule. This means that she 85. (1) Since, in case of water images, usually the image drawn
is industrious. But still she finds time for rest. This is just under the actual object. So the shaded and dotted
means that she is an organised person. So, both I and II portion will come up on their respective sides, because
follow. the portion of the object near water surface will remain
75. (1) According to the statement, funding is necessary to near the water surface in case of the image as well.
improve quality and India is allocating funds to
education. This means that quality of education will
improve in India. So, I follows. But funding alone is
sufficient to enhance quality, is not true and doesn’t
Water surface
have a mention in the statement. So, II does not follow.
76. (1) Either there were less options available or there was no
desire to influence the destiny in the man of the past
that’s why he couldn’t influence his destiny by the
choice he made. 86. (4) The required region is the one which is common to the
77. (1) (I) BC = 5 = AB circle and the triangle only i.e., D
p = q = 40º 87. (4) The required region is the one which is common to the
r = 180º – (40 + 40) triangle and the rectangle but lies outside the circle i.e, Z.
Hence, r can be determined by using (I) alone. 88. (3) C lies inside the triangle only. So, C represents surgical
(II) it is not sufficient to answer the question. Specialists.
M 89. (2) The year 1979 being an ordinary year, it has 1 odd day.
78. (2) (1) K
3
So, the day on 12th January 1980 is one day beyond on
M the day on 12th January, 1979.
may be odd or even. Therefore it is not sufficient
3 But, January 12, 1980 being Saturday.
to answer the question. January 12, 1979 was Friday.
M 90. (3) 55 min spaces are gained in 60 min
(II) = 2J + 1, which is always odd because J is an
3
35 min spaces will be gained in 38.18 min.
integer.
Answer = 7 hrs + 38.18 min.
Exemplar Practice Questions M-37
91. (3) The term ‘usually’ indicates that some possibility
might be possible that X goes out during rain. A
Nothing has been said about the urgency of the work.
Thus neither I nor II follow. E
92. (3) All the members of the Golf Club are necessarily rich. F G
Since Mrs. X is a member of the Golf Club, she is rich.
93. (2) Since all employees of the company. A have identity
B D C
cards and Ram is an employee of the company, he
must have an identity card.
101. (2) Canada, Mexico are in North America and Australia is a
Sol. for (94 to 96) : continent and located in the Pacific (1) - Asia; (3) Africa
We may prepare a table as follows: & (4) Europe. Except (2) all are in one countries
102. (1) Except (1) all are natural disaster. Seismograph is an
Nehru House Gandhi House Tall S hort Glas ses No glas ses instruments scientists use to measure earthquakes.
A 103. (1) Except (1) rest of all are divisible by 3.
B 104. (3) In the alphabet series there are 2 letters between ‘O’
C and ‘R’. Here also there are 2 letters (AZ) between them.
D
Again, there is no letter between R & S.
E
F R A Z O R S
94. (3) B and E are short and not wearing glasses. 105. (3) TUB when interchanged will term a meaningful word
95. (2) E belongs to Gandhi House, is short and does not wear BUT.
glasses. 106. (4) The order of words in dictionary will be CAR, HID,
96. (4) F belongs to Gandhi House, is tall and is not wearing LAP, SON, TUB
glasses. 107. (3) Painter cannot be made from the letters of the given
word as I letter is not mentioned in the given word.
97. (4) We may prepare a table as follows:
108. (4) Trible cannot be made from the letters of the given
word as E letter is not mentioned in the given word.
Green Yellow New Old Law Medical
Cover Cover Volume Volume Reports Extracts
109. (3) Recognise cannot be made from the letters of the given
A word as C and E letters are is not mentioned in the
B given word.
C 110. (4) Groundnut belongs to same group.
D 111. (2) Clutch, Brake, Horn, Steering are parts of Car.
E
F (× 2)–1
112. (4) (7, 5, 3)
Clearly, E and F are old volumes which have green (17, 15, 8)
covers and are medical extracts.
(× 2) + 1
98. (3) The four vocal musicians and one actress in the group
are given. Two dancers are Shailja and Tanuja. Two 8 × 2 = 16 – 1= 15 3×2=6–1=5
violinists are Girija and Vanaja. Since Jalaja and Shailja 8 × 2 = 16 + 1 = 17 3×2=6+1=7
cannot be violinists, so remaining two violinists are 113. (3) (8, 3, 2) (63, 8, 3)
Tanuja and Pooja. Clearly, Tanuja is both a violinist and
a dancer. 22 = 4 – 1 = 3 32 = 9 – 1 = 8
2
3 =9–1=8 82 = 64 – 1 = 63
99. (3) Eleven pieces consist of 10 smaller equal pieces and
one half cake piece. 114. (2) BOAT = 4 + 12 + 6 + 8 = 30
Weight of half piece with ten small pieces of weight 25 TAME = 8 + 6 + 6 + 9 = 29
grams each = 25 × 10 g i.e. 250 grams GATE = 8 + 6 + 8 + 9 = 31
Total weight of the cake = 2 × 250 grams i.e. TENT = 8 + 9 + 12 + 8 = 37
500 grams. Hence, TAME has the lowest value.
100. (3) Number of triangles in this figure are 16: AEG, AFG, BFG, 115. (3) ATPOG = 6 + 8 + 16 + 12 + 8 = 50
BDG, CDG, CEG, ABG, BCG, ACG, ABD, ADC, ABE, BCE, BMNET = 4 + 6 + 12 + 9 + 8 = 39
ACF, BCF and ABC. PEGNO = 16 + 9 + 8 + 12 + 12 = 57
MAETB = 6 + 6 + 9 + 8 + 4 = 33
Hence, PEGNO has the highest value.
EBD_7332
M-38 NTSE Stage 1 Question Bank
116. (1) Let present age of Amit’s son = x yrs. 123. (2) 26 January 2015 = (2014 years + period from 1.1.2015 to
and age of Amit = 6x yrs. 26.1.2015)
Four years from now, (6x + 4) + (x + 4) = 43 Odd days in 1600 years = 0
7x = 35 Odd days in 400 years = 0
x = 5, i.e., present age of Amit = 6x = 6 × 5 = 30 yrs. 14 years = (11 ordinary years + 3 leap years)
117. (4) Aman has taken a rectangular path as is clear from the = 11 × 1 + 3 × 2 = 17 odd days
diagram below. January 26 days 5 odd days
Total number of odd days = 17 + 5 = 22 = 1 odd day.
2 km
N Given day is Monday.
124. (1) Candidates failed in social science only = (Candidates
5 km failed in social science) – (candidates failed in all three
subjects + candidates passed in science only +
12 km W E candidates passed in one subject
7 km = 175 – (60 + 52 + 48) = 175 – 160 = 15
125. (3) Candidates failed in one subject only = (total number
S of candidates) – (candidates passed in all subjects +
2 km
candidates failed in all the subjects + candidates passed
118. (3) Let A takes ‘n’ days to paint the room. in one subject only)
So B will take 3n days to paint the room. = 450 – (167 + 60 + 62 + 48 + 52) = 450 – 389 = 61
3n – n = 60 126. (2) Candidates failed in science only
or n = 30 = 191 – (62 + 60 + 48) = 21
A = 30, B = 90 Candidates failed in social science only = 15
1 1 4 2 Candidates passed in Maths and at least one more
A & B will do work in a day subject
30 90 90 45
= (21 + 15 + 167) = 203
45 1 127. (3) 7+ 8 5×6–4
So they will complete the work in 22 days
2 2
48 35 48 20 63 3
92.4 km 15.4 1000m 7 4 12
119. (1) 92.4 km/hr = 60 min = 15400 m in 10 min 5 5 5 5
10 min
128. (1) 15 3 + 24 – 12 × 2 5 + 24 – 24 = 5
120. (3) Net ascent of monkey in 1 hr = (30 – 20) = 10 feet 129. (4) 40 + 30 – 50 = 20
So, the monkey ascends 90 feet in 9 hours i.e. till 5 p.m. 130. (2) 60 – 12 = 8 × 5 + 8
Clearly, in the next 1 hour i.e., 6 pm the monkey ascends 48 = 48
remaining 30 feet to touch the flag.
131. (1) L E M 4 5 9
121. (3) Earlier + M E L T + 9 5 4 0
K M Position 9 9 9 9 9 9 9 9
5L 6R
132. (2) S 6, E 4, T 1
New 4 3 1
M K Position + 1 4 3 6
5L 6R after 1 8 6 7
13 L interchanging
So the numbre of girls in a row = 6 + 13 – 1 = 18 E N
Mona’s position from right = 18 – 5 + 1 = 14 NW NE
133. (4)
122. (3) NE A B
W E
(12 noon) NE D
N E
SW SE
S
NW SE C

W S 134. (1) Only Science = b


SW 135. (3) M H only = e
Now, hour hand will point to East.
Exemplar Practice Questions M-39

136. (3) father-in-law 144. (3) P A P E R


+3 +2 +4 +2 +5
(only child) son wife (Mamta)
S C T G W
son Similarly,
Mamta’s father-in-law’s only child is Mamta’s husband M O T H E R
and only child’s only son is her husband’s only son +3 +2 +4 +2 +5 +2
i.e. Mamta’s son.
P Q X J J T
3 12 27 48 75 108 147
137. (1) 145. (2) P O P U L A R
+3 × 3 +3 × 5 +3 × 7 +3 × 9 +3 × 11 +3 × 13 +1 +1 +1 +1 +1 +1 +1
138. (4) 1
1 × 4 = 4; 4–2=2 Q P Q V M B S
2 × 4 = 8; 8–2=6 Similarly,
6 × 4 = 24; 24 – 2 = 22 G B N P V T
22 × 4 = 88; 88 – 2 = 86
–1 –1 –1 –1 –1 –1
2 2 2 2
139. (3) Ist Letter : K I G E C
F A M O U S
2nd Letter:
146. (1) C A L C U T T A
3 3 3 3 +4 +4 +4 +4 +4 +4 +4 +4
M P S V Y

2 3 5 7 G E P G Y X X E
Number : 1 3 6 11 18
Similarly,
140. (1) a b c / a b c / c b a / c b a
F S Q F C E
i.e, c a c a.
141. (2) x x y y / y y z z / x x y y / z z x x / y y z z –4 –4 –4 –4 –4 –4
i.e. x z y x
142. (3) Tractor is used to plough a field but here is code tractor B O M B Y A
is called car.
+2 +2
143. (3) P + R Q
P+R P is father of R
R Q R is daughter of Q.
89 78 86 80 85 82 83
Making diagram from the above expression, we get, 147. (3)
P Q
–2 –2 –2
The term 86 is wrong and it can be replaced by 87.
R 148. (2)
2 2 2 2 2
1 (1) 3 (3) 6 (6) 10 (10) 15 (15)
Male ; Female
Hence, P is the husband of Q.
+2 +3 +4 +5
Hence, the term 16 is wrong and it can be replaced by 15.

444 300 200 136 100 84 80


149. (4) 2 2
2 2 2 2
– (12) = 144 – (10) = 100 – (8) = 64 – (6) = 36 – (4) = 16 – (2) = 4
EBD_7332
M-40 NTSE Stage 1 Question Bank
160. (4) Fig. (i) 15 – 9 = 6
150. (2) 22 2 2 92 9 72 82 8 56 22 – 16 = 6
Common Diff. = 6 square it = 62 = 36
32 3 6 12 1 0 72 7 42
Fig. (ii) 13 – 9 = 4
2 2 2
4 4 12 5 5 20 6 6 30 11 – 7 = 4
Common Diff. = 4 square it = 42 = 16
151. (1) In the first column,
Fig. 21 – 13 = 8
91 + 84 + 25 = 200
23 – 15 = 8
In the second column,
Common Diff. = 8 square it = 82 = 64
64 + 76 + 60 = 200
161. (2) Method (I)
Similarly,
18 × 5 = 90 + 12 = 102
73 + 61 + ? = 200
12 × 4 = 48 + 6 = 54
? = 200 – 73 – 61 = 66
9 × 3 = 27 + 3 = 30
152. (2) 1 Method (II)
2 3 4
9 8 7 6 5
+24 54 +48
10 11 12 13 14 15 16
30 ? 102
25 24 23 22 21 20 19 18 17
26 27 28 29 30 31 32 33 34 35 36 +12
49 48 47 46 45 44 43 42 41 40 39 38 37 18 9
Start
Correct answer is 30, 21, 23. 12 +3 from here
+6
153. (4) By observation
154. (1) By observation.
155-157. Letter Code Letter Code
32
A s B l 162. (3) 13 + 19 = 4
8
C q E t
G u H f 80
71 + 9 = 10
I p J a 8
L g M d
N k O b 160
128 + 32 = 20
P r R w 8
S o T x 163. (1) Experienced and Trained teachers work in Rural Area–
U e V z A no which comes in all diagram ( , , )
W y X v It is = 5
Z n 164. (2) Experiences and Trained teachers but not working in
Rural Area–
155. (3) T x, O b, I p, L g
A No which comes in and both. But Not in
156. (1) C q, O b,S o, T x
It is = 4
157. (4) U e, L g, C q, E t, R w 165. (4) O = 16, FOR = 42
F=6 + 1 =7
158. (4) Birds R = 18 + 1 = 19
O = 15 + 1 = 16
Crows N = 14 + 1 = 15
Cats
T = 20 + 1 = 21
78
Then FRONT = 78
166. (1) TER M I NAT I ON M OT I O N
159. (1)
4 8 158 6
1 2 3 4 5 6 7 158 6
Land Water
Creatures Creatures 167. (2) a a b b a a /aa b b a a
Hence a a b a is answer.
168. (2) b a a / bb a / abb/ a a b
Snakes Hence, b a a is answer.
Exemplar Practice Questions M-41
169. (1) a a b b b b a a a a b b b b a a 182. (3) d b h f k l t h f b h k l t b d b h t d b b d h k
Hence, b b a a is answer.
170. (1) a a b b / a a a b / b b a a 183. (1) Triangles
171. (3) +3 +4 +5 A
D B
acute angled
4 7 11 16 C triangle
equilateral
7 13 21 31 triangle
isosceles triangle
+6 +8 + 10 184. (2) 13 is the correct answer
172. (2) 173. (2) 14 17 24 35 50
174. (3) In first figure : (25 + 17) 6 = 7 185. (3)
In second figure : (38 + 18) 8 = 7 +3 +7 + 11 + 15
(89 + 16) ? = 7 (186 - 188). The correct order is:
105 Sun Mon Tues Wed Thus Fri Sat
105 ? = 7 ?= 15
7
D B E F C A
175. (2) In first figure,
3 × 2 = 6; 3 × 3 = 9; 3 × 6 = 18 186. (1) There is no lecture on monday.
Similarly, 187. (4) Three lectures are organised between C and D – B, E
4 × 2 = 8; 4 × 3 = 12; 4 × 6 = 24 and F.
188. (1) F is organised on thursday.
176. (2)
(189 - 191). Teacher Subjects
P English
Clips Green Red
Q Geography
R Maths
S Hindi
Conclusions: I : False T History
II : True
189. (4) The History teacher is T.
177. (1) 190. (4) The Hindi teacher is S.
191. (2) The teaching subject of P is English.
Papers Files Pens
(192 - 194) A Physics Chemistry Electronics
B Physics Chemistry Maths Electronics
C Chemistry Maths
Pens
D Physics Chemistry Maths Electronics
E Maths Electronics

192. (3) B studies most number of subjects from the given


Conclusions: I : True
options.
II : False
193. (1) Chemistry is common subject of A, D and C.
178. (2) The investment agent calculates the market risk with
194. (3) C and E study only two subjects.
certainty.
179. (3) Students play kabbadi only = (1 + 3) = 4 195. (4)
180. (1) Students common to all the sports i.e. 2.
181. (2)
36996869888963968 9 6 9
6 preceded
by 9
9 preceded
by 6
EBD_7332
M-42 NTSE Stage 1 Question Bank

A
196. (2)

E F
G

B D C

The triangles composed of three components each are –


ABD, ADC, i.e. 2 in number.
197. (3) 198. (1)
The triangle composed of six components is – ABC.
199 - 201. Green So, total triangles are – (6 + 2 + 2 + 1) = 11.
208. (4) The figure may be labelled as shown :

D H C

E G

Blue A B
F
199. (4) There are four cubes in the middle layer which have The simplest triangles are – DHI, DEI, IFB, IGB, i.e. 4 in
one face painted only in blue. number.
200. (2) There is one (central) cube in top layer and one (central) The triangles composed of three components each
cube in the bottom layer which have one face painted are – DAB, DCB i.e. 2 in number.
in green only. So, total triangles are = (4 + 2) = 6
201. (1) Four (corner) cubes in the top layer and four (corner) The simple quadrilaterals are – DEHI, HIGC, EIFA, FIGB
cubes in the bottom layer have three sides painted. i.e. 4 in number.
There are no such cubes in the middle layer. Hence, The quadrilaterals composed of two components each
there are 8 such cubes. are – DHFA, HFBC, DECG, EABG, i.e. 4 in number.
202. (1) 203. (4) The quadrilateral composed of your components is –
204. (1) 205. (4) ABCD. So quadrilaterals are = (4 + 4 + 1) = 9
206. (4) The figure may be labelled as shown :
209. (2)
A 210. (4) In each step, shaded circle shifts one side ACW while
plus symbol shifts one side CW.
211. (1) In each step, each bar as well as small dots increases
J
E F by 1.
I 212. (4) In all the figures except (4) inside circle is rotating in
G H
anticlockwise direction while outer curve rotating in
C
clockwise direction. Hence option (4) is odd figure.
B
D 213. (2) By visualizing the figure, we figure out that option (2)
is odd one out.
The simplest triangle are–
AEJ and AJF i.e. 2 in number. 214. (4) The cubes at all the corners have three coloured
The triangles composed of two components each are– surfaces. Therefore, there are 24 such cubes having
AGI, AIH, AEF i.e. 3 in number. three surfaces coloured.
The triangles composed of three components each are– 215. (1) There is no such cube having two surface coloured.
ABD, ADC i.e. 2 in number. 216. (4) The cubes which are located at central of every surface
The triangle composed of four components is AGH i.e. having one face coloured. Therefore, in each layer there
1 in number. are 4 such cubes. Hence, 24 number of blocks having
The triangle composed of six components is ABC. i.e. 1 one face coloured.
in number.
So total triangles are = (2 + 3 + 2 + 1 + 1) = 9 217. (4)
207. (4) The figure may be labelled as shown:
The simplest triangles are – 218. (3) First letter : +2, +3, +4, +5 etc.
AEG, AGF, EGD, FGD, BED, DFC, Second letter : –2, –3, –4, –5 etc.
i.e. 6 in number.
219. (3) +1, +2, +3, +4 in letters; +3, +5, +7, +9 in numbers.
The triangles composed of two components each are –
AED, AFD i.e. 2 in number.
Exemplar Practice Questions M-43
220. (2) There is a gap of four letters between first and second, 235. (4) Consider pair of numbers
second and third letter of each term. Also there is a gap (1, 1) = (1, 13)
of 4 letters between the last letter of a term and the first (2, 4) = (2, 22)
letter of the next term.
(3, 27) = (3, 33)
221. (4) The first letters in odd numbered terms from the series J,
(4, 16) = (4, 42)
I, H and in even numbered terms from the series K, L, M.
Next number will be 5.
The sequence followed by the numbers is +2, +3, +4,
+5, +6. The third letter of each term is moved two steps 236. (3) The series is mno/nopq/opqrs/pqrst
backward to obtain the third letter of the next term. 237. (3) Consider first three terms. Differences are 2, 4. In the
222. (4) There are two alternate series . next three, Differences are 6, 8. In the next three
differences should be 10, 12.
+1 +1 +1
238. (3)
B Y C X D W E V
D O C U M E N T A T I O N
–1 –1 –1
Series I : BCDE (natural order) 1 2 3 4 5 6 7 8 9 10 11 12 13

Series II : YXWV (reverse order)


223. (4) 3 = 22 – 1, 8 = 32 – 1 : The eleventh from right is T.
35 = 62 – 1, 48 = 72 – 1: 239. (4) Clearly, the given letters, when arranged in the order 5,
? = 102 – 1, 120 = 112 – 1. 1, 2, 3, 4 from the word ‘TRACE’.
224. (2) The difference between the letters increases at each 240. (3) 241. (2) 242. (2)
step after beginning with two. 243. (3) 244. (3)
245. (4) J is the tenth letter in the first half. The tenth letter in
B D G K P V the later half is W.
+2 +3 +4 +5 +6 246. (4) 22nd letter from the left is V and 21st letter from the
right is F. The letter midway F and V is N.
225. (3) This is a series of prime number
247. (4) In all other pairs, second is a part of the first.
226. (4) Let x = 8 248. (2) In all other pairs, second is the result of the first.
then 15 = 2x – 1 = y 249. (2) The words in all other pairs are synonyms.
250. (4) Except (4) The two words in the other options are opposite
28 = 2y – 2 = z to each other.
53 = 2z – 3 = m 251. (2) The total no. of digits is seven, others are six.
Next term in the pattern should be 2m – 4 = 2 x 53 – 4 = 102 252. (3) It is the set of all prime numbers, other are not.
253. (4) The sum is not divisible, like other sets.
227. (3) Note that 0 = 13 – 1 254. (2) The sum of all numbers in each set is 100 except
6 = 23 – 2 (2) 100 .
255. (2) Other letters are consecutive.
24 = 33 – 3
256. (3) All other groups contain alternate letters of the alphabet in-
228. (4) The terms exhibit the pattern n 3 + 1, n taking values 1, order.
2, 3....... 257. (3) A hungry person requires food and a weary person
229. (3) 2 = 12 + 13 requires rest.
12 = 22 + 23 258. (2) All are natural sources.
259. (3) All are computer languages.
36 = 32 + 33 and so on.
230. (3) The series is + 5, + 7, + 9, + 11, ... x2
260. (3) The logic is x : .
231. (4) The series is 2, 1.5 alternately.. 2
232. (3) The series is an alternate series, having 261. (2) Sum of the digits of the first number is 2 more than the
S1 = 2 5 14 41; × 3 – 1 in each term sum of the digits of the second number.
S2 = 3 8 23 69: × 3 – 1 in each term 262. (2) Tie is worn in the neck and belt is worn on the waist.
263. (2) A, C, F, J are 1st, 3rd, 6th and 10th letters from the start
233. (3) The differences are 1– 0 = 1 = 13 ; 9 – 1= 8
of alphabets whereas Z, X, U, Q are the 1st, 3rd, 6th and
= 23; 36 – 9 = 27 = 33; 99 – 36 = 63 43, 10th letters from the end. In the same way, EGJN are
but 100 – 36 = 64 = 43; 225 – 100 = 125 = 53 ; related to VTQM.
441– 225 = 216 = 63 264. (4) Each letter of JKLM stands for each corresponding
234. (4) ab – – a – dcacb – acd – letter of XYZA, 14 places before.
Option (4) i.e, cdbdb provides the pattern abcd/ abdc/ 265. (3) First tackles the second.
acbd/acdb
EBD_7332
M-44 NTSE Stage 1 Question Bank
266. (3) Type – Alphanumeric coding. 274. (1) E R O D E – 66, 10, 76, 02, 85
Therefore, Codes Words
Word V I S H A L
Value in 22 9 19 8 1 12 pee are
alphabet series hee flowers
1 2 3 4 5 silk red
Code 22 10 21 11 5 17
mit vegetables
similarly, for the word S A C H I N il roses / blue
19 1 3 8 9 14 be blue / roses
1 2 3 4 5 275. (3) Let the number of boys be x.
Final code 19 2 5 11 13 19 Then, number of girls = 2x.
Thus code is 1925111319, option (3). x + 2x = 60 or 3x = 60 or x = 20.
So, number of boys = 20 and number of girls = 40.
267. (1) Type – Direct coding (Direct fashion)
Number of students behind Kamal in rank
5 P, 4 O, 6 E, 1 A, 3 R. (60 – 17) = 43.
Therefore, 54613 is coded as POEAR. Number of girls ahead of Kamal in rank = 9.
268. (1) Type – Simple arrangement (SWAP CODING) Number of girls behind Kamal in rank
Interchange F and D, R and N and I and E. = (40 – 9) = 31
Similarly, DESERT is coded as TRESED. Number of boys behind Kamal in rank
269. (2) Type – Simple Arrangement (Swap coding). = (43 – 31) = 12.
In the word AFFAIR, the positions of first A & first F 276. (4) Changing the positions of the digits as mentioned, we
are interchanged second A & second F are get the number : 49138657.
interchanged and I and R are interchanged. The seventh digit from the left end of this number is 5.
Similarly, FERRARIS is coded as EFRRRASI. The third digit to the left of 5 is 3.
277. (3) The numbers satisfying the given conditions, can be
Sol. (270 - 272)
Type Direct coding (Jumbled fashion) seen as follow: 8 9 5 3 2 5 3 8 5 5 6 8 7 3 3 5 7 7 5

S. No. Code Sentence 3 6 5 3 35738


1. il be pee roses are blue
Clearly there are three such numbers.
2. silk hee red flowers
3. pee mit hee flowers are vegetables.
278. (4) Clearly, number of boys in a line = 12 + 4 – 1 = 15
Number of boys to be added = 35 – 15 = 20.
Common word in sentences 1 & 3 ‘are’ and code ‘pee’
279. (3) If day before yesterday was Thursday, so today is
Common word in sentences 2 & 3 ‘flowers’ and code
Saturday. So tomorrow will be Sunday.
‘hee’.
280. (3) Total students
270. (2) silk.
271. (4) il or be, cannot be determined. = [Rakesh’s position from upwards + Rakesh’s position
from downwards] – 1
272. (1) niit silk hee pee.
Sol. (273 - 277) = [9 + 38] – 1 = 46
281. (3) Suman is 17th from the last and Raman is 7 ranks ahead
MATRIX-I T – 00, 12, 24, 31, 43
of Suman. So Raman is 24 th from the last.
N – 01, 13, 20, 32, 44
Raman rank from the start is 39 + 1 – 24 i.e, 16 th
D – 02, 14, 21, 33, 40
282. (2) Father's Wife — Mother; Mother's daughter — Sister
R – 03, 10, 22, 34, 41 Deepak's sister's younger brother— Deepak's
I – 04, 11, 23, 30, 42 brother.
MATRIX-II M – 55, 67, 79, 86, 98 283. (1) Father's wife means mother ; mother's only son
W – 56, 68, 75, 87, 99 means himself and thus the girls is the daughter of
the man.
O – 57, 69, 76, 88, 95
284. (2) M + J means 'M' is the daughter of 'J' J ÷ K means 'J' is
F – 58, 65, 77, 89, 96 the son of 'K'
E – 59, 66, 78, 85, 97
273. (2) T I M E – 43, 11, 55, 66
Exemplar Practice Questions M-45
294. (1) Given : A + B = 2C ..........(1)
K
and C + D = 2A ......... (2)
(Father )/(Mother
Adding (1) and (2) we get :
J A + B + C + D = 2C + 2A B + D = A + C.
295. (1) The movements of Raj are as shown in fig (X to Y, Y to
(Son) (Grandfather)/(Grandmother) A, A to B , B to C ).

C 80m Y
X
P
M 50 m 50 m
(Daughter)
285. (4) C is B's daughter and D is B's son. So, D is the brother B 70m A
of C.E is a male married to C so, E is the husband of C,
Raj's distance from the starting point
whose brother is D. Thus, D is the brother in-law of E.
= XC = (XY – YC) = (XY – BA)
286. (2) It is clear that C is the Brother of B but how B is related = (80 – 70) m = 10 m.
to C depends on the sex of B.
25 metres
287. (3) ‘S + Q’ & ‘R + S’ means R is the grandfather of Q. Now

15 metres
15 metres
P R means P is daughter of R. This clearly means P is
aunt of Q.
296. (4)
288. (1) P – R + Q, represents R is the father of Q, and P is the Starting 20 metres
wife of R. Point E Finishing Point

P is the mother of Q. 297. (3) The directions to be followed will be :


289. (3) Using proper notations, we have:
11 left
(1) given statement is 3 ÷ 2 + 4 < 9 ÷3 – 1 or < 2, left
2
which is not true. N
right
(2) given statement is 3 + 2 + 4 < 18 ÷ 3 – 2 or 9 < 4,
which is not true. W E
(3) given statement is 3 + 2 – 4 > 8 ÷ 4 – 2 or 1 > 0,
which is true. S
5 298. (3) The track of both the friends is
(4) given statement is 3 ÷ 2 – 4 > 9 ÷ 3 – 3 or – > 0,
2
Jack

Bunny

which is not true . So, the statement (3) is true.


50mts

290. (1) Given that : 20 – 10 = 200.


But, actually 20 × 10 = 200, so – means ×.
Given that 8 ÷ 4 = 12, But actually 8 + 4 = 12. 60mts 40mts
So, means +.
50mts 50mts
Given that : 6 × 2 = 4 But actually 6 – 2 = 4.
So,× means –
Thus, in the given mathematical language – means ×,
÷ means + and × means – So, ÷ Given expression Two friends are 60 + 40 = 100 mts apart
= 100 × 10 – 1000 + 1000 100 – 10 299. (2) N
NE School
1000 – 1000 + 10 – 10 = 0. NW
291. (2) Using the correct symbols, we have
W E
Given expression = (3 × 15 + 19) ÷ 8 – 6
= 64 ÷ 8 – 6 = 8 – 6 = 2.
SW NW Home
2 S
a b
292. (3) The rule is a + b = It is clear from the diagram that school is in North-west
2
direction with respect to home.
2 2 300. (1) In morning, sun rise in the east so shadow of a object
3 5 11 3
3+5= = 16 etc. 11 + 3 = = 49 falls towards the west. Now, Kavita’s shadow falls to the
2 2
293. (4) The rule is the digits are jumbled in an order. rights of Reeta. Hence, Reeta is facing South and Kavita is
facing North.
EBD_7332
M-46 NTSE Stage 1 Question Bank
307. (2) Married teachers are represented by the region common
N to the square and the triangle i.e., B and C. But,
Reeta according to the given conditions, the persons should
be living in joint families. So, the required region should
W E
be a part of the circle. Since B lies inside the circle, so
Kavita's the given conditions are satisfied by the persons
Kavita
Shadow S denoted by the region B.
308. (1) Between 2 and 4 O’clock, 8 and 10 O’clock, the hands
301. (2) The movements of the man from A to F are as shown in are at right thrice in each case. Between 4 and 8’O clock,
fig Clearly DC = AB + EF. the hands are at right angles.
A 20m B So total number of times the hands are at right angles
E 15 m
= 3 + 3 + (2 × 4) = 14.
F 10m
309. (2) The year 1979 being an ordinary year, it has 1 odd day.
5m
So, the day on 12th January 1980 is one day beyond on
D C the day on 12th January, 1979.
35m
F is in line with A. But, January 12, 1980 being Saturday.
Also, AF = (BC – DE) = 5 m. January 12, 1979 was Friday.
Men 310. (2) A leap year has (52 weeks + 2 days). So, the number of
302. (2) odd days in a leap year is 2.
311. (3) The hands coincide 11 times in every 12 hours (between
Educated 11 and 1 O’clock there is a common position at 12
Citizens O’clock). Hence, the hands coincide 22 times in a day.
Some educated are citizens. 312. (1) At 2’O Clock, Minute Hand will be 10 × 6 = 60°
behind the Hour Hand.
Some citizens are educated.
But, both educated and citizens are men. 1
In 30 minutes, Minute Hand will gain 5 × 30
2
303. (1) = 150 + 15 = 165°
Angle between Hour Hand and Minute Hand
Moon = 165 – 60 = 105°
313. (2) Time from 8 a.m. on Sunday to 8 p.m. on following
Sunday
Sun Stars = 7 days 12 hours = 180 hours

Sun is a Star. But, Moon is entirely different. 4 54


The watch gains 5 5 min. or min. in 180 hrs.
5 5
304. (5) Planets
54
Now min. are gained in 180 hrs.
5
Mars 5
Mercury 5 min. are gained in 180 5 hrs. = 83 hrs 20 min.
54
Mercury and Mars are entirely different. But, both are = 3 days 11 hrs 20 min.
planets. Watch is correct 3 days 11 hrs 20 min. after 8 a.m. of
305. (3) Married persons living in joint families are presented Sunday
by the region common to the square and the circle i.e., It will be correct at 20 min. past 7 p.m. on Wednesday..
D and B. But, according to the conditions, the persons 314. (1) Time interval indicated by incorrect clock
should not be school teachers. So, B is to be excluded. = 6 p.m. – 1 p.m. = 5 hours.
Hence, the required condition is denoted by region D. Time gained by incorrect clock in one hour
306. (3) Persons living in joint families are represented by the
1
circle. According to the given conditions, the persons = + 1 minute = + hour
should be unmarried and not working as school 60
teachers. So, the region should not be a part of either Using the formula,
the square of the triangle. Thus, the given conditions True time interval
are satisfied by the region E.
Time interval in incorrect clock
Exemplar Practice Questions M-47
missing letter in the first row will be three steps behind
1
= Z, which is W. The missing letter in the second row will
1 hour gained in 1 hour by incorrect clock be four steps behind O, which is K. The missing letter
in the third row will be three steps ahead of G, which is
True time interval 1
= J.
5 1 325. (1) Three, four, five ....... line segments are added to the figure
1
60 sequentailly at the lower end of the figure. Clearly, there
should be nine line segments in fig. (3) Hence, fig. (1) is
5 60 56
True time interval = =4 the answer.
61 61 326. (3) In each step, each one of the arrow rotates 90º CW.
56 327. (3) The ‘S’ shaped figure reverses its direction and gets
True time = 1 p.m. + 4 hrs. rotated through 45ºCW in each step.
61
328. (5) In each step, the figure gets vertically inverted and a
56 56 line segment is added to the RHS -end.
= 5 p.m. + hrs. = 5 p.m. + × 60 min. 329. (3) In each step, each elements of the figures shift one side
61 61
clockwise direction with rotation of 180°. So, option (3)
5 is correct.
= 55 minutes past 5. 330. (3) Option (3) will replaces the question mark.
61
315. (2) (7 × 8) – (5 × 4) = 36 331. (1) All other are vowels.
(9 × 3) – (5 × 2) = 17 332. (2) Each one of the figures except fig. (2), contains – one
(8 × 5) – (6 × 3) = 22 complete square, one cup-shaped element having side,
316. (1) Subtract the sum of the even numbers from the sum of one ‘L’-shaped element having two sides and one
straight line.
the odd numbers.
333. (3) All other figures can be rotated into each other.
7 + 9 = 16 9 + 3 = 12
334. (3) In each one of the figures except fig. (3), the angle
2+4=6 4 + 6 = 10 indicated between the two arrows is acute.
16 – 6 = 10 12 – 10 = 2 335. (3) The two lower arcs of the right hand side design of problem
317. (2) The number inside the circle is the difference of the figure (A) turns over vertically, then the whole design of
numbers on its left and right. right hand side reverses horizontally and shifts to left
318. (4) Multiply all the numbers around the circle and then hand side in the problem figure (B). The uppermost arc of
divide it by 10 to get the number at the centre, viz., left hand side design of problem figure (A) reverses
7 3 8 5 vertically and shifts to the right hand side in the Problem
84 Figure (B). Applying the same rule with Problem Figure
10
319. (1) In each row, out of the letters A, B and C, each of these (C), the answer figure (3) is obtained.
must appear once. Also, in each column. The product 336. (4) Clearly fig.(A) rotate through 1350 CW to from fig. (B)
of first and third numbers is equal to the second num- Similar relationship will give fig. (4) from fig. (C).
bers, so the missing number will be (2 × 4) i.e., 8 and the Hence, fig. (4) is the answer.
missing letter will be C. Thus, the answer is 8C. 337. (4) The contents of the outer main figure rotate through
320. (2) The above three numbers are multiples of the number 45ºCW.
at the bottom. Clearly 36, 18 and 27 are all multiples of 338. (3) The upper-left and the lower-right elements rotate 90°CW
9. So, the missing number is 9. while the upper-right and the lower-left elements rotate
90°ACW.
321. (4) We have : 36 + 64 + 25 + 49 = 26 ; 339. (5) The figure gets laterally inverted.
9 + 25 + 16 + 81 = 21 340. (2)
So, missing number
= 25 + 144 + 36 + 64 = (5 + 12 + 6 + 8) = 31
322. (2) In fig. (A),
(101 + 15) – (35 + 43) = 116 – 78 = 38.
In fig. (B), Missing number
= (48 + 184) – (56 + 34) = 232 – 90 = 142.
323. (2) In fig. (A), (915 – 364) = 551. 341. (3)
In fig. (B), (789 – 543) = 246.
In fig. (C), missing number = (863 – 241) = 622.
324. (3) The letter in the second column is three steps behind
that in the first column, and the letter in the third column
is four steps behind that in the second column. So, the
EBD_7332
M-48 NTSE Stage 1 Question Bank
356 - 358.
342. (2) E F

G B

D C
B H R
G
343. (3)
G
R

A B
Here,
Faces ABCD, BCFG are painted with red.
344. (2) Faces ADEH, EFGH are painted with red.
Faces CDEF, ABGH are painted with green.
345. (3) No. of divisions on the faces of cubes = 4

346. (4) Mirror image for the letters ‘j’ is , ‘e’ is ‘ ’, ‘a’ is ‘ ’, ‘I’
is I, ‘O’ is ‘O’, ‘u’ is ‘ ’ and ‘S’ is ‘ ’
347. (3) If we rotate the mirror image, i.e., ‘ ’ to 90°
anticlockwise, then it will be ‘ ’.

348. (2)

Hence option (2) is the answer.


Now, the figure may be analysed by dividing it into
349. (2) If we rotate the mirror iamge i.e., ‘ ’ to 90° anti- four horizontal layers :
In Layer I : Four central cubes only one face painted
clockwise, then it will be ‘ ’. Thus, option (2) is the answer.. green, four cubes at the corners have three faces painted
red, green and black. Out of the remaining 8 cubes, four
350. (2) 351. (1) cubes have two faces painted red and green and four
cubes have two faces painted black and green.
Black Green painted

352. (1)
k
ac
Bl

Red painted

9.30 2.30 Bottom painted

353. (4) When the sheet in fig.(X) is folded to form a cube, then Red painted
the face bearing a dot appears opposite to a blank face, In layer II and III : Four central cubes have no faces
the face bearing a ‘+’ sign appears opposite to another painted, four cubes at the corners have two face painted
blank face and the face bearing a circle appears opposite red and black. Out of the remaining 8 cubes four cubes
to the third blank face. Clearly, all the four cubes shown
in figures (1), (2),(3) and (4) can be formed. have only one face painted black and four cubes have
354. (3) Clearly, in the figure there are --1 column containing only one face painted red.
3 cubes, 2 columns containing 2 cubes each and 3 Black Unpainted
columns containing 1 cube each.
Number of cubes in columns of 3 cubes = 1 × 3 = 3;
Number of cubes in columns of 2 cubes = 2 × 2 = 4;
k
ac

Number of cubes in columns of 1 cubes = 3 × 1 = 3;


Bl

Therefore, total number of cubes = 3 + 4 + 3 = 10. Red painted


355. (4) From figures (i) and (iv) we conclude that 6, 5, 2 and 3
Bottom painted
lie adjacent to 4. It follows that 1 lies opposite 4.

Red painted
Exemplar Practice Questions M-49
In layer IV : Four central cubes have only one face Sol. (372 - 374) We prepare a table from the given data as under:
painted green, four cubes at the corners have three faces
painted red, green and black. Out of the remaining 8 Maths Geography His tory Political S c. Biology
cubes four cubes have two faces painted green and black A × × ×
and four cubes have two faces painted red and green. B × ×
Black Unpainted C × ×
D × ×
E × ×
k
ac
Bl

Red painted 372. (2) Clearly, from the table D is intelligent in Political Science,
Geography and Biology.
Bottom painted 373. (2) B is intelligent in Mathematics, Political Science and
Green Geography.
374. (3) A is intelligent in Mathematics and History but not in
Red painted
Geography.
356. (3) There are 8 cubes having no face painted. Sol. (375 - 379)
357. (3) Cubes only one face painted We may prepare a table as follows:
= 4 8 8 4 24 Nehru House Gandhi House Tall Short Glasses No glasses
(I layer) (II layer) (III layer) (IV layer)
A
or (n – 2)2 × 6 = 24 B
358. (3) In the figure there are 34 columns containing 2 cubes C
each. D
Total number of cubes = (34 × 2) = 68 E
359. (2) In the figure, there are 4 columns containing 5 cubes F

each and 38 columns containing 1 cube each.


375. (3) B and E are short and not wearing glasses.
Total number of cubes = (4 × 5) + (38 × 1) = 20 + 38 = 58.
376. (2) E belongs to Gandhi House, is short and does not wear
360. (2) 361. (3) 362. (3) 363. (4)
glasses.
364. (4) 365. (4)
377. (4) F belongs to Gandhi House, is tall and is not wearing
glasses.
378. (4) Consider the longest vertical strip.
It has six rectangles (excluding squares).
In the second vertical strip there are 3
366. (4) 367. (3) rectangles. In the second vertical strip there
are 3 rectangles.
This pattern is repeated with the largest
horizontal strip and the second horizontal
strip.
Next consider the third vertical and horizontal
strips. Each has one rectangle.
Again, consider two vertical and horizontal
368. (4)
strips together . Each has one rectangle.
Hence total no. of rectangles
= 2 (6 + 3 + 1 + 1) = 22
Sol. (369 - 371) 379. (3) Name the squares as square 1, 2, 3, 4 5 beginning with
Person Vehicle Profession Sex the largest.
A II Teacher Female The space between square 1 and 2 is subdivided into 4
B I Engineer Male triangles.
C III Doctor Female The space between square 2 and 3 has 12 triangles as
D II Doctor Male
below.
E III Teacher Male
From each vertex of square no. 2, there are two small
F II Engineer Female
G I Teacher Female
triangles and one large triangle consisting of these two
triangles.
369. (3) C travels in vehicle III.
Thus from four vertices, there are 12 triangles.
370. (2) There are four ladies in the group — A, C, F and G.
Each diagonal of square no. 2 divides it into two
371. (2) F is an engineer by profession.
triangles, making 4 triangles in this manner, each half
the size of the square.
EBD_7332
M-50 NTSE Stage 1 Question Bank
The two diagonals together sub- divide the square into 389. (2) S, C, A & R are denoted by 7, 9, 2 and 3 respectively, the
four triangles, each 1/4th the size of the square . representation for the word S C A R should be a combination
Thus there are 20 triangles in square no. of 7, 9, 2, & 3. Thus option (2) 2973 is the answer.
2 Pattern of square no. 1 and 2 is repeated in no. 3 and 390. (2) S O A P 7021. As 9 + O = 9, so O = 0
4 respectively. and P = C – D = 9 – 8 = 1.
There are no triangles in square no. Sol. (391 to 392) :
Hence, total no. of triangles = 4 + 20 + 4 + 20 = 48 As A = 2 and N = 3.
380. (1) One point lies in all three figures. Two points lie in B 2 3 2
between two figures. 3 2 3 2
381. (3) In fig. (X), the dot is placed in the region common to the
V L W L
circle and the triangle. Amongst the four alternatives
only in figure (3), we have a region common to circle and 2 + 2 = L = 4 and 3 + 3 = W = 6.
triangle only. Hence figure, (3) is the answer. 391. (2) If V is a prime number, it can hold the values 2, 3, 5 or 7.
382. (1) In fig. (X), one of the dots lies in the region common to As A = 2, N = 3 and if V = 5, then B will be 2 which is not
the square and the triangle only, another dot lies in the possible. Hence, V = 7 thus B = 4.
region common to the circle and the triangle only and 392. (3) V = 7.
the third dot lies in the region common to the triangle 393. (1) Option (1) – A C B
and the rectangle only. In fig. (2), there is no region Locks
common to the square and the triangle only. In fig. (3), Balls
there is no region common to the circle and the triangle
only. In fig. (4) there is no region common to the triangle Keys
and the rectangle only. Only fig. (1) consists of all the
three types of regions.
383. (2) In fig. (X), one of the dots lies in the region common to
All keys are balls and all balls are locks. Therefore,
the circle and the triangle only and the other dot lies in
all keys are locks.
the region common to the circle and the square only. In
Therefore, option (1) is the correct answer, where B
each of the figures (1), (3) and (4), there is no region
logically follows the 2 sentence A and C.
common to the circle and the triangle only. Only fig. (2)
394. (3) Option (1) – A B E
consists of both the types of regions.
Case I Case II
384. (4) In fig. (X), one of the dots lies in the region common to
P P
the circle and the triangle only, another dot lies in the
region common to all the three figures–the circle, the C S C S
square and the triangle and the third dot lies in the
region common to the circle and the square only. In
each of the figures (1) and (3), there is no region common
to the circle and the square only and in fig. (2), there is All pillows are sheets may be or may not be true. Thus
no region common to the circle and the triangle only. this option is incorrect.
Only fig. (4) consists of all the three types of regions. Option (2) – B C D
385. (2) As N = 6, M = 7. Therefore, A = Z – M = 8 – 7 = 1. Case I Case II
Therefore, option (2).
386. (2) M = 7, A = 1 and N = 6. Therefore, the numbers that P S C P S
stand for M A N have to be a combination of 7, 1 and 6.
Therefore, option (2) 716 is the answer.
387. (1) P O A R should be a combination of the numbers 4, 5, 1 & 3.
C
Thus option (1) 4513 is the answer. As it is given that O = 5.
Sol. (388 to 390): Pillows and cushions may or may not be related. Thus
For simplicity, let us replace all the Cs with 9, as C = 9 option (2) is not the correct.
(given) Option (3) – EAC
A B 9 D S
S R O P P
9 9 9 9 C

388. (4) C = 9, A = 2 but value for R is unknown. Thus the


number for C A R cannot be determined.
As A = 2, R < 5 where R is a prime number, the only Some pillows are cushions.
option available is R = 3. Thus option (3) is the correct answer where sentence C
follows the 2 sentences E and A.
Exemplar Practice Questions M-51
395. (1) Option (1) – A B E
Case II
Pretty Jungles
Heroines Popular Tigers Rains Birds
girls

Some popular girls are pretty. Thus this is the correct


answer. Case III
396. (3) Option (1) – A B D
Jungles
Cobra Tigers Rains Birds
Abra Dabra

So, only III and either I or II follow.


Sentence D (conclusion) does not follow the 399. (3)
preceding 2 sentences.
Rivers Mountains Hillocks
Option (2) – A E F
Case II
Case I Abra Dabra
Cobra Abra Dabra Valleys

Cobra So, Either II or III follows.


400. (2) Houses
Sentence F (conclusion) may or may not be true. Roads
Therefore, option (2) is not correct. Pens
Option (3) – B C F

Cobra
Abra So, conclusion II follows, therefore, option (2) is correct.
Dabra
Temples
401. (4)
Mansions
Some cobra are dabra, is the correct conclusion. Huts
Thus option (3) is the correct answer.
397. (4) Both statement II & III are true.

Flowers Fruits
Both I and II follow.
Trees
Branches So, option (4) is the correct answer.

402. (2) Chairs


Chalks
Case I
Pens
398. (3) Jungles
Tigers Bird Rains

Only conclusion II follows.


403. (2) Since one-half of the four children are girls, two must
be boys. It is not clear which children have blue or
brown eyes.
EBD_7332
M-52 NTSE Stage 1 Question Bank
404. (4) No answer description available for this question. Thus, required expenditures are C and B or D and E.
405. (3) The first statement cannot be true because only female or A and B or A and E.
birds lay eggs. Statement II is true because some hens For (Qs. 415 - 418)
are chickens and all chickens are birds. Statement III is Subjects Students Girls Boys
also true because if only some chickens are hens, then Art 216 168 48
some must not be hens. Biology 234 168 66
406. (1) Since the statement talks of putting the child in school Law 360 360 Nil
at the age of 5, it means that the child is mentally Computers 270 120 150
prepared for the same at this age. So, I is implicit. But Maths 90 24 66
nothing about admission after 6 years of age is Political Science 630 360 270
mentioned in the statement. So, II is not implicit. Total 1800 1200 600
407. (2) Assumption I goes against the statement. So, it is not
implicit. The allowance will serve as a reward to the 415. (1) Total number of assistants = 15% of 7000
employees and shall provoke them to come on time. 15
= 7000 1050
So, II is implicit. 100
408. (4) Since the product of any numbers is an even number if and Number of direct recruits = 40% of 1050
only if at least one of them is an even number. Now,
(I) A is an even number Product of A, B, C is even. 40
1050 420
(II) A × B is an even number and A × C is also an even 100
number. Now, number of promotee assistants = 1050 – 420 = 630
at least one of them (A, B, C) is even
Hence, required difference = 630 – 420 = 210
their product is an even number.
Each statement alone is sufficient to answer the question.
416. (3) Number of clerk I = 19% of 7000
409. (4) (I) C.P. can be calculated since 19
= × 7000 = 1330
S.P. 100 100
C.P.
(100 %profit ) Number of direct recruits = 90% of 1330
Also, since Profit = S.P. – C.P. 90 1330
Profit can be calculated using statement (I) alone. = 1197
100
(II) C.P. 3 Profit and Profit = S.P. – C.P. Now, number of promotees = 1330 – 1197 = 133
Therefore, each statement alone is sufficient to answer the
question.
Hence, required percentage
410. (2) From (I) : Q ( F) N( M) S 133 13300
= 100 = 11%
daughter 1197 1200
P( M) L(F) 417. (4) Number of supervisors = 17% of 7000
N is maternal uncle of P. 17
= 7000 1190
From (II) : (M )M K (M ) L (F) 100
daughter
G (F) P 418. (2) Type of employees Number
M is the paternal uncle of P
Hence we can give the answer using statement alone.
10 30
Steno 7000 210
411. (4) % of Royalty = 15% 100 100
15 40
15 Assistant 7000 420
Angle made by Royalty = 360 54 100 100
100
412. (4) Marked price = 1.2 × Cost price 17 50
Supervisor 7000 595
100 100
30
Cost price = = Rs. 25 19 90
1.2 Clerk I 7000 1197
100 100
Cost of paper = 25 × 20% = Rs. 5
20 3
413. (3) We have, 360° = 100% Clerk II 7000 420
100 100
100% 90 8
108 108 30% 7000 504
360 Officer I
100 100
From the given pie chart. (A + D) = 30% 11 70
Officer II 7000 539
18 100 100
414. (4) 18 100% 5%
360
Total direct recruitment
So, the difference in percentages of expenditures be
= 210 + 420 + 595 + 1197 + 420 + 504 + 539 = 3885
5%
SAT Exemplar Practice Questions

Scholastic Aptitude Test – SAT

SECTION 1. PHYSICS

1. An electric iron draws a current of 15A from a 220 V supply. 6. Three equal resistors connected in series across a source of
What is the cost of using iron for 30 min everyday for 15 e.m.f. dissipate 10 watts of power. What will be the power
days if the cost of unit (1 unit = 1 kW hr) is 2 rupees? dissipated in watts if the same resistors are connected in
parallel across the same source of e.m.f.?
(1) ` 49.5 (2) ` 60 (1) 10 W (2) 30 W
(3) ` 40 (4) ` 10 (3) 90 W (4) 40 W
7. A technician has 10 resistors each of resistance 0.1 The
2. Two different sources of light A and B have wavelength
largest and smallest resistance that he can obtain by
0.7 m and 0.3 m respectively. Then which of the following
combining these resistors are
statement is true
(1) 10 and 1 resp. (2) 1 and 0.1 resp.
(1) A has greater energy than B
(3) 1 and 0.01 resp. (4) 0.1 and 0.01 resp.
(2) B has greater energy than A
8. An object is placed in front of a concave mirror of radius of
(3) Both has equal energy curvature 15 cm, at a distance of 10 cm. The position and
(4) None of the above nature of the image formed is :
3. The type of radiation absorbed by CO 2 molecule in (1) +30 cm, virtual and erect
atmosphere are (2) + 30 cm, real and inverted
(1) X-rays (2) Gamma rays (3) – 30 cm, virtual and erect
(4) –30 cm, real and inverted
(3) Infra-red rays (4) UV-rays
9. An electric lamp whose resistance is 10 ohm and a conductor
4. The speed-time relation of a car whose weight is 1500 kg as of 2 ohm resistance are connected in series with a 6 V battery.
shown in the given graph. How much braking force has The total current through the circuit and the potential
been applied at the end of 7 sec. to stop the car in 2 sec? difference across the electric lamp are :
(1) 3.6 A, 6 V (2) 0.5 A, 5 V
(3) 2 .0 A, 0.2 V (4) 0.3 A, 3 V
10. Several electric bulbs designed to be used on a 220 V electric
16 supply are rated 20 W each. How many lamps can be
connected in parallel with each other across the two wires
12 of 220 V line if the maximum allowable current is 5 A?
Speed (m/s)

(1) 50 (2) 110


8 (3) 55 (4) 60
11. If the temperature is increased, what will be the effect on the
4
resistance of a conductor?
(1) does not change (2) decreases
1
2 3 4 5 6 7 8 9 10 (3) increases (4) cannot say
Time (sec) 12. The area under velocity–time graph gives:
(1) 2000 N (2) 9000 N (1) acceleration (2) distance
(3) 4800 N (4) 8400 N (3) displacement (4) velocity
5. The distance covered by a body moving along X-axis with 13. The Moon is constantly falling towards the Earth.
initial velocity ‘u’ and uniform acceleration ‘a’ is given by
(1) This statement is absurd.
1 2
x ut at . This result is a consequence of (2) This statement is correct.
2
(1) Newton’s Ist law (2) Newton’s IInd law (3) This statement is wrong.
(3) Newton’s IIIrd law (4) None of the above (4) Nothing can be said.
EBD_7332
2
S- NTSE Stage 1 Question Bank

14. Voice of which of the following is likely to have maximum 22. The mass of a body on earth is 60 kg. Its mass on moon will
frequency? be :
(1) man (2) cow (1) 360 kg (2) 60 kg
(3) bird (4) dog (3) 10 kg (4) 1/6 kg
15. Match the terms in column I with those of column II. 23. Which of the following sound of given frequencies can be
heard by us ?
Column I Column II
(1) 10 Hz (2) 10 kHz
(i) Electric fuse A. chemical effect
(3) 10 MHz (4) 10 GHz
(ii) Relay B. electric discharge 24. Example of fossil fuel is :
(iii) CFL C. magnetic effect (1) Alcohol (2) Hydrogen
(iv) Button Cell D. heating effect (3) Petrol (4) Diesel
(1) (i)–C, (ii)–B, (iii)–A, (iv)–D 25. Which is not the unit of Force?
(2) (i)–B, (ii)–A, (iii)–C, (iv)–D (1) Poundal (2) Dyne
(3) (i)–D, (ii)–C, (iii)–B, (iv)–A (3) Joule (4) Newton
26. Which country is called the country of Wind mills?
(4) (i)–D, (ii)–B, (iii)–C, (iv)–A
(1) Holland (2) Britain
16. The rate of evaporation increases with:
(3) India (4) America
(1) Increase of surface area, 27. One Volt is equal to-
increase of temperature, (1) 1 Joule (2) 1 Newton/Coulomb
decrease in humidity and (3) 1 Joule/Coulomb (4) 1 Coulomb/Newton
increase in wind speed. 28. The Unit of frequency is-
(2) Increase of surface area, (1) Hertz (2) Joule
(3) Ohm (4) Kilo Calorie
decrease of temperature, 29. Boiling point of Water is-
decrease in humidity and (1) 273 K (2) 0 K
decrease in wind speed. (3) 373 K (4) 100 K
(3) decrease of surface area, 30. A velocity-time graph for a moving object is shown below.
increase of temperature, What would be the total displacement during time
t = 0 to t = 6s?
Increase in humidity and
increase in wind speed.
(4) decrease of surface area,
increase of temperature,
decrease in humidity and
decrease in wind speed.
17. The centre of curvature of a concave mirror
(1) 10 m (2) 20 m
(1) lies in front of it
(3) 30 m (4) 40 m
(2) lies behind it
31. The radius of the orbit of a geosynchronous satellite is
(3) lies on the surface of mirror
36000 km, then the period of revolution of a satellite with its
(4) is apart of mirror orbital radius 9000 km would be
18. If a lens has power -2.5D, then it is a : (1) 24 hrs (2) 12 hrs
(1) convex lens, with a focal length of 40 cm (3) 6 hrs (4) 3 hrs
(2) concave lens with a focal length of 40 cm 32. An electric bulb is rated 100W at 220V. When it is connected
(3) convex lens with a focal length of 0.4 cm to 110V main supply, the power consumed would be:
(4) concave lens with a focal length of 0.4 cm (1) 100W (2) 75W
19. One atomic mass unit (a.m.u.) is equal to: (3) 50W (4) 25W
(1) 1 eV of energy (2) 931 eV of energy 33. An electric kettle has two thermal coils A and B. When
(3) 1 MeV of energy (4) 931 MeV of energy current is passed through A, water in the kettle boils in 6
20. The planet nearest to Sun is : minutes. When current is passed through B, same volume
(1) Mercury (2) Mars of water in the kettle boils in 8 minutes. If both A and B
(3) Saturn (4) Venus would be joined in series and current would be passed, water
21. The S.I. unit of momentum is : would boil in
(1) kg ms–1 (2) kg ms–2 (1) 14 minutes (2) 7 minutes
2 –1 –1
(3) kg ms (4) kg m s (3) 24 minutes (4) 28 minutes
Exemplar Practice Questions S-3
34. What should be the minimum height of a plane mirror so 45. Maximum refraction of light rays entering the eye takes place
that a 6ft tall man can see his full view image standing in from :
front of the mirror? (1) Cornea (2) Iris
(1) 6 ft (2) 12 ft (3) Retina (4) Eye lens
(3) 3 ft (4) 2 ft 46. At what position should be the candle kept by Sunidhi to
35. A lens of power + 3D and another of power-1.5 D are placed obtain a virtual image of the candle using concave mirror?
in contact. What would be the focal length of the (1) Between infinity and the centre of curvature.
combination? (2) Between the focus and the centre of curvature
(1) 1.5 m (2) – 0.67 m (3) Between the pole and the principal focus
(3) 0.67 m (4) – 67.0 m (4) Between the infinity and focus
36. The mass of two bodies are 1 kg and 2 kg respectively and 47. According to the new cartesian sign convention, the ..............
their kinetic energy are 1J and 2J respectively. The ratio of is taken as origin.
their momentum is given by (1) Centre of curvature
(1) 1 : 1 (2) 1 : 4 (2) Pole of the mirror
(3) 4 : 1 (4) 1 : 2 (3) Focus
37. Which of the following principles has been followed to (4) Any point on the principal axis
propagate light waves through an optical fibre? 48. The refractive index of alcohol is :
(1) Difraction (2) Interference
(1) 1.44 (2) 1.46
(3) Total internal reflection (4) Scattering
(3) 1.47 (4) 1.50
38. The frequency of light of wave length 5000 Å as
(1) 1.5 × 105 Hz (2) 6 × 108 Hz 49. One Newton is the force which produces an acceleration of
14
(3) 6 × 10 Hz (4) 7.5 × 1015 Hz (1) 1 m. s–2 on a body of mass 1 gm
39. Five resistances are joined as shown in the figure. The (2) 1 cm. s–2 on a body of mass 1 kg
equivalence resistance between the points A and C is (3) 1cm. s–2 on a body of mass 1 gm
B (4) 1m. s–2 on a body of mass 1 kg
3
2 50. Two objects of mass ratio 1 : 4 are dropped from the same
A height. The ratio between their velocities when they strike
7 C the ground is
4
(1) Both objects will have the same velocity
6 (2) The velocity of the first object is twice that of the sec-
D
ond one
10
(1) (2) 22 (3) The velocity of the 2nd object is one fourth of that of the
3
1st object
(3) 15 (4) 10.6
40. If the decay constant of radioactive radium is 4.28 × 10–4 year–1, (4) The velocity of the 2nd object is 4 times that of the 1st one
its half life will be 51. The electromagnetic radiations used for taking photographs
of objects in dark
(1) 2000 years (2) 1240 years
(1) X-rays (2) Infra-red rays
(3) 63 years (4) 1620 years
(3) -rays (4) UV rays
41. 1 kilowatt hr. = ................................... 52. The work done by a force on a body will be positive if the
(1) 10.6 × 106 Joule (2) 3.6 × 106 Joule body
(3) 30.6 × 106 Joule (4) 3.6 × 105 Joule (1) moves perpendicular to the direction of applied force
42. Find the odd one out : (2) does not move
(1) Constantan (2) Manganin (3) moves along the direction of applied force
(3) Nichrome (4) Chromium (4) moves opposite to the direction of applied force
43. Which of the following instruments is used to measure the 53. Identify the energy changes in the following two cases-
electric current? A : A car moving up a hill
(1) Voltmeter (2) Ammeter B : Photographic film is exposed to sun-light
(3) Barometer (4) Galvanometer (1) In ‘A’ mechanical energy in moving car is converted to
44. Household electrical appliances are joined using .............. potential energy and in ‘B’ potential energy is converted
combination of resistors. to chemical energy
(1) Parallel (2) Alternating (2) In ‘A’ potential energy in moving car is converted to
kinetic energy and in ‘B’ chemical energy is converted to
(3) Continuous (4) Series
light energy
EBD_7332
4
S- NTSE Stage 1 Question Bank

(3) In ‘A’ kinetic energy in moving car is converted to 65. One Joule is equivalent to :
potential energy and in ‘B’ potential energy is converted (1) 4.18 calories (2) 4.12 calories
to light energy (3) 0.239 calories (4) 0.283 calories
(4) In ‘A’ kinetic energy in moving car is converted to 66. The figure given below is a plot of lines of force due to
Charges q1 and q2. Figure out the Sign of the two Charges:
potential energy and in ‘B’ light energy is converted to
chemical energy
54. The instrument that is based on the principle that when an
object is placed between first principal focus and the optic q1
centre of convex lens, an upright, virtual and enlarged image
on the same side of the object is formed, is
(1) Telescope (2) Projector
(3) Camera (4) Simple microscope q2
55. A charge of 1000 C flows through a conductor for 3 minutes
and 20 seconds. Find the magnitude of current flowing
through the conductor
(1) 5A (2) 2A (1) both positive
(3) 0.5 A (4) 10 A (2) both negative
56. The particle with mass equal to 9.1 × 10–31 kg and charge (3) q1 positive and q2 negative
equal to – 1.6 × 10–19 C is… (4) q2 negative and q1 positive
(1) (2) 67. Two bulbs X and Y painted black and white respectively are
(3) (4) X filled with air and connected by a U tube partly filled with
57. The error and the correction to be made when the zeroth alcohol. What happens to levels of alcohol in the limbs X
division of the head scale in a screw gauge is above index and Y. When an electric bulb placed mid way between the
line of the pitch scale respectively are bulbs is lighted :
(1) positive, negative (2) negative, negative
(3) negative, positive (4) positive, negative
58. The isotopes that emit these radiations are used as radioac- Painted
tive tracers in medical science Painted White
(1) – radiations (2) – radiations Black
(3) – radiations (4) All the three
59. Einstein’s Photo electric equation :
1 1
(1) h = mv2 + W (2) h = mv2 – W Alcohol
2 2
1 1
(3) h = mv + W (4) h = mv – W
2 2
60. Kepler’s third law of planetary motion : (1) The level of alcohol falls in both the limbs
(2) The level of alcohol in limb X rises while that in limb Y
(1) V0 = Rg (2) F = – F flls
(3) r 2 T 3 (4) r3 T2 (3) The level of alcohol in limb X falls while that in Y rises
61. Fronhoffer’s spectral lines are also called : (4) There is no change in the levels of alcohol in the two
(1) Line absorption spectra limbs
(2) Line emission spectra 68. The uncharged Metallic Sphere X suspended as shown in
(3) Continuous spectra figure. The Metallic sphere is given a push so that it moves
(4) Continuous emission spectra towards the +ve plate. Which of the following statement is
62. Current is flowing throught the circular loop, then the
correct?
imaginary magnetic poles are produced at :
(1) Top and bottom of the loop
(2) Right and left parts of the loop
(3) Both side the plane of loop
(4) Each and every part of the loop –
+ –
63. In the nuclear reaction + –
2 2 3
1H + 1H 2He + X : X stands for : + –
(1) +1 e 0 (2) 0 n 1 + x –
0 + –
(3) –1 e (4) H1 –
+
64. High energetic radiations among the followings : + –
(1) = 0.01 Å (2) = 0.10 Å –
(3) = 1.00 Å (4) = 40.0 Å
Exemplar Practice Questions S-5
(1) X touches the +ve plate and remains in contact with it 78. Which of the following graph corectly represents the energy-
(2) X touches +ve plate and then moves towards negative displacement relationship for a spring-mass system?
plate and emains in contact with it
(3) X moves to and fro between the two plates with a
contact time period
(4) X moves to and fro between the two plates with an PE
inceasing time period.

Energy
69. The most intense man-made light score is :
(1) Laser (2) LED (1)
(3) Maser (4) Mercury Vapour Lamp KE
O
70. Which of the following cannot be accelerated in a cyclotron?
(1) Proton (2) -particle +x
Displacement
(3) Electron (4) Neutron
71. The velocity of particle moving with a uniform speed
changes with time according to the relation v = 2 – 3t + 4t2,
KE
then v/t graph of the particle is a :
(1) straight line parallel to x-axis

Energy
(2) a straight line parallel to y-axis
(3) a parabola (2)
(4) a circle O PE
72. Distribution of electric power from one place to antoher is
done of high A.C. (Alternating Current) Voltage, because +x
Displacement
(1) Wastage of electricity is minimised
(2) The distribution of power is economical
(3) Stealing of electric wires is prevented
(4) It is safe to handle
73. A Pond is covered with a layer of ice and the external KE
temperature is –30°C. The temperature of water in contact
Energy

with lower surface of ice is :


(1) 4°C (2) 0°C (3)
(3) –15°C (4) –30°C PE
74. A car is moving with a velocity of 10 m/sec. Its mass is 1000 O
kg. If the velocity-time graph for this car is a horizontal line +x
parallel to the time axis, then the velocity of car at the end of Displacement
25 sec. will be :
(1) 25 m/sec (2) 40 m/sec
(3) 10 m/sec (4) 250 m/sec
75. According to 3rd Law of Motion which one of the following PE
statement is not true?
Energy

(1) When one object applies force on the other, the other
also applies force on the first object simultaneously
(2) Magnitude of both the force is same (4)
(3) Direction of both the forces is opposite O KE
(4) Both the forces act on one object but in opposite
direction +x
Displacement
76. Two objects are made to fall freely from heights h1 and h2.
The ratio of time taken by them to reach ground is (Neglect 79. An object dropped from top of tower falls through 40 m
air friction) during the last two seconds of its falls. The height of tower
h1 (g = 10 m/s2) will be-
(1) 1 (2) (1) 60 m (2) 45 m
h2
2
(3) 80 m (4) 50 m
h1 h1 80. Spring of spring constant k is cut into n parts. The new
(3) h2 (4) spring constant of each part will be
h2
77. A bullet moving with velocity 'v' penetrates a sand bag upto n
distance 'x'. Same bullet with velocity '2v' can penetate bag (1) nk (2)
k
upto distance:
(1) x (2) 2x k 1
(3) (4)
(3) 3x (4) 4x n nk
EBD_7332
6
S- NTSE Stage 1 Question Bank

81. A balloon has 5g air. A small hole is pierced into it the air The relation between the three measurement would most
escapes at a uniform rate with a velocity of 4cm/s if the likely be :
balloon strikes completely in 2.5s, then the average force (1) f1 = f2 = f3 (2) f1 < f2 and f3
acting on the balloon is
(1) 2 dyne (2) 50 dyne (3) f1 < f2 and f1 = f3 (4) None of these
(3) 8 dyne (4) 8 N 89. A rainbow viewed from an airplane may form a complete
82. A body covers the first half distance with a velocity v and circle. The shadow of the airplane will appear
the second half in double the time taken for first half, then (1) outside the circle of the rainbow
the average velocity is (2) within the circle of the rainbow
(1) v (2) v/2 (3) shadow doesn’t appear with respect to the rainbow
(3) 2 v/3 (4) 2/3v (4) none of these
83. The F-x graph of particle of mass 100g is shown. If particle 90. A lemon is kept in a glass tumbler with water. How does it
begin to move from rest x = 0 its velocity at x = 12 is appear due to refraction of light?
(1) The lemon appears to float above the surface of water.
F
(2) The lemon appears to be smaller than its actual size.
(N) 10 (3) The lemon appears to be bigger than its actual size.
(4) The lemon appears to be of the same size as its actual
size.
91. A given ray of light suffers minimum deviation in an
equilateral prism P. Additional prisms Q and R of identical
0 4 8 12 shape and material are now added to P as shown in the
(Meters) figure. The ray will suffer
x
(1) 10 m/s (2) 20 m/s
(3) 30 m/s (4) 40 m/s
84. A bright × (cross) mark is made on a sheet of white paper.
Over the white paper a rectangular glass-slab of thickness 3
(1) greater deviation (2) same deviation
cm is placed. On looking through, the image of the mark
appears above the mark. It is below the upper surface of the (3) no deviation (4) total internal reflection
slab by 92. Five equal resistors connected in series across a source of
(1) 2.5 cm (2) 1.5 cm e.m.f. dissipate 20 watts of power. What will be the power
(3) 2 cm (4) 1.75 cm dissipated in watts if the same resistors are connected in
85. On keeping the incident ray fixed, the mirror is rotated by an parallel across the same source of e.m.f.?
angle , about an axis in the plane of mirror, the reflected ray (1) 100 W (2) 300 W
is rotated through an angle
(3) 500 W (4) 600 W
(1) /2 (2)
93. A current of 1 A is drawn by a filament of an electric bulb.
(3) 2 (4) 3
86. The linear magnification for a mirror is the ratio of the size of Number of electrons passing through a cross-section of the
the image to the size of the object, and is denoted by m. filament in 16 seconds would be roughly
Then, m is equal to (symbols have their usual meanings): (1) 1020 (2) 1016
f f u (3) 1018 (4) 1023
(1) (2)
f u f 94. In the figure three similar lamps L1, L2 and L3 connected
f f v across a power supply. If the Lamp l3 fuse, how will the
(3) (4) light emitted by L1 and L2 change?
f v f
87. You are given water, mustard oil, glycerine and kerosene. In
which of these media a ray of light incident obliquely at
same angle would bend the most? L1
(1) Kerosene (2) Water E
(3) Mustard Oil (4) Glycerine L3
88. Three student measured the focal length of a convex lens S
L2
using parallel rays from a distant object. All of them measured
the distance between the lens and the inverted image on the
screen.
Student A saw a sharp image on the screen and labelled the (1) L1 increases, L2 increases
distance as f1.
(2) L1 increases, L2 decreases
Student B saw a slightly larger blurred image on the screen
and labelled the distance as f2. (3) L1 decrease, L2 increases
Student C saw slightly larger blurred image on the screen (4) L1 decreases, L2 decreases
and labelled the distance as f3.
Exemplar Practice Questions S-7

95. The graph of the potential difference V against the current I 98. A circular loop placed in a plane perpendicular to the
for a device is as shown below. plane of paper carries a current when the key is ON.
The current as seen from points A and B (in the plane of
paper and on the axis of the coil) is anti clockwise and
V clockwise respectively. The magnetic field lines point from
B to A. The N-pole of the resultant magnet is on the face
close to
(1) A
(2) B
I (3) A if the current is small, and B if the current is large
O
(4) B if the current is small and A if the current is large

Which of the following is the correct deduction? A. B. Variable


(1) The device obeys Ohm’s law. resistance
(2) The resistance of the device increases when the current + –
increases.
(3) The resistance of the device remains constant when
the p.d. increases. + –
(4) The resistance of the device remains constant when A
the current increases.
K
96. To obtain maximum intensity of magnetic field at a point the 99. The nucleus of a heavy atom such as uranium, plutonium or
angle between position vector of point and small elements thorium when bombarded with low-energy neutrons, can
of length of the conductor is be split into lighter nuclei and a tremendous amount of
(1) 0 (2) /4 energy in
(3) /2 (4) (1) nuclear fission, which is a chain reaction
97. Which of the following factors regarding (2) nuclear fusion, which requires large energy to initiate
the reaction
solenoid is incorrect? (3) nuclear warfare
1 (4) biotechnological processes
A. The strength of current B
I 100. Joining of lighter nuclei to make a heavier nucleus, most
B. The number of turns of wire forming a solenoid. B n commonly hydrogen isotopes to helium such as
2 2 3
1 1
H + 1H 1
He + n with release of large amount of energy is
C. Nature of material inside the solevoid B
(1) nuclear fusion that is the source of energy in the Sun
and other stars
(1) A & B (2) A&C (2) nuclear fission
(3) B & C (4) All of these (3) Both (a) and (b)
(4) None of these

SECTION 2. CHEMISTRY
1. The electronic configuration of an ion M2+ is 2, 8, 14. If its (3) Only H2S is oxidized
mass is 56, the number of neutrons in its nucleus is: (4) Both H2S and Cl2 are reduced
(1) 30 (2) 32 5. Which of the following is the functional group for carboxylic
(3) 34 (4) 42 acids?
2. The normality of 0.3 M phosphoric acid is: O
(1) 0.1 (2) 0.9 (1) (2) C
C O
(3) 0.3 (4) 0.6
3. In the presence of concentrated sulphuric acid, acetic acid H
reacts with ethyl alcohol to produce: O
(1) aldehyde (2) alcohol
(3) OH (4) C
(3) ester (4) carboxylic acid
4. H2S(g) + Cl2(g) 2HCl(g) + S(s) OH
The reaction is interpreted as: 6. Which of the following gases is known as tear gas?
(1) H2S is getting oxidized and Cl2 is getting reduced (1) Methyl isocyanide (2) Sulphur dioxide
(2) H2S is getting reduced and Cl2 is getting oxidized (3) Chloropicrin (4) Nitrous oxide
EBD_7332
8
S- NTSE Stage 1 Question Bank

7. The number of atoms in 8g oxygen molecules are : 17. Method used for purifying Petroleum is :
23 23
(1) 6.022 ×10 (2) 3.011×10 (1) Simple distillation (2) Steam distillation
23
(3) 1.51 × 10 (4) 12.044 × 1023 (3) Vacuum distillation (4) Fractional distillation
79 18. Unsaturated hydrocarbon is :
8. Bromine atom is available in two isotopes, 35 Br (49.7%)
81 (1) CH4 (2) C2H6
and 35 Br (50.3%), the average atomic mass of bromine atom
(3) C2H4 (4) C2H5OH
is :
(1) 79.016 (2) 80.076 19. Brass contains :
(3) 80.006 (4) 81.016 (1) Cu and Sn (2) Cu and Ni
9. Which of the following statement is correct? (3) Cu and Zn (4) Mg and Al
(i) German silver is an alloy of silver, copper and zinc. 20. The formula of ammonium sulphate is-
(ii) There is no zinc in brass. (1) NH4SO4 (2) (NH4)2SO4
(iii) Bronze is an alloy of copper and tin. (3) NH4(SO4)2 (4) NH2(SO4)2
(1) (i), (ii) and (iii) (2) only (iii) 21. pH of any neutral solution is-
(3) (i) and (iii) (4) (i) and (ii) (1) 0 (2) 1
10. Two metals which will displace hydrogen and two metals (3) 14 (4) 7
which will not displace hydrogen from dilute acids, 22. Which of the following is correct electronic configuration
respectively are : of argon-
(1) potassium, calcium, aluminium and zinc (1) 2, 8, 3 (2) 2, 8, 8
(3) 2, 8, 1 (4) 8, 2 , 8
(2) sodium, calcium, zinc and iron
23. Which of the following pair is isoelectonic?
(3) zinc, iron, copper and mercury
(1) Na+, Ar (2) Mg2+, Ca2+
(4) copper, mercury, silver and gold +
(3) K , Ar (4) Na+, K+
11. The pH of solution formed by mixing of 40 mL of 0.10 M HCl 24. The atomic number of third alkali metal is _________.
and 10 mL of 0.45 M of NaOH is : (1) 3 (2) 11
(1) 10 (2) 12 (3) 19 (4) 37
(3) 8 (4) 6 25. Silicon carbide is also known as _________.
12. Food cans are coated with tin and not with zinc because: (1) carborundum (2) cyanogen
(1) zinc is costlier than tin (3) silane (4) silicone
(2) zinc has a higher melting point than tin 26. Amount of sodium hydroxide present in 500 mL of 0.2M
(3) zinc is more reactive than tin solution is ________.
(4) zinc is less reactive than tin (1) 4 g (2) 8 g
13. Which statement is correct about a proton ? (3) 40 g (4) 20 g
(1) It is nucleus of deuterium. 27. The mixture of CO and N2 is called as __________.
(2) It is ionised hydrogen molecule. (1) water gas (2) producer gas
(3) synthetic gas (4) marsh gas
(3) It is ionised hydrogen atom.
28. Among the following compounds which one is used for
(4) It is - particle.
cooling refrigerator
14. Which of the following statements is incorrect ? (1) NH4Cl (2) CO2
(1) Charges on an electron and proton are equal and (3) NH4OH (4) liquid NH3 and CO2
opposite. 29. From the following metals whose nitrate produces NO2 gas
(2) Neutron have no charge. on heating
(3) Electron and proton have same mass. (1) Na (2) K
(4) Masses of proton and neutron are nearly the same. (3) Pb (4) None of the above
15. In periodic table generally following similarity is found in 30. Which of the following elements belong to lanthanide series?
elements of same group : (1) Ce (2) Hg
(1) Atomic number (3) Th (4) Mn
(2) Number of electrons in outermost orbit of an atom 31. Which gas is evolved from fire extinguisher?
(3) Number of isotopes (1) CO2 (2) CO
(3) CO3 (4) H2
(4) Atomic volume
32. Atomic number of element ....................... is 19.
16. When two atoms combine to form molecule then :
(1) Mg (2) K
(1) Energy is released (3) Ar (4) Ca
(2) Energy is absorbed 33. Gallium is in ...................... state at room temperature.
(3) Energy is neither released nor absorbed (1) plasma (2) liquid
(4) Energy may either be released or absorbed (3) solid (4) gaseous
Exemplar Practice Questions S-9
34. What is IUPAC name of the following compound? 44. When 500 mL ethyl alcohol (pure) is mixed in 500 mL water
CH3—CH2—CH2—OH (pure), the volume of the mixture will be
(1) Propan-1-ol (2) Propan-2-ol (1) 1.0 L (2) less than 1.0 L
(3) Ethan-1-ol (4) Ethan-2-ol (3) immiscible (4) more than 1.0 L
45. Which of the following statement is not correct?
35. Electronic configuration of copper (atomic No. Cu – 29) is :
(1) Milk is an example of colloidal solution.
(1) 1s22s22p63s23p63d84s24p1 (2) Colloidal solution shows electrophoresis.
(2) 1s22s 22p63s23p63d94s14p1 (3) Colloidal solution is homogeneous mixture.
(3) 1s22s22p63s23p63d84s14p1 (4) Sky is blue due to Tyndall effect.
(4) 1s 22s22p63s23p63d104s14p0 46. Which does not equal to 1 mole of nitrogen gas?
36. The composition of alloy, German silver is : (1) 22.4 L of N2 at STP
(1) Copper, zinc and nickel (2) 28 g of nitrogen
(3) 6.022 × 1023 nitrogen atoms
(2) Copper, tin and lead
(4) 22400 cc. of N2 at STP
(3) Copper, nickel and lead 47. What is cetane number?
(4) Copper, lead and silver (1) Parameter to measure water pollution
37. Which of the following statement is correct? (2) Parameter to measure air pollution
(1) Fusion of 1 gram of hydrogen gives more energy than (3) Parameter to measure quality of diesel
fission of 1 gram of uranium 235. (4) Parameter to measure quality of CNG
(2) Fusion of 1 gram of hydrogen gives less energy than 48. Which of the following contains maximum number of atoms?
(1) 1 g CO2 (2) 1 g N2
fission of 1 gram of uranium 235.
(3) 1 g O2 (4) 1 g CH4
(3) Fusion of 1 gram of hydrogen gives same energy as in 49. The chemical formula of blue vitriol is
fission of 1 gram of uranium 235. (1) Na2SO4.10H2O (2) ZnSO4.7H2O
(4) Fusion of 1 gram of uranium gives less energy than the (3) FeSO4.7H2O (4) CuSO4.5H2O
combustion of 1 ton of coal.
38. The chemical composition of plaster of Paris is : Partition

(1) CaSO4.2H2O (2) CaSO4.H2O 50.


V(L) of V(L) of
(3) CaSO4.1/2H2O (4) CaSO4.3H2O Gas A Gas B
39. The equation given below indicates 400 Torr 400 Torr
TK TK
NaCl + AgNO3 AgCl + NaNO3 :
(1) Chemical decomposition Partition removed
at same temp.
(2) Chemical combination
(3) Chemical displacement
Gas A + Gas B
(4) Chemical double decomposition Pressure = P Torr
40. In the process of vulcanization : TK
(1) Natural rubber is heated with sulphur
(2) Natural rubber is heated with carbon In the bove experiment the final pressure P will be
(3) Natural rubber is heated with phosphorus (1) 400 torr (2) 600 torr
(4) Natural rubber is heated with sodium (3) 800 torr (4) Between 400 and 800 torr
41. The pair of the solutions which have pH value less than 7 : 51. At 20°C the solubility of salt x is 34.7 g in 100 g of water. If
(1) Solution of washing soda and vinegar the density of saturated solution is 1.3 g/mL, the weight/
(2) Solution of soap and solution of washing soda volume (W/V) percentage of solution is :
(3) Solution of copper sulphate and solution of washing (1) 25.76 (2) 32.98
soda (3) 33.49 (4) 22.56
(4) Solution of copper sulphate and vinegar 52. Chemical formula of potash alum is
42. The mass of oxygen required for the complete combustion of (1) K2SO4.Al(SO4)2.24H2O
1.6 kg of methane is : (2) K2SO4.Al2(SO4)2.24H2O
(1) 6.4 kg (2) 3.2 kg (3) K2SO4.Al(SO4)3.24H2O
(3) 1.2 kg (4) 12.8 kg (4) K2SO4.Al2(SO4)3.12H2O
43. Bose-Einstein Condensate have 53. When iron nails are added to CuSO4 solution:
(1) ability to be used as model black-hole (1) A colourless solution is formed
(2) existence at very high temperature (2) A light green coloured solution is formed
(3) isolated gaseous atoms (3) A yellow coloured solution is formed
(4) high kinetic energy (4) No change in colour of solution
EBD_7332
10
S- NTSE Stage 1 Question Bank

54. A student dissolved 50 g sugar in 200 mL water at room 65. Acid present in 'Tamarind' is
temperature. He then heated the solution, till the final volume (1) oxalic Acid (2) formic Acid
became 100 mL. How much sugar is still present in the (3) lactic Acid (4) tartaric Acid
solution? 66. ______ is used to cure "Tumors".
(1) 0 g (2) 25 g (1) Na - 24 (2) I - 131
(3) 50 g (4) 100 g (3) Co - 60 (4) As - 74
55. The oxide of a metal has molecular formula M2O. If the 67. The term ‘rancidity’ represents
molecular weight of oxide is 94 u. The molecular weight of (1) acid rain
chloride salt of this metal will be (2) oxidation of fatty food
(1) 118.5 u (2) 74.5 u (3) rottening of fruit
(3) 114.0 u (4) 110.0 u (4) fading of coloured lothes in the sun
68. Mention the reaction which is utilized to upthrust space
56. X is a yellow coloured non-metal, when X is burnt it produces shuttle.
a pungent smelling gas Y. Gas Y gets mixed with rain water (1) N2(g) + 3H2(g) 2NH3 (g)
to cause acid rain, which is harmful for building and crops (2) CH4(g) + 2O2(g) CO2(g) + 2H2O(g)
both. Identify X and Y. (3) 2H2(l) + O2(l) 2H2O(l)
(1) P4, P2O5 (2) N2, NH3 (4) 2SO2(g) + O2(g) 2SO3(g)
(3) C, CO2 (4) S, SO2 69. A dilute ferrous sulphate solution was gradually added to
57. Orthoboric acid used as mild antiseptic for eyes is the beaker containing acidified permanganate solution. The
(1) H3BO2 (2) H2B4O7 light purple colour of the solution fades and finally
(3) B(OH)3 (4) Na2B4O7 disappears.Which of the following is the correct explanation
58. The diagram given below show the electron arrangement in for the observation?
the outer shell of 1, 2, 3, 4 and 5. All the elements are from (1) KMnO4 is an oxidising agent, it oxidises FeSO4.
period 3 of the periodic table. (2) FeSO4 acts as an oxidising agent and oxidises KMnO4.
(3) The colour disappears due to dilution ; no reaction is
involved.
(4) KMnO4 is an unstable compound and decomposes in
presence of FeSO4 to a colourless compound.
70. Copper on exposure to air reacts with moisture and CO2 to
1 2 3 4 5 develop a green layer which is chemically
Which two elements form a covalent compound with YZ2 (1) basic copper carbonate
type formula. (2) coppper sulphate
(1) 5 and 2 (2) 1 and 2 (4) copper carbonate
(3) 3 and 4 (4) 1 and 5 (4) copper nitrate
59. The label of a breakfast cereal showed that it contained 71. Which of the following gases can be used for storage of
110 mg of sodium per 100 g of the cereal. The mass fresh sample of an oil for a long time?
percentage of sodium in the cereal is : (1) Carbon dioxide or oxygen
(1) 1.10% (2) 0.110% (2) Nitrogen or Oxygen
(3) 0.011% (4) 11.0% (3) Carbon dioxide or helium
60. Arrange the following in order of their increasing calorific (4) Helium or nitrogen
value: 72. Solid calcium oxide reacts vigorously with water to form
(i) Petrol (ii) Wood calcium hydroxide accompanied by liberation of heat. This
(iii) Cow dung cake (iv) Biogas process is called slaking of lime. Calcium hydroxide dissolves
(1) (i) < (ii) < (iii) < (iv) (2) (ii) < (i) < (iv) < (iii) in water to form its solution called lime water. Which among
(3) (iii) < (ii) < (i) < (iv) (4) (iv) < (ii) < (i) < (iii) the following is (are) true about slaking of lime and the
61. Acid present in 'spinach' is solution formed?
(1) oxalic acid (2) lactic acid A. It is an endothermic reaction.
(3) tartaric acid (4) formic acid B. It is an exothermic reaction.
62. In Cu2O, Cu is: C. The pH of the resulting solution will be more than seven.
(1) Monovalent (2) Bivalent
D. The pH of the resulting solution will be less than seven.
(3) Trivalent (4) Neutral
63. ______ radio active isotope is used to determine the activity (1) A and B (2) B and C
of thyroid gland. (3) A and D (4) C and D
(1) Cobalt - 60 (2) Uranium - 235 73. Bleaching powder gives smell of chlorine because it –
(3) Iodine - 130 (4) Iodine - 131 (1) is unstable.
64. Valence of permanganate ion is: (2) gives chlorine on exposure to atmosphere.
(1) 2 (2) 1 (3) is a mixture of chlorine and slaked lime.
(3) 3 (4) 4 (4) contains excess of chlorine.
Exemplar Practice Questions S-11
74. The aqueous solution of aluminium sulphate is: 85. Oils on treating with hydrogen in the presence of palladium
(1) Acidic (2) Basic or nickel catalyst form fats. This is an example of
(3) Amphoteric (4) Both (b) and (c) (1) addition reaction
75. Peroxy disulphuric acid is also known as: (2) substitution reaction
(1) Caro acid (3) displacement reaction
(2) Thiosulphurous acid (4) oxidation reaction
(3) Marshall acid 86. The gases X and Y, have the following properties.
(4) Pyrosulphuric acid • X dissolves in aqueous sodium hydroxide but Y is
76. What is the chemical formula of oleum? insoluble.
(1) H2S2O6 (2) H2S2O7 • Y burns in excess oxygen to give X and water only.
(3) H2S2O5 (4) H2S2O2
• Y does not decolourise aqueous bromine.
77. Compound ‘X’ in the aqueous form reacts with ammonium
What are gases X and Y?
carbonate to give an acidic gas. What is compound ‘X’?
X Y
(1) Calcium hydroxide (2) Ethanoic acid (1) Carbon monoxide Ethene
(3) Sodium hydroxide (4) Sulphuric acid (2) Carbon monoxide Ethane
78. Chemical ‘A’ is used for water softening to remove temporary (3) Carbon dioxide Ethane
hardness. ‘A’ reacts with sodium carbonate to generate (4) Carbon dioxide Ethene
caustic soda. What is ‘A’? 87. Arrange the following elements in the order of their
(1) Gypsum (2) Slaked lime increasing non-metallic character
(3) Quick lime (4) Limestone
Li, O, C, Be, F
79. Aluminium is used for making cooking utensils. Which of (1) F < O < C < Be < Li (2) Li < Be < C < O < F
the following properties of aluminium are responsible for (3) F < O < C < Be < Li (4) F < O < Be < C < Li
the same ? 88. Which of the following is not isoelectronic with O2– ?
(i) Good thermal conductivity (1) N3– (2) Na+
(3) F – (4) Ti+
(ii) Good electrical conductivity
89. Eka-aluminium predicted by Mendeleev is:
(iii) Ductility
(1) Scandium. (2) Gallium.
(iv) High melting point
(3) Germanium. (4) Tellurium.
(1) (i) and (ii) (2) (i) and (iii) 90. Which of the following is an actinide?
(3) (ii) and (iii) (4) (i) and (iv) (1) Plutonium (2) Polonium
80. Silver articles become black on prolonged exposure to air. (3) Promethium (4) Palladium
This is due to the formation of 91. Elements A, B, C, D have same number of valence electrons.
(1) Ag3 N (2) Ag2 O Their melting points are 458K, 370K, 525K and 245K
respectively. Arrange the elements in increasing order of
(3) Ag2 S (4) Ag2S and Ag3N
their atomic numbers.
81. What is anode mud ?
(1) A < B < C < D (2) C < A < B < D
(1) Fan of anode.
(3) D < C < A < B (4) B < D < A < C
(2) Metal of anode.
92. Which of the following is a borderline element?
(3) Impurities collected at anode in electrolysis during
(1) Rubidium (2) Technitium
purification of metals.
(3) Polonium (4) Zirconium
(4) All of these
93. Three elements X, Y and Z form a Dobereiner triad. Their
82. Which of the following is the correct ratio, for mixing conc.
HCl with conc. HNO3 to form aqua regia? atomic weights are in the ratio 5:11:17. If the sum of the
(1) 3 : 1 (2) 1 : 3 atomic weights of extreme elements is 176, then find the
(3) 4 : 1 (4) 1 : 4 atomic weights of X,Y and Z.
83. Among the following groups of oxides, the group that is X Y Z
not reduced by smelting is: (1) 40 80 176
(1) ZnO, Fe2O3 (2) SnO2, PbO (2) 40 88 136
(3) MnO2, ZnO (4) CaO, MgO (3) 40 80 120
84. ‘Drinking alcohol’ is very harmful and it ruins the health. (4) 80 100 120
‘Drinking alcohol’ stands for: 94. Which of the following are not ionic compounds ?
(1) Drinking methyl alcohol (i) KCl (ii) HCl
(2) Drinking ethyl alcohol (iii) CCl4 (iv) NaCl
(3) Drinking propyl alcohol (1) (i) and (ii) (2) (ii) and (iii)
(3) (iii) and (iv) (4) (i) and (iii)
(4) Drinking isopropyl alcohol
EBD_7332
12
S- NTSE Stage 1 Question Bank

95. Which observation most strongly suggests that a solid 98. Ethene and ethane can be distinguished from each other by :
element X is a non-metal? (1) Blue litmus paper
(1) X forms an acidic oxide. (2) Red litmus paper
(2) X has a high melting point. (3) Bromine water
(3) X is a conductor of electricity. (4) Sodium hydrogen carbonate
(4) X reacts vigorously with chlorine. 99. The elements of group sixteen are called
96. Pick up the odd one out. (1) halogens (2) chalcogens
(1) Air (3) pnicogens (4) noble gases
(2) Brass 100. On the basis of following features identify the correct group
(3) A crystal of green vitriol of materials
(4) Gun powder (I) They are corrosive in nature.
97. Pentane has the molecular formula C5H12. It has (II) They have wide industrial applications.
(1) 5 covalent bonds (III) They have sour taste.
(2) 12 covalent bonds (IV) They furnish hydrogen or hydronium ions in aqueous
(3) 16 covalent bonds solutions.
(4) 17 covalent bonds (1) Acids (2) Bases
(3) Salts (4) Metals

SECTION 3. BIOLOGY
1. Mitosis (3) Dominant trait cannot be expressed in heterozygous
(1) leads to recombinant daughter cells condition
(2) is a reduction division (4) Recessive trait cannot be expressed in heterozygous
(3) leads to formation of parental type of daughter cells condition
(4) occurs in gametes 8. Which of the following causes Kalaazar?
2. Mitochondria and Chloroplasts are similar because (1) Leishmania (2) Trypanosoma
(3) Ascaris lumbricoides (4) Helicobacter pylori
(1) Both have nuclei
9. Hydrochloric acid facilitates the action of which enzyme?
(2) Both have 80S ribosomes
(1) salivary amylase (2) pepsin
(3) Both have DNA
(3) Trypsin (4) lipase
(4) Both have single membrane envelope 10. Connective tissue with a fluid matrix is :
3. Cut leaves remain green for longer time if dipped in (1) ligament (2) tendons
(1) Auxin (2) Cytokinins (3) blood (4) cartilage
(3) Ethylene (4) Gibberellins 11. Asexual reproduction takes place through budding in:
4. The enzyme pepsin is secreted by (1) Amoeba (2) Yeast
(1) Inner lining of oesophagus (3) Plasmodium (4) Leishmania
(2) Gastric lining of stomach 12. The experiment conducted by Stanley L. Miller and Harold
(3) Inner lining of duodenum C. Drey in 1953 to show how organic molecules arise in
nature, they assembled an atmosphere consisted of :
(4) Gall bladder
(1) ammonia, methane and oxygen.
5. Pick the odd one out
(2) ammonia, hydrogen sulphide and oxygen.
(1) Down syndrome (2) Haemophilia (3) ammonia, hydrogen sulphide and methane.
(3) Malaria (4) Phenylketonuria (4) methane, hydrogen sulphide and oxygen.
6. Vegetative propagation refers to formation of new plants 13. An example of homologous organs is :
from: (1) our arm and a dog’s fore–leg.
(1) Stem, roots and leaves (2) our teeth and an elephant’s tusks.
(2) Leaves, flowers and seeds (3) potato and runners of grass.
(3) Stem, roots and seeds (4) all of the above
(4) Fruits, seeds and spores 14. Ribosomes are the centre for :
7. Which one of the following is a correct statement ? (1) Respiration (2) Protein synthesis
(1) Dominant trait can be expressed in homozygous (3) Photosynthesis (4) Fat synthesis
condition only 15. Binomial nomenclature was introduced by :
(2) Recessive trait can be expressed in homozygous (1) John Ray (2) Aristotle
condition only (3) A.P. De Candolle (4) Carolus Linnaeus
Exemplar Practice Questions S-13
16. Which of the following is rich in vitamin A ? (3) Phtophosphorylation
(1) Carrot (2) Amla (4) Starch synthesis
(3) Apple (4) Green vegetables 29. The stage of interphase in which the number of cell
17. Which of the following is the high milk yielding variety of organelle increases in
cow ?
(1) G1 (2) S
(1) Holstein (2) Sahiwal
(3) Red Sindhi (4) Mehsana (3) G2 (4) G0
18. Phloem tissues in plants are responsible for- 30. Corpus callosum present in
(1) Transportation of Water (1) Medulla Oblongata (2) Pons
(2) Transportation of Food (3) Cerebrum (4) Cerebellum
(3) Transportation of Amino Acids 31. Isogamy, Aniosogamy and Oogamy types of sexual
(4) Transportation of Oxygen reproduces occur in
19. Which of the following is not a plant harmone- (1) Mucor Sp (2) Monocystis Sp
(1) Auxin (2) Gibberellins
(3) Spirogyra Sp (4) Chlamydomonous Sp
(3) Cytokinin (4) Adrenaline
20. Which of the following two diseases have no specific 32. In which of the following alternatives the correct order of
processes is given?
treatment?
(1) Assimilation Absorption Digestion Ingestion
(1) Hepatitis and influenza
Egestion
(2) Tuberculosis and influenza
(2) Absorption Digestion Ingestion Assimilation
(3) Leprosy and Hepatitis Egestion
(4) Hydrophobia and Tuberculosis (3) Digestion Ingestion Assimilation Absorption
21. Choose the correct pairing of fish and its feeding zone from Egestion
the choices given below. (4) Ingestion Digestion Absorption Assimilation
(1) Rohu-Bottom Zone Egestion
(2) Catla-Middle Zone 33. Show the pathway of electric impulse :
(3) Common carp-Bottom Zone (1) Dendrite Cell body Axon to its end
(4) Mrigal-Surface Zone (2) Cell body Dendrite Axon to its end
22. Which two of the following are growth promoting hormones (3) Axon Cell body Dendrite to its end
in plant? (4) Dendrite Axon Cell body to its end
(1) Auxin and Cytokinin 34. The systematic regulation of various activities means :
(2) Gibberellin and Ethylene (1) Control (2) Co-ordination
(3) Abscisic acid and Auxin (3) Taming (4) Randomness
(4) Cytokinin and Ethylene 35. State the order in which various parts of flower are arranged
23. Which one of the following is having conducting tissue? from outer side to inner side :
(1) Fern (2) Funaria
(1) Carpel Stamen Petals Sepals
(3) Riccia (4) Marchantia
(2) Sepals Petals Stamen Carpel
24. Which one of the following compound contains two carbon
atoms? (3) Petals Sepals Stamen Carpel
(1) Ethanol (2) Pyruvic acid (4) Sepals Petals Carpel Stamen
(3) Lactic acid (4) Glucose 36. All the cats in the world belong to ................ same species.
25 Darwin was most influenced by Research work of (1) Panthera leo (2) Panthera tigris
(1) Alfred Russel Wallace (2) Herbart Spencer (3) Panthera pardus (4) Felis domestics
(3) Malthus (4) Weismann 37. .......................... this animal lays eggs as reptiles but has
26. Which one is more permanent ecosystem? mammary glands and hair on the skin like mammals.
(1) Forest (2) Mountain (1) Duckbill Duckbill (2) Peripatus
(3) Desert (4) Sea (3) Bear (4) Lungfish
27. In photosynthesis maximum amount of starch are produce 38. A fragment of DNA that provides complete information about
at which part? one protein is referred to as ....................... for that protein.
(1) Spongy parenchyma (2) Palisade parenchyma (1) Nucleotide (2) Enzyme
(3) Guard cell (4) Vascular Bundle (3) Nucleoside (4) Gene
28. Which reaction take place in Mitochondria? 39. The process which helps in perpetuation of a race
(1) Oxidative phosphorylation (1) Nutrition (2) Photosynthesis
(2) Photolysis (3) Excretion (4) Reproduction
EBD_7332
14
S- NTSE Stage 1 Question Bank

40. The circulatory system in Cockroach consists of ….. Tropism Stimulus


(1) heart, sinuses and alary muscle (1) Hydrotropism (i) Earth
(2) Cardiac muscle, heart, blood vessels (2) Geotropism (ii) Light
(3) blood vessels, heart, atrium (3) Chemotropism (iii) Water
(4) Veins, heart and atrium (4) Phototropism (iv) Chemical
41. The major natural Auxin is …. (1) 1–ii, 2–i, 3–iv, 4–iii (2) 1–iii, 2–i, 3–iv, 4–ii
(1) IBA (2) 2, 4D (3) 1–iv, 2–iii, 3–i, 4–ii (4) 1–iii, 2–i, 3–ii, 4–iv
(3) IAA (4) NAA 52. The two hormones secreted by adrenal glands which
42. Ramu collected epiphyllous buds from a plant. The plant from regulates the rate of metabolism and the concentration of
which he has collected these buds could be …. salts in blood and prepares the body to face the emergency
(1) Murraya (2) Bryophyllum are _____ and _____
(3) Neem (4) Hibuscus (1) Insulin and Glucagon
43. Identify the correct statement from below (2) Testosterone and Estrogen
(1) The Zygote develops into embryo-sac (3) Cortisone and Dopamine
(2) Synergids are situated near the chalazal end of embryosac (4) Cortisone and Adrenaline
(3) Mature embryo-sac has eight cells 53. Concentrated nitric acid is used in a test to detect adulteration
(4) Secondary nucleus in a mature embryo-sac is diploid of :
44. One of the following is a wrong combination (1) Cooking oil (2) Milk
(1) Paramoecium - Exconjugats (3) Salt (4) Tea powder
(2) Clitellum – Earthworm 54. A biotechnological process that has found a significant
(3) Flies – Internal fertilization application in the field of forensic science is :
(4) Ampluxory pads – female frog (1) Recombinant DNA technology
(2) Tissue culture
45. In humans, disorders of nervous system are caused due to
the deficiency of vitamin (3) DNA finger print technology
(4) Genetic engineering
(1) Vit B12 (2) Retinol
55. The flow of energy from autotrophs to herbiivores is :
(3) Phylloquinone (4) Ascorbic acid (1) Bidirectional (2) Unidirectional
46. The normal systolic and diastolic pressure in human beings (3) Reverse directional (4) Multi directional
is : 56. National Science Day :
(1) 80/120 mm of Hg (2) 130/90 mm of Hg (1) 15th September (2) 14th February
(3) 120/80 mm of Hg (4) 140/80 mm of Hg th
(3) 5 April (4) 28th February
47. While observing the cross section of an angiosperm stem, if 57. The process of carrying food from leaves to other parts of a
you find complex permanent tissue, it could be : plant is called
(1) Epidermal tissue and collenchyma (1) Transpiration (2) Translocation
(2) Sclerenchyma and parenchyma (3) Transportation (4) Transformation
(3) Parenchyman and epidermal tissue 58. Peptic ulcer is caused by
(4) Xylem and Phloem (1) Helicobacter pylori (2) Trypanosoma
48. One of the basic characteristics of living organisms which is (3) Staphylococci (4) None of the above
well developed in nervous system is : 59. The organelle involved in membrane biogenesis is :
(1) Movement (2) Growth (1) Ribosome (2) Mitochondria
(3) Endoplasmic Reticulam (4) Lysosome
(3) Irritability (4) Nutrition
60. Concentrates given as feed to the cattle are :
49. If a person cannot walk in a straight line or cannot balance a (1) high in fibres and proteins
ride on bicycle, probably which part of his brain is not working (2) high in fibres but low in proteins
properly? (3) low in fibres and proteins
(1) Cerebrum (2) Cranium (4) low in fibres and high in proteins
(3) Cerebellum (4) Hypothalamus 61. Due to active immune system local effects like swelling and
50. If the urine excreted by a person contain glucose the condition pain are seen due to :
is called (1) Inflamation (2) Antibiotics
(1) Galacturia (2) Diabetes mellitus (3) Microbia growth (4) Multiplication of
(3) Diabetes insipidus (4) Gluacoma pathogen
51. Choose the correct alternate in which the type of tropism 62. In Nigrogen cycle which bacteria is responsible for
exhibited by plants is correctly matched with the stimulus to nitrification:
which they respond (1) Rhizobium (2) Clostridium
(3) Nitrosomonas (4) Azotobactor
Exemplar Practice Questions S-15
63. The organisms present in the faeces of mammals which are 77. Which ions are involved in clotting of blood?
indicators of quality of water are : (a) Na+ (2) K+
(1) Bacteria (2) Fungus (3) Fe3+ (4) Ca2+
(3) Algae (4) Virus 78. The vitamin which is generally excreted by human is:
64. The antibiotic penicillin inhibits the growth of bacteria by (1) Vitamin - C (2) Vitamin - A
blocking the process which helps to build its : (3) Vitamin - D (4) Vitamin - E
79. What will happen to RBCs, if they are placed in hypertonic
(1) Cell wall (2) Cell membrane
solution?
(3) Endoplasmic Reticulum(4) Mitochondria
(1) The cells will swell up but not burst.
65. The plants having inconspicuous or hidden reproductive (2) The cells will shrink.
organs are placed in : (3) The cells will remain uneffected.
(1) Cryptogamae (2) Phenerogamae (4) The cells will burst.
(3) Angiospermae (4) Gymnospermae 80. A river with high 'Biochemical Oxygen Demand '(BOD) value
66. The fibrous tissue with great strength but limited flexibility is:
is : (1) Highly polluted
(1) Tendon (2) Ligament (2) Highly clean
(3) Catilage (4) Areola Tissue (3) High in oxygen level
67. The structue marked as (A) in the figure below is : (4) None of these
81. Identify the process that requires ATP energy in order to
take place.
(1) Osmosis
(2) Diffusion
(1) Cytosome (2) Water Vacuole (3) Facilitated transport
(3) Food Vacuole (4) Cytopyge (4) Active transport
68. "The Origin of Species" is written by : 82. The filtration of blood in a nephron takes place in,
(1) Charles Brown (2) Lamarck (1) Tubular part (2) Collecting duct
(3) Robert Hooke (4) Charles Darwin (3) Renal artery (4) Bowman’s capsule
69. Dolly Sheep was produced by a technique known as : 83. Sphincter muscle are present in the
(1) Sexual Reproduction (2) Fragmentation (1) Oesophagus (2) Stomach
(3) Regeneration (4) Cloning (3) Small intestine (4) Liver
70. Moss and Ferns are found in moist and shady places, 84. In mammals the urine expulsion organ is called
because they: (1) Ureter (2) Urinary bladder
(1) Need low temperature for nutrition (3) Urethra (4) Nephron
(2) Do not need sun light for photosynthesis. 85. Erythropoesis may be stimulated by the
(3) Require water for fertilisation. deficiency of
(4) Can not compete with sun loving plants. (1) Iron (2) Oxygen
71. The species of plants and animals found exclusively in a (3) Protein (4) None of the above
particular area are called: 86. In which of the following vertebrate group/groups, heart
(1) Endemic (2) Endangered does not pump oxygenated blood to different parts of the
(3) Biological (4) Alien body?
72. Which of the following organism does not follow 'Cell (1) Pisces and amphibians
Theory': (2) Amphibians and reptiles
(1) Bacteria (2) Virus (3) Amphibians only
(3) Fungi (4) Plants (4) Pisces only
73. 'Sleeping Sickness' is caused by: 87. Which of the following is used to remove nitrogen waste
(1) Staphylococci (2) Leishmania products from the blood?
(3) Trypanosoma (4) SARS Virus (1) Ventilator (2) Transfusion
74. Outer covering of Virus is made up of: (3) Hemodialysis (4) Angiogram
(1) Lipid (2) Protein 88. Pepsin activity needs
(3) RNA (4) DNA (1) Acidic medium (2) alkaline medium
75. Which one of the following pigment is most abundant in (3) Neutral medium (4) None of the above
green plants? 89. Name the hormone that lowers the blood sugar level?
(1) Chlorophyll - a (2) Chlorophyll - b (1) Adrenaline (2) Estrogen
(3) Carotene (4) Xanthophyll (3) Glucagon (4) Insulin
76. The cell organelle which involves in detoxification of poison 90. Co-ordination is achieved through nervous system as well
and drugs is: as circulatory system by respective agents like
(1) Golgi Apparatus (1) Neurotransmitters and proteins
(2) Lysosome (2) Neurotransmitters and hormones
(3) Smooth Endoplasmic Reticulum (3) Neurotransmitters and sugars
(4) Rough Endoplasmic Reticulum (4) Sugars and hormones
EBD_7332
16
S- NTSE Stage 1 Question Bank

91. The growth of pollen tubes towards ovules is due to 96. Length of pollen tube depends on the distance between
(1) Hydrotropism (2) Chemotropism (1) Pollen grain and upper surface of stigma
(3) Geotropism (4) Phototropism (2) Pollen grain on upper surface of stigma and ovule
92. The nerve cell has branches that develop during cell (3) Pollen grain in anther and upper surface of stigma
specialization. These branches help the nerve cell to perform (4) Upper surface of stigma and lower part of style
which of the following functions? 97. In human beings, a recessive gene can express it in
(1) Communication with other cells homozygous condition as in
(2) Moving from one location to the other (1) Autosome
(3) Storing extra DNA (2) X-chromosome of male
(4) Exerting force on non-nervous tissue (3) X-Chromosome of females
93. Characters that are transmitted from parents to offspring (4) An autosome or X-Chromosome
during reproduction show 98. Mendel Laws of Heredity were rediscovered by
(1) Only similarities with parents (1) Charles darwin
(2) Only variations with parents (2) Lamark
(3) Both similarities and variations with parents (3) Devries Correns and Tschermark
(4) Neither similarities nor variations (4) Morgan
94. The main method of propagation of banana, orange, rose 99. Genetics is the study of –
and jasmine is (1) Inheritance (2) Cell structure
(1) Sexual reproduction (2) Vegetative reproduction (3) Only plants (4) Only animals
(3) Fission (4) Fusion 100. New species may be formed if
95. What is not common among these between sperm and ova? A. DNA undergoes significant changes in germ cells
(1) Both have nucleus B. chromosome number changes in the gamete
(2) Both are produced in germ cells C. there is no change in the genetic material
(3) Both have 21 chromosomes D. mating does not take place
(4) Both have mitochondria (1) A and B (2) A and C
(3) B, C and D (4) A, B and C

SECTION 4. MATHEMATICS
1. A shopkeeper mixes 80 kg sugar worth of ` 6.75 per kg with (1) 15 (2) 25
120 kg of sugar worth of ` 8 per kg. He earns a profit of 20% (3) 30 (4) 35
by selling the mixture. He sells it at the rate 6. The sum of all two digit numbers each of which leaves
(1) ` 7.50 per kg (2) ` 9 per kg remainder 3 when divided by 5 is
(3) ` 8.20 per kg (4) ` 8.85 per kg (1) 952 (2) 999
1 1 (3) 1064 (4) 1120
2. If x + = 3, then the value of x 6 + is
x x6 7. In right triangle ABC, BC = 7 cm, AC – AB = 1 cm and
(1) 927 (2) 114 B = 90°. The value of cos A + cos B + cos C is
(3) 364 (4) 322 1 32
(1) (2)
3. If the zeros of the polynomial f(x) = k2x2 – 17x + k + 2, (k > 0) 7 24
are reciprocal of each other. then the value of k is 31 25
(3) (4)
(1) 2 (2) –1 25 31
(3) –2 (4) 1 8. The angles of elevations of the top of the tower from two
4. A bag contains 20 balls out of which x are black. If 10 more points in the same straight line and at a distance of 9 m. and
16 m from the base of the tower are complementary. The
black balls are put in the box, the probability of drawing a
height of the tower is
black ball is double of what it was before. The value of x is (1) 18 m (2) 16 m
(1) 0 (2) 5 (3) 10 m (4) 12 m
(3) 10 (4) 40 9. Let P(4, k) be any point on the line y = 6 – x. If the vertical
5. for the distribution segment PQ is rotated about y-axis, volume of the resulting
Marks Number of students cylinder is
Below 5 10
Below 10 25 P
Below 15 37
Below 20 57
Below 25 66 Q

the sum of the lower limits of the median class and the model (1) 32 (2) 16
class is 32
(3) p (4) 8
3
Exemplar Practice Questions S- 17
10. Coordinates of P and Q are (4, – 3) and (–1, 7). The abscissa 19. If a number with 12 has the same ratio as 8 having with 10
then the number is :
PR 3
of a point R on the line segment PQ such that = is (1) 15 (2) 9.6
PQ 5 (3) 7.5 (4) 10
18 17 20. If the sum of the roots of the equation ax2 + bx + c = 0 is
(1) (2) equal to product of their reciprocal, then:
5 5 2 2
(1) a + bc = 0 (2) b + ca = 0
2
17 (3) c + ab = 0 (4) b + c = 0
(3) 1 (4)
8 1
21. cot 2
is equal to :
11. An aeroplane is flying horizontally at a height of 3150 m 2
sin
above a horizontal plane ground. At a particular instant is (1) 1 (2) – 1
passes another plane vertically below it. At this instant, the (3) 2 (4) – 2
angles of elevation of the planes from a point on the ground
22. On the level ground, the angle of elevation of the top of a
are 30° and 60°. Hence, the distance between the two planes
tower is 30°. On moving 20 metres nearer to it the angle of
at that instant is :
elevation becomes 60°. The height of the tower is :
(1) 1050 m (2) 2100 m
(3) 4200 m (4) 5250 m (1) 10 m (2) 15 m
12. Given that a(a + b) = 36 and b(a + b) = 64, where a and b are (3) 20 m (4) 10 3 m
positive, (a – b) equals: 23. If sin + cos = 1, then sin cos is equal to :
(1) 2.8 (2) 3.2 1
(3) –2.8 (4) –2.5 (1) 0 (2)
3 1
a c
13. If a, b, c are positive, then is
b c 1 2
(3) 1 (4)
a 1 3
(1) always smaller than 24. The ratio of the areas of two similar triangles is equal to :
b
a (1) The ratio of corresponding medians
(2) always greater than (2) The ratio of corresponding sides
b
a (3) The ratio of the squares of corresponding sides
(3) greater than only if a > b (4) None of these
b
25. If b2 – 4ac 0 then the roots of quadratic equation
a
(4) greater than only if a < b ax2 + bx + c = 0 is-
b
2 b b2 4ac b b 2 4ac
14. If a b c 0, then the value of a b – c is: (1) (2)
2a 2a 2a 2a
(1) 2ab (2) 2bc
(3) 4ab (4) 4ac b b2 4ac b b 2 4ac
(3) (4)
15. A bag contains 5 red balls and some blue balls. If the 2a 2a 2a 2a
probability of drawing a blue ball is double that of a red ball, 26. If points (x, 0), (0, y) and (1, 1) are Collinear then the relation
then the number of blue balls in the bag are : is-
(1) 20 (2) 15 (1) x + y = 1 (2) x + y = xy
(3) 12 (4) 10 (3) x + y + l = 0 (4) x + y + xy = 0
16. Consider the points A(a, b + c), B(b, c + a), and C(c, a + b) be
the vertices of ABC. The area of ABC is : 3 3
27. If Sin (A + B) = , Cos (A – B) = and 0 < A + B 90°,
(1) 2(a2 + b2 + c2) (2) a 2 b 2 c 2 2 2
(3) 2(ab + bc + ca) (4) None of these if A > B then the value of A and B are-
17. The probability of getting a number greater than 2 by (1) A = 45°, B = 15° (2) A = 60°, B = 30°
throwing a fair dice is : (3) A = 0°, B = 30° (4) A = 30°, B = 0°
(1) 2/3 (2) 1/3 28. A drinking glass is in the shape of a frusturm of a cone of
(3) 1 (4) 3/5 height 14 cm. Two diameter of its two circular ends are 4 cm
18. Which one of the following is a factor of the expression and 2 cm. then the capacity of glass is-
(a + b)3 – (a – b)3 ? 2 1
(1) 102 cm3 (2) 102 cm3
(1) a (2) 3a2 – b 3 3
(3) 2b (4) (a + b) (a – b) 2 1 3
(3) 101 cm 3 (4) 101 cm
3 3
EBD_7332
18
S- NTSE Stage 1 Question Bank

29. If the sum of the two roots of the equation 38. If D is any point on the side BC of a ABC, then
1 1 1 (1) AB + BC + CA > 2AD
x+a x+b b (2) AB + BC + CA < 2AD
is zero, then the product of the two roots is __________. (3) AB + BC + CA > 3AD
(4) None
a2 b2
(1) 0 (2) 39. On increasing each of the radius of the base and the height
2
of a cone by 20% its volume will be increased by _________.
a b (a 2 b 2 )
(3) (4) – (1) 20% (2) 40%
2 2
30. The remainder when 11997 + 21997 + ...... + 19961997 is divded (3) 72.2% (4) 72.8%
by 1997 is _________. 40. Each side of an equilateral triangle is a cm. Find out the ratio
(1) 0 (2) 1 of areas of the circumcircle and the incircle of the triangle
(3) 197 (4) 1996 (1) 1 : 2 (2) 2 : 1
31. If the product of 1000 positive integers is 1000 then their
(3) 4 : 1 (4) 1 : 4
maximum sum is ____________.
(1) 1000 (2) 1999 41. If the measures of sides of a triangle are (x2 – 1) cm, (x 2 + 1)
(3) 2000 (4) 2999 cm, and 2x cm, then the triangle will be
(1) right angled (2) obtuse angled
2 f (n) 1
32. If f (n 1) , n 1, 2, ...... and f (1) = 2, then (3) equilateral (4) isosceles
2
f (101) = ____________. 42. ‘+’ means ‘ ’, ‘ ’ means ‘×’, ‘×’ means ‘–’ and ‘–’ means ‘+’;
(1) 53 (2) 52 what will be the value of 20 5 × 6 + 2 – 10 ?
(3) 51 (4) 50 (1) 107 (2) 100
33. A triangle with integral sides has perimeter 8 units. The area (3) 92 (4) 114
of the triangle is __________ sq units.
43. The height of a cone is 30 cm. A small cone is cut off by a
(1) 2 (2) 2 2 1
plane parallel to its base. If its volume be th of the volume
(3) 3 2 (4) 4 27
of the given cone, at what height above the base is the
34. At a party of married couples, each man shook hand with section cut?
everyone except his spouse and no handshakes took place
(1) 21 cm (2) 20.5 cm
between women. If 13 married couples attended the party,
the number of handshakes among these 26 people was (3) 18 cm (4) 20 cm
__________. 44. Circumference of a circle is S cm. and its area is A sq.cm.
(1) 230 (2) 22 which one of the following relations is true?
(3) 233 (4) 234
(1) S2 > 4A (2) S2 < 2 A
x x x x x x
35. If the median of , , , x, , , is 8, then the value of (3) S = 4 A (4) S2 = 4 A
7 5 6 4 3 2 2
x is __________. 45. If x – 6x + 5 < 0, then real values of x satisfies
(1) 8 (2) 24 (1) 1 < x < 5 (2) 3 < x < 7
(3) 32 (4) 48 (3) 4 < x < 5 (4) 7 < x < 10
36. If m = 7777 ……… 7777 is a 99 digit number and n = 999
………. 999 is 77 digit number then the sum of the digits in 46. If the sum of three consecutive terms in a geometric
the product m × n is __________. progression is 21 and their product is 216, then which of the
(1) 890 (2) 891 following is the middle term of the given geometric
(3) 892 (4) 893 progression?
37. In the given figure m A + m B + m C + m D + m E +
m F + m G = _________. (1) 3 (2) 12
(3) 6 (4) 8
7 5
47. If x y 5 ; 4x + 2y = 7, then what is the value of
2 2
x–y?
(1) 1 (2) 4
(1) 360° (2) 500° (3) 2 (4) –2
(3) 520° (4) 540°
Exemplar Practice Questions S-19
48. In the figure given below, point O is orthocentre of ABC 56. Equation of the line passing through (– 1, 2) and perpendicu-
and points D, E and F are foot of the perpendiculars, then lar to x – y + 2 = 0 is ….
how many sets make the 4 cyclic points from the point O? (1) x + y =1 (2) x – y =1
A (3) x + y = 2 (4) x – y + 1= 0
57. If A(– 2,5) and B(3,2) are the two points on a straight line. If
AB is extended to ‘C’ such that AC = 2BC, then the co-
ordinates of ‘C’ are ….
F E
O
1 3 7 1
(1) , (2) ,
2 2 2 2
(3) (8, –1) (4) (–1, 8)
B D C 58. Mean of 100 items is 49. It was found that the three items
(1) 4 (2) 3 which should have beeen 60, 70, 80 were wrongly read as 40,
(3) 2 (4) 6 20, 50 respectively. The correct mean is
49. Equations of the three sides of a rectangle are x = 4.5, y = –3, (1) 48 (2) 49
x = –3.5 and length of a diagonal is 10 cm. What is the (3) 50 (4) 60
equation of fourth side? 59. The value of ‘P’ for which the equation
(1) y = –9 (2) y = 2
a2 + 4 = (P + 2)x has equal roots?
(3) y = –6 (4) y = 5
(1) (2, – 6) (2) (– 2, + 6)
9 6 (3) (–2, – 6) (4) (2, 6)
50. If = a ; =b , then the relation between a and b
11 2 3 3 60. There are 3 hockey players Vinay, Raja and Jayanth, who are
is …. equally capable of scoring a goal in a match, it was known
(1) a < b (2) a > b that the team scored 3 goals, then the probability that Vinay
scored one goal and Raja scored two goals is
(3) a + b >1 (4) a b
51. If ‘r’ and ‘s’ are the roots of the equation ax2 + bx + c = 0, then 1 6
(1) (2)
1 1 27 27
the value of is….
r2 s2 2 1
(3) (4)
b2 4ac 9 10
(1) b2 – 4ac (2) 61. The ratio of the area of a square to that of the square drawn
2a on its diagonal is :
b2 4ac b2 2ac (1) 1 : 1 (2) 2 : 1
(3) 2 (4) 2
c c (3) 1 : 2 (4) 1 : 4
52. When the sum of the first ten terms of an A.P. is four times the 62. ABCD is a cylic quadrilateral, then the angles of the
sum of the first five terms. Then the k term is….. quadrilateral in the same order are :
(1) a (2k +1) (2) a(2k –1) (1) 70°, 120°, 110°, 60° (2) 120°, 110°, 70°, 60°
(3) 2k +1 (4) 2k + 3 (3) 110°, 70°, 60°, 120° (4) 60°, 120°, 70°, 110°
4 4 1 1
3 6 6 3 63. If x + = a + b, x – = a – b then
53. The value of a9 a9 is …. x x
(1) ab = 1 (2) ab = 2
(1) a16 (2) a12
(3) a + b = 0 (4) a = b
(3) a 8
(4) a4
54. The sum of three numbers is 98. The ratio of the first to the 64. If fifth term of GP is 2, then the product of its 9 terms is:
2 (1) 256 (2) 512
second term is and the ratio of the second to the third is
3 (3) 1024 (4) none of these
5 65. If , are the roots of the equation ax2 + bx + c = 0, then
. Then the second number is
8
(1) 15 (2) 20 ?
b a b
(3) 30 (4) 32 (1) 2/a (2) 2/b
55. A cylindrical pencil of diameter 1.2 cm has one of its ends (3) 2/c (4) –2/a
sharpened into a conical shape of height 1.4 cm. The volume
of the material removed is (in cub. cms)…. 66. If the length of the chord of a circle is equal to the radius,
then the angle subtended by it at the centre is :
(1) 4.224 (2) 1.056
(3) 10.56 (4) 42.24 (1) 30° (2) 90°
(3) 60° (4) 120°
EBD_7332
20
S- NTSE Stage 1 Question Bank

67. The coach of a cricket team buys 7 bats and 6 balls for ` 77. If x2 + 7ax + 40 and x2 + 2ax – 60 has a common factor, then
3,800. Later he buys 3 bats and 5 balls for ` 1,750. The cost of the value of 'a' is
each bat and each ball is : (1) ±1 (2) ±2
(1) ` 3,800, ` 1,750 (2) ` 50, ` 500 (3) ±3 (4) ±4
(3) ` 500, ` 50 (4) ` 2,150, ` 750 78. If 642x – 5 = 4 × 8x – 5, then the value of x is:
68. What is the remainder when (x4 + 1) is divided by (x – 2) ?
(1) 17 (2) 15 17 17
(1) (2)
(3) 7 (4) 1 9 10
69. The number of coins of 1.5 cm in diameter and 0.2 cm thick, to
be melted to form a right circular cylinder of height 10 cm and 20 9
diameter 4.5 cm is : (3) (4)
9 17
(1) 150 (2) 250 79. If a = b3x, b = c3y and c = a3z, then value of xyz is
(3) 450 (4) 500
1
1 1 1 1 (1) 27 (2)
70. If + + = where (a + b + c) 0 and abc 0. 27
a b c a +b+c
What is the value of (a + b) (b + c) (c + a)? 1
(3) 9 (4)
(1) 0 (2) 1 9
(3) – 1 (4) 2
71. The value of x satisfying the equation 1 21 4 8
80. Value of is
x2 + p2 = (q – x)2 is : 1 x 1 x 1 x2 1 x4 1 x8

p2 q 2 16 8
q 2 p2 (1) (2)
(1) (2) 1 x16 1 x16
2 2q
16 32
q2 p2 p2 q2 (3) (4)
(3) (4) 1 x16 1 x16
2q 2p
81. If x + y = 8, xy = 15, then the value of x4 + x2y2 + y4 is
72. On a road, three consecutive traffic lights change after 36, 42
(1) 34 (2) 1156
and 72 seconds. If the lights are first switched on at 9.00 am,
at what time will they change simultaneously? (3) 931 (4) 1381
(1) 9 : 08 : 04 (2) 9 : 08 : 24 82. The coefficient of x2 in the expansion of (x2 – x + 1)2 + (x2 +
x + 1)2 is
(3) 9 : 08 : 44 (4) 9 : 08 : 58
(1) 6 (2) 5
73. AB and CD are two parallel chords of a circle of radius 3 cms.
If AB = 4 cms and CD = 5 cms. Then the distance between (3) 4 (4) 3
them in cms is 83. A farmer can plough a farm in 10 days by working 5 hours a
day. In how many days can 5 farmers plough 10 such farms
5 working at 5 hours a day?
(1) 11 (2) 5 11
2 (1) 20 days (2) 25 days
(3) 15 days (4) 24 days
11 11
(3) 5 (4) 2
2 5 a 4 b4 c 4
84. If a + b + c = 0, then the value of 2 2 2 2
74. The value of (256)0.16 × (256)0.09 is : a b c (a b 2 )
(1) 2 (2) 8 (1) 0 (2) 1
(3) 6 (4) 4 (3) 2 (4) 4
2 1 85. If x = (–23) + 22 + (–23) + 22 + ..... (40 terms) and y = 11 + (–
75. If x2 – 5x – 1 = 0 then the value of x is 10) + 11 + (–10) + .... (20 terms) then value of (y – x) is
x2
(1) 20 (2) 27 (1) 30 (2) 40
(3) 25 (4) –25 (3) 20 (4) 10
1/ 2 86. If abc = 1, then the value of
x6 y 3 x 1 y2 a b
76. If x y , then the value of (a + b 1 1 1
x 2 y3 x3 y 2 1 1 is
1 a b 1 b c 1 c a 1
+ 1) is
(1) 0 (2) –1
(1) 0 (2) 2
(3) –1 (4) –2 (3) 1 (4) 1 + a + ab
Exemplar Practice Questions 21
S-

87. Ajay wanted to type the first 180 natural numbers. Find the 95. XYZ is a triangle right angled at X. XP YZ. The length
number of times he had to press the numbered keys: of perpendicular XP drawn on YZ is
(1) 416 (2) 430
(3) 432 (4) 448 X
88. If x = 3 + 3 + 3 , then the value of x - 9 x + 18 x -12
1/3 2/3 3 2

is
63 16
(1) 0 (2) –1
(3) 1 (4) 2
89. If figue, AC = CB = AD, then the value of x is :
Y Z
P
65

A
99° (1) 15.5 units (2) 13.5 units
(3) 10.5 units (4) 15.0 units
96. If 9x – 2 = 3x + 1 , then the value of 21+x is
x (1) 64 (2) 32
B
C D (3) 16 (4) 5
97. If x + y + z = 2, xy + yz + zx = –1 and xyz = –2, then value of
(1) 49.5° (2) 30°
(3) 33° (4) 49° x3 + y3 + z3 is :
(1) 20 (2) 16
æ 3 ö÷1-2 x 17
ç = 4 , then the value of x is
(3) 8 (4) 0
90. If ççè 5 ø÷÷ 27 98. In the given figure, if AF || BE || CD, AF = 7.5 cm, CD = 4.5 cm,
ED = 3 cm, BE = x cm, AE = y cm, then value of x and y are
-2 2
(1) (2)
7 7 F

-7 7
(3) (4) D
2 2 3 cm
E
7.5 cm

91. The value of 0.2 + 0.23 is :

4.5 cm
y
(1) 0.43 (2) x
0.43
A B C
(3) 0.45 (4) 0.45
92. Travelling at (4/5)th of his usual speed, a man is 15 minutes
13
late. His usual time to cover the same distance is: (1) x 2
cm, y 3 cm
16
(1) 45 minutes (2) 60 minutes (2) x = 3 cm, y = 5 cm
(3) x = 5 cm, y = 3 cm
(3) 75 minutes (4) 90 minutes
13
93. If (x + 3) and (x + 2) are factors of 2 x 3 + mx 2 + 7 x + n, then (4) x 2 cm, y 5 cm
16
the values of m and n are :
1 1
(1) m = 9, n = –6 (2) m = –9, n = 6 99. If xa = yb = zc and y2 =zx, then the value of is
a c
(3) m = –9, n = –6 (4) m = 9, n = 6 b c
(1) (2)
94. A man buys apples at a certain price per dozen and sells 2 2
them at eight time that price per hundred, find his gain or 2 2
(3) (4)
loss percent b a
100. For the equation, 2a + 3 = 4 a+2 – 48, the value of a is
(1) Gain 4% (2) Loss 4% (1) 0 (2) 1
(3) Gain 5% (4) Loss 5% (3) –1 (4) –2
EBD_7332
22
S- NTSE Stage 1 Question Bank

SECTION 5. HISTORY
1. Until medieval times Jews lived in separately marked areas 13. Which one of the following is the ancient name of Tokyo?
known as: (1) Osaka (2) Nagam
(1) Ghettos (2) Lebensraum (3) Edo (4) Gifu
(3) Synagogues (4) Gas Chambers 14. In which city of India the first cotton mill was established?
2. The ‘slavs’ belong to a geographical region: (1) Ahmadabad (2) Surat
(1) Russia (2) Balkans (3) Bombay (Mumbai) (4) Kanpur
(3) Hungary (4) Germany 15. ‘Raikas’ the Pastoral community lived in which of the Indian
3. A city that was a group of seven Islands under Portuguese state?
control in the Seventeenth century: (1) Andhra Pradesh (2) Jharkhand
(1) Surat (2) Bombay (3) Chhattisgarh (4) Rajasthan
(3) Calcutta (4) Madras 16. How was Jamil related to Swami Mahavir ?
4. The novel that deals with Indian Peasantry:
(1) Son (2) Friend
(1) Godan (2) Rangbhoomi
(3) Son-in-law (4) Father
(3) Anandmath (4) Padmarag
17. Who is regarded as Light of Asia ?
5. Samburu National Park is in:
(1) Gautam Buddha (2) Gandhiji
(1) Kenya (2) Tanzania
(2) Swami Mahavir (4) Mao-Tse Tung
(3) Namibia (4) Zimbabwe
6. The ‘Swaraj flag’ designed by Gandhiji consorted of 18. God Rudra mentioned in Rigveda is :
following three colours: (1) Brahma (2) Vishnu
(1) red, green and white (2) red, white and green (3) Mahesh (4) Yamraj
(3) red, yellow and green (4) red, white and yellow 19. When was Hajrat Mohammad born ?
7. Which of the following common foods did not come from (1) 550 A.D. (2) 560 A.D.
the ‘New World’? (3) 570 A.D. (4) 580 A.D.
(1) Potatoes and Tomatoes 20. Who was the author of the book 'Divine Comedy' ?
(2) Maize and Chillies (1) Acquinas (2) Marsilio
(3) Groundnut and Sweet Potatoes (3) John of Paris (4) Dante
(4) Sugarcane and shali variety of rice
21. Who was the founder of British colonial empire in America ?
8. Indicate the correct chronological order in which of the
(1) James I (2) Edward I
following artisans helped in cloth production?
A. Stapler (3) George V (4) Charles II
B. Weavers 22. Who was the author of the book 'Republic' ?
C. Fullers (1) Aristotle (2) Socrates
D. Spinners (3) Machiavelli (4) Plato
E. Dyers 23. "Power corrupts and absolute power corrupts absolutely."
(1) A — E — B — D — C (2) A — C — B — E — D Who said it ?
(3) A — B — C — E — D (4) A — D — B — C — E (1) Lord Acton (2) Abraham Lincoln
9. Who wrote the book ‘Hind Swaraj’? (3) Garner (4) Easton
(1) Pt. Jawahar Lal Nehru 24. In which year India's rule was given to the British Crown ?
(2) Moti Lal Nehru (1) 1773 (2) 1813
(3) Mahatma Gandhi (3) 1833 (4) 1858
(4) Subhash Chandra Bose 25. In 1913, Dada Saheb Phalke made the movie-
10. When the French Revolution was took place? (1) Basant (2) Raja Harishchandra
(1) 1789 (2) 1786 (3) Anari (4) Paying guest
(3) 1795 (4) 1781
26. What is Guillotine?
11. The “Great Depression was a period of
(1) Political crisis (2) Global crisis (1) Mine of Coal (2) Human Settlement
(3) Social crisis (4) Economic crisis (3) Death Machine (4) Shifting Agriculture
12. Printing was first developed in : 27. Who opened the first Cricket Club in India?
(1) Japan (2) Portugal (1) Britisher (2) Hindu
(3) China (4) Germany (3) Jurestreian (4) Muslim
Exemplar Practice Questions S-23

28. In which year Indian National Congress was established? 42. Through whose writings did the people find an expression
to their feelings in France?
(1) 1889 (2) 1885
(1) Montesquieu (2) Voltaire
(3) 1985 (4) 1905
(3) Necker (4) Calon
29. Who had given the title of “Mahatma” to “Gandhiji”? 43. Which of the following events does not occur due to the
(1) Bal Gangadhar Tilak (2) Gopal Krishna Gokhle cold war?
(3) Motilal Nehru (4) Rabindra Nath Tagore (1) Partition of Germany (2) Indo-China War
30. Who was the political guru of Mahatma Gandhi? (3) Partition of Korea (4) America-Iraq War
(1) Bal Gangadhar Tilak (2) Dadabhai Naroji 44. Which event led to the end of feudalism?
(1) Constantinople’s fall
(3) Badruddin Tyabji (4) Gopal Krishna Gokhale
(2) First Religious War
31. Who among the following was responsible for the
(3) French Revolution
foundation of I.N.A.
(4) American War of Independence
(1) Ras behari Bose (2) Subhas Chandra Bose
45. The ‘Cutting of the Chinese Melon’ means :
(3) Captain Mohan singh (4) Major Fuzihara
(1) Equal rights to trade anywhere in China for western
32. Who among the following leaders was associated with the countries.
Revolt of 1857?
(2) Process of division of China due to western powers
(1) Azimullah Khan (2) Ahmed Shah
(3) China opened few ports for westerners
(3) Kunwar Singh (4) Sir Syed Ahmed Khan
(4) Special rights for trade given to America by China
33. Who wrote ‘Das Capital’? 46. Arrange the following events in chronological order :
(1) Tolstoy (2) Lenin (i) Existence of Independent Vietnam
(3) Karl Marx (4) Kerensky (ii) Existence of Bangladesh
34. The first convention of Facist party was held at _______ (iii) South Rhodesia became independent
(1) Rome (2) Vatican City (iv) Burma became independent
(3) Milan (4) Venice (1) (iv), (ii), (i), (iii) (2) (i), (ii), (iii), (iv)
35. Harappa is situated in (3) (ii), (iv), (iii), (i) (4) (iii), (i), (iv), (ii)
(1) Punjab (2) Sindh 47. The League of Nations settled boundary dispute between
(3) Baluchistan (4) Gujarat which countries?
36. ‘Chaitanya Charitarnrito’ was composed by (1) Sweden-Finland (2) Greece-Bulgaria
(3) Iraq-Turkey (4) Iran-Poland
(1) Brindavan Das (2) Krishnadas Kabiraj
48. Which statement is not applicable for capitalism?
(3) Maladhar Basu (4) Poet Chandidas
(1) Economic system based on mass production
37. The famine of Seventysix in Bengal occurred in (2) Means of production are Government owned
(1) 1768 A.D. (2) 1770 A.D. (3) Economic system based on mass distribution
(3) 1772 A.D. (4) 1774 A.D. (4) The capitalist aims at maximizing profits.
38. ‘Mitra Mela’ was later on renamed as 49. Which symbol represents ‘Fascism’?
(1) Anusilan Samiti (2) Abinava Bharat
(3) Jugantar (4) Hindu Mela
39. Not as a member of ‘Hindustan Socialist Republican (1) (2)
Association’.
(1) Sukdev (2) Bhagat Singh
(3) Rajguru (4) Lala Lajpat Rai
40. ‘Indian Penal Code’ was prepared during the time of (3) (4)
(1) Lord Cornwallis (2) Lord William Bentinck
(3) Lord Dalhousie (4) Lord Canning 50. ‘Father Gapon’ is related to which event?
41. Which time period is known as the ‘Revolutionary Age’? (1) Leader of Petrograd Workers
(1) 1st half of 19th century
(2) Confiscation of landed estates
(2) 2nd half of 19th century
(3) Experimentation of collective farming
(3) 1st half of 20th century
(4) The Bloody Sunday
(4) 2nd half of 20th century
EBD_7332
24
S- NTSE Stage 1 Question Bank

51. “Crusader”, means (2) Satyartha Prakash (2) Ishwara Chandra


(1) A procedure adopted to propagate religion Vidya Sagar
(2) Priests (3) S.N.D.T. University (3) Dayananda
(3) Local Governors during Roman’s period Saraswathi
(4) Religious wars fought between Christians and Muslims (4) Poona Sarvajanika Sabha (4) Jyothibha Phule
52. In western countries the influence of Renaissance took place (5) Mahadeva
between Govinda Ranade
(1) 500 – 1500 A.D. (2) 500 – 1800 A.D. (1) A-3, B-4, C-2, D-5 (2) A-4, B-2, C-1, D-3
(3) 1300 – 1500 A.D. (4) 1300 – 1800 A.D. (3) A-5, B-1, C-4, D-2 (4) A-4, B-3, C-1, D-5
53. This was not the guiding principle of Congress of Vienna
63. The First Newspaper published in India was :
(1) Restoration and legitimacy
(1) The Bengal (2) The Bengal Gazette
(2) Balance of power and compensation
(3) The Hindu (4) The Indian Mirror
(3) Rewards and punishments
64. Which of the following is related to sumptuary laws?
(4) Principles and feelings of Nationality
(1) wars (2) clothes
54. He entered Prussian civil service, but returned out with the (3) foods (4) none of these
remark, “deficiency in regularity and discipline”
65. Who among the following were well-known skilled forest
(1) Napoleon (2) Karl Marx cutters?
(3) Bismarck (4) Louis Blanc (1) Maasais of Africa (2) Kalongs of Jawa
55. The immediate cause for the First World War (3) Gonds of Orissa (4) Mundas of Chota Nagpur
(1) Aggressive Nationalism 66. Which of the following Pastoral groups lived in the region
(2) Imperialism of Kenya and Tanzania?
(3) Secret alliance (1) Somali (2) Bedouins
(4) Murder of the crown prince of Austria (3) Maasai (4) Boran
56. The slogan “Jai Hind” was given by : 67. Which Indian city is known as Manschester of India?
(1) Lal Bahadur Shastri (2) Jawaharlal Nehru (1) Delhi (2) Kolkata
(3) Surat (4) Ahmedabad
(3) Subhash Chandra Bose (4) Ras Behari Bose
68. Who among the following first propounded the idea of
57. Hydro-Electric Power provided in Shivana Samudra was first 'Basic Education'?
of its kind in the whole of Asia. The Diwan of Mysore (1) Jawahar Lal Nehru
responsible for this is (2) Raja Ram Mohan Rao
(1) Dewan Rangacharlu (2) Sir M. Visweswaraiah (3) Mahatma Gandhi
(3) Dewan Sheshadri Iyer (4) Sir Mirza Ismail (4) Dayanand Saraswati
69. Who started schools under a new teaching scheme, known
58. The dictatorship of the proletariat as the only solution for all
as 'Nai Taleem'?
feudal problems was advocated by :
(1) Mahatma Gandhi
(1) Czar Nicholas II (2) Karl Marx
(2) Rabindranath Tagore
(3) Vladimir Lenin (4) Joseph Stalin
(3) Gopal Krishna Gokhale
59. Israel was proclaimed as a New State in 1948 after dividing :
(4) Sir Sayed Ahmad Khan
(1) Saudi Arabia (2) Palestine 70. Who among the following was known as 'Lokhitwadi'
(3) Syria (4) Jordan (1) Jyoba Phule
60. The State of Mysore came to be called as Karnataka during (2) Gopal Hari Deshmukh
the Chief Ministership of : (3) Swami Vivekananda
(1) Sri Devaraj Urs (4) Mahadev Govind Ranade
(2) Sri S. Nijalingappa 71. The 1878 Act divided forests into ..........
(3) Sri Kengal Hanumanthaiah (1) Four categories (2) Two categories
(4) Sri Ramakrishna Hegde (3) Three categories (4) Five categories
61. Sir Charles Wood’s Despatch on Educational reforms led to 72. Under which Governor General was the Indian Penal Code
the establishment of first three universities in India. they were drawn up:
established during the year : (1) Lord Macaulay (2) Lord Curzon
(1) 1854 (2) 1857 (3) Lord Dufferin (4) Lord Minto
(3) 1858 (4) 1904 73. The Weimar Republic in Germany was formed after the :
62. Match List I with List II by using the code given below : (1) Battle of Water Loo
List I List II (2) Death of Bismark
(1) Satyashodak Samaj (1) Dhondo Keshav (3) End of the First World War
Karve (4) Break of the Second World War
Exemplar Practice Questions S-25
74. An infamous film, which was made to create hatred for Jews (3) It was not the result of a sudden upheavel or revolution,
was? but the result of a long-drawn-out process.
(1) The Essential Jew (2) The Evergreen Jew (4) All the above
(3) The Eternal Jew (4) The Emigrant Jew 85. When and how French Indo-China was formed?
75. Who questioned the state ownership of the forest in Java? (1) French Indo-China was formed in 1880, after the French
(1) Suronitko Das (2) Suronitko Samin defeated China.
(3) Suroniko Naveed (4) Suronitko Sayeed (2) Conquest of Tonkin and Anaam in 1887, led to the
76. What was Jungvolk? formation of French Indochina.
(1) Nazi youth group for children below 14. (3) Conquest of Laos, Cambodia and Vietnam led to the
(2) Nazi youth group for children above 14. formation of French Indochina.
(3) It was other name for youth league. (4) All the above
(4) It referred to the undesirable German children. 86. Vietnam’s religious beliefs were a mixture of:
77. In 1927, Bardoli Satyagraha was led by : (1) Local practices, worship of the supernatural
(1) Mahatma Gandhi (2) Pandit Jawaharlal Nehru (2) Buddhism and Confucianism
(3) Sardar Patel (4) Manubhai Patel (3) Buddhism, Confucianism, and local practices with
78. The political aim of the Birsa Movement was to drive away: reverence shown to the supernatural
(1) The money lenders (4) All the above
(2) Missionaries 87. The primary objective of the ‘Go East Movement’ was:
(3) Hindu landlords and the government (1) To acquire modern education in Japan
(4) All of these
(2) To drive out the French from Vietnam, overthrow the
79. Careful use of resources and giving time to renew them, is
puppet emperor and re-establish the dynasty
known as:
(3) To establish a republic in Vietnam
(1) Technological skills (2) Depletion
(4) All the above
(3) Development level (4) Conservation
88. The Trungh sisters were idealised and glorified because:
80. The first clear expression of Nationalism in Europe came
with: (1) They represented the indomitable will an intense
(1) The American Revolution patriotism of the Vietnamese.
(2) The French Revolution (2) They preferred to commit suicide, instead of
(3) The Russian Revolution surrendering to the Chinese when defeated.
(4) The Industrial Revolution (3) Phan Boi Chau wrote about them in his play.
81. Frederic Sorrieu, a French artist, in his series of four prints (4) They gathered over 30,000 soldiers and fought the
(1848) visualised his dream of a world as: Chinese for two years.
(1) A world made of democratic and social republics 89. ‘Forced recruitment’ means a process by which
(2) A world made up of one nation, one world (1) Indians were forced by the British rulers to finance the
(3) A world with one absolute ruler British army.
(4) A world following one religion, one language (2) The Indian princes had to supply soldiers to fight for
82. Name one kind of revolt that started in Europe in 1848. the British.
(1) Linguistic Revolt in Germany (3) The colonial state forced people in rural areas to join
(2) Artisans, industrial workers and peasants revolted the army.
against economic hardships (4) None of the above
(3) Revolt against monarchy in Switzerland 90. Baba Ramchandra was
(4) Revolt for freedom in Greece (1) A sanyasi, who was earlier an indentured labourer
83. How was the process of German unification completed and (2) Leader of the peasants revolts in Awadh
who was proclaimed the ruler? (3) Founder of the Kishan Sabha of Awadh in October
(1) After many wars, Kaiser William became the ruler. 1920 along with J. L. Nehru
(2) After 3 wars fought over 7 years with Austria, Denmark (4) All the above
and France, Kaiser William I became the ruler of a unified 91. Name two industrial organisations established by Indian
Germany. merchants and industrialists to protect their business
(3) The Germans defeated the Habsburg Empire and made interests.
Kaiser William I the ruler. (1) The Confederation of Indian Industry (CII)
(4) Otto von Bismarck became the ruler after defeating (2) The Indian Industrial and Commercial Congress (1922)
France. (3) The Federation of the Indian Chambers of Commerce
84. Some historian consider Great Britain as a model of the nation and Industry (FICCI) in 1927
state because: (4) Both (2) and (3)
(1) It was created after a long-drawn-out wars and political 92. Who was the first writer to create the image of ‘Bharat Mata’
struggle. as an identity of India and how?
(2) It became a nation-state after forcing Scotland, Wales (1) Abanindranath Tagore by his paintings of a mother
and Ireland to submit to it. figure in 1905.
EBD_7332
26
S- NTSE Stage 1 Question Bank

(2) Rabindranath Tagore through his collection of ballads, (2) The injustices of the caste system.
nursery rhymes and myths. (3) Restriction on the Vernacular press.
(3) Bankim Chandra Chattopadhyay in 1870, by writing (4) III treatment of widows.
the song “Vande Mataram” and later including it in his 97. Written in the form of series of letters
novel ‘Anand Math’. (1) Episolatory novel (2) Literature
(4) None of the above (3) Workbook (4) None of these
93. Her writing became important in defining a new type of 98. Charles Dicken’s ‘Hard Times’ shows a fictious industrial
women town which is named as
(1) Jane Austen (2) The Bronte sisters (1) Coketown (2) Porttown
(3) George Eliot (4) All of these (3) Courttown (4) None of these
94. Which female writer highlighted the experience of woman? 99. Her novels give us a glimpse of the world of women in rural
(1) Rash Sundari Devi (2) Kailashbashini Devi society in early nineteen century.
(3) Tarabai Shinde (4) Pandita Rama Bai (1) Jane Austen (2) Rokeya Hussein
95. The first printing press was developed by (3) Charlotte Bronte (4) Emile Zola
(1) Marco Polo (2) Kitagawa Utamaro 100. Chandu Menon tried to translate ‘Henreitta Temple’ into
(3) Johann Gutenburg (4) Erasmus (1) Malayalam (2) Kannada
96. The book Gulamgiri was written about (3) Tamil (4) Hindi
(1) The link between caste and class exploitation.

SECTION 6. CIVICS
1. The constitution of India declares India to be: 9. Which of the following is not a permanent member, of UN
(1) India is a Union of States Security Council?
(2) India is a federation (1) China (2) France
(3) India is a Unitary States (3) Japan (4) Russia
(4) India is a Union of provinces
10. Which one of the following is a directly elected house?
2. Which of the following states enjoy special status according
to the constitution? (1) Parliament (2) Rajya Sabha
(1) Jammu and Kashmir (2) Punjab (3) Lok Sabha (4) Vidhan Parishad
(3) Tamil Nadu (4) Kerala 11. Who said that religion can never be separated from the
3. Coalition Government means: politics ?
(1) Government formed by two or more parties (1) Acharya Vinoba Bhave (2) Mahatma Gandhi
(2) Government that remains in power for five years (3) Sarojini Naidu (4) Dr. Rajendra Prasad
(3) Government formed with the State parties
(4) Government that makes law with consensus 12. Who among the following is a part of Political Executive?
4. Party that remained in power for 30 years without any break (1) District collector
and believes in Marxism: (2) Secretary of the ministry of Home Affairs
(1) Communist Party of India (3) Home Minister
(2) Communist Party of India Marxist (4) Director General of Police
(3) Communist Party of India Marxist Leninist 13. Apartheid was the name of a system unique to :
(4) Communist Alliance
(1) South America (2) South Africa
5. Who was the Chairperson of the Drafting Committee of the
Constituent Assembly of India? (3) Asia (4) Europe
(1) Dr. BR Ambedkar (2) Pt. Jawaharlal Nehru 14. When was Universal Adult Franchise granted in India?
(3) Abdul Kalam Azad (4) Dr. Rajendra Prasad (1) 1948 (2) 1950
6. How many Fundamental Rights are there in Indian (3) 1952 (4) 1954
Constitution? 15. Which state has more than 30 Lok Sabha constituencies?
(1) 4 (2) 5
(1) Assam (2) Kerala
(3) 6 (4) 7
7. What does the term ‘Secular’ mean in the Indian Context? (3) Rajasthan (4) Tamil Nadu
(1) No State Religion 16. In which year IX Schedule was included in the Indian
(2) One State Religion Constitution ?
(3) No Religion (1) 1950 (2) 1951
(4) Sarvadharma Sambhav (3) 1952 (4) 1953
8. In India seats are reserved for women in: 17. Which part became 22nd State of India on 26th April, 1975 ?
(1) Lok Sabha (2) Rajya Sabha
(1) Nagaland (2) Tripura
(3) Panchayati Raj (4) Cabinet
(3) Himachal Pradesh (4) Sikkim
Exemplar Practice Questions 27
S-

18. Who is the first Person of India ? 28. The Right to Property lost its place among the Fundamental
(1) Prime Minister (2) President Rights and became just a statutory right in the year.
(3) Governor (4) Chief Minister (1) 1975 (2) 1976
(3) 1977 (4) 1978
19. The Country that is not permanent member of United Nation
29. ‘Politics’ was written by
Organisation is?
(1) Plato (2) Aristotle
(1) Russia (2) Britain
(3) Hegel (4) Marx
(2) China (4) India
30. The total number of members in the Security Council of the
20. How many languages are there under article 8th in the Indian United Nations Organisation is
Constitution?
(1) 10 (2) 15
(1) 22 (2) 20 (3) 18 (4) 20
(3) 25 (4) 15 31. Which of the following thinkers may be called the proponent
21. Which of the following Country has adopted the one (single) of the principle of popular sovereignty?
Party System? (1) Hobbes (2) Locke
(1) lndia (2) America (3) Rousseau (4) Hegel
(3) Japan (4) China 32. How many times has Financial Emergency (art.360) been
22. Disputes relating to the election of the President of India declared in India?
can be adjudicated in which if the following courts? (1) Once (2) Twice
(1) Supreme Court of India (3) Thrice (4) Never
(2) State High Court 33. Which one of the following is the financial powers of the
(3) Subordinate Courts President?
(4) All of the above Courts (1) To declare a Financial Emergency.
(2) To call for a joint session of both the Houses of
23. Read the following statement and indicate which one is
Parliament.
correct?
(3) To appoint the Finance Commission after every five years.
Statement :
(4) To grant pardon to a convict if a petition of mercy is
(i) Democracy is a form of government in which the submitted.
Governing Body is a comparatively large fraction of 34. Which one of the following is not of Public Interest
the entire population Litigation?
(ii) Democracy really means nothing more or less than the (1) Environmental Protection (2) Child Labour
rule of the whole people expressing their sovereign (3) Bonded Labour (4) Dowry System
will be their votes. 35. In a City ‘A’ the Municipal Council have established ward
Indications : committees because :
(1) (i) is true, (ii) is false (2) (i) is false, (ii) is true (1) City ‘A’ is a hill station.
(3) Both (i) and (ii) are true (4) Both i and ii are false (2) City ‘A’ has a population of more than three lakhs.
24. In which year in India was the Protection of Human Rights (3) Due to provision of 74th constitutional amendment.
Act enacted? (4) City ‘A’ has a population of more than ten thousand
and less than twenty five thousand.
(1) 1992 (2) 1993
36. The Zilla Parishad get ........................ share in the lands
(3) 1994 (4) 1995 revenue collected from the district.
25. Since when the Right to Information Act was implemented (1) 50% (2) 75%
in India? (3) 70% (4) 100%
(1) September 12, 2005 (2) October 12, 2005 37. Which one is not the member of the Union Executive?
(3) November 19, 2005 (4) December 19, 2005 (1) Speaker of Loksabha (2) President
26. In which year was the United Nations Peace Keeping Forces (3) Council of Ministers (4) Prime Minister
awarded Nobel Peace Prize? 38. This was added in the preamble to the constitution through
(1) 1988 (2) 1990 42nd constitutional amendment
(3) 1992 (4) 1994 (1) Socialist (2) Democratic
27. Who appoints the Chief Minister of any State in India? (3) Sovereign (4) Republic
(1) The Prime Minister of India 39. The population of a village is between 500 to 1500. The num-
(2) The Home Minister of India ber of members to be elected to the gram panchayat
(3) The Governor of the State (1) 5 (2) 7
(4) The Chief Justice of the High Court of the State (3) 9 (4) 11
EBD_7332
28
S- NTSE Stage 1 Question Bank

40. The term ‘Third World’ represents (3) All kinds of discrimination based on castes, creed and
(1) A large number of newly independent and developing gender should be eliminated
nations (4) All should be granted right to freedom of religion
(2) A large number of developed nations 53. Which of the following guiding values of the constitution
(3) A large number of socialist countries of India means people have supreme right to make decision?
(4) A large number of western group of nations (1) Sovereign (2) Secular
(3) Republic (4) Fraternity
41. The 17th SAARC Summit was held in Maldives in 2011. Its
Chairperson was : 54. Indian constitutional amendment 122th is concerned with
(1) Health (2) Education
(1) Man Mohan Singh (2) Rajpaksha
(3) Tax (4) Sport
(3) Begum Haseena (4) Mahmud Nasheed
55. Article 22 of the Constitution and Criminal Law guarantee
42. 193rd member of UN admitted in 2011 was : to very arrested person :
(1) Switzerland (2) Sudan (i) The person to be arrested, has to be informed the
(3) South Sudan (4) East Timor reason of this arrest at the time of arrest.
43. Which one of the following Articles provides equality of (ii) The arrested person has to be presented before a
opportunity in matters of public employment in India? magistrate, within 24 hours of his arrest.
(1) Article 14 (2) Article 15 (iii) Confessions made, while in police custody, are not
(3) Article 16 (4) Article 17 evidences and cannot be used in court.
44. The members of the Commonweath Nations were those : (iv) A boy below the 15 years of age woman connot be
(1) Who were subjected to British Rule called for interrogation to the police station
(1) (i), (ii), (iii) (2) (ii), (iii), (iv)
(2) Who still accept Birtish queen as their head
(3) (i), (iii), (iv) (4) (i), (ii), (iii), (iv)
(3) Who did not support British during World War I
56. In a Parlimentary form of government :
(4) Who did not support British during World War II (1) The Council of Ministers exercises all powers headed
45. The Second Gulf War was fought between : by the Prime Minister.
(1) Iraq and U.S.A. (2) The head of the State, President, enjoys real powers.
(2) Iraq and Kuwait (3) Union and States enjoy equal powers.
(3) Iraq and Iran (4) Right to vote is limited.
(4) Iraq and Sudan 57. Which of the following is incorrect regarding a unitary
46. The Judges of International court of Justice are elected by : government?
(1) There is either only one level of government or the
(1) Security Council
sub-units are subordinate to the central government
(2) General Assembly (2) The central government can pass on orders to the
(3) Security Council and Trusteeship Council provincial government
(4) Security Council and General Assembly (3) A state government is answerable to central
47. Which Political Party used the Slogan of 'Land to the Tiller' government.
in the West Bengal Assembly Elections in 1977 : (4) The powers of state governments are guaranteed by
(1) Left Front (2) Telugu Desam Party the Constitution
(3) AIADMK (4) Assam Gan Parishad 58. What is true regarding sources of revenue in a federal
48. Japan's parliament is known as : system?
(1) Diet (1) States have no financial powers or independent sources
(2) National Assembly of revenue.
(3) Yuan (2) States are dependent for revenue or funds on the central
(4) National Peoples Congress government.
49. Which article of Constitution of India says "No Child below (3) Sources of revenue for each level of government are
the age of fourteen years shall be employed to work in any clearly specified to ensure its financial autonomy.
hazardous employment"? (4) States have no financial autonomy.
(1) Article 24 (2) Article 45 59. Which is not true regarding changes in power-sharing
(3) Article 330 (4) Article 368 arrangement between the centre and the states?
50. In Parliament session zero house is at the discretion of (1) The Parliament cannot on its own change this
(1) Prime Minister (2) Speaker arrangement.
(3) Opposition Leader (4) President (2) Any change to it has to be first passed by both the
51. Autobiography of Nelson Mandela is : Houses with at least two-thirds majority.
(1) Wings of Fire (2) The Race of My Life (3) Then, it has to be ratified by the legislatures of at least
(3) Long Walk to Freedom (4) My Country My Life half of the total states.
52. What is meant by 'Social Justice'? (4) The Parliament alone has the power to amend the
(1) All should have same economic rights provisions regarding power-sharing.
(2) All should have same political rights
Exemplar Practice Questions S-29
60. Which period saw the rise of regional political parties in 69. What did social reformers like Jyotiba Phule work towards?
many states of the country? (1) Freedom of expression
(1) Period after 1990 (2) Period after 2000 (2) Removing caste inequalities
(3) Period after 1980 (4) Period after 1970 (3) Freedom to practise any religion
61. Which of these sentences is not correct about the “Black (4) Removing sexual division of labour
Power Movement”? 70. The first step towards thinking carefully about the
(1) It emerged in 1996 ________ of democracy is to recognize that democracy is
(2) It lasted till 1975 just a form of government.
(3) It was a more militant movement (1) government (2) people
(4) It advocated peaceful methods (3) outcome (4) None of the above
62. How did the San Jose State University honour Tommie Smith 71. What is the most distinctive feature of democracy?
and Carlos? (1) Its examination never gets over
(1) By giving them bravery awards (2) Its examination gets over after a period
(3) It never fulfills people’s demands
(2) By installing their statues in the University Campus
(4) None of the above
(3) By starting a sports organisation in their name
72. What does the term ultra-rich mean?
(4) By giving them jobs
(1) highly wealthy (2) wealthy
63. Which of these is incorrect about the Catholic Christians of
(3) highly healthy (4) not very wealthy
Northern Ireland? 73. What type of challenge is faced by a non-democratic country
(1) They are likely to be poor. for democratic set up?
(2) They may have suffered a history of discrimination. (1) foundational challenge
(3) Catholics have lived peacefully with Protestants. (2) challenge of deepening
(4) All the above. (3) challenge of expansion
64. Which of these sentences is not false? (4) challenge of money power
(1) Migrants bring with them their own culture. 74. Which are the two countries that face challenge of expansion
(2) They tend to form a different social community. of democracy?
(3) They help in making the world multicultural. (1) China & Nepal (2) Myanmar & Pakistan
(4) All the above. (3) India & U.S. (4) France & Nepal
65. Dealing with social divisions, which one of the following 75. Those communists who believe in the ideology of Mao, the
statements is not correct about democracy? leader of the Chinese Revolution. They seek to overthrow
(1) Democracy is the best way to accommodate social the government through an armed revolution so as to
diversity. establish the rule of the peasants and workers.
(2) Democracy always leads to disintegration of society (1) Maoist (2) Marxist
on the basis of social divisions. (3) Catholics (4) Romans
(3) In a democracy, it is possible for communities to voice 76. What was the result of Bolivia water war?
their grievances in a peaceful manner. (1) People were forced to pay the increase water rates
(4) Due to political competition in a democracy, social (2) People lost the war
divisions get reflected in politices. (3) Government of Bolivia resisted
66. Which among the following statements about India’s (4) The water contract with the MNC was cancelled and
water supply was restored to the municipality to the old
constitution is wrong? It
rates.
(1) prohibits discrimination on grounds of religion
77. Which one of the following is the ‘Third Wave’ country
(2) gives official status to one religion that had won democracy in 1990?
(3) provides to all individuals freedom to profess any (1) Bolivia (2) Belgium
religion (3) Bangladesh (4) Nepal
(4) ensures equality of citizens within r eligious 78. Which of the following statements is not true about Nepal?
communities. (1) King Gyanendra the New King of Nepal was not
67. A system in which all work inside the home is either done by prepared to accept democratic rule.
the women of the family, or organised by them through the (2) Nepal witnessed an extra ordinary popular movement
domestic helpers in April 2001.
(1) Domestic system (3) On 24th April King Gyanendra conceded to all the
(2) Female system of labour demands of SPA.
(3) Sexual division of labour (4) Girja Prasad Koirala chosen as the new Prime Minister
(4) None of these of the Interim Government.
68. This concept is used to refer to a system that values men 79. Which pressure group seeks to promote collective good
(1) Sectional Interest group
more and gives them power over women
(2) Public interest group
(1) Patriarchy (2) Monarchy
(3) Movement group
(3) Polygamy (4) Dyarchy
(4) Loose Organisation
EBD_7332
30
S- NTSE Stage 1 Question Bank

80. A group fighting against bonded labour’ is an example of (4) Securing the Interest of the educated classes
(1) Promotional group (2) Public Interest group 90. Changing party allegiance from the party on which a person
(3) Professional group (4) Sectional interest group got elected (to a legislative body) to a different party.
81. What does NAPM signify (1) Defection (2) Diversion
(1) National Agreement for People’s Movements (3) Separation (4) Cancellation
(2) National alliance of Public Movements 91. A direct vote in which an entire electorate is asked to either
(3) National alliance for People Movement accept or reject a particular proposal is termed as:
(4) National alliance for people mobilisation (a) Coalition (b) Referendum
82. What are a group of people who come together to contest (c) Veto (d) Election
elections and hold power in the government they are known 92. Why are China and Mexico not regarded as democracies
as despite holding elections?
(1) Cabinet (2) Council (1) Because they are communist countries.
(3) Candidate (4) Political Party (2) Because they are monarchies.
83. Which one of the following is true regarding the coalition (3) Because China and Mexico are under military rule.
government? (4) Because elections in China and Mexico do not offer
(1) Only two parties form an alliance and contest election any political alternatives and the people cannot remove the
(2) Several parties compete for power existing rulers.
(3) The government is formed by two or more parties 93. A democracy requires that the rulers have to attend to the
coming together __________ of the people.
(4) Several parties form an alliance and compete for power (1) requirements (2) needs
84. Which one of the following is the guiding philosophy of (3) dreams (4) wishes
Bhartiya Janta Party? 94. Compare the Preambles to the Constitution of following
(1) Bahujan Samaj countries. Which of the following does not invoke God?
(2) Revolutionary democracy (1) The United States of America
(3) Cultural Nationalism (2) India
(4) Modernity (3) South Africa
85. Which party believes in concept of Marxism-Leninism (4) Afghanistan
(1) Communist Party of India 95. Which of the following is/are an essential element of the
(2) Bhartiya Janta Party state
(3) Bahujan Samajwadi Party (1) Sovereignty (2) Government
(4) Indian National Congress (3) Territory (4) All of these
86. Which party wants that high offices in government be 96. Who often bitterly criticised Mahatma Gandhi and his
confined to natural born citizens of the country visions and why?
(1) Nationalist Congress Party (1) Pt. Jawaharlal Nehru
(2) Bhartiya Janta Party (2) Dr. B.R. Ambedkar
(3) Indian National Congress
(3) Mr. Shyama Prasad Mukherjee
(4) None of these
(4) Mr. Somnath Lahiri
87. An Affidavit signifies
97. Who is the chief administratione officer of the UN.
(1) Where a person makes a sworn statement regarding
(1) General Secretary (2) Secretary General
his/her personal information.
(3) Security Council (4) General
(2) A signed document submitted to an officer
98. Kosovo was a province of __________ before it splits.
(3) Signed document where a person makes sworn
statement regarding his or her antecedents (1) Vietnam (2) Zimbabwe
(4) All of the above (3) Sri Lanka (4) Yugoslavia
88. Which one of the following is not a challenge to political 99. Who appoints NHRC?
party? (1) President (2) Judge of S.C.
(1) Lack of internal democracy (3) Vice-President (4) Prime Ministor
(2) Dynastic succession 100. One of the forms of explaitation as mentioned in the
(3) Growing role of money and muscle power Constitution is traffic. What does it mean?
(4) All of these (1) transport system
89. The Bahujan Samaj Party stands for (2) buying and selling of human beings
(1) Securing the Interest of the operessed (3) buying and selling of goods
(2) Securing the Interest of the capital class (4) None of the above
(3) Securing the Interest of the elite class
Exemplar Practice Questions S-31

SECTION 7. ECONOMICS
1. Which of the following agencies markets steel for the public 13. Which of the following set of wars was mainly responsible
sector plants? for failure of the Third 5 Year Plan?
(1) HAIL (2) SAIL (1) Indo-China war of 1962 and Indo-Pak war of 1947
(3) Tata Steel (4) MMTC (2) Indo-China war of 1962 and Indo-Pak war of 1965
2. What is the life expectancy of Indians, as per the 2001 (3) Indo-Pak war of 1971 and the Kargil war of 1999
Census? (4) Indo-China war of 1962 and the Indo-Pak war of 1971
(1) 72 Yrs. (2) 53 Yrs. 14. A customer pruchases a washing machine. She realises that
(3) 64 Yrs. (4) 70 Yrs. it is defective. She complains to the dealer and the
3. Which one of the following is associated with Primary manufacturer, to no effect, which one of the following right is
Sector? denied to her, in this case?
(1) right to choose
(1) Lawyer (2) Doctor
(2) right to consumer’s safety
(3) Priest (4) Farmer
(3) right to information about goods and services
4. Where is the headquarters of Life Insurance Corporation
(4) right to represent.
situated ?
15. Schemes like ‘Akshara Dasoha’ and ‘Ksheera Yojana’ are
(1) Kolkata (2) Chennai associated with :
(3) Mumbai (4) New Delhi (1) Milk production
5. When was the "National Rural Employment Gurantee Act" (2) Food production
Passed? (3) Illiteracy
(1) 2001 (2) 2003 (4) Mid day Meal
(3) 2005 (4) 2007 16. Universalisation of primary education is the aim of National
6. When is the National Consumer Day celebrated in India? Policy of education. It came into effect during the year :
(1) 1980 (2) 1985
(1) December, 24 (2) September, 16
(3) 1986 (4) 1989
(3) March, 8 (4) May, 25 17. Taxation is a toll of :
7. In which year New Economic Reforms were started in India? (1) Monetry policy (2) Fiscal policy
(1) 1981 (2) 1991 (3) Price policy (4) Wage policy
(3) 2001 (4) 1995 18. Which online facility has been launched by the government
to provide defect solution to problem of agriculture sector?
8. Which sector provides Livelihood to majority of Indians?
(1) e-Krishi Samasya
(1) Industry (2) Agriculture (2) e-Krishi Samvad
(3) Service (4) Commerce (3) e-Krishi Samveda
9. The natural resources used in production are (4) e-Krishi Solution
(1) renewable resources 19. Which one of the following is a non-economic activity?
(2) non-renewable resources (1) Milkman selling milk
(3) renewable and non-renewable resources (2) Farmer sowing seeds
(4) none of the above (3) A mother cooking for her family
10. The term ‘Globalisation’ refers to
(4) Nurse working in a hospital
(1) integration among the economy of different countries
(2) decreasing government control over different sectors 20. Which of the following is responsible for high poverty rates?
(3) free trade among the different countries (1) Huge income inequalities
(4) all of the above (2) Unequal distribution of land
11. Which of the following activities is not involved in the term
(3) Lack of effective implementation of land reforms
‘economic growth’?
(1) development of industries (4) All of these
(2) increase in the share of agriculture 21. Who advocated that India would be truly independent only
(3) trade growth
(4) increase in banking and construction services when the poorest of its people become free of human
12. The government has reduced the Public Distribution System suffering?
prices by ................. % from July 2001. (1) Mahatma Gandhi (2) Indira Gandhi
(1) 10 (2) 20
(3) 30 (4) 50 (3) Jawahar Lal Nehru (4) Subhash Chandra Bose
EBD_7332
32
S- NTSE Stage 1 Question Bank

22. The calorie requirement is higher in the rural areas because 34. Interest payment will be higher on a house loan on account
(1) they do not enjoy as much as people in the urban areas of
(2) food items are expensive (1) high tenure of loan
(3) they are engaged in mental work (2) high rate of interest
(4) people are engaged in physical labour
(3) high risk of borrower profile
23. Which of the following is not a valid reason for the poverty
alleviation programme in India? (4) All of these
(1) Lack of proper implementation 35. In a SHG most of the decisions regarding savings and loan
(2) Lack of right targeting activities are taken by the
(3) Corruption at the highest level (1) Bank
(4) Overlapping of schemes (2) Members
24. The rationing system was revived in the wake of an acute (3) Non-government organisation
food shortage during the 1960s, prior to the (4) None of these
(1) White Revolution (2) Great Revolution
36. In October 2005, the Government of India enacted a law,
(3) Green Revolution (4) Blue Revolution
popularly known as RTI (Right to Information) Act, which
25. In Maharashtra, Academy of Development Science (ADS)
ensures its citizens all the information about the functions of
has facilitated a network of NGOs for setting up ________ in
different regions. (1) Private departments (2) Public departments
(1) Milk banks (2) Money banks (3) Local departments (4) Government departments
(3) Cloth banks (4) Grain banks 37. Rajesh had applied for an electricity connections. However,
he was not provided power supply. In this situation,
26. Public Distribution System (PDS) is the most important step (1) he can approach the consumer court to seek-redressal
taken by the Government of India towards (2) he cannot approach the consumer court to seek redressal
(1) Ensuring water security (2) Ensuring milk security
(3) he can file a civil suit in the court of law against the
(3) Ensuring food security (4) Ensuring fans security
Electricity Board
27. Which are the special target groups in Antyodaya Anna
(4) Only (b) and (c)
Yojana?
(1) Poorest of the poor (2) Poor and non-poor 38. Which of the following does not fall under Consumer Rights?
(3) Backward class (4) None of these (1) Right to be informed
28. Average income is also called (2) Right to choose
(1) Per capita incentive (2) Per capita income (3) Right to seek government help
(3) Percent capital income (4) Per capital information (4) Right to represent the consumer’s court
29. Kerala has a low Infant Mortality Rate because it has adequate
39. Which is a function of PDS?
provision of basic health and
(1) Educational facilities (2) Sports facilities (1) Distribution of black market products
(3) Security facilities (4) Entertainment facilities (2) Control Hoarding, control prices and control over
30. In the year 2003, the tertiary sector emerged as the largest charging.
producing sector in India replacing the (3) Popularise black-marketing in the country
(1) First sector (2) Main sector (4) None of these
(3) One sector (4) Primary sector 40. GDP is the sum total of the value of ................... goods
31. Grameen Bank of Bangladesh is one of the biggest success produced during a particular year :
stories in reaching out to the poor to meet their credit needs
(1) Primary (2) Secondary
at
(3) Tertiary (4) Information technology
(1) First rates (2) Same rates
(3) Reasonable rates (4) Best rates 41. What is disguised unemployments ?
32. Most loans from informal lenders carry a very high interest (1) When more people are made to work less than their
rate and do little to increase the income of the potential
(1) Borrowers (2) Lenders (2) When people are ready to work but they have no jobs or
(3) People (4) Buyers work to do
33. There is restriction of withdrawing money in a (3) When less people are made to work more than their
(1) savings account (2) current account potential
(3) fixed deposit account (4) None of these (4) When every one is made to work according to his potential
Exemplar Practice Questions 33
S-

42. Which was the largest producing sector of India in 2003 ? 52. Which of the following is not an advantage of self-help group?
(1) Tertiary Sector (2) Quaternary Sector (1) Grant of timely loans
(3) Primary Sectors (4) Private Sector (2) Reasonable Interests
43. Underemployment occurs when people: (3) Reasonable interests
(1) Do not work at all (4) Does not help women to become self-reliant
(2) Work slowly 53. What do you mean by collateral ?
(3) Are not paid for work (1) It is the total sum of money with a person
(4) Work less than their potential (2) It is the things kept in the locker
44. Producing a good by exploiting the natural resources is an (3) It is the guarantee given by the lender to the borrower
activity of the : (4) It is the security to a lender until the loan is repaid
(1) Secondary sector (2) Primary sector 54. Which of the following is not true regarding the in
(3) Service sector (4) None of these convenience of Barter Exchange ?
45. A study by the _____ estimates that 20 lakh jobs can be (1) Lack of double coincidence of want
created in the educational sector alone. (2) Absence of divisibility
(1) Ministry of Affairs (3) Difficulty in storing wealth
(2) Planning Commission (4) Availability of money as a medium of exchange
(3) Human Development Report 55. Which one of the following is the important characteristic of
(4) None of these modern form of currency ?
46. There are many people who work in the ______ sector have (1) It is made from precious metal
towork at a low wage and accept conditions that are not fair. (2) It is made from thing of everyday use
(1) MNC (2) Unorganized (3) It is authorised by the commercial banks
(3) Organized (4) None of the above (4) It is authorised by the Government of the country
47. What is the consumer court at the national level called ? 56. Which one of the following is a major reason that prevents
(1) The National Consumer Disputes Redressal Commission the poor from getting loans from the banks?
(2) State Consumer Disputes Redressal Commission (1) Lack of capital
(3) The District Forum (2) Not affordable due to high rate of interest
(4) None of the above (3) Absence of collateral security
48. COPRA is also known as (4) Absence of mediators
(1) Consumer Safety Act 57. Anything which is generally accepted by the people in
(2) Consumer Protection Council exchange of goods and services is called :
(3) Consumer Forum (1) money (2) barter
(4) Consumer Protection Act (3) credit (4) loans
49. What one of the following is not given on the packing of 58. Banks doe not given loans :
commodities. (1) to small farmers
(1) price (2) to marginal farmers
(2) batch number (3) to industries
(3) expiry date (4) without proper collateral and documents
(4) name of the manufacturer
59. The functioning of the formal sources of credit are supervised
50. AGMARK is the certification maintained for standardization by :
of ______
(1) Government of India (2) Reserve Bank of India
(1) jewellery (2) clothes
(3) Ministry of finance (4) State Bank of India
(3) agricultural goods (4) furniture
60. Which is not the main source of credit from the following for
51. Big companies with huge wealth, power and reach can ____
rural households in India?
the market in various ways.
(1) Traders (2) Relatives and friends
(1) prove (2) product
(3) Commercial Banks (4) Moneylenders
(3) mingle (4) manipulate
EBD_7332
34
S- NTSE Stage 1 Question Bank

61. Cheaper imports, inadequate investment in infrastructure lead 71. Which social group is most vulnerable to poverty in India ?
to (1) Scheduled castes (2) Scheduled tribes
(1) slowdown in agricultural sector (3) Casual labours (4) All of these
(2) replace the demand for domestic production 72. In which state is the public distribution system responsible
(3) slowdown in industrial sector for the reduction in poverty ?
(4) all the above (1) Andhra Pradesh (2) Tamil Nadu
62. Globalisation results in (3) Both (1) and (2) (4) None of the above
(1) lesser competition among producers 73. In rural sector, which of the following is not poor
(2) greater competition among producers (1) Landless agricultural workers
(3) no change in competition among producers (2) Backward classes
(4) none of the above (3) Rural artisans
63. Globalisation has led to improvement in (4) Medium farmers
(1) choice to consumers 74. Trade barriers refer to:
(2) quality of goods and services (1) Taxes on imports (2) Globalization
(3) foreign investment (3) Liberalization (4) None of the above
(4) all the above 75. MNCs prefer investment in foreign countries through:
64. Which of the following factors has not facilitated globalisation ? (1) Setting up new factories
(1) Technology (2) Buying existing local companies
(2) Liberalisation of trade (3) entering into partnership with existing local companies
(3) WTO (4) All the above
(4) Nationalisation of banks 76. Which of the following terms is used for measuring the crop
produced on a given piece of land during a single year?
65. Multinational corporations have succeeded in entering global
markets through
(1) Yield (2) Productivity
(1) WTO (2) UNO
(3) Cultivation (4) Output
(3) UNESCO (4) None of the above
77. Which of the following statements is not true about small-
66. When economic activities in a country are influenced by scale manufacturing in villages ?
economic activities in other countries, it is called (1) Farmers engage in it to supplement their income
(1) foreign trade (2) competition (2) Farmers takes help of their family members
(3) globalisation (4) All the above (3) The production is done mostly at home
67. Investment means spending on (4) Farmers produce articles for their own use
(1) factory building (2) machines 78. The rural population migrates to urban areas due to
(3) equipments (4) all the above (1) rising population in rural areas
68. Integration of markets means (2) in search of better employment opportunities
(1) operating beyond the domestic markets (3) lack of demand of labour in agriculture
(4) all of the above
(2) wider choice of goods
79. Which states from the following try out the modern farming
(3) competitive price
methods in India ?
(4) All the above
(1) Haryana, Madhya Pradesh and Tamil Nadu
69. Liberalisation refers to
(2) Punjab, Haryana and Gujarat
(1) freeing the economy from direct control (3) Punjab, Himachal Pradesh and Uttar Pradesh
(2) putting an end to various restrictions (4) Punjab, Haryana and Uttar Pradesh
(3) opening up the economy 80. Why are rural women employed in low-paid jobs ?
(4) all the above (1) They do not need to work in high-paid jobs
70. FDI (Foreign Direct Investment) attracted by globalisation in (2) They lack education or the necessary skill
India belongs to the (3) They are not allowed by their families to do high paid
(1) World trade (2) Multinationals jobs
(3) Foreign governments (4) None of the above (4) They are not aware of the wage structure
Exemplar Practice Questions S-35

81. ________ is the most labour-absorbing sector of Indian 90. Mother Diary is making strides in provision of milk and
economy. vegetables to the consumers at controlled rate decided by
(1) Agriculture (1) Government of Bihar (2) Government of Gujarat
(2) Transport (3) Government of Punjab (4) Government of Delhi
91. Hunger is another aspect indicating
(3) Seasonal unemployment
(1) Pool insecurity (3) Paint insecurity
(4) Manufacturing
(2) Flood insecurity (4) Food insecurity
82. Reduction in infant mortality involves: 92. To whom is the yellow card issued?
(1) Preventing infants from dying (1) To shopkeepers
(2) Protecting expectant mothers from infection (2) To landlords
(3) Protection of children from infection, ensuring nutrition (3) To government employees
alongwith care for mother and child (4) People below the poverty line
(4) Providing health services for children 93. Development of a country generally is determined by
83. Which of the following are the factors of production ? (1) its per capita income
(2) its literacy level
(1) Health, Education, Technology and Training
(3) health status of its people
(2) Land, Labour, Capital and Human resources
(4) all of these
(3) Literacy rate, Health, Life expectancy and Skill 94. Formal sources of credit include
(4) Marketing and non-marketing activities (1) Banks, Cooperatives and Private Lenders
84. A student admitted to a Managing Institute is most likely to (2) Bans, Cooperatives and LIC
get a bank loan because: (3) Banks, Societies and Private Lenders
(1) His father is a rich man (4) None of these
(2) He is a brilliant student 95. Which of the following is the most important component for
(3) He is likely to go abroad comparing different countries?
(4) He is certain to get a high salary job (1) Population (2) Income
(3) Per capita income (4) Resources
85. Quality of population depends upon the
96. Which of the following is not a component of HDI?
(1) Literacy rate (1) Per Capita Income (2) Life Expectancy
(2) Health of a person indicated by life expectancy (3) Literacy Rate (4) Poverty Rate
(3) Skill formation 97. Which of the following is most likely to be a development
(4) All of these goal for landless rural labourers as:
86. Death rate is the number of people per _______ who die (1) Highest support prices for their crops
during a particular period of time. (2) They should be able to settle their children abroad
(1) 10,000 (2) 1,000 (3) Raised wages
(3) 100 (4) 10 (4) None of these
98. Which of the following do we get when we divide the National
87. Increase in longevity of life is an indicator of
Income of a country by its total population?
(1) good quality of life (1) Per Capita Income
(2) improvement in health sector (2) Gross Development Product
(3) better HDI (Human Development Index) (3) Human Development Index
(4) All of these (4) None of above
88. Nutritional level of food energy is expressed in the form of 99. Which of the following Acts would not apply to a company
(1) calories per day like TISCO?
(1) Minimum Wages Act
(2) wheat consumption
(2) National Rural Employment Guarantee Act
(3) rice consumption per day
(3) Factories Act
(4) none of these (4) Payment of Gratuity Act
89. Which one is not the major cause of income inequality in 100. The value of all goods and services produced within a country
India? in a certain year is called its:
(1) Unequal distribution of land (1) National Product
(2) Lack of fertile land (2) Net Domestic Product
(3) Gap between the rich and the poor
(3) Gross Domestic Product
(4) Increase in population
(4) None of these
EBD_7332
36
S- NTSE Stage 1 Question Bank

SECTION 8. GEOGRAPHY
1. Which of the following countries does not share its 15. Which country of Europe is called 'Playground of Europe' ?
boundaries with India? (1) England (2) Holland
(1) Bhutan (2) Tajikistan (3) Switzerland (4) Belgium
(3) Bangladesh (4) Nepal
16. Which State of India does not have common boundary with
2. The Eastern most longitude of India is Myanmar ?
(1) 97° 25 E (2) 77° 6 E
(1) Arunachal Pradesh (2) Tripura
(3) 68° 7 E (4) 82° 32 E
(3) Nagaland (4) Manipur
3. Under which of the following type of resources can tidal
energy be classified? 17. Which of the following countries is not in Indian sub-
(1) Replenishable (2) Human made continent ?
(3) Abiotic (4) Non Recyclable (1) Maldives (2) Pakistan
4. Which one of the following is the main cause of land (3) Bangladesh (4) Nepal
degradation in Punjab? 18. Which State has Satpura hills?
(1) Deforestation (2) Over Irrigation (1) Uttar Pradesh (2) Bihar
(3) Over grazing (4) Intensive cultivation (3) Andhra Pradesh (4) Madhya Pradesh
5. Which of the following states is not connected with the 19. Kaila Devi Sanctuary is situated in which district?
Hazira-Vijaipur-Jagdishpur (H. V. J.) Pipeline? (1) Alwar (2) Dungarpur
(1) Madhya Pradesh (2) Maharashtra (3) Karauli (4) Udaipur
(3) Gujarat (4) Uttar Pradesh
20. Massai Mara national park is located in-
6. In which of the following States is Black soil found?
(1) Jammu and Kashmir (2) Rajasthan (1) India (2) Pakistan
(3) Maharashtra (4) Jharkhand (3) Sudan (4) Kenya
7. Which type of drainage pattern is formed when the river 21. Ranthambore is situated in-
channel follows the slope of the terrain? (1) Rajasthan (2) Arunachal Pradesh
(1) Radial (2) Rectangular
(3) Assam (4) Madhya Pradesh
(3) Trellis (4) Dendritic
8. In which year the southernmost point of the Indian union – 22. The full form of C.N.G. is-
‘Indira Point’ submerged under the sea water. (1) Compound Natural Gas
(1) 2000 (2) 2002 (2) Complex Natural Gas
(3) 1998 (4) 2004 (3) Compound New Gas
9. In India which of the following river forms a second largest (4) Compressed Natural Gas
waterfall? 23. Gender ratio in India is-
(1) Narmada (2) Godavari (1) 880/1000 (2) 940/1000
(3) Kaveri (4) Krishna (3) 300/1000 (4) 400/1000
10. Sugarcane crop grows well in the areas with a rainfall of
24. Bhakra-Nangal Project is situated on the River?
_______________.
(1) Satluj (2) Tungbhadra
(1) 100 – 150 cm (2) 75 – 100 cm
(3) Damodar (4) Mahi
(3) 150 – 200 cm (4) 200 cm and above
11. Which port was developed in the wake of loss of Karachi 25. Salty Water lake is-
port ? (1) Jaisamand lake (2) Rajsamand lake
(1) Mumbai (2) Paradeep (3) Didwana lake (4) Gapsagar lake
(3) Kandla (4) Marmagoa 26. The Rainfall that occurs during winter season is known as-
12. India’s total area accounts ______________ per cent of (1) Monsoon (2) Cyclone
the total geographical area of the world. (3) Mango Shower (4) Mavath
(1) 5.0 (2) 4.0 27. Out of the following which is not a union territory?
(3) 2.8 (4) 2.4 (1) Puducherry (2) Chandigarh
13. How much part of Earth is covered by land ? (3) Goa (4) Daman and Div
(1) 26% (2) 27% 28. Which of the following is the oldest mountain range of
(3) 28% (4) 29% India?
14. In which continent, there is no active volcano ?
(1) Aravali (2) Nilgiri
(1) Asia (2) Africa
(3) Vindhya (4) Karakoram
(3) Europe (4) Australia
Exemplar Practice Questions S- 37
29. Which of the following reservoirs is related to the Bhakra- 42. Which station is not situated on the Mumbai-Delhi railroute
Nangal Project? on western railway?
(1) Krishnaraj Sagar (2) Govind Sagar (1) Jhansi (2) Surat
(3) Gandhi Sagar (4) Nizam Sagar (3) Vadodara (4) Ratlam
30. Which is the main cash crop of Odisha? 43. Which thermal power station is not in Andhra Pradesh?
(1) Cotton (2) Maize (1) Ramagundam (2) Kolaghat
(3) Wheat (4) Sugarcane (3) Kottagundam (4) Sinhadripuram
31. In the production of which mineral India ranks first in the 44. Find the wrong pair producing nitrate and phosphate :
world? Column ‘A’ Column ‘B’
(1) Manganese (2) Bauxite (State) (Industry)
(3) Tin (4) Mica (1) Maharashtra (1) Turbhe
32. What type of resource copper is? (2) Kerala (2) Cochi
(1) Exhaustible (2) Non-exhaustible (3) Odhisha (3) Rourkela
(3) Potential (4) National (4) Gujarat (4) Kalol
33. In which State of India the Kandala Port is located? 45. The following graph represents climatic conditions of :
50
(1) Tamil Nadu (2) Kerala
(3) Maharashtra (4) Gujarat 45

34. The city located on the equator is 40


(1) Quito (2) New Orleans
35
(3) Kuala Lampur (4) Addis Ababa Temperature in °C

35. The father of modern Geography are

Rainfall (in cm)


30 30

(1) Ptolemy and Layman 25 25


(2) X.Le. Pichon and Wegner
20
(3) Alexander Von Humbolt and Carl Ritter 20

(4) Park and Thornbury 15 15


36. One problem of the economic exploitation of equatorial Ever
10 10
green forest is
(1) heavy rainfall 5 5

(2) heterogenous growth of vegetation 0 0


J F M A M J J A S O N D
(3) homogenous growth of vegetation Months
(4) the trees are not suitable for economic utilisation (1) Shilong (2) Leh
37. A river builds delta when it (3) Mumbai (4) Delhi
(1) carries huge water 46. Backwater inland waterway is in :
(1) Gujarat (2) Kerala
(2) carries enough load
(3) Maharashtra (4) West Bengal
(3) passes a very long distance 47. Which cotton textile centre is not in Tamil Nadu?
(4) cannot carry its load (1) Tuticorin (2) Coimbatore
38. The main route of communication between Srinagar and (3) Salem (4) Trichur
Leh is 48. Which of the following is not associated with the
(1) Shipkila (2) Banihal manufacture of MIG Aircraft?
(3) Zojila (4) Nathula (1) Ozar (2) Koraput
39. The mother of all industries is (3) Hyderabad (4) Bengaluru
49. The primitive tribe that is found in hot deserts
(1) Petrochemical industry
(1) The Semang (2) The Pygmies
(2) Automobile industry
(3) The Sakai (4) The Bushmen
(3) Iron and Steel industry 50. The grasslands of Eurasia are called as
(4) Information technology industry (1) Veldts (2) Pampas
40. In India which irrigation sources has the maximum area? (3) Downs (4) Steppes
(1) Rivers (2) Dams 51. The soil that has very poor fertility status
(3) Wells and tubewells (4) Canals (1) Alluvial soils
41. Which is the northernmost atomic project? (2) Black cotton or regular soils
(1) Kakrapar (2) Rawatbhata (3) Laterite soils
(3) Kalpakkam (4) Tarapore (4) Red soils
EBD_7332
38
S- NTSE Stage 1 Question Bank

52. The multipurpose project that is administered by Madhya 61. Column I is the list of minerals and Column II is the list of
Pradesh and Rajasthan mode of formation of minerals.
(1) Damodar valley project (2) Chambal project Column I Column II
(3) Kosi project (4) Hirakund project 1. Copper (i) by deposition in layers of sedimentary
53. This is the single largest item of import rocks
(1) Fertilisers (2) Newsprint 2. Aluminium (ii) solidification of liquids in cracks and
(3) Petroleum (4) Machinery equipment crevices in igneous rocks
54. 38th parallel is the dividing line between the countries of : 3. Coal (iii) Decomposition of surface rocks
(1) North Vietnam and South Vietnam 4. Platinum (iv) Evaporation of water from the surface
(2) North Korea and South Korea of rocks
(3) Communist China and Taiwan (v) deposition in sand beds of valleys and
(4) India and Pakistan in foothills
55. Which one of the following is the correct sequence of rivers Which one of the following is correctly matched set?
of India from north to south? (1) 1–(v), 2–(iii), 3–(iv), 4–(i)
(1) Godavari (2) The Ganga
(2) 1–(iv), 2–(ii), 3–(i), 4–(v)
(3) Narmada (4) Cauvery
(3) 1–(iii), 2–(i), 3–(v), 4–(ii)
(i) 2, 3, 1, 4 (ii) 4, 2, 3, 1
(4) 1–(ii), 2–(iii), 3–(i), 4–(v)
(iii) 1, 3, 2, 4 (4) 4, 1, 3, 2
62. The following is the list of iron and steel plants located in
56. Consider the following statements : various parts of India.
(1) The north Indian plain is very fertile
(1) Vishakhapatnam (2) Bhadravati
(2) Perennial rivers flow across the plain.
(3) Jamshedpur (4) Salem
(3) It’s a flat land, fit for irrigation and inland navigation and
(5) Rourkela (6) Bokaro
establishment of industries.
Which one of the following is the best concluding statement? (7) Vijaynagar (8) Durgapur
(1) It’s a land of sacred rivers. Which one of the following represents iron & steel industries
(2) It’s useful to people to some extent. located outside the chotanagpur region?
(3) 40% of total population lives in this region. (1) 1, 4, 7, 8 (2) 2, 3, 6, 7
(4) The region is famous for IT companies. (3) 1, 2, 4, 7 (4) 2, 5, 7, 8
57. Which one of the following cities never gets the vertical rays 63. The following is the list of ports located along the east coast
of the sun? of India.
(1) Simla (2) Bombay (Mumbai) (1) Paradeep (2) Kolkata
(3) Ahmedabad (4) Bhopal (3) Vishakhapatnam (4) Tuticorin
58. The south west monsoons start retreating from north to south Which one of the following is the correct sequence of ports
in winter in India. The phenomenon involved in this is : from south to north?
(1) Winds blow from hot regions to cold regions. (1) 3, 4, 2, 1 (2) 4, 3, 1, 2
(2) Monsoons always move downwards. (3) 2, 1, 4, 3 (4) 1, 3, 4, 2
(3) Winds blow from a high pressure area to a low pressure 64. Kal Baisakhi is associated with :
area. (1) Punjab (2) Kashmir
(4) Winds blow from a low pressure area to high pressure (3) Himachal (4) West Bengal
area. 65. In 2014, Government of which state was awarded with the
59. Some statements are given below : UN appreciation for better management of cyclone 'Phalin'?
(1) Mangrove forests are found in some delta regions in (1) West Bengal (2) Andhra Pradesh
India (3) Tamil Nadu (4) Odisha
(2) Coniferous forests are found in tropical regions of India. 66. Which is the west coast port in India?
(3) Date palms are a part of desert vegetation. (1) Chennai (2) Kochi
(4) Evergreen forests are very dense forests. (3) Tuticorin (4) Vishakhapatnam
Which of the above statements are true? 67. "Desert Oak" is a tree whose rots go deep into the ground
(1) 1, 2 and 3 (2) 1, 3 and 4 till they reach water. The depth of these roots nearly 30
(3) 2, 3 and 4 (4) 1, 2 and 4 times the height of the tree. This tree is found in-
(1) Rajasthan (2) Abu Dhabi
60. A. Assertion :
(3) Australia (4) Russia
Pulses except turdal are grown in rotation with other crops.
68. The longest sea beach in India:-
R. Reason :
(1) Chapora Beach (2) Diu Beach
They are leguminous and helps in restoring fertility of soil by (3) Aksa Beach (4) Mariana Beach
fixing nitrogen from air in the roots. 69. The Kishtwar National Park is located in which state?
(1) Both ‘A’ and ‘R’ are true and ‘R’ explains ‘A’. (1) Jammu and Kashmir
(2) Both ‘A’ and ‘R’ are true but ‘R’ doesn’t explain ‘A’. (2) Himachal Pradesh
(3) ‘A’ is true but ‘R’ is false. (3) Punjab
(4) ‘A’ is false but ‘R’ is true. (4) Sikkim
Exemplar Practice Questions S-39
70. Highly indented coastline found along the coast of- 83. Which of the following hydraulic structures is not a feature
(1) Atlantic Ocean (2) Pacific Ocean of ancient times?
(3) Indian Ocean (4) Arctic Ocean (1) Multipurpose river valley projects
71. The state having a largest area of forest cover in India is (2) Dams built of stone rubble
(1) Arunachal Pradesh (2) Madhya Pradesh (3) Reservoirs or lakes
(3) Haryana (4) Assam (4) Embankments and canals
72. In which of the following stations maintained by India 84. Which of the following multipurpose projects is found in
meteorological department does the monsoon arrive first? the Satluj-Beas river basin?
(1) Thiruvananthapuram (2) Kolkata (1) Hirakund Project
(3) Chennai (4) Delhi (2) Damodar Valley Corporation
73. Which of the following climatic phenomenon is defined by (3) Bhakra Nangal Project
the Ferrel's Law? (4) Rihand Project
(1) ITCZ (2) Jet Stream 85. Which of the following environmental damages are not
(3) Coriolis force (4) Monsoon induced due to multipurpose projects?
74. Ozone hole refers to : (1) Water-borne diseases and pests
(1) hole in ozone layer (2) Pollution resulting from excessive use of water
(2) decrease in the ozone layer in troposphere (3) Earthquakes
(3) decrease in thickness of ozone layer in stratosphere (4) Volcanic activity
(4) increase in the thickness of ozone layer in troposphere 86. Which of the following structures are known as ‘tankas’?
75. Pine, fir, spruce, cedar, larch and cypress are the famous (1) Underground tanks for storing rainwater harvested
timber – yielding plants of which several also occur widely from roof tops for drinking purpose.
in the hilly regions of India. All these belongs to : (2) Tanks constructed on rooftops for storing rainwater.
(1) angiosperms (2) gymnosperms (3) Tanks constructed in agricultural fields to store
(3) monocotyledons (4) dicotyledons rainwater.
76. Which one of the following determines the land use pattern? (4) Tanks constructed to store floodwater.
(1) Climate (2) Topography 87. What is Primitive Subsistence Farming known as in north-
(3) Minerals (4) All of these eastern states like Assam, Meghalaya, Mizoram and
77. Which of the following plays a key role in the ecological Nagaland?
system? (1) Horticulture (2) Penda
(1) Rocks (2) Roads (3) Jhumming (4) Milpa
(3) Forests (4) None of above 88. Rice is a subsistence crop in Odisha. In which of the
78. Of the estimated 47,000 plant species found in India, about following states, is rice a commercial crop?
15,000 flowering species belong to which category? (1) West Bengal and Bihar
(1) Endangered species (2) Extinct species (2) Jammu and Kashmir
(3) Endermic species (4) Vulnerable species (3) Punjab and Haryana
79. What is the Himalayan Yew? (4) Tamil Nadu and Kerala
(1) A type of deer 89. Which of the following are known as coarse grains?
(2) A medicinal plant (1) Wheat and Rice
(3) A species of bird (2) Millets-Jowar, bajra and ragi
(3) Pulses-urad, arhar, gram
(4) A food crop grown in the Himalayas
(4) Oilseeds
80. Who among the following is in charge of management of
90. Which of the following is a kharif crop and accounts for
forests and wildlife resources of India?
about half of the major oilseeds produced in the country?
(1) World Wildlife Foundation
(1) Mustard (2) Coconut
(2) Geological Survey of India
(3) Groundnut (4) Soyabean
(3) Forest Department
91. Which of the following types of farming is practised in areas
(4) Non-government organisations
with high population pressure on land?
81. In which of the following tiger reserves have the local
(1) Primitive Subsistence Farming
communities fought for conservation of the forests? (2) Intensive Subsistence Farming
(1) Manas Tiger Reserve (3) Commercial Farming
(2) Periyar Tiger Reserve (4) Plantations
(3) Simplipal Bio Reserve 92. Minerals formed from solidification of molten matter in the
(4) Sariska Tiger Reserve crack, crevies, faults or joints are found in which types of
82. Which of the following is a reason for water scarcity in a rocks?
region with sufficient water to meet the requirements of the (1) stratified rocks
people? (2) igneous and metamorphic rocks
(1) Huge population (2) Less rainfall (3) sedimentary rocks
(3) Power requirement (4) Pollution (4) none of the above
EBD_7332
40
S- NTSE Stage 1 Question Bank

93. Which of the following sedimentary minerals is formed as a 97. Which of the following group of factors is a prime group for
result of evaporation especially in arid regions? the location of aluminium smelting plant?
(1) Coal (2) Potash salt (1) Capital and Market
(3) Iron ore (4) Sulphur (2) Raw material and Electricity
94. Which of the following regions of India contain most of the (3) Labour and Raw material
reserves of coal, metallic minerals, mica and many other non- (4) Capital and Transport
metallic minerals? 98. Which one of the following factors is considered as the
(1) The Himalayas most prominent one in industrial location in a region?
(2) Alluvial plains of north India (1) Availability of raw materials
(3) Thar desert (2) Market
(4) Peninsular plateau region (3) Cheap Labour
95. Which one of the following mineral ores is formed by (4) None of these
decomposition of rocks, leaving a residual mass of 99. Which one of the following groups of states have the largest
weathered material? number of cotton textile centres?
(1) Coal (2) Bauxite (1) Gujarat and Maharashtra
(3) Gold (4) Zinc (2) Karnataka and Tamil Nadu
96. On the basis of character of raw material and finished (3) Maharashtra and Madhya Pradesh
product, iron and steel industry belongs to which category? (4) Uttar Pradesh and Gujarat
(1) Heavy industry 100. Which one of the following terms is used to describe trade
(2) Medium industry between two or more countries?
(3) Light industry (1) Internal trade (2) International trade
(4) Perishable goods industry (3) External trade (4) Local trade
HINTS & SOLUTIONS
SECTION 1. PHYSICS R 15
8. (4) Given : u = – 10 cm, R = 15 cm, f= cm
1. (1) Given : I = 15A, V = 220V, t = 30 min × 15 = 7.5 hr 2 2

Energy = P × t = VI t From mirror formula


= 220 × 15 × 7.5 1 1 1 1 1 2
v = – 30 cm
1 v u f v 10 15
= 3300 × × 15 W hr
2 Nature of image is real and inverted.
= 3.3 kW × 7.5 hr 9. (2) In series combination,
= 24.75 kW hr
Total cost = 24.75 × 2 = ` 49.50
2. (2) A= 0.7µm = 0.7 × 10 –6 m,

B = 0.34 µm = 0.3 × 10 –6 m
Lamp conductor
hc
Energy, E =

Since, A> B I 6V I
Hence, EA < EB
R = RLamp + Rconductor = 10 +2 = 12
3. (3) CO2 in the atmosphere does not allow infrared rays
from the earth to escape causing green house effect. V 6V
I= 0.5 A
R 12
12
4. (2) Slope = = 6 m/s2 (Retardation) VLamp = IR = 0.5 × 10 = 5V
2
Required force, F = m × a = 1500 × 6 = 9000 N
10. (3) Bulb1
5. (4) It is the fundamental equation of motion so it is not the i
consequence of Newton’s Laws of motion.
6. (3) Let the resistance of each resistor be R Bulb2
i
In series combination,
Bulb3
Rs = 3R , P = 10W, i

V2 V2 V2
P= 10 30 5A
Rs 3R R
Bulbn
i
In parallel combination,
220V 5A
V2 V2
P = 3 3 30 90 W
R/3 R
7. (3) To get largest resistance all ten resistors should be For a Bulb, Vi = P = 20 W
connected in series and in parallel to get smallest
P 20 1
resistance. i= A
V 220 11
RS = 10 × 0.1 = 1 (largest)
1
0.1 I = ni 5A = n A
RP = = 0.01 (smallest) 11
10
n = 55
EBD_7332
42
S- NTSE Stage 1 Question Bank

11. (3) Rt = R0 (1 + t) 22. (2) As mass is a property so it doesn’t changes. Mass is a


Hence, resistance of a conductor increases with constant quantity. Weight (W = mg) is a variable.
increase in temperature. 23. (2) Audible freq. 20 to 20000 Hz
12. (3) Displacement = velocity × time 24. (3)
25. (3) Joule is the SI unit of work and energy.
Velocity Pounded unit of force in FPS system
Dyne unit of force in CGS system
Newton unit is of force in SI system.

O 26. (1) Holland/Denmark is called the country of wind mills.


Time
x=v t
27. (3) 1 volt = 1 joule
= area of v-t graph coulomb
13. (2) As mass of the moon (= 1023 kg) is less than mass of
28. (1) Physical quantity Unit
the earth (= 1025 kg)
Frequency hertz
Gm1m2
And the gravitational force of attraction F = work/Energy joule
r2
14. (3) Among the voice of man, cow, bird and dog the voice Resistance ohm
of bird is shriller than other. Hence the pitch or frequency Heat (energy) kilocal (cas) system
of bird voice is more than man, cow and dog.
29. (3) The boiling point of water is 100°C or 100 + 273
15. (3) Electric fuse works on heating effect of current. CFL
works on electric discharge, relay based on magnetic = 373 K
effect, button cell based on chemical effect. 30. (1) Total displacement during t = 0 to t = 6s is equal to area
16. (1) The rate of evaporation increases with increase in under the graph = A1 + A2 + A3
surface area, temperature and wind speed.
5
4
m/s 3
17. (1) P 2
C 1 A1 A2
time
–1 1 2 34 5 6 7 8 9
Concave mirror
–2
Point ‘C’ is the centre of curvature and is lies in front of –3
concave mirror. –4 A3
–5
1
18. (2) P 1 1
f (in meter) Displacement 5 2 2 5 5 2
2 2
1 10 1000
f = P – 2.5 – 25 f = cm = 40 cm = 10 + 5 – 5 = 10m
25 31. (4) According to Kepler’s law of planetary motion, T2 R3
Focal length (f ) of a concave lens is always negative.
3
19. (4) E = mc2 (Einstein’s mass-energy equivalence equation)
R2 2
mass defect T2 = T1
1 amu = 1.66 × 10 –27 kg R1
E = 1.66 × 10 –27 × (3 × 108 )2 [ c = 3 × 108 ms–1] 3

= 1.66 × 9 × 10–11 J 9000 2


T2 = 24
11 36000
1.66 9 10
= 931 MeV [ geosynchronous satellite, T = 24 hrs.]
1.6 10 19 106 3
20. (1) Mercury is the nearest to sun. 1 2
21. (1) p = mv unit of m = kg T2 = 24
4
unit of v = ms–1
unit of momentum 1
T2 = 24 × 3hrs
P = mv is kg ms
–1 8
Exemplar Practice Questions 43
S-

V2 (220)2
32. (4) Resistance of bulb = = 484 0.693 0.693
P 100 40. (4) T1/2 4
1620 years
when supply is 110V, then power consumed 4.28 10

=
(110) 2
= 25W 41. (2) 1 kilowatt hour = 1000 × 3600 joule = 3.6 × 106 joule
484 42. (4) Except chromium other three constantan, manganin and
33. (1) Let R1 & R2 be the resistance of two coils A & B. Nichrome are alloy.
V 2 t1 V2t2 43. (2) Ammeter is used to measure the electric current.
Then, H = [Heat consumed will be 44. (1) Household electrical appliances are always joined in
R1 R2
parallel.
same]
45. (1) Refraction of light rays entering the eye takes place
t1 t from eye lens, cornea and vitreous humour but maximum
= 2
R1 R2 from the cornea of the eye.
But t1 = 6 min, t2 = 8 min 46. (3) We get virtual and erect image of an object only when
it is placed between the pole and the principal focus of
6 8 the concave mirror.
=
R1 R 2

6
R1 = R
8 2
3
= R
4 2 C F O P I
If both R1 & R2 are connected in series, then
R = R1 + R2
3 7 47. (2) Pole of the mirror is taken as origin according to the
R= R + R2 R= R new cartesian sign convention.
4 2 4 2
V2t V2 t2 t t
Now, H = R = R2 R1
= 2
1 R2 Pole
7 C F (origin)
As we know, t2 = 8 min, R = R.
4 2
8 7
t= R
R2 4 2 48. (1) Infacts refractive index of alcohol is 1.37 1.44
t = 14 min. 49. (4) The unit of force in SI system is newton.
34. (3) To see full view of image standing in front of mirror, And force, f = mass × acceleration
height of plane mirror should be half i.e., 3 ft. 1newton = 1 kg. 1 ms–2
1 [ unit of mass is kg and of acceleration ms–2 in SI
35. (3) We know P =
f system]
1 1 50. (1) Let the masses of the two objects be m1 and m2
f1 = , f2 =
3 1.5 u1, u2 be their initial velocities respectively. Let h be
1 1 1 1 the height from where the objects are dropped.
Now, = = 3 + (–1.5)
f f1 f2 f Let v1, v2 be the their final velocities and t1, t2 be the
time taken to strike the ground respectively
1 1
= 1.5 f = = 0.67 m. u1 = u2 = 0, h1 = h2 = h
f 1.5
1 2
h gt ...(1)
p1 E1 m1 1 1 2 1
36. (4) p 2Em 1: 2
p2 E2 m2 2 2 1 2
h gt ... (2)
2 2
37. (3)
2h
t1 t2 t ... (3)
c 3 108 g
38. (3) 6 1014 Hz
5000 10 10 now v1 = gt1 v2 = gt2
From equation (3), v1 = v2 = gt
10 v1 1
39. (1) RAC = (2 3) (4 6)
3 v2 1
EBD_7332
44
S- NTSE Stage 1 Question Bank

51. (2) Infrared radiations find applications in physio – therapy 57. (3) In screw gauge
and are used to take photographs of objects in dark. Negative zero error – If the zeroth division of the head
52. (3) Work done by a force on a body = Fs cos scale is above the index line the error is said to be
where F = Force acting on the body negative and the correction has to be positive.
s = Displacement due to the force applied Positive zero error – If the zeroth division of the head
= Angle between F and s scale is below the index line of the pitch scale, the error
Work done will be positive if cos is positive i.e. is said to be positive and the correction is negative.
0 < 90° 58. (2) -radiations emitted by radio isotopes are used as
When the body moves along the direction of applied radioactive tracers in medical science.
force i.e., = 0° -being EM radiations has more penetrating power than
Work done = Fs cos = Fs a and b particles and are unaffected by electric and
Work done here is positive. magnetic fields.
53. (4) A : A car moving up a hill 59. (1) Einstein’s photoelectric equation,
Moving car has kinetic energy, kinetic energy gets
1
converted to potential energy as it gains height (h). E= mv2 = h –h 0
And potential energy = mgh 2
B : Photographic film is exposed to sun – light 1 2
When photographic film is exposed to sun the silver or, h =mv W ( h 0 = W = work function)
2
bromide present in it decomposes 60. (4) According to Kepler’s third law of planetary motion,
such that the film cannot be developed. square of time period of revolution of a planet is directly
Light energy gets converted to chemical energy. proportional to the cube of semi-major axis. i.e.,
54. (4) In astronomical telescope 2 convex lens called eyepiece T2 r 3
and objective lens are used and object is placed before 61. (1) Frannhoffer’s spectral lines are also called line
eyepiece lens, such that final image inverted, a camera absorption spectra.
and eye also form inverted image on the screen. 62. (3) Upper side of the loop behaves as the north pole and
Whereas simple microscope gives an erect, virtual and the lower side as the south pole.
enlarged image of the object placed between first
principal focus and the optic nerve of the convex lens.

circular
loop

63. (2) The given equation or reaction represents nuclear


fusion reaction.
2
1H +1 H2 2 He3 +0 n1 + 3.27 Mev(energy)
In a projector, the image formed is real, inverted Hence X stands for 0n1
magnified on the other side of the lens. This inverted
image is again inverted by the film. hc
64. (1) Energy, E = h =
55. (1) Q = 1000 C; t = 3 min 20 sec = 200 sec
Hence, highest energetic rediation is whose wavelength
Q
Current flowing through a conductor is I = is least, i.e., = 0.01A°
t 65. (1) 1 joule = 4.18 4.2 cal
1000C 66. (2) Both are negative as electric field lines are coming
I= 5A towards charge.
200 sec
67. (2)
56. (1) -particles are electrons originating in the nucleus 68. (1) When the uncharged metallic sphere is brought nearby
Mass of an electron = 9.1 × 10–31 kg the positively charged plate. A negative charge is
Charge of an electron = – 1.6 × 10–19 C induced in sphere which causes attraction.
4 69. (4) Mercury vapour lamp is most intense man-made light
-particle is a doubly ionized helium atom 2 He source.
Its mass is 4 times mass of proton = 6.68 × 10–27 kg 70. (4) Cyclotron is a device that is used to accelerate charged
Its charge is 2 times the charge of proton = 3.2 × 10–19 C particles. Since a neutron is neutral by charge.
-rays are not particles but radiations 71. (3) v-t graph of particle is parabola.
X – rays are not particles but EM radiations 72. (1) As wastage of electricity is minimized.
73. (2) Water starts freezing at 0 degree.
Exemplar Practice Questions 45
S-

74. (3) Since V – t graph is parallel to time axis hence car has
2s s
constant velocity. Time taken for second half covered
75. (4) According to Newton's 3rd law action and reaction 2v v
acts on different objects.
s s 3s
Total time taken = + =
1 2 2h 2v v 2v
76. (3) h gt ; t
2 g
s 2
= = v
77. (4) v2 u2 2as Average velocity 3s 3
2 2
...(i) 2v
(0) v 2 a ( x)
83. (4) As we know,
(0) 2 (2v)2 2a( x1 ) ...(ii) work done = change in kinetic energy
x1 4 x 1 1
78. (1) Potential energy in stretched spring 2 4 10 4 10 100 v 2
2 2
1 2
U kx U x2 40 + 40 = 50v2
2 80 = 50v2
79. (2) Let the body fall through the height of tower in t
a 80
seconds. From, Dn = u + 2n –1 we have, total distance v= = 40 m / s
2 50 ×10 –3
travelled in last 2 seconds of fall is 84. (3) It is below the upper surface of the slab by 2 cm.
D = Dt + D(t–1) 85. (3) If the mirror is rotated by an angle , about an axis in
the plane of mirror, the reflected ray is rotated through
g g
= 0+ 2t –1 + 0 + 2 t –1 –1 an angle 2 .
2 2
f
g g g 86. (1) m =
= 2t – 1 + 2t – 3 = 4t – 4 f u
2 2 2
87. (4) Glycerine
10 88. (2) f1 < f2 and f3
= 4 t 1
2 89. (2) Rainbow is a three dimensional cone of dispersed light
or, 40 = 20 (t – 1) or t = 2 + 1 = 3s it will appear as complete circle. The shadow of the
Distance travelled in t second is
airplane will appear within the circle of the rainbow.
1 2 1 90. (3) The lemon appears to be bigger than its actual size.
s = ut +at = 0 + × 10 × (3)2 = 45 m.
2 2 91. (1) The ray will suffer greater deviation.
80. (1) The spring constant is inversely proportional to length 92. (3)
1 93. (1) Number of electrons = 1020
of spring i.e. k 94. (3) Let R be the resistance of each lamp. If E is applied
l
The new spring constant, emf, then the current in the circuit flowing through L2
k' = nk
81. (3) From Newton's second law of motion 1 2E E
or L3 =
2 3R 3R
d(mv)
F=
dt When L3 is fused, the whole current flows through L1
As is constant and L2.
dm E E
F= I2
dt R R 2R
dm 5 Current through L 1 decreases and through L 2
and 2
dt 2.5 increases.
F = 4 × 2 = 8 dyne 95. (2) According to the graph, the resistance of the device
82. (3) Let s be the total distance covered increases when the current increases.
s 96. (3) Angle should be /2
Time taken for first half covered =
2v 97. (2) (A) and (C) are incorrect.
EBD_7332
46
S- NTSE Stage 1 Question Bank

98. (1) The N-pole of the resultant magnet is on the face close
to A. 1 6.022 1023
7. (2) 8g oxygen = mole of oxygen = atoms
99. (1) A chain reaction of nuclear fission is triggered when 2 2
the nucleus of a heavy atom such as uranium, = 3.011 × 1023 atoms
plutonium or thorium is bombarded with low-energy
neutrons to split into lighter nuclei, three neutrons and 49.7 50.3
8. (3) Average atomic wt. = 79 81 = 80.006
a tremendous amount of energy. 100 100
100. (1) Nuclear fusion is joining of lighter nuclei to make a 9. (2) German silver is an alloy of Cu, Zn and Ni. Brass
heavier nucleus and a neutron. For example two contains Cu and Zn. Bronze is an alloy of Cu and Sn.
hydrogen isotopes join to give helium such as
10. (3) In electrochemical series position of Zn and iron is
2 2 3
1H + 1H 1He + n with release of large amount of above hydrogen where as copper and mercury are
placed below hydrogen. Hence Zn and Fe can replace
energy. That is the source of energy in the Sun and
hydrogen from dilute acids whereas copper and
other stars.
mercury do not.
11. (2) Molality of the solution
SECTION 2. CHEMISTRY
0.45 10 0.10 40 4.5 4.0 0.5
1. (1) Given : M2+ = 2, 8, 14; = = 0.01 M
50 50 50
Hence Electronic Configuration of M = 2, 8, 16
So, total electrons in M shell will be = 26 [OH ] = 0.01 M
i.e., atomic number (Z) = 26, Mass number (given) = 56 pOH = – log [0.01] = 2 [ pH + pOH = 14]
Number of neutrons = A – Z = 56 – 26 = 30 pH = 14 – 2 = 12
2. (2) Normality = basicity of acid × molarity 12. (3) Since Zn is more reactive than Sn, hence can be
Since H3PO4 is a tribasic acid hence corroded faster than Sn.
Normality = 0.3 × 3 = 0.9 N 13. (3) Proton = 1H1
Hydrogen atom contain 1 proton and 1 electron.
1
3. (3) CH3COOH C2H5OH conc.H 2SO 4
CH3COOC 2H 5 H 2O H1 H+ + e–. Hence H+ is a proton
ethylacetate
(ester) 14. (3) Mass of a proton is about 1835 times mass of an
electron.
This reaction is known as esterification. me = 9.1 × 10–31 kg
mp = 1.67 × 10–27 kg
4. (1) 15. (2) Elements belonging to the same group of the periodic
table shows same valency i.e. similar valence shell
electronic configuration
16. (1) When two free atoms combine to form a bond, some
heat is always evolved which is known as the bond
O formation energy or the bond energy.
5. (4) 17. (4) Crude petroleum is mainly a mixture of many
C Carboxylic acid group. hydrocarbons, with a wide range of boiling points.
OH H H

H C C H
18. (3)
C O Ketone group;
H H

O H H

C Aldehyde group and H C C O H


H
H H
OH alcohol group. due to presence of double bond ethelene is an
6. (3) Chloropicrin is commonly used as tear gas. Its chemical unsaturated hydrocarbon.
formula is CCl3NO2. 19. (3) Brass is an alloy of Zn and Cu
Exemplar Practice Questions 47
S-

20. (2) 40. (1) Vulcanization is a chemical process by which the


21. (4) pH of a neutral solution is 7. physical properties of natural or synthetic rubber are
22. (2) Argon is an inert gas hence have complete octet. improved. In its simplest form, vulcanization is brought
23. (3) K+ = 19 – 1 = 18 e– about by heating rubber with sulphur.
Ar = 18e– 41. (4) Since the compounds present in solution contain acidic
Hence they are isoelectronic. radicals SO 24 – and CH3COO– ions respectively. Hence
24. (3) The third alkali metal is K and its atomic no. is 19.
in their solution they will produce acids. hence the pH
25. (1) Silicon carbide is also known as carborundum.
of the solution will be less than 7.
26. (1) Molarity = no. of mole / vol of solution in ltr
42. (1) CH 4 2O2 CO 2 2H 2O
W 1000 1 mole 2 mole 1 mole 2 mole
0.2 = w= 4g 16 g 64 g
40 500
27. (2) Producer gas is mixture of carbon monoxide and Amount of oxygen needed for the combustion of 16 g
nitrogen. of CH4 = 64 g
28. (4) A mixture of liquid NH3 and CO2 is used for cooling 64
refrigerator. 1.6 kg or 1600 g = 1600 = 6400 g
16
or 6.4 kg.
29. (3) 2Pb (NO3)2 2PbO + 4NO2 + O2
43. (1) Bose - Einstein condensate is the fifth state of matter.
30. (1) Lanthanide series consists of 14 elements from BEC is super cooled solid.
Ce(Z = 58) to Lu (Z = 71) in which the last electron 44. (4) The cohesive force between H2O molecules is more
enters one of the 4f-Orbitals. Ce is a lanthanide with than the adhesive force between C2H5OH and H2O.
electronic configuration [Xe]4f1 5d16s2
45. (3) Colloidal solution is a type of heterogeneous mixture.
31. (1) CO2 being non-flammable, inert and denser than air
46. (3) 1 mole of N2 gas = 22.4 L of STP = 28 g of N2
and used in fire extinguisher.
32. (2) Atomic number potassium (K) is 19. = 22400 c.c. of N2 at STP
33. (2) Gallium melts at a temperature of 29.76°C. = 6.022 1023 molecules of N2 gas.
34. (1) The given compound is 1° alcohol with 3 carbon 47. (3) Cetane number is a quantity indicating the ignition
straight chain. properties of diesel fuel relative to cetane as a standard.
35. (4) This is expected that the configuration of copper is 48. (4) 1 g CO2 n = 1/44 = 0.02
3d9 4s2. However, it turns out that the 3d10 4s1 Number of atoms = 0.02 × 3 × NA = 0.06 NA atoms
configuration is more stable, because that way the 3d 1 g N2 1/28 = 0.035, Number of atoms = 0.071NA
sub-shell is full, which is a far more stable arrangement 1 g O2 1/32 = 0.031, Number of atoms = 0.0625NA
than 3d9.
1 g CH4 1/16 = 0.625, Number of atoms = 0.312NA
36. (1) German silver or nickel silver is a metal alloy of copper
with nickel and zinc. The usual formulation is 60% Hence maximum number of atoms present in 1 g CH4.
copper, 20% nickel and 20% zinc. It is named for its 49. (4) CuSO4.5H2O – Blue vitriol, ZnSO4.7H2O – White
silvery appearance, but contains no elemental silver. vitriol, FeSO4.7H2O – Green vitriol.
37. (1) Fusion of 1 g of hydrogen generate 6.4 × 1011 joule. 50. (1) P1V1 + P2V2 = Presult.Vresult;
Whereas the fission of one atom of U-235 generate P1 = P2 = P ; V1 = V2;
202.5 MeV = 3.244 × 10–11 J, PV + PV = PR (2V)
Now since 235 g of uranium contains
= 6.023 × 1023 atom 2PV
PR P = 400 torr
2V
6.023 10 23
since 1 g of uranium contains 51. (3) Mass of solute + Mass of solvent = 34.7 + 100 g = 134.7 g
235
hence fission of 1 g of 0.235 generates Mass of solution = 134.7 g
6.023 1023 Density of solution = 1.3 g/mL
11
3.244 10 J = 8.31 × 1010 J Volume of solution = 134.7/1.3 = 103.62
235
W/V percentage
41 H1 2 He
4
21 e0
38. (3) Gypsum plaster, or plaster of Paris, is produced by Mass of solute 34.7 100
= 100 33.49%
heating gypsum to about 300°F. Its chemical Volume of solution 103.62
composition is CaSO4·½H2O. 52. (3) K2SO4.Al2(SO4)3.24H2O.
39. (4) A double decomposition reaction is a chemical reaction
between two compounds in which parts of each are 53. (2) Fe(s) CuSO4 (aq) FeSO4(aq) Cu(s)
(Iron nails) (Green coloured
interchanged to form two new compounds. Solution formed)
EBD_7332
48
S- NTSE Stage 1 Question Bank

54. (3) Sugar is non volatile, only solvent will evaporate on 81. (3) Impurities collected at anode in electrolysis during
heating, so solute amount will remain constant. purification of metals.
55. (2) M2O = 94 82. (1) Aqua regia, is a freshly prepared mixture of concentrated
HCl and conc. HNO3 in the ratio 3 : 1, which can even
2M + 16 = 94 dissolve gold and platinum in it.
M = 39, Cl = 35.5 83. (4) Oxides of reactive metals like Ca & Mg forms very
Metal is K. Metal chloride KCl = 74.5 u strong bond with oxygen which cannot be broken by
smelting.
56. (4) S O2 SO 2 84. (2) Drinking ethyl alcohol is very harmful and it ruins the
X (Pungent smelling gas)
health.
85. (1) This is an example of addition reaction
SO 2 H 2O H 2SO3
(Sulphurous acid) 86. (3) X-carbon dioxide, Y-ethane.
87. (2) Non-metallic character increases on moving from left
57. (3) H3BO3 or B(OH)3.
to right in a period.
58. (1) As both shows non-metal configuration. 88. (4) Isoelectronic species have same number of electrons.
3 N3-, Na+ and F- all have ten electrons just like O2-.
110 10
59. (2) As 100 0.110% of Na. However, Ti+ (Z = 22) has twenty one (21) electrons.
100 89. (2) Eka-aluminium predicted by Mendeleev is gallium.
60. (3) Cow dung cake < Wood < Petrol < Biogas. 90. (1) The element plutonium has atomic no. 94, therefore it
is called as actinide.
61. (1) Mainly oxalic acid is present in spinach. 91. (2) A, B, C and D belongs to same group as they have
62. (1) 63. (4) same number of electrons in valence shell. As we move
64. (2) 65. (4) down in a group atomic number increases and
66. (4) electrostatic force of attraction between nucleus and
67. (2) The oxidation of oils and fats in a food resulting into a valence electrons decreases. Thus on moving down in
bad smell and bad taste is called rancidity. a group melting point decreases.
92. (3) Polonium is a borderline element, as it is a semi-metal.
68. (3) Reaction between liquid oxygen and liquid hydrogen
93. (2) Let the given ratio of atomic weights of X, Y, Z be 5x,
to form water is an extreme exothermic reaction, the
11x, 17x respectively.
large amount of energy is released which is utilized to
upthrust space shuttle. Sum of extreme elements (5x + 17x) = 176 (given).
176
2H 2 l O2 l 2H 2O l 22x 176, x 8
22
572kJ 286 kJ / mol At wt. of X = 5 × 8 = 40, Y = 11 × 8 = 88, Z = 17 × 8
= 136.
69. (1) KMnO4 is an oxidising agent.
94. (2) HCl and CCl4 are covalent compounds.
70. (1) Basic copper carbonate which is green in colour is
95. (1) Non-metals usually form acidic oxides.
formed when copper reacts with CO2 in presence of
moisture in the air. 96. (1) Air
71. (4) H H H H H
72. (2) | | | | |
73. (2) Bleaching powder gives chlorine on exposure to 97. (3) H—C— C— C— C— C—H
| | | | |
atmosphere.
H H H H H
74. (1) Al2(SO4)3 on hydrolysis gives Al(OH)3 & H2SO4, and pentane
therefore will be acidic in nature.
98. (3) Bromine water will discharge the colour of ethene when
75. (3) Peroxy disulphuric acid is also known as Marshall acid.
the vapours of the gas are passed through it.
76. (2) Chemical formula of oleum - H2S2O7
H H
77. (4) compound ‘X’ is sulphuric acid that reacts with | |
ammonium carbonate to form a weak, acidic, ammonium H—C C — H + Br2 H—C C—H
sulphate along with carbon dioxide gas and water. Ethene (yellow) | |
Br Br
78. (2) Slaked lime Ca(OH)2 (colourless)

79. (4) Aluminium has high melting point and good thermal Ethane will not react with bromine water and the colour
conductivity. will not be discharged.
80. (3) Silver articles become black on exposure to air due to 99. (2) Elements of oxygen family are known as chalcogens.
the formation of Ag2S. 100. (1) Acids
Exemplar Practice Questions 49
S-

SECTION 3. BIOLOGY 15. (4) Carolus Linnaeus gave the system of ‘Binomial
Nomenclature’ for providing scientific names to known
1. (3) In mitosis, the number of chromosomes in the parent organisms. Each name has two components–the
and progeny cells are the same. Generic name and the specific epithet.
It is also known as equational division. There is no The term ‘species’ was coined by John Ray.
crossing over takes place in mitosis cell division. Father of Biology is Aristotle.
2. (3) Mitochondria and chloroplast semi-autonomous A.P. Decandolle introduced the term ‘taxonomy’.
organelles. Mitochondrial matrix possesses single 16. (1) Carrot contains and – carotene which are partly
circular DNA molecule and chloroplast also contains metabolised into vitamin A in humans.
small, double-stranded circular DNA molecules. Both 17. (1) Holstein is a breed of cattle known today as the world’s
have 70s ribosomes. highest production dairy animal. It is black and white
3. (2) Cytokinins promote nutrient mobilisation and delay due to artifical selection by breeders.
senescence (ageing) so chrolophyll will be retained 18. (2) Phloem transports food materials, usually from leaves
and leaves will remain green for longer time. to other parts of the plant.
19. (4) Adrenaline is an animal hormone secreted by adrenal
4. (1) Pepsin is major enzyme of gastric juice secreted from
gland.
the lining of stomach and functions in acidic medium
(pH 2.0 – 3.5). 20. (1) There is no single medicine for hepatitis. Treatment is
specific to its cause, which means the physician will
5. (3) Malaria is a communicable (protozoan) disease caused choose the best therapy based on diagonosis. People
by Plasmodium vivax while others are congenital with the flu are advised to get plenty of rest, drink
(genetic) disorders. Plasmodium enters the human plenty of liquids, avoid using alcohol & tobacco and if
body as sporozoites (infections form) through the bite necessary, take medications.
of infected female Anopheles mosquito.
21. (3) Rohu — Column feeder
6. (1) Vegetative propagation refers to asexual mode of Cotta — Surface feeder
reproduction in which formation of new plants occur Mrigal — Bottm feeder
from vegetative parts (stem, leaves and root) of the 22. (1) Auxin and Cytokinin are plant growth promoters. They
plant.
are involved in various growth promoting activities,
7. (2) (i) According to Mendelian Inheritance an allele which like cell division, cell enlargement, flowering, fruiting
cannot express itself in presence of other is and seed formation.
recessive, hence can be expressed only in 23. (1) Funaria, Riccia and Marchantia belong to the
homozygous condition. Bryophytes which don’t have any conducting tissue
(ii) Exceptions are there in non Mendelian in heritance whereas fern belongs to pteridophytes. Evolutionarily,
(in complete dominance, co-dominance). pteridophytes, are the first terrestrial plants to possess.
Vascular tissues — xylem and pholem.
8. (1) Kalaazar or Dumdum fever is caused by a flagellate
24. (1) Pyruvic acid and lactic acid are 3-carbon compounds,
protozaon Leishmania donovani. The primary host for
glucose is 6-carbon compound. Ethanol (C2H5OH) is a
this parasite is human and the secondary host is sand
2-carbon compound.
fly (Phlebotomus).
25. (3) Darwin was most influenced by Research work of
9. (2) Pepsin is a major enzyme of gastric juice functions in
Malthus when he returned from the voyage in 1836
acidic medium. HCl makes the food acidic (pH 1-2) for
and got a chance to read the book ‘Essay on Principles
proper functioning of pepsin.
of Human Population by Malthus. Malthus in his book
10. (3) Blood is a fluid connective tissue containing plasma, described that human population increases in geometric
red blood cells (RBC), white blood cells (WBC) and ratio while food increases in arithmatic ratio.
platelets.
26. (4) A large proportion of all life on Earth exists in the oceans
11. (2) Budding is an unequal division of the parent where the
which provide 300 times as much habitable volume as
identity of the parent body is still maintained. Examples
terrestrial habitats. Marine habitats range from surface
are sponges, Hydra, among fungi it is formed in yeast.
water to the deepest oceanic trenches, including coral
12. (3) Stanley L.Miller and Harold C. Urey performed reefs, seagrass meadows, muddy, sandy and rocky
experiment by using ‘Spark discharge apparatus. They bottoms, and the open pelogic zone.
used the mixture of water vapours (H2O), hydrogen 27. (2) Palisade parenchyma cells are found within the
(H2), ammonia (NH3) and methane (CH4), at 800ºC. They
mesophyll in leaves of dicotyledonous plants. They
observed the formation of amino acids.
contain the largest number of chloroplasts per cell and
13. (4) The homologous organs are similar in origin or converting the energy in light to the chemical energy
fundamental structure or basic plan, but may or may
of carbohydrates. Their cylindrical shape allows a large
not be similar in functions, i.e. can differ in functions.
14. (2) Ribosomes are also known as ‘Protein factories of Cell’. amount of light absorbed by the chloroplasts.
They are present in both prokaryotes (70s) and 28. (1) Oxidative phosphorylation takes place in mitochondria.
eukaryot es (80s). Mitochondria is responsible for It is the synthesis of energy rich ATP from ADP and
respiration. Chloroplast helps in photosynthesis. inorganic phosphate, that is connected to oxidation of
Smooth Endoplasmic Reticulum helps in fat synthesis.
reduced coenxymes produced in cellular respiration.
EBD_7332
50
S- NTSE Stage 1 Question Bank

29. (3) G2-Phase is second growth phase, also called gap II, 47. (4) In plants, permanent tissues are of two types - Simple
post–synthetic phase or pre-mitotic phase. In this and Complex. (1) Simple permanent tissues are a
phase, cell size increases, nucleus grows in size, collection of similar cells that perform a common
centrosome replicates, mitochondria and plastids function these are parenchyma, collenchyma and
undergo division and multiplication of other cell sclerenchyma. (2) Complex permanent tissues are a
collection of different types of cells performing a
organelles also occurs.
common function. Xylem and phloem are the complex
30. (3) Cerebrum forms the major part of the human brain
tissue found in vascular tissue fo all vascular plants.
divides longitudinally into two halves left and right 48. (3) Irritability or sensitivity is the characteristic of living
cerebral hemispheres. The hemispheres are connected organisms responding to a stimulus. Animals show
by a tract of nerve fibres called Corpus callosum. quick response to a stimulus because they additionally
31. (4) Chlamydomonous species are belong to the members possess nervous system, e.g., pain on being pricked,
of chlorophyceae (commonly called green algae). In running away from the sight of predator.
Chlamydomonas, sexual reproduction occurs through 49. (3) Cerebellum is second largest part of the brain. It
isogamy, anisogamy and oogamy. coordinates muscular activity of the body. It also
32. (4) There are five steps in holozoic (organic food is taken maintains equilibrium or posture of the body as during
from outside) nutrition–ingestion, digestion, walking, jumping, lifting, catching and bending etc.
absorption, assimilation and egestion. 50. (2) Prolonged increase in blood sugar leads to complex
33. (1) Impulse is a self-propagated electrochemical current disorder called diabets mellitus which is associated
that travels from one end to another of a neuron for the with loss of glucose through urine and formation of
passage of a message. The pathway is Stimulus harmful compounds known as ketone bodies.
Dendrite Cell body Axon Axon terminal 51. (2) Mydrotropism tropism is the turning or bending
passage of stimulus. movement of an organism or a part toward or away
34. (2) Coordination is orderly or harmonious working of form an external stimulus such as light, water, gravity
different but inter-related parts of the body so as to and earth.
perform one or more activities very smoothly. 52. (4) Adrenaline and cortisone are secreted in response to a
35. (2) Each flower normally has four floral whorls, i.e., calyx perceived threat in the environment. The effects of
(sepal), corolla (petal), androecium (stamen) and stress hormones on blood glucose, heart rate and
gynoecium (carpel). respiration increase oxygen and nutrient supplies to
36. (4) Felis is a genus of cats in the family Felidae, including muscles and temporarily shut down the maintenance
the familiar domestic cats and its closet wild relatives. of the body’s other systems.
37. () Duck billed platypus is the only mammal that lays eggs 53. (1) To test the adulteration of cooking oil, add few drops
instead of giving birth. of conc. nitric acid (HNO3) and shake. Appearance of
38. (4) Fragment of DNA that provides complete information red colour in the acid layer indicates the presence of
about one protein is referred to as gene for that protein. adulteration.
39. (4) Reproduction 54. (3) DNA finger printing is a technique used in forensic lab
40. (1) Blood vascular system of cockroach is an open type. to identify criminals to check paternity etc. In this
Heart consist of enlongated museular tube. It is technique, DNA from tissues such as blood, hair, skin
and semen etc. are taken from an individual to show
differentiated into furnel shaped chambers Blood from
the same degree of vartiation at genetic level.
Sinuses enter heart through Ostia.
55. (2) The flow of energy from autotrophs to herbivores is
41. (3) Major Natural Auxin is indole - 3 - acetic acid (IAA) unidirectional.
occuring in biological membrane of both plant aqnd 56. (4)
animal cell. 57. (2) Translocation is a process of transportation of
42. (2) The small buds which are araises on leaf margins. photosynthetic food through phloem.
43. (3) 58. (1) Helicobacter pylori causes peptic ulcer.
44. (3) The female make ovipositor to make hole in the manure, 59. (3) Endoplasmic Reticulum helps in the formation of fat
garbage or other decaying material she has selected and lipid (SER) and protein (RER).
and releases her fertilized eggs. 60. (1) Concentrates given as feed to the cattle to improve
45. (1) Vit B12 immunity and lactation because it has high fibrous and
Deficiency Symtoms include lack of cordination, pain, Nutritional value.
61. (1) Inflamation occurs due to swelling and pain.
numbness & tingling in hands or feet, sensory loss.
62. (3) Nitrosomonas is responsible for the Nitrification.
46. (3) The pressure exerted by the blood on the walls of blood
63. (1) E. coli is the most commonly known species, of faecal
vessels is called blood pressure. In human beings 120 matter which indicates the quality of water.
mm of Hg is systolic pressure during contraction of 64. (1) Antibiotic penicillin inhibits the formation of cell wall
ventricles, and 80 mm of Hg is diastolic pressure during and hence stops the growth of bacteria.
relaxation of ventricles.
Exemplar Practice Questions 51
S-

65. (1) Crypto means hidden and gamae means marriae. So 94. (2) When an individual organism increases in size via cell
cryptogamae has hidden reproductive organ. multiplication and remains intact, the process is called
66. (2) Ligament connects bone to bone which has limited "vegetative growth". However, in vegetative reproduc-
flexibility. tion, the new plants that result are new individuals
67. (4) Cytopyge helps in the removal of undigested food. almost similar in every respect except genetically.
68. (4) During the study of evolution "Charles Darwin" wrote 95. (3) Both sperm and ova contain 23 chromosomes, total 46
a book called "Origin of species". chromosomes
69. (4) Cloning is a technique of making a new individual with 96. (2) It depends upon distance between pollen grain on upper
the help of a cell. surface of stigma and ovule.
70. (3) Ferns and mosses reproduce by releasing millions of 97. (1) If both parents are homozygous recessive then all the
sports through the air. The sperm is released from the children will express recessive trait.
antheridium and go through water to a nearby arch
98. (3) Mendels laws of heredity were rediscovered by
gonium to mix with the egg. Devries, Correns and Tschermark.
71. (1) Endemism is the ecological state of a species being
99. (1) Study of heredity and its inheritance is known as
unique to a defined geographic location.
genetics.
72. (2) Virus are not made up of cells, and do not use cells in
any of their processes. 100. (1)
73. (3) African tryponamiasis also known as sleeping
sickness, is an insect-borne parasitic disease of SECTION 4. MATHEMATICS
human and other animals.
74. (2) Outer Covering of virus made up of protein is capsid. 1. (2) Total cost = 80 × 6.75 + 120 × 8 = ` 1500
75. (1) Chlorophyll a absorbs blue and red light it reflect green
light strongly so it appear green to us. 120
Total Selling price = 1500 × = ` 1800
76. (3) Smooth Endoplasmic Reticulum 100
77. (4) Clothing requires calcium ions (Ca2t) (which is why
1800
blood banks use a chelating agent to bind the calcium S.P. per kg = =`9
in donated blood so the blood will not clot in the bag) 200
78. (1) 1
79. (2) When placed in hypertonic solution, a RBC will lose 2. (4) x+ =3
x
water and undergo shrinkage.
80. (1) Biological Oxygen Demand contribute to organic waste 1 æ 1ö
in the water, which is then decomposed by bacteria. x3 + + 3ççç x + ÷÷÷ = 27
x3 è xø
Low BOD indicate good Quality water, while a hight
BOD indicates polluted water. 1
81. (4) Active transport is associated with accumulating hight x3 + = 27 – 9 = 18
x3
concentration of molecules that the cell needs. If the
process uses chemical energy inform of ATP, it is æ 3 1ö
2
ççç x + 3 ÷÷÷ = x + 6 + 2
6 1
termed as primary active transport.
è x ø x
82. (4) The filtration of blood takes place in Bowman’s
capsule. 1
324 = x + 6 + 2
6
83. (2)
84. (2) The urine expulsion organ in mammals is urinary x
bladder
1
85. (2) Deficiency of proteins initiates Erythropoesis. x6 + = 322
86. (4) x6
87. (3) Hemodialysis is used to remove nitrogenous waste
products from the blood. 1
3. (1) , are the roots of k2x2–17x + (k+2)
88. (1) Pepsin acts in acidic medium.
89. (4) Insulin hormone is responsible for of the regulation of
1 k 2
blood sugar level in the body.
90. (2) k2
91. (2) Because ovule contains a sugary substance which k2 = k + 2
induces the pollen tubes towards ovule.
k2 – k – 2 = 0
92. (1) These branches help the nerve cell to perform
communication with other cell as seen in reflex action. k = 2 and k = – 1
93. (3)
But k > 0, k=2
EBD_7332
52
S- NTSE Stage 1 Question Bank

x PR : RQ = 3 : 2
4. (2) Probability of black balls =
20 Let co-ordinate of R be (x, y)
x 10 By section formula :
Probability of black balls (New) =
30 Abscissa of point R =
x 10 2x
= (Given) m1x 2 + m 2 x1
30 20 x=
m1 + m 2
x= 5
3(–1) + 4 × 2
5. (2) Marks f cf x=
3+ 2
0–5 10 10
5 –10 15 25
–3 + 8 5
10 – 15 12 37 x= 1
5 5
15 – 20 20 57
20 – 25 9 66 11. (2) Let BC be the ground and plane at A. Let the other
plane at Dvertically below A. Let the distance between
Median class is 10 – 15 the two planes be x meter
Modal class is 15 – 20 In DBC,
Sum of lower limits = 25 A
6. (2) 13, 18 ......98
In this A.P. we have 18 terms x
18 3150 m
S18 = [2 × 13 × 17 + × 5] = 999 D
2
A
(3) AC2 = AB2 + 49

3150-x
7.
60°
(AC – AB)(AC + AB) = 49 25 30°
24
B
AC + AB = 49 C
AC = 25, AB = 24 C 3150 – x 1
B tan 30 ... (i)
BC 3
24 7 31
cos A + cos B + cos C = +0+ = Now, in ABC,
25 25 25
3150
h tan 60 3 ... (ii)
8. (4) tan = BC
9 A
From (i) and (ii),
h
tan (90 – ) = h 3150 x 1
16
90°– 3150 3
B 9 C D
2 16 3(3150 – x) = 3150
h
tan × cot = h = 3 × 4 = 12 m
9 16 3x = 6300
9. (1) P(4, k) lies on y = 6 – x x = 2100 metre.
k=2 12. (3) Given a(a + b) = 36 and b (a + b) = 64
Volume of cylinder = × (4)2 × 2 = 32 By adding both,
10. (3) PR : PQ = 3 : 5 a (a + b) + b (a + b) = 36 + 64
(a + b) (a + b = 100
3 (a + b)2 = 100
a + b = 10 ( a+b – 10)
P R Q
(4,1–3) (x,y) (–1,7) 10a = 36 and 10b = 64
10 (a – b) = – 28
5
a – b = – 2.8
Exemplar Practice Questions 53
S-

a c a 20. (1) Given equation is ax2 + bx + c = 0


13. (4) Let and be the roots. Then
b c b
+ = – b/a, = c/a
ab + bc > ab + ca
1 1 a
bc > ca Given, Also
c
b>a b a
a2 bc a2 + bc = 0
a < b. a c
21. (1) Consider,
14. (3) Let a b c 0
1 1 cos2 1 cos2
cot 2 =
a b c sin 2 sin 2 sin 2 sin 2
Squaring both the sides, we get sin 2
1
2 sin 2
a b c 22. (4) Let AB be the tower and BD be the ground.
Let h be the height of the tower.
a+b+2 a b c h
In ADB, tan 30º
20 x
a+b–c=–2 a b
1 h 20 x
h
2 3 20 x 3 A
(a + b – c)2 = 2 a b = 4ab In ABC,
15. (4) Number of red balls = 5 h
tan 60º = h
Let number of blue balls = x x
Total number of balls = 5 + x h= 3x 30° 60°
D C B
20 x
Probability of drawing one blue ball = 2 × Probability 20 x
of red ball Now, 3x =
3
3x =20 + x x = 10
x 5
2
x 5 x 5 h = 3 10 10 3 m
23. (1) Let sin + cos = 1
x = 10
Squaring both side, we get
Hence, number of blue balls = 10 sin2 + cos2 + 2 sin cos = 1
16. (4) A(a, b + c), B(b, c + a), C(c, a + b) sin cos = 0
24. (3) It is a property.
1
Area ( ABC) = |a(c + a) – b(b + c) + b(a + b) b b2 – 4ac
2 25. (4) Roots will be
2a 2a
– c(c + a) + c(b + c) – a(a + b)|= 0 26. (2) Since, (x, 0), (0, y) and (1, 1) are collinear
17. (1) S = {1, 2, 3, 4, 5, 6} Then, area of ABC = 0
n(S) = 6 1
[x (y – 1) + 0 + 1(0 – y)] = 0
E = event of getting a number greater than 2. 2
xy – x – y = 0 x + y = xy
E = {3, 4, 5, 6} n(E) = 4
3
n(E) 4 2 27. (1) Let sin (A + B) =
Required prob = P(E) 2
n(S) 6 3
sin (A + B) = sin 60°
18. (3) Consider (a + b)3 – (a – b)3 A + B = 60° ...(1)
3 3 2 2 3 3 2 2
= a + b + 3a b + 3ab – (a – b + 3ab – 3a b)
3 2 2 2
3
= 2b + 6a b = 2b(b + 3a ) Let cos (A – B) =
2
19. (1) Let the required number be x. cos (A – B) = cos 30°
12 8 A – B = 30° ...(2)
x 15 Solving (1) and (2)
x 10
A = 45°, B = 15°
EBD_7332
54
S- NTSE Stage 1 Question Bank

1 33. (2) Area =


28. (1) Volume of frustum = h (r12 r22 r1r2 ) 4 4 a 4 b 4 c
3
1 22 = 4 4 3 4 3 4 2 = 2 2 4(2)
= 14(2 2 12 2)
3 7 a = 3, b = 3 and c = 2
1 308 2 ( a < 4, b < 4, c < 4 and a + b + c = 8 a+b>
= 22 2 7 = = 102 cm3 c, b + c > a and c + a > b)
3 3 3 34. (4) A female can handshake only with all males and except
29. (4) Given equation is her husband, there will be 13 handshake.
1 1 1 Total handshake by female is 12 × 13 = 156
= A male can handshake with 12 females & 12 males
x a x b c
x2 + (a + b – 2c)x + ab – (a + b)c = 0 excluding his wife & himself
Let and be two roots of this equation. so total hanshake is 13 × 24
Given : + = 0 But since handshake by a with b is also handshake by
b with a.
– (a + b – 2c) = 0 a + b = 2c ……(i)
Now, = ab – (a + b)c 12 24
Total handshake will be 13 × = 234
a b 2
= ab – (a + b) [from (i)] 35. (3) Given observations are
2
2 x x x x x x x x x x x x
2ab a b , , , x, , , , , , , , ,x
= 7 5 6 4 3 2 7 6 5 4 3 2
2 Given Median = 8
2ab a 2 b2 2ab a 2 b2 Since number of observation is odd
= =–
2 2 7 1
th
Median = obs. = 4th obs = x/4
30. (1) According to fermat’s Little Theorem, 2
If p be a prime number, then remainder x/4 = x = 32
when Qb divided by p is ‘Q’.
m× n = 7777......7×9999......9
14243 1424 3
Since 1997 is a prime number, therefore when (1)1997, 36. (2) 99 Digits 77 Digits
(2)1997 , ... , (1006)1997 are divided by 1997 then
remainder will be respectively 1, 2, ..., 1996
1 = 7777......7
14243 × 100......0
1
4243 1
Now 1 + 2 + ... + 1996 = × 1996 × 1997 = 998 × 1997, 99 Digits 77 Zeros
2
Which is divisible by 1997. = 7777......7000......0
14243 1 424 3 – 7777......7
14243
Hence remainder = 0 99 Digits 77 Zeros 99 Digits
31. (2) The product of 1000 integers is 1000
= 77......79......99
1
424 31 424 3 – 2......2223
1424 3
Since we need to find the maximum sum of these 76 Digits 22Digits 76Digits
integers.
Sum o f Digits = 532 + 6 + 198 + 152 + 3 = 891
We take, integer as the largest factor of 1000 which 37. (4)
is 1000 and the rest 999 integers be all 1's 1= 1+ A ...(i)
(By Enterior Angle Property)
Hence the maximum sum is 1000 + 999 times 1 = 1999
2= 2+ A ... (ii)
2f n 1
32. (2) Given f (n + 1) =
2
and f (1) = 2 then f (101) = 2
2f 1 1 5
for n = 1, f (2) = =
2 2
2f 2 1
for n = 2, f (3) = =3
2 After adding eqn (i) and (ii), we get
2f 3 1 6 1 7 1+ 2 = ( 1 + 2) + 2 A
f (4) = = = = (180 – A) + 2 A (By Angle Sum Property)
2 2 2
1 + 2 = 180 + A
5 7
So, f (1), f (2), f (3), f (4), ……, 2, ,3, Now,
2 2 (180° + A) + (180 + B) + ...... + (180 + G)
3 n = 2 (Sum of interior angles of Heptagon)
f(n) = = 2 × (7 – 2) × 180.
2
3 101 = 10 × 180°
f(101) = = 52 Therefore, A + B + .... + G = (10 – 7) × 180° = 540°
2
Exemplar Practice Questions 55
S-

38. (1)
r 2 10
39. (4) Let r be the radius and h be the height of the cone. r12h1 =
9 h1
1 2
Volume of the cone = r h 2
3 r1 10
. h1 = r1
r 20 r r 6r r 9
Now, New radius = r + = = Since, both cones are similar h
100 5 5
h h 20 6h r 30 r
New height = = r1 h1
100 5
2 2
1 6r 6h h1 10
New volume = Thus, h1
3 5 5 30 9
h1 = 10 cm h = 30 – h1
1 36r 2 6h 1 2 = 30 – 10 = 20 cm
r h
3 5 5 3 44. (4) Given, 2 r = S and r2 = A
% increased = 100
1 2 4 2r2 = S2 (on squaring)
r h
3 4 ( r2) = S2
1 2 216 4 A = S2
r h 1 100
3 125 91 S2 = 4 A
= = 100 = 72.8%
1 2 125
r h 45. (1) Consider x2 – 6x + 5 < 0
3
x2 – x – 5x + 5 < 0
a (x – 1) (x – 5) < 0
40. (3) Circum radius R =
3 1<x<5
a 46. (3) Given G.P is a, ar, ar2
Inradius r = where ‘a’ is the first term and ‘r’ is the common ratio.
2 3
Also, given a + ar + ar2 = 21
a2 and a(ar) (ar2) = 216
Area of circumcircle 3 12 4 (ar)3 = 216 = (6)3 ar = 6
Now,
Area of incirlce a2 3 1 Hence, middle term = ar = 6.
12 7 5
47. (2) Let x y 5
41. (1) Consider AB2 + BC2 2 2
A 7x + 5y = 10 ...(1)
and 4x + 2y = 7 ...(2)
2
x +1
2 (1) and (2) can be written as
x –1
28x + 20y = 40
28x + 14y = 49
B C (–) (–) (–1)
2x
6y = –9
= (x2 – 1)2 + (2x)2 = x4 + 2x2 + 1 3 7 3 10 5
= (x2 + 1)2 = (AC)2 y= and x =
2 4 4 2
Hence, ABC is right angled triangle.
42. (1) Given 20 5 × 6 + 2 – 10 5 3
Now, x – y = 4
After applying the given notations, we get 2 2
20 × 5 – 6 2 + 10 = 20 × 5 – 3 + 10 48. (2) Since opposite angle of a cyclic quadrilateral is
= 100 – 3 + 10 = 107 supplementary therefore here is three sets of 4 cyclic
1 points. Four cyclic quadrilateral are OFAE, ODBF and
43. (4) Volume of small cone = volume of big cone. OECD.
27
49. (1) Let equation of fourth side be y = k.
1 Then (k + 3)2 + 82 = (10)2 (By distance formula)
Volume of smaller cone = volume of given cone
27 (k + 3)2 = 100 – 64 = 36
1 2 1 1 (k + 3) = ± 6 k = –9 or k = 3
r1 h1 r 2 30
3 27 3 Hence, required line is y = – 9
EBD_7332
56
S- NTSE Stage 1 Question Bank

9 11 2 57. (3)
50. (2) Let a 11 2 (by A B C
11 – 2 11 2 AC 2
rationalizing) Given where A (– 2, 5) and B (3, 2)
BC 1
2 2 3
Let b Since, ‘C’ divides the line AB is externally in the ratio 2 : 1
3 3
clearly a > b By section formula we have,
51. (4) Let r and s be the roots of equation ax2 + bx + c = 0 2(3) –1(–2) 2(2) –1(5)
–b c C , C (8, –1)
So, r s , rs 2 –1 2 –1
a a
1 1 s2 r2 ( s r )2 – 2rs 58. (3) Sum of all the number = 49 × 100 = 4900
Now, Sum of numbers in error = 110
r2 s2 (rs )2 (rs )2
Sum of numbers (corrected) = 210
b2 c 4900 110 210 5000
–2 50
a 2 a b2 – 2ac So, Actual mean
100 100
c2 c2 59. (1) Given equation is
a 2 x2 + 4 = (P + 2) x
52. (2) Let ‘a’ be the first term and ‘d’ be the common difference x2 – (P + 2) x + 4 = 0 a = 1, b = – (P + 2), C = 4
of an A.P. Since, given equation has equal roots
Given sum of first ten terms = 4 × sum of first five terms
b2 – 4ac = 0 [– (P + 2)]2 – 4(4) = 0
10 5
[2a (10 –1)d ] 4 [2a (5 –1)d ] P + 2 = 4 P = 2, – 6
2 2 60. (4)
2a + 9d = 4a + 8d No. of Goals
2a = d ... (1) Vinay Raja Jayanth
kth term of an A.P. is a + (k – 1) d 0 0 3
= a + (k – 1) 2a {by (1)}
= 2ak – a = a (2k – 1) 0 1 2
4 4 0 2 1
36 9 63 9 0 3 0
53. (4) Consider a a
1 0 2
1 1 1
9
4
9
4 91 4 91 4
= = . . 1 2 0
3 a 6
6 a 3
a6 3 a3 6 2 0 1
= a2. a2 = a4
54. (3) Let a, b, c be three numbers 2 1 0
Given a + b + c = 98 ... (1) 3 0 0
a 2 b 5 2b 8b No. of favorable cases
and , a ,c Required probability
b 3 c 8 3 5 No. of all cases
2b 8b 10b 15b 24b 1
from (1), b 98 98
3 5 15 10
D a C
49b = 98 × 15 b = 30
55. (2) Given: h =1.4cm, r = 0.6 cm
Volume of the material removed
= Volume of cylinder ABCD a
61. (3) a
2a

– volume of conical shape APB


2 1 2 2 2h
= r h– r h = r
3 3
2
A a B
6 2 14 Let ‘a’ be the side of the square ABCD.
= 1.056
10 3 10 then Area = a2
56. (1) Given line can be written as y = x + 2
Slope of given line is 1 Diagonal AC = a 2 a2 2a
since required line is to given line Area of square drawn on diagonal ( 2a) 2 2a 2
Slope of required line is – 1
The equation of line passing through (– 1, 2) having a2 1
Hence, Required Ratio 2
slope – 1 is y – 2 = – 1 (x + 1) 2a 2
x+y=1
Exemplar Practice Questions 57
S-

62. (1) In cyclic quadrilateral opposite angles add upto 180°. 67. (3) Let cost of 1 bat = ` x and
A + C = 180° and B + D = 180° cost of 1 ball = ` y
According to the question, we have
D C
7x + 6y = 3800
3x + 5y = 1750
These equations can be re-written as
A B
21x + 18y = 11400
So, from options, only option (1) satisfies the given 21x + 35y = 12250
condition Now, on subtracting we get
A = 70°, B = 120°, C = 110°, D = 60° y = 50 and x = 500
1 1 Hence, cost of 1 bat = ` 500 and
63. (1) Let x a b and x a b cost of 1 ball = ` 50
x x
By adding both the equations, we get 68. (1)
x 2 x4 1 x3 2x 2 4x 8
1 1 x 4 2x3
x x a b a b
x x (–) (+)
2x = 2a x = a
Thus, given eqns. becomes 2x3 1
1 1 2x 3 4x 2
a a b and a a b (–) (+)
a a
on multiplying both, we get 4x 2 1
1 1 4x 2 8x
a a (a b)(a b) (–) (+)
a a
1 8x 1
a2 a 2 b 2 a 2b 2 1 ab 1 8x 16
a2 (–) (+)
64. (2) Fifth term of a G.P is 2 17
ar4 = 2; Hence, Remainder = 17
Product required 2
4.5
= a × ar × ar2 × ........ × ar8 10
2
= a9 r36 69. (3) Required number of coins 2
1.5 2
= (ar4)9 = (2)9 = 512
2 10
b c 2 2 (b2 2ac )
65. (4) , and 45 45 10 20 20 10
a a a 2 = 9 × 10 × 5 = 450
20 20 15 15 2
(a b) (a b) 1 1 1 1
Now 70. (1) Let
b a b (a b ) ( a b) a b c a b c
(b2 2ac) b where a + b + c 0 and abc 0.
2 2 a 2
b bc ac ab 1
a( ) b( ) a a
= abc a b c
a2 ab ( ) b2 c 2
a ab
b
b2
a a (a + b + c) (bc + ac + ab) = abc
abc + a2c + a2b + b2c + abc + ab2 + bc2 + ac2 + abc
b 2 2ac b 2 2ac 2
= = abc
a 2 c ab 2 ab 2 a2c a a2c + a2b + b2c + b2a + bc2 + ac2 + 3abc = abc
a2c + a2b + b2c + b2a + bc2 + ac2 + 2abc = 0
abc + a2b + ac2 + a2c + b2c + b2a + bc2 + abc = 0
(ab + ac + b2 + bc) (c + a) = 0
66. (3) O (a + b) (b + c) (c + a) = 0
r r 71. (3) Given equation is
A r B x2 + p2 = (q – x)2
x2 + p2 = q2 + x2 – 2qx
Let O be the centre and r be the radius of the circle.
p2 = q2 – 2qx 2qx = q2 – p2
Let AB be the chord. According to the question
AB = OA = OB = r q2 p2
Now, OAB is an equilateral triangle. x
2q
AOB = 60°
EBD_7332
58
S- NTSE Stage 1 Question Bank

72. (1) The lights will change simultaneously after every 77. (2) x2 + 7ax + 40 and x2 + 2ax – 60 have a common factor.
interval = (L.C.M of 36, 42 and 72) seconds. Solving using cross multiplication method :
2 36, 42, 72 x2 x
2 18, 21, 36 (7 a )( 60) (2a)(40) (40)(1) ( 60)(1)
3 9, 21, 18
3 3, 7, 6 1
2 1, 7, 2 (2a)(1) (7)(1)
7 1, 7, 1 x2 x 1
1, 1, 1 x 10 a 2
500a 100 5a
LCM = 2 × 2 × 3 × 3 ×2 × 7 = 504
504 seconds = 8 minute and 4 seconds 84 x 10
78. (1) 642 x 5
4 8x 5
4
The next change will be at 9 : 08 : 04. 8x 5

A 2 2 B 83 x 5
4 29 x 15
22
F 9 x 17 x 17 / 9
3
79. (2) c a 3z

73. (3) O 3z
c (b3 x )3 z ( c3 y ) 3 x c1 c27 xyz
3
E 1
C 5/2 5/2 D xyz
27
Let AB and CD be two parallel chords. 1 1 2
Radius = 3 cm 80. (3)
1 x 1 x 1 x2
Now, from OFB,
OF2 = 9 – 4 = 5 OF = 5 2 2 4
From OEC, 1 x2 1 x2 1 x4
25 11 11 4 4 8
OE 2 9 OE
4 4 2 1 x4 1 x4 1 x8
So, Distance between AB and CD
8 8 16
11 1 x8 1 x8 1 x16
OF OE 5
2
74. (4) Consider (256)0.16 × (256)0.09 81. (3) x4 2 x2 y 2 y4 x2 y 2 (x2 y 2 )2 x2 y 2
= (256)0.16 + 0.09 = (256)0.25 [( x y ) 2 2 xy ]2 x2 y2 [(8) 2 2 15]2 (15)2 931
= (44)0.25 = (4)1 = 4
82. (1) ( x2 x 1)2 ( x 2 x 1)2
2 1
75. (2) x 1 5x x 5
x x4 x2 1 2 x2 2x 2x2 x4 x2 1 2x3 2 x 2 x2

1
2 3x 2 3 x 2 6 x 2
x– = 52 Coefficient of x2 = 6
x 83. (1) Time taken to plough the farm = 50 hrs.
1 Required number of days = (500 / 25) = 20 days.
x2 + = 25 + 2
x2 84. (3) (a 2 b 2 c 2 )2 a4 b4 c4 2(a 2 b2 b2 c 2 c2 a2 )
= 27
[(a b c) 2 2(ab bc ca)]2
1/2 1/3
6 3 1 2
x y x y
76. (1) x a yb a 4 b 4 c 4 2(a 2b2 b2 c2 c 2 a 2 )
x 2 y3 x3 y 2
4[(a 2b2 b2 c 2 c 2 a 2 ) 2abc (a b c)]
( x8 y 6 ) 1/2
( x 4 y4 ) 1/3
x a yb
a 4 b 4 c 4 2(a 2b2 b2 c2 c 2 a 2 )
(x 4 y3 ) a b
x y
( x 4/3 )( y 4/3 ) 2(a 2b2 b2 c2 c 2 a 2 ) a 4 b4 c 4
16/3 13/3
85. (1) x = –460 + 440 = – 20
x y xa yb y = 110 – 100 = 10
16 13 y – x = 10 – (–20) = 30
a b 1 1 1 1 0
3 3
Exemplar Practice Questions S-59
97. (3) Since x3 + y3 + z3 – 3xyz = (x + y + z) (x2 + y2 + z2 – xy –
x y z
86. (3) Let a ;b ;c yz – zx)
y z x
x3 + y3 + z3 + 6 = 2 (x2 + y2 + z2 + 1)
x3 + y3 + z3 = 2 (x2 + y2 + z2) – 4 ...... (1)
1 1 1
x z y x z y Now (x + y + z)2 = x2 + y2 + z2 + 2(xy + yz + zx)
1 1 1 (2)2 = x2 + y2 + z2 –2
y y z z x x
x2 + y2 + z2 = 6 ...... (2)
y z x putting value of eqn (2) in eq (1) we have
y x z z y x x z y x3 + y3 + z3 = 2(6) – 4
= 12 – 4 = 8
y z x 98. (4) In triangle ADC BE | | CD
1
y x z Using proportionality theorem
87. (3) 1 – 9 one key 9 × 1 = 9 y AB x
10 – 99 two keys 90 × 2 = 180 = = ...... (1)
y + 3 AC 4.5
100 – 180 3 keys 81 × 3 = 243
Similarly in triangle AFC
number of times key pressed is 9 + 180 + 243
x BC AB
88. (1) x3 27 9 x 2 27 x 12 9 x 27 = =1–
7.5 AC AC
x 3 9 x 2 18 x 12 0
AB x
89. (3) x 2 x 99 x 33 [ exterior = sum of two = 1– ...... (2)
AC 7.5
interior angles of a ]
6 2x 1
From eqn (1) & (2)
5 5 7
90. (4) 2x 7 x x x
3 3 2 = 1–
4 5 7 5
91. (3) 0.2 0.23 0.45
5 45 13
92. (2) t 15 t t 60 min. on solving x = = 2 cm
4 16 16
93. (1) f ( 3) f ( 2) 0 m 9, n 6. Now from (1)
94. (4) Let the CP be Re 1/dozen y x
for one apple CP = 1/12 =
y + 3 4.5
As he sold apples at Rs 8 per 100.
So SP for one apple = 8/100 = 2/25 Substituting the value of x & on solving

1 –2 1 875
12 25 ×100 = 4% y= = 5cm .
Now less% = 1 375
12 99. (3) Since xa = yb = zc
95. (1) Area of XYZ = 1 2 XZ × XP = 1 2 × 65 × XP x = yb/a and z = yb/c
...(i) Now y2 = zx
Also according to hero's formula y2 = yb/c yb/a
Area of XYZ = S(S – a) S – b S – c b b 2 1 1
2 + = +
c a b c a
63 +16 + 65
Now S = 72 100. (2) 2a+3 = 4a+2 – 48
2
23.2a = 22a . 24– 48 ...... (1)
Area of XYZ = 72 a 56 7 ...(2) Let 2a = x, eqn (1) becomes
From (1) and (2) 8x = 16x2 – 48 16x2 – 8x – 48 = 0
1 65× XP = 504 XP = 15.5 units 2x2 – x – 6 = 0
2
(x – 2) (2x + 3) = 0
96. (1) 9x-2 = 3x+1
32x-4 = 3x+1 2x – 4 = x + 1 x = 3/2, 2
x=5 2a = 3/2, 2 possible value = 2
21 + x = 26 = 64 2a = 21 = 1 a = 1
EBD_7332
60
S- NTSE Stage 1 Question Bank

SECTION 5. HISTORY 12. (3) Printing was first developed in China. The earliest form
of printing was woodblock printing, which existed in
1. (1) A ghetto is a part of a city in which members of a minority China before 220 A.D. and Egypt in the fourth century.
group live, especially because of social, legal, or Later the developments in printing include the movable
economic pressure. The term was originally used in type, first developed by Bi Sheng in China.
Venice to describe the part of a city to which Jews were 13. (3) Edo was the ancient name of Tokyo. When Emperor
restricted and segregated. Meiji moved Japan's capital from Kyoto to Edo, he
2. (2) The people living in Balkan region where known as named it Tokyo. Before Kyoto, Nara was the ancient
capital of Japan.
Slavs.
14. (3) The first Indian cotton mill, "The Bombay Spinning
3. (2) The city of Bombay originally consisted of seven Mill", was opened in 1854 in Bombay by Cowasji
islands, namely Colaba, Mazagaon, Old Womans Island, Nanabhai Davar.
Wadala, Mahim, Parel, and Matunga-Sion. It was 15. (4) 'Raikas' the Pastoral community lives in Rajasthan.
occupied by the Portuguese initially. They named their Raikas combined cultivation with pastoralism.
new possession as "Bom Baia" which in Portuguese 16. (3) Jamil was the son in law of Swami Mahavir.
means "Good Bay". A hundred and twenty-eight years 17. (1) Gautama Buddha, founder of Buddhism is considered
later the islands were given to the English King Charles 'The Light of Asia'.
18. (3) Rudra is a Rig vedic deity, associated with wind or
II in dowry on his marriage to Portuguese Princess
storm. It is also the other name for Shiva or Mahesh.
Catherine of Braganza in 1662. 19. (3) Muhammad was Born in 570 CE in the Arabian city of
4. (1) Godaan was written by Munshi Premchand, and was Mecca. He was a religious, political, and military leader
first published in 1936. The story revolves around the who unified Arabia into a single religious polity under
socio-economic deprivation as well as the exploitation Islam.
of poor peasants. 20. (4) The Divine Comedy is an epic poem written by Dante
Alighieri. It is considered one of the prominent works
5. (1) The Samburu National Reserve is located on the banks of Italian literature.
of the Ewaso Ng'iro river in Kenya. It is 165 km2 in size 21. (1) James I
and 350 kilometers from Nairobi and ranges in altitude 22. (4) The Republic is written by Plato. It is the longest of his
from 800 to 1230m above sea level. works with the exception of the Laws, and is certainly
the greatest of them.
6. (1) By 1921, Gandhiji had designed the Swaraj flag. It was
23. (1) John Emerich Edward Dalberg Acton, (1834-1902). The
again a tricolour (red, green and white) and had a
historian and moralist, who was otherwise known
spinning wheel in the centre, representing the Gandhian simply as Lord Acton, expressed this opinion in a letter
ideal of self-help. Carrying the flag, holding it aloft, to Bishop Mandell Creighton in 1887:
during marches became a symbol of defiance. "Power tends to corrupt, and absolute power corrupts
7. (4) It's amazing the amount of food the New World absolutely. Great men are almost always bad men."
24. (4) After the revolt of 1857 and as a consequence of
provided the Old. Corn, tomatoes, potatoes, peppers,
Government of India Act 1858, the British Government
chilies, groundnuts, maize, different types of beans,
assumed the task of directly administering India in the
sweet potatoes, chocolate, peanuts, and many other new British Raj. So India's rule was given to the British
things & were introduced by the travellers and Crown.
explorers. 25. (2) Dhundiraj Govind Phalke, popularly known as
8. (4) A merchant clothier in England purchased wool from a Dadasaheb Phalke (30 April 1870 - 16 February 1944)
wool stapler, and carried it to the spinners; the yarn was an Indian producer-director-screenwriter, known
(thread) that was spun was taken in subsequent stages as the father of Indian cinema. Starting with his debut
of production to weavers, fullers, and then to dyers. film, Raja Harishchandra in 1913, now known as India's
first full-length feature, he made 95 movies and 26 short
9. (3) Hind Swaraj also known as Indian Home Rule was
films in his career spanning 19 years.
written by Mahatma Gandhi in the year 1909. This book
expresses his view on swaraj, modern civilization, 26. (3) The guillotine is a device designed for carrying out
mechanization etc. executions by decapitation. It consists of a tall upright
frame in which a weighted and angled blade is raised to
10. (1) On 14th July 1789 French Revolution took place. It led
the top and suspended. The condemned person is
to the end of monarchy in France and also affected the
secured at the bottom of the frame, with his or her neck
whole of Europe. A society based on privileges gave
way to a new system of governance. held directly below the blade. The blade is then released,
to fall swiftly and sheared the head from the body.
11. (4)
Exemplar Practice Questions S-61

27. (3) The first Indian cricket club was founded by the Parsis 40. (4) 'Indian Penal Code' was prepared during the time of
in Mumbai in 1848 by the name of 'oriental cricket club'. Lord Canning.
28. (2) Indian National Congress was established in the year 41. (1) The Revolutionary movement for Indian independence
1885 by Allan Octavian Hume in Bombay. comprised the actions of the underground
revolutionary factions. Groups believing in armed
29. (4) Mahatma means a great soul, is often taken to be
revolution against the ruling British fall into this
Gandhi’s given name in the West. Poet and Nobel
category, as opposed to the generally peaceful civil
laureate Rabindranath Tagore bestowed the title on
disobedience movement spearheaded by Mohandas
Gandhi in 1915 while writing his autobiography.
Karamchand Gandhi. The revolutionary groups were
30. (4) Gopal Krishna Gokhale was famously a mentor to
mainly concentrated in Bengal, Bihar, the United
Mahatma Gandhi in his formative years. In 1912,
Provinces and Punjab. The revolutionary philosophies
Gokhale visited South Africa at Gandhi’s invitation. As
and movement made its presence felt during the 1905
a young barrister, Gandhi returned from his struggles
Partition of Bengal.
against the Empire in South Africa and received
42. (1) Montesquieu, was a French social commentator and
personal guidance from Gokhale.
political thinker who lived during the Age of
31. (3) The First Indian National Army was formed under
Enlightenment. He is famous for his articulation of the
Mohan Singh in 1942 and received considerable
theory of separation of powers, which is implemented
Japanese aid and support. In a series of meetings
in many constitutions throughout the world.
between the INA leaders and the Japanese in 1943, it
43. (4) America-Iraq war was a controversial event that began
was decided to cede the leadership of the IIL and the
after the end of Cold War( started in 1945 and ended in
INA to Subhas Chandra Bose.
1991) . Partition of Germany, Korea and Indo-China War
32. (3) Kunwar Singh at the age of 80 years, during India's all occurred during the period of Cold War.
First War of Independence (1857-58), he actively led a 44. (3) On August 4, 1789, a body known as the National
select band of armed soldiers against the troops under Assembly announced the end of feudalism in France.
the command of the British Raj, and also recorded This stopped the Church from collecting taxes and most
victories in many battles. importantly, stopped the forced labor from the poor
33. (3) ‘Das Capital’ was written by Karl Marx. It is basically a peasants. This also allowed positions in Church,
critical analysis of political economy, meant to reveal government, and army to be open to all citizens of
the economic laws of the capitalist mode of production. France.
34. (3) The first convention of Facist party was held at Milan. 45. (3) In 1840, the First Opium War broke out between the
35. (1) Harappa is located in the Punjab region of eastern English and the Chinese. The military weakness of
Pakistan, and was part of the Indus river civilizations China led to their defeat. By the Treaty of Nanking,
around 2600 BC. China gave the Island of Hong Kong and a war
indemnity to Britain. She opened five ports for the
36. (2) Shri Chaitanya Charitamrita is the biography of Lord English trade. The Treaty of Nanking opened doors to
Chaitanya Mahaprabhu composed by Shrila Britain in China. The Second Opium War forced China
Krishnadas Kaviraja Goswami. to legalize the opium trade and also to open more ports
for foreign trade.
37. (2) The Bengal famine of 1770 was a catastrophic famine
46. (3) Bangladesh came into existence in1947 during the
between 1769 and 1773 that affected the lower Gangetic partition of India and it became independent in 1971.
plain of India. The famine is estimated to have caused Burma became independent in 1948. South Rhodesia
the deaths of 10 million people, reducing the population became independent in 1950 and today it is Zimbabwe.
to thirty million in Bengal, which included Bihar and Vietnam came into existence in 1945 but it became a
parts of Odisha. unified independent country in 1976.
47. (1) In 1920 the League of nations settled the boundary
38. (2) Vinayak Damodar Savarkar was one of the most
dispute over the ownership of Islands in the Baltic.
dynamic, outspoken and revolutionary leaders of the
48. (2) Means of production are owned by the capitalist
freedom struggle. He renamed the "Mitra Mela" as entrepreneurs.
"Abhinav Bharat" and declared "India must be 49. (1)
independent". 50. (4) On January 22, 1905, the day after a general strike
39. (4) Hindustan Socialist Republican Association (HSRA) burst out in St. Petersburg, Father Gapon organized a
was a revolutionary organisation, also known as workers' procession to present a petition to the Tsar,
Hindustan Socialist Republican Army established in which ended tragically (Bloody Sunday 1905). Gapon's
1928 at Feroz Shah Kotla New Delhi by Chandrasekhar life was saved by Pinchas Rutenberg, who took him
Azad, Bhagat Singh, Sukhdev, Rajguru and others. Lala away from the gunfire. He then became the guest of
Maxim Gorky.
Lajpat Rai was not a member of this association.
EBD_7332
62
S- NTSE Stage 1 Question Bank

51. (4) These are the religious wars fought between Christians Executive Head of the World Zionist Organization and
and Muslims. The Crusades had major political, president of the Jewish Agency for Palestine, declared
economic, and social impacts on Western Europe. It "the establishment of a Jewish state in Eretz Israel, to
resulted in a substantial weakening of the Byzantine be known as the State of Israel," a state independent
Empire, which fell several centuries later to the Ottoman upon the termination of the British Mandate for
Empire. Palestine, 15 May 1948.
52. (1) Renaissance took place in western countries between 60. (2) Sri. S. Nijalingappa, the chief minister of the state of
500 - 1500 AD. It is essentially an intellectual movement. Mysore named it as Karnataka.
It is meant that new enthusiasm for classical literature, 61. (1) In 1854, Sir Charles Wood in his Despatch on
learning, and art which sprang up in Western countries Educational Reforms led to the establishment of first
towards the close of the Middle Ages, and which during
three universities in India at Bombay, Calcutta and
the course of the 500 - 1500 centuries gave a new culture
Madras.
to Europe.
62. (4) Satyashodhak Samaj was formed by Jyotibha Phule,
53. (4) There were four guiding principles for the Congress of
Vienna. They included restoring the balance of power, Satyartha Prakash was formed by Dayanand
the containment of France, the restoration of rightful Saraswathi, S.N.D.T.University was established by
rulers, and rewarding & punishing people involved in Dhondho Keshava Karve, Poona Sarvajanik Sabha was
the Napoleonic Wars. Principles & feelings of formed by Mahadeva Govind Ranade.
Nationality was not there. 63. (2) Hicky's Bengal Gazette was an English newspaper
54. (3) Bismarck had a short career in the civil service for published from Kolkata (then Calcutta), India. It was
"deficiency in regularity and discipline"; the first major newspaper in India, started in 1780. It
In 1847 Bismarck started his political career as a member was published for two years.
of the Prussian Diet in Berlin. There he opposed every 64. (3) 65. (2)
liberal proposal and made a name for himself as a 66. (3) 67. (4)
determined reactionary. 68. (3) 69. (1)
55. (4) The immediate cause for the First World War was the 70. (1) 71. (3)
assassination of the crown prince of Austria i.e. 72. (2) 73. (3)
Archduke Franz Ferdinand and his wife on 28 June 74. (3) 75. (2)
1914. Rests of the options are other important causes
76. (1) 77. (3)
but not the immediate cause.
78. (4) 79. (4)
56. (3) Jai Hind is a salutation, slogan and battle cry most
commonly used in India in speeches and 80. (2) 81. (1)
communications pertaining to or referring to patriotism 82. (2) 83. (2)
towards India (also known as Hind). It translates 84. (3) 85. (2)
roughly to "Hail India" or "Victory to India" or "Long 86. (3) 87. (2)
live India". The term was claimed to be coined by 88. (1) 89. (3)
Chempakaraman Pillai, of Indian Independence 90. (3) 91. (4)
Movement and Hindu-German Conspiracy. But 92. (1) 93. (4)
researchers had stated that it was first coined by Major 94. (2) 95. (3)
Abid Hasan Safrani of the Indian National Army as a 96. (2) 97. (1)
shortened version of Jai Hindustan Ki (translation: 98. (1) 99. (1)
Victory to India). It has since captured the imagination 100. (1)
of Indians and has been immortalized by Subhas
Chandra Bose as the battle cry of the Indian National SECTION 6. CIVICS
Army (Azad Hind Fauj).
57. (3) Dewan Seshadri Iyer. Shivanasamudra is a small town 1. (1) The constitution of India declares India to be a Union
in the Mandya District of the state of Karnataka, of States.
situated on the banks of the river Kaveri. It was the 2. (1) Article 370 of the Indian constitution is a law that grants
first hydro-electric power station in Asia, which was
special autonomous status to Jammu and Kashmir.
set up in the year 1902. The construction of this project
received anger from Tamil Nadu leaders as it was 3. (1) A coalition government is a type of government where
constructed at a time when both these states were two or more political parties join together in parliament/
having a Kavery river water dispute. This dispute has assembly to run the government and agree on a policy
remained prevalent in the modern day.
programme.
58. (2) The dictatorship of the proletariat as the only solution
for all feudal problems was advocated by Karl Marx. 4. (2) Communist Party of India Marxist has remained in
59. (2) On 29 November 1947, the United Nations General power in West Bengal for 30 years and Jyoti Basu as
Assembly recommended th e adoption and the longest serving Chief Minister of any State of India.
implementation of the partition plan of Mandatory
Palestine. On 14 May 1948, David Ben-Gurion, the 5. (1) Dr. BR Ambedkar was the Chairperson of the Drafting
Committee of the Constituent Assembly of India.
Exemplar Practice Questions S-63

6. (3) There are six Fundamental Rights in the Indian 16. (2) Article 31-B was inserted by the First Constitutional
Constitution. They are as follows: 1. Right to Equality, (Amendment) Act 1951. Article 31-B of the Constitution
2. Right to Freedom, 3. Right against Exploitation, of India ensured that any law in the Ninth Schedule
could not be challenged in courts and Government can
4. Right to Freedom of Religion, 5. Cultural and rationalize its programme of social engineering by
Educational Rights, 6. Right to Constitutional reforming land and agrarian laws.
Remedies. 17. (4) Sikkim became the 22nd Indian State on April 26, 1975.
7. (1) 'Secular' means in the Indian Context that there is no On May 16, 1975, Sikkim officially became a state of the
Indian Union and Lhendup Dorji became head of State
official religion for the Indian state. (chief minister).
8. (3) Reservations of seats for women in local elections have 18. (2) President is the first person in India.
been perhaps most significant political step taken by 19. (4)
independent India to provide overdue political spaces 20. (1) The Eighth Schedule to the Indian constitution lists 22
for women. It also helped to give local development a languages that the Government of India has the
much desired direction. responsibility to develop.
9. (3) The permanent members of the United Nations Security 21. (4) China has adopted the one party system.
Council, include the following five governments: 22. (1) Disputes related to the election of the President of India
China, France, Russia, the United Kingdom, and the can be adjudicated in Supreme Court.
United States. There have been proposals suggesting 23. (3) Bryce believes that “Democracy really means nothing
the introduction of new permanent members. The more or less than the rule of the whole people,
candidates usually mentioned are Brazil, Germany, expressing their Sovereign will by their votes”.
Dicey believes, “Democracy is a form of government
India, and Japan. They comprise the group of G4
in which the governing body is a comparatively large
nations, mutually supporting one another's bids for fraction of the entire nation”.
permanent seats. 24. (2) In1993 the Protection of Human Rights Act was enacted
10. (3) The Lok Sabha also known as House of the People is in India.
the lower house of the Parliament of India. Members of 25. (2) This law was passed by Parliament on 15 June 2005
the Lok Sabha are elected by direct election under and came fully into force on 12 October 2005. The Act
universal adult franchise. It is the people of the country has increased transparency and greater accountability
in the functioning of the government and hence played
who are directly involve in this election. Each Lok Sabha
a significant role in exposing and reducing corruption
is formed for a five-year term, after which it is
to some extent.
automatically dissolved.
26. (1) In the year 1988 the United Nations Peace Keeping
11. (2) Gandhiji used to say that religion can never be Forces awarded Nobel Peace Prize. The Nobel
separated from politics. What he meant by religion was Committee awarded the pr ize because “Th e
not any particular religion like Hinduism or Islam but peacekeeping forces of the United Nations have, under
moral values that inform all religions. He believed that extremely difficult conditions, contributed to reducing
politics must be guided by ethics drawn from religion. tensions where an armistice has been negotiated but a
12. (3) Home minister is a part of Political Executive. The peace treaty has yet to be established”.
Ministry of Home Affairs (MHA) or Home Ministry is 27. (3) It is the Governor of the state who calls the head of the
a ministry of the Government of India. It is mainly party with the majority in the election to form the
responsible for the maintenance of internal security government. So we can say the Governor of the state
and domestic policy. The Home Ministry is headed by appoints the chief minister.
Union Minister of Home Affairs.
28. (4) The Right to Property lost its place among the
13. (2) The system of apartheid -- literally, "apartness" --
Fundamental Rights and became just a statutory right
enforced the separation of South Africa's whites from
in the year 1978.
the blacks, Asians, and people of mixed race. This
discriminatory system was carried out through political, 29. (2) 'Politics' is written by Aristotle in 350 B.C.E. It has been
economic, and social institutions and continued translated by Benjamin Jowett.
through the early 1990s.
30. (2) The United Nations Security Council (UNSC) is one of
14. (2) Universal Adult Franchise means the right to vote to the six principal organs of the United Nations. There
every adult of the country. The right to vote is not are 15 members of the Security Council. This includes
restricted by race, sex, belief, wealth, or social status. five veto-wielding permanent members-China, France,
In India it was granted in 1950. Russia, the United Kingdom, and the United States-
15. (4) Tamil Nadu has more than 30 Lok Sabha constituencies. based on the great powers that were the victors of
World War II.
EBD_7332
64
S- NTSE Stage 1 Question Bank

31. (1) Popular sovereignty in its modern sense, that is, 44. (1) The Commonwealth of Nations is an intergovernmental
including all the people and not just noblemen, is an organisation of 54 member states that were mostly
idea that dates to the social contracts school (mid-17th territories of the British Empire. The Commonwealth
operates by intergovernmental consensus of the
to mid-18th centuries), represented by Thomas Hobbes member states, organised through the Commonwealth
(1588-1679), John Locke (1632-1704), and Jean-Jacques Secretariat, and non-governmental organisations,
Rousseau (1712-1778). organised through the Commonwealth Foundation.
32. (4) It has never been declared. But it had gone up to that 45. (1) The second gulf war was fought between Iraq and
condition once. They had then deposited gold. U.S.A.
33. (1) To declare a financial emergency is one of the financial 46. (4) The International Court of Justice is composed of 15
powers of the President. judges elected to nine-year terms of office by the United
Nations General Assembly and the Security Council.
34. (1) In Indian law, Article 32 of the Indian constitution These organs vote simultaneously but separately. In
contains a tool which directly joints the public with order to be elected, a candidate must receive an absolute
judiciary. PIL may be introduced in a court of law by majority of the votes in both bodies. This sometimes
the court itself (suo motu), rather than the aggrieved makes it necessary for a number of rounds of voting to
party or another third party. For the exercise of the be carried out.
court's jurisdiction, it is unnecessary for the victim of 47. (1) 48. (1)
the violation of his or her rights to personally approach 49. (1) 50. (2)
51. (3) 52. (3)
the court. 53. (1) 54. (3)
35. (2) Municipal councils make ward committees when the 55. (4) 56. (1)
population is more than three lakhs. 57. (4) 58. (3)
36. (3) 70% share of land revenue is given to Zilla Parishad by 59. (4) 60. (1)
the district. 61. (4) 62. (2)
37. (2) The speaker of Lok Sabha is not the member of union 63. (4) 64. (4)
executive. 65. (4) 66. (2)
38. (1) The characterization of India as 'Sovereign Democratic 67. (3) 68. (1)
69. (2) 70. (3)
Republic' has been changed to 'Sovereign Socialist
71. (1) 72. (1)
Secular Democratic Republic'. The words 'Unity of the 73. (1) 74. (3)
nation' have been changed to 'Unity and integrity of 75. (1) 76. (4)
the nation'. The rest ideals were in constitution from its 77. (4) 78. (2)
force i.e. 26 Jan 1950. 79. (2) 80. (2)
39. (2) The population of a village is between 500 to 1500. The 81. (3) 82. (4)
number of members to be elected to the gram Panchayat 83. (3) 84. (3)
is 7. 85. (1) 86. (1)
40. (1) The third world represents a large number of newly 87. (4) 88. (4)
independent and developing nations. The term Third 89. (1) 90. (1)
World arose during the Cold War to define countries 91. (2) 92. (4)
that remained non-aligned with either NATO or the 93. (2)
Communist Block. 94. (2) The “Indian Constitution” does not invoke God
41. (4) The Seventeenth Summit was held from 10-11 of whereas the “Constitutions of America and South
November 2011 in Addu City, Maldives. The Meeting, Africa” seek the blessings of God in the making of
their Constitution.
which was held at the Equatorial Convention Centre,
95. (4)
Addu City was opened by the outgoing Chair of
96. (2) Dr. B. R. Ambedkar often bitterly criticised Mahatma
SAARC, Prime Minister of the Royal Government of
Gandhi regarding the elimination of inequality in India
Bhutan, H.E.Lyonchhen Jigmi Yoezer Thinley. H.E. and reservation of Scheduled Castes and Tribes.
Mohamed Nasheed was elected as the Chairperson of
97. (2) 98. (4)
the 17th SAARC Summit. In his inaugural address
99. (1) 100. (2)
President Nasheed highlighted three areas of
cooperation in which progress should be made; trade,
transport and economic integration; security issues SECTION 7. ECONOMICS
such piracy and climate change; and good governance. 1. (2) SAIL is a public sector company and is the largest
42. (3) 14 July 2011 - The General Assembly admitted the integrated iron and steel producer in India. It is
Republic of South Sudan as the 193rd member of the responsible for the marketing of steel for the public
United Nations, welcoming the newly independent sectors in India.
country to the community of nations. South Sudan's 2. (3) Death rates have declined from 25 per 1000 population in
independence from the rest of Sudan is the result of 1951 to 8.1 per 1000 in 2001 and life expectancy at birth has
the January 2011 referendum held under the terms of increased from 36.7 years in 1951 to 64.6 years in 2001.
the 2005 Comprehensive Peace Agreement (CPA) that The substantial improvement is the result of many factors
ended the decades-long civil war between the North including improvement in public health, prevention of
and the South. infectious diseases and application of modern medical
43. (3) practices in diagnosis and treatment of ailments.
Exemplar Practice Questions S-65
3. (4) 17. (4) 18. (2)
4. (3) Life Insurance Corporation of India (LIC) is the largest 19. (3) 20. (4)
insurance group and investment company in India. 21. (3) 22. (4)
Government of India has 100% stake in it. It was 23. (2) 24. (2)
founded in 1956 with the merger of 245 insurance 25. (4) 26. (3)
companies and provident societies. It's headquarter is 27. (1) 28. (2)
in Mumbai. 29. (1) 30. (4)
5. (3) The National Rural Employment Guarantee Act 31. (3) 32. (1)
(NREGA), implemented by the Government during the 33. (1) 34. (3)
year 2005, is an illustrious programme that reaches the 35. (2) 36. (4)
37. (4) 38. (3)
nucleus of the indispensable requisites of the poor,
39. (2) 40. (2)
and augments to their dire needs by creating and
41. (1) 42. (1)
providing employment even for the elders of the family. 43. (4) 44. (2)
The hand - to - mouth existence is thus eradicated, 45. (2) 46. (2)
paving way for an enhanced future. 47. (4) 48. (4)
6. (1) National Consumer Day is celebrated on 24 December. 49. (1) 50. (3)
7. (2) Due to the fall of the Soviet Union and the problems in 51. (4) 52. (4)
balance of payment accounts, the country faced 53. (4) 54. (4)
economic crisis and the IMF asked for the bailout loan. 55. (4) 56. (3)
To get out of the situation, the then Finance Minister, 57. (2) 58. (4)
Manmohan Singh initiated the economic liberation 59. (2) 60. (3)
reform in the year 1991. This is considered to be one of 61. (4) 62. (2)
the milestones in India economic reform as it changed 63. (4) 64. (4)
65. (1) 66. (3)
the market and financial scenario of the country. Under
67. (4) 68. (4)
the liberalization program, foreign direct investment was
69. (4) 70. (2)
encouraged, public monopolies were stopped, and
71. (4) 72. (3)
service and tertiary sectors were developed.
73. (4) 74. (1)
8. (2) The contribution from agriculture has been
75. (3) 76. (1)
continuously falling from 55.1% in 1950-51 to 37.6% in
77. (4) The farmers sell the manufactured articles in market
1981-82 & further to 18.5% in 2006-07. But agriculture
which brings them additional income.
still continues to be the main sector because it provides
78. (4) The rural population shift to urban areas due to rising
livelihood to a majority of the people.
population in rural areas, in search of better job
9. (3) The natural resources used in production are renewable opportunities and due to lack of demand of labour in
and non-renewable resources. agriculture.
79. (4) The use of advanced technology was initiated with
10. (4) The term 'Globalisation' refers to integration among the
crops of wheat, and later, rice. Punjab, Haryana, Uttar
economy of different countries, decreasing government Pradesh (Western) are important wheat-growing areas,
control over different sectors and free trade among the and the Green Revolution has led to maximum benefits
different countries. to these areas.
11. (2) Increase in the share of agriculture. 80. (2) The rural women lack education and the necessary
12. (4) Keeping in view the consensus on increasing the skills. They are thus willing to work for lowpaid work
allocation of food grains to BPL families, and to better which is available to them.
target the food subsidy, Government of India increased 81. (1) Agriculture by the nature nature of occupation absorbs
the allocation to BPL families from 10 kg. to 20 kg of the maximum labour in India.
food grains per family per month at 50% of the economic 82. (3) To reduce infant mortality it is essential to reduce risk
cost and allocation to APL families at economic cost. of infection among infants. It also involves ensuring
13. (2) The term of the Third Five Year Plan was from 1960 to nutrition along with care for mother as well as child.
1965 in between which these two wars broke out the 83. (2) 84. (2)
devastated the Indian economy and led to the failure 85. (4) 86. (2)
of this plan. 87. (1) 88. (1)
14. (2) Right to consumer's safety 89. (2) 90. (4)
15. (4) These schemes are associated with mid day meals 91. (4) 92. (4)
provided in government aided schools for poor 93. (4) 94. (2)
children. 95. (3) 96. (4)
16. (3) In 1986 universalisation of primary education was 97. (3) 98. (1)
included in national policy of education. 99. (2) 100. (3)
EBD_7332
66
S- NTSE Stage 1 Question Bank

SECTION 8. GEOGRAPHY 13. (4) 29% of Earth is covered by land. 71% of the earth's
surface is covered with water. And most of it is covered
1. (2) Tajikistan is the only country which does not share its by oceans.
boundary with India. Four countries share land 14. (4) Australia is the only continent without any active
boundary with Tajikistan, they are Afghanistan, China, volcanoes.
Kyrgyzstan and Uzbekistan. 15. (3) Switzerland is one of the beautiful countries. It is
2. (1) 97° 25 E is the Eastern most longitude of India. located in the Central Europe. The borders of
Switzerland touch Germany to the north, France to the
3. (1) Energy generated from tides is called Tidal energy. It is
west, Austria to the east and Italy to the south.
a non-conventional source of energy and replenishable Mountains of the Switzerland are very popular, Alps in
or renewable as long as long as we have the ocean south and Jura in northwest. It has a central plateau of
water and the moon. rolling hills, plains and large lakes. So, Switzerland is
4. (4) Intensive cultivation enhanced by excessive use of the playground of Europe.
chemical fertilizers has caused land degradation in 16. (2) Tripura is a state in North East India. The third-smallest
Punjab. Also irrational use of surface and ground water state in the country, it covers 10,491 km2 (4,051 sq mi) and
for irrigation. is bordered by Bangladesh to the north, south, and west,
and the Indian states of Assam and Mizoram to the east.
5. (2) Gas pipeline from Hazira in Gujarat connects Jagdishpur
It does not have common boundary with Myanmar.
in Uttar Pradesh, via Vijaipur in Madhya Pradesh. It 17. (1) Indian sub-continent includes Bangladesh, Bhutan,
has branches to Kota in Rajasthan, Shahajahanpur, India, Nepal, Pakistan, and Sri Lanka. The Indian
Babrala and other places in Uttar Pradesh. subcontinent is a southerly region of Asia, mostly
6. (3) Black soil is mainly found in Maharashtra, Saurashtra, situated on the Indian Plate and projecting southward
Malwa, Madhya Pradesh and Chhattisgarh. They are into the Indian Ocean.
well known for their capacity to hold moisture and are 18. (4) Satpura hills are part of the Deccan plateau in Madhya
very rich in soil nutrients. Pradesh. The hills stretch for some 560 miles (900 km)
across the widest part of peninsular India, through
7. (4) The dendritic pattern develops where the river channel
Maharashtra and Madhya Pradesh states. The range,
follows the slope of the terrain. The stream with its the name of which means "Seven Folds," forms the
tributaries resembles the braches of a tree, thus the watershed between the Narmada (north) and Tapti
name dendritic. A river joined by its tributaries, at (south) rivers.
approximately right angles, develops a trellis pattern. 19. (3) This Wildlife Sanctuary is located at a distance of
A trellis drainage pattern develops where hard and soft 25 Km from Karauli City in the hilly area of Aravalli range
rocks exist parallel to each other. A rectangular drainage in the holy town of Kaila Devi, District Karauli, Rajasthan.
pattern develops on a strongly jointed rocky terrain. It was established in the year 1983 and is spread over an
area of 676.40 Sq. Km. It was made a part of the famous
The radial pattern develops when streams flow in
Ranthambhore Tiger Reserve in the year 1991.
different directions from a central peak or dome like 20. (4) Maasai Mara (Masai Mara) is known as one of Africa's
structure. Greatest Wildlife Reserves, situated in southwest
8. (4) During 2004 Tsunami the southernmost point of Indian Kenya and is part of the northern section of the
union 'Indira Point' got submerged under the sea water. Serengeti National Park. Famous for the abundance of
It was situated on Great Nicobar Island in the Nicobar the big cats, Lion, Leopard, Cheetah and the Great
Islands, which are located in the eastern Indian Ocean Wildebeast Migration and the Maasai people, well
at 6°45 10 N and 93°49 36 E. known for their distinctive custom and dress.
9. (3) River Kaveri forms the second biggest waterfall. 21. (1) Ranthambore National Park is in Sawai Madhopur
10. (2) Sugarcane is a tropical and subtropical crop. It grows District of Rajasthan state. Ranthambore national park
well in hot and humid climate with a temperature of was declared a wildlife sanctuary in 1957 and in 1974 it
21º C to 27º C and an annual rainfall between 75 cm and gained the protection of "Project Tiger". It got its status
100 cm. of a National Park in 1981.
11. (3) Kandla was constructed in the 1950s as the chief 22. (4) Compressed natural gas (CNG) is a fossil fuel substitute
seaport serving western India, after the partition of for gasoline (petrol), Diesel fuel, and propane/LPG.
India from Pakistan left the port of Karachi in Pakistan. Although CNG's combustion does produce greenhouse
12. (4) India's total area accounts for about 2.4 per cent of the gases, it is widely considered a more environmentally
total geographical area of the world. India is the seventh "clean" alternative to conventional fuels; plus, it is
much safer than other fuels in the event of a spill (as
largest country of the world. India has a land boundary
natural gas is lighter than air, and disperses quickly
of about 15,200 km and the total length of the coast line
when released). CNG may also be mixed with biogas
of the mainland including Andaman and Nicobar and
(produced from landfills or wastewater).
Lakshadweep is 7,516.6 km.
Exemplar Practice Questions 67
S-

23. (2) According to Census of India 2011, Indian sex ratio 36. (4) One problem of the economic exploitation of equatorial
has shown some improvement in the last 10 years. It Ever green forest is the trees are not suitable for
has gone up from 933 in 2001 to 940 in 2011 census of economic utilization.
India. 37. (4) A river ends its journey when it flows into ocean, sea,
24. (2) Bhakra Dam is a concrete gravity dam across the Sutlej estuary, lake or reservoir. It is a landform that is formed
River, and is near the border between Punjab and at the mouth of a river. Because the river water is no
Himachal Pradesh in northern India. Nangal dam is longer flowing downhill, the speed of the water slows
another dam downstream of Bhakra dam. Sometimes down. It makes sediments settle down at the bottom.
both the dams together are called Bhakra-Nangal dam Over long periods of time, this deposition builds the
though they are two separate dams. characteristic geographic pattern of a river delta.
25. (3) Dhebar Lake (also known as Jaisamand Lake) is Asia's 38. (3) The highway runs alongside the Indus River and through
second-largest artificial lake. some extremely harsh and barren terrain including the
26. (4) three high mountain passes named Zozi La (3528 Mtrs),
Namki La (3815 Mtrs) and Fotu La (4108 Mtrs).
27. (3) Goa is a state of India rest all others are Union
39. (3) Iron and Steel industry is called the mother of all
Territories.
industries. It provides raw materials to ship building,
28. (1) The Aravalli ranges are the oldest fold mountains in
transport equipments, machine and tool making
India. The Aravali Hills lie on the western and
northwestern margins of the peninsular plateau. These industry, Aircraft etc.
are highly eroded hills and are found as broken hills. 40. (3) Wells (including tube wells) account for more than 30
They extend from Gujarat to Delhi in a southwest- million acres (more than 55 per cent of total irrigated
northeast direction. area). Well irrigation is most common in alluvial plain
areas where the water table is fairly high. Owing to the
29. (2) The reservoir which is related to the Bhakra-Nangal
soft nature of the soil, wells are easy to dig and the
Project is known as the “Gobind Sagar”, stores up to
yield of crops from the land after irrigation is rewarding.
9.34 billion cubic metres of water. The Bhakra-Nangal
multipurpose dams were among the earliest river valley 41. (2) Rawatbhata atomic power station is in Rajasthan.
development schemes undertaken by India after 42. (1) Jhansi lies in central India and does not come in that
independence. route.
43. (2) Kolaghat Thermal Power Station is a major thermal
30. (4) Sugarcane is the main cash crop of Odisha.
power station in West Bengal. It is located at Mecheda,
31. (4) India ranks first in the world in the production of Mica. approx. 55 km from Kolkata in the Purba Medinipur.
Due to its excellent di-electric strength, low power loss
44. (4) Steel industries are found at Kalol in Gujarat.
factor, insulating properties and resistance to high
45. (4)
voltage, mica is one of the most indispensable minerals
46. (2) The Kerala Backwaters are a network of interconnected
used in electronic industries.
canals, rivers, lakes and inlets, a labyrinthine system
32. (1) Copper is an exhaustible resource. A renewable formed by more than 900 km of waterways, and
resource is like solar power, which keeps on coming no sometimes compared to the American Bayou. In the
matter how much we use it. Copper is mined out of the midst of this landscape there are a number of towns
earth. When it is all finished there will be no more. That and cities, which serve as the starting and end points
is why recycling copper is so important. of backwater cruises. National Waterway No. 3 from
33. (4) Kandla port is in Gujarat and was the first port Kollam to Kottapuram, covers a distance of 205 km and
developed soon after Independence to ease the volume runs almost parallel to the coast line of southern Kerala
of trade on the Mumbai port, when we lost Karachi facilitating both cargo movement and backwater
port to Pakistan after partition. tourism.
34. (1) The city located on the equator is Quito. It is the capital 47. (4) Trichur is in the state of Kerala.
city of Ecuador. The central square of Quito is located 48. (1) Ozar is not associated with MIG aircrafts.
about 25 kilometres (16 mi) south of the equator; the 49. (4) The 'Bushmen' are the oldest inhabitants of southern
city itself extends to within about 1 kilometre (0.62 mi) Africa, where they have lived for at least 20,000 years.
of zero latitude. Their home is in the vast expanse of the Kalahari Desert.
The rest are in equatorial region.
35. (3) Alexander von Humboldt, a German traveler, scientist, 50. (4) Steppes - Eurasia, Veldts - South Africa, Downs -
and geographer from 1769-1859, is commonly known Australia, Pampas - South America.
as the "father of modern geography." German 51. (3) Lateritic soils are characterized by their low soil fertility.
geographer Carl Ritter is commonly associated with Due to the high rate of weathering, and resulting low
Alexander von Humboldt as one of the founders of charge minerals, the soil is unable to retain the nutrients
modern geography. needed for the growth of plants.
EBD_7332
68
S- NTSE Stage 1 Question Bank

52. (2) Chambal project - MP, Rajasthan, Damodar - WB, Kosi 61. (4) Copper- solidification of liquid in cracks and crevices
- Bihar, Hirakund - Orissa of igneous rocks, aluminium- decomposition of surface
The Chambal irrigation project is one of the rocks, coal- by deposition in layers of sedimentary
multipurpose river valley projects taken during the first rocks, platinum- deposition in sand beds of valleys
five year plan of the country. To make use of the and foothills.
unharnessed water of Chambal, the Government of 62. (3) Vishakhapatnam, Bhadravati, Salem and Vijayanagar
Madhya Pradesh and Rajasthan started a joint are located outside the Chotanagpur region.
integrated scheme of three dams and a reservoir and Chotanagpur plateau region has the maximum
the net work of canals in 1953 for irrigation. concentration of iron and steel industries. It is because
53. (3) Petroleum products power virtually all motor vehicles, of low cost of iron ore, high grade raw materials in
aircraft, marine vessels, and trains around the globe. In proximity, cheap labour and vast growth potential in
total, products derived from oil, such as motor gasoline, the home market.
jet fuel, diesel fuel, and heating oil, supply nearly 40 63. (2) Tuticorin- Tamil Nadu, Vishakhapatanam- Andhra
percent of the energy consumed by households, Pradesh, Paradeep- Orissa, Kolkata- West Bengal.
businesses, and manufacturers worldwide. That is why 64. (4) 65. (4)
it is the single largest item of import. 66. (2) 67. (3)
54. (2) The 38th parallel north is a circle of latitude that is 38 68. (4) 69. (1)
degrees north of the Earth's equatorial plane. It crosses 70. (1) 71. (2)
Europe, the Mediterranean Sea, Asia, the Pacific Ocean, 72. (1) 73. (3)
North America, and the Atlantic Ocean. The 38th
74. (3) 75. (2)
parallel north has been especially important in the recent
76. (4) 77. (3)
history of Korea.
78. (3) 79. (2)
55. (1) The Ganga, Narmada, Godavari, Cauvery
80. (3) 81. (4)
56. (3) 40% of the Indian population lives in the Indo-Gangetic
Plains. 82. (4) 83. (1)
57. (1) It is because Simla lies in the Temparate Zone where 84. (3) 85. (4)
the slanting rays of the sun falls. 86. (1) 87. (3)
58. (3) It is because winds blow from a high pressure area to a 88 (3) 89. (2)
low pressure area. 90. (3) 91. (2)
59. (2) Coniferous forests are found mountainous areas or 92. (2) 93. (2)
regions of high altitudes. 94. (4) 95. (2)
60. (1) As pulses are leguminous and help in restoring fertility 96. (1) 97. (2)
of soil by fixing nitrogen from air in the roots so they 98. (1) 99. (1)
are grown in rotation with other crops. 100. (2)

You might also like